Download as pdf or txt
Download as pdf or txt
You are on page 1of 1269

MACHINE DESIGN - An Integrated Approach, 4th Ed.

1-1-1
PROBLEM 1-1
Statement: It is often said, "Build a better mousetrap and the world will beat a path to your door." Consider
this problem and write a goal statement and a set of at least 12 task specifications that you would
apply to its solution. Then suggest 3 possible concepts to achieve the goal. Make annotated,
freehand sketches of the concepts.

Solution:

Goal Statement: Create a mouse-free environment.

Task Specifications:

1. Cost less than $1.00 per use or application.


2. Allow disposal without human contact with mouse.
3. Be safe for other animals such as house pets.
4. Provide no threat to children or adults in normal use.
5. Be a humane method for the mouse.
6. Be environmentally friendly.
7. Have a shelf-life of at least 3 months.
8. Leave no residue.
9. Create minimum audible noise in use.
10. Create no detectable odors within 1 day of use.
11. Be biodegradable.
12. Be simple to use with minimal written instructions necessary.

Concepts and sketches are left to the student. There are an infinity of possibilities.

© 2011 Pearson Education, Inc., Upper Saddle River, NJ. All rights reserved. This publication is protected by Copyright and written permission should be
MACHINE DESIGN - An Integrated Approach, 4th Ed. 1-2-1
PROBLEM 1-2
Statement: A bowling machine is desired to allow quadriplegic youths, who can only move a joystick, to
engage in the sport of bowling at a conventional bowling alley. Consider the factors involved,
write a goal statement, and develop a set of at least 12 task specifications that constrain this
problem. Then suggest 3 possible concepts to achieve the goal. Make annotated, freehand
sketches of the concepts.

Solution:

Goal Statement: Create a means to allow a quadriplegic to bowl.

Task Specifications:

1. Cost no more than $2 000.


2. Portable by no more than two able-bodied adults.
3. Fit through a standard doorway.
4. Provide no threat of injury to user in normal use.
5. Operate from a 110 V, 60 Hz, 20 amp circuit.
6. Be visually unthreatening.
7. Be easily positioned at bowling alley.
8. Have ball-aiming ability, controllable by user.
9. Automatically reload returned balls.
10. Require no more than 1 able-bodied adult for assistance in use.
11. Ball release requires no more than a mouth stick-switch closure.
12. Be simple to use with minimal written instructions necessary.

Concepts and sketches are left to the student. There are an infinity of possibilities.

© 2011 Pearson Education, Inc., Upper Saddle River, NJ. All rights reserved. This publication is protected by Copyright and written permission should be
MACHINE DESIGN - An Integrated Approach, 4th Ed. 1-3-1
PROBLEM 1-3
Statement: A quadriplegic needs an automated page turner to allow her to read books without assistance.
Consider the factors involved, write a goal statement, and develop a set of at least 12 task
specifications that constrain this problem. Then suggest 3 possible concepts to achieve the goal.
Make annotated, freehand sketches of the concepts.

Solution:

Goal Statement: Create a means to allow a quadriplegic to read standard books with minimum assistance.

Task Specifications:

1. Cost no more than $1 000.


2. Useable in bed or from a seated position
3. Accept standard books from 8.5 x 11 in to 4 x 6 in in planform and up to 1.5 in thick.
4. Book may be placed, and device set up, by able-bodied person.
5. Operate from a 110 V, 60 Hz, 15 amp circuit or by battery power.
6. Be visually unthreatening and safe to use.
7. Require no more than 1 able-bodied adult for assistance in use.
8. Useable in absence of assistant once set up.
9. Not damage books.
10. Timing controlled by user.
11. Page turning requires no more than a mouth stick-switch closure.
12. Be simple to use with minimal written instructions necessary.

Concepts and sketches are left to the student. There are an infinity of possibilities.

© 2011 Pearson Education, Inc., Upper Saddle River, NJ. All rights reserved. This publication is protected by Copyright and written permission should be
MACHINE DESIGN - An Integrated Approach, 4th Ed. 1-4-1
PROBLEM 1-4
Statement: Convert a mass of 1 000 lbm to (a) lbf, (b) slugs, (c) blobs, (d) kg.
2
lbf  sec
Units: blob :=
in

Given: Mass M := 1000  lb

Solution: See Mathcad file P0104.

1. To determine the weight of the given mass, multiply the mass value by the acceleration due to gravity, g.

W := M  g W = 1000 lbf

2. Convert mass units by assigning different units to the units place-holder when displaying the mass value.

Slugs M = 31.081 slug

Blobs M = 2.59 blob

Kilograms M = 453.592 kg

© 2011 Pearson Education, Inc., Upper Saddle River, NJ. All rights reserved. This publication is protected by Copyright and written permission should be
MACHINE DESIGN - An Integrated Approach, 4th Ed. 1-5-1
PROBLEM 1-5
Statement: A 250-lbm mass is accelerated at 40 in/sec2. Find the force in lb needed for this acceleration.
in
Given: Mass M := 250 lb Acceleration a := 40
2
sec

Solution: See Mathcad file P0105.

1. To determine the force required, multiply the mass value, in slugs, by the acceleration in feet per second squared.

Convert mass to slugs: M = 7.770 slug


-2
Convert acceleration to feet per second squared: a = 3.333s  ft

F := M  a F = 25.9 lbf

© 2011 Pearson Education, Inc., Upper Saddle River, NJ. All rights reserved. This publication is protected by Copyright and written permission should be
MACHINE DESIGN - An Integrated Approach, 4th Ed. 1-6-1
PROBLEM 1-6
Statement: Express a 100-kg mass in units of slugs, blobs, and lbm. How much does this mass weigh?
2
lbf  sec
Units: blob 
in

Given: M  100  kg

Assumptions: The mass is at sea-level and the gravitational acceleration is


ft in m
g  32.174 or g  386.089  or g  9.807 
2 2 2
sec sec sec

Solution: See Mathcad file P0106.


1. Convert mass units by assigning different units to the units place-holder when displaying the mass value.

The mass, in slugs, is M  6.85 slug

The mass, in blobs, is M  0.571  blob

The mass, in lbm, is M  220.5  lb

Note: Mathcad uses lbf for pound-force, and lb for pound-mass.

2. To determine the weight of the given mass, multiply the mass value by the acceleration due to gravity, g.

The weight, in lbf, is W  M  g W  220.5  lbf

The weight, in N, is W  M  g W  980.7  N

© 2011 Pearson Education, Inc., Upper Saddle River, NJ. All rights reserved. This publication is protected by Copyright and written permission should be
MACHINE DESIGN - An Integrated Approach, 4th Ed. 1-7-1

PROBLEM 1-7
Statement: Prepare an interactive computer program (using, for example, Excell, Mathcad, or TKSolver) from
which the cross-sectional properties for the shapes shown in the inside front cover can be
calculated. Arrange the program to deal with both ips and SI unit systems and convert the results
between those systems.

Solution: See the inside front cover and Mathcad file P0107.
1. Rectangle, let:
b  3  in h  4  in
2
Area A  b  h A  12.000 in
2
A  7742 mm
3
b h 4
Moment about x-axis Ix  Ix  16.000 in
12 6 4
Ix  6.660  10  mm
3
h b 4
Moment about y-axis Iy  Iy  9.000  in
12
6 4
Iy  3.746  10  mm

Ix
Radius of gyration about x-axis kx  kx  1.155  in
A
kx  29.329 mm

Iy
Radius of gyration about y-axis ky  ky  0.866  in
A
ky  21.997 mm

4
Polar moment of inertia Jz  Ix  Iy Jz  25.000 in
7 4
Jz  1.041  10  mm
2. Solid circle, let:
D  3  in
2
π D 2
Area A  A  7.069  in
4 2
A  4560 mm

4
π D 4
Moment about x-axis Ix  Ix  3.976  in
64 6 4
Ix  1.655  10  mm
4
π D 4
Moment about y-axis Iy  Iy  3.976  in
64
6 4
Iy  1.655  10  mm

Ix
Radius of gyration about x-axis kx  kx  0.750  in
A
kx  19.05  mm

© 2011 Pearson Education, Inc., Upper Saddle River, NJ. All rights reserved. This publication is protected by Copyright and written permission should be
MACHINE DESIGN - An Integrated Approach, 4th Ed. 1-7-2

Iy
Radius of gyration about y-axis ky  ky  0.750  in
A ky  19.05  mm
4
π D 4
Polar moment of inertia Jz  Jz  7.952  in
32 6 4
Jz  3.310  10  mm

3. Hollow circle, let:

D  3  in d  1  in

Area A 
π
4
  D d
2 
2
A  6.283  in
2

2
A  4054 mm

Moment about x-axis Ix 


π
64

 D d
4 4  4
Ix  3.927  in
6 4
Ix  1.635  10  mm

Moment about y-axis Iy 


π
64
  D d
4 4  4
Iy  3.927  in
6 4
Iy  1.635  10  mm

Ix
Radius of gyration about x-axis kx  kx  0.791  in
A
kx  20.08  mm

Iy
Radius of gyration about y-axis ky  ky  0.791  in
A
ky  20.08  mm

Polar moment of inertia Jz 


π
32
  D d
4 4  Jz  7.854  in
4

6 4
Jz  3.269  10  mm

4. Solid semicircle, let:

D  3  in R  0.5 D R  1.5 in
2
π D 2
Area A  A  3.534  in
8
2
A  2280 mm

4 4
Moment about x-axis Ix  0.1098 R Ix  0.556  in
5 4
Ix  2.314  10  mm
4
π R 4
Moment about y-axis Iy  Iy  1.988  in
8
5 4
Iy  8.275  10  mm

© 2011 Pearson Education, Inc., Upper Saddle River, NJ. All rights reserved. This publication is protected by Copyright and written permission should be
MACHINE DESIGN - An Integrated Approach, 4th Ed. 1-7-3
Ix
Radius of gyration about x-axis kx  kx  0.397  in
A
kx  10.073 mm

Iy
Radius of gyration about y-axis ky  ky  0.750  in
A
ky  19.05  mm
4
Polar moment of inertia Jz  Ix  Iy Jz  2.544  in
6 4
Jz  1.059  10  mm

Distances to centroid a  0.4244 R a  0.637  in


a  16.17  mm

b  0.5756 R b  0.863  in
b  21.93  mm

5. Right triangle, let:


b  2  in h  1  in

b h 2
Area A  A  1.000  in
2 2
A  645  mm

3
b h 4
Moment about x-axis Ix  Ix  0.056  in
36 4 4
Ix  2.312  10  mm
3
h b 4
Moment about y-axis Iy  Iy  0.222  in
36
4 4
Iy  9.250  10  mm

Ix
Radius of gyration about x-axis kx  kx  0.236  in
A
kx  5.987  mm

Iy
Radius of gyration about y-axis ky  ky  0.471  in
A
ky  11.974 mm

4
Polar moment of inertia Jz  Ix  Iy Jz  0.278  in
5 4
Jz  1.156  10  mm

© 2011 Pearson Education, Inc., Upper Saddle River, NJ. All rights reserved. This publication is protected by Copyright and written permission should be
MACHINE DESIGN - An Integrated Approach, 4th Ed. 1-8-1
PROBLEM 1-8
Statement: Prepare an interactive computer program (using, for example, Excell, Mathcad, or TKSolver) from
which the mass properties for the solids shown in the page opposite the inside front cover can be
calculated. Arrange the program to deal with both ips and SI unit systems and convert the results
between those systems.
2
lbf  sec
Units: blob 
in
Solution: See the page opposite the inside front cover and Mathcad file P0108.
3
1. Rectangular prism, let: a  2  in b  3  in c  4  in γ  0.28 lbf  in
3
Volume V  a  b  c V  24.000 in
3
V  393290 mm
V γ
Mass M  M  0.017  blob
g
M  3.048  kg

Moment about x-axis Ix 


2
M a  b
2  Ix  0.019  blob in
2
12
2
Ix  2130.4 kg mm

Moment about y-axis Iy 


2
M a  c
2 Iy  0.029  blob in
2
12
2
Iy  3277.6 kg mm

Moment about z-axis Iz 


2
M b  c
2 Iz  0.036  blob in
2
12
2
Iz  4097.0 kg mm

Ix
Radius of gyration about x-axis kx  kx  1.041  in
M
kx  26.437 mm

Iy
Radius of gyration about y-axis ky  ky  1.291  in
M
ky  32.791 mm

Iz
Radius of gyration about z-axis kz  kz  1.443  in
M
kz  36.662 mm

3
2.Cylinder, let: r  2  in L  3  in γ  0.30 lbf  in

2 3
Volume V  π r  L V  37.699 in
3
V  617778 mm
V γ
Mass M  M  0.029  blob
g
M  5.13 kg
© 2011 Pearson Education, Inc., Upper Saddle River, NJ. All rights reserved. This publication is protected by Copyright and written permission should be
MACHINE DESIGN - An Integrated Approach, 4th Ed. 1-8-2

2
M r 2
Moment about x-axis Ix  Ix  0.059  blob in
2
2
Ix  6619.4 kg mm

Moment about y-axis Iy 



M  3 r  L
2 2  Iy  0.051  blob in
2
12
2
Iy  5791.9 kg mm

Moment about z-axis Iz 



M  3 r  L
2 2  Iz  0.051  blob in
2
12
2
Iz  5791.9 kg mm

Ix
Radius of gyration about x-axis kx  kx  1.414  in
M
kx  35.921 mm

Iy
Radius of gyration about y-axis ky  ky  1.323  in
M
ky  33.601 mm

Iz
Radius of gyration about z-axis kz  kz  1.323  in
M
kz  33.601 mm

3. Hollow cylinder, let:


3
a  2  in b  3  in L  4  in γ  0.28 lbf  in

Volume 2
V  π b  a  L 
2
V  62.832 in
3

3
V  1029630  mm

V γ
Mass M  M  0.046  blob
g
M  7.98 kg

Moment about x-axis Ix 


M
2
 a b
2  Ix  0.296  blob in
2
2 4 2
Ix  3.3  10  kg mm

Moment about y-axis Iy 


M
 2
 3 a  3 b  L
2 
2
Iy  0.209  blob in
2
12 4 2
Iy  2.4  10  kg mm

Moment about z-axis Iz 


M
 2
 3 a  3 b  L
2 
2
Iz  0.209  blob in
2
12 4 2
Iz  2.4  10  kg mm

Ix
Radius of gyration about x-axis kx  kx  2.550  in
M
kx  64.758 mm

Iy
Radius of gyration about y-axis ky  ky  2.141  in
M
ky  54.378 mm
© 2011 Pearson Education, Inc., Upper Saddle River, NJ. All rights reserved. This publication is protected by Copyright and written permission should be
MACHINE DESIGN - An Integrated Approach, 4th Ed. 1-8-3
Iz
Radius of gyration about z-axis kz  kz  2.141  in
M
kz  54.378 mm

4. Right circular cone, let:

3
r  2  in h  5  in γ  0.28 lbf  in
2
π r  h 3
Volume V  V  20.944 in
3
3
V  343210 mm
V γ
Mass M  M  0.015  blob
g
M  2.66 kg

3 2 2
Moment about x-axis Ix  M r Ix  0.018  blob in
10
2
Ix  2059.4 kg mm

Moment about y-axis Iy  M 


12r2  3h2 Iy  0.023  blob in
2
80
2
Iy  2638.5 kg mm

Moment about z-axis Iz  M 


12r2  3h2 Iz  0.023  blob in
2
80
2
Iz  2638.5 kg mm
Ix
Radius of gyration about x-axis kx  kx  1.095  in
M
kx  27.824 mm

Iy
Radius of gyration about y-axis ky  ky  1.240  in
M
ky  31.495 mm

Iz
Radius of gyration about z-axis kz  kz  1.240  in
M
kz  31.495 mm
5. Sphere, let:
r  3  in

4 3 3
Volume V   π r V  113.097  in
3 3
V  1853333  mm

V γ
Mass M  M  0.082  blob
g
M  14.364 kg

2 2 2
Moment about x-axis Ix  M r Ix  0.295  blob in
5 2
Ix  33362  kg mm
© 2011 Pearson Education, Inc., Upper Saddle River, NJ. All rights reserved. This publication is protected by Copyright and written permission should be
MACHINE DESIGN - An Integrated Approach, 4th Ed. 1-8-4
2 2 2
Moment about y-axis Iy  M r Iy  0.295  blob in
5
2
Iy  33362  kg mm
2 2 2
Moment about z-axis Iz  M r Iz  0.295  blob in
5 2
Iz  33362  kg mm

Ix
Radius of gyration about x-axis kx  kx  1.897  in
M
kx  48.193 mm

Iy
Radius of gyration about y-axis ky  ky  1.897  in
M
ky  48.193 mm

Iz
Radius of gyration about z-axis kz  kz  1.897  in
M
kz  48.193 mm

© 2011 Pearson Education, Inc., Upper Saddle River, NJ. All rights reserved. This publication is protected by Copyright and written permission should be
MACHINE DESIGN - An Integrated Approach, 4th Ed. 1-9-1
PROBLEM 1-9
Statement: Convert the template in Problem 1-7 to have and use a set of functions or subroutines that can be
called from within any program in that language to solve for the cross-sectional properties of the
shapes shown on the inside front cover.

Solution: See inside front cover and Mathcad file P0109.

1. Rectangle: Area A ( b h )  b  h


3
b h
Moment about x-axis Ix( b h ) 
12
3
h b
Moment about y-axis Iy( b h ) 
12

2
π D
2. Solid circle: Area A ( D) 
4
4
π D
Moment about x-axis Ix( D) 
64
4
π D
Moment about y-axis Iy( D) 
64

3. Hollow circle: Area A ( D d ) 


π
 2
 D d
2
4

Moment about x-axis Ix( D d ) 


π
 4
 D d
4 
64

Moment about y-axis Iy( D d ) 


π
 4
 D d
4 
64

4. Solid semicircle:
2
π D
Area A ( D) 
8

4
Moment about x-axis Ix( R)  0.1098 R

4
π R
Moment about y-axis Iy( R) 
8

5. Right triangle:
b h
Area A ( b h ) 
2

3
b h
Moment about x-axis Ix( b h ) 
36
3
h b
Moment about y-axis Iy( b h ) 
36
© 2011 Pearson Education, Inc., Upper Saddle River, NJ. All rights reserved. This publication is protected by Copyright and written permission should be
MACHINE DESIGN - An Integrated Approach, 4th Ed. 1-10-1
PROBLEM 1-10
Statement: Convert the template in Problem 1-8 to have and use a set of functions or subroutines that can be
called from within any program in that language to solve for the cross-sectional properties of the
shapes shown on the page opposite the inside front cover.
Solution: See the page opposite the inside front cover and Mathcad file P0110.
1 Rectangular prism:

Volume V ( a b c)  a  b  c

V ( a b c)  γ
Mass M ( a b c γ) 
g

Moment about x-axis Ix( a b c γ) 


M ( a b c γ)  a  b 2 2 
12

Moment about y-axis Iy( a b c γ) 


M ( a b c γ)  a  c 2 
2

12

Moment about z-axis Iz( a b c γ) 


M ( a b c γ)  b  c 2 
2

12
2. Cylinder:
2
Volume V ( r L)  π r  L

V ( r L)  γ
Mass M ( r L γ) 
g

2
M ( r L γ)  r
Moment about x-axis Ix( r L γ) 
2

Moment about y-axis Iy( r L γ) 



M ( r L γ)  3  r  L
2 
2

12

Moment about z-axis Iz( r L γ) 



M ( r L γ)  3  r  L
2 
2

12

3. Hollow cylinder:

Volume 2
V ( a b L)  π b  a  L
2
V ( a b L)  γ
Mass M ( a b L γ) 
g

Moment about x-axis Ix( a b L γ) 


M ( a b L γ)
2
 a b
2 
2

Moment about y-axis Iy( a b L γ) 


M ( a b L γ)
 2
 3 a  3 b  L
2 
2
12

Moment about z-axis Iz( a b L γ) 


M ( a b L γ)
 2
 3 a  3 b  L
2 
2
12
© 2011 Pearson Education, Inc., Upper Saddle River, NJ. All rights reserved. This publication is protected by Copyright and written permission should be
MACHINE DESIGN - An Integrated Approach, 4th Ed. 1-10-2
4. Right circular cone:
2
π r  h
Volume V ( r h ) 
3

V ( r h )  γ
Mass M ( r h γ) 
g

3 2
Moment about x-axis Ix( r h γ)   M ( r h γ)  r
10

Moment about y-axis I ( r h γ)  M ( r h γ) 


 12 r  3  h 
2 2
y
80

Moment about z-axis Iz( r h γ)  M ( r h γ) 


12r2  3h2
80
5. Sphere:
4 3
Volume V ( r)   π r
3

V ( r)  γ
Mass M ( r γ) 
g

2 2
Moment about x-axis Ix( r γ)   M ( r γ)  r
5

2 2
Moment about y-axis Iy( r γ)   M ( r γ)  r
5

2 2
Moment about z-axis Iz( r γ)   M ( r γ)  r
5

© 2011 Pearson Education, Inc., Upper Saddle River, NJ. All rights reserved. This publication is protected by Copyright and written permission should be
MACHINE DESIGN - An Integrated Approach, 4th Ed. 2-1-1
PROBLEM 2-1
Statement: Figure P2-1 shows stress-strain curves for three failed tensile-test specimens. All are plotted on
the same scale.
(a) Characterize each material as brittle or ductile.
(b) Which is the stiffest?
(c) Which has the highest ultimate strength?
(d) Which has the largest modulus of resilience?
(e) Which has the largest modulus of toughness?

Solution: See Figure P2-1 and Mathcad file P0201.

1. The material in Figure P2-1(a) has a moderate amount of strain beyond the yield point, P2-1(b) has very little, and
P2-1(c) has considerably more than either of the other two. Based on this observation, the material in Figure
P2-1(a) is mildly ductile, that in P2-1(b)is brittle, and that in P2-1(c) is ductile.
2. The stiffest material is the one with the grearesr slope in the elastic range. Determine this by dividing the rise by
the run of the straight-line portion of each curve. The material in Figure P2-1(c) has a slope of 5 stress units per
strain unit, which is the greatest of the three. Therefore, P2-1(c) is the stiffest.
3. Ultimate strength corresponds to the highest stress that is achieved by a material under test. The material in
Figure P2-1(b) has a maximum stress of 10 units, which is considerably more than either of the other two.
Therefore, P2-1(b) has the highest ultimate strength.

4. The modulus of resilience is the area under the elastic portion of the stress-starin curve. From observation of the
three graphs, the stress and strain values at the yield points are:
P2-1(a) σya := 5 εya := 5

P2-1(b) σyb := 9 εyb := 2

P2-1(c) σyc := 5 εyc := 1

Using equation (2.7), the modulus of resiliency for each material is, approximately,

1
P21a := ⋅ σya ⋅ ε ya P21a = 12.5
2

1
P21b := ⋅ σyb ⋅ ε yb P21b = 9
2

1
P21c := ⋅ σyc⋅ ε yc P21c = 2.5
2

P2-1 (a) has the largest modulus of resilience

5. The modulus of toughness is the area under the stress-starin curve up to the point of fracture. By inspection,
P2-1 (c) has the largest area under the stress-strain curve therefore, it has the largest modulus of toughness.

© 2011 Pearson Education, Inc., Upper Saddle River, NJ. All rights reserved. This publication is protected by Copyright and written permission should be
MACHINE DESIGN - An Integrated Approach, 4th Ed. 2-2-1
PROBLEM 2-2
Statement: Determine an approximate ratio between the yield strength and ultimate strength for each material
shown in Figure P2-1.

Solution: See Figure P2-1 and Mathcad file P0202.

1. The yield strength is the value of stress at which the stress-strain curve begins to be nonlinear. The ultimate
strength is the maximum value of stress attained during the test. From the figure, we have the following values
of yield strength and tensile strength:

Figure P2-1(a) S ya := 5 S ua := 6

Figure P2-1(b) S yb := 9 S ub := 10

Figure P2-1(c) S yc := 5 S uc := 8

2. The ratio of yield strength to ultimate strength for each material is:

S ya
Figure P2-1(a) ratioa := ratioa = 0.83
S ua

S yb
Figure P2-1(b) ratiob := ratiob = 0.90
S ub

S yc
Figure P2-1(c) ratioc := ratioc = 0.63
S uc

© 2011 Pearson Education, Inc., Upper Saddle River, NJ. All rights reserved. This publication is protected by Copyright and written permission should be
MACHINE DESIGN - An Integrated Approach, 4th Ed. 2-3-1
PROBLEM 2-3
Statement: Which of the steel alloys shown in Figure 2-19 would you choose to obtain
(a) Maximum strength
(b) Maximum modulus of resilience
(c) Maximum modulus of toughness
(d) Maximum stiffness

Given: Young's modulus for steel E  207  GPa


Solution: See Figure 2-19 and Mathcad file P0203.
1. Determine from the graph: values for yield strength, ultimate strength and strain at fracture for each material.

Steel Yield Strength Ultimate Strength Fracture Strain


AISI 1020: Sy1020  300  MPa Sut 1020  400  MPa εf 1020  0.365

AISI 1095: Sy1095  550  MPa Sut 1095  1050 MPa εf 1095  0.11

AISI 4142: Sy4142  1600 MPa Sut 4142  2430 MPa εf 4142  0.06

Note: The 0.2% offset method was used to define a yield strength for the AISI 1095 and the 4142 steels.
2. From the values of Sut above it is clear that the AISI 4142 has maximum strength.
3. Using equation (2-7) and the data above, determine the modulus of resilience.
2
1 Sy1020 MN  m
UR1020   UR1020  0.22
2 E 3
m
2
1 Sy1095 MN  m
UR1095   UR1095  0.73
2 E 3
m
2
1 Sy4142 MN  m
UR4142   UR4142  6.18
2 E 3
m
Even though the data is approximate, the AISI 4142 clearly has the largest modulus of resilience.
4. Using equation (2-8) and the data above, determine the modulus of toughness.
MN  m
  Sy1020  Sut 1020  εf 1020
1
UT1020  UT1020  128 
2 3
m
MN  m
  Sy1095  Sut 1095  εf 1095
1
UT1095  UT1095  88
2 3
m
MN  m
  Sy4142  Sut 4142  εf 4142
1
UT4142  UT4142  121 
2 3
m
Since the data is approximate, there is no significant difference between the 1020 and 4142 steels. Because of
the wide difference in shape and character of the curves, one should also determine the area under the
curves by graphical means. When this is done, the area under the curve is about 62 square units for 1020
and 66 for 4142. Thus, they seem to have about equal toughness, which is about 50% greater than that for
the 1095 steel.

5. All three materials are steel therefore, the stiffnesses are the same.

© 2011 Pearson Education, Inc., Upper Saddle River, NJ. All rights reserved. This publication is protected by Copyright and written permission should be
MACHINE DESIGN - An Integrated Approach, 4th Ed. 2-4-1
PROBLEM 2-4
Statement: Which of the aluminum alloys shown in Figure 2-21 would you choose to obtain
(a) Maximum strength
(b) Maximum modulus of resilience
(c) Maximum modulus of toughness
(d) Maximum stiffness

Given: Young's modulus for aluminum E  71.7 GPa


Solution: See Figure 2-21 and Mathcad file P0204.
1. Determine, from the graph, values for yield strength, ultimate strength and strain at fracture for each material.

Alum Yield Strength Ultimate Strength Fracture Strain


1100: Sy1100  120  MPa Sut 1100  130  MPa εf 1100  0.170

2024-T351: Sy2024  330  MPa Sut 2024  480  MPa εf 2024  0.195

7075-T6: Sy7075  510  MPa Sut 7075  560  MPa εf 7075  0.165

Note: The 0.2% offset method was used to define a yield strength for all of the aluminums.
2. From the values of Sut above it is clear that the 7075-T6 has maximum strength.
3. Using equation (2-7) and the data above, determine the modulus of resilience.
2
1 Sy1100 MN  m
UR1100   UR1100  0.10
2 E 3
m
2
1 Sy2024 MN  m
UR2024   UR2024  0.76
2 E 3
m
2
1 Sy7075 MN  m
UR7075   UR7075  1.81
2 E 3
m
Even though the data is approximate, the 7075-T6 clearly has the largest modulus of resilience.
4. Using equation (2-8) and the data above, determine the modulus of toughness.
MN  m
  Sy1100  Sut 1100  εf 1100
1
UT1100  UT1100  21
2 3
m
MN  m
  Sy2024  Sut 2024  εf 2024
1
UT2024  UT2024  79
2 3
m
MN  m
  Sy7075  Sut 7075  εf 7075
1
UT7075  UT7075  88
2 3
m
Even though the data is approximate, the 7075-T6 has the largest modulus of toughness.

5. All three materials are aluminum therefore, the stiffnesses are the same.

© 2011 Pearson Education, Inc., Upper Saddle River, NJ. All rights reserved. This publication is protected by Copyright and written permission should be
MACHINE DESIGN - An Integrated Approach, 4th Ed. 2-5-1
PROBLEM 2-5
Statement: Which of the thermoplastic polymers shown in Figure 2-22 would you choose to obtain
(a) Maximum strength
(b) Maximum modulus of resilience
(c) Maximum modulus of toughness
(d) Maximum stiffness

Solution: See Figure 2-22 and Mathcad file P0205.


1. Determine, from the graph, values for yield strength, ultimate strength, strain at fracture, and modulus of
elasticity for each material.

Plastic Yield Strength Ultimate Strength Fracture Strain Mod of Elasticity


Nylon 101: SyNylon  63 MPa Sut Nylon  80 MPa εf Nylon  0.52 ENylon  1.1 GPa

HDPE: SyHDPE  15 MPa Sut HDPE  23 MPa εf HDPE  3.0 EHDPE  0.7 GPa

PTFE: SyPTFE  8.3 MPa Sut PTFE  13 MPa εf PTFE  0.51 EPTFE  0.8 GPa

2. From the values of Sut above it is clear that the Nylon 101 has maximum strength.

3. Using equation (2-7) and the data above, determine the modulus of resilience.
2
1 SyNylon MN  m
URNylon   URNylon  1.8
2 ENylon 3
m
2
1 SyHDPE MN  m
URHDPE   URHDPE  0.16
2 EHDPE 3
m
2
1 SyPTFE MN  m
URPTFE   URPTFE  0.04
2 EPTFE 3
m

Even though the data is approximate, the Nylon 101 clearly has the largest modulus of resilience.

4. Using equation (2-8) and the data above, determine the modulus of toughness.

MN  m
  SyNylon  Sut Nylon  εf Nylon
1
UTNylon  UTNylon  37
2 3
m

MN  m
  SyHDPE  SutHDPE  εf HDPE
1
UTHDPE  UTHDPE  57
2 3
m

MN  m
  SyPTFE  SutPTFE  εf PTFE
1
UTPTFE  UTPTFE  5 
2 3
m
Even though the data is approximate, the HDPE has the largest modulus of toughness.

5. The Nylon 101 has the steepest slope in the (approximately) elastic range and is, therefore, the stiffest of the
three materials..

© 2011 Pearson Education, Inc., Upper Saddle River, NJ. All rights reserved. This publication is protected by Copyright and written permission should be
MACHINE DESIGN - An Integrated Approach, 4th Ed. 2-6-1

PROBLEM 2-6
Statement: A metal has a strength of 414 MPa at its elastic limit and the strain at that point is 0.002. What is
the modulus of elasticity? What is the strain energy at the elastic limit? Assume that the test
speimen is 12.8-mm dia and has a 50-mm gage length. Can you define the type of metal based on
the given data?

Given: Elastic limit: Strength S el  414  MPa Strain ε el  0.002

Test specimen: Diameter d o  12.8 mm Length Lo  50 mm

Solution: See Mathcad file P0206.


1. The modulus of elasticity is the slope of the stress-strain curve, which is a straight line, in the elastic region.
Since one end of this line is at the origin, the slope (modulus of elasticity) is

S el
E  E  207  GPa
ε el

2. The strain energy per unit volume at the elastic limit is the area under the stress-strain curve up to the elastic
limit. Since the curve is a straight line up to this limit, the area is one-half the base times the height, or
1 kN  m
U'el   S el ε el U'el  414 
2 3
m

The total strain energy in the specimen is the strain energy per unit volume times the volume,

2
π d o
Uel  U'el  Lo Uel  2.7 N  m
4

3. Based on the modulus of elasticity and using Table C-1, the material is steel.

© 2011 Pearson Education, Inc., Upper Saddle River, NJ. All rights reserved. This publication is protected by Copyright and written permission should be
MACHINE DESIGN - An Integrated Approach, 4th Ed. 2-7-1

PROBLEM 2-7
Statement: A metal has a strength of 41.2 kpsi (284 MPa) at its elastic limit and the strain at that point is
0.004. What is the modulus of elasticity? What is the strain energy at the elastic limit?
Assume that the test speimen is 0.505-in dia and has a 2-in gage length. Can you define the
type of metal based on the given data?

Given: Elastic limit: Strength S el  41.2 ksi Strain ε el  0.004 S el  284  MPa
Test specimen: Diameter d o  0.505  in Length Lo  2.00 in

Solution: See Mathcad file P0207.

1. The modulus of elasticity is the slope of the stress-strain curve, which is a straight line, in the elastic region.
Since one end of this line is at the origin, the slope (modulus of elasticity) is

S el 6
E  E  10.3 10  psi E  71 GPa
ε el

2. The strain energy per unit volume at the elastic limit is the area under the stress-strain curve up to the elastic
limit. Since the curve is a straight line up to this limit, the area is one-half the base times the height, or

1 lbf  in kN  m
U'el   S el ε el U'el  82.4 U'el  568 
2 3 3
in m
The total strain energy in the specimen is the strain energy per unit volume times the volume,
2
π d o
Uel  U'el  Lo Uel  33.0 in lbf
4

3. Based on the modulus of elasticity and using Table C-1, the material is aluminum.

© 2011 Pearson Education, Inc., Upper Saddle River, NJ. All rights reserved. This publication is protected by Copyright and written permission should be
MACHINE DESIGN - An Integrated Approach, 4th Ed. 2-8-1
PROBLEM 2-8

Statement: A metal has a strength of 134 MPa at its elastic limit and the strain at that point is 0.006. What is
the modulus of elasticity? What is the strain energy at the elastic limit? Assume that the test
speimen is 12.8-mm dia and has a 50-mm gage length. Can you define the type of metal based on
the given data?

Given: Elastic limit: Strength S el  134  MPa Strain ε el  0.003

Test specimen: Diameter d o  12.8 mm Length Lo  50 mm

Solution: See Mathcad file P0208.

1. The modulus of elasticity is the slope of the stress-strain curve, which is a straight line, in the elastic region.
Since one end of this line is at the origin, the slope (modulus of elasticity) is

S el
E  E  45 GPa
ε el

2. The strain energy per unit volume at the elastic limit is the area under the stress-strain curve up to the elastic
limit. Since the curve is a straight line up to this limit, the area is one-half the base times the height, or

1 kN  m
U'el   S el ε el U'el  201 
2 3
m

The total strain energy in the specimen is the strain energy per unit volume times the volume,

2
π d o
Uel  U'el  Lo Uel  1.3 N  m
4

3. Based on the modulus of elasticity and using Table C-1, the material is magnesium.

© 2011 Pearson Education, Inc., Upper Saddle River, NJ. All rights reserved. This publication is protected by Copyright and written permission should be
MACHINE DESIGN - An Integrated Approach, 4th Ed. 2-9-1

PROBLEM 2-9

Statement: A metal has a strength of 100 kpsi (689 MPa) at its elastic limit and the strain at that point is
0.006. What is the modulus of elasticity? What is the strain energy at the elastic limit?
Assume that the test speimen is 0.505-in dia and has a 2-in gage length. Can you define the
type of metal based on the given data?

Given: Elastic limit: Strength S el  100  ksi Strain ε el  0.006


S el  689  MPa
Test specimen: Diameter d o  0.505  in Length Lo  2.00 in

Solution: See Mathcad file P0209.


1. The modulus of elasticity is the slope of the stress-strain curve, which is a straight line, in the elastic region.
Since one end of this line is at the origin, the slope (modulus of elasticity) is

S el 6
E  E  16.7 10  psi E  115  GPa
ε el

2. The strain energy per unit volume at the elastic limit is the area under the stress-strain curve up to the elastic
limit. Since the curve is a straight line up to this limit, the area is one-half the base times the height, or

1 lbf  in 3 kN  m
U'el   S el ε el U'el  300  U'el  2  10 
2 3 3
in m

The total strain energy in the specimen is the strain energy per unit volume times the volume,

2
π d o
Uel  U'el  Lo Uel  120.18 in lbf
4

3. Based on the modulus of elasticity and using Table C-1, the material is titanium.

© 2011 Pearson Education, Inc., Upper Saddle River, NJ. All rights reserved. This publication is protected by Copyright and written permission should be
MACHINE DESIGN - An Integrated Approach, 4th Ed. 2-10-1

PROBLEM 2-10
Statement: A material has a yield strength of 689 MPa at an offset of 0.6% strain. What is its modulus of
resilience?
6
Units: MJ  10  joule

Given: Yield strength S y  689  MPa

Yield strain ε y  0.006

Solution: See Mathcad file P0210.

1. The modulus of resilience (strain energy per unit volume) is given by Equation (2.7) and is approximately

1 MJ
UR   S y ε y UR  2.067  UR  2.1 MPa
2 3
m

© 2011 Pearson Education, Inc., Upper Saddle River, NJ. All rights reserved. This publication is protected by Copyright and written permission should be
MACHINE DESIGN - An Integrated Approach, 4th Ed. 2-11-1

PROBLEM 2-11

Statement: A material has a yield strength of 60 ksi (414 MPa) at an offset of 0.2% strain. What is its
modulus of resilience?
6
Units: MJ  10  joule

Given: Yield strength S y  60 ksi S y  414  MPa

Yield strain ε y  0.002

Solution: See Mathcad file P0211.

1. The modulus of resilience (strain energy per unit volume) is given by Equation (2.7) and is approximately

1 in lbf MJ
UR   S y ε y UR  60 UR  0.414  UR  0.414  MPa
2 3 3
in m

© 2011 Pearson Education, Inc., Upper Saddle River, NJ. All rights reserved. This publication is protected by Copyright and written permission should be
MACHINE DESIGN - An Integrated Approach, 4th Ed. 2-12-1

PROBLEM 2-12
Statement: A steel has a yield strength of 414 MPa, an ultimate tensile strength of 689 MPa, and an
elongation at fracture of 15%. What is its approximate modulus of toughness? What is the
approximate modulus of resilience?

Given: S y  414  MPa S ut  689  MPa ε f  0.15

Solution: See Mathcad file P0212.

1. Determine the modulus of toughness using Equation (2.8).

 Sy  S ut  MN  m
UT     εf UT  82.7 UT  82.7 MPa
 2  m
3

2. Determine the modulus of resilience using Equation (2.7) and Young's modulus for steel: E  207  GPa

2
1 Sy kN  m
UR   UR  414  UR  0.41 MPa
2 E 3
m

© 2011 Pearson Education, Inc., Upper Saddle River, NJ. All rights reserved. This publication is protected by Copyright and written permission should be
MACHINE DESIGN - An Integrated Approach, 4th Ed. 2-13-1

PROBLEM 2-13
Statement: The Brinell hardness of a steel specimen was measured to be 250 HB. What is the material's
approximate tensile strength? What is the hardness on the Vickers scale? The Rockwell scale?

Given: Brinell hardness of specimen HB  250

Solution: See Mathcad file P0213.


1. Determine the approximate tensile strength of the material from equations (2.10), not Table 2-3.

S ut  0.5 HB ksi S ut  125  ksi S ut  862  MPa

2. From Table 2-3 (using linear interpolation) the hardness on the Vickers scale is

HB  241
HV   ( 292  253 )  253 HV  263
277  241

3. From Table 2-3 (using linear interpolation) the hardness on the Rockwell C scale is

HB  241
HRC   ( 28.8  22.8)  22.8 HRC  24.3
277  241

© 2011 Pearson Education, Inc., Upper Saddle River, NJ. All rights reserved. This publication is protected by Copyright and written permission should be
MACHINE DESIGN - An Integrated Approach, 4th Ed. 2-14-1

PROBLEM 2-14
Statement: The Brinell hardness of a steel specimen was measured to be 340 HB. What is the material's
approximate tensile strength? What is the hardness on the Vickers scale? The Rockwell scale?

Given: Brinell hardness of specimen HB  340

Solution: See Mathcad file P0214.


1. Determine the approximate tensile strength of the material from equations (2.10), not Table 2-3.

S ut  0.5 HB ksi S ut  170  ksi S ut  1172 MPa

2. From Table 2-3 (using linear interpolation) the hardness on the Vickers scale is

HB  311
HV   ( 360  328 )  328 HV  359
341  311

3. From Table 2-3 (using linear interpolation) the hardness on the Rockwell C scale is

HB  311
HRC   ( 36.6  33.1)  33.1 HRC  36.5
341  311

© 2011 Pearson Education, Inc., Upper Saddle River, NJ. All rights reserved. This publication is protected by Copyright and written permission should be
MACHINE DESIGN - An Integrated Approach, 4th Ed. 2-15-1
PROBLEM 2-15
Statement: What are the principal alloy elements of an AISI 4340 steel? How much carbon does it have? Is it
hardenable? By what techniques?
Solution: See Mathcad file P0215.
1. Determine the principal alloying elements from Table 2-5 for 43xx steel..
1.82% Nickel
0.50 or 0.80% Chromium
0.25% Molybdenum

2. From "Steel Numbering Systems" in Section 2.6, the carbon content is

From the last two digits, the carbon content is 0.40%.

3. Is it hardenable? Yes, all of the alloying elements increase the hardenability. By what techniques? It can be
through hardened by heating, quenching and tempering; and it can also be case hardened (See Section 2.4).

© 2011 Pearson Education, Inc., Upper Saddle River, NJ. All rights reserved. This publication is protected by Copyright and written permission should be
MACHINE DESIGN - An Integrated Approach, 4th Ed. 2-16-1
PROBLEM 2-16
Statement: What are the principal alloy elements of an AISI 1095 steel? How much carbon does it have? Is it
hardenable? By what techniques?
Solution: See Mathcad file P0216.
1. Determine the principal alloying elements from Table 2-5 for 10xx steel.
Carbon only, no alloying elements
2. From "Steel Numbering Systems" in Section 2.6, the carbon content is

From the last two digits, the carbon content is 0.95%.

3. Is it hardenable? Yes, as a high-carbon steel, it has sufficient carbon content for hardening. By what
techniques? It can be through hardened by heating, quenching and tempering; and it can also be case
hardened (See Section 2.4).

© 2011 Pearson Education, Inc., Upper Saddle River, NJ. All rights reserved. This publication is protected by Copyright and written permission should be
MACHINE DESIGN - An Integrated Approach, 4th Ed. 2-17-1
PROBLEM 2-17
Statement: What are the principal alloy elements of an AISI 6180 steel? How much carbon does it have? Is it
hardenable? By what techniques?
Solution: See Mathcad file P0217.
1. Determine the principal alloying elements from Table 2-5 for 61xx steel..
0.15% Vanadium
0.60 to 0.95% Chromium
2. From "Steel Numbering Systems" in Section 2.6, the carbon content is

From the last two digits, the carbon content is 0.80%.

3. Is it hardenable? Yes, all of the alloying elements increase the hardenability. By what techniques? It can be
through hardened by heating, quenching and tempering; and it can also be case hardened (See Section 2.4).

© 2011 Pearson Education, Inc., Upper Saddle River, NJ. All rights reserved. This publication is protected by Copyright and written permission should be
MACHINE DESIGN - An Integrated Approach, 4th Ed. 2-18-1
PROBLEM 2-18
Statement: Which of the steels in Problems 2-15, 2-16, and 2-17 is the stiffest?

Solution: See Mathcad file P0218.


1. None. All steel alloys have the same Young's modulus, which determines stiffness.

© 2011 Pearson Education, Inc., Upper Saddle River, NJ. All rights reserved. This publication is protected by Copyright and written permission should be
MACHINE DESIGN - An Integrated Approach, 4th Ed. 2-19-1

PROBLEM 2-19
Statement: Calculate the specific strength and specific stiffness of the following materials and pick one for
use in an aircraft wing spar.

Given: Material Code Ultimate Strength Young's Modulus Weight Density


6 lbf
Steel st  0 Sut  80 ksi E  30 10  psi γ  0.28
st st st 3
in
6 lbf
Aluminum al  1 Sut  60 ksi E  10.4 10  psi γ  0.10
al al al 3
in
6 lbf
Titanium ti  2 Sut  90 ksi E  16.5 10  psi γ  0.16
ti ti ti 3
in

Index i  0 1  2

Solution: See Mathcad file P0219.


1. Specific strength is the ultimate tensile strength divided by the weight density and specific stiffness is the
modulus of elasticity divided by the weight density. The text does not give a symbol to these quantities.

Sut E
i 1 i 1
Specific strength   Specific stiffness  
γ in γ in
i i
286·103 Steel 107·106
600·103 Aluminum 104·106
Titanium
563·103 103·106

2. Based on the results above, all three materials have the same specific stiffness but the aluminum has the largest
specific strength. Aluminum for the aircraft wing spar is recommended.

© 2011 Pearson Education, Inc., Upper Saddle River, NJ. All rights reserved. This publication is protected by Copyright and written permission should be
MACHINE DESIGN - An Integrated Approach, 4th Ed. 2-20-1
PROBLEM 2-20
Statement: If maximum impact resistance were desired in a part, which material properties would you look for?
Solution: See Mathcad file P0220.
1. Ductility and a large modulus of toughness (see "Impact Resistance" in Section 2.1).

© 2011 Pearson Education, Inc., Upper Saddle River, NJ. All rights reserved. This publication is protected by Copyright and written permission should be
MACHINE DESIGN - An Integrated Approach, 4th Ed. 2-21-1

PROBLEM 2-21 _____

Statement: Refer to the tables of material data in Appendix A and determine the strength-to-weight ratios of
the following material alloys based on their tensile yield strengths: heat-treated 2024 aluminum,
SAE 1040 cold-rolled steel, Ti-75A titanium, type 302 cold-rolled stainless steel.

Given: Material Yield Strength Specific Weight


3 kN
Mat  "2024 Aluminum, HT" Sy  290  MPa γ  0.10 lbf  in γ  27.14 
1 1 1 1 3
m
3 kN
Mat  "1040 CR Steel" Sy  490  MPa γ  0.28 lbf  in γ  76.01 
2 2 2 2 3
m
3 kN
Mat  "Ti-75A Titanium" Sy  517  MPa γ  0.16 lbf  in γ  43.43 
3 3 3 3 3
m
3 kN
Mat  "Type 302 CR SS" Sy  1138 MPa γ  0.28 lbf  in γ  76.01 
4 4 4 4 3
m

i  1 2  4
Solution: See Mathcad file P0221.

1. Calculate the strength-to-weight ratio for each material as described in Section 2.1.

Sy SWR
i i
SWR  
i γ 4
i 10  m
 "2024 Aluminum, HT"  1.068
0.645
Mat   
"1040 CR Steel"
i  "Ti-75A Titanium"  1.190
 "Type 302 CR SS" 
  1.497

© 2011 Pearson Education, Inc., Upper Saddle River, NJ. All rights reserved. This publication is protected by Copyright and written permission should be
MACHINE DESIGN - An Integrated Approach, 4th Ed. 2-22-1

PROBLEM 2-22 _____

Statement: Refer to the tables of material data in Appendix A and determine the strength-to-weight ratios of
the following material alloys based on their ultimate tensile strengths: heat-treated 2024 aluminum
SAE 1040 cold-rolled steel, unfilled acetal plastic, Ti-75A titanium, type 302 cold-rolled stainless
steel.

Given: Material Tensile Strength Specific Weight


3 3
Mat  "2024 Aluminum, HT" Sut  441  MPa γ  0.10 lbf  in γ  27.14  kN  m
1 1 1 1
3 3
Mat  "1040 CR Steel" Sut  586  MPa γ  0.28 lbf  in γ  76.01  kN  m
2 2 2 2
3 3
Mat  "Acetal, unfilled" Sut  60.7 MPa γ  0.051  lbf  in γ  13.84  kN  m
3 3 3 3
3 3
Mat  "Ti-75A Titanium" Sut  586  MPa γ  0.16 lbf  in γ  43.43  kN  m
4 4 4 4
3 3
Mat  "Type 302 CR SS" Sut  1310 MPa γ  0.28 lbf  in γ  76.01  kN  m
5 5 5 5

i  1 2  5

Solution: See Mathcad file P0222.

1. Calculate the strength-to-weight ratio for each material as described in Section 2.1.

Sut SWR
i i
SWR  
i γ 4
i 10  m
 "2024 Aluminum, HT"  1.625
 "1040 CR Steel"  0.771
 
Mat   "Acetal, unfilled"  0.438
i
 "Ti-75A Titanium"  1.349
  1.724
 "Type 302 CR SS" 

© 2011 Pearson Education, Inc., Upper Saddle River, NJ. All rights reserved. This publication is protected by Copyright and written permission should be
MACHINE DESIGN - An Integrated Approach, 4th Ed. 2-23-1

PROBLEM 2-23 _____

Statement: Refer to the tables of material data in Appendix A and calculate the specific stiffness of
aluminum, titanium, gray cast iron, ductile iron, bronze, carbon steel, and stainless steel.
Rank them in increasing order of this property and discuss the engineering significance of
these data.

3
Units: Mg  10  kg

Given: Material Modulus of Elasticity Density


3
Mat  "Aluminum" E  71.7 GPa ρ  2.8 Mg m
1 1 1
3
Mat  "Titanium" E  113.8  GPa ρ  4.4 Mg m
2 2 2
3
Mat  "Gray cast iron" E  103.4  GPa ρ  7.2 Mg m
3 3 3
3
Mat  "Ductile iron" E  168.9  GPa ρ  6.9 Mg m
4 4 4
3
Mat  "Bronze" E  110.3  GPa ρ  8.6 Mg m
5 5 5
3
Mat  "Carbon steel" E  206.8  GPa ρ  7.8 Mg m
6 6 6
3
Mat  "Stainless steel" E  189.6  GPa ρ  7.8 Mg m
7 7 7

i  1 2  7

Solution: See Mathcad file P0223.


1. Calculate the specific stiffness for each material as described in Section 2.1.

E E' 2
i i s
E'   
i ρ 6 2
i 10 m
 "Aluminum" 
  25.6
 "Titanium"  25.9
 "Gray cast iron"  14.4
Mat   "Ductile iron" 
i   24.5
 "Bronze"  12.8
 "Carbon steel"  26.5
 "Stainless steel" 
  24.3

2. Rank them in increasing order of specific stiffness.


E' 2
5 s
Mat  "Bronze"   12.8
5 6 2
10 m
E' 2
3 s
Mat  "Gray cast iron"   14.4
3 6 2
10 m
E' 2
7 s
Mat  "Stainless steel"   24.3
7 6 2
10 m
© 2011 Pearson Education, Inc., Upper Saddle River, NJ. All rights reserved. This publication is protected by Copyright and written permission should be
MACHINE DESIGN - An Integrated Approach, 4th Ed. 2-23-2

E' 2
4 s
Mat  "Ductile iron"   24.5
4 6 2
10 m
E' 2
1 s
Mat  "Aluminum"   25.6
1 6 2
10 m
E' 2
2 s
Mat  "Titanium"   25.9
2 6 2
10 m

E' 2
6 s
Mat  "Carbon steel"   26.5
6 6 2
10 m

3. Bending and axial deflection are inversely proportional to the modulus of elasticity. For the same shape and
dimensions, the material with the highest specific stiffness will give the smallest deflection. Or, put another
way, for a given deflection, using the material with the highest specific stiffness will result in the least
weight.

© 2011 Pearson Education, Inc., Upper Saddle River, NJ. All rights reserved. This publication is protected by Copyright and written permission should be
MACHINE DESIGN - An Integrated Approach, 4th Ed. 2-24-1

PROBLEM 2-24
Statement: Call your local steel and aluminum distributors (consult the Yellow Pages) and obtain current
costs per pound for round stock of consistent size in low-carbon (SAE 1020) steel, SAE 4340
steel, 2024-T4 aluminum, and 6061-T6 aluminum. Calculate a strength/dollar ratio and a
stiffness/dollar ratio for each alloy. Which would be your first choice on a cost-efficiency
basis for an axial-tension-loaded round rod
(a) If maximum strength were needed?
(b) If maximum stiffness were needed?

Solution: Left to the student as data will vary with time and location.

© 2011 Pearson Education, Inc., Upper Saddle River, NJ. All rights reserved. This publication is protected by Copyright and written permission should be
MACHINE DESIGN - An Integrated Approach, 4th Ed. 2-25-1

PROBLEM 2-25
Statement: Call your local plastic stock-shapes distributors (consult the Yellow Pages) and obtain current
costs per pound for round rod or tubing of consistent size in plexiglass, acetal, nylon 6/6, and
PVC. Calculate a strength/dollar ratio and a stiffness/dollar ratio for each alloy. Which would
be your first choice on a cost-efficiency basis for an axial-tension-loaded round rod or tube of
particular diameters.
(a) If maximum strength were needed?
(b) If maximum stiffness were needed?

Solution: Left to the student as data will vary with time and location.

© 2011 Pearson Education, Inc., Upper Saddle River, NJ. All rights reserved. This publication is protected by Copyright and written permission should be
MACHINE DESIGN - An Integrated Approach, 4th Ed. 2-26-1
PROBLEM 2-26
Statement: A part has been designed and its dimensions cannot be changed. To minimize its deflections
under the same loading in all directions irrespective of stress levels, which material woulod you
choose among the following: aluminum, titanium, steel, or stainless steel?

Solution: See Mathcad file P0226.

1. Choose the material with the highest modulus of elasticity because deflection is inversely proportional to
modulus of elasticity. Thus, choose steel unless there is a corrosive atmosphere, in which case, choose
stainless steel.

© 2011 Pearson Education, Inc., Upper Saddle River, NJ. All rights reserved. This publication is protected by Copyright and written permission should be
MACHINE DESIGN - An Integrated Approach, 4th Ed. 2-27-1
PROBLEM 2-27
Statement: Assuming that the mechanical properties data given in Appendix Table A-9 for some carbon
steels represents mean values, what is the value of the tensile yield strength for 1050 steel
quenched and tempered at 400F if a reliability of 99.9% is required?

Given: Mean yield strength S y  117  ksi S y  807  MPa

Solution: See Mathcad file P0227.

1. From Table 2-2 the reliability factor for 99.9% is Re  0.753. Applying this to the mean tensile strength gives

S y99.9  S y Re S y99.9  88.1 ksi S y99.9  607  MPa

© 2011 Pearson Education, Inc., Upper Saddle River, NJ. All rights reserved. This publication is protected by Copyright and written permission should be
MACHINE DESIGN - An Integrated Approach, 4th Ed. 2-28-1

PROBLEM 2-28
Statement: Assuming that the mechanical properties data given in Appendix Table A-9 for some carbon
steels represents mean values, what is the value of the ultimate tensile strength for 4340 steel
quenched and tempered at 800F if a reliability of 99.99% is required?

Given: Mean ultimate tensile strength S ut  213  ksi S ut  1469 MPa

Solution: See Mathcad file P0228.

1. From Table 2-2 the reliability factor for 99.99% is Re  0.702. Applying this to the mean ultimate tensile
strength gives

S ut99.99  S ut Re S ut99.99  150  ksi S ut99.99  1031 MPa

© 2011 Pearson Education, Inc., Upper Saddle River, NJ. All rights reserved. This publication is protected by Copyright and written permission should be
MACHINE DESIGN - An Integrated Approach, 4th Ed. 2-29-1

PROBLEM 2-29
Statement: Assuming that the mechanical properties data given in Appendix Table A-9 for some carbon
steels represents mean values, what is the value of the ultimate tensile strength for 4130 steel
quenched and tempered at 400F if a reliability of 90% is required?

Given: Mean ultimate tensile strength S ut  236  ksi S ut  1627 MPa

Solution: See Mathcad file P0229.

1. From Table 2-2 the reliability factor for 90% is Re  0.897. Applying this to the mean ultimate
tensile strength gives

S ut99.99  S ut Re S ut99.99  212  ksi S ut99.99  1460 MPa

© 2011 Pearson Education, Inc., Upper Saddle River, NJ. All rights reserved. This publication is protected by Copyright and written permission should be
MACHINE DESIGN - An Integrated Approach, 4th Ed. 2-30-1

PROBLEM 2-30
Statement: Assuming that the mechanical properties data given in Appendix Table A-9 for some carbon
steels represents mean values, what is the value of the tensile yield strength for 4140 steel
quenched and tempered at 800F if a reliability of 99.999% is required?

Given: Mean yield strength S y  165  ksi S y  1138 MPa

Solution: See Mathcad file P0230.

1. From Table 2-2 the reliability factor for 99.999% is Re  0.659. Applying this to the mean
tensile
strength gives
S y99.9  S y Re S y99.9  109  ksi S y99.9  750  MPa

© 2011 Pearson Education, Inc., Upper Saddle River, NJ. All rights reserved. This publication is protected by Copyright and written permission should be
MACHINE DESIGN - An Integrated Approach, 4th Ed. 2-31-1

PROBLEM 2-31
Statement: A steel part is to be plated to give it better corrosion resistance. Two materials are being
considered: cadmium and nickel. Considering only the problem of galvanic action, which would
you chose? Why?

Solution: See Mathcad file P0231.

1. From Table 2-4 we see that cadmium is closer to steel than nickel. Therefore, from the standpoint of reduced
galvanic action, cadmium is the better choice. Also, since cadmium is less noble than steel it will be the material
that is consumed by the galvanic action. If nickel were used the steel would be consumed by galvanic action.

© 2011 Pearson Education, Inc., Upper Saddle River, NJ. All rights reserved. This publication is protected by Copyright and written permission should be
MACHINE DESIGN - An Integrated Approach, 4th Ed. 2-32-1

PROBLEM 2-32
Statement: A steel part with many holes and sharp corners is to be plated with nickel. Two processes are
being considered: electroplating and electroless plating. Which process would you chose? Why?

Solution: See Mathcad file P0232.

1. Electroless plating is the better choice since it will give a uniform coating thickness in the sharp corners and in
the holes. It also provides a relatively hard surface of about 43 HRC.

© 2011 Pearson Education, Inc., Upper Saddle River, NJ. All rights reserved. This publication is protected by Copyright and written permission should be
MACHINE DESIGN - An Integrated Approach, 4th Ed. 2-33-1

PROBLEM 2-33
Statement: What is the common treatment used on aluminum to prevent oxidation? What other metals can
also be treated with this method? What options are available with this method?

Solution: See Mathcad file P0233.

1. Aluminum is commonly treated by anodizing, which creates a thin layer of aluminum oxide on the surface.
Titanium, magnesium, and zinc can also be anodized. Common options include tinting to give various colors to
the surface and the use of "hard anodizing" to create a thicker, harder surface.

© 2011 Pearson Education, Inc., Upper Saddle River, NJ. All rights reserved. This publication is protected by Copyright and written permission should be
MACHINE DESIGN - An Integrated Approach, 4th Ed. 2-34-1

PROBLEM 2-34
Statement: Steel is often plated with a less nobel metal that acts as a sacrificial anode that will corrode instead
of the steel. What metal is commonly used for this purpose (when the finished product will not be
exposed to saltwater), what is the coating process called, and what are the common processes used
to obtain the finished product?

Solution: See Mathcad file P0234.

1. The most commonly used metal is zinc. The process is called "galvanizing" and it is accomplished by
electroplating or hot dipping.

© 2011 Pearson Education, Inc., Upper Saddle River, NJ. All rights reserved. This publication is protected by Copyright and written permission should be
MACHINE DESIGN - An Integrated Approach, 4th Ed. 2-35-1

PROBLEM 2-35
Statement: A low-carbon steel part is to be heat-treated to increase its strength. If an ultimate tensile
strength of approximately 550 MPa is required, what mean Brinell hardness should the part
have after treatment? What is the equivalent hardness on the Rockwell scale?

Given: Approximate tensile strength S ut  550  MPa

Solution: See Mathcad file P0235.

1. Use equation (2.10), solving for the Brinell hardness, HB.

S ut
S ut = 3.45 HB HB  HB  159
3.45 MPa

2. From Table 2-3, the equivalent hardness on the Rocwell scale is 83.9HRB.

© 2011 Pearson Education, Inc., Upper Saddle River, NJ. All rights reserved. This publication is protected by Copyright and written permission should be
MACHINE DESIGN - An Integrated Approach, 4th Ed. 2-36-1

PROBLEM 2-36
Statement: A low-carbon steel part has been tested for hardness using the Brinell method and is found to
have a hardness of 220 HB. What are the approximate lower and upper limits of the ultimate
tensile strength of this part in MPa?

Given: Hardness HB  220

Solution: See Mathcad file P0236.

1. Use equation (2.10), solving for ultimate tensile strength.

Minimum: S utmin  ( 3.45 HB  0.2 HB)  MPa S utmin  715  MPa

Maximum: S utmax  ( 3.45 HB  0.2 HB)  MPa S utmax  803  MPa

© 2011 Pearson Education, Inc., Upper Saddle River, NJ. All rights reserved. This publication is protected by Copyright and written permission should be
MACHINE DESIGN - An Integrated Approach, 4th Ed. 2-37-1

PROBLEM 2-37
Statement: Figure 2-24 shows "guide lines" for minimum weight design when failure is the criterion. The
guide line, or index, for minimizing the weight of a beam in bending is f2/3/, where f is the
yield strength of a material and  is its mass density. For a given cross-section shape the
weight of a beam with given loading will be minimized when this index is maximized. The
following materials are being considered for a beam application: 5052 aluminum, cold rolled;
CA-170 beryllium copper, hard plus aged; and 4130 steel, Q & T @ 1200F. The use of which of
these three materials will result in the least-weight beam?
3
Units: Mg  kg
3
Given: 5052 Aluminum S ya  255  MPa ρa  2.8 Mg m
3
CA-170 beryllium copper S yb  1172 MPa ρb  8.3 Mg m
3
4130 steel S ys  703  MPa ρs  7.8 Mg m

Solution: See Mathcad file P0237.


1. The values for the mass density are taken from Appendix Table A-1 and the values of yield strength come
from from Tables A-2, A-4, and A-9 for aluminum, beryllium copper, and steel, respectively.
2. Calculate the index value for each material.

0.667 3
Sy Mg m
Index S y ρ   
ρ 0.667
MPa

Aluminum Ia  Index S ya ρa  Ia  14.4

Beryllium copper Ib  Index S yb ρb  Ib  13.4

Steel Is  Index S ys ρs  Is  10.2

The 5052 aluminum has the highest value of the index and would be the best choice to minimize weight.

© 2011 Pearson Education, Inc., Upper Saddle River, NJ. All rights reserved. This publication is protected by Copyright and written permission should be
MACHINE DESIGN - An Integrated Approach, 4th Ed. 2-38-1

PROBLEM 2-38
Statement: Figure 2-24 shows "guide lines" for minimum weight design when failure is the criterion. The
guide line, or index, for minimizing the weight of a member in tension is f/, where f is the
yield strength of a material and  is its mass density. The weight of a member with given
loading will be minimized when this index is maximized. For the three materials given in Problem
2-37, which will result in the least weight tension member?
3
Units: Mg  kg
3
Given: 5052 Aluminum S ya  255  MPa ρa  2.8 Mg m
3
CA-170 beryllium copper S yb  1172 MPa ρb  8.3 Mg m
3
4130 steel S ys  703  MPa ρs  7.8 Mg m

Solution: See Mathcad file P0238.


1. The values for the mass density are taken from Appendix Table A-1 and the values of yield strength come
from from Tables A-2, A-4, and A-9 for aluminum, beryllium copper, and steel, respectively.
2. Calculate the index value for each material.

S y Mg m 3
Index S y ρ   
ρ MPa

Aluminum Ia  Index S ya ρa  Ia  91.1

Beryllium copper Ib  Index S yb ρb  Ib  141.2

Steel Is  Index S ys ρs  Is  90.1

The beryllium copper has the highest value of the index and would be the best choice to minimize weight.

© 2011 Pearson Education, Inc., Upper Saddle River, NJ. All rights reserved. This publication is protected by Copyright and written permission should be
MACHINE DESIGN - An Integrated Approach, 4th Ed. 2-39-1

PROBLEM 2-39
Statement: Figure 2-23 shows "guide lines" for minimum weight design when stiffness is the criterion. The
guide line, or index, for minimizing the weight of a beam in bending is E1/2/, where E is the
modulus of elasticity of a material and  is its mass density. For a given cross-section shape the
weight of a beam with given stiffness will be minimized when this index is maximized. The
following materials are being considered for a beam application: 5052 aluminum, cold rolled;
CA-170 beryllium copper, hard plus aged; and 4130 steel, Q & T @ 1200F. The use of which of
these three materials will result in the least-weight beam?
3
Units: Mg  kg
3
Given: 5052 Aluminum Ea  71.7 GPa ρa  2.8 Mg m
3
CA-170 beryllium copper Eb  127.6  GPa ρb  8.3 Mg m
3
4130 steel Es  206.8  GPa ρs  7.8 Mg m

Solution: See Mathcad file P0239.


1. The values for the mass density and modulus are taken from Appendix Table A-1.
2. Calculate the index value for each material.

0.5 3
E Mg m
Index( E ρ )  
ρ 0.5
GPa

Aluminum Ia  Index Ea ρa  Ia  3.0

Beryllium copper Ib  Index Eb ρb  Ib  1.4

Steel Is  Index Es ρs  Is  1.8

The 5052 aluminum has the highest value of the index and would be the best choice to minimize weight.

© 2011 Pearson Education, Inc., Upper Saddle River, NJ. All rights reserved. This publication is protected by Copyright and written permission should be
MACHINE DESIGN - An Integrated Approach, 4th Ed. 2-40-1

PROBLEM 2-40
Statement: Figure 2-24 shows "guide lines" for minimum weight design when stiffness is the criterion. The
guide line, or index, for minimizing the weight of a member in tension is E/, where E is the
modulus of elasticity of a material and  is its mass density. The weight of a member with given
stiffness will be minimized when this index is maximized. For the three materials given in Problem
2-39, which will result in the least weight tension member?

3
Units: Mg  kg
3
Given: 5052 Aluminum Ea  71.7 GPa ρa  2.8 Mg m
3
CA-170 beryllium copper Eb  127.6  GPa ρb  8.3 Mg m
3
4130 steel Es  206.8  GPa ρs  7.8 Mg m

Solution: See Mathcad file P0240.


1. The values for the mass density and modulus are taken from Appendix Table A-1.
2. Calculate the index value for each material.

3
E Mg m
Index( E ρ )  
ρ GPa

Aluminum Ia  Index Ea ρa  Ia  25.6

Beryllium copper Ib  Index Eb ρb  Ib  15.4

Steel Is  Index Es ρs  Is  26.5

The steel has the highest value of the index and would be the best choice to minimize weight.

© 2011 Pearson Education, Inc., Upper Saddle River, NJ. All rights reserved. This publication is protected by Copyright and written permission should be
MACHINE DESIGN - An Integrated Approach, 4th Ed. 3-1-1
PROBLEM 3-1
Statement: Which load class from Table 3-1 best suits these systems?
(a) Bicycle frame
(b) Flag pole
(c) Boat oar
(d) Diving board
(e) Pipe wrench
(f) Golf club.

Solution: See Mathcad file P0301.

1. Determine whether the system has stationary or moving elements, and whether the there are constant or
time-varying loads.

(a) Bicycle frame Class 4 (Moving element, time-varying loads)

(b) Flag pole Class 2 (Stationary element, time-varying loads)

(c) Boat oar Class 2 (Low acceleration element, time-varying loads)

(d) Diving board Class 2 (Stationary element, time-varying loads)

(e) Pipe wrench Class 2 (Low acceleration elements, time-varying loads)

(f) Golf club Class 4 (Moving element, time-varying loads)

© 2011 Pearson Education, Inc., Upper Saddle River, NJ. All rights reserved. This publication is protected by Copyright and written permission should be
MACHINE DESIGN - An Integrated Approach, 4th Ed. 3-2a-1
PROBLEM 3-2a
Statement: Draw free-body diagrams for the system of Problem 3-1a (bicycle frame).
Assumptions: 1. A two-dimensional model is adequate.
2. The lower front-fork bearing at C takes all of the thrust load from the front forks.
3. There are no significant forces on the handle bars.

Solution: See Mathcad file P0302a.

1. A typical bicycle frame is shown in Figure 3-2a. There are five points on the frame where external forces and
moments are present. The rider's seat is mounted through a tube at A. This is a rigid connection, capable of
transmitting two force components and a moment. The handle bars and front-wheel forks are supported by the
frame through two bearings, located at B and C. These bearings are capable of transmitting radial and axial
loads. The pedal-arm assembly is supported by bearings at D. These bearings are capable of transmitting radial
loads. The rear wheel-sprocket assembly is supported by bearings mounted on an axle fixed to the frame at E.

Ma Ra Rb B
A
Fax Fbr Fct
Fcr
Rc α

Fay C

Rd
Re

Fey

Fex
E
Fdx D

Fdy

FIGURE 3-2a
Free Body Diagram for Problem 3-2a

2. The loads at B and C can be determined by analyzing a FBD of the front wheel-front forks assembly. The loads
at D can be determined by analyzing a FBD of the pedal-arm and front sprocket (see Problem 3-3), and the loads at E
can be determined by analyzing a FBD of the rear wheel-sprocket assembly.

3. With the loads at B, C, D, and E known, we can apply equations 3.3b to the FBD of the frame and solve for Fax ,
Fay , and Ma.

Σ Fx : −Fax − Fbr⋅ cos( α) + Fcr⋅ cos( α) − Fct⋅ sin( α) − Fdx + Fex = 0 (1)

Σ Fy : −Fay − Fbr⋅ sin( α) + Fcr⋅ sin( α) + Fct⋅ cos( α) − Fdy + Fey = 0 (2)

Σ Mz: Ma + ( Rbx ⋅ Fby − Rby⋅ Fbx ) + ( Rcx⋅ Fcy − Rcy⋅ Fcx) ... = 0 (3)
+ ( Rdx⋅ Fdy − Rdy ⋅ Fdx) + ( Rex⋅ Fey − Rey⋅ Fex)

© 2011 Pearson Education, Inc., Upper Saddle River, NJ. All rights reserved. This publication is protected by Copyright and written permission should be
MACHINE DESIGN - An Integrated Approach, 4th Ed. 3-2e-1
PROBLEM 3-2e
Statement: Draw free-body diagrams for the system of Problem 3-1e (pipe wrench).
Assumptions: A two-dimensional model is adequate.
Solution: See Mathcad file P0302e.

1. A typical pipe wrench with a pipe clamped in its jaw is shown in Figure 3-2e(a). When a force Fhand is applied on
the wrench, the piping system provides an equal and opposite force and a resisting torque, Tpipe.

Fhand

Tpipe
Fhand

(a) FBD of pipe wrench and pipe

Fbt

Fbn
Fax A

α
Fay

d b

(b) FBD of pipe wrench only

FIGURE 3-2e
Free Body Diagrams for Problem 3-2e

2. The pipe reacts with the wrench at the points of contact A and B. The forces here will be directed along the
common normals and tangents. The jaws are slightly tapered and, as a result, the action of Fhand tends to drive the
wrench further into the taper, increasing the normal forces. This, in turn, allows for increasing tangential forces. It is
the tangential forces that produce the turning torque.
3. Applying equations 3.3b to the FBD of the pipe wrench,

Σ Fx : −Fax + Fbn⋅ cos( α) − Fbt⋅ sin( α) = 0 (1)

Σ Fy : −Fay + Fbn⋅ sin( α) + Fbt⋅ cos( α) − Fhand = 0 (2)

Σ M A: d ⋅ ( Fbt⋅ cos( α) + Fbn⋅ sin( α) ) − ( d + a ) ⋅ Fhand = 0 (3)

4. These equations can be solved for the vertical forces if we assume α is small so that sin(α) = 0 and cos (α) = 1.
© 2011 Pearson Education, Inc., Upper Saddle River, NJ. All rights reserved. This publication is protected by Copyright and written permission should be
MACHINE DESIGN - An Integrated Approach, 4th Ed. 3-3-1
PROBLEM 3-3
Statement: Draw a free-body diagram of the pedal-arm assembly from a bicycle with the pedal arms in the
horizontal position and dimensions as shown in Figure P3-1. (Consider the two arms, pedals and
pivot as one piece). Assuming a rider-applied force of 1500 N at the pedal, determine the torque
applied to the chain sprocket and the maximum bending moment and torque in the pedal arm.

Given: a  170  mm b  60 mm Frider  1.5 kN

Assumptions: The pedal-arm assembly is supprted by bearings at A and at B.


Solution: See Figure 3-3 and Mathcad file P0303.
1. The free-body diagram (FBD) of the pedal-arm assembly (including the sprocket) is shown in Figure 3-3a. The
rider-applied force is Frider and the force applied by the chain (not shown) is Fchain. The radial bearing
reactions are Fax, Faz, Fbx, and Fbz. Thus, there are five unknowns Fchain, Fax, Faz, Fbx, and Fbz. In general, we
can write six equilibrium equations for a three-dimensional force system, but in this system there are no forces
in the y-direction so five equations are available to solve for the unknowns.
z

Fchain

Sprocket
Faz
a Fbz

Frider
A
b Arm B Arm (sectioned)
Fax
Fbx y
Pedal

(a) FBD of complete pedal-arm assembly

a
Tc
Frider
Mc
b Arm

Fc y
Pedal

(b) FBD of pedal and arm with section through the origin

FIGURE 3-3
Dimensions and Free Body Diagram of the pedal-arm assembly for Problem 3-3

2. The torque available to turn the sprocket is found by summing moments about the sprocket axis. From Figure
3-3a, it is
© 2011 Pearson Education, Inc., Upper Saddle River, NJ. All rights reserved. This publication is protected by Copyright and written permission should be
MACHINE DESIGN - An Integrated Approach, 4th Ed. 3-3-2

 Ty-axis: a  Frider  r Fchain = a  Frider  Tsprocket = 0

Tsprocket  a  Frider Tsprocket  255  N  m

where r is the sprocket pitch radius.

3. In order to determine the bending moment and twisting torque in the pedal arm, we will cut the arm with a
section plane that goes through the origin and is parallel to the y-z plane, removing everything beyond that
plane and replacing it with the internal forces and moments in the pedal arm at the section. The resulting
FBD is shown in Figure 3-3b. The internal force at section C is Fc the internal bending moment is Mc, and
the internal twisting moment (torque) is Tc. We can write three equilibrium equations to solve for these
three unknowns:

Shear force in pedal arm at section C


 Fz : Fc  Frider = 0 Fc  Frider Fc  1.5 kN

Bending moment in pedal arm at section C

 My-axis: a  Frider  Mc = 0 Mc  a  Frider Mc  255  N  m

Twisting moment in pedal arm at section C

 Mx-axis: b  Frider  Tc = 0 Tc  b  Frider Tc  90 N  m

© 2011 Pearson Education, Inc., Upper Saddle River, NJ. All rights reserved. This publication is protected by Copyright and written permission should be
MACHINE DESIGN - An Integrated Approach,4th Ed. 3-4-1
PROBLEM 3-4
Statement: The trailer hitch from Figure 1-1 has loads applied as shown in Figure P3-2. The tongue weight
of 100 kg acts downward and the pull force of 4905 N acts horizontally. Using the dimensions
of the ball bracket in Figure 1-5 (p. 15), draw a free-body diagram of the ball bracket and find the
tensile and shear loads applied to the two bolts that attach the bracket to the channel in Figure
1-1.

Given: a  40 mm b  31 mm c  70 mm d  20 mm


Mtongue  100  kg Fpull  4.905  kN t  19 mm

Assumptions: 1. The nuts are just snug-tight (no pre-load), which is the worst case.
2. All reactions will be concentrated loads rather than distributed loads or pressures.
Solution: See Figure 3-4 and Mathcad file P0304.

1. The weight on the tongue is

Wtongue  Mtongue g Wtongue  0.981  kN

2. The FBD of the hitch and bracket assembly is shown in Figure 3-4. The known external forces that act on the
ball are Fpull and Wtongue . The reactions on the bracket are at points C and D. The bolts at C provide tensile
(Fc2x) and shear (Fc2y) forces, and the bracket resists rotation about point D where the reaction force Fd2 is
applied by the channel to which the bracket is bolted.
3. Solving for the reactions by summing the horizontal and vertical forces and the moments about D:

 Fx :  Fpull  Fc2x  Fd2 = 0 (1)

 Fy : Fc2y  Wtongue = 0 (2)

 MD : Fc2x d  Fpull  ( a  t  b  d )  Wtongue c = 0 (3)

W tongue
70 = c

1 F pull 1

40 = a

2 A 2
B 19 = t B

31 = b Fc2x
C C

20 = d
D
D Fd2

F c2y

FIGURE 3-4
Dimensions and Free Body Diagram for Problem 3-4

4. Solving equation (3) for Fc2x


© 2011 Pearson Education, Inc., Upper Saddle River, NJ. All rights reserved. This publication is protected by Copyright and written permission should be
MACHINE DESIGN - An Integrated Approach,4th Ed. 3-4-2

Fpull  ( a  t  b  d )  Wtongue c
Fc2x  Fc2x  30.41  kN (4)
d

5. Substituting into (1) and solving for Fd2

Fd2  Fc2x  Fpull Fd2  25.505 kN (5)

6. Solving (2) for Fc2y

Fc2y  Wtongue Fc2y  0.981  kN (6)

7. The loads applied to the two bolts that attach the bracket to the channel are:

Axial force on two bolts Fc2x  30.4 kN

Shear force on two bolts Fc2y  0.98 kN

We assume that each bolt would carry one half of these loads.

© 2011 Pearson Education, Inc., Upper Saddle River, NJ. All rights reserved. This publication is protected by Copyright and written permission should be
MACHINE DESIGN - An Integrated Approach, 4th Ed. 3-5-1
PROBLEM 3-5
Statement: For the trailer hitch of Problem 3-4, determine the horizontal force that will result on the ball from
accelerating a 2000-kg trailer to 60 m/sec in 20 sec.
Given: Mass of trailer Mtrailer  2000 kg
m
Final velocity vf  60
sec
Time to reach velocity τ  20 sec

Assumptions: 1. Acceleration is constant.


2. The rolling resistance of the tires and the wheel bearings is negligible.
Solution: See Mathcad file P0305.

1. From elementary kinematics, the acceleration required is

vf m
a  a  3.00 (1)
τ 2
sec
2. Using Newton's second law to find the force required to accelerate the trailer,

Fhitch  Mtrailer  a Fhitch  6.00 kN (2)

© 2011 Pearson Education, Inc., Upper Saddle River, NJ. All rights reserved. This publication is protected by Copyright and written permission should be
MACHINE DESIGN - An Integrated Approach, 4th Ed. 3-6-1

PROBLEM 3-6
Statement: For the trailer hitch of Problem 3-4, determine the horizontal force that will result on the ball from
an impact between the ball and the tongue of the 2000-kg trailer if the hitch deflects 2.8 mm
dynamically on impact. The tractor weighs 1000 kg. The velocity at impact is 0.3 m/sec.

Given: Mass of trailer Mtrailer  2000 kg


Dynamic deflection δi  2.8 mm
Mass of tractor Mtractor  1000 kg
m
Impact velocity vi  0.3
sec
Assumptions: 1. The tractor is the "struck member" because the hitch is on the tractor and it is the hitch that
deflects.
2. Equations (3.9) and (3.10) can be used to model the impact.

Solution: See Mathcad file P0306.

1. The weight of the trailer (the "striking member") is

Wtrailer  Mtrailer  g Wtrailer  19.613 kN

2. The correction factor, from equation (3-15), is


1
η  η  0.857
Mtractor
1
3  Mtrailer

3. Eliminating E from equations (3.9a) and (3.10) and solving for the horizontal force on the ball Fi yields

 Fi δi = η   Mtrailer  vi 
1 1 2
2 2 

2
η Mtrailer  vi
Fi  Fi  55.1 kN
δi

© 2011 Pearson Education, Inc., Upper Saddle River, NJ. All rights reserved. This publication is protected by Copyright and written permission should be
MACHINE DESIGN - An Integrated Approach, 4th Ed. 3-7-1

PROBLEM 3-7
Statement: The piston of an internal-combustion engine is connected to its connecting rod with a "wrist
pin." Find the force on the wrist pin if the 0.5-kg piston has an acceleration of 2 500 g.

Given: Mass of piston Mpiston  0.5 kg


Acceleration of piston a piston  2500 g

Assumptions: The force on the wrist pin due to the weight of the piston is very small compared with the
acceleration force.

Solution: See Mathcad file P0307.


4 m
1. The acceleration in m/s is a piston  2.452  10 
2
sec

2. Using Newton's Second Law expressed in equation (3.1a), the force on the wrist pin is

Fwristpin  Mpiston  a piston Fwristpin  12.258 kN

© 2011 Pearson Education, Inc., Upper Saddle River, NJ. All rights reserved. This publication is protected by Copyright and written permission should be
MACHINE DESIGN - An Integrated Approach, 4th Ed. 3-8-1

PROBLEM 3-8 _____

Statement: A cam-follower system similar to that shown in Figure 3-15 has a mass m = 1 kg, a spring
constant k = 1000 N/m, and a damping coefficient d = 19.4 N-s/m. Find the undamped and
damped natural frequencies of this system.

−1
Units: cps := 2 ⋅ π⋅ rad⋅ sec

−1
Given: Mass M := 1 ⋅ kg, Spring constant k := 1000⋅ N ⋅ m
−1
Damping coefficient d := 19.4⋅ N ⋅ s⋅ m

Solution: See Figure 3-15 and Mathcad file P0308.

1. Calculate the undamped natural frequency using equation 3.4.

k rad
ωn := ωn = 31.6 ωn = 5.03 cps
M sec

2. Calculate the undamped natural frequency using equation 3.7.

2
k d rad
ωd := − ωd = 30.1 ωd = 4.79 cps
M 2⋅ M sec

© 2011 Pearson Education, Inc., Upper Saddle River, NJ. All rights reserved. This publication is protected by Copyright and written permission should be
MACHINE DESIGN - An Integrated Approach, 4th Ed. 3-9-1

PROBLEM 3-9
Statement: A ViseGrip plier-wrench is drawn to scale in Figure P3-3. Scale the drawing for dimensions. Find
the forces acting on each pin and member of the assembly for an assumed clamping force of P =
4000 N in the position shown. What force F is required to keep it in the clamped position shown?

Given: Clamping force P  4.00 kN


Dimensions a  50.0 mm e  28.0 mm α  21.0 deg
b  55.0 mm f  26.9 mm β  129.2  deg
c  39.5 mm g  2.8 mm
d  22.0 mm h  21.2 mm

Assumptions: Links 3 and 4 are in a toggle position, i.e., the pin that joins links 3 and 4 is in line with the pins
that join 1 with 4 and 2 with 3.
Solution: See Figure 3-9 and Mathcad file P0309.

1. The FBDs of the assembly and each individual link are shown in Figure 3-9. The dimensions, as scaled from
Figure P3-3 in the text, are given above and are shown on the link FBDs.

F 4
P
1

3 2
P

55.0 = b 50.0 = a
F14
39.5 = c 22.0 = d
F

129.2° 1 
4

F41 F34
F21 P


28.0 = e


P 2.8 = g


F43

3 F12
21.2 = h
F23 2
F32

26.9 = f

FIGURE 3-9
Free Body Diagrams for Problem 3-9

2. Looking first at Part 3, we see that it is a three-force body. Therefore, the lines of action of the three forces
must intersect at a point. But, since Parts 3 and 4 are in a toggle position, F43 and F23 are colinear, which
means that their x- and y-components must be equal and opposite, leading to the conclussion that F = 0.
© 2011 Pearson Education, Inc., Upper Saddle River, NJ. All rights reserved. This publication is protected by Copyright and written permission should be
MACHINE DESIGN - An Integrated Approach, 4th Ed. 3-9-2

3. Now, looking at Part 1, we see that (for F = 0) it is also a three-force body, as is Part 2. In fact, the forces on
Part 1 are identical to those on Part 2. Solving for the unknown reactions on Parts 1 and 2,

 Fx: F41 cos( 180  deg  α)  F21 cos( β  180  deg) = 0 (a)

 Fy: F41 sin( 180  deg  α)  F21 sin( β  180  deg)  P = 0 (b)

Solving equation (a) for F21

F41 cos( 180  deg  α)


F21 =  (c)
cos( β  180  deg)

Substituting equation (c) into (b)

F41 cos( 180  deg  α)


F41 sin( 180  deg  α)   sin( β  180  deg)  P = 0 (d)
cos( β  180  deg)

Solving equation (d) for F41

P
F41  
cos( 180  deg  α)
sin( 180  deg  α)   sin( β  180  deg)
cos( β  180  deg)

F41 cos( 180  deg  α) F41  5.1 kN


F21  
cos( β  180  deg)
F21  7.5 kN

Checking moment balance on Part 1,

F41 sin( α)  c  F21 sin( β  90 deg)  d  P a  0  kN  m

The result is, within the accuracy of the scaled dimensions, zero as it must be.

4. The x and y components of the pin forces on Part 1 are

F41x  F41 cos( 180  deg  α) F41x  4.749  kN

F41y  F41 sin( 180  deg  α) F41y  1.823  kN

F21x  F21 cos( β  180  deg) F21x  4.749  kN

F21y  F21 sin( β  180  deg) F21y  5.823  kN

5. The forces on the pins at the ends of Part 4 are

F14  F41 F14  5.1 kN

F34  F14 F34  5.1 kN


6. The forces on the pins at the ends of Part 3 are
F43  F34 F43  5.1 kN

F23  F43 F23  5.1 kN


© 2011 Pearson Education, Inc., Upper Saddle River, NJ. All rights reserved. This publication is protected by Copyright and written permission should be
MACHINE DESIGN - An Integrated Approach, 4th Ed. 3-9-3

7. The forces on the pins at the ends of Part 2 are

F12  F21 F12  7.5 kN

F32  F23 F32  5.1 kN

Checking moment equilibrium on Part 2,


F12 ( e cos( β  90 deg)  g  sin( β  90 deg) )   0  kN  m
 F32 ( h  cos( α)  f  sin( α) )

which is zero, as it must be.

© 2011 Pearson Education, Inc., Upper Saddle River, NJ. All rights reserved. This publication is protected by Copyright and written permission should be
MACHINE DESIGN - An Integrated Approach, 4th Ed. 3-10-1
PROBLEM 3-10
Statement: An overhung diving board is shown in Figure P3-4a. Find the reaction forces and construct the
shear and moment diagrams for this board with a 100 kg person standing at the free end.
Determine the maximum shear force, maximum moment and their locations.

Given: Beam length L  2000 mm 2000 = L


Distance to support a  700  mm R1 P
Mass at free end M  100  kg

Assumptions: The weight of the beam is negligible


compared to the applied load and so can R2
be ignored.
700 = a
Solution: See Figure 3-10 and Mathcad file P0310.
FIGURE 3-10A
Free Body Diagram for Problem 3-10

1. From inspection of Figure 3-10, write the load function equation


q(x) = -R1<x - 0>-1 + R2<x - a>-1 - P<x - L >-1

2. Integrate this equation from - to x to obtain shear, V(x)


V(x) = -R1<x - 0>0 + R2<x - a>0 - P<x - L >0

3. Integrate this equation from - to x to obtain moment, M(x)


M(x) = -R1<x - 0>1 + R2<x - a>1 - P<x - L >1

4. Determine the magnitude of the force, P P  M  g P  980.7  N

5. Solve for the reactions by evaluating the shear and moment equations at a point just to the right of x = L, where
both are zero.

At x = L+, V = M = 0 V = R1  R2  P = 0 M = R1 L  R2 ( L  a ) = 0

La
R1  P R1  1821 N
a

R2  P  R1 R2  2802 N

6. Define the range for x x  0  in 0.005  L  L

7. For a Mathcad solution, define a step function S. This function will have a value of zero when x is less than z,
and a value of one when it is greater than or equal to z.

S ( x z)  if ( x  z 1 0 )

8. Write the shear and moment equations in Mathcad form, using the function S as a multiplying factor to get the
effect of the singularity functions.

V ( x)  R1 S ( x 0  in)  R2 S ( x a )  P S ( x L)

M ( x)  R1 S ( x 0  in)  x  R2 S ( x a )  ( x  a )  P S ( x L)  ( x  L)

© 2011 Pearson Education, Inc., Upper Saddle River, NJ. All rights reserved. This publication is protected by Copyright and written permission should be
MACHINE DESIGN - An Integrated Approach, 4th Ed. 3-10-2

9. Plot the shear and moment diagrams.

Shear 1000
Diagram

0
V ( x)
N
 1000

 2000
0 500 1000 1500 2000
x
mm

Moment 0
Diagram

 375

M ( x)
 750
Nm

 1125

 1500
0 500 1000 1500 2000
x
mm

FIGURE 3-10B
Shear and Moment Diagrams for Problem 3-10

10. The maximum value of the shear force ocuurs throughout the distance from x = 0 to x = a and is
R1  1821 N
11. Find the maximum value of the bending moment by determining the value of x where the shear is zero.
Inspection of the shear diagram shows that this occurs at x = a.

Mmax  M ( a ) Mmax  1275 N  m

© 2011 Pearson Education, Inc., Upper Saddle River, NJ. All rights reserved. This publication is protected by Copyright and written permission should be
MACHINE DESIGN - An Integrated Approach, 4th Ed. 3-11-1
PROBLEM 3-11
Statement: Determine the impact force and dynamic deflection that will result when the 100-kg person in
Problem 3-10 jumps up 250 mm and lands back on the board. Assume that the board weighs
29 kg and deflects 131 mm statically when the person stands on it. Find the reaction forces
and construct the shear and moment diagrams for this dynamic loading. Determine the
maximum shear force, maximum moment and their locations along the length of the board.

Given: Beam length L  2000 mm 2000 = L


Distance to support a  700  mm R1 Fi
Mass of person mpers  100  kg
Mass of board mboard  29 kg
R2
Static deflection δst  131  mm
Height of jump h  250  mm 700 = a

Assumptions: Equation (3.15) can be used to approximate a FIGURE 3-11A


mass correction factor. Free Body Diagram for Problem 3-11

Solution: See Figure 3-11 and Mathcad file P0311.

1. The person is the moving object and the board is the struck object. The mass ratio to be used in equation
(3.15) for the correction factor is
mpers
massratio  massratio  3.448
mboard

2. From equation (3.15), the correction factor is


1
η  η  0.912
1
1
3  massratio

3. The weight of the moving mass is Wpers  mpers g Wpers  0.981  kN

4. The dynamic force is found by solving equation (3.14) for Fi.

 2  η h 
Fi  Wpers  1  1  Fi  3.056  kN
 δst 

Fi
From this we see that the dynamic force ratio is  3.12
Wpers

5. From inspection of Figure 3-11, write the load function equation

q(x) = -R1<x - 0>-1 + R2<x - a>-1 - Fi<x - L >-1

6. Integrate this equation from - to x to obtain shear, V(x)


V(x) = -R1<x - 0>0 + R2<x - a>0 - Fi<x - L >0

7. Integrate this equation from - to x to obtain moment, M(x)


M(x) = -R1<x - 0>1 + R2<x - a>1 - Fi<x - L >1
© 2011 Pearson Education, Inc., Upper Saddle River, NJ. All rights reserved. This publication is protected by Copyright and written permission should be
MACHINE DESIGN - An Integrated Approach, 4th Ed. 3-11-2

8. Solve for the reactions by evaluating the shear and moment equations at a point just to the right of x = L,
where both are zero.
At x = L+, V = M = 0 V = R1  R2  Fi = 0 M = R1 L  R2 ( L  a ) = 0

L a
R1  Fi R1  5676 N
a

R2  Fi  R1 R2  8733 N

9. Define the range for x x  0  in 0.005  L  L

10. For a Mathcad solution, define a step function S. This function will have a value of zero when x is less than
z, and a value of one when it is greater than or equal to z.

S ( x z)  if ( x  z 1 0 )

11. Write the shear and moment equations in Mathcad form, using the function S as a multiplying factor to get
the effect of the singularity functions.

V ( x)  R1 S ( x 0  in)  R2 S ( x a )  Fi S ( x L)

M ( x)  R1 S ( x 0  in)  x  R2 S ( x a )  ( x  a )  Fi S ( x L)  ( x  L)

12. Plot the shear and moment diagrams.


Shear Diagram Moment Diagram

4 0

2
1

0
V ( x) M ( x)
2
kN kN  m
2

3
4

6 4
0 0.5 1 1.5 2 0 0.5 1 1.5 2
x x
m m

FIGURE 3-11B
Shear and Moment Diagrams for Problem 3-11

13. The maximum value of the shear force ocuurs throughout the distance from x = 0 to x = a and is
R1  5676 N
14. Find the maximum value of the bending moment by determining the value of x where the shear is zero.
Inspection of the shear diagram shows that this occurs at x = a.

Mmax  M ( a ) Mmax  3973 N  m

© 2011 Pearson Education, Inc., Upper Saddle River, NJ. All rights reserved. This publication is protected by Copyright and written permission should be
MACHINE DESIGN - An Integrated Approach, 4th Ed. 3-12-1
PROBLEM 3-12
Statement: An overhung diving board is shown in Figure P3-4b. Find the reaction forces and construct the
shear and moment diagrams for this board with a 100 kg person standing at the free end.
Determine the maximum shear force, maximum moment and their locations.

Given: Beam length L  1300 mm 2000

Mass at free end M  100  kg 1300 = L

P
Assumptions: 1. The weight of the beam is negligible
compared to the applied load and so can
be ignored.
M1
R1
Solution: See Figure 3-12 and Mathcad file P0312.
700
1. From inspection of Figure 3-12, write the load function
equation FIGURE 3-12A
Free Body Diagram for Problem 3-12
q(x) = -M1<x - 0>-2 + R1<x - a>-1 - P<x - L >-1

2. Integrate this equation from - to x to obtain shear, V(x)


V(x) = -M1<x - 0>-1 + R1<x - a>0 - P<x - L >0

3. Integrate this equation from - to x to obtain moment, M(x)


M(x) = -M1<x - 0>0 + R1<x - a>1 - P<x - L >1

4. Determine the magnitude of the force, P P  M  g P  980.7  N

5. Solve for the reactions by evaluating the shear and moment equations at a point just to the right of x = L,
where both are zero.

At x = L+, V = M = 0 V = R1  P = 0 M = M1  R1 L = 0

R1  P R1  981  N

M1  R1 L M1  1275 m N

6. Define the range for x x  0  in 0.005  L  L

7. For a Mathcad solution, define a step function S. This function will have a value of zero when x is less than z,
and a value of one when it is greater than or equal to z.

S ( x z)  if ( x  z 1 0 )

8. Write the shear and moment equations in Mathcad form, using the function S as a multiplying factor to get the
effect of the singularity functions.

V ( x)  R1 S ( x 0  mm)  P S ( x L)

M ( x)  M1 S ( x 0  mm)  R1 S ( x 0  mm)  ( x  0  mm)  P S ( x L)  ( x  L)

9. Plot the shear and moment diagrams.


© 2011 Pearson Education, Inc., Upper Saddle River, NJ. All rights reserved. This publication is protected by Copyright and written permission should be
MACHINE DESIGN - An Integrated Approach, 4th Ed. 3-12-2

Shear 1000
Diagram
800

600
V ( x)
N
400

200

0
0 0.5 1 1.5 2
x
m

Moment 0
Diagram
 300

 600
M ( x)
Nm
 900

 1200

 1500
0 0.5 1 1.5 2
x
m

FIGURE 3-12B
Shear and Moment Diagrams for Problem 3-12

10. The maximum value of the shear force ocuurs throughout the distance from x = 0 to x = L and is
R1  981  N
11. Find the maximum value of the bending moment by determining the value of x where the shear is zero.
Inspection of the shear diagram shows that this occurs at x = 0.

Mmax  M ( 0  mm) Mmax  1275 N  m

© 2011 Pearson Education, Inc., Upper Saddle River, NJ. All rights reserved. This publication is protected by Copyright and written permission should be
MACHINE DESIGN - An Integrated Approach, 4th Ed. 3-13-1
PROBLEM 3-13
Statement: Determine the impact force and dynamic deflection that will result when a 100-kg person jumps
up 25 cm and lands back on the board. Assume the board weighs 19 kg and deflects 8.5 cm
statically when the person stands on it. Find the reaction forces and construct the shear and
moment diagrams for this dynamic loading. Determine the maximum shear force, maximum
moment, and their locations along the length of the board.

Given: Total board length b  2000 mm


Supported length a  700  mm 2000

Mass of board mboard  19 kg 1300 = L

Static board deflection δstat  85 mm Fi


Mass of person mperson  100  kg
Height of jump h  250  mm M1
R1

Assumptions: 1. The board can be modelled as a cantilever


beam with maximum shear and moment at the 700
edge of the support.
FIGURE 3-13A
Solution: See Figure 3-13 and Mathcad file P0313.
Free Body Diagram for Problem 3-13

1. The person impacts the board upon landing. Thus, the board is the struck object and the person is the
striking object. To determine the force exerted by the person we will first need to know the impact correction
factor from equation (3.15).

1
η  η  0.94 (1)
mboard
1
3  mperson

2. We can now use equation (3.14) to determine the impact force, Fi,

 2  η h 
Fi  mperson g   1  1  Fi  3.487  kN (2)
 δstat 

3. Write an equation for the load function in terms of equations 3.17 and integrate the resulting function twice
using equations 3.18 to obtain the shear and moment functions. Note use of the unit doublet function to
represent the moment at the wall. For the beam in Figure 3-13,

q(x) = -M1<x - 0>-2 + R1<x - 0>-1 - Fi<x - l>-1 (3)

V(x) = -M1<x - 0>-1 + R1<x - 0>0 - Fi<x - l>0 + C1 (4)

M(x) = -M1<x - 0>0 + R1<x - 0>1 - Fi<x - l>1 + C1x+ C2 (5)

The reaction moment M1 at the wall is in the z direction and the forces R1 and Fi are in the y direction in
equation (4). All moments in equation (5) are in the z direction.

4. Because the reactions have been included in the loading function, the shear and moment diagrams both
close to zero at each end of the beam, making C1 = C2 = 0.
5. The reaction force R1 and the reaction moment M1 can be calculated from equations (4) and (5) respectively
by substituting the boundary conditions x = l+, V = 0, M = 0. Note that we can substitute l for l + since their
difference is vanishingly small.
© 2011 Pearson Education, Inc., Upper Saddle River, NJ. All rights reserved. This publication is protected by Copyright and written permission should be
MACHINE DESIGN - An Integrated Approach, 4th Ed. 3-13-2

Unsupported beam length l  b  a l  1300 mm

V(l) = -M1<l - 0>-1 + R1<l - 0>0 - Fi<l - l>0 = 0

V = R1  Fi = 0 (6)

R1  Fi R1  3.487  kN

M(l) = -M1<l - 0>0 + R1<l - 0>1 - Fi<l - l>1 = 0

M = M1  R1 l  Fi ( l  l) = 0 (7)

M1  R1 l M1  4533 N  m

6. To generate the shear and moment functions over the length of the beam, equations (4) and (5) must be
evaluated for a range of values of x from 0 to l, after substituting the above values of C1, C2, R1, and M1 in them.
For a Mathcad solution, define a step function S. This function will have a value of zero when x is less than the
dummy variable z, and a value of one when it is greater than or equal to z. It will have the same effect as the
singularity function.
Range of x x  0  in 0.005  l  l
Unit step function S ( x z)  if ( x  z 1 0 )

Write the shear and moment equations in Mathcad form, using the function S as a multiplying factor to get
the effect of the singularity functions.

0 0
V ( x)  R1 S ( x 0  in)  ( x  0 )  Fi S ( x l)  ( x  l)
(8)
0 1 1
M ( x)  M1 S ( x 0  in)  ( x  0 )  R1 S ( x 0  in)  ( x  0 )  Fi S ( x l )  ( x  l)

Plot the shear and moment diagrams (see below).


Shear Diagram Moment Diagram

1
3

2
V ( x) M ( x)
2
kN kN  m
3

1
4

0 5
0 0.5 1 0 0.5 1 1.5
x x
m m
7. The graphs show that the shear force and the moment are both largest at x = 0. The function values of
these points can be calculated from equations (4) and (5) respectively by substituting x = 0 and evaluating the
singularity functions:

Vmax = V(0) = R1<0 - 0>0 - Fi<0 - l>0 = R1 (9)

© 2011 Pearson Education, Inc., Upper Saddle River, NJ. All rights reserved. This publication is protected by Copyright and written permission should be
MACHINE DESIGN - An Integrated Approach, 4th Ed. 3-13-3

Vmax  R1 Vmax  3.49 kN

M.max = M(0) = -M1<0 - 0>0 + R1<0 - 0>1 - Fi<0 - l>1 = -M1 (10)

Mmax  M1 Mmax  4533 N  m

© 2011 Pearson Education, Inc., Upper Saddle River, NJ. All rights reserved. This publication is protected by Copyright and written permission should be
MACHINE DESIGN - An Integrated Approach, 4th Ed. 3-14-1
PROBLEM 3-14
Statement: Figure P3-5 shows a child's toy called a pogo stick. The child stands on the pads, applying half
her weight on each side. She jumps off the ground, holding the pads up against her feet, and
bounces along with the spring cushioning the impact and storing energy to help each rebound.
Find the natural frequency of the system, the static deflection of the spring with the child
standing still, and the dynamic force and deflection when the child lands after jumping 2 in off
the ground.

2
lbf ⋅ sec
Units: blob :=
in
Given: Child's weight Wc := 60⋅ lbf
−1
Spring constant k := 100⋅ lbf ⋅ in
Pogo stick weight Wp := 5 ⋅ lbf
Height of drop h := 2 ⋅ in

Assumptions: 1. An approximate energy method will be acceptable.


2. The correction factor for energy dissipation will be applied.
Solution: See Figure 3-14 and Mathcad file P0314.

1. Find the natural frequency of the (child/spring) system. Fi /2 Fi /2

Wc
Mass of child (striker) m := m = 0.155⋅ blob
g
Wp
Mass of stick (struck) mb := mb = 0.013⋅ blob
g

k rad
Natural frequency ω := ω = 25.367⋅
m sec

ω P
f := f = 4.037⋅ Hz
2⋅ π
FIGURE 3-14
Free Body Diagram for Problem 3-14
2. The static deflection of the spring with the child standing still is

Wc
Static deflection of spring δst := δst = 0.6⋅ in
k

3. Determine the mass ratio correction factor from equation (3.15):

1
Correction factor η := η = 0.973
mb
1+
3⋅ m

4. Using equation (3.14), determine the dynamic force.

2 ⋅ η⋅ h
Fi := Wc⋅ 1 + 1+ Fi = 224⋅ lbf
δst

© 2011 Pearson Education, Inc., Upper Saddle River, NJ. All rights reserved. This publication is protected by Copyright and written permission should be
MACHINE DESIGN - An Integrated Approach, 4th Ed. 3-15-1

PROBLEM 3-15
Statement: A pen plotter imparts a constant acceleration of 2.5 m/sec2 to the pen assembly, which travels in
a straight line across the paper. The moving pen assembly weighs 0.5 kg. The plotter weighs 5
kg. What coefficient of friction is needed between the plotter feet and the table top on which it
sits to prevent the plotter from moving when the pen accelerates?

2
Given: Acceleration of pen ass'y a  2.5 m sec
Mass of pen ass'y mpen  0.5 kg
Mass of plotter mplot  5  kg
Solution: See Mathcad file P0315.

1. The force imparted to the pen assembly by the internal drive mechanism must be reacted at the table top by
the plotter feet. The horizontal force at the feet will be equal to the force on the pen assembly and must be
less than or equal to the maximum friction force, which is the product of the coefficient of friction and the
normal force, which is the weight of the plotter.

Horizontal driving
force on pen ass'y Fpen  mpen a Fpen  1.25 N

Weight of plotter Wplot  mplot  g Wplot  49.033 N

Minimum coefficient Fpen


of friction μ  μ  0.025
Wplot

© 2011 Pearson Education, Inc., Upper Saddle River, NJ. All rights reserved. This publication is protected by Copyright and written permission should be
MACHINE DESIGN - An Integrated Approach, 4th Ed. 3-16-1

PROBLEM 3-16
Statement: A track to guide bowling balls is designed with two round rods as shown in Figure P3-6. The
rods are not parallel to one another but have a small angle between them. The balls roll on the
rods until they fall between them and drop onto another track. The angle between the rods is
varied to cause the ball to drop at different locations. Each rod's unsupported length is 30 in
and the angle between them is 3.2 deg. The balls are 4.5 in dia and weigh 2.5 lb. The center
distance between the 1-in-dia rods is 4.2 in at the narrow end. Find the distance from the
narrow end at which the ball drops through and determine the worst-case shear and moment
maximum for the rods as the ball rolls a distance from the narrow end that is 98% of the distance
to drop. Assume that the rods are simply supported at each end and have zero deflection under
the applied loading. (Note that assuming zero deflection is unrealistic. This assumption will be
relaxed in the next chapter after deflection has been discussed.)

Given: Unsupported rod length L  30 in Bowling ball weight W  2.5 lbf
Half-angle between rods α  1.6 deg Rod diameter d  1.0 in
Bowling ball diameter D  4.5 in Half width of rod gap c  2.1 in

Solution: See Figure 3-16 and Mathcad file P0316.

1. Calculate the distance between the ball and rod


centers.
A A
D d
Distance between centers h  h  2.75 in
2

c
TOP VIEW


u F

x

W/2
F

SECTION A-A
width(x)
(a) Distance between the roll axis and (b) Partial FBD of the bowling ball.
the rod axis.

FIGURE 3-16
Dimensions and Free Body Diagrams for Problem 3-16

© 2011 Pearson Education, Inc., Upper Saddle River, NJ. All rights reserved. This publication is protected by Copyright and written permission should be
MACHINE DESIGN - An Integrated Approach, 4th Ed. 3-16-2

2. Let x be the distance along the roll axis, and u be the corresponding distance to the point of contact between
the ball and rods, measured along the rods. Then the distance from the center plane of the ball to the center
of a rod as shown in Figure 3-16(a) is,

width( x)  c cos( α)  x sin( α) (1)

And the distance from the narrow end to the point at which the ball drops (assuming rigid rods) is
h  c cos( α)
xdrop  xdrop  23.31  in
sin( α)

The distance along the rod corresponding to xdrop is

xdrop  h  sin( α)
u drop  u drop  23.24  in
cos( α)

3. The angle made by a line through the ball-rod centers and the horizontal plane (see Figure 3-16b) is

θ ( x)  acos
width( x) 

 h 

When x = 0, this is θ0  θ ( 0  in) θ0  40.241 deg

When x = 0.98xdrop, this is θ98%  θ  0.98 xdrop θ98%  5.577  deg

4. The loading on the ball is symmetric about its center plane along the x-axis. Figure 3-16(b) shows a FBD of
one half of the ball with the internal forces along the plane of symmetry due to the reaction at the other rod
omitted. With these forces omitted we may only sum forces in the vertical direction.

W
 Fy : F  sin( θ )  μ  F  cos( θ )  =0 (2)
2

W
F= (3)
2  ( sin( θ )  μ  cos( θ ) )

5. The ball will drop through the rods when  is zero. If there were no friction force present ( = 0) then F would
become very large as  approached zero. The presence of the friction term in the denominator of equation (3)
limits F to finite values. However, with the assumption that the rods are rigid, there is no way for the rods to
provide a normal force when  reaches zero. Thus, we will need to limit the range of  for this analysis.

Let μ  0 and θmin  θ98%

h  cos θmin  c cos( α)


Then xmax  xmax  22.84  in
sin( α)

xmax  h  sin( α)
u max  u max  22.77  in
cos( α)

W
Fmax  Fmax  12.86  lbf (4)
2  sin θmin

© 2011 Pearson Education, Inc., Upper Saddle River, NJ. All rights reserved. This publication is protected by Copyright and written permission should be
MACHINE DESIGN - An Integrated Approach, 4th Ed. 3-16-3

6. Determine the worst-case shear and moment maximum for the rods as the ball rolls along their length from
Figure B-2(a) in Appendix B where a in the figure is u max. Then,

 u max 
Mmax  Fmax u max  1   Mmax  70.6 in lbf (5)
 L 

For the shear, we must find the reactions, which are

 u max 
R1  Fmax  1   R1  3.10 lbf
 L 
R2  Fmax  R1 R2  9.76 lbf

The maximum absolute value of shear is the larger of these two. Thus

Vmax  R2 Vmax  9.8 lbf (6)

© 2011 Pearson Education, Inc., Upper Saddle River, NJ. All rights reserved. This publication is protected by Copyright and written permission should be
MACHINE DESIGN - An Integrated Approach, 4th Ed. 3-17-1
PROBLEM 3-17
Statement: A pair of ice tongs is shown in Figure P3-7. The ice weighs 50 lb and is 10 in wide across the
tongs. The distance between the handles is 4 in, and the mean radius r of a tong is 6 in. Draw
free-body diagrams of the two tongs and find all forces acting on them. Determine the bending
moment at point A.

Given: Weight of ice W  50 lbf


Distances a x  11.0 in a y  6.0 in
b x  5.0 in b y  12.0 in
cx  2.0 in cy  3.5 in

Assumptions: Assume that the horizontal force at C (the handle) is zero, thus Fc  0  lbf (1)
Solution: See Figure 3-17 and Mathcad file P0317.

F F F

C
FC
3.5 = cy
O
FO
11.0 = ax 2.0 = cx

A
12.0 = by

5.0 = bx
FB
B

W/2

W
FIGURE 3-17A
Free Body Diagrams for Problem 3-17

1. Summing forces and moments on a single tong (see FBD above right).

 Fx FO  FB  FC = 0 (2)

 Fy W
F=0 (3)
2

 MC FO cy  FB  b y  cy    b x  cx = 0


W
(4)
2

2. From equations (1) and (2), FO = FB (5)

3. Eliminating FO from equations (4) and (5) and solving for FB

W   b x  cx
FB  FB  14.58  lbf
2 by

© 2011 Pearson Education, Inc., Upper Saddle River, NJ. All rights reserved. This publication is protected by Copyright and written permission should be
MACHINE DESIGN - An Integrated Approach, 4th Ed. 3-17-2

F
4. From equation (3), the vertical force
on one handle is
C
W FC
F  F  25 lbf
2 3.5 = cy
O
FO
5. From Figure 3-17B we see that, at any
section that we might take through the 11.0 = ax 2.0 = cx
tong, there will be an internal moment,
shear force, and axial force present. The
A
bending moment will be a maximum at 
point A because it is the fartherest point
from the centroid of the system.
Summing forces and moments:
D
FDs M D FDn
 Fx -FDs cos  + FDn sin 
+ FO = 0 (6)
FIGURE 3-17B
Free Body Diagram with section at D for Problem 3-17
 Fy -FDs sin  - FDn cos 
+F=0 (7)

 MO F cx - M D - (FDs cos + FDn sin )(ay + rc sin )


+ (FDs sin + FDn cos )[ax - rc (1 - cos)] = 0
(8)
6. Solving equations (6) and (7) for FDs and FDn

FDn sin( θ )  FO
FDn = F  cos( θ )  FO sin( θ ) FDs =
cos( θ )

7. The maximum value of MD will occur at  = 0 deg. At  = 0 deg,

FO  FB

FDn  F FDn  25 lbf

FDs  FO FDs  14.58  lbf

MD  F  cx  FDs a y  FDn a x MD  237.5  lbf  in

© 2011 Pearson Education, Inc., Upper Saddle River, NJ. All rights reserved. This publication is protected by Copyright and written permission should be
MACHINE DESIGN - An Integrated Approach, 4th Ed. 3-18-1
PROBLEM 3-18
Statement: A tractor-trailer tipped over while negotiating an on-ramp to the New York Thruway. The road
has a 50-ft radius at that point and tilts 3 deg toward the outside of the curve. The 45-ft-long by
8-ft-wide by 8.5-ft-high trailer box (13 ft from ground to top) was loaded 44 415 lb of paper rolls in
two rows by two high as shown in Figure P3-8. The rolls are 40-in-dia by 38-in-long and weigh
about 900 lb each. They are wedged against rolling backward but not against sliding sidewards.
The empty trailer weighed 14 000 lb. The driver claims that he was traveling at less than 15 mph
and that the load of paper shifted inside the trailer, struck the trailer sidewall, and tipped the
truck. The paper company that loaded the truck claims the load was properly stowed and would
not shift at that speed. Independent test of the coefficient of friction between similar paper rolls
and a similar trailer floor give a value of 0.43 +/- 0.08. The composite center of gravity of the
loaded trailer is estimated to be 7.5 ft above the road. Determine the truck speed that would
cause the truck to just begin to tip and the speed at which the rolls will just begin to slide
sidways. What do you think caused the accident?
Given: Weight of paper Wp := 44415⋅ lbf
Weight of trailer Wt := 14000⋅ lbf
Radius of curve r := 50⋅ ft
Nominal coefficient of friction μnom := 0.43
Coefficient of friction uncertainty u μ := 0.08
Trailer width w := 8 ⋅ ft
Height of CG from pavement h := 7.5⋅ ft
Assumptions: 1. The paper rolls act as a monolith since they are tightly strapped together with steel bands.
2. The tractor has about 15 deg of potential roll freedom versus the trailer due to their relative angle
in plan during the turn combined with the substantial pitch freedom in the fifth wheel. So the trailer
can tip independently of the tractor.
3. The outside track width of the trailer tires is equal to the width of the trailer.

Solution: See Figure 3-18 and Mathcad file P0318.

1. First, calculate the location of the


trailer's CG with respect to the outside
wheel when it is on the reverse-banked
curve. From Figure 3-18A,

Tilt angle θ := 3 ⋅ deg

a := h ⋅ tan( θ ) a = 0.393⋅ ft

w
b := −a b = 3.607⋅ ft
2 7.500'
ybar
xbar := b ⋅ cos( θ ) xbar = 3.602⋅ ft

h
ybar := b⋅ sin( θ ) +
cos( θ )

ybar = 7.699⋅ ft a b
The coordinates of the CG of the xbar
loaded trailer with respect to the lower 4.000'
outside corner of the tires are:
FIGURE 3-18A
xbar = 3.602⋅ ft ybar = 7.699⋅ ft Location of CG for Problem 3-18

© 2011 Pearson Education, Inc., Upper Saddle River, NJ. All rights reserved. This publication is protected by Copyright and written permission should be
MACHINE DESIGN - An Integrated Approach, 4th Ed. 3-18-2
2. The trailer is on the verge of tipping over when the copule due to centrifugal force is equal to the couple formed
by the weight of the loaded trailer acting through its CG and the vertical reaction at the outside edge of the tires.
At this instant, it is assumed that the entire weight of the trailer is reacted at the outside tires with the inside tires
carrying none of the weight. Any increase in tangential velocity of the tractor and trailer will result in tipping.
Summing moments about the tire edge (see Figure 3-18B),

ΣM Fw⋅ xbar − Fc⋅ ybar = 0 (1)

where Fc is the centrifugal force due


to the normal acceleration as the tractor
and trailer go through the curve. The
normal acceleration is Fc
2
vtip
a tip = (2)
r Fw
and the force necessary to keep the
tractor trailer following a circular path ybar
is
Fc = mtot⋅ a tip (3)

Rx
where mtot is the total mass of the
trailer and its payload. Combining
equations (2) and (3) and solving for
Ry
vtip, we have
xbar
Fc⋅ r
vtip = (4)
mtot FIGURE 3-18B
FBD of Trailer on the Verge of Tipping
or,
Fc⋅ r⋅ g
vtip = (5)
Fw

3. Calculate the minimum tipping velocity of the tractor/trailer. From equations (1) and (5),

Total weight Fw := Wt + Wp Fw = 58415⋅ lbf

xbar
Centrifugal force required Fc := ⋅ Fw Fc = 27329⋅ lbf
to tip the trailer ybar

Fc⋅ r⋅ g
Minimum tipping speed vtip := vtip = 18.7⋅ mph
Fw

Thus, with the assumptions that we have made, the trailer would not begin to tip over until it reached a speed of

vtip = 18.7⋅ mph

4. The load will slip when the friction force between the paper rolls and the trailer floor is no longer sufficient to
react the centrifugal force on the paper rolls. Looking at the FBD of the paper rolls in Figure 3-18C, we see that

Normal force between


Fn = Wp⋅ cos( θ ) − Fcp ⋅ sin( θ ) (6)
paper and floor

© 2011 Pearson Education, Inc., Upper Saddle River, NJ. All rights reserved. This publication is protected by Copyright and written permission should be
MACHINE DESIGN - An Integrated Approach, 4th Ed. 3-18-3
Tangential force tending to slide the paper

Ft = Wp⋅ sin( θ ) + Fcp ⋅ cos( θ ) (7) Fcp

Centrifugal force on the paper


Wp
2
Wp W p vs
Fcp = ⋅ as = ⋅ (8) Ft
g g r
Fn
But, the maximum friction force is
FIGURE 3-18C
Ff = μ ⋅ Fn = Ft (9)
FBD of Paper on the Verge of Sliding

Substituting equation (9) into (7), then combining (6) and (7) to eliminate Fn, and solving for Fcp yields

Wp⋅ ( μ ⋅ cos( θ ) − sin( θ ) )


Fcp = (10)
μ ⋅ sin( θ ) + cos( θ )

Substituting equation (10) into (8), to eliminate Fcp , and solving for vs yields

( μ ⋅ cos( θ ) − sin( θ ) )
vs = ⋅ r⋅ g (11)
μ ⋅ sin( θ ) + cos( θ )

5. Use the upper and lower limit on the coefficient of friction to determine an upper and lower limit on the speed
necessary to cause sliding.

Maximium coefficient μmax := μnom + u μ μmax = 0.51

Minimium coefficient μmin := μnom − uμ μmin = 0.35

Maximum velocity to cause sliding

(μmax ⋅ cos( θ ) − sin( θ ) )


vsmax := ⋅ r⋅ g vsmax = 18.3⋅ mph
μmax ⋅ sin( θ ) + cos( θ )

Minimum velocity to cause sliding

(μmin ⋅ cos( θ ) − sin( θ ) )


vsmin := ⋅ r⋅ g vsmin = 14.8⋅ mph
μmin ⋅ sin( θ ) + cos( θ )

6. This very rough analysis shows that , if the coefficient of friction was at or near the low end of its measured
value, the paper load could slide at a tractor/trailer speed of 15 mph, which would lead to the trailer tipping over.
In any case, it appears that the paper load would slide before the truck would tip with the load in place.

© 2011 Pearson Education, Inc., Upper Saddle River, NJ. All rights reserved. This publication is protected by Copyright and written permission should be
MACHINE DESIGN - An Integrated Approach, 4th Ed. 3-19-1
PROBLEM 3-19
Statement: Assume that the CG of the paper rolls in Problem 3-18 is 2.5 ft above the floor of the trailer. At what
speed on the same curve will the pile of rolls tip over (not slide) with respect to the trailer?
Given: Weight of paper Wp := 44415⋅ lbf
Radius of curve r := 50⋅ ft
Paper roll length L := 38⋅ in L = 3.167⋅ ft
Height of CG from floor h := 2.5⋅ ft
Assumptions: The paper rolls act as a single, lumped mass and tip about one corner where they are braced
against sliding. The brace provides no moment support.
Solution: See Figure 3-19 and Mathcad file P0319.

1. First, calculate the location of the paper's CG with


respect to the outside corner when it is on the
reverse-banked curve. From Figure 3-19,
Fcp
Tilt angle θ := 3⋅ deg

a := h⋅ tan ( θ) a = 0.131⋅ ft 2.500' ybar


Wp
b := L − a b = 3.036⋅ ft a Rx
b
xbar R y
xbar := b ⋅ cos ( θ) xbar = 3.031⋅ ft
3.167'
h
ybar := b⋅ sin ( θ) +
cos ( θ) FIGURE 3-19
FBD of Paper on the Verge of Tipping
ybar = 2.662⋅ ft

The coordinates of the CG of the paper with respect to the lower outside corner are:
xbar = 3.031⋅ ft ybar = 2.662⋅ ft

2. The paper is on the verge of tipping over when the couple due to centrifugal force is equal to the couple formed
by the weight of the paper acting through its CG and the vertical reaction at the outside edge of the rolls. At this
instant, it is assumed that the entire weight of the paper is reacted at the outside corner. Any increase in
tangential velocity of the tractor and trailer will result in tipping. Summing moments about the outside corner
nearest the floor (see Figure 3-19),

ΣM Wp⋅ xbar − Fcp ⋅ ybar = 0 (1)

where Fcp is the centrifugal force due to the normal acceleration as the tractor and trailer go through the curve.
The normal acceleration is
2
vtip
a tip = (2)
r

and the force necessary to keep the tractor trailer following a circular path is
Fcp = mp⋅ a tip (3)

where mp is the mass of the paper. Combining equations (2) and (3) and solving for vtip, we have

© 2011 Pearson Education, Inc., Upper Saddle River, NJ. All rights reserved. This publication is protected by Copyright and written permission should be
MACHINE DESIGN - An Integrated Approach, 4th Ed. 3-19-2
Fcp ⋅ r
vtip = (4)
mp

or,
Fcp ⋅ r⋅ g
vtip = (5)
Wp

3. Calculate the minimum paper tipping velocity of the tractor/trailer. From equations (1) and (5),

xbar
Centrifugal force required Fcp := ⋅ Wp Fcp = 50574⋅ lbf
to tip the paper ybar

Fcp ⋅ r⋅ g
Minimum tipping speed vtip := vtip = 29.2⋅ mph
Wp

Thus, with the assumptions that we have made, the paper would not begin to tip over until the tractor/trailor
reached a speed of

vtip = 29.2⋅ mph

© 2011 Pearson Education, Inc., Upper Saddle River, NJ. All rights reserved. This publication is protected by Copyright and written permission should be
MACHINE DESIGN - An Integrated Approach, 4th Ed. 3-20-1
PROBLEM 3-20
Statement: Assume that the load of paper rolls in Problem 3-18 will slide sideways at a truck speed of 20 mph
on the curve in question. Estimate the impact force of the cargo against the trailer wall. The
force-deflection characteristic of the trailer wall has been measured as approximately 400 lb/in.

Given: Weight of paper Wp := 44415⋅ lbf


Weight of trailer Wt := 14000⋅ lbf
Speed of tractor/trailer vt := 20⋅ mph
Radius of curve r := 50⋅ ft
Trailer width w := 8 ⋅ ft
Paper roll length L := 38⋅ in L = 3.167⋅ ft
lbf
Trailer wall stiffness k := 400
in
Assumptions: 1. The paper rolls act as a monolith since they are tightly strapped together with steel bands.
2. The worst case will result if friction between the floor and the paper is neglected.
Solution: See Figure P3-8 and Mathcad file P0320.

1. Calculate the distance that the rolls will slide before impacting the wall.
1
s := ⋅ ( w − 2⋅ L) s = 10⋅ in
2
2. Determine the centripetal acceleration at 20 mph.
2
vt in
a p := a p = 206.507⋅
r 2
sec

3. From elementary particle dynamics, estimate the velocity at impact due to the centripetal acceleration

in
vi := 2⋅ a p⋅ s vi = 64.266⋅
sec

4. With the paper as the moving mass and the trailer as the stationary or struck mass, calculate the correction factor
using equation (3.15)

1
η := η = 0.905
Wt
1+
3 ⋅ Wp

5. Calculate the static deflection caused by the paper against the trailer wall.

Wp
δst := δst = 111.037⋅ in
k
6. Using equation (3.12), estimate the dynamic force of the paper rolls impacting the trailer wall.

η
Fi := Wp⋅ vi⋅ Fi = 13114⋅ lbf
g ⋅ δst

© 2011 Pearson Education, Inc., Upper Saddle River, NJ. All rights reserved. This publication is protected by Copyright and written permission should be
MACHINE DESIGN - An Integrated Approach, 4th Ed. 3-21-1
PROBLEM 3-21
Statement: Figure P3-9 shows an automobile wheel with two common styles of lug wrench being used to
tighten the wheel nuts, a single-ended wrench in (a), and a double-ended wrench in (b). In each
case two hands are required to provide forces respectively at A and B as shown. The distance
between points A and B is 1 ft in both cases. The wheel nuts require a torque of 70 ft-lb. Draw
free body diagrams for both wrenches and determine the magnitudes of all forces and moments
on each wrench. Is there any difference between the way these two wrenches perform their
assigned task? Is one design better than the other? If so, why? Explain.
Given: Distance between A and B d AB := 1 ⋅ ft
Tightening torque T := 70⋅ ft ⋅ lbf

Assumptions: 1. The forces exerted by the user's hands lie in a plane through the wrench that is also parallel to
the plane of the wheel.
2. The applied torque is perpendicular to the plane of the forces.
3. By virtue of 1 and 2 above, this is a planar problem that can be described in a 2D FBD.

Solution: See Figure 3-21 and Mathcad file P0321.

1. Summing moments about the left end of the 12" = dAB


wrench (for either case) F

T − F ⋅ d AB = 0

2. Solving for F
T
T
F := F = 70⋅ lbf F
d AB
(a) Single-ended Wrench

3. This result is the same for both wrenches.


12" = dAB
Is there any difference between the way these F
6"
two wrenches perform their assigned task?

No, they both require the same


two-handed exertion of 70 lb from each
hand. T

Is one design better than the other? If so, F


(b) Double-ended Wrench
why? Explain.
FIGURE 3-21
Design (b) has advantages over (a)
Free Body Diagrams for Problem 3-21
because it is balanced about the wheel
nut. This allows the user to spin the
wrench once the nut is loosened. It is
also slightly easier to apply the upward
and downward forces (F) in a plane with
design (b).

© 2011 Pearson Education, Inc., Upper Saddle River, NJ. All rights reserved. This publication is protected by Copyright and written permission should be
MACHINE DESIGN - An Integrated Approach, 4th Ed. 3-22-1
PROBLEM 3-22
Statement: A roller-blade skate is shown in Figure P3-10. The polyurethane wheels are 72 mm dia. The
skate-boot-foot combination weighs 2 kg. The effective "spring rate" of the person-skate
subsystem is 6000 N/m. Find the forces on the wheels' axles for a 100-kg person landing a 0.5-m
jump on one foot. (a) Assume all 4 wheels land simultaneously. (b) Assume that one wheel
absorbs all the landing force.

Given: Mass of struck member Msys  2  kg


N
Stiffness of struck member k  6000
m
Mass of striking member Mperson  100  kg
Height of drop h  0.5 m

Assumptions: Equation (3.14) applies in this case.

Solution: See Figure P3-10 and Mathcad file P0322.

1. The weight of the striking mass is


Wperson  Mperson g Wperson  980.7  N

2. The static deflection of the subsystem is

Wperson
δst  δst  163.444  mm
k

3. The correction factor is

1
η  η  0.993
Msys
1
3  Mperson

4. From equation (3.14), the force of impact is

 2  η h 
Fi   1  1   Wperson Fi  3.59 kN
 δst 

(a) If this will be absorbed by 4 wheel axles, the force per axle is

Fi
Fa  Fa  897  N
4

(b) If one wheel absorbs all force Fb  Fi Fb  3.59 kN

© 2011 Pearson Education, Inc., Upper Saddle River, NJ. All rights reserved. This publication is protected by Copyright and written permission should be
MACHINE DESIGN - An Integrated Approach, 4th Ed. 3-23a-1
PROBLEM 3-23a
Statement: A beam is supported and loaded as shown in Figure P3-11a. Find the reactions, maximum shear,
and maximum moment for the data given in row a from Table P3-1.

Given: Beam length L  1  m L


Distance to distributed load a  0.4 m b
Distance to concentrated load b  0.6 m a
F
1 w
Distributed load magnitude w  200  N  m
Concentrated load F  500  N
R1 R2
Solution: See Figures 3-23 and Mathcad file P0323a.
FIGURE 3-23A
Free Body Diagram for Problem 3-23

1. From inspection of Figure P3-11a, write the load function equation

q(x) = R1<x - 0>-1 - w<x - 0>0 + w<x - a>0 - F<x - b>-1 + R2<x - L>-1

2. Integrate this equation from - to x to obtain shear, V(x)

V(x) = R1<x - 0>0 - w<x - 0>1 + w<x - a>1 - F<x - b>0 + R2<x - L>0

3. Integrate this equation from - to x to obtain moment, M(x)

M(x) = R1<x - 0>1 - w<x - 0>2/2 + w<x - a>2/2 - F<x - b>1 + R2<x - L>1

4. Solve for the reactions by evaluating the shear and moment equations at a point just to the right of x = L,
where both are zero.
At x = L+, V = M = 0

V = R1  w ( L)  w ( L  a )  F  R2 = 0

w 2 w 2
M = R 1 L  L   ( L  a)  F  ( L  b) = 0
2 2

w F w 2
R1  L   ( L  b)   ( L  a) R1  264  N
2 L 2 L

R2  w a  F  R1 R2  316  N

5. Define the range for x x  0  m 0.005  L  L

6. For a Mathcad solution, define a step function S. This function will have a value of zero when x is less than
z, and a value of one when it is greater than or equal to z.

S ( x z)  if ( x  z 1 0 )

7. Write the shear and moment equations in Mathcad form, using the function S as a multiplying factor to get
the effect of the singularity functions.

V ( x)  R1 S ( x 0  m)  w S ( x 0  m)  ( x)  w S ( x a )  ( x  a )  F  S ( x b )  R2 S ( x L)

w 2 w 2
M ( x)  R1 S ( x 0  m)  x   S ( x 0  m)  x   S ( x a )  ( x  a )  F  S ( x b )  ( x  b )
2 2
© 2011 Pearson Education, Inc., Upper Saddle River, NJ. All rights reserved. This publication is protected by Copyright and written permission should be
MACHINE DESIGN - An Integrated Approach, 4th Ed. 3-23a-2

8. Plot the shear and moment diagrams.

Shear 400
Diagram
200

V ( x)
0
N

 200

 400
0 0.2 0.4 0.6 0.8
x
m

Moment 150
Diagram

100
M ( x)
Nm

50

0
0 0.2 0.4 0.6 0.8
x
m

FIGURE 3-23aB
Shear and Moment Diagrams for Problem 3-23a

9. Determine the maximum shear and maximum moment from inspection of the diagrams.

Maximum shear: Vmax  V ( b ) Vmax  316  N

Maximum moment occurs where V is zero, which is x = b:

Mmax  M ( b ) Mmax  126.4  N  m

© 2011 Pearson Education, Inc., Upper Saddle River, NJ. All rights reserved. This publication is protected by Copyright and written permission should be
MACHINE DESIGN - An Integrated Approach, 4th Ed. 3-24a-1
PROBLEM 3-24a
Statement: A beam is supported and loaded as shown in Figure P3-11b. Find the reactions, maximum shear,
and maximum moment for the data given in row a from Table P3-1.
Given: Beam length L  1  m L
Distance to distributed load a  0.4 m a
1
Distributed load magnitude w  200  N  m F
w
Concentrated load F  500  N

Solution: See Figures 3-24 and Mathcad file P0324a. M1


R1
1. From inspection of Figure P3-11b, write the load function
equation
FIGURE 3-24A
Free Body Diagram for Problem 3-24

q(x) = -M1<x - 0>-2 + R1<x - 0>-1 - w<x - a>0 - F<x - L>-1

2. Integrate this equation from - to x to obtain shear, V(x)

V(x) = -M1<x - 0>-1 + R1<x - 0>0 - w<x - a>1 - F<x - L>0

3. Integrate this equation from - to x to obtain moment, M(x)

M(x) = -M1<x - 0>0 + R1<x - 0>1 - w<x - a>2/2 - F<x - L>1

4. Solve for the reactions by evaluating the shear and moment equations at a point just to the right of x = L,
where both are zero.
At x = L+, V = M = 0

V = R1  [ w  ( L  a )  F ] = 0

w 2
M = M1  R1 L   ( L  a) = 0
2

R1  w ( L  a )  F R1  620  N

w 2
M1   ( L  a )  R1 L M1  584  N  m
2

5. Define the range for x x  0  m 0.005  L  L

6. For a Mathcad solution, define a step function S. This function will have a value of zero when x is less than
z, and a value of one when it is greater than or equal to z.

S ( x z)  if ( x  z 1 0 )

7. Write the shear and moment equations in Mathcad form, using the function S as a multiplying factor to get
the effect of the singularity functions.

V ( x)  R1 S ( x 0  m)  w S ( x a )  ( x  a )  F  S ( x L)

w 2
M ( x)  M1  R1 S ( x 0  m)  x   S ( x a )  ( x  a )  F  S ( x L)  ( x  L)
2

© 2011 Pearson Education, Inc., Upper Saddle River, NJ. All rights reserved. This publication is protected by Copyright and written permission should be
MACHINE DESIGN - An Integrated Approach, 4th Ed. 3-24a-2

8. Plot the shear and moment diagrams.

Shear
Diagram
600

V ( x)
400
N

200

0
0 0.2 0.4 0.6 0.8
x
m

Moment 0
Diagram

 150

M ( x)
 300
Nm

 450

 600
0 0.2 0.4 0.6 0.8
x
m

FIGURE 3-24aB
Shear and Moment Diagrams for Problem 3-24a

9. Determine the maximum shear and maximum moment from inspection of the diagrams.

Maximum shear: Vmax  V ( 0  m) Vmax  620  N

Maximum moment occurs where V is zero, which is x = 0:

Mmax  M ( 0  m) Mmax  584  N  m

© 2011 Pearson Education, Inc., Upper Saddle River, NJ. All rights reserved. This publication is protected by Copyright and written permission should be
MACHINE DESIGN - An Integrated Approach, 4th Ed. 3-25a-1
PROBLEM 3-25a
Statement: A beam is supported and loaded as shown in Figure P3-11d. Find the reactions, maximum shear,
and maximum moment for the data given in row a from Table P3-1.

Given: Beam length L  1  m L


Distance to distributed load a  0.4 m b
Distance to concentrated load b  0.6 m a F
1 w
Distributed load magnitude w  200  N  m
Concentrated load F  500  N
R1 R2
Solution: See Figures 3-25 and Mathcad file P0325a.
FIGURE 3-25A
Free Body Diagram for Problem 3-25

1. From inspection of Figure P3-11c, write the load function equation

q(x) = R1<x - 0>-1 - w<x - a>0 + R2<x - b>-1 - F<x - L>-1

2. Integrate this equation from - to x to obtain shear, V(x)

V(x) = R1<x - 0>0 - w<x - a>1 + R2<x - b>0 - F<x - L>0

3. Integrate this equation from - to x to obtain moment, M(x)

M(x) = R1<x - 0>1 - w<x - a>2/2 + R2<x - b>1 - F<x - L>1

4. Solve for the reactions by evaluating the shear and moment equations at a point just to the right of x = L,
where both are zero.
At x = L+, V = M = 0

V = R1  w  ( L  a )  R2  F = 0

w 2
M = R 1 L   ( L  a )  R2 ( L  b ) = 0
2

   ( L  a )  F  ( L  b )  w ( L  a )  ( L  b )
1 w 2
R1  R1  353  N
b 2 
R2  w ( L  a )  F  R1 R2  973  N

5. Define the range for x x  0  m 0.005  L  L

6. For a Mathcad solution, define a step function S. This function will have a value of zero when x is less than
z, and a value of one when it is greater than or equal to z.

S ( x z)  if ( x  z 1 0 )

7. Write the shear and moment equations in Mathcad form, using the function S as a multiplying factor to get
the effect of the singularity functions.

V ( x)  R1 S ( x 0  m)  w S ( x a )  ( x  a )  R2 S ( x b )  F  S ( x L)

w 2
M ( x)  R1 S ( x 0  m)  x   S ( x a )  ( x  a )  R2 S ( x b )  ( x  b )
2
© 2011 Pearson Education, Inc., Upper Saddle River, NJ. All rights reserved. This publication is protected by Copyright and written permission should be
MACHINE DESIGN - An Integrated Approach, 4th Ed. 3-25a-2

8. Plot the shear and moment diagrams.

Shear 1
Diagram
0.5

V ( x)
0
kN

 0.5

1
0 0.2 0.4 0.6 0.8
x
m

Moment
Diagram 0

 75

M ( x)
 150
Nm

 225

 300
0 0.2 0.4 0.6 0.8
x
m
FIGURE 3-25aB
Shear and Moment Diagrams for Problem 3-25a

9. Determine the maximum shear and maximum moment from inspection of the diagrams.

Maximum shear: Vmax  V ( b ) Vmax  580.0  N

Maximum moment occurs where V is zero, which is x = a:

Mmax  M ( b ) Mmax  216  N  m

© 2011 Pearson Education, Inc., Upper Saddle River, NJ. All rights reserved. This publication is protected by Copyright and written permission should be
MACHINE DESIGN - An Integrated Approach, 4th Ed. 3-26a-1
PROBLEM 3-26a
Statement: A beam is supported and loaded as shown in Figure P3-11d. Find the reactions, maximum shear,
and maximum moment for the data given in row a from Table P3-1.
Given: Beam length L  1  m
L
Distance to distributed load a  0.4 m
b
Distance to reaction load b  0.6 m a F
1
Distributed load magnitude w  200  N  m w

Concentrated load F  500  N


R1 R2
Solution: See Figures 3-26 and Mathcad file P0326a.
FIGURE 3-26A
Free Body Diagram for Problem 3-26

1. From inspection of Figure 3-26aA, write the load function equation

q(x) = R1<x - 0>-1 - w<x - a>0 + R2<x - b>-1 - F<x - a>-1

2. Integrate this equation from - to x to obtain shear, V(x)

V(x) = R1<x - 0>0 - w<x - a>1 + R2<x - b>0 - F<x - a>0

3. Integrate this equation from - to x to obtain moment, M(x)

M(x) = R1<x - 0>1 - w<x - a>2/2 + R2<x - b>1 - F<x - a>1

4. Solve for the reactions by evaluating the shear and moment equations at a point just to the right of x = L,
where both are zero.
At x = L+, V = M = 0

V = R1  w  ( L  a )  R2  F = 0

w 2
M = R 1 L   ( L  a )  R2 ( L  b )  F ( L  a ) = 0
2

   ( L  a )  F  ( b  a )  w ( L  a )  ( L  b )
1 w 2
R1  R1  147  N
b 2 
R2  w ( L  a )  F  R1 R2  473  N

5. Define the range for x x  0  m 0.005  L  L

6. For a Mathcad solution, define a step function S. This function will have a value of zero when x is less than
z, and a value of one when it is greater than or equal to z.

S ( x z)  if ( x  z 1 0 )

7. Write the shear and moment equations in Mathcad form, using the function S as a multiplying factor to get
the effect of the singularity functions.

V ( x)  R1 S ( x 0  m)  w S ( x a )  ( x  a )  R2 S ( x b )  F  S ( x a )

w 2
M ( x)  R1 S ( x 0  m)  x   S ( x a )  ( x  a )  R2 S ( x b )  ( x  b )  F  S ( x a )  ( x  a )
2
© 2011 Pearson Education, Inc., Upper Saddle River, NJ. All rights reserved. This publication is protected by Copyright and written permission should be
MACHINE DESIGN - An Integrated Approach, 4th Ed. 3-26a-2

8. Plot the shear and moment diagrams.

Shear 500
Diagram
250

V ( x)
0
N

 250

 500
0 0.2 0.4 0.6 0.8
x
m

Moment 60
Diagram

40

M ( x)
20
Nm

 20
0 0.2 0.4 0.6 0.8
x
m

FIGURE 3-26aB
Shear and Moment Diagrams for Problem 3-26a

9. Determine the maximum shear and maximum moment from inspection of the diagrams.

Maximum shear: Vmax  V ( b  0.001  mm) Vmax  393  N

Maximum moment occurs where V is zero, which is x = a:

Mmax  M ( a ) Mmax  58.7 N  m

© 2011 Pearson Education, Inc., Upper Saddle River, NJ. All rights reserved. This publication is protected by Copyright and written permission should be
MACHINE DESIGN - An Integrated Approach, 4th Ed. 3-27-1
PROBLEM 3-27
Statement: A storage rack is to be designed to hold the paper roll of Problem 3-8 as shown in Figure
P3-12. Determine the reactions and draw the shear and moment diagrams for the mandrel that
extends 50% into the roll.

Given: Paper roll dimensions OD  1.50 m


ID  0.22 m
Lroll  3.23 m
3
Roll density ρ  984  kg m

Assumptions: 1. The paper roll's weight creates a concentrated load acting at the tip of the mandrel.
2. The mandrel's root in the stanchion experiences a distributed load over its length of engagemen
Solution: See Figure 3-27 and Mathcad file P0327.
W
1. Determine the weight of the roll
and the length of the mandrel.

W 
π
4
 2 2 
 OD  ID  Lroll  ρ  g

W  53.9 kN
M1 Lm
R1
Lm  0.5 Lroll
FIGURE 3-27
Lm  1.615  m Free Body Diagram for Problem 3-27

2. From inspection of Figure 3-27, write the load function equation

q(x) = -M1<x - 0>-2 + R1<x - 0>-1 - W<x - L>-1

3. Integrate this equation from - to x to obtain shear, V(x)

V(x) = -M1<x - 0>-1 + R1<x - 0>0 - W<x - L>0

4. Integrate this equation from - to x to obtain moment, M(x)

M(x) = -M1<x - 0>0 + R1<x - 0>1 - W<x - L>1

5. Solve for the reactions by evaluating the shear and moment equations at a point just to the right of x = L,
where both are zero.
At x = L+, V = M = 0

V = R1  W = 0 M = M1  R1 L = 0

R1  W R1  53.895 kN

M1  R1 Lm M1  87.040 kN  m

6. Define the range for x x  0  m 0.005  Lm  Lm

© 2011 Pearson Education, Inc., Upper Saddle River, NJ. All rights reserved. This publication is protected by Copyright and written permission should be
MACHINE DESIGN - An Integrated Approach, 4th Ed. 3-27-2

7. For a Mathcad solution, define a step function S. This function will have a value of zero when x is less than
z, and a value of one when it is greater than or equal to z.

S ( x z)  if ( x  z 1 0 )

8. Write the shear and moment equations in Mathcad form, using the function S as a multiplying factor to get
the effect of the singularity functions.

V ( x)  R1 S ( x 0  m)  W  S  x Lm

M ( x)  M1  R1 S ( x 0  m)  x  W  S  x Lm   x  Lm

9. Plot the shear and moment diagrams.

Shear
Diagram

40
V ( x)
kN
20

0
0 0.5 1 1.5 2
x
m

Moment 20
Diagram 1.615

 10

M ( x)
 40
kN  m

 70

 100
0 0.5 1 1.5 2
x
m

FIGURE 3-27B
Shear and Moment Diagrams for Problem 3-27

© 2011 Pearson Education, Inc., Upper Saddle River, NJ. All rights reserved. This publication is protected by Copyright and written permission should be
MACHINE DESIGN - An Integrated Approach, 4th Ed. 3-28-1
PROBLEM 3-28
Statement: Figure P3-13 shows a forklift truck negotiating a 15 deg ramp to to drive onto a 4-ft-high loading
platform. The truck weighs 5 000 lb and has a 42-in wheelbase. Determine the reactions and draw
the shear and moment diagrams for the worst case of loading as the truck travels up the ramp.

Given: Ramp angle θ  15 deg


Platform height h  4  ft h  48 in
Truck weight W  5000 lbf
Truck wheelbase Lt  42 in

Assumptions: 1. The worst case is when the truck CG is located at the center of the beam's span.
2. Use a coordinate frame that has the x-axis along the long axis of the beam.
3. Ignore traction forces and the weight components along the x-axis of the beam.
4. There are two ramps, one for each side of the forklift.
5. The location of the CG in Figure P3-13 is 32 in from the front wheel and 10 in from the rear
wheel.
CGa  32 in CGb  10 in
Solution: See Figure 3-28 and Mathcad file P0328.

L
b
a CG a
y
CG b

R1

Fa Fb x
Wa
Wb
R2

FIGURE 3-28A
Dimensions and Free Body Diagram for Problem 3-28

1. Determine the length of the beam between supports and the distances a and b.
h
Length of beam L  L  15.455 ft
sin( θ )

L L
With the CG at midspan, we have a  CGa = a   CGa a  5.061  ft
2 2

L
and b   CGb b  8.561  ft
2

© 2011 Pearson Education, Inc., Upper Saddle River, NJ. All rights reserved. This publication is protected by Copyright and written permission should be
MACHINE DESIGN - An Integrated Approach, 4th Ed. 3-28-2

2. The weight distribution on the wheels is determined from the distance from the front wheel to the CG. Each
wheel weight is divided by 2 to get the weight on a single ramp.
CGb W
Weight on front wheel Wa   Wa  595  lbf
Lt 2

W
Weight on rear wheel Wb   Wa Wb  1905 lbf
2

3. The normal force on the ramp at each wheel is adjusted for the ramp angle.

Load at front wheel Fa  Wa cos( θ ) Fa  575  lbf


Load at rear wheel Fb  Wb cos( θ ) Fb  1840 lbf

4. From inspection of Figure 3-28A, write the load function equation

q(x) = R1<x - 0>-1 - Fa<x - a>-1 - Fb<x - b>-1 + R2<x - L>-1

5. Integrate this equation from - to x to obtain shear, V(x)

V(x) = R1<x - 0>0 - Fa<x - a>0 - Fb<x - b>0 + R2<x - L>0

6. Integrate this equation from - to x to obtain moment, M(x)

M(x) = R1<x - 0>1 - Fa<x - a>1 - Fb<x - b>1 + R2<x - L>1

7. Solve for the reactions by evaluating the shear and moment equations at a point just to the right of x = L,
where both are zero.
At x = L+, V = M = 0

V = R1  Fa  Fb  R2 = 0

M = R1 L  Fa ( L  a )  Fb ( L  b ) = 0

1
R1   Fa ( L  a )  Fb ( L  b ) R1  1207 lbf
L

R2  Fa  Fb  R1 R2  1207 lbf

8. Define the range for x x  0  m 0.005  L  L

9. For a Mathcad solution, define a step function S. This function will have a value of zero when x is less than
z, and a value of one when it is greater than or equal to z.

S ( x z)  if ( x  z 1 0 )

10. Write the shear and moment equations in Mathcad form, using the function S as a multiplying factor to get the
effect of the singularity functions.

V ( x)  R1 S ( x 0  m)  Fa S ( x a )  Fb S ( x b )  R2 S ( x L)

M ( x)  R1 S ( x 0  m)  x  Fa S ( x a )  ( x  a )  Fb S ( x b )  ( x  b )  R2 S ( x L)  ( x  L)

© 2011 Pearson Education, Inc., Upper Saddle River, NJ. All rights reserved. This publication is protected by Copyright and written permission should be
MACHINE DESIGN - An Integrated Approach, 4th Ed. 3-28-3

11. Plot the shear and moment diagrams.

Shear 2000
Diagram
1000

V ( x)
0
lbf

 1000

 2000
0 2 4 6 8 10 12 14 16
x
ft

Moment 10000
Diagram 15.455

8000

6000
M ( x)
ft  lbf
4000

2000

0
0 2 4 6 8 10 12 14 16
x
ft

FIGURE 3-28B
Shear and Moment Diagrams for Problem 3-28

© 2011 Pearson Education, Inc., Upper Saddle River, NJ. All rights reserved. This publication is protected by Copyright and written permission should be
MACHINE DESIGN - An Integrated Approach, 4th Ed. 3-29-1

PROBLEM 3-29 _____

Statement: Run the TKSolver or Mathcad model for Case Study 1A and move the point of application of the
hand force along the lever by changing the values of Rb2, recalculate and observe the changes to
the forces and moments.
Problem: Determine the forces on the elements of the bicycle brake lever assembly shown in Figure 3-1
during braking.
Given: The geometry of each element is known. The average human's hand can develop a grip force of
about 267 N (60 lb) in the lever position shown.
Magnitude of handle force Fb2 Fb2 := 267⋅ N
Direction of handle force Fb2 θb2 := 270⋅ deg
Direction of cable force Fc2 θc2 := 184⋅ deg
Direction of cable force Fcable θcable := 180⋅ deg

Position vector components (Change the value of Rb2x and note the results)
Rb2x := 19⋅ mm Rc2x := −25⋅ mm R12x := −12⋅ mm
Rb2y := −4⋅ mm Rc2y := 0⋅ mm R12y := −7⋅ mm

R21x := 7⋅ mm Rb1x := 47.5⋅ mm R31x := −27⋅ mm


R21y := 19⋅ mm Rb1y := −14⋅ mm R31y := 30⋅ mm

Assumptions: The accelerations are negligible. All forces are coplanar and two-dimensional. A class 1 load
model is appropriate and a static analysis is acceptable. The higher applied load will be used as a
worst case, assuming that it can be reached before bottoming the tip of the handle on the handgrip.
If that occurs, it will change the beam's boundary conditions and the analysis.

Solution: See Figures 3-1, 3-2, and Mathcad file P0329.


1. Figure 3-1 shows the hand brake lever assembly, which consists of three subassemblies: the handlebar (1), the
lever (2), and the cable (3). The lever is pivoted to the handlebar and the cable is connected to the lever. The
cable runs within a plastic-lined sheath (for low friction) down to the brake caliper assembly at the bicycle's
wheel rim. The user's hand applies equal and opposite forces at some point on the lever and handgrip. These
forces are transformed to a larger force in the cable by reason of the lever ratio of part 2.

Figure 3-1 is a free-body diagram of the entire assembly since it shows all the forces and moments acting on it
except for its weight, which is small compared to the applied forces and is thus neglected for this analysis. The
"broken away" portion of the handlebar provides internal x and y force components and a moment. These are
arbitrarily shown as positive in sign. Their actual signs will "come out in the wash" in the calculations. The
known applied forces are shown in their actual directions and senses.

2. Figure 3-2 shows the three subassembly elements separated and drawn as free-body diagrams with all relevant
forces and moments applied to each element, again neglecting the weights of the parts. The lever (part 2) has
three forces on it, Fb2, Fc2, and F12. The two-character subscript notation used here should be read as, force of
element 1 on 2 (F12) or force at B on 2 (Fb2), etc. This defines the source of the forces (first subscript) and the
element on which it acts (second subscript).

This notation will be used consistently throughout this text for both forces and position vectors such as Rb2,
Rc2, and R12 in Figure 3-2, which serve to locate the above three forces in a local, non rotating coordinate system
whose origin is at the center of gravity (CG) of the element or subassembly being analyzed. (See foot note on
page 83 of the text).

On this brake lever, Fb2 is an applied force whose magnitude and direction are known. Fc2 is the force in the

© 2011 Pearson Education, Inc., Upper Saddle River, NJ. All rights reserved. This publication is protected by Copyright and written permission should be
MACHINE DESIGN - An Integrated Approach, 4th Ed. 3-29-2
cable. Its direction is known but not its magnitude. Force F12 is provided by part 1 on part 2 at the pivot pin.

Its magnitude and direction are both unknown. We can write equations 3.3b for this element to sum forces
in the x and y directions and sum moments about the CG. Note that all unknown reactive forces and moments are
initially assumed positive in the equations. Their true signs will come out in the calculation. (See foot note on
page 84 of the text).

ΣFx = Fb2x + Fc2x + F12x = 0

ΣFy = Fb2y + Fc2y + F12y = 0 (a)

ΣMz = ( R12 × F12) + ( Rb2 × Fb2) + ( Rc2 × Fc2 ) = 0

The cross products in the moment equation represent the "turning forces" or moments created by the
application of these forces at points remote from the CG of the element. Recall that these cross products can be
expanded to

ΣMz = ( R12x⋅ F12y − R12y⋅ F12x) ... =0


+ ( Rb2x⋅ Fb2y − Rb2y⋅ Fb2x) ... (b)
+ ( Rc2x⋅ Fc2y − Rc2y⋅ Fc2x)

We have three equations and four unknowns (F12x, F12y, Fc2x, Fc2y) at this point, so we need another equation. It
is available from the fact that the direction of Fc2 is known. (The cable can pull only along its axis). We can
express one component of the cable force Fc2 in terms of its other component and the known angle θc2 of the
cable.
Fc2y = Fc2x⋅ tan( θc2 ) (c)

We will now use a Mathcad solve block to solve equations a through c.

Calculate components of Fb2

Fb2x := Fb2⋅ cos( θb2) Fb2x = −0 ⋅ N

Fb2y := Fb2⋅ sin( θb2) Fb2y = −267⋅ N

Guess F12x := 1000⋅ N Fc2x := −1000⋅ N F12y := 1000⋅ N Fc2y := −1000⋅ N

Given Fb2x + Fc2x + F12x = 0

Fb2y + Fc2y + F12y = 0

(R12x⋅ F12y − R12y⋅ F12x) ... = 0


+ ( Rb2x⋅ Fb2y − Rb2y⋅ Fb2x) ...
+ ( Rc2x⋅ Fc2y − Rc2y⋅ Fc2x)

Fc2y = Fc2x⋅ tan( θc2 )

© 2011 Pearson Education, Inc., Upper Saddle River, NJ. All rights reserved. This publication is protected by Copyright and written permission should be
MACHINE DESIGN - An Integrated Approach, 4th Ed. 3-29-3
F12x
F12y
:= Find ( F12x , F12y , Fc2x , Fc2y)
Fc2x
Fc2y

Components of the unknown forces F12, and Fc2

F12x = 1047⋅ N Fc2x = −1047⋅ N F12y = 340⋅ N Fc2y = −73.2⋅ N

3. Part 3 in Figure 3-2 is the cable that passes through a hole in part 1. This hole is lined with a low friction material,
which allows us to assume no friction at the joint between parts 1 and 3. We will further assume that the three
forces F13, Fc3, and Fcable form a concurrent system of forces acting through the CG and thus create no moment.
With this assumption, only a summation of forces is necessary for this element.
ΣFx = Fcablex + F13x + Fc3x = 0
(d)
ΣFy = Fcabley + F13y + Fc3y = 0

Using Newton's third law, we have Fc3x := −Fc2x and Fc3y := −Fc2y.

We also assume that the cable entering from the left is horizontal and that the reaction F13 is vertical, thus

Fcabley := 0 ⋅ N and F13x := 0 ⋅ N (e)

We can now solve for the forces on part 3 directly,


Fcablex := −F13x − Fc3x Fcablex = −1047⋅ N

F13y := −Fcabley − Fc3y F13y = −73.2⋅ N

The assembly of elements labeled part 1 in Figure 3-2 has both force and moments on it (i.e., it is not a concurrent
system), so the three equations 3.3b are needed.

ΣFx = F21x + Fb1x + F31x + Px + Fsheathx = 0

ΣFy = F21y + Fb1y + F31y + Py = 0 (f)

ΣMz = Mh + ( R21 × F21) + ( Rb1 × Fb1) + ( R31 × F31) ... = 0


+ ( Rp × Fp) + ( Rd × Fsheath )

Expanding cross products in the moment equation gives the moment magnitude as

ΣMz = Mh + ( R21x⋅ F21y − R21y⋅ F21x) ... = 0


+ ( Rb1x⋅ Fb1y − Rb1y⋅ Fb1x) ...
+ ( R31x⋅ F31y − R31y⋅ F31x) ... (g)
+ ( RPx⋅ FPy − RPy ⋅ FPx) ...
+ ( 0 − Rdy ⋅ Fsheathx)

Using Newton's third law, we have


F31x := −F13x F21x := −F12x Fb1x := −Fb2x
(h)
F31y := −F13y F21y := −F12y Fb1y := −Fb2y

© 2011 Pearson Education, Inc., Upper Saddle River, NJ. All rights reserved. This publication is protected by Copyright and written permission should be
MACHINE DESIGN - An Integrated Approach, 4th Ed. 3-29-4
Fsheathx := −Fcablex

Given RPx := −27⋅ mm RPy := 0 ⋅ mm

Rdx := −41⋅ mm Rdy := 27⋅ mm

We will now use a Mathcad solve block to solve equations (f) through (h).

Guess Px := 1000⋅ N Mh := −100⋅ N ⋅ m Py := 0 ⋅ N

Given F21x + Fb1x + F31x + Px + Fsheathx = 0

F21y + Fb1y + F31y + Py = 0

Mh + ( R21x⋅ F21y − R21y⋅ F21x) ... = 0


+ ( Rb1x⋅ Fb1y − Rb1y⋅ Fb1x) ...
+ ( R31x⋅ F31y − R31y⋅ F31x) ...
+ ( RPx⋅ Py − RPy ⋅ Px) ...
+ ( 0 ⋅ N ⋅ m − Rdy⋅ Fsheathx)

Px
Py := Find( Px , Py , Mh)
Mh

Summarizing, the results obtained for a grip force Fb2 = 267⋅ N are:

Handlebar (1) Fb1x = 0 ⋅ N Fb1y = 267⋅ N


F21x = −1047⋅ N F21y = −340⋅ N
F31x = 0 ⋅ N F31y = 73.2⋅ N

−6
Px = 1 × 10 ⋅N Py = 0 ⋅ N
Mh = 0.0⋅ N ⋅ m

Lever (2) Fc2x = −1047⋅ N Fc2y = −73.2⋅ N


F12x = 1047⋅ N F12y = 340⋅ N

Cable (3) Fc3x = 1047⋅ N Fc3y = 73.2⋅ N


F13x = 0 ⋅ N F13y = −73.2⋅ N
Fcablex = −1047⋅ N Fcabley = 0 ⋅ N

© 2011 Pearson Education, Inc., Upper Saddle River, NJ. All rights reserved. This publication is protected by Copyright and written permission should be
MACHINE DESIGN - An Integrated Approach, 4th Ed. 3-30-1

PROBLEM 3-30 _____


Statement: Run the TKSolver or Mathcad model for Case Study 2A and move the point of application of the
crimp force along the jaw by changing the values of Rhand, recalculate and observe the changes to
the forces and moments.
Problem: Determine the forces on the elements of the crimping tool shown in Figure 3-3 during a crimp
operation.
Given: The geometry is known and the tool develops a crimp force of 2000 lb (8896 N) at closure in the
position shown.
Applied crimp force Fc4x := −1956.30⋅ lbf Fc4y := 415.82⋅ lbf
Position vector components (Change the value of Rhand and note the results)

Rc4x := 0.454⋅ in R12x := 1.399⋅ in R32x := 2.199⋅ in


Rc4y := 0.337⋅ in R12y := 0.049⋅ in R32y := 0.077⋅ in
R23x := −0.602⋅ in R43x := 0.602⋅ in R14x := −0.161⋅ in
R23y := 0.127⋅ in R43y := −0.127⋅ in R14y := −0.758⋅ in
R34x := 0.161⋅ in R34y := 0.758⋅ in Rhand := −4.40⋅ in

Assumptions: The accelerations are negligible. All forces are coplanar and two-dimensional. A class 1 load
model is appropriate and a static analysis is acceptable.
Solution: See Figures 3-3, 3-4, and Mathcad file P0330.

1. Figure 3-3 shows the tool in the closed position, in the process of crimping a metal connector onto a wire. The
user's hand provides the input forces between links 1 and 2, shown as the reaction pair Fhand. The user can grip
the handle anywhere along its length but we are assuming a nominal moment arm of Rhand for the application of
the resultant of the user's grip force (see Figure 3-4). The high mechanical advantage of the tool transforms the
grip force to a large force at the crimp.

Figure 3-3 is a free-body diagram of the entire assembly, neglecting the weight of the tool, which is small
compared to the crimp force. There are four elements, or links, in the assembly, all pinned together. Link 1 can
be considered to be the "ground" link, with the other links moving with respect to it as the jaw is closed. The
desired magnitude of the crimp force Fc is defined and its direction will be normal to the surfaces at the crimp.

2. Figure 3-4 shows the elements of the crimping tool assembly separated and drawn as free-body diagrams with all
forces applied to each element, again neglecting their weights as being insignificant compared to the applied
forces. The centers of gravity of the respective elements are used as the origins of the local, non rotating
coordinate systems in which the points of application of all forces on the element are located. (See footnote on
page 116 of the text).

3. We will consider link 1 to be the ground plane and analyze the remaining moving links. Note that all unknown
forces and moments are initially assumed positive. Link 4 has three forces acting on it: Fc4 is the known (desired)
force at the crimp, and F14 and F34 are the reaction forces from links 1 and 3, respectively. The magnitudes of
these two forces are unknown as is the direction of F14. The direction of F34 will be the same as link 3, since it is
a two-force member. Writing equations 3.3b for this element:
ΣFx = F14x + F34x + Fc4x = 0

ΣFy = F14y + F34y + Fc4y = 0 (a)

ΣMz = ( R14x⋅ F14y − R14y⋅ F14x) ... =0


+ ( R34x⋅ F34y − R34y⋅ F34x) ...
+ ( Rc4x⋅ Fc4y − Rc4y⋅ Fc4x)
© 2011 Pearson Education, Inc., Upper Saddle River, NJ. All rights reserved. This publication is protected by Copyright and written permission should be
MACHINE DESIGN - An Integrated Approach, 4th Ed. 3-30-2

We have three equations and four unknowns (F14x, F14y, F34x, F34y) at this point, so we need another equation.
It is available from the fact that the direction of F34 is known. We can express one component of the force F34 in
terms of its other component and the known angle θ3 of link 3.
(b)
F34y = F34x⋅ tan( θ3)
(c)
where θ3 := 168⋅ deg

Guess F14x := 500⋅ lbf F34x := 1000⋅ lbf F14y := −100⋅ lbf F34y := −100⋅ lbf

Given F14x + F34x + Fc4x = 0 F14y + F34y + Fc4y = 0

(Rc4x⋅ Fc4y − Rc4y⋅ Fc4x) ... = 0


+ ( R14x⋅ F14y − R14y⋅ F14x) ...
+ ( R34x⋅ F34y − R34y⋅ F34x)

F34y = F34x⋅ tan( θ3)

F14x
F14y
:= Find ( F14x , F14y , F34x , F34y)
F34x
F34y

Components of the unknown forces F14, and F34

F14x = 442.9⋅ lbf F14y = −94.1⋅ lbf F34x = 1513.4⋅ lbf F34y = −321.7⋅ lbf

4. Link 3 has two forces on it, F23 and F43. Because this is a two-force link, these two forces are equal in magnitude
and opposite in direction. Also, from Newton's third law, F43 = - F34. Thus,
F43x := −F34x F43y := −F34y F23x := −F43x F23y := −F43y (d)

F43x = −1513.4⋅ lbf F43y = 321.7⋅ lbf F23x = 1513.4⋅ lbf F23y = −321.7⋅ lbf

5. Link 2 has three forces acting on it: Fhand is the unknown force from the hand, and F12 and F32 are the reaction
forces from links 1 and 3, respectively. Force F12 is provided by part 1 on part 2 at the pivot pin and force F32 is
provided by part 3 acting on part 2 at their pivot pin. The magnitude and direction of F32 is known and the
direction of Fhand is known. Using equations 3.3b, we can solve for the magnitude of Fhand and the two
components of F12. From the third law,
F32x := −F23x F32y := −F23y F32x = −1513.4⋅ lbf F32y = 321.7⋅ lbf

ΣFx = F12x + F32x = 0

ΣFy = Fhand + F12y + F32y = 0 (e)

ΣMz = ( R12 × F12) + ( R32 × F32) ... = 0


+ ( Rhand × Fhand )

Guess F12x := 1500⋅ lbf F12y := −100⋅ lbf Fhand := 100⋅ lbf

© 2011 Pearson Education, Inc., Upper Saddle River, NJ. All rights reserved. This publication is protected by Copyright and written permission should be
MACHINE DESIGN - An Integrated Approach, 4th Ed. 3-30-3

Given F12x + F32x = 0

F12y + F32y + Fhand = 0

(R12x⋅ F12y − R12y⋅ F12x) ... = 0


+ ( R32x⋅ F32y − R32y⋅ F32x) ...
+ Rhand ⋅ Fhand

F12x
F12y := Find( F12x , F12y , Fhand )
Fhand

F12x = 1513.4⋅ lbf F12y = −373.4⋅ lbf Fhand = 51.7⋅ lbf

6. The four forces on link 1 can now be determined using the third law.

F21x := −F12x F21y := −F12y F41x := −F14x F41y := −F14y

F21x = −1513.4⋅ lbf F21y = 373.4⋅ lbf F41x = −442.9⋅ lbf F41y = 94.1⋅ lbf

Fc1x := −Fc4x Fc1y := −Fc4y Fc1x = 1956.3⋅ lbf Fc1y = −415.8⋅ lbf

7. The solution to this problem for the scaled dimensions in Figure 3-3 assuming a 2000-lb (8896-N) force applied at
the crimp, normal to the crimp surface, is given above. The total forces at the pivot points are:
0.5
2 2
Pivot A F12 := F12x + F12y F12 = 1559⋅ lbf
0.5
2 2
Pivot B F32 := F32x + F32y F32 = 1547⋅ lbf
0.5
2 2
Pivot C F43 := F43x + F43y F43 = 1547⋅ lbf
0.5
2 2
Pivot D F14 := F14x + F14y F14 = 453⋅ lbf

The moment that must be applied to the handles to generate the crimp force of
0.5
2 2
Crimp force Fc4 := Fc4x + Fc4y Fc4 = 2000⋅ lbf

Moment Mh := Rhand ⋅ Fhand Mh = 227⋅ lbf ⋅ in

This moment can be obtained with a force of Fhand = 52⋅ lbf applied at mid-handle. This force is within the
physiological grip-force capacity of the average human.

© 2011 Pearson Education, Inc., Upper Saddle River, NJ. All rights reserved. This publication is protected by Copyright and written permission should be
MACHINE DESIGN - An Integrated Approach, 4th Ed. 3-31-1

PROBLEM 3-31 _____


Statement: Run the TKSolver or Mathcad model for Case Study 2A and move the point of application of P
along the x direction by changing the values of Rpx, recalculate and observe the changes to the
forces and moments. What happens when the vertical force P is centered on link 3? Also, change
the angle of the applied force P to create an x component and observe the effects on the forces and
moments on the elements.
Problem: Determine the forces on the elements of the scissors-jack in the position shown in Figure 3-5.
Given: The geometry is known and the jack supports a force of 1000 lb (4448 N) in the position shown.
Support force Px := 0.0⋅ lbf Py := −1000⋅ lbf
Position vector components (Change the value of Rpx and note the results)

Rpx := −0.50⋅ in R12x := −3.12⋅ in R32x := 2.08⋅ in


Rpy := 0.87⋅ in R12y := −1.80⋅ in R32y := 1.20⋅ in

R42x := 2.71⋅ in R23x := −0.78⋅ in R43x := 0.78⋅ in


R42y := 1.00⋅ in R23y := −0.78⋅ in R43y := −0.78⋅ in

R14x := 3.12⋅ in R24x := −2.58⋅ in R34x := −2.08⋅ in


R14y := −1.80⋅ in R24y := 1.04⋅ in R34y := 1.20⋅ in

Angle of gear teeth common normal θ := −45.0⋅ deg


Assumptions: The accelerations are negligible. The jack is on level ground. The angle of the elevated car chassis
does not impart an overturning moment to the jack. All forces are coplanar and two-dimensional.
A class 1 load model is appropriate and a static analysis is acceptable.

Solution: See Figures 3-5 through 3-8, and Mathcad file P0331.
1. Figure 3-5 shows a schematic of a simple scissors jack used to raise a car. It consists of six links that are
pivoted and/or geared together and a seventh link in the form of a lead screw that is turned to raise the jack.
While this is clearly a three-dimensional device, it can be analyzed as a two-dimensional one if we assume that
the applied load (from the car) and the jack are exactly vertical (in the z direction). If so, all forces will be in the xy
plane. This assumption is valid if the car is jacked from a level surface. If not, then there will be some forces in
the yz and xz planes as well. The jack designer needs to consider the more general case, but for our simple
example we will initially assume two-dimensional loading. For the overall assembly as shown in Figure 3-5, we
can solve for the reaction force Fg, given force P, by summing forces: Fg = -P.

2. Figure 3-6 shows a set of free-body diagrams for the entire jack. Each element or subassembly of interest has
been separated from the others and the forces and moments shown acting on it (except for its weight, which is
small compared to the applied forces and is thus neglected for this analysis). The forces and moments can be
either internal reactions at interconnections with other elements or external loads from the "outside world." The
centers of gravity of the respective elements are used as the origins of the local, non rotating coordinate systems
in which the points of application of all forces on the element are located. In this design, stability is achieved by
the mating of two pairs of crude (non involute) gear segments acting between links 2 and 4 and between links 5
and 7. These interactions are modeled as forces acting along a common normal shared by the two teeth. This
common normal is perpendicular to the common tangent at the contact point.

There are 3 second-law equations available for each of the seven elements allowing 21 unknowns. An additional
10 third-law equations will be needed for a total of 31. This is a cumbersome system to solve for such a simple
device, but we can use its symmetry to advantage in order to simplify the problem.

3. Figure 3-7 shows the upper half of the jack assembly. Because of the mirror symmetry between the upper
and lower portions, the lower half can be removed to simplify the analysis. The forces calculated for this half will
be duplicated in the other. If we wished, we could solve for the reaction forces at A and B using equations 3.3b
from this free-body diagram of the half-jack assembly.
© 2011 Pearson Education, Inc., Upper Saddle River, NJ. All rights reserved. This publication is protected by Copyright and written permission should be
MACHINE DESIGN - An Integrated Approach, 4th Ed. 3-31-2

4. Figure 3-8a shows the free-body diagrams for the upper half of the jack assembly, which are essentially the same
as those of Figure 3-6. We now have four elements but can consider the subassembly labeled 1 to be the
"ground," leaving three elements on which to apply equations 3.3. Note that all forces and moments are initially
assumed positive in the equations.

5. Link 2 has three forces acting on it: F42 is the unknown force at the gear tooth contact with link 4; F12 and F32 are
the unknown reaction forces from links 1 and 3, respectively. Force F12 is provided by part 1 on part 2 at the
pivot pin and force F32 is provided by part 3 acting on part 2 at their pivot pin. The magnitudes and the
directions of these pin forces and the magnitude of F42 are unknown. The direction of F42 is along the common
normal shown in Figure 3-8b. Write equations 3.3b for this element to sum the forces in the x and y directions
and sum moments about the CG (with the cross products expanded).
ΣFx = F12x + F32x + F42x = 0

ΣFy = F12y + F32y + F42y = 0 (a)

ΣMz = ( R12x⋅ F12y − R12y⋅ F12x) ... =0


+ ( R32x⋅ F32y − R32y⋅ F32x) ...
+ ( R42x⋅ F42y − R42y⋅ F42x)

6. Link 3 has three forces acting on it: P, F23 and F43. Only P is known. Writing equations 3.3b for this element
gives
ΣFx = F23x + F43x + Px = 0

ΣFy = F23y + F43y + Py = 0 (b)

ΣMz = ( R23x⋅ F23y − R23y⋅ F23x) ... =0


+ ( R43x⋅ F43y − R43y⋅ F43x) ...
+ ( Rpx⋅ Py − Rpy⋅ Px)

7. Link 4 has three forces acting on it: F24 is the unknown force from link 2; F14 and F34 are the unknown reaction
forces from links 1 and 3, respectively.

ΣFx = F14x + F24x + F34x = 0

ΣFy = F14y + F24y + F34y = 0 (c)

ΣMz = ( R14x⋅ F14y − R14y⋅ F14x) ... =0


+ ( R24x⋅ F24y − R24y⋅ F24x) ...
+ ( R34x⋅ F34y − R34y⋅ F34x)

8. The nine equations in sets a through c have 16 unknowns in them, F12x, F12y, F32x, F32y, F23x, F23y, F43x, F43y,
F14x , F14y, F34x, F34y, F24x, F24y, F42x, F42y. We can write the third-law relationships between action-reaction pairs
at each of the joints to obtain six of the seven additional equations needed:
F32x = −F23x F32y = −F23y
F34x = −F43x F34y = −F43y (d)
F42x = −F24x F42y = −F24y

9. The last equation needed comes from the relationship between the x and y components of the force F24 (or
F42) at the tooth/tooth contact point. Such a contact (or half) joint can transmit force (excepting friction force)
only along the common normal , which is perpendicular to the joint's common tangent as shown in Figure 3-8b.
The common normal is also called the axis of transmission.
© 2011 Pearson Education, Inc., Upper Saddle River, NJ. All rights reserved. This publication is protected by Copyright and written permission should be
MACHINE DESIGN - An Integrated Approach, 4th Ed. 3-31-3

The tangent of the angle of this common normal relates the two components of the force at the joint:
F24y = F24x⋅ tan( θ ) (e)

10. Equations (d) and (e) will be substituted into equations (a) through (c) to create a set of nine simultaneous
equations for solution.
Guess F12x := 500⋅ lbf F12y := 500⋅ lbf F14x := −500⋅ lbf F14y := 500⋅ lbf

F23x := 500⋅ lbf F23y := 500⋅ lbf F24x := 500⋅ lbf

F43x := −500⋅ lbf F43y := 500⋅ lbf

Given (R12x⋅ F12y − R12y⋅ F12x) ... =0 F12x − F23x − F24x = 0


+ ( −R32x⋅ F23y + R32y⋅ F23x) ...
+ ( −R42x⋅ F24x⋅ tan( θ ) + R42y⋅ F24x) F12y − F23y − F24x⋅ tan( θ ) = 0

(R23x⋅ F23y − R23y⋅ F23x) ... = 0 F23x + F43x + Px = 0


+ ( R43x⋅ F43y − R43y⋅ F43x) ...
+ ( Rpx⋅ Py − Rpy⋅ Px) F23y + F43y + Py = 0

(R14x⋅ F14y − R14y⋅ F14x) ... =0 F14x + F24x − F43x = 0


+ ( R24x⋅ F24x⋅ tan( θ ) − R24y⋅ F24x) ...
+ ( −R34x⋅ F43y + R34y⋅ F43x) F14y + F24x⋅ tan( θ ) − F43y = 0

F12x
F12y
F14x
F14y
F23x := Find ( F12x , F12y , F14x , F14y , F23x , F23y , F24x , F43x , F43y)
F23y
F24x
F43x
F43y

Results: F14x = −877.8⋅ lbf F14y = 469.6⋅ lbf


F24x = 290.1⋅ lbf F24y := F24x⋅ tan( θ ) F24y = −290.1⋅ lbf

F34x := −F43x F34y := −F43y

F23x = 587.7⋅ lbf F23y = 820.5⋅ lbf F43x = −587.7⋅ lbf F43y = 179.5⋅ lbf

F12x = 877.8⋅ lbf F12y = 530.4⋅ lbf F32x := −F23x F32y := −F23y

F42x := −F24x F42y := −F24y

© 2011 Pearson Education, Inc., Upper Saddle River, NJ. All rights reserved. This publication is protected by Copyright and written permission should be
MACHINE DESIGN - An Integrated Approach, 4th Ed. 3-32-1

PROBLEM 3-32 _____

Statement: Figure P3-14 shows a cam-follower arm. If the load P = 200 lb, what spring force is needed at the
right end to maintain a minimum load between cam and follower of 25 lb? Find the maximum shear
force and bending moment in the follower arm. Plot the shear and moment diagrams.

Given: Load at left end of beam P := 200⋅ lbf


Load at cam follower Pcam := 25⋅ lbf
Distance from left end to: Pivot point a := 10⋅ in
Cam follower b := 22⋅ in
Spring c := 29⋅ in

Solution: See Figure P3-14 and Mathcad file P0332.

1. Draw a FBD of the cam-follower arm (beam).

c
b

R Pcam Fspring

2. From inspection of the FBD, write the load function equation


q(x) = -P<x - 0>-1 + R<x - a>-1 + Pcam<x - b >-1 - Fspring<x - 0>-1
3. Integrate this equation from -∞ to x to obtain shear, V(x)

V(x) = -P<x - 0>0 + R<x - a>0 + Pcam<x - b >0 - Fspring<x - 0>0


4. Integrate this equation from -∞ to x to obtain moment, M(x)

M(x) = -P<x - 0>1 + R<x - a>1 + Pcam<x - b >1 - Fspring<x - 0>1

5. Solve for the reactions by evaluating the shear and moment equations at a point just to the right of x = c, where
both are zero.
At x = c+, V = M = 0 V = −P + R + Pcam − Fspring = 0

M = −P⋅ c + R⋅ ( c − a ) + Pcam⋅ ( c − b ) = 0

P⋅ a + Pcam⋅ ( b − a )
Fspring := Fspring = 121.05⋅ lbf
c−a

R := Fspring + P − Pcam R = 296.05⋅ lbf

6. Define the range for x x := 0 ⋅ in , 0.002⋅ c .. c


© 2011 Pearson Education, Inc., Upper Saddle River, NJ. All rights reserved. This publication is protected by Copyright and written permission should be
MACHINE DESIGN - An Integrated Approach, 4th Ed. 3-32-2

7. For a Mathcad solution, define a step function S. This function will have a value of zero when x is less than z,
and a value of one when it is greater than or equal to z.

S ( x , z) := if ( x ≥ z , 1 , 0 )
8. Write the shear and moment equations in Mathcad form, using the function S as a multiplying factor to get the
effect of the singularity functions.

V ( x) := −P⋅ S ( x , 0 ⋅ in) + R⋅ S ( x , a ) + Pcam⋅ S ( x , b ) − Fspring⋅ S ( x , c)

M ( x) := −P⋅ S ( x , 0 ⋅ in) ⋅ x + R⋅ S ( x , a ) ⋅ ( x − a) + Pcam⋅ S ( x , b ) ⋅ ( x − b ) − Fspring⋅ S ( x , c) ⋅ ( x − c)

9. Plot the shear and moment diagrams and find the maximum shear force and bending moment.

SHEAR DIAGRAM
200

100

0
V ( x)
lbf
− 100

− 200

− 300
0 10 20 30
x
in
Vmax := V ( 0 ⋅ in) Vmax = 200⋅ lbf

MOMENT DIAGRAM
0

− 500

M ( x)
− 1000
in⋅ lbf

− 1500

− 2000
0 10 20 30
x
in

Mmax := M ( a ) Mmax = 2000⋅ in⋅ lbf

© 2011 Pearson Education, Inc., Upper Saddle River, NJ. All rights reserved. This publication is protected by Copyright and written permission should be
MACHINE DESIGN - An Integrated Approach, 4th Ed. 3-33-1

PROBLEM 3-33 _____

Statement: Write a computer program or equation solver model to calculate all the singularity functions listed
in equations 3.17. Set them up as functions that can be called from within any other program or
model.

Solution: See Mathcad file P0333.

1. No solution is provided for this programming problem.

© 2011 Pearson Education, Inc., Upper Saddle River, NJ. All rights reserved. This publication is protected by Copyright and written permission should be
MACHINE DESIGN - An Integrated Approach, 4th Ed. 3-34a-1

PROBLEM 3-34a
Statement: A beam is supported and loaded as shown in Figure P3-15. Find the reactions, maximum shear, and
maximum moment for the data given in row a from Table P3-2.

Given: Beam length L := 20⋅ in b


P
Distance to RH bearing a := 16⋅ in
Distance to concentrated load b := 18⋅ in
R1 R2
Concentrated load P := 1000⋅ lbf a

FIGURE 3-34aA
Solution: See Figure 3-34 and Mathcad file P0334a. Free Body Diagram for Problem 3-34

1. From inspection of Figure 3-34, write the load function equation

q(x) = R1<x - 0>-1 + R2<x - b>-1 - P<x - L>-1

2. Integrate this equation from -∞ to x to obtain shear, V(x)

V(x) = R1<x - 0>0 + R2<x - b>0 - P<x - L>0

3. Integrate this equation from -∞ to x to obtain moment, M(x)

M(x) = R1<x - 0>1 + R2<x - b>1 - P<x - L>1

4. Solve for the reactions by evaluating the shear and moment equations at a point just to the right of x = b, where
both are zero.
At x = b +, V = M = 0
V = R1 + R2 − P = 0
M = R1 ⋅ b + R2 ⋅ ( b − a ) = 0

P
R1 := ⋅ (a − b) R1 = −125⋅ lbf
a

R2 := P − R1 R2 = 1125⋅ lbf

5. Define the range for x x := 0 ⋅ m , 0.002⋅ L .. L

6. For a Mathcad solution, define a step function S. This function will have a value of zero when x is less than z,
and a value of one when it is greater than or equal to z.

S ( x , z) := if ( x ≥ z , 1 , 0 )

7. Write the shear and moment equations in Mathcad form, using the function S as a multiplying factor to get the
effect of the singularity functions.

V ( x) := R1⋅ S ( x , 0 ⋅ in) + R2⋅ S ( x , a ) − P⋅ S ( x , b )

M ( x) := R1⋅ S ( x , 0 ⋅ in) ⋅ x + R2⋅ S ( x , a ) ⋅ ( x − a ) − P⋅ S ( x , b ) ⋅ ( x − b)

8. Plot the shear and moment diagrams.

© 2011 Pearson Education, Inc., Upper Saddle River, NJ. All rights reserved. This publication is protected by Copyright and written permission should be
MACHINE DESIGN - An Integrated Approach, 4th Ed. 3-34a-2

SHEAR DIAGRAM MOMENT DIAGRAM


1000 1000

0
500
V ( x) M ( x)
− 1000
lbf in⋅ lbf

0
− 2000

− 500 − 3000
0 5 10 15 20 0 5 10 15 20
x x
in in

FIGURE 3-34aB
Shear and Moment Diagrams for Problem 3-34a

9. Determine the maximum shear and maximum moment from inspection of the diagrams.

Maximum shear: Vmax := V ( a ) Vmax = 1000⋅ lbf

Maximum moment occurs where V is zero, which is x = a:

Mmax := M ( a ) Mmax = 2000⋅ in⋅ lbf

© 2011 Pearson Education, Inc., Upper Saddle River, NJ. All rights reserved. This publication is protected by Copyright and written permission should be
MACHINE DESIGN - An Integrated Approach, 4th Ed. 3-35a-1

PROBLEM 3-35a
Statement: A beam is supported and loaded as shown in Figure P3-15. Write a computer program or equation
solver model to find the reactions and calculate and plot the loading, shear, and moment functions.
Test the program with the data given in row a from Table P3-2.

Input data: Enter data in highlighted areas b


F
Beam length L := 20⋅ in
Distance to RH bearing a := 16⋅ in
R1 R2
Distance to concentrated load b := 18⋅ in a
Concentrated load F := 1000⋅ lbf
FIGURE 3-34aA
Solution: See Figures 3-35 and Mathcad file P0335a. Free Body Diagram for Problem 3-34

1. From inspection of Figure 3-35, write the load function equation

q(x) = R1<x - 0>-1 + R2<x - b>-1 - F<x - L>-1

2. Integrate this equation from -∞ to x to obtain shear, V(x)

V(x) = R1<x - 0>0 + R2<x - b>0 - F<x - L>0

3. Integrate this equation from -∞ to x to obtain moment, M(x)

M(x) = R1<x - 0>1 + R2<x - b>1 - F<x - L>1

4. Solve for the reactions by evaluating the shear and moment equations at a point just to the right of x = b, where
both are zero.
At x = b +, V = M = 0
V = R1 + R2 − F = 0
M = R1 ⋅ b + R2 ⋅ ( b − a ) = 0

F
R1 := ⋅ ( a − b) R1 = −125⋅ lbf
a

R2 := F − R1 R2 = 1125⋅ lbf

5. Define the range for x x := 0 ⋅ m , 0.002⋅ L .. L

6. For a Mathcad solution, define a step function S. This function will have a value of zero when x is less than z,
and a value of one when it is greater than or equal to z.

S ( x , z) := if ( x ≥ z , 1 , 0 )

7. Write the shear and moment equations in Mathcad form, using the function S as a multiplying factor to get the
effect of the singularity functions.

V ( x) := R1⋅ S ( x , 0 ⋅ in) + R2⋅ S ( x , a ) − F ⋅ S ( x , b )

M ( x) := R1⋅ S ( x , 0 ⋅ in) ⋅ x + R2⋅ S ( x , a ) ⋅ ( x − a ) − F ⋅ S ( x , b ) ⋅ ( x − b )

8. Plot the shear and moment diagrams.


© 2011 Pearson Education, Inc., Upper Saddle River, NJ. All rights reserved. This publication is protected by Copyright and written permission should be
MACHINE DESIGN - An Integrated Approach, 4th Ed. 3-35a-2

SHEAR DIAGRAM MOMENT DIAGRAM


1000 1000

0
500
V ( x) M ( x)
− 1000
lbf in⋅ lbf

0
− 2000

− 500 − 3000
0 5 10 15 20 0 5 10 15 20
x x
in in

FIGURE 3-34aB
Shear and Moment Diagrams for Problem 3-35a

9. Determine the maximum shear and maximum moment from inspection of the diagrams.

Maximum shear: Vmax := V ( a ) Vmax = 1000⋅ lbf

Maximum moment occurs where V is zero, which is x = a:

Mmax := M ( a ) Mmax = 2000⋅ in⋅ lbf

© 2011 Pearson Education, Inc., Upper Saddle River, NJ. All rights reserved. This publication is protected by Copyright and written permission should be
MACHINE DESIGN - An Integrated Approach, 4th Ed. 3-36a-1

PROBLEM 3-36a
Statement: A beam is supported and loaded as shown in Figure P3-16. Find the reactions, maximum shear, and
maximum moment for the data given in row a from Table P3-2.
Given: Beam length L := 20⋅ in
Distance to RH bearing L := 20⋅ in b
a
Distance to start of load a := 16⋅ in p

Distance to end of load b := 18⋅ in


lbf R1 L R2
Distributed load p := 1000⋅
in
FIGURE 3-36aA
Solution: See Figures 3-36 and Mathcad file P0336a. Free Body Diagram for Problem 3-36

1. From inspection of Figure 3-36, write the load function equation

q(x) = R1<x - 0>-1 - p<x - a>0 + p<x - b>0 + R2<x - L>-1

2. Integrate this equation from -∞ to x to obtain shear, V(x)

V(x) = R1<x - 0>0 - p<x - a>1 + p<x - b>1 + R2<x - L>0

3. Integrate this equation from -∞ to x to obtain moment, M(x)

M(x) = R1<x - 0>1 - p<x - a>2/2 + p<x - b>2/2 + R2<x - L>1

4. Solve for the reactions by evaluating the shear and moment equations at a point just to the right of x = L, where
both are zero.
At x = L+, V = M = 0

V = R1 − p ⋅ ( L − a ) + p ⋅ ( L − b ) + R2 = 0
p 2 p 2
M = R1 ⋅ L − ⋅ (L − a) + ⋅ ( L − b ) + R2⋅ ( L − b) = 0
2 2

p 2 2
R1 := ⋅ 2⋅ (b − a)⋅ L + a − b R1 = 300⋅ lbf
2⋅ L

R2 := p ⋅ ( b − a ) − R1 R2 = 1700⋅ lbf

5. Define the range for x x := 0 ⋅ in , 0.002⋅ L .. L

6. For a Mathcad solution, define a step function S. This function will have a value of zero when x is less than z,
and a value of one when it is greater than or equal to z.

S ( x , z) := if ( x ≥ z , 1 , 0 )

7. Write the shear and moment equations in Mathcad form, using the function S as a multiplying factor to get the
effect of the singularity functions.

V ( x) := R1⋅ S ( x , 0 ⋅ in) − p ⋅ S ( x , a ) ⋅ ( x − a) + p ⋅ S ( x , b ) ⋅ ( x − b ) + R2⋅ S ( x , L)

p 2 p 2
M ( x) := R1⋅ S ( x , 0 ⋅ in) ⋅ x − ⋅ S(x , a)⋅ ( x − a) + ⋅ S ( x , b ) ⋅ ( x − b ) + R2⋅ S ( x , L) ⋅ ( x − L)
2 2

© 2011 Pearson Education, Inc., Upper Saddle River, NJ. All rights reserved. This publication is protected by Copyright and written permission should be
MACHINE DESIGN - An Integrated Approach, 4th Ed. 3-36a-2

8. Plot the shear and moment diagrams.

SHEAR DIAGRAM MOMENT DIAGRAM


1000 5000

4000

0
3000
V ( x) M ( x)
lbf in⋅ lbf
2000
− 1000

1000

− 2000 0
0 5 10 15 20 0 5 10 15 20
x x
in in

FIGURE 3-36aB
Shear and Moment Diagrams for Problem 3-36a

9. Determine the maximum shear and maximum moment from inspection of the diagrams.

Maximum shear: Vmax := V ( b ) Vmax = 1700⋅ lbf

Maximum moment occurs where V is zero, which is x = c, where:

R1 ⋅ b + R2 ⋅ a
c := c = 16.3⋅ in
R1 + R2

Mmax := M ( c) Mmax = 4845⋅ in⋅ lbf

© 2011 Pearson Education, Inc., Upper Saddle River, NJ. All rights reserved. This publication is protected by Copyright and written permission should be
MACHINE DESIGN - An Integrated Approach, 4th Ed. 3-37a-1

PROBLEM 3-37a
Statement: A beam is supported and loaded as shown in Figure P3-16. Write a computer program or equation
solver model to find the reactions and calculate and plot the loading, shear, and moment functions.
Test the program with the data given in row a from Table P3-2.

Input data: Enter data in highlighted areas


Beam length L := 20⋅ in b
a
Distance to RH bearing L := 20⋅ in p

Distance to start of load a := 16⋅ in


Distance to end of load R1 L R2
b := 18⋅ in
lbf
Distributed load p := 1000⋅ FIGURE 3-37aA
in Free Body Diagram for Problem 3-37

Solution: See Figures 3-37 and Mathcad file P0337a.


1. From inspection of Figure 3-37, write the load function equation
q(x) = R1<x - 0>-1 - p<x - a>0 + p<x - b>0 + R2<x - L>-1

2. Integrate this equation from -∞ to x to obtain shear, V(x)


V(x) = R1<x - 0>0 - p<x - a>1 + p<x - b>1 + R2<x - L>0

3. Integrate this equation from -∞ to x to obtain moment, M(x)


M(x) = R1<x - 0>1 - p<x - a>2/2 + p<x - b>2/2 + R2<x - L>1

4. Solve for the reactions by evaluating the shear and moment equations at a point just to the right of x = L, where
both are zero.
At x = L+, V = M = 0

V = R1 − p ⋅ ( L − a ) + p ⋅ ( L − b ) + R2 = 0
p 2 p 2
M = R1 ⋅ L − ⋅ (L − a) + ⋅ ( L − b ) + R2⋅ ( L − b) = 0
2 2

p 2 2
R1 := ⋅ 2⋅ (b − a)⋅ L + a − b R1 = 300⋅ lbf
2⋅ L

R2 := p ⋅ ( b − a ) − R1 R2 = 1700⋅ lbf

5. Define the range for x x := 0 ⋅ in , 0.002⋅ L .. L

6. For a Mathcad solution, define a step function S. This function will have a value of zero when x is less than z,
and a value of one when it is greater than or equal to z.

S ( x , z) := if ( x ≥ z , 1 , 0 )

7. Write the shear and moment equations in Mathcad form, using the function S as a multiplying factor to get the
effect of the singularity functions.

V ( x) := R1⋅ S ( x , 0 ⋅ in) − p ⋅ S ( x , a ) ⋅ ( x − a) + p ⋅ S ( x , b ) ⋅ ( x − b ) + R2⋅ S ( x , L)

p 2 p 2
M ( x) := R1⋅ S ( x , 0 ⋅ in) ⋅ x − ⋅ S(x , a)⋅ ( x − a) + ⋅ S ( x , b ) ⋅ ( x − b ) + R2⋅ S ( x , L) ⋅ ( x − L)
2 2
© 2011 Pearson Education, Inc., Upper Saddle River, NJ. All rights reserved. This publication is protected by Copyright and written permission should be
MACHINE DESIGN - An Integrated Approach, 4th Ed. 3-37a-2

8. Plot the shear and moment diagrams.

SHEAR DIAGRAM MOMENT DIAGRAM


1000 5000

4000

0
3000
V ( x) M ( x)
lbf in⋅ lbf
2000
− 1000

1000

− 2000 0
0 5 10 15 20 0 5 10 15 20
x x
in in

FIGURE 3-37aB
Shear and Moment Diagrams for Problem 3-37a

9. Determine the maximum shear and maximum moment from inspection of the diagrams.

Maximum shear: Vmax := V ( b ) Vmax = 1700⋅ lbf

Maximum moment occurs where V is zero, which is x = c, where:

R1 ⋅ b + R2 ⋅ a
c := c = 16.3⋅ in
R1 + R2

Mmax := M ( c) Mmax = 4845⋅ in⋅ lbf

© 2011 Pearson Education, Inc., Upper Saddle River, NJ. All rights reserved. This publication is protected by Copyright and written permission should be
MACHINE DESIGN - An Integrated Approach, 4th Ed. 3-38a-1

PROBLEM 3-38a
Statement: A beam is supported and loaded as shown in Figure P3-17. Find the reactions, maximum shear, and
maximum moment for the data given in row a from Table P3-2.

Given: Beam length L := 20⋅ in b


P
p
Distance to RH bearing a := 16⋅ in
Distance to concentrated load b := 18⋅ in
R1 R2
Concentrated load P := 1000⋅ lbf a
−1
Distributed load p := 1000⋅ lbf ⋅ in
FIGURE 3-38aA
Solution: See Figure 3-38 and Mathcad file P0338a. Free Body Diagram for Problem 3-38

1. Determine the distance from the origin to the left and right ends of the roller.
Distance to left end e := 0.1⋅ a e = 1.600⋅ in
Distance to right end f := 0.9⋅ a f = 14.400in

2. From inspection of Figure 3-38, write the load function equation
q(x) = R1<x - 0>-1 - p<x - e>0 + p<x - f>0 + R2<x - a>-1 - P<x - b>-1
3. Integrate this equation from -∞ to x to obtain shear, V(x)
V(x) = R1<x - 0>0 - p<x - e>1 + p<x - f>1 + R2<x - a>0 - P<x - b>0
4. Integrate this equation from -∞ to x to obtain moment, M(x)
M(x) = R1<x - 0>1 - p<x - e>2/2 + p<x - f>2/2 + R2<x - a>1 - P<x - b>1
5. Solve for the reactions by evaluating the shear and moment equations at a point just to the right of x = b, where
both are zero.
At x = b +, V = M = 0
V = R1 − p ⋅ ( b − e ) + p ⋅ ( b − f ) + R2 − P = 0
p 2 p 2
M = R1 ⋅ b − ⋅ ( b − e) + ⋅ ( b − f ) + R2⋅ ( b − a) = 0
2 2
2 2
e −f b−a
R1 := + f − e ⋅p − ⋅P R1 = 6275⋅ lbf
2⋅ a a

R2 := p ⋅ ( f − e) − R1 + P R2 = 7525⋅ lbf

6. Define the range for x x := 0 ⋅ m , 0.002⋅ L .. L

7. For a Mathcad solution, define a step function S. This function will have a value of zero when x is less than z,
and a value of one when it is greater than or equal to z.

S ( x , z) := if ( x ≥ z , 1 , 0 )
8. Write the shear and moment equations in Mathcad form, using the function S as a multiplying factor to get the
effect of the singularity functions.

V ( x) := R1⋅ S ( x , 0 ⋅ m) − p ⋅ S ( x , e) ⋅ ( x − e) + p ⋅ S ( x , f ) ⋅ ( x − f ) + R2⋅ S ( x , a ) − P⋅ S ( x , b )

p 2 p 2
M ( x) := R1⋅ S ( x , 0 ⋅ m) ⋅ x − ⋅ S ( x , e) ⋅ ( x − e) + ⋅ S ( x , f ) ⋅ ( x − f ) ...
2 2
+ R2⋅ S ( x , a ) ⋅ ( x − a) − P⋅ S ( x , b ) ⋅ ( x − b )
© 2011 Pearson Education, Inc., Upper Saddle River, NJ. All rights reserved. This publication is protected by Copyright and written permission should be
MACHINE DESIGN - An Integrated Approach, 4th Ed. 3-38a-2

9. Plot the shear and moment diagrams.

SHEAR DIAGRAM MOMENT DIAGRAM


10000 30000

5000 20000

V ( x) M ( x)
0 10000
lbf in⋅ lbf

− 5000 0

− 10000 − 10000
0 5 10 15 20 0 5 10 15 20
x x
in in

FIGURE 3-38aB
Shear and Moment Diagrams for Problem 3-38a

9. Determine the maximum shear and maximum moment from inspection of the diagrams.

Maximum shear: Vmax := V ( f ) Vmax = 6525⋅ lbf

Maximum moment occurs where V is zero, which is x = c:

c−e f −c f ⋅ R1 + e ⋅ R2 − e ⋅ P
= c := c = 7.875⋅ in
R1 R2 − P R1 + R2 − P

Mmax := M ( c) Mmax = 29728⋅ in⋅ lbf

© 2011 Pearson Education, Inc., Upper Saddle River, NJ. All rights reserved. This publication is protected by Copyright and written permission should be
MACHINE DESIGN - An Integrated Approach, 4th Ed. 3-39a-1

PROBLEM 3-39a
Statement: A beam is supported and loaded as shown in Figure P3-17. Write a computer program or equation
solver model to find the reactions and calculate and plot the loading, shear, and moment functions.
Test the program with the data given in row a from Table P3-2.

Input data: Enter data in highlighted areas b


P
Beam length L := 20⋅ in p

Distance to RH bearing a := 16⋅ in


Distance to concentrated load b := 18⋅ in R1 R2
a
Concentrated load P := 1000⋅ lbf
FIGURE 3-39aA
−1
Distributed load p := 1000⋅ lbf ⋅ in Free Body Diagram for Problem 3-39

Solution: See Figure 3-39 and Mathcad file P0339a.

1. Determine the distance from the origin to the left and right ends of the roller.
Distance to left end e := 0.1⋅ a e = 40.64⋅ mm
Distance to right end f := 0.9⋅ a f = 365.76mm

2. From inspection of Figure 3-39, write the load function equation
q(x) = R1<x - 0>-1 - p<x - e>0 + p<x - f>0 + R2<x - a>-1 - P<x - b>-1
3. Integrate this equation from -∞ to x to obtain shear, V(x)
V(x) = R1<x - 0>0 - p<x - e>1 + p<x - f>1 + R2<x - a>0 - P<x - b>0
4. Integrate this equation from -∞ to x to obtain moment, M(x)
M(x) = R1<x - 0>1 - p<x - e>2/2 + p<x - f>2/2 + R2<x - a>1 - P<x - b>1
5. Solve for the reactions by evaluating the shear and moment equations at a point just to the right of x = b, where
both are zero.
At x = b +, V = M = 0
V = R1 − p ⋅ ( b − e ) + p ⋅ ( b − f ) + R2 − P = 0
p 2 p 2
M = R1 ⋅ b − ⋅ ( b − e) + ⋅ ( b − f ) + R2⋅ ( b − a) = 0
2 2
2 2
e −f b−a
R1 := + f − e ⋅p − ⋅P R1 = 6275⋅ lbf
2⋅ a a

R2 := p ⋅ ( f − e) − R1 + P R2 = 7525⋅ lbf

6. Define the range for x x := 0 ⋅ m , 0.002⋅ L .. L

7. For a Mathcad solution, define a step function S. This function will have a value of zero when x is less than z,
and a value of one when it is greater than or equal to z.

S ( x , z) := if ( x ≥ z , 1 , 0 )
8. Write the shear and moment equations in Mathcad form, using the function S as a multiplying factor to get the
effect of the singularity functions.

V ( x) := R1⋅ S ( x , 0 ⋅ m) − p ⋅ S ( x , e) ⋅ ( x − e) + p ⋅ S ( x , f ) ⋅ ( x − f ) + R2⋅ S ( x , a ) − P⋅ S ( x , b )

© 2011 Pearson Education, Inc., Upper Saddle River, NJ. All rights reserved. This publication is protected by Copyright and written permission should be
MACHINE DESIGN - An Integrated Approach, 4th Ed. 3-39a-2

p 2 p 2
M ( x) := R1⋅ S ( x , 0 ⋅ m) ⋅ x − ⋅ S ( x , e) ⋅ ( x − e) + ⋅ S ( x , f ) ⋅ ( x − f ) ...
2 2
+ R2⋅ S ( x , a ) ⋅ ( x − a) − P⋅ S ( x , b ) ⋅ ( x − b )

9. Plot the shear and moment diagrams.

SHEAR DIAGRAM MOMENT DIAGRAM


10000 30000

5000 20000

V ( x) M ( x)
0 10000
lbf in⋅ lbf

− 5000 0

− 10000 − 10000
0 5 10 15 20 0 5 10 15 20
x x
in in

FIGURE 3-39aB
Shear and Moment Diagrams for Problem 3-39a

9. Determine the maximum shear and maximum moment from inspection of the diagrams.

Maximum shear: Vmax := V ( f ) Vmax = 6525⋅ lbf

Maximum moment occurs where V is zero, which is x = c:

c−e f −c f ⋅ R1 + e ⋅ R2 − e ⋅ P
= c := c = 7.875⋅ in
R1 R2 − P R1 + R2 − P

Mmax := M ( c) Mmax = 29728⋅ in⋅ lbf

© 2011 Pearson Education, Inc., Upper Saddle River, NJ. All rights reserved. This publication is protected by Copyright and written permission should be
MACHINE DESIGN - An Integrated Approach, 4th Ed. 3-40a-1

PROBLEM 3-40a
Statement: A beam is supported and loaded as shown in Figure P3-18. Find the reactions, maximum shear, and
maximum moment for the data given in row a from Table P3-2.

a
P1 P2
Given: Distance to gear 2 L := 20⋅ in
Distance to gear 1 a := 16⋅ in
Distance to RH bearing b := 18⋅ in R1 R2
b
Concentrated load at gear 2 P2 := 1000⋅ lbf
L
Concentrated load at gear 1 P1 := 0.4⋅ P2
FIGURE 3-40a
Solution: See Figure 3-40 and Mathcad file P0340a. Free Body Diagram for Problem 3-40

1. From inspection of Figure 3-40, write the load function equation

q(x) = R1<x - 0>-1 - P1<x - a>-1 + R2<x - b>-1 - P2<x - L>-1

2. Integrate this equation from -∞ to x to obtain shear, V(x)

V(x) = R1<x - 0>0 - P1<x - a>0 + R2<x - b>0 - P2<x - L>0

3. Integrate this equation from -∞ to x to obtain moment, M(x)

M(x) = R1<x - 0>1 - P1<x - a>1 + R2<x - b>1 - P<x - L>1

4. Solve for the reactions by evaluating the shear and moment equations at a point just to the right of x = L, where
both are zero.
At x = L+, V = M = 0
V = R1 − P1 + R2 − P2 = 0
M = R1⋅ L − P1⋅ ( L − a ) + R2⋅ ( b − a ) = 0

a L
R1 := P1⋅ 1 − + P2⋅ 1 − R1 = −67⋅ lbf
b b

R2 := P1 + P2 − R1 R2 = 1467⋅ lbf

5. Define the range for x x := 0 ⋅ m , 0.002⋅ L .. L

6. For a Mathcad solution, define a step function S. This function will have a value of zero when x is less than z,
and a value of one when it is greater than or equal to z.

S ( x , z) := if ( x ≥ z , 1 , 0 )

7. Write the shear and moment equations in Mathcad form, using the function S as a multiplying factor to get the
effect of the singularity functions.

V ( x) := R1⋅ S ( x , 0 ⋅ in) − P1⋅ S ( x , a ) + R2⋅ S ( x , b ) − P2⋅ S ( x , L)

M ( x) := R1⋅ S ( x , 0 ⋅ mm) ⋅ ( x − 0 ⋅ mm) − P1⋅ S ( x , a ) ⋅ ( x − a ) ...


+ R2⋅ S ( x , b ) ⋅ ( x − b) − P2⋅ S ( x , L) ⋅ ( x − L)

8. Plot the shear and moment diagrams.


© 2011 Pearson Education, Inc., Upper Saddle River, NJ. All rights reserved. This publication is protected by Copyright and written permission should be
MACHINE DESIGN - An Integrated Approach, 4th Ed. 3-40a-2

SHEAR DIAGRAM MOMENT DIAGRAM


1000 0

500 − 1000
V ( x) M ( x)
lbf in⋅ lbf

0 − 2000

− 500 − 3000
0 5 10 15 20 0 5 10 15 20
x x
in in

FIGURE 3-40aB
Shear and Moment Diagrams for Problem 3-40a

9. Determine the maximum shear and maximum moment from inspection of the diagrams.

Maximum shear: Vmax := V ( b ) Vmax = 1000⋅ lbf

Maximum moment occurs where V is zero, which is x = b:

Mmax := M ( b ) Mmax = 2000⋅ in⋅ lbf

© 2011 Pearson Education, Inc., Upper Saddle River, NJ. All rights reserved. This publication is protected by Copyright and written permission should be
MACHINE DESIGN - An Integrated Approach, 4th Ed. 3-41a-1

PROBLEM 3-41a
Statement: A beam is supported and loaded as shown in Figure P3-18. Write a computer program or equation
solver model to find the reactions and calculate and plot the loading, shear, and moment functions.
Test the program with the data given in row a from Table P3-2.

Input data: Enter data in highlighted areas a


P1 P2
Distance to gear 2 L := 20⋅ in
Distance to gear 1 a := 16⋅ in
Distance to RH bearing b := 18⋅ in R1 R2
b
Concentrated load at gear 2 P2 := 1000⋅ lbf
L
Concentrated load at gear 1 P1 := 0.4⋅ P2
FIGURE 3-41aA
Solution: See Figure 3-41 and Mathcad file P0341a. Free Body Diagram for Problem 3-41

1. From inspection of Figure 3-41, write the load function equation

q(x) = R1<x - 0>-1 - P1<x - a>-1 + R2<x - b>-1 - P2<x - L>-1

2. Integrate this equation from -∞ to x to obtain shear, V(x)

V(x) = R1<x - 0>0 - P1<x - a>0 + R2<x - b>0 - P2<x - L>0

3. Integrate this equation from -∞ to x to obtain moment, M(x)

M(x) = R1<x - 0>1 - P1<x - a>1 + R2<x - b>1 - P<x - L>1

4. Solve for the reactions by evaluating the shear and moment equations at a point just to the right of x = L, where
both are zero.
At x = L+, V = M = 0
V = R1 − P1 + R2 − P2 = 0
M = R1⋅ L − P1⋅ ( L − a ) + R2⋅ ( b − a ) = 0

a L
R1 := P1⋅ 1 − + P2⋅ 1 − R1 = −67⋅ lbf
b b

R2 := P1 + P2 − R1 R2 = 1467⋅ lbf

5. Define the range for x x := 0 ⋅ m , 0.002⋅ L .. L

6. For a Mathcad solution, define a step function S. This function will have a value of zero when x is less than z,
and a value of one when it is greater than or equal to z.

S ( x , z) := if ( x ≥ z , 1 , 0 )

7. Write the shear and moment equations in Mathcad form, using the function S as a multiplying factor to get the
effect of the singularity functions.

V ( z) := R1⋅ S ( z , 0 ⋅ in) − P1⋅ S ( z , a) + R2⋅ S ( z , b) − P2⋅ S ( z , L)

M ( z) := R1⋅ S ( z , 0 ⋅ mm ) ⋅ ( z − 0 ⋅ mm ) − P1⋅ S ( z , a) ⋅ ( z − a ) ...


+ R2⋅ S ( z , b) ⋅ ( z − b ) − P2⋅ S ( z , L) ⋅ ( z − L)

© 2011 Pearson Education, Inc., Upper Saddle River, NJ. All rights reserved. This publication is protected by Copyright and written permission should be
MACHINE DESIGN - An Integrated Approach, 4th Ed. 3-41a-2

8. Plot the shear and moment diagrams.

SHEAR DIAGRAM MOMENT DIAGRAM


1000 0

500 − 1000
V ( x) M ( x)
lbf in⋅ lbf

0 − 2000

− 500 − 3000
0 5 10 15 20 0 5 10 15 20
x x
in in

FIGURE 3-41aB
Shear and Moment Diagrams for Problem 3-41a

9. Determine the maximum shear and maximum moment from inspection of the diagrams.

Maximum shear: Vmax := V ( b ) Vmax = 1000⋅ lbf

Maximum moment occurs where V is zero, which is x = b:

Mmax := M ( b ) Mmax = 2000⋅ in⋅ lbf

© 2011 Pearson Education, Inc., Upper Saddle River, NJ. All rights reserved. This publication is protected by Copyright and written permission should be
MACHINE DESIGN - An Integrated Approach, 4th Ed. 3-42-1

PROBLEM 3-42 _____


Statement: A 1000 kg speedboat reaches a speed of 16 kph at the instant it takes up the slack in a 100 m-long
tow rope attached to a surfboard carrying a 100 kg passenger. If the rope has k = 5 N/m, what is
the dynamic force exerted on the surfboard?

Given: Mass of speedboat ms := 1000⋅ kg


Speed of boat vi := 16⋅ kph
Mass of passenger mp := 100⋅ kg
−1
Rope stiffness k := 5 ⋅ N ⋅ m
Assumptions: 1. The water does not influence the dynamic force.
2. An impact model can be used to estimate the dynamic force.

Solution: See Mathcad file P0342.

1. For the impact model, the passenger is the "struck" mass and the speedboat is the "striking mass". Thus, from
equation 3.15, the energy correction factor is:

1
η := η = 0.97
mp
1+
3 ⋅ ms

2. Use equation 3.11 to estimate the dynamic force on the surfboard/passenger.

Fi := vi⋅ η⋅ ms⋅ k Fi = 309⋅ N

© 2011 Pearson Education, Inc., Upper Saddle River, NJ. All rights reserved. This publication is protected by Copyright and written permission should be
MACHINE DESIGN - An Integrated Approach, 4th Ed. 3-43-1

PROBLEM 3-43
Statement: Figure P3-19 shows an oil-field pump jack. For the position shown, draw free-body diagrams of the
crank (2), connecting rod (3) and walking beam (4) using variable names similar to those used in
Case Studies 1A and 2A. Assume that the crank turns slowly enough that accelerations can be
ignored. Include the weight acting at the CG of the walking beam and the crank but not the
connecting rod.

Assumptions: 1. A two-dimensional model is adequate.


2. Inertia forces may be ignored.
Solution: See Mathcad file P0343.
1. Isolate each of the elements to be analyzed, starting with the walking beam, since the external forces on it are
known. Place the known force, Fcable, at the point P and the known weight at the CG. Assume the forces at the
interfaces O4 and B to be positive. The position vectors R14, R34, and Rp will be known as will the angle, θ3,that
the connecting rod makes with the horizontal axis.
F34

R34
y
R 14
θ3
RP B
head end F x
14y

P 4 counterweigh

O4 F 14x

W4
F cable
F43

2. The connecting rod is a two-force member with


the forces acting at the interfaces A and B along y
the line joining points A and B. The assumption B
made in step 1 is that these are compressive
forces on link 3. R 43

3 θ3

R23

F 23
© 2011 Pearson Education, Inc., Upper Saddle River, NJ. All rights reserved. This publication is protected by Copyright and written permission should be
MACHINE DESIGN - An Integrated Approach, 4th Ed. 3-43-2

3. The crank is acted on by forces at A and O2, its weight at its CG, and a torque which we will assume to be
positive (CCW). As in step 1, assume that the unknown reaction force at O2 is positive.

F32
y

F θ3
12y A
x
2
O2 F 12x
T2
W2

© 2011 Pearson Education, Inc., Upper Saddle River, NJ. All rights reserved. This publication is protected by Copyright and written permission should be
MACHINE DESIGN - An Integrated Approach, 4th Ed. 3-44-1

PROBLEM 3-44
Statement: For the pump jack of Problem 3-43 and the data of Table P3-3, determine the pin forces on the
walking beam, connecting rod, and crank and the reaction torque on the crank.
Given: R12 := 13.2⋅ in θ12 := 135⋅ deg R14 := 79.22⋅ in θ14 := 196⋅ deg
R32 := 0.80⋅ in θ32 := 45⋅ deg R34 := 32.00⋅ in θ34 := 169⋅ deg
Fcable := 2970⋅ lbf W2 := 598⋅ lbf W4 := 2706⋅ lbf
θ3 := 98.5⋅ deg RP := 124.44⋅ in θP := 185⋅ deg

Solution: See Mathcad files P0343 and P0344.


1. Draw free-body diagrams of each element (see Problem 3-43).
F34

R34
y
R 14
θ3
RP B
head end F x
14y

P 4 counterweigh

O4 F 14x

W4
F cable

F43

y
B
F32
y R 43

θ3 3 θ3
F 12y A
x x
2
O2 F 12x
R23
T2
W2
A

F 23

© 2011 Pearson Education, Inc., Upper Saddle River, NJ. All rights reserved. This publication is protected by Copyright and written permission should be
MACHINE DESIGN - An Integrated Approach, 4th Ed. 3-44-2
2. Calculate the x- and y-components of the position vectors.
R12x := R12⋅ cos( θ12) R12x = −9.334⋅ in R12y := R12⋅ sin( θ12) R12y = 9.334⋅ in

R14x := R14⋅ cos( θ14) R14x = −76.151⋅ in R14y := R14⋅ sin( θ14) R14y = −21.836⋅ in

R32x := R32⋅ cos( θ32) R32x = 0.566⋅ in R32y := R32⋅ sin( θ32) R32y = 0.566⋅ in

R34x := R34⋅ cos( θ34) R34x = −31.412⋅ in R34y := R34⋅ sin( θ34) R34y = 6.106⋅ in

RPx := RP⋅ cos( θP) RPx = −123.966 ⋅ in RPy := RP⋅ sin( θP) RPy = −10.846⋅ in

3. Write equations 3(b) for link 4, the walking beam.

Σ Fx: F14x + F34x = 0 (1)

Σ Fy: −Fcable + F14y + F34y − W4 = 0 (2)

Σ Mz: Rpx⋅ Fcable + ( R14x⋅ F14y − R14y⋅ F14x) + ( R34x⋅ F34y − R34y⋅ F34x) = 0 (3)

4. The direction (but not the sense) of F34 is known so write the equation that relates the x- and y-components of
this force.

F34y − F34x⋅ tan( θ3) = 0 (4)

5. There are four unknowns in the four equations above. Solving for F14x, F14y, F34x, and F34y,

1 0 1 0 0
Fcable + W4 F14x
0 1 0 1
A := −R14y R14x −R34y R34x lbf F14y −1
B := := A ⋅ B⋅ lbf
in in in in −RPx ⋅ Fcable F34x
0 0 −tan( θ3) 1 in⋅ lbf F34y
0
F14x = 2446⋅ lbf F14y = −10687⋅ lbf F34x = −2446⋅ lbf F34y = 16363⋅ lbf

6. From Newton's thrid law and, since the connecting rod (3) is a two-force member

F43x := −F34x F43x = 2446⋅ lbf F43y := −F34y F43y = −16363⋅ lbf

F23x := −F43x F23x = −2446⋅ lbf F23y := −F43y F23y = 16363⋅ lbf

7. Write equations 3(b) for link 2, the crank.


Σ Fx: F12x + F32x = 0 (5)

Σ Fy: F12y + F32y − W2 = 0 (6)

Σ Mz: T2 + ( R12x⋅ F12y − R12y⋅ F12x) + ( R32x⋅ F32y − R32y⋅ F32x) = 0 (7)

8. There are three unknowns in the three equations above. Solving for F12x, F12y, and T2, since
F32x := −F23x F32x = 2446⋅ lbf F32y := −F23y F32y = −16363⋅ lbf
© 2011 Pearson Education, Inc., Upper Saddle River, NJ. All rights reserved. This publication is protected by Copyright and written permission should be
MACHINE DESIGN - An Integrated Approach, 4th Ed. 3-44-3

−F32x
1 0 0 lbf
F12x
0 1 0 W2 − F32y −1
A := B := F12y := A ⋅B
−R12y R12x lbf
1 T2
in in −( R32x⋅ F32y − R32y⋅ F32x)
in⋅ lbf

F12x = −2446 lbf F12y = 16961 lbf

T2 = 146128 in-lbf

© 2011 Pearson Education, Inc., Upper Saddle River, NJ. All rights reserved. This publication is protected by Copyright and written permission should be
MACHINE DESIGN - An Integrated Approach, 4th Ed. 3-45-1

PROBLEM 3-45
Statement: Figure P3-20 shows an aircraft overhead bin mechanism in end view. For the position shown, draw
free-body diagrams of links 2 and 4 and the door (3) using variable names similar to those used in
Case Studies 1A and 2A. There are stops that prevent further clockwise motion of link 2 (and the
identical link behind it at the other end of the door) resulting in horizontal forces being applied to
the door at points A. Assume that the mechanism is symmetrical so that each set of links 2 and 4
carry one half of the door weight. Ignore the weight of links 2 and 4 as they are negligible.

Assumptions: 1. A two-dimensional model is adequate.


2. Inertia forces may be ignored as the mechanism is at rest against stops.
Solution: See Mathcad file P0345.
1. Isolate each of the elements to be analyzed, starting with the door. Place the force, Fstop, at the point A and the
known weight at the CG. Assume the forces in links 2 and 4 to be positive (tensile). The position vectors R43
and R23 will be known as will the angles θ2 and θ4 that links 2 and 4 make with the horizontal axis.

F 23
θ4

F 43
θ2
y B
F stop R43
A R23
3
x

W3
2 F 12

2. Links 2 and 4 are two-force members with the θ2


forces acting at the pinned ends along the line y
joining the pin centers. The assumption made in
step 1 is that these are tensile forces on links 2 O2
and 4.
R12
2
x
θ4 y
R32
F 14
A
O4 x
R14
4 R34
B F 34

F 32

© 2011 Pearson Education, Inc., Upper Saddle River, NJ. All rights reserved. This publication is protected by Copyright and written permission should be
MACHINE DESIGN - An Integrated Approach, 4th Ed. 3-46-1

PROBLEM 3-46
Statement: For the overhead bin mechanism of Problem 3-45 and the data of Table P3-4, determine the pin
forces on the door (3), and links 2 & 4 and the reaction force on each of the two stops.
Given: R23 := 180.0⋅ mm θ23 := 160.345⋅ deg R43 := 180.0⋅ mm θ43 := 27.862⋅ deg
W3 := 45⋅ N θ2 := 85.879⋅ deg θ4 := 172.352 ⋅ deg

Solution: See Mathcad files P0345 and P0346.


1. Draw free-body diagrams of each element (see Problem 3-45).

F 12

y θ2

O2
θ4 y
R12 F 14
2
O4 x
x R14
4 R34
B F 34
R32

F 23
θ4

F 43
F 32 θ2
y B
F stop R43
A R23
3
x

W3
2

2. Calculate the x- and y-components of the position vectors on the door (3).
R23x := R23⋅ cos( θ23) R23x = −169.512 ⋅ mm R23y := R23⋅ sin( θ23) R23y = 60.544⋅ mm

R43x := R43⋅ cos( θ43) R43x = 159.134 ⋅ mm R43y := R43⋅ sin( θ43) R43y = 84.122⋅ mm

3. Write equations 3(b) for link 3, the door.

Σ Fx: Fstop + F23x + F43x = 0 (1)

© 2011 Pearson Education, Inc., Upper Saddle River, NJ. All rights reserved. This publication is protected by Copyright and written permission should be
MACHINE DESIGN - An Integrated Approach, 4th Ed. 3-46-2
Σ Fy: F23y + F43y − 0.5⋅ W3 = 0 (2)

Σ Mz: −R23x⋅ Fstop + ( R23x⋅ F23y − R23y⋅ F23x) + ( R43x⋅ F43y − R43y⋅ F43x) = 0 (3)

4. The directions (but not the sense) of F 23 and F43 are known so write the equations that relates the x- and
y-components of these forces.

F23y − F23x⋅ tan( θ2) = 0 (4)

F43y − F43x⋅ tan( θ4) = 0 (5)

5. There are five unknowns in the five equations above. Solving for F23x, F23y, F43x, F43y, and Fstop:

1 0 1 0 1 0 F23x
0 1 0 1 0 0.5⋅ W3
F23y
−R23y R23x −R43y R43x −R23x N −1
A := B := F43x := A ⋅ B⋅ N
mm mm mm mm mm 0
F43y
−tan( θ2) 1 0 0 0 0
Fstop
0 0 −tan( θ4) 1 0 0

F23x = 1.49⋅ N F23y = 20.63⋅ N F43x = −13.96⋅ N F43y = 1.87⋅ N

The pin forces at A and B are:

2 2 2 2
F23 := F23x + F23y F23 = 20.68⋅ N F43 := F43x + F43y F43 = 14.08⋅ N

The force on each stop is: Fstop = 12.47⋅ N

6. From Newton's thrid law and, since links 2 and 4 are two-force members

F34x := −F43x F34x = 13.96⋅ N F34y := −F43y F34y = −1.87⋅ N

F32x := −F23x F32x = −1.49⋅ N F32y := −F23y F32y = −20.63⋅ N

The pin forces at O2 and O4 are numerically equal to those at A and B, respectively.

© 2011 Pearson Education, Inc., Upper Saddle River, NJ. All rights reserved. This publication is protected by Copyright and written permission should be
MACHINE DESIGN - An Integrated Approach, 4th Ed. 3-47-1

PROBLEM 3-47
Statement: A particular automobile front suspension consists of two A-arms , the wheel (with tire), a coil
spring, and a shock absorber (damper). The effective stiffness of the suspension (called the "ride
rate") is a function of the coil spring stiffness and the tire stiffness. The A-arms are designed to
give the wheel a nearly vertical displacement as the tire rides over bumps in the road. The entire
assembly can be modeled as a spring-mass-damper system as shown in Figure 3-15(b). If the
sprung mass (mass of the portion of the vehicle supported by the suspension system) weighs 675
lb, determine the ride rate that is required to achieve an undamped natural frequency of 1.4 Hz.
What is the static deflection of the suspension for the calculated ride rate?
−1
Units: Hz := 2 ⋅ π⋅ rad⋅ sec

Given: Sprung mass Ws := 675⋅ lbf Natural frequency ωn := 1.4⋅ Hz

Solution: See Figure 3-15(b) and Mathcad file P0347.

Ws 2 −1
1. Calculate the sprung mass Ms := Ms = 1.748 lbf ⋅ sec ⋅ in
g

2. Using equation 3.4, calculate the required ride rate

2 lbf
Ride rate k := ωn ⋅ Ms k = 135.28
in

3. Calculate the static deflection using equation 3.5

Ws
Static deflection δ := δ = 4.99in
k

© 2011 Pearson Education, Inc., Upper Saddle River, NJ. All rights reserved. This publication is protected by Copyright and written permission should be
MACHINE DESIGN - An Integrated Approach, 4th Ed. 3-48-1

PROBLEM 3-48
Statement: The independent suspension system of Problem 3-47 has an unsprung weight (the weight of the
axle, wheel, A-arms, etc.) of 106 lb. Calculate the natural frequency (hop resonance) of the
unsprung mass if the sum of the tire and coil spring stiffnesses is 1100 lb/in.

−1
Units: Hz := 2 ⋅ π⋅ rad⋅ sec
lbf
Given: Unsprung mass Wu := 106⋅ lbf Stiffness k := 1100⋅
in

Solution: See Figure 3-15(b) and Mathcad file P0348.

Wu 2 −1
1. Calculate the unsprung mass Mu := Mu = 0.275 lbf ⋅ sec ⋅ in
g

2. Using equation 3.4, calculate the natural frequency

k
Natural frequency ωn := ωn = 10.1 Hz
Mu

© 2011 Pearson Education, Inc., Upper Saddle River, NJ. All rights reserved. This publication is protected by Copyright and written permission should be
MACHINE DESIGN - An Integrated Approach, 4th Ed. 3-49-1

PROBLEM 3-49
Statement: The independent suspension system of Problem 3-47 has a sprung weight of 675 lb and a ride rate
of 135 lb/in. Calculate the damped natural frequency of the sprung mass if the damping coefficient
of the shock absorber is a constant 12 lb-sec/in.

−1
Units: Hz := 2 ⋅ π⋅ rad⋅ sec
lbf
Given: Sprung mass Ws := 675⋅ lbf Ride rate k := 135⋅
in
lbf ⋅ sec
Damping coefficient d := 12⋅
in

Solution: See Figure 3-15(b) and Mathcad file P0349.

Ws 2 −1
1. Calculate the sprung mass Ms := Ms = 1.748lbf ⋅ sec ⋅ in
g

2. Using equation 3.7, calculate the damped natural frequency


2
k d
Damped natural frequency ωd := − ωd = 1.29Hz
Ms 2⋅ Ms

© 2011 Pearson Education, Inc., Upper Saddle River, NJ. All rights reserved. This publication is protected by Copyright and written permission should be
MACHINE DESIGN - An Integrated Approach, 4th Ed. 3-50-1

PROBLEM 3-50_______________________________________________________

Statement: Figure P3-22 shows a powder compaction mechanism. For the position shown, draw free-body
diagrams of the input arm (2), connecting rod (3) and compacting ram (4) using variable names
similar to those used in Case Studies 1A and 2A. Assume that the input arm turns slowly
enough that accelerations can be ignored. Ignore the weights of the arm, connecting rod, and
compacting ram. Neglect friction.

Assumptions: 1. A two-dimensional model is adequate.


2. Inertia forces may be ignored.
3. The reactions at slider bearings E and F can be modeled as concentrated forces acting
horizontally at the center of each bearing.

Solution: See Mathcad file P0350. y


1. Isolate each of the elements to be analyzed, starting with the
compacting rod, since the external force on it is known. Place the
known force, Fcom , at the point P. The position vectors R14E, R14E
F14E E
R14F, and R p will be known as will the angle, q3,that the R34
compacting ram makes with the vertical axis.
D
2. The connecting rod is a two-force member with the forces x
acting at the interfaces B and D along the line joining points B
and D. The assumption made in step 1 is that these are tensile
forces on link 3. F34 R14F
F
3. The input arm is acted on by forces at A, B, and C. Assume that the F14F 3
unknown reaction force at A is positive.
RP
F43 P

Fcom
D
y Compacting Ram (4)

R43

x C

R23
y
B Fin
F32

x
Rin
F23
F12y
B
R32
Connecting Rod (3)
A
F12x
R12

Input Arm (2)


© 2011 Pearson Education, Inc., Upper Saddle River, NJ. All rights reserved. This publication is protected by Copyright and written permission should be
MACHINE DESIGN - An Integrated Approach, 4th Ed. 3-51-1

PROBLEM 3-51______________________________________________________
Statement: For the compaction mechanism of Problem 3-50 and the data of Table P3-5, determine the pin forc
on the compacting ram, connecting rod, and input arm.
Given: R12 := 148.4⋅ mm θ12 := −45⋅ deg R14E := 57.0⋅ mm θ14E := 90⋅ deg
R14F := 62.9⋅ mm θ14F := 270⋅ deg R34 := 32.00⋅ in θ34 := −105.64⋅ deg
R23 := 87.6⋅ mm θ23 := 254.36⋅ deg R43 := 87.6⋅ mm θ43 := 74.36⋅ deg
R32 := 42.9⋅ mm θ32 := 74.36⋅ deg R34 := 15.0⋅ mm θ34 := 90⋅ deg
Rin := 152.6⋅ mm θin := 225⋅ deg RP := 105.0⋅ mm θP := 270⋅ deg
Fcom := 100⋅ N θ3 := 254.36⋅ deg

Solution: See Mathcad files P0350 and P0351.


1. Draw free-body diagrams of each element (see Problem
3-50).
y

R14E
F14E E
R34
F43
D
x
D F34 R14F
y F
F14F θ3
R43 RP
P
x

R23 Fcom

Compacting Ram (4)


B
C

F23
y
Connecting Rod (3)
Fin
F32

x
Rin
F12y
B
R32
A
F12x
R12
Input Arm (2)

© 2011 Pearson Education, Inc., Upper Saddle River, NJ. All rights reserved. This publication is protected by Copyright and written permission should be
MACHINE DESIGN - An Integrated Approach, 4th Ed. 3-51-2

2. Calculate the x- and y-components of the position vectors.


R12x := R12⋅ cos( θ12) R12x = 104.935 ⋅ mm R12y := R12⋅ sin( θ12) R12y = −104.935 ⋅ mm
R14Ex := R14E⋅ cos( θ14E) R14Ex = 0 ⋅ mm R14Ey := R14E⋅ sin( θ14E) R14Ey = 57.000⋅ mm
R14Fx := R14F ⋅ cos( θ14F ) R14Fx = −0.000⋅ mm R14Fy := R14F ⋅ sin( θ14F ) R14Fy = −62.900⋅ mm
R23x := R23⋅ cos( θ23) R23x = −23.616⋅ mm R23y := R23⋅ sin( θ23) R23y = −84.357⋅ mm
R32x := R32⋅ cos( θ32) R32x = 11.566⋅ mm R32y := R32⋅ sin( θ32) R32y = 41.312⋅ mm
R34x := R34⋅ cos( θ34) R34x = 0.000⋅ mm R34y := R34⋅ sin( θ34) R34y = 15.000⋅ mm
R43x := R43⋅ cos( θ43) R43x = 23.616⋅ mm R43y := R43⋅ sin( θ43) R43y = 84.357⋅ mm
RPx := RP⋅ cos( θP) RPx = −0.000⋅ mm RPy := RP⋅ sin( θP) RPy = −105.000 ⋅ mm
Rinx := Rin⋅ cos( θin) Rinx = −107.904 ⋅ mm Riny := Rin⋅ sin( θin) Riny = −107.904 ⋅ mm

3. Write equations 3(b) for link 4, the compacting ram.

Σ Fx: F14E + F14F + F34x = 0 (1)

Σ Fy: Fcom + F34y = 0 (2)

Σ Mz: (−R14Ey⋅ F14E) + (−R14Fy⋅ F14F ) + (R34x⋅ F34y − R34y⋅ F34x) = 0 (3)

4. The direction (but not the sense) of F34 is known so write the equation that relates the x- and y-components of
this force.

F34y − F34x⋅ tan( θ3) = 0 (4)

5. There are four unknowns in the four equations above. Solving for F14x, F14y, F34x, and F34y,

1 1 1 0
0 F14E
0 0 0 1
Fcom
A := −R14Ey −R14Fy −R34y R34x − F14F −1
B := N := A ⋅ B⋅ N
mm mm mm mm F34x
0
0 0 −tan( θ3) 1 F34y
0

F14E = 18.2⋅ N F14F = 9.8⋅ N F34x = −28.0⋅ N F34y = −100.0⋅ N

6. From Newton's thrid law and, since the connecting rod (3) is a two-force member

F43x := −F34x F43x = 28.0⋅ N F43y := −F34y F43y = 100.0⋅ N

F23x := −F43x F23x = −28.0⋅ N F23y := −F43y F23y = −100.0⋅ N

7. Write equations 3(b) for link 2, the input arm.


Σ Fx: F12x + F32x + Finx = 0 (5)

Σ Fy: F12y + F32y + Finy = 0 (6)

Σ Mz: (R12x⋅ F12y − R12y⋅ F12x) + (R32x⋅ F32y − R32y⋅ F32x) + (Rinx⋅ Finy − Riny⋅ Finx) = 0 (7)

© 2011 Pearson Education, Inc., Upper Saddle River, NJ. All rights reserved. This publication is protected by Copyright and written permission should be
MACHINE DESIGN - An Integrated Approach, 4th Ed. 3-51-3

8. The direction (but not the sense) of Fin is known so write the equation that relates the x- and
y-components of this force.

Finy − Finx⋅ tan( θin) = 0 (8)

9. There are four unknowns in the four equations above. Solving for F12x, F12y, Finx, and Finy, since

F32x := −F23x F32x = 28⋅ N F32y := −F23y F32y = 100⋅ N

−F32x
1 0 1 0
N F12x
0 1 0 1
−F32y
A := −R12y R12x −Riny Rinx F12y −1
B := N := A ⋅ B⋅ N
mm mm mm mm Finx
−( R32x⋅ F32y − R32y⋅ F32x)
0 0 −tan( θin) 1 Finy
N ⋅ mm
0

F12x = 36.0⋅ N F12y = −36.0⋅ N Finx = −64.0⋅ N Finx = −64.0⋅ N

2 2 2 2
F12 := F12x + F12y Fin := Finx + Finy

F12 = 51⋅ N Fin = 91⋅ N

© 2011 Pearson Education, Inc., Upper Saddle River, NJ. All rights reserved. This publication is protected by Copyright and written permission should be
MACHINE DESIGN - An Integrated Approach, 4th Ed. 3-52-1

PROBLEM 3-52
Statement: Figure P3-23 shows a drag link slider crank mechanism. For the position shown, draw free-body
diagrams of links 2 through 6 using variable names similar to those used in Case Studies 1A and
2A. Assume that the crank turns slowly enough that accelerations can be ignored. Ignore the
weights of the links and any friction forces or torques.

Assumptions: 1. A two-dimensional model is adequate.


2. Inertia forces may be ignored.
3. Links 4 and 6 are three-force bodies.

Solution: See Figure P3-23 and Mathcad file P0352.


1. Isolate each of the elements to be analyzed, y
starting with the slider, link 6, since the external
forces on it are known. Place the known force, F56
FP, at the point P. This is a three-force member θ5 P
so the forces are coincident at point D and there x FP
is no turning moiment on the link. The angle, D
θ5,that link 5 makes with the horizontal axis is
known.
2. Link 5 is a two-force member with the forces F16
acting at the interfaces C and D along the line
joining points C and D. The assumption made in Slider block 6
step 1 is that these are compressive forces on link
5.

F45 C
y

x
θ5
R45
D

R65 F65

Link 5

3. Link 4 is a three-force body with the three forces meeting at a point. The position vectors R 14, R34, and
R54 will be known as will the angles, 3 and 5,that links 3 and 5, respectively, make with the horizontal
axis.

© 2011 Pearson Education, Inc., Upper Saddle River, NJ. All rights reserved. This publication is protected by Copyright and written permission should be
MACHINE DESIGN - An Integrated Approach, 4th Ed. 3-52-2

C
y
R34 F54
E

x R54
F14y
B
O4
F14x
F34 R14

F43
Link 4
y

B R43

4. Link 3 is a two-force member with the forces acting at the


interfaces A and B along the line joining points A and B.
x

R23
5. The crank is acted on by forces at A and O2, and a torque A
which we will assume to be positive (CCW). As in step 1,
assume that the unknown reaction force at O2 is positive.
F23

Link 3
F
12y

y
T
R32 F12x
O2
x
R12
A

F32

Link 2

© 2011 Pearson Education, Inc., Upper Saddle River, NJ. All rights reserved. This publication is protected by Copyright and written permission should be
MACHINE DESIGN - An Integrated Approach, 4th Ed. 3-53-1

PROBLEM 3-53
Statement: For the drag link slider crank mechanism of Problem 3-52 and the data of Table P3-6, determine the
pin forces on the slider, connecting rods, and crank and the reaction torque on the crank.
Given: R12 := 63.5⋅ mm θ12 := 45.38⋅ deg R14 := 93.6⋅ mm θ14 := −55.89⋅ deg
R23 := 63.5⋅ mm θ23 := 267.8⋅ deg R32 := 63.5⋅ mm θ32 := 225.38⋅ deg
R34 := 103.5⋅ mm θ34 := 202.68⋅ deg
R43 := 63.5⋅ mm θ43 := 87.80⋅ deg R45 := 190.5⋅ mm θ45 := 156.65⋅ deg
R54 := 103.5⋅ mm θ54 := 45.34⋅ deg R65 := 190.5⋅ mm θ65 := −23.35⋅ deg
FP := 85⋅ N θ5 := 156.65deg θ3 := 87.80⋅ deg

Solution: See Mathcad files P0352 and P0353.


1. Draw free-body diagrams of each element (see
Problem 3-52). y
F56
θ5 P
Slider block 6 FP
D x

F16

F45 C
y

x
θ5
R45
D

Link 5 F65
R65

2. Calculate the x- and y-components of the position vectors.


R12x := R12⋅ cos( θ12) R12x = 44.602⋅ mm R12y := R12⋅ sin( θ12) R12y = 45.198⋅ mm

R14x := R14⋅ cos( θ14) R14x = 52.489⋅ mm R14y := R14⋅ sin( θ14) R14y = −77.497⋅ mm

R23x := R23⋅ cos( θ23) R23x = −2.438⋅ mm R23y := R23⋅ sin( θ23) R23y = −63.453⋅ mm

R32x := R32⋅ cos( θ32) R32x = −44.602⋅ mm R32y := R32⋅ sin( θ32) R32y = −45.198⋅ mm

R34x := R34⋅ cos( θ34) R34x = −95.497⋅ mm R34y := R34⋅ sin( θ34) R34y = −39.908⋅ mm

R43x := R43⋅ cos( θ43) R43x = 2.438⋅ mm R43y := R43⋅ sin( θ43) R43y = 63.453⋅ mm
© 2011 Pearson Education, Inc., Upper Saddle River, NJ. All rights reserved. This publication is protected by Copyright and written permission should be
MACHINE DESIGN - An Integrated Approach, 4th Ed. 3-53-2
R45x := R45⋅ cos( θ45) R45x = −174.898 ⋅ mm R45y := R45⋅ sin( θ45) R45y = 75.504⋅ mm

R54x := R54⋅ cos( θ54) R54x = 72.75⋅ mm R54y := R54⋅ sin( θ54) R54y = 73.619⋅ mm

R65x := R65⋅ cos( θ65) R65x = 174.898 ⋅ mm R65y := R65⋅ sin( θ65) R65y = −75.504⋅ mm

C
F43
y
y
R34 F54
E
B R43
x R54
F14y
B
x
O4
F14x
R23 F34 R14
A

Link 4
F23

Link 3 F
12y

y
T
R32 F12x
O2
x
R12
A

F32

Link 2
3. Write equations 3(b) for link 5, the slider.

Σ Fx: F56x − FP = 0 (1)

Σ Fy: F16 + F56y = 0 (2)

4. The direction (but not the sense) of F56 is known so write the equation that relates the x- and y-components of
this force.

F56y − F56x⋅ tan( θ5) = 0 (3)

5. There are three unknowns in the three equations above. Solving for F56x, F56y, and F16,
© 2011 Pearson Education, Inc., Upper Saddle River, NJ. All rights reserved. This publication is protected by Copyright and written permission should be
MACHINE DESIGN - An Integrated Approach, 4th Ed. 3-53-3

1 0 0 FP
F56x
A := 0 1 1 N −1
B := F56y := A ⋅ B⋅ N
−tan( θ5) 1 0 0
F16
0

F56x = 85.0⋅ N F56y = −36.7⋅ N F16 = 36.7⋅ N

6. From Newton's thrid law and, since the connecting rod (5) is a two-force member

F65x := −F56x F65x = −85⋅ N F65y := −F56y F65y = 36.7⋅ N

F45x := −F65x F45x = 85⋅ N F45y := −F65y F45y = −36.7⋅ N


and, for link 4
F54x := −F45x F54x = −85⋅ N F54y := −F45y F54y = 36.7⋅ N

7. Write equations 3(b) for link 4, the rocker.


Σ Fx: F34x + F54x + F14x = 0 (4)

Σ Fy: F34y + F54y + F14y = 0 (5)

Σ Mz: (R14x⋅ F14y − R14y⋅ F14x) + (R34x⋅ F34y − R34y⋅ F34x) + (R54x⋅ F54y − R54y⋅ F54x) = 0
8. The direction (but not the sense) of F34 is known so write the equation that relates the x- and y-components of
this force.

F34y − F34x⋅ tan( θ3) = 0 (7)

9. There are four unknowns in the four equations above. Solving for F34x, F34y, F14x, and F14y,

−F54x
1 0 1 0 N F34x
0 1 0 1 −F54y
F34y −1
A := −R34y R34x −R14y R14x B := N := A ⋅ B⋅ N
F14x
mm mm mm mm −( R54x⋅ F54y − R54y⋅ F54x)
F14y
−tan( θ3) 1 0 0 N ⋅ mm
0

F34x = 3.5⋅ N F34y = 90.9⋅ N F14x = 81.5⋅ N F14y = −127.6⋅ N

10. From Newton's thrid law and, since the connecting rod (3) is a two-force member

F43x := −F34x F43x = −3.5⋅ N F43y := −F34y F43y = −90.9⋅ N

F23x := −F43x F23x = 3 ⋅ N F23y := −F43y F23y = 90.9⋅ N


and, for link 2
F32x := −F23x F32x = −3.5⋅ N F32y := −F23y F32y = −90.9⋅ N

© 2011 Pearson Education, Inc., Upper Saddle River, NJ. All rights reserved. This publication is protected by Copyright and written permission should be
MACHINE DESIGN - An Integrated Approach, 4th Ed. 3-53-4
11. Write equations 3(b) for link 2, the crank.
Σ Fx: F12x + F32x = 0 (8)

Σ Fy: F12y + F32y = 0 (9)

Σ Mz: T2 + ( R12x⋅ F12y − R12y⋅ F12x) + ( R32x⋅ F32y − R32y⋅ F32x) = 0 (10)

12. There are three unknowns in the three equations above. Solving for F12x, F12y, and T2

−F32x
1 0 0 N
F12x
0 1 0 −F32y −1
A := B := F12y := A ⋅B
−R12y R12x N
1 T2
mm mm −( R32x⋅ F32y − R32y⋅ F32x)
N ⋅ mm

F12x = 3.5 N F12y = 90.9 N T2 = −7796 N*mm

© 2011 Pearson Education, Inc., Upper Saddle River, NJ. All rights reserved. This publication is protected by Copyright and written permission should be
MACHINE DESIGN - An Integrated Approach, 4th Ed. 4-1a-1
PROBLEM 4-1a
Statement: A differential stress element has a set of applied stresses on it as indicated in each row of Table
P4-1. For row a, draw the stress element showing the applied stresses, find the principal stresses
and maximum shear stress using Mohr's circle diagram, and draw the rotated stress element
showing the principal stresses.
Given: σx := 1000 σy := 0 σz := 0
τxy := 500 τyz := 0 τzx := 0
Solution: See Figure 4-1a and Mathcad file P0401a.
500
y
1. Draw the stress element, indicating the x and y axes.
2. Draw the Mohr's circle axes, indicating the τ and σ x 1000
axes with CW up and CCW down.
3. Plot the positive x-face point, which is (+1000, -500),
and label it with an "x."
FIGURE 4-1aA
4. Plot the positive y-face point, which is (0, +500), and Stress Element for Problem 4-1a
label it with a "y."
5. Draw a straight line from point x to point y. Using the point where this line intersects the σ-axis as the center of
the Mohr circle, draw a circle that goes through points x and y.

σx + σy
6. The center of the circle will be at σc := σc = 500
2

7. The circle will intersect the σ-axis at two of the principal stresses. In this case, we see that one is positive and
the other is negative so they will be σ1 and σ3. The third principal stress is σ2 = 0.

2
σx − σy 2
8. Calculate the radius of the circle R := + τxy R = 707.1
2

τ CW τ CW

τ1-3
τ 1-2
500 y 500

τ2-3
-500 500 1000 1500 -500 500 1000 1500
σ3 σ σ3 σ
0 σ1 0 σ2 σ1

500 x 500

τ CCW τ CCW

FIGURE 4-1aB
2D and 3D Mohr's Circle Diagrams for Problem 4-1a

© 2011 Pearson Education, Inc., Upper Saddle River, NJ. All rights reserved. This publication is protected by Copyright and written permission should be
MACHINE DESIGN - An Integrated Approach, 4th Ed. 4-1a-2

9. Calculate the principal stresses σ1 := σc + R σ1 = 1207 σ2 := 0

σ3 := σc − R σ3 = −207

10. Draw the three Mohr's circles to represent the complete 3D stress state.

11. Calculate the principal shear stresses y

τ12 := 0.5⋅ ( σ1 − σ2) τ12 = 603.6 207

τ23 := 0.5⋅ ( σ2 − σ3) τ23 = 103.6 1207

τ13 := 0.5⋅ ( σ1 − σ3) τ13 = 707.1 22.5°

The maximum principal stress is always τ13. x

12. Determine the orientation of the principal normal


stress (σ1) with respect to the x-axis. From the 2D
Mohr's circle diagram, we see that the angle 2φ from x
to σ1 is CCW and is given by
FIGURE 4-1aC
1 σx − σc Rotated Stress Element for Problem 4-1a
ϕ := ⋅ acos ϕ = 22.5 deg
2 R

13. Draw the rotated 2D stress element showing the two nonzero principal stresses.

© 2011 Pearson Education, Inc., Upper Saddle River, NJ. All rights reserved. This publication is protected by Copyright and written permission should be
MACHINE DESIGN - An Integrated Approach, 4th Ed. 4-1h-1
PROBLEM 4-1h
Statement: A differential stress element has a set of applied stresses on it as indicated in each row of Table
P4-1. For row h, draw the stress element showing the applied stresses, find the principal stresses
and maximum shear stress and draw the Mohr's circle diagram.

Given: σx := 750 σy := 500 σz := 250


τxy := 500 τyz := 0 τzx := 0
Solution: See Figures 4-1h and Mathcad file P0401h. z

1. Calculate the coefficients (stress invariants) of equation (4.4c). 250

3
C2 := σx + σy + σz C2 = 1.500 × 10

σx τxy σx τzx σy τyz


C1 := + +
τxy σy τzx σz τyz σz
750 500
500 500
5
C1 = 4.375 × 10 x y

σx τxy τzx
7
C0 := τxy σy τyz C0 = 3.125 × 10 FIGURE 4-1hA
Stress Element for Problem 4-1h
τzx τyz σz
3 2
2. Find the roots of the triaxial stress equation: σ − C2⋅ σ + C 1⋅ σ − C0 = 0
−C0
110
C1
v := r := polyroots ( v) r= 250
−C2
1140
1 τ CW

3. Extract the principal stresses from


the vector r by inspection.

σ1 := r σ1 = 1140 τ1-3
3
500 τ 1-2
σ2 := r σ2 = 250
2

σ3 := r σ3 = 110
1 τ2-3
-500 500 1000 1500
σ
0 σ3 σ2 σ1
4. Using equations (4.5), evaluate
the principal shear stresses.
σ1 − σ3
τ13 := τ13 = 515
2 500
σ1 − σ2
τ12 := τ12 = 445
2
σ2 − σ3
τ23 := τ23 = 70 τ CCW
2
FIGURE 4-1hB
5. Draw the three-circle Mohr diagram. The Three Mohr's Circles for Problem 4-1h
© 2011 Pearson Education, Inc., Upper Saddle River, NJ. All rights reserved. This publication is protected by Copyright and written permission should be
MACHINE DESIGN - An Integrated Approach, 4th Ed. 4-2-1
PROBLEM 4-2
Statement: A 400-lb chandelier is to be hung from two 10-ft-long solid steel cables in tension. Choose a
suitable diameter for the cable such that the stress will not exceed 5000 psi. What will be the
deflection of the cables? State all assumptions.

Given: Weight of chandelier W := 400⋅ lbf


Length of cable L := 10⋅ ft L = 120 in
Allowable stress σallow := 5000⋅ psi
Number of cables N := 2
6
Young's modulus E := 30⋅ 10 ⋅ psi

Assumptions: The cables share the load equally.


Solution: See Mathcad file P0402.
W
1. Determine the load on each cable P := P = 200 lbf
N

2. The stress in each cable will be equal to the load on the cable divided by its cross-sectional area. Using
equation (4.7), and setting the stress equal to the allowable stress, we have

4⋅ P
σallow =
2
π⋅ d

3. Solve this equation for the unknown cable diameter.

4⋅ P
d := d = 0.226 in
π⋅ σallow

4. Round this up to the next higher decimal equivalent of a common fractional size: d := 0.250⋅ in
5. Using equation (4.8), determine the deflection in each cable.
2
π⋅ d 2
Cross-section area A := A = 0.049 in
4

P⋅ L
Cable deflection ∆s := ∆s = 0.016 in
A ⋅E

© 2011 Pearson Education, Inc., Upper Saddle River, NJ. All rights reserved. This publication is protected by Copyright and written permission should be
MACHINE DESIGN - An Integrated Approach, 4th Ed. 4-3-1
PROBLEM 4-3
Statement: For the bicycle pedal-arm assembly in Figure 4-1 with rider-applied force of 1500 N at the pedal,
determine the maximum principal stress in the pedal arm if its cross-section is 15 mm in dia. The
pedal attaches to the pedal arm with a 12-mm screw thread. What is the stress in the pedal screw?

Given: Distances (see figure) a  170  mm b  60 mm


Rider-applied force Frider  1.5 kN
Pedal arm diameter d pa  15 mm
Screw thread diameter d sc  12 mm
z

Solution: See Figure 4-3 and Mathcad file P0403.


a
C Tc
1. From the FBD in Figure 4-3A (and on
Frider
the solution for Problem 3-3), we see that
the force from the rider is reacted in the Mc
b Arm
pedal arm internally by a moment, a
torque, and a vertical shear force. There
are two points at section C (Figure 4-3B) Fc y
that we should investigate, one at z = 0.5 Pedal
d pa (point A), and one at y = 0.5 d pa (point
B). x

2. Refering to the FBD resulting from FIGURE 4-3A


taking a section through the arm at C, the Free Body Diagram for Problem 4-3
maximum bending moment Mc is found by
summing moments about the y-axis, and
the maximum torque Tc is found by
z
summing moments about the x-axis.
 M y: Frider a  Mc = 0

 M x: Frider b  Tc = 0
Section C
A
Maximum bending moment:
Mc  Frider a Mc  255  N  m B
Arm

Maximum torque:
x
Tc  Frider b Tc  90 N  m
y

Vertical shear:
FIGURE 4-3B
Fc  Frider Fc  1.500  kN Points A and B at Section C

3. Determine the stress components at point A where we have the effects of bending and torsion, but where the
transverse shear due to bending is zero because A is at the outer fiber. Looking down the z-axis at a stress
element on the surface at A,

Distance to neutral axis cpa  0.5 d pa cpa  7.5 mm

© 2011 Pearson Education, Inc., Upper Saddle River, NJ. All rights reserved. This publication is protected by Copyright and written permission should be
MACHINE DESIGN - An Integrated Approach, 4th Ed. 4-3-2

4
Moment of inertia of π d pa 3 4
pedal-arm Ipa  Ipa  2.485  10  mm
64

Mc cpa
Bending stress σx  σx  769.6  MPa
(x-direction) Ipa

Stress in y-direction σy  0  MPa

Torsional stress Tc cpa


due to Tc τxy  τxy  135.8  MPa CW
2  Ipa

2
Principal stresses at A, σx  σy  σx  σy  2
equation (4.6a) σ1A      τxy
2  2 

2
σx  σy  σx  σy  2
σ3A      τxy
2  2 

σ1A  793  MPa σ2A  0  MPa σ3A  23 MPa

4. Determine the stress components at point B where we have the effects of transverse shear and torsion, but
where the bending stress is zero because B is on the neutral plane. Looking down the y-axis at a stress
element at B,

2
Cross-section area π d pa 2
of pedal-arm Apa  Apa  176.7  mm
4

Torsional stress
4 Fc
due to Tc and shear τzx    τxy τzx  124.5  MPa CW
stress due to Fc 3 Apa

Normal stresses σx  0  MPa σz  0  MPa

2
σx  σz  σx  σz  2
Principal stresses at B σ1B      τzx
2  2 

2
σx  σz  σx  σz  2
σ3B      τzx
2  2 

σ1B  124  MPa σ2B  0  MPa σ3B  124  MPa

5. The maximum principal stress is at point A and is σ1A  793  MPa

© 2011 Pearson Education, Inc., Upper Saddle River, NJ. All rights reserved. This publication is protected by Copyright and written permission should be
MACHINE DESIGN - An Integrated Approach, 4th Ed. 4-3-3

6. Determine the stress in the pedal screw.

Bending moment Msc  Frider b Msc  90 N  m

Distance to neutral axis csc  0.5 d sc csc  6  mm

4
Moment of inertia of π d sc 3 4
pedal screw Isc  Isc  1.018  10  mm
64

Msc csc
Bending stress σy  σy  530.5  MPa
(y-direction) Isc

Stress in z-direction σz  0  MPa

Torsional stress τxy  0  MPa

Since there is no shear stress present at the top of the screw where the bending stress is a maximum, the
maximum principal stress in the pedal screw is

σ1  σy σ1  530.5  MPa

© 2011 Pearson Education, Inc., Upper Saddle River, NJ. All rights reserved. This publication is protected by Copyright and written permission should be
MACHINE DESIGN - An Integrated Approach, 4th Ed. 4-4-1
PROBLEM 4-4
Statement: The trailer hitch shown in Figure P4-2 and Figure 1-1 (p. 12) has loads applied as defined in
Problem 3-4. The tongue weight of 100 kg acts downward and the pull force of 4905 N acts
horizontally. Using the dimensions of the ball bracket shown in Figure 1-5 (p. 15), determine:
(a) The principal stresses in the shank of the ball where it joins the ball bracket.
(b) The bearing stress in the ball bracket hole.
(c) The tearout stress in the ball bracket.
(d) The normal and shear stresses in the 19-mm diameter attachment holes.
(e) The principal stresses in the ball bracket as a cantilever.
Given: a  40 mm b  31 mm c  70 mm d  20 mm
Mtongue  100  kg Fpull  4.905  kN d sh  26 mm t  19 mm

Assumptions: 1. The nuts are just snug-tight (no pre-load), which is the worst case.
2. All reactions will be concentrated loads rather than distributed loads or pressures.
Solution: See Figure 4-4 and Mathcad file P0404.

1. The weight on the tongue is Wtongue  Mtongue g Wtongue  0.981  kN

2. Solving first for the reactions on the ball by summing the horizontal and vertical forces and the moments
about A.
W tongue

70 = c

1 F pull 1

40 = a

2 A A F a1x
B 19 = t

F b1 B
31 = b
C F a1y
20 = d F a2y

D
Fa2x

F b2
Fc2x
C

D
Fd2

F c2y

FIGURE 4-4A
Dimensions and Free Body Diagram for Problem 4-4

 Fx : Fpull  Fa1x  Fb1 = 0 (1)

 Fy : Fa1y  Wtongue = 0 (2)


© 2011 Pearson Education, Inc., Upper Saddle River, NJ. All rights reserved. This publication is protected by Copyright and written permission should be
MACHINE DESIGN - An Integrated Approach, 4th Ed. 4-4-2

 MA: Fb1 t  Fpull  a = 0 (3)

Fpull  a
3. Solving equation (3) for Fb1 Fb1  Fb1  10.326 kN
t

4. Substituting into (1) and solving for Fa1x Fa1x  Fpull  Fb1 Fa1x  15.231 kN

5. Solving (2) for Fa1y Fa1y  Wtongue Fa1y  0.981  kN

6. Now, refering to the FBD of the bracket, we can apply the equations of equilibrium to determine the reactions
at C and D on the bracket.
 Fx : Fa2x  Fb2  Fc2x  Fd2 = 0 (4)

 Fy : Fc2y  Fa2y = 0 (5)

 MC: Fd2 d  Fb2 b  Fa2x ( b  t)  Fa2y c = 0 (6)

7. Note also that the interface forces between part 1 (ball) and part 2 (bracket) have been drawn on their
respective FBDs in opposite senses. Therefore,

Fa2x  Fa1x Fa2y  Fa1y Fb2  Fb1

Fa2x ( b  t)  Fa2y c  Fb2 b


8. Solving equation (6) for Fd2 Fd2  Fd2  25.505 kN
d

9. Substituting into (4) and solving for Fc2x Fc2x  Fa2x  Fb2  Fd2 Fc2x  30.41  kN

10. Solving (5) for Fa1y Fc2y  Fa2y Fc2y  0.981  kN

11. Determine the principal stresses in the shank of the ball where it joins the ball bracket.

The internal bending moment at A on the FBD of the ball is


M  Fpull  a M  196.2  N  m

Distance to neutral axis csh  0.5 d sh csh  13 mm

π  d sh
4
4 4
Moment of inertia of shank Ish  Ish  2.243  10  mm
64
M  csh
Bending stress (x-direction) σx  σx  113.7  MPa
Ish

Stress in y-direction σy  0  MPa

Shear stress at A τxy  0  MPa

Since the shear stress is zero, x is the maximum principal stress, thus
σ1  σx σ1  114  MPa σ2  0  MPa σ3  0  MPa

© 2011 Pearson Education, Inc., Upper Saddle River, NJ. All rights reserved. This publication is protected by Copyright and written permission should be
MACHINE DESIGN - An Integrated Approach, 4th Ed. 4-4-3

12. Determine the bearing stress in the ball bracket hole.


2
Bearing area Abearing  d sh t Abearing  494  mm

Fpull
Bearing stress σbearing  σbearing  9.93 MPa
Abearing

13. Determine the tearout stress in the ball bracket.


Tearout length
Shear area (see Figure 4-4B)

2 2
Atear = 2  t R  ( 0.5 d )

Atear  2  t ( 32 mm)   0.5 d sh


2 2

2
Atear  1111 mm

Stress
d R
Fpull
τtear 
Atear FIGURE 4-4B
Tearout Diagram for Problem 4-4
τtear  4.41 MPa

14. Determine the normal and shear stresses in the attachment bolts if they are 19-mm dia. d bolt  19 mm

Bolt cross-section area (2 bolts)

2
π d bolt 2
Abolt  2  Abolt  567.1  mm
4
Fc2x
Normal stress (tension) σbolt  σbolt  53.6 MPa
Abolt

© 2011 Pearson Education, Inc., Upper Saddle River, NJ. All rights reserved. This publication is protected by Copyright and written permission should be
MACHINE DESIGN - An Integrated Approach, 4th Ed. 4-4-4
Shear stress W tongue

Fc2y
τbolt 
Abolt τbolt  1.7 MPa
F pull 1

15. Determine the principal stresses in the ball bracket as a


cantilever (see Figure 4-4C). a

Bending moment
2
M  Fpull  a  Wtongue c M  264.8  N  m

M
Width of bracket w  64 mm c

3 R
w t 4
Moment of inertia I  I  36581  mm
12 FIGURE 4-4C
Cantilever FBD for Problem 4-4
M t Fpull
Total tensile stress σ  
2 I w t σ  72.8 MPa

Since there are no shear stress at the top and bottom of the bracket where the bending stresses are maximum,
they are also the principal stresses, thus

σ1  σ σ1  72.8 MPa σ2  0  MPa σ3  0  MPa


σ
τmax  τmax  36.4 MPa
2

© 2011 Pearson Education, Inc., Upper Saddle River, NJ. All rights reserved. This publication is protected by Copyright and written permission should be
MACHINE DESIGN - An Integrated Approach, 4th Ed. 4-5-1
PROBLEM 4-5
Statement: Repeat Problem 4-4 for the loading conditions of Problem 3-5, i.e., determine the stresses due to a
horizontal force that will result on the ball from accelerating a 2000-kg trailer to 60 m/sec in 20 se
Assume a constant acceleration. From Problem 3-5, the pull force is 6000 N. Determine:
(a) The principal stresses in the shank of the ball where it joins the ball bracket.
(b) The bearing stress in the ball bracket hole.
(c) The tearout stress in the ball bracket.
(d) The normal and shear stresses in the 19-mm diameter attachment holes.
(e) The principal stresses in the ball bracket as a cantilever.

Given: a  40 mm b  31 mm c  70 mm d  20 mm


Mtongue  100  kg Fpull  6  kN d sh  26 mm t  19 mm

Assumptions: 1. The nuts are just snug-tight (no pre-load), which is the worst case.
2. All reactions will be concentrated loads rather than distributed loads or pressures.
Solution: See Figure 4-5 and Mathcad file P0405.

1. The weight on the tongue is Wtongue  Mtongue g Wtongue  0.981  kN

2. Solving first for the reactions on the ball by summing the horizontal and vertical forces and the moments about
A.
W tongue

70 = c

1 F pull 1

40 = a

2 A A F a1x
B 19 = t

F b1 B
31 = b
C F a1y
20 = d F a2y

D
Fa2x

F b2
Fc2x
C

D
Fd2

F c2y

FIGURE 4-5A
Dimensions and Free Body Diagram for Problem 4-5

 Fx : Fpull  Fa1x  Fb1 = 0 (1)

 Fy : Fa1y  Wtongue = 0 (2)


© 2011 Pearson Education, Inc., Upper Saddle River, NJ. All rights reserved. This publication is protected by Copyright and written permission should be
MACHINE DESIGN - An Integrated Approach, 4th Ed. 4-5-2

 MA: Fb1 t  Fpull  a = 0 (3)

Fpull  a
3. Solving equation (3) for Fb1 Fb1  Fb1  12.632 kN
t

4. Substituting into (1) and solving for Fa1x Fa1x  Fpull  Fb1 Fa1x  18.632 kN

5. Solving (2) for Fa1y Fa1y  Wtongue Fa1y  0.981  kN

6. Now, refering to the FBD of the bracket, we can apply the equations of equilibrium to determine the reactions
at C and D on the bracket.
 Fx : Fa2x  Fb2  Fc2x  Fd2 = 0 (4)

 Fy : Fc2y  Fa2y = 0 (5)

 MC: Fd2 d  Fb2 b  Fa2x ( b  t)  Fa2y c = 0 (6)

7. Note also that the interface forces between part 1 (ball) and part 2 (bracket) have been drawn on their respective
FBDs in opposite senses. Therefore,

Fa2x  Fa1x Fa2y  Fa1y Fb2  Fb1

Fa2x ( b  t)  Fa2y c  Fb2 b


8. Solving equation (6) for Fd2 Fd2  Fd2  30.432 kN
d

9. Substituting into (4) and solving for Fc2x Fc2x  Fa2x  Fb2  Fd2 Fc2x  36.432 kN

10. Solving (5) for Fa1y Fc2y  Fa2y Fc2y  0.981  kN

11. Determine the principal stresses in the shank of the ball where it joins the ball bracket.

The internal bending moment at A on the FBD of the ball is


M  Fpull  a M  240  N  m

Distance to neutral axis csh  0.5 d sh csh  13 mm

π  d sh
4
4 4
Moment of inertia of shank Ish  Ish  2.243  10  mm
64
M  csh
Bending stress (x-direction) σx  σx  139.1  MPa
Ish

Stress in y-direction σy  0  MPa

Shear stress at A τxy  0  MPa

Since the shear stress is zero, x is the maximum principal stress, thus

σ1  σx σ1  139  MPa σ2  0  MPa σ3  0  MPa

12. Determine the bearing stress in the ball bracket hole.


© 2011 Pearson Education, Inc., Upper Saddle River, NJ. All rights reserved. This publication is protected by Copyright and written permission should be
MACHINE DESIGN - An Integrated Approach, 4th Ed. 4-5-3

2
Bearing area Abearing  d sh t Abearing  494  mm

Fpull
Bearing stress σbearing  σbearing  12.15  MPa
Abearing

13. Determine the tearout stress in the ball bracket.


Tearout length
Shear area (see Figure 4-4B)

2 2
Atear = 2  t R  ( 0.5 d )

Atear  2  t ( 32 mm)   0.5 d sh


2 2

2
Atear  1111 mm

Stress
d R
Fpull
τtear 
Atear FIGURE 4-5B
τtear  5.4 MPa Tearout Diagram for Problem 4-5

14. Determine the normal and shear stresses in the attachment bolts if they are 19-mm dia. d bolt  19 mm

Bolt cross-section area (2 bolts)

2
π d bolt 2
Abolt  2  Abolt  567.1  mm
4
Fc2x
Normal stress (tension) σbolt  σbolt  64.2 MPa
Abolt

Shear stress W tongue

Fc2y
τbolt 
Abolt τbolt  1.7 MPa
F pull 1

15. Determine the principal stresses in the ball bracket as a


cantilever (see Figure 4-4C). a

Bending moment
2
M  Fpull  a  Wtongue c M  308.6  N  m

M
Width of bracket w  64 mm c

3 R
w t 4
Moment of inertia I  I  36581  mm
12 FIGURE 4-5C
Cantilever FBD for Problem 4-5

M t Fpull
Total tensile stress σ   σ  85.1 MPa
2 I w t
© 2011 Pearson Education, Inc., Upper Saddle River, NJ. All rights reserved. This publication is protected by Copyright and written permission should be
MACHINE DESIGN - An Integrated Approach, 4th Ed. 4-5-4

Since there are no shear stress at the top and bottom of the bracket where the bending stresses are maximum,
they are also the principal stresses, thus

σ1  σ σ1  85.1 MPa σ2  0  MPa σ3  0  MPa


σ
τmax  τmax  42.5 MPa
2

© 2011 Pearson Education, Inc., Upper Saddle River, NJ. All rights reserved. This publication is protected by Copyright and written permission should be
MACHINE DESIGN - An Integrated Approach, 4th Ed. 4-6-1
PROBLEM 4-6
Statement: Repeat Problem 4-4 for the loading conditions of Problem 3-6, i.e., determine the stresses due to a
horizontal force that will results from an impact between the ball and the tongue of the 2000-kg
trailer if the hitch deflects 2.8 mm dynamically on impact. The tractor weighs 1000 kg and the
velocity at impact is 0.3 m/sec. Determine:
(a) The principal stresses in the shank of the ball where it joins the ball bracket.
(b) The bearing stress in the ball bracket hole.
(c) The tearout stress in the ball bracket.
(d) The normal and shear stresses in the 19-mm diameter attachment holes.
(e) The principal stresses in the ball bracket as a cantilever.
Given: a  40 mm b  31 mm c  70 mm d  20 mm
Mtongue  100  kg Fpull  55.1 kN d sh  26 mm t  19 mm

Assumptions: 1. The nuts are just snug-tight (no pre-load), which is the worst case.
2. All reactions will be concentrated loads rather than distributed loads or pressures.
Solution: See Figure 4-6 and Mathcad file P0406.

1. The weight on the tongue is Wtongue  Mtongue g Wtongue  0.981  kN

2. Solving first for the reactions on the ball by summing the horizontal and vertical forces and the moments
about A.
W tongue

70 = c

1 F pull 1

40 = a

2 A A F a1x
B 19 = t

F b1 B
31 = b
C F a1y
20 = d F a2y

D
Fa2x

F b2
Fc2x
C

D
Fd2

F c2y

FIGURE 4-6A
Dimensions and Free Body Diagram for Problem 4-6

 Fx : Fpull  Fa1x  Fb1 = 0 (1)


© 2011 Pearson Education, Inc., Upper Saddle River, NJ. All rights reserved. This publication is protected by Copyright and written permission should be
MACHINE DESIGN - An Integrated Approach, 4th Ed. 4-6-2

 Fy : Fa1y  Wtongue = 0 (2)

 MA: Fb1 t  Fpull  a = 0 (3)

Fpull  a
3. Solving equation (3) for Fb1 Fb1  Fb1  116  kN
t

4. Substituting into (1) and solving for Fa1x Fa1x  Fpull  Fb1 Fa1x  171.1  kN

5. Solving (2) for Fa1y Fa1y  Wtongue Fa1y  0.981  kN

6. Now, refering to the FBD of the bracket, we can apply the equations of equilibrium to determine the reactions
at C and D on the bracket.
 Fx : Fa2x  Fb2  Fc2x  Fd2 = 0 (4)

 Fy : Fc2y  Fa2y = 0 (5)

 MC: Fd2 d  Fb2 b  Fa2x ( b  t)  Fa2y c = 0 (6)

7. Note also that the interface forces between part 1 (ball) and part 2 (bracket) have been drawn on their respective
FBDs in opposite senses. Therefore,

Fa2x  Fa1x Fa2y  Fa1y Fb2  Fb1

Fa2x ( b  t)  Fa2y c  Fb2 b


8. Solving equation (6) for Fd2 Fd2  Fd2  251.382  kN
d

9. Substituting into (4) and solving for Fc2x Fc2x  Fa2x  Fb2  Fd2 Fc2x  306.482  kN

10. Solving (5) for Fa1y Fc2y  Fa2y Fc2y  0.981  kN

11. Determine the principal stresses in the shank of the ball where it joins the ball bracket.

The internal bending moment at A on the FBD of the ball is


3
M  Fpull  a M  2.204  10  N  m

Distance to neutral axis csh  0.5 d sh csh  13 mm

π  d sh
4
4 4
Moment of inertia of shank Ish  Ish  2.243  10  mm
64
M  csh
Bending stress (x-direction) σx  σx  1277 MPa
Ish

Stress in y-direction σy  0  MPa

Shear stress at A τxy  0  MPa

Since the shear stress is zero, x is the maximum principal stress, thus

σ1  σx σ1  1277 MPa σ2  0  MPa σ3  0  MPa


© 2011 Pearson Education, Inc., Upper Saddle River, NJ. All rights reserved. This publication is protected by Copyright and written permission should be
MACHINE DESIGN - An Integrated Approach, 4th Ed. 4-6-3
12. Determine the bearing stress in the ball bracket hole.
2
Bearing area Abearing  d sh t Abearing  494  mm

Fpull
Bearing stress σbearing  σbearing  111.54 MPa
Abearing

13. Determine the tearout stress in the ball bracket.


Tearout length
Shear area (see Figure 4-4B)

2 2
Atear = 2  t R  ( 0.5 d )

Atear  2  t ( 32 mm)   0.5 d sh


2 2

2
Atear  1111 mm

Stress
d R
Fpull
τtear 
Atear FIGURE 4-6B
Tearout Diagram for Problem 4-6
τtear  49.59  MPa

14. Determine the normal and shear stresses in the attachment bolts if they are 19-mm dia. d bolt  19 mm
Bolt cross-section area (2 bolts)

2
π d bolt 2
Abolt  2  Abolt  567.1  mm
4
Fc2x
Normal stress (tension) σbolt  σbolt  540  MPa
Abolt

Shear stress W tongue

Fc2y
τbolt 
Abolt τbolt  1.7 MPa
F pull 1

15. Determine the principal stresses in the ball bracket as a


cantilever (see Figure 4-4C). a

Bending moment
2
3
M  Fpull  a  Wtongue c M  2.3  10  N  m

M
Width of bracket w  64 mm c

3 R
w t 4
Moment of inertia I  I  36581  mm
12 FIGURE 4-6C
Cantilever FBD for Problem 4-6
M t Fpull
Total tensile stress σ   σ  635.5  MPa
2 I w t
© 2011 Pearson Education, Inc., Upper Saddle River, NJ. All rights reserved. This publication is protected by Copyright and written permission should be
MACHINE DESIGN - An Integrated Approach, 4th Ed. 4-6-4

Since there are no shear stress at the top and bottom of the bracket where the bending stresses are maximum,
they are also the principal stresses, thus

σ1  σ σ1  636  MPa σ2  0  MPa σ3  0  MPa


σ
τmax  τmax  318  MPa
2

© 2011 Pearson Education, Inc., Upper Saddle River, NJ. All rights reserved. This publication is protected by Copyright and written permission should be
MACHINE DESIGN - An Integrated Approach, 4th Ed. 4-7-1
PROBLEM 4-7
Statement: Design the wrist pin of Problem 3-7 for a maximum allowable principal stress of 20 ksi if the pin is
hollow and loaded in double shear.

Given: Force on wrist pin Fwristpin  12.258 kN Fwristpin  2756 lbf


Allowable stress σallow  20 ksi
Assumptions: Choose a suitable outside diameter, say od  0.375  in

Solution: See Figure 4-12 in the text and Mathcad file P0407.

Fwristpin
1. The force at each shear plane is F  F  1378 lbf
2

2. With only the direct shear acting on the plane, the Mohr diagram will be a circle with center at the origin and
radius equal to the shear stress. Thus, the principal normal stress is numerically equal to the shear stress, which
in this case is also the principal shear stress, so we have  = 1 = allow.

F 4 F
3. The shear stress at each shear plane is τ= = = σallow
π  od  id 
A 2 2

2 4 F
4. Solving for the inside diameter, id  od  id  0.230  in
π σallow

© 2011 Pearson Education, Inc., Upper Saddle River, NJ. All rights reserved. This publication is protected by Copyright and written permission should be
MACHINE DESIGN - An Integrated Approach, 4th Ed. 4-8-1
PROBLEM 4-8
Statement: A paper mill processes rolls of paper having a density of 984 kg/m3. The paper roll is 1.50 m
outside dia (OD) by 0.22 m inside dia (ID) by 3.23 m long and is on a simply supported, hollow,
steel shaft. Find the shaft ID needed to obtain a maximum deflection at the center of 3 mm if the
shaft OD is 22 cm.

kg
Given: Paper density ρ  984 
3
m
Roll dimensions Shaft outside dia od  220  mm
Outside diameter OD  1.50 m Young's modulus E  207  GPa
Inside diameter ID  0.22 m Allowable deflection δ  3  mm
Lemgth L  3.23 m
Assumptions: The shaft (beam) supporting the paper roll is simply-supported at the ends and is the same
length as the paper roll. The paper acts as a distributed load over the length of the shaft.
Solution: See Mathcad file P0408.

1. The weight of the paper roll is equal to its volume times the paper density times g.

Wroll 
π
4
 2 2
 OD  ID  L ρ  g  Wroll  53.89  kN

Wroll N
2. The intensity of the distributed load is w  w  16.686
L mm

3. Using Figure B-2(b) in Appendix B with a = 0, the maximum deflection is at the midspan and is

y =
w x
24 E I
 2
 2  L x  x  L
3 3 
4
5  w L
For x = L/2, this reduces to y =
384  E I
4
5  w L 7 4
Letting  = -y and solving for I, we have I  I  3.808  10  mm
384  E δ

4. The area moment of inertia for a hollow circular cross-section is I =


π
 4
 od  id
4 
64

1
4
64 I 
id   od 
4
Solving this for the id yields  id  198.954  mm
 π 

Round this down (for slightly less deflection) to id  198  mm

© 2011 Pearson Education, Inc., Upper Saddle River, NJ. All rights reserved. This publication is protected by Copyright and written permission should be
MACHINE DESIGN - An Integrated Approach, 4th Ed. 4-9-1
PROBLEM 4-9
Statement: A ViseGrip plier-wrench is drawn to scale in Figure P4-3, and for which the forces were analyzed
in Problem 3-9, find the stresses in each pin for an assumed clamping force of P = 4000 N in the
position shown. The pins are 8-mm dia and are all in double shear.
Given: Pin forces as calculated in Problem 3-9:
Member 1 F21  7.5 kN F41  5.1 kN
Member 2 F12  7.5 kN F32  5.1 kN
Member 3 F23  5.1 kN F43  5.1 kN
Member 4 F14  5.1 kN F34  5.1 kN
Pin diameter d  8  mm
Assumptions: Links 3 and 4 are in a toggle position, i.e., the pin that joins links 3 and 4 is in line with the pins
that join 1 with 4 and 2 with 3.
Solution: See Figure 4-9 and Mathcad file P0409.

1. The FBDs of the assembly and each individual link are shown in Figure 4-9. The dimensions, as scaled from
Figure P4-3 in the text, are shown on the link FBDs.

F 4
P
1

3 2
P

55.0 = b 50.0 = a
F14
39.5 = c 22.0 = d
F

129.2° 1 
4

F41 F34
F21 P


28.0 = e


P 2.8 = g


F43

3 F12
21.2 = h
F23 2
F32

26.9 = f

FIGURE 4-9
Free Body Diagrams for Problem 4-9

2
π d 2
2. The cross-sectional area for all pins is the same and is A  A  50.265 mm
4
© 2011 Pearson Education, Inc., Upper Saddle River, NJ. All rights reserved. This publication is protected by Copyright and written permission should be
MACHINE DESIGN - An Integrated Approach, 4th Ed. 4-9-2

3. The pin that joins members 1 and 2 is the most highly stressed while the stress on each of the remaining pins
is the same. Since the pins are in double shear, we will divide the pin load by 2 in each case.

F12
Pin joining 1 and 2 τ12  τ12  74.6 MPa
2 A

F14
All other pins τ14  τ14  50.7 MPa
2 A

© 2011 Pearson Education, Inc., Upper Saddle River, NJ. All rights reserved. This publication is protected by Copyright and written permission should be
MACHINE DESIGN - An Integrated Approach, 4th Ed. 4-10-1
PROBLEM 4-10
Statement: The over-hung diving board of problem 3-10 is shown in Figure P4-4a. Assume cross-section
dimensions of 305 mm x 32 mm. The material has E = 10.3 GPa. Find the largest principal stress
at any location in the board when a 100-kg person is standing at the free end.

Given: Weight of person W  100  kgf 2000 = L


Board dimensions
R1 P
Distance to support a  0.7 m
Length of board L  2  m
R2
Cross-section w  305  mm
t  32 mm 700 = a

FIGURE 4-10
Assumptions: The weight of the beam is negligible Free Body Diagram for Problem 4-10
compared to the applied load and so can
be ignored.

Solution: See Figure 4-10 and Mathcad file P0410.

1. From the FBD of the diving board and Figure B-3 (Appendix B), the reactions at the supports are

R1  W   1 
L
 R1  1821 N
 a

R2  W  
L
 R2  2802 N
a

2. Also from Figure D-3, the maximum bending moment occurs at the right-hand support where, in the FBD
above, x = a.

Mmax  R1 a Mmax  1275 N  m

3. The maximum bending stress will occur on the top and bottom surfaces of the board at the section where the
maximum bending moment occurs which, in this case, is at x = a. The only stress present on the top or
bottom surface of the board is the bending stress x. Therefore, on the top surface where the stress is
tensile, x. is the principal stress 1 . Thus,

t
Distance to extreme fiber c  c  16 mm
2
3
w t 5 4
Moment of inertia I  I  8.329  10  mm
12
Mmax c
Bending stress σx  σx  24.492 MPa
I

Maximum principal stress σ1  σx σ1  24.5 MPa

© 2011 Pearson Education, Inc., Upper Saddle River, NJ. All rights reserved. This publication is protected by Copyright and written permission should be
MACHINE DESIGN - An Integrated Approach, 4th Ed. 4-11-1
PROBLEM 4-11
Statement: Repeat Problem 4-10 using the loading conditions of Problem 3-11. Assume the board weighs 29
kg and deflects 13.1 cm statically when the person stands on it. Find the largest principal stress
at any location in the board when the 100-kg person in Problem 4-10 jumps up 25 cm and lands
back on the board. Find the maximum deflection.

Given: Beam length L  2000 mm 2000 = L


Distance to support a  700  mm R1 Fi
Mass of person mpers  100  kg
Mass of board mboard  29 kg
R2
Static deflection δst  131  mm
Height of jump h  250  mm 700 = a
Cross-section w  305  mm
FIGURE 4-11
t  32 mm Free Body Diagram for Problem 4-11

Assumptions: The apparent Young's modulus for fiberglas is


4
E  1.03 10  MPa

Solution: See Figure 4-11 and Mathcad file P0411.


1. From Problem 3-11, the dynamic load resulting from the impact of the person with the board isFi  3.056  kN
2. From the FBD of the diving board and Figure B-3(a) (Appendix B), the reactions at the supports are

R1  Fi  1 
L
 R1  5.675  kN
 a

R2  Fi 
L
 R2  8.731  kN
a
3. Also from Figure D-3(a), the maximum bending moment occurs at the right-hand support where, in the
FBD above, x = a.

Mmax  R1 a Mmax  3.973  kN  m


4. The maximum bending stress will occur on the top and bottom surfaces of the board at the section where the
maximum bending moment occurs which, in this case, is at x = a. The only stress present on the top or bottom
surface of the board is the bending stress x. Therefore, on the top surface where the stress is tensile, x is the
principal stress 1 . Thus,
t
Distance to extreme fiber c  c  16 mm
2
3
w t 5 4
Moment of inertia I  I  8.329  10  mm
12
Mmax c
Bending stress σx  σx  76.322 MPa
I

Maximum principal stress σ1  σx σ1  76.3 MPa

5. Calculate the maximum deflection from the equation given in Figure D-3(a) at x = L. Let b in the figure be a in
our problem and let a in the figure be equal to L, then

 ( a  L)  L  L ( L  a )  a  ( L  a )  L
Fi 3 3 2
ymax  ymax  401.4  mm
6  a  E I
© 2011 Pearson Education, Inc., Upper Saddle River, NJ. All rights reserved. This publication is protected by Copyright and written permission should be
MACHINE DESIGN - An Integrated Approach, 4th Ed. 4-12-1
PROBLEM 4-12
Statement: Repeat Problem 4-10 using the cantilevered diving board design in Figure P4-4b.

Given: Beam length L  1300 mm 2000

Weight at free end P  100  kgf 1300 = L


Cross-section w  305  mm P
t  32 mm
Assumptions: The apparent Young's modulus for
M1
fiberglas is R1
4
E  1.03 10  MPa
700

Solution: See Figure 4-12 and Mathcad file P0412. FIGURE 4-12
Free Body Diagram for Problem 4-12

1. From the FBD of the diving board and Figure B-1(a) (Appendix B), the reactions at the supports are

R1  P R1  981  N

M1  P L M1  1275 N  m

2. Also from Figure D-1, the maximum bending moment occurs at the support where, in the FBD above, x = 0.
Mmax  M1 Mmax  1275 N  m

3. The maximum bending stress will occur on the top and bottom surfaces of the board at the section where the
maximum bending moment occurs which, in this case, is at x = 0. The only stress present on the top or bottom
surface of the board is the bending stress x. Therefore, on the top surface where the stress is tensile, x is the
principal stress 1 . Thus,

t
Distance to extreme fiber c  c  16 mm
2
3
w t 5 4
Moment of inertia I  I  8.329  10  mm
12
Mmax c
Bending stress σx  σx  24.492 MPa
I

Maximum principal stress σ1  σx σ1  24.5 MPa

© 2011 Pearson Education, Inc., Upper Saddle River, NJ. All rights reserved. This publication is protected by Copyright and written permission should be
MACHINE DESIGN - An Integrated Approach, 4th Ed. 4-13-1
PROBLEM 4-13
Statement: Repeat Problem 4-11 using the diving board design shown in Figure P4-4b. Assume the board
weighs 19 kg and deflects 8.5 cm statically when the person stands on it.

Given: Unsupported length L  1300 mm


2000
Mass of board mboard  19 kg
1300 = L
Static board deflection δstat  85 mm
Fi
Mass of person mperson  100  kg
Height of jump h  250  mm
M1
Cross-section w  305  mm R1

t  32 mm
700
Assumptions: The apparent Young's modulus for fiberglas is
4 FIGURE 4-13
E  1.03 10  MPa
Free Body Diagram for Problem 4-13
Solution: See Figure 4-13 and Mathcad file P0413.

1. From Problem 3-13, the dynamic load resulting from the impact of the person with the board isFi  3.487  kN

2. From the FBD of the diving board and Figure B-1(a) (Appendix B), the reactions at the supports are

R1  Fi R1  3487 N

M1  Fi L M1  4533 N  m

3. Also from Figure D-1, the maximum bending moment occurs at the support where, in the FBD above, x = 0.
Mmax  M1 Mmax  4533 N  m

4. The maximum bending stress will occur on the top and bottom surfaces of the board at the section where the
maximum bending moment occurs which, in this case, is at x = 0. The only stress present on the top or bottom
surface of the board is the bending stress x. Therefore, on the top surface where the stress is tensile, x is the
principal stress 1 . Thus,
t
Distance to extreme fiber c  c  16 mm
2
3
w t 5 4
Moment of inertia I  I  8.329  10  mm
12
Mmax c
Bending stress σx  σx  87.086 MPa
I

Maximum principal stress σ1  σx σ1  87.1 MPa

5. Calculate the maximum deflection from the equation given in Figure D-3(a) at x = L. Let b in the figure be a in
our problem and let a in the figure be equal to L, then

3
Fi L
ymax  ymax  297.7  mm
3  E I

© 2011 Pearson Education, Inc., Upper Saddle River, NJ. All rights reserved. This publication is protected by Copyright and written permission should be
MACHINE DESIGN - An Integrated Approach, 4th Ed. 4-14-1
PROBLEM 4-14
Statement: Figure P4-5 shows a child's toy called a pogo stick. The child stands on the pads, applying
half her weight on each side. She jumps off the ground, holding the pads up against her
feet, and bounces along with the spring cushioning the impact and storing energy to help
each rebound. Design the aluminum cantilever beam sections on which she stands to
survive jumping 2 in off the ground. Assume an allowable stress of 20 ksi. Define and size
the beam shape.

Given: Allowable stress σallow  20 ksi


6
Young's modulus E  10.3 10  psi

Assumptions: The beam will have a rectangular


cross-section with the load applied at a
distance of 5 in from the central support.

L  5  in
Solution: See Figure 4-14 and Mathcad file P0414.

1. From Problem 3-14, the total dynamic force on both


foot supports is
Fi  224  lbf
Therefore, the load on each support is Fi /2 Fi /2

Fi
P  P  112  lbf
2

2. To give adequate support to the childs foot, let the


width of the support beam be

w  1.5 in
3. From Figure B-1(a) in Appendix B, the maximum
bending moment at x = 0 is
P
M  P L M  560  in lbf
FIGURE 4-14
4. We can now calculate the minimum required section modulus, Z = I/c.
Free Body Diagram for Problem 4-14

M
Bending stress σ= = σallow
Z
M 3
Solving for Z, Z  Z  458.8  mm
σallow

3 2
w t t w t
5. For a rectangular cross-section, I = and c = so Z =
12 2 6

6 Z
Solving for t, t  t  0.335  in
w

Round this up to the next higher decimal equivalent of a common fraction, t  0.375  in

© 2011 Pearson Education, Inc., Upper Saddle River, NJ. All rights reserved. This publication is protected by Copyright and written permission should be
MACHINE DESIGN - An Integrated Approach, 4th Ed. 4-15-1
PROBLEM 4-15
Statement: Design a shear pin for the propeller shaft of an outboard motor if the shaft through which the
pin is placed is 25-mm diameter, the propeller is 20-cm diameter, and the pin must fail when a
force > 400 N is applied to the propeller tip. Assume an ultimate shear strength for the pin
material of 100 MPa.

Given: Propeller shaft dia d  25 mm Fpin


T Propeller Hub
Propeller dia D  200  mm
Max propeller tip force Fmax  400  N
Ultimate shear strength S us  100  MPa Shear Pin

Assumptions: A shear pin is in direct, double shear.


Fpin Propeller Shaft
Solution: See Figure 4-15 and Mathcad file P0415.
d
1. Calculate the torque on the propeller shaft that will FIGURE 4-15
result from a tip force on the propeller of Fmax. Free Body Diagram for Problem 4-15

D
T  Fmax T  40000  N  mm
2
2. This will be reacted by the shear pin's couple on the shaft. Determine the magnitude of the direct shear force.
T
Fpin  Fpin  1600 N
d

3. Determine the maximum pin diameter that will shear at this force.

Fpin 4  Fpin
Direct shear stress τ= = = S us
A 2
π d pin

4  Fpin
Solving for the pin diameter d pin  d pin  4.514  mm
π S us

Round this to d pin  4.5 mm

© 2011 Pearson Education, Inc., Upper Saddle River, NJ. All rights reserved. This publication is protected by Copyright and written permission should be
MACHINE DESIGN - An Integrated Approach, 4th Ed. 4-16-1
PROBLEM 4-16
Statement: A track to guide bowling balls is designed with two round rods as shown in Figure P4-6. The
rods are not parallel to one another but have a small angle between them. The balls roll on the
rods until they fall between them and drop onto another track. The angle between the rods is
varied to cause the ball to drop at different locations. Find the maximum stress and deflection
in the rods assuming that they are
(a) simply supported at each end, and
(b) fixed at each end.

Given: Rod length L  30 in


a Fball
Rod diameter d  1.00 in
Distance to load a  23.15  in
6 R1 L R2
Young's modulus E  30 10  psi

Assumptions: The analysis of Problem 3-16 yielded


FIGURE 4-16A
the following for a simply supported
beam: Free Body Diagram for Problem 4-16(a), taken
on a plane through the rod axis and ball center
Max ball load Fball  13.89  lbf
Max moment Mmax  73.4 in lbf
Reactions R1  3.17 lbf
R2  10.72  lbf

Solution: See Figure 4-16 and Mathcad file P0416.


1. The maximum bending stress will occur at the outer fibers of the rod at the section where the maximum
bending moment occurs which, in this case, is at x = a. The only stress present on the top or bottom surface of
the rod is the bending stress x. Therefore, on the bottom surface where the stress is tensile, x is the principal
stress 1 . Thus, for a simply supported rod,

d
Distance to extreme fiber c  c  0.5 in
2
4
π d 4
Moment of inertia I  I  0.0491 in
64
Mmax c
Bending stress σx  σx  748  psi
I

Maximum principal stress σ1  σx σ1  748  psi

2. Calculate the maximum deflection for the simply supported case from the equation given in Figure D-2(a),

Fball  a
4 
  2 a  
3 2
ymax   L a ymax  0.0013 in
6  E I  L 
a Fball
3. For the case where the rod is built in at each end,
the beam is statically indeterminate. As seen in
Figure 4-16B, there are four unknown reactions and M1
only two equilibrium equations can be written using R1 L R 2 M2
statics. We will find the reactions using Example 4-7
as a model. FIGURE 4-16B
Free Body Diagram for Problem 4-16(b), taken on a
plane through the rod axis and ball center
© 2011 Pearson Education, Inc., Upper Saddle River, NJ. All rights reserved. This publication is protected by Copyright and written permission should be
MACHINE DESIGN - An Integrated Approach, 4th Ed. 4-16-2

4. Write an equation for the load function in terms of equations 3.17 and integrate the resulting function four
times using equations 3.18 to obtain the shear and moment functions. Note use of the unit doublet function to
represent the moment at the wall. For the beam in Figure 4-16B,

q(x) = -M1<x - 0>-2 + R1<x - 0>-1 - F<x - a>-1 + R2<x - L>-1 + M2<x - L>-2

V(x) = -M1<x - 0>-1 + R1<x - 0>0 - F<x - a>0 + R2<x - L>0 + M2<x - L>-1 + C1

M(x) = -M1<x - 0>0 + R1<x - 0>1 - F<x - a>1 + R2<x - L>1 + M2<x - L>0 + C1x+ C2

(x) = ( -M1<x - 0>1 + R1<x - 0>2/2 - F<x - a>2/2 + R2<x - L>2/2 + M2<x - L>1 + C1x2/2 + C2x + C3) / EI

y(x) = ( -M1<x - 0>2/2 + R1<x - 0>3/6 - F<x - a>3/6 + R2<x - L>3/6 + M2<x - L>2 /2+ C1x3/6 + C2x2/2 + C3x + C4) / EI

5. Because the reactions have been included in the loading function, the shear and moment diagrams both
close to zero at each end of the beam, making C1 = C2 = 0. This leaves six unknowns; the four reactions and the
constants of integration, C3 and C4. There are four boundary conditions that we can use and two equilibrium
equations. The boundary conditions are: at x = 0,  = 0 and y = 0; and at x = L,  = 0 and y = 0. Applying the
boundary conditions at x = 0 results in C3 = C4 = 0. Applying the BCs at x = L results in the following two
equations, which are solved for R1 and M1.
R1 2 F 2
At x = L, θ=0 0=  L  M 1 L   ( L  a)
2 2
R1 3 M1 2 F 3
y =0 0= L   L   ( L  a)
6 2 6

Solving these two equations simultaneously for R1 and M1,

Fball  ( L  a)
3
 ( L  a )  
2
M1  M1  16.765 in lbf
L  L 
2
M1 ( L  a)
R1  2   Fball  R1  1.842  lbf
L 2
L

6. The remaing two reactions can be found by using the equations of equilibrium.
 Fy = 0: R1  Fball  R2 = 0

 M = 0: M1  Fball  a  R2 L  M2 = 0

Solving these two equations simultaneously for R2 and M2,

R2  Fball  R1 R2  12.048 lbf

M2  M1  Fball  a  R2 L M2  56.657 in lbf


7. Define the range for x, x  0  in 0.005  L  L
8. For a Mathcad solution, define a step function S. This function will have a value of zero when x is less than z,
and a value of one when it is greater than or equal to z.

S ( x z)  if ( x  z 1 0 )


9. Write the shear, moment, slope, and deflection equations in Mathcad form, using the function S as a multiplying
factor to get the effect of the singularity functions.
© 2011 Pearson Education, Inc., Upper Saddle River, NJ. All rights reserved. This publication is protected by Copyright and written permission should be
MACHINE DESIGN - An Integrated Approach, 4th Ed. 4-16-3

V ( x)  R1 S ( x 0  mm)  Fball  S ( x a )  R2 S ( x L)

M ( x)  M1 S ( x 0  mm)  R1 S ( x 0  mm)  ( x  0  mm)  Fball  S ( x a )  ( x  a ) 
 M2 S ( x L)  R2 S ( x L)  ( x  L)

1  R1 2 Fball 2 
θ ( x)   M1 S ( x 0  mm)  x   S ( x 0  mm)  ( x  0  mm)   S ( x a )  ( x  a ) 
E I 2 2
 R 
 M2 S( x L)  ( x  L)  2  S( x L)  ( x  L) 2 
 2 

1  M1 2 R1 3 Fball 3 
y ( x)    S ( x 0  mm)  x   S ( x 0  mm)  ( x  0  mm)   S ( x a )  ( x  a ) 
E I 2 6 6
 M2 R 
  S ( x L)  ( x  L) 2  2  S ( x L)  ( x  L) 3 
 2 6 
10. Plot the shear, moment, slope, and deflection diagrams.

(a) Shear Diagram (b) Moment Diagram

5 40

20
0
Moment, M - lb in
Shear, V - lb

0
5
 20

 10
 40

 15  60
0 10 20 30 0 10 20 30

Distance along beam, x - in Distance along beam, x - in

(c) Slope Diagram (d) Deflection Diagram

0.1 0
Deflection - thousandths of in
Slope - Thousands of Rad

 0.2

0  0.4

 0.6

 0.1  0.8
0 10 20 30 0 10 20 30

Distance along beam, x - in Distance along beam, x - in

FIGURE 4-16C
Shear, Moment, Slope, and Deflection Diagrams for Problem 4-16(b)
© 2011 Pearson Education, Inc., Upper Saddle River, NJ. All rights reserved. This publication is protected by Copyright and written permission should be
MACHINE DESIGN - An Integrated Approach, 4th Ed. 4-16-4

11 The maximum moment occurs at x = L and is


Mmax  M2 Mmax  56.7 in lbf

12 Calculate the maximum bending and principal stresses.

Mmax c
Bending stress σx  σx  577  psi
I

Maximum principal stress σ1  σx σ1  577  psi

13. To find the maximum deflection, first determine at what point on the beam the slope is zero. Let this be at x = e.
From the slope diagram, we see that e < a. Using the slope equation and setting it equal to zero, we have

R1 2
For  = 0 0 = M1 e  e
2
2  M1
Solving for e e  e  18.204 in
R1

Maximum deflection ymax  y ( e) ymax  0.00063  in

© 2011 Pearson Education, Inc., Upper Saddle River, NJ. All rights reserved. This publication is protected by Copyright and written permission should be
MACHINE DESIGN - An Integrated Approach, 4th Ed. 4-17-1
PROBLEM 4-17
Statement: A pair of ice tongs is shown in Figure P4-7. The ice weighs 50 lb and is 10 in wide across the
tongs. The distance between the handles is 4 in, and the mean radius r of the tong is 6 in. The
rectangular cross-sectional dimensions are 0.75 x 0.312 in. Find the stress in the tongs.

Given: Mean radius of tong rc  6.00 in F


Tong width w  0.312  in
C
Tong depth h  0.75 in FC
3.5 = cy
O
Assumptions: The tong can be analyzed as a curved FO
beam.
11.0 = ax 2.0 = cx

Solution: See Problem 3-17, Figure 4-17, A


12.0 = by
and Mathcad file P0417.
5.0 = bx
1. The maximum bending moment and axial force in the
tong were found in Problem 3-17 at point A. They are FB
B
Maximum moment MA  237.5  in lbf W/2
Axial force at D FAn  25 lbf
FIGURE 4-17
Free Body Diagram for Problem 4-17
2. Calculate the section area, inside radius and outside radus.
2
Area of section A  h  w A  0.234  in
Inside and outside radii ri  rc  0.5 h ri  5.625  in
of section
ro  rc  0.5 h ro  6.375  in

3. Use the equation in the footnote on page 195 of the text to calculate the radius of the neutral axis.
ro  ri
Radius of neutral axis rn  rn  5.992  in
 ro 
ln 
 ri 
4. Calculate the eccentricty and the distances from the neutral axis to the extreme fibers.
Eccentricity e  rc  rn e  0.007821 in

Distances from neutral ci  rn  ri ci  0.3672 in


axis to extreme fibers
co  ro  rn co  0.3828 in

MA c i FAn
Stresses at inner and σi    σi  8.58 ksi
outer radii e A ri A

 MA co  FAn
σo     σo  7.69 ksi
 e A ro  A

5. The shear stress is zero at the outer fibers. Therefore, these are the principal stresses. At the inner surface
σ1  σi σ1  8.58 ksi σ2  0  ksi σ3  0  ksi

© 2011 Pearson Education, Inc., Upper Saddle River, NJ. All rights reserved. This publication is protected by Copyright and written permission should be
MACHINE DESIGN - An Integrated Approach, 4th Ed. 4-18-1
PROBLEM 4-18
Statement: A set of steel reinforcing rods is to be stretched axially in tension to create a tensile stress of 30
ksi prior to being cast in concrete to form a beam. Determine how much force will be required to
stretch them the required amount and how much deflection is required. There are 10 rods; each
is 0.75-in diameter and 30 ft long.

Given: Desired stress σ  30 ksi Rod diameter d  0.75 in


6
Number of rods Nrods  10 Young's modulus E  30 10  psi
Rod length L  30 ft

Assumptions: The rods share the load equally.


Solution: See Mathcad file P0418.
2
π d 2
1. Calculate the cross-sectional area of one rod. A  A  0.442  in
4

2. Determine the force required to achieve the desired stress level in one rod.
F
σ= F  σ  A F  13.254 kip
A

3. Determine the total force required to achieve the desired stress level in all rods.
Ftotal  Nrods F Ftotal  132.5  kip

4. Determine the amount the rods will deflect under the applied load.
F L
δ  δ  0.360  in
A E

© 2011 Pearson Education, Inc., Upper Saddle River, NJ. All rights reserved. This publication is protected by Copyright and written permission should be
MACHINE DESIGN - An Integrated Approach, 4th Ed. 4-19-1
PROBLEM 4-19
Statement: The clamping fixture used to pull the rods in Problem 4-18 is conected to the hydraulic ram by a
clevis like that shown in Figure P4-8. Determine the size of the clevis pin needed to withstand
the applied force. Assume an allowable shear stress of 20 000 psi and an allowable normal
stress of 40 000 psi. Determine the required outside radius of the clevis end to not exceed the
above allowable stresses in either tear out or bearing if the clevis flanges are each 0.8 in thick.

Given: Desired rod stress σrod  30 ksi Rod diameter d  0.75 in
6
Number of rods Nrods  10 Young's modulus E  30 10  psi
Rod length L  30 ft Clevis flange thickness t  0.8 in
Clevis strength S sallow  20 ksi
S ballow  40 ksi

Assumptions: The rods share the load equally, and there is one clevis for all ten rods.
Solution: See Figures 4-12 and 4-13 in the text, Figure 4-19, and Mathcad file P0419.
2
π d 2
1. Calculate the cross-sectional area of one rod. A  A  0.442  in
4

2. Determine the force required to achieve the desired stress level in one rod.
F
σrod = F  σrod  A F  13.254 kip
A

3. Determine the total force required to achieve the desired stress level in all rods.
Ftotal  Nrods F Ftotal  132.5  kip

This force is transmitted through the clevis pin, which is in double shear.

4. Calculate the minimum required clevis pin diameter for the allowable shear stress. Divide the load by 2
because of the double shear loading.

Ftotal 2  Ftotal
τpin = = = S sallow
2  Apin 2
π d

2  Ftotal
Solving for the pin diameter d  d  2.054  in
π S sallow

Round this up to the next higher decimal equivalent of a common fraction ( 2 1/8) d  2.125  in

5. Check the bearing stress in the clevis due to the pin on one side of the clevis.
2
Bearing stress area Ab  d  t Ab  1.700  in

Ftotal
Bearing force Fb  Fb  66.268 kip
2
Fb
Bearing stress σb  σb  39.0 ksi
Ab

Since this is less than S ballow, this pin diameter is acceptable.

© 2011 Pearson Education, Inc., Upper Saddle River, NJ. All rights reserved. This publication is protected by Copyright and written permission should be
MACHINE DESIGN - An Integrated Approach, 4th Ed. 4-19-2

6. Determine the tearout stress in the clevis.

Tearout length
2 2
Shear area (see Figure 4-19) Atear = 2  t R  ( 0.5 d )

Shear force

Ftotal
Ftear  Ftear  66.268 kip
2

Shear stress and strength


d R
Ftear Ftear FIGURE 4-19
τ= = = S sallow
Tearout Diagram for Problem 4-19
Atear 2 2
2  t R  ( 0.5 d )

2
 Ftear  2
Solving for the clevis radius, R R     ( 0.5 d) R  2.328  in
 2  t  S sallow 

Round this up to the next higher decimal equivalent of a common fraction ( 2 3/8) R  2.375  in

© 2011 Pearson Education, Inc., Upper Saddle River, NJ. All rights reserved. This publication is protected by Copyright and written permission should be
MACHINE DESIGN - An Integrated Approach, 4th Ed. 4-20-1
PROBLEM 4-20
Statement: Repeat Problem 4-19 for 12 rods, each 1 cm in diameter and 10 m long. The desired rod stress is 20
MPa. The allowable normal stress in the clevis and pin is 280 MPa and their allowable shear stress
is 140 MPa. Each clevis flange is 2 cm wide.
3 6 9
Units: kN  10  newton MPa  10  Pa GPa  10  Pa
Given: Desired rod stress σrod  200  MPa Rod diameter d  10 mm
Number of rods Nrods  12 Young's modulus E  207  GPa
Rod length L  10 m Clevis flange thickness t  20 mm
Clevis strength S sallow  140  MPa
S ballow  280  MPa

Assumptions: The rods share the load equally, and there is one clevis for all twelve rods.
Solution: See Figures 4-12 and 4-13 in the text, Figure 4-20, and Mathcad file P0420.
2
π d 2
1. Calculate the cross-sectional area of one rod. A  A  78.54 mm
4

2. Determine the force required to achieve the desired stress level in one rod.
F
σrod = F  σrod  A F  15.708 kN
A

3. Determine the total force required to achieve the desired stress level in all rods.
Ftotal  Nrods F Ftotal  188.5 kN

This force is transmitted through the clevis pin, which is in double shear.

4. Calculate the minimum required clevis pin diameter for the allowable shear stress. Divide the load by 2 because
of the double shear loading.

Ftotal 2  Ftotal
τpin = = = S sallow
2  Apin 2
π d

2  Ftotal
Solving for the pin diameter d  d  29.277 mm
π S sallow

Round this up to the next higher even mm d  30 mm

5. Check the bearing stress in the clevis due to the pin on one side of the clevis.
2
Bearing stress area Ab  d  t Ab  600 mm

Ftotal
Bearing force Fb  Fb  94.248 kN
2
Fb
Bearing stress σb  σb  157.1 MPa
Ab

Since this is less than S ballow, this pin diameter is acceptable.

© 2011 Pearson Education, Inc., Upper Saddle River, NJ. All rights reserved. This publication is protected by Copyright and written permission should be
MACHINE DESIGN - An Integrated Approach, 4th Ed. 4-20-2

6. Determine the tearout stress in the clevis.

Tearout length
2 2
Shear area (see Figure 4-19) Atear = 2  t R  ( 0.5 d )

Shear force

Ftotal
Ftear  Ftear  94.248 kN
2

Shear stress and strength


d R
Ftear Ftear
τ= = = S sallow
Atear 2 2 FIGURE 4-20
2  t R  ( 0.5 d ) Tearout Diagram for Problem 4-20

2
 Ftear  2
Solving for the clevis radius, R R     ( 0.5 d) R  22.544 mm
 2  t  S sallow 

Round this up to the next higher even mm R  24 mm

© 2011 Pearson Education, Inc., Upper Saddle River, NJ. All rights reserved. This publication is protected by Copyright and written permission should be
MACHINE DESIGN - An Integrated Approach, 4th Ed. 4-21-1
PROBLEM 4-21
Statement: Figure P4-9 shows an automobile wheel with two common styles of lug wrench being used to
tighten the wheel nuts, a single-ended wrench in (a), and a double-ended wrench in (b). In each
case two hands are required to provide forces respectively at A and B as shown. The distance
between points A and B is 1 ft in both cases and the handle diameter is 0.625 in. The wheel nuts
require a torque of 70 ft-lb. Find the maximum principle stress and maximum deflection in each
wrench design.

Given: Distance between A and B d AB  1  ft


Tightening torque T  70 ft  lbf
Wrench diameter d  0.625  in
Assumptions: 1. The forces exerted by the user's hands lie in a plane through the wrench that is also parallel
to the plane of the wheel.
2. The applied torque is perpendicular to the plane of the forces.
3. By virtue of 1 and 2 above, this is a planar problem that can be described in a 2D FBD.

Solution: See Figure 4-21 and Mathcad file P0421.


12" = dAB
1. In Problem 3-21 we found that for both cases F

F  70 lbf

2. From examination of the FBDs, we see that, in


both cases, the arms are in bending and the stub T
that holds the socket wrench is in pure torsion.
The maximum bending stress in the arm will occur F
near the point where the arm transitions to the (a) Single-ended Wrench
stub. The stress state at this transition is very
complicated, but we can find the nominal bending
stress there by treating the arm as a cantilever 12" = dAB
beam, fixed at the transition point. For both cases
F
the torque in the stub is the same. 6"

Case (a)
T
2. The bending moment at the transition is
F
(b) Double-ended Wrench
Ma  F  d AB Ma  840  lbf  in
FIGURE 4-21
3. The tensile stress at this point is found from Free Body Diagrams for Problem 4-21

4
π d 4
Moment of inertia I  I  0.00749  in
64
Dist to extreme fibre c  0.5 d c  0.313  in
M a c
Stress σx  σx  35.05  ksi
I
4. There are no other stress components present at this point, so x is the maximum principle stress here and

σ1  σx σ1  35.0 ksi σ2  0  psi σ3  0  psi

5. The torque in the stub is T  840  in lbf


6. The shear stress at any point on the outside surface of the stub is found from
© 2011 Pearson Education, Inc., Upper Saddle River, NJ. All rights reserved. This publication is protected by Copyright and written permission should be
MACHINE DESIGN - An Integrated Approach, 4th Ed. 4-21-2

4
Polar moment of inertia J  2  I J  0.0150 in
Tc
Shear stress τxy  τxy  17.52  ksi
J

7. There are no other stress components present along the outside surface of the stub, so
σ1  τxy σ1  17.5 ksi σ2  0  psi σ3  σ1

8. Thus, the maximum principle stress for case (a) is on the upper surface of the handle (arm) near the point
where it transitions to the stub.

There will be two deflection components that we can calculate separately and then add (superposition).
One will come from the bending of the arm and one will come from the twisting of the stub, projected out
to the end of the arm.

9. Deflection of the arm due to bending only for a stub length of stub  3  in:

6 6
Assuming that the wrenches are made from steel E  30 10  psi G  11.7 10  psi

3
F  d AB
From Figure B-1(a), Appendix B, yarm  yarm  0.179  in
3  E I

From equation (4.24), the T  stub


angular twist of the stub is θstub  θstub  0.014  rad
J G
The deflection at the end of
the arm due to the stub twist ystub  d AB θstub ystub  0.173  in
is
So, the total deflection is ya  yarm  ystub ya  0.352  in

Case (b)
F  d AB
10. The bending moment at the transition is Mb  Mb  420  lbf  in
2

11. The tensile stress at this point is found from

M b c
Stress σx  σx  17.52  ksi
I

12. There are no other stress components present at this point, so x is the maximum principle stress here and

σ1  σx σ1  17.5 ksi σ2  0  psi σ3  0  psi

13. The torque in the stub is T  840  in lbf

14. The shear stress at any point on the outside surface of the stub is found from
Tc
Shear stress τxy  τxy  17.52  ksi
J
15. There are no other stress components present along the outside surface of the stub, so
σ1  τxy σ1  17.5 ksi σ2  0  psi σ3  σ1

© 2011 Pearson Education, Inc., Upper Saddle River, NJ. All rights reserved. This publication is protected by Copyright and written permission should be
MACHINE DESIGN - An Integrated Approach, 4th Ed. 4-21-3
16. Thus, the maximum principle stress for case (b) is the same on the upper surface of the handle (arm) near the
point where it transitions to the stub, and on the outside surface of the stub.

There will be two deflection components that we can calculate separately and then add (superposition).
One will come from the bending of the arm and one will come from the twisting of the stub, projected out to
the end of the arm.

F   0.5 d AB
Deflection of the arm due to bending only: 3
From Figure B-1(a), Appendix B, yarm  yarm  0.022  in
3  E I

From equation (4.24), the T  stub


angular twist of the stub is θstub  θstub  0.014  rad
J G

The deflection at the end of d AB


the arm due to the stub twist ystub   θstub ystub  0.086  in
2
is

So, the total deflection is yb  yarm  ystub yb  0.109  in

© 2011 Pearson Education, Inc., Upper Saddle River, NJ. All rights reserved. This publication is protected by Copyright and written permission should be
MACHINE DESIGN - An Integrated Approach, 4th Ed. 4-22-1
PROBLEM 4-22
Statement: A roller-blade skate is shown in Figure P4-10. The polyurethane wheels are 72 mm dia. The
skate-boot-foot combination weighs 2 kg. The effective "spring rate" of the person-skate
subsystem is 6000 N/m. The axles are 10-mm-dia steel pins in double shear. Find the stress in
the pins for a 100-kg person landing a 0.5-m jump on one foot. (a) Assume all 4 wheels land
simultaneously. (b) Assume that one wheel absorbs all the landing force.

Given: Axle pin diameter d  10 mm


Solution: See Figure P4-10 and Mathcad file P0422.

1. From Problem 3-22, we have the forces for cases (a) and (b): Fa  897  N Fb  3.59 kN
2. In both cases, this is the force on one axle. The shear force will be one half of these forces because the pins
are in double shear.
2
π d 2
Shear area As  As  78.54  mm
4

Shear stress
Fa
Case (a) all wheels landing τa  τa  5.71 MPa
2  As

Fb
Case (b) one wheel landing τb  τb  22.9 MPa
2  As

© 2011 Pearson Education, Inc., Upper Saddle River, NJ. All rights reserved. This publication is protected by Copyright and written permission should be
MACHINE DESIGN - An Integrated Approach, 4th Ed. 4-23a-1
PROBLEM 4-23a
Statement: A beam is supported and loaded as shown in Figure P4-11a. Find the reactions, maximum shear,
maximum moment, maximum slope, maximum bending stress, and maximum deflection for the
data given in row a from Table P4-2.

Given: Beam length L  1  m


L
Distance to distributed load a  0.4 m
b
Distance to concentrated load b  0.6 m a
1 F
Distributed load magnitude w  200  N  m w

Concentrated load F  500  N


8 4 R2
Moment of inertia I  2.85 10 m R1
2
Distance to extreme fiber c  2.00 10 m FIGURE 4-23A
Free Body Diagram for Problem 4-23
Solution: See Figures 4-23 and Mathcad file P0423a.
1. The reactions, maximum shear and maximum moment were all found in Problem 3-23a. Those results are
summarized here.
Load function q(x) = R1<x - 0>-1 - w<x - 0>0 + w<x - a>0 - F<x - b>-1 + R2<x - L>-1
Shear function V(x) = R1<x - 0>0 - w<x - 0>1 + w<x - a>1 - F<x - b>0 + R2<x - L>0
Moment function M(x) = R1<x - 0>1 - w<x - 0>2/2 + w<x - a>2/2 - F<x - b>1 + R2<x - L>1
Modulus of elasticity E  207  GPa
Reactions R1  264.0  N R2  316.0  N
Maximum shear Vmax  316  N (negative, from x = b to x =L)
Maximum moment Mmax  126.4  N  m (at x = b)

2. Integrate the moment function, multiplying by 1/EI, to get the slope.


(x) = [R1<x>2/2 - w<x>3/6 + w<x - a>3/6 - F<x - b>2/2 + R2<x - L>2/2 + C3]/EI
3. Integrate again to get the deflection.
y(x) = [R1<x>3/6 - w<x>4/24 + w<x - a>4/24 - F<x - b>3/6 + R2<x-L>3/6 + C3x +C4]/EI
4. Evaluate C3 and C4
At x = 0 and x = L, y = 0, therefore, C4 = 0.

R1 3 w 4 w 4 F 3
0= L  L   ( L  a)   ( L  b )  C 3 L
6 24 24 6

1  R1 3 w 4 w 4 F 3 2
C3   
L  L   ( L  a)   ( L  b)  C3  31.413 N  m
L  6 24 24 6 
5. Define the range for x x  0  m 0.005  L  L

6. For a Mathcad solution, define a step function S. This function will have a value of zero when x is less than z,
and a value of one when it is greater than or equal to z.

S ( x z)  if ( x  z 1 0 )


7. Write the slope and deflection equations in Mathcad form, using the function S as a multiplying factor to
get the effect of the singularity functions. See Figure 4-23aB where these functions are plotted.

© 2011 Pearson Education, Inc., Upper Saddle River, NJ. All rights reserved. This publication is protected by Copyright and written permission should be
MACHINE DESIGN - An Integrated Approach, 4th Ed. 4-23a-2

1  R1 2 w 3 w 3 
θ ( x)    S ( x 0  in)  x   S ( x 0  in)  x   S ( x a )  ( x  a ) 
E I 2 6 6
 R 
 2  S ( x L)  ( x  L) 2   F  S( x b)  ( x  b ) 2  C3 
 2 2 

1  R1 3 w 4 w 4 
y ( x)    S ( x 0  in)  x   S ( x 0  in)  x   S ( x a )  ( x  a ) 
E I 6 24 24
 R 
 2  S( x L)  ( x  L) 3   F  S ( x b )  ( x  b) 3  C3 x 
 6 6 
8. Maximum slope occurs at x = L θmax  θ ( L) θmax  0.335  deg

9. Maximum deflection occurs at x = c, where  = 0 and c < b.

1  R1 2 w 3 w 3 
θ0 =  c   c   ( c  a )  C3 = 0
E I  2 6 6 
Solving for c,
R1 w w 2 w 3
A   3 a B  3  a C  C3  a
2 6 6 6
2
A  92.000 N B  16.000 N  m C  33.547 N  m

2
B  B  4 A  C
c  c  0.523  m
2 A

Substituting c into the deflection equation, ymax  y ( c) ymax  1.82 mm

SLOPE, radians DEFECTION, mm


0.01 0

0.005  0.5

y ( x)
θ( x) 0 1
mm

 0.005  1.5

 0.01 2
0 0.2 0.4 0.6 0.8 1 0 0.2 0.4 0.6 0.8 1
x x
m m

FIGURE 4-23aB
Slope and Deflection Diagrams for Problem 4-23a
10. The maximum bending stress occurs at x = b, where the moment is a maximum. For
2
c  2.00 10 m c  20 mm

Mmax c
σmax  σmax  88.7 MPa
I
© 2011 Pearson Education, Inc., Upper Saddle River, NJ. All rights reserved. This publication is protected by Copyright and written permission should be
MACHINE DESIGN - An Integrated Approach, 4th Ed. 4-24a-1
PROBLEM 4-24a
Statement: A beam is supported and loaded as shown in Figure P4-11b. Find the reactions, maximum
shear, maximum moment, maximum slope, maximum bending stress, and maximum deflection for
the data given in row a from Table P4-2.

Given: Beam length L  1  m L


a
Distance to distributed load a  0.4 m
F
Distance to concentrated load b  0.6 m
w
1
Distributed load magnitude w  200  N  m
Concentrated load F  500  N M1
8 4
Moment of inertia I  2.85 10 m R1
2
Distance to extreme fiber c  2.00 10 m
FIGURE 4-24A
Free Body Diagram for Problem 4-24
Solution: See Figures 4-24 and Mathcad file P0424a.

1. The reactions, maximum shear and maximum moment were all found in Problem 3-24a. Those results are
summarized here.
Load function q(x) = -M1<x - 0>-2 + R1<x - 0>-1 - w<x - a>0 - F<x - L>-1
Shear function V(x) = -M1<x - 0>-1 + R1<x - 0>0 - w<x - a>1 - F<x - L>0
Moment function M(x) = -M1<x - 0>0 + R1<x - 0>1 - w<x - a>2/2 - F<x - L>1
Modulus of elasticity E  207  GPa
Reactions R1  620.0  N M1  584.0  N  m
Maximum shear Vmax  620  N (positive, at x = 0)
Maximum moment Mmax  584  N  m (negative, at x = 0)

2. Integrate the moment function, multiplying by 1/EI, to get the slope.


(x) = [-M1<x-0>1 + R1<x - 0>2/2 - w<x - a>3/6 - F<x - L>2/2 + C3]/EI

3. Integrate again to get the deflection.


y(x) = [-M1<x-0>2/2 + R1<x - 0>3/6 - w<x - a>4/24 - F<x - L>3/6 + C3x +C4]/EI

4. Evaluate C3 and C4. At x = 0,  = 0 and y = 0, therefore, C3 = 0 and C4 = 0.

5. Define the range for x x  0  m 0.005  L  L

6. For a Mathcad solution, define a step function S. This function will have a value of zero when x is less than z,
and a value of one when it is greater than or equal to z.

S ( x z)  if ( x  z 1 0 )

7. Write the slope and deflection equations in Mathcad form, using the function S as a multiplying factor to get
the effect of the singularity functions. See Figure 4-24aB where these functions are plotted.

1  R1 2 w 3 
θ ( x)   M1 S ( x 0  in)  x   S ( x 0  in)  x   S ( x a )  ( x  a ) 
E I 2 6
 F 
   S( x L)  ( x  L) 2 
 2 

© 2011 Pearson Education, Inc., Upper Saddle River, NJ. All rights reserved. This publication is protected by Copyright and written permission should be
MACHINE DESIGN - An Integrated Approach, 4th Ed. 4-24a-2

1  M1 2 R1 3 w 4 
y ( x)     S ( x 0  in)  x   S ( x 0  in)  x   S ( x a )  ( x  a ) 
E I 2 6 24
 F 
   S ( x L)  ( x  L) 3 
 6 

8. Maximum slope occurs at x = L θmax  θ ( L) θmax  2.73 deg

9. Maximum deflection occurs at x = L ymax  y ( L) ymax  32.2 mm

10. The maximum bending stress occurs at x = 0, where the moment is a maximum. For c  20 mm

M1 c
σmax  σmax  410  MPa
I

SLOPE, radians DEFLECTION, mm

0 0

 0.01
 10

 0.02
y ( x)
θ( x)  20
mm
 0.03

 30
 0.04

 0.05  40
0 0.2 0.4 0.6 0.8 1 0 0.2 0.4 0.6 0.8 1
x x
m m

FIGURE 4-24aB
Slope and Deflection Diagrams for Problem 4-24a

© 2011 Pearson Education, Inc., Upper Saddle River, NJ. All rights reserved. This publication is protected by Copyright and written permission should be
MACHINE DESIGN - An Integrated Approach, 4th Ed. 4-25a-1
PROBLEM 4-25a
Statement: A beam is supported and loaded as shown in Figure P4-11c. Find the reactions, maximum
shear, maximum moment, maximum slope, maximum bending stress, and maximum deflection for
the data given in row a from Table P4-2.

Given: Beam length L  1  m L


Distance to distributed load a  0.4 m b
Distance to reaction load b  0.6 m a F
1 w
Distributed load magnitude w  200  N  m
Concentrated load F  500  N
8 4 R1 R2
Moment of inertia I  2.85 10 m
2
Distance to extreme fiber c  2.00 10 m FIGURE 4-25A
Free Body Diagram for Problem 4-25
Solution: See Figures 4-25 and Mathcad file P0425a.

1. The reactions, maximum shear and maximum moment were all found in Problem 3-25a. Those results are
summarized here.
Load function q(x) = R1<x - 0>-1 - w<x - a>0 + R2<x - b>-1 - F<x - L>-1
Shear function V(x) = R1<x - 0>0 - w<x - a>1 + R2<x - b>0 - F<x - L>0
Moment function M(x) = R1<x - 0>1 - w<x - a>2/2 + R2<x - b>1 - F<x - L>1
Modulus of elasticity E  207  GPa
Reactions R1  353.3  N R2  973.3  N
Maximum shear Vmax  580  N (positive, at x = b)
Maximum moment Mmax  216  N  m (negative, at x = b)

2. Integrate the moment function, multiplying by 1/EI, to get the slope.


(x) = [R1<x - 0>2/2 - w<x - a>3/6 + R2<x - b>2/2 - F<x - L>2/2 + C3]/EI

3. Integrate again to get the deflection.


y(x) = [R1<x - 0>3/6 - w<x - a>4/24 + R2<x-b>3/6 - F<x - L>3/6 + C3x +C4]/EI

4. Evaluate C3 and C4
At x = 0 and x = b, y = 0, therefore, C4 = 0.

R1 3 w 4
0= b   ( b  a )  C3 b
6 24

1  R1 3 w 4 2
C3      b   ( b  a)  C3  21.22  N  m
b 6 24 

5. Define the range for x x  0  m 0.005  L  L


6. For a Mathcad solution, define a step function S. This function will have a value of zero when x is less than z,
and a value of one when it is greater than or equal to z.

S ( x z)  if ( x  z 1 0 )

7. Write the slope and deflection equations in Mathcad form, using the function S as a multiplying factor to get
the effect of the singularity functions. See Figure 4-25aB where these functions are plotted.
© 2011 Pearson Education, Inc., Upper Saddle River, NJ. All rights reserved. This publication is protected by Copyright and written permission should be
MACHINE DESIGN - An Integrated Approach, 4th Ed. 4-25a-2

1  R1 2 w 3
θ ( x)    S ( x 0  in)  x   S ( x a )  ( x  a ) 

E I 2 6
 R 
 2  S ( x b )  ( x  b) 2   F  S( x L)  ( x  L) 2  C3 
 2 2 

1  R1 3 w 4 
y ( x)    S ( x 0  in)  x   S ( x a )  ( x  a )  
E I 6 24
 R 
 2  S( x b)  ( x  b ) 3   F  S ( x L)  ( x  L) 3  C3 x 
 6 6 

8. Maximum slope occurs at x = L θmax  θ ( L) θmax  0.823  deg

9. Maximum deflection occurs at x = L. ymax  y ( L) ymax  4.81 mm

10. The maximum bending stress occurs at x = b, where the moment is a maximum. For c  20 mm

Mmax c
σmax  σmax  152  MPa
I

SLOPE, radians DEFLECTION, mm


0.005 2

0 0

y ( x)
θ( x)  0.005 2
mm

 0.01 4

 0.015 6
0 0.2 0.4 0.6 0.8 1 0 0.2 0.4 0.6 0.8 1
x x
m m

FIGURE 4-25aB
Slope and Deflection Diagrams for Problem 4-25a

© 2011 Pearson Education, Inc., Upper Saddle River, NJ. All rights reserved. This publication is protected by Copyright and written permission should be
MACHINE DESIGN - An Integrated Approach, 4th Ed. 4-26a-1
PROBLEM 4-26a
Statement: A beam is supported and loaded as shown in Figure P4-11d. Find the reactions, maximum
shear, maximum moment, maximum slope, maximum bending stress, and maximum deflection for
the data given in row a from Table P4-2.

Given: Beam length L  1  m


L
Distance to distributed load a  0.4 m
b
Distance to R2 b  0.6 m a F
1
Distributed load magnitude w  200  N  m w

Concentrated load F  500  N


8 4 R2 R3
Moment of inertia I  2.85 10 m R1
2
Distance to extreme fiber c  2.00 10 m FIGURE 4-26A
Free Body Diagram for Problem 4-26
Modulus of elasticity E  207  GPa

Solution: See Figures 4-26 and Mathcad file P0426a.


1. From inspection of Figure P4-11d, write the load function equation
q(x) = R1<x>-1 - F<x - a>-1 - w<x - a>0 + R2<x - b>-1 - R3<x - L>-1
2. Integrate this equation from - to x to obtain shear, V(x)
V(x) = R1<x>0 - F<x - a>0 - w<x - a>1 + R2<x - b>0 - R3<x - L>0
3. Integrate this equation from - to x to obtain moment, M(x)
M(x) = R1<x>1 - F<x - a>1 - w<x - a>2/2 + R2<x - b>1 - R3<x - L>1
4. Integrate the moment function, multiplying by 1/EI, to get the slope.
(x) = [R1<x>2/2 - F<x - a>2/2 - w<x - a>3/6 + R2<x - b>2/2 + R3<x - L>2/2 + C3]/EI
5. Integrate again to get the deflection.
y(x) = [R1<x>3/6 - F<x - a>3/6 - w<x - a>4/24 + R2<x - b>3/6 + R3<x - L>3/6 + C3x + C4]/EI

6. Evaluate R1, R2, R3, C3 and C4


At x = 0, x = b, and x = L; y = 0, therefore, C4 = 0.
At x = L+, V = M = 0
2
Guess R1  100  N R2  100  N R3  100  N C3  5  N  m

Given

R1 3 F 3 w 4 3
 b   ( b  a)   ( b  a )  C3 b = 0  N  m
6 6 24

R1 3 F 3 w 4 R2 3 3
 L   ( L  a)   ( L  a)   ( L  b )  C 3 L = 0  N  m
6 6 24 6

R1  F  w  ( L  a )  R2  R3 = 0  N

w 2
R 1 L  F  ( L  a )   ( L  a )  R 2 ( L  b ) = 0  N  m
2

© 2011 Pearson Education, Inc., Upper Saddle River, NJ. All rights reserved. This publication is protected by Copyright and written permission should be
MACHINE DESIGN - An Integrated Approach, 4th Ed. 4-26a-2

 R1 
 
 R2   Find  R R R C 
 R3  1 2 3 3

 
 C3 
2
R1  112.33 N R2  559.17 N R3  51.50  N C3  5.607  N  m

7. Define the range for x x  0  in 0.002  L  L

8. For a Mathcad solution, define a step function S. This function will have a value of zero when x is less than z,
and a value of one when it is greater than or equal to z.
S ( x z)  if ( x  z 1 0 )
9. Write the shear and moment equations in Mathcad form, using the function S as a multiplying factor to get
the effect of the singularity functions.

V ( x)  R1 S ( x 0  in)  F  S ( x a )  w S ( x a )  ( x  a )  R2 S ( x b )  R3 S ( x L)


w 2
M ( x)  R1 S ( x 0  in)  x  F  S ( x a )  ( x  a )   S ( x a )  ( x  a ) 
2
 R2 S ( x b )  ( x  b )

10. Plot the shear and moment diagrams.

SHEAR, N MOMENT, N-m


200 60

0 35

V ( x) M ( x)
 200 10
N Nm

 400  15

 600  40
3 3
0 200 400 600 800 1 10 0 200 400 600 800 1 10
x x
mm mm

FIGURE 4-26aB
Shear and Moment Diagrams for Problem 4-26a

11. From the diagram, we see that maximum shear occurs at x = b -,


Vmax  V ( b  0.001  mm) Vmax  428  N

12. The maximum moment occurs at x = a,


Mmax  M ( a ) Mmax  44.9 N  m
13. Write the slope and deflection equations in Mathcad form, using the function S as a multiplying factor to get t
effect of the singularity functions. See Figure 4-26aB where these functions are plotted.
© 2011 Pearson Education, Inc., Upper Saddle River, NJ. All rights reserved. This publication is protected by Copyright and written permission should be
MACHINE DESIGN - An Integrated Approach, 4th Ed. 4-26a-3

1  R1 2 F 2 w 3 
θ ( x)    S ( x 0  in)  x   S ( x a )  ( x  a )   S ( x a )  ( x  a ) 
E I 2 2 6
 R R 
 2  S ( x b )  ( x  b) 2  3  S ( x L)  ( x  L) 2  C3 
 2 2 

1  R1 3 F 3 w 4 
y ( x)    S ( x 0  in)  x   S ( x a )  ( x  a )   S ( x a )  ( x  a ) 
E I 6 6 24
 R R 
 2  S( x b)  ( x  b ) 3  3  S( x L)  ( x  L) 3  C3 x 
 6 6 

14. Maximum slope occurs between x = a and x = b θmax  0.0576 deg

15. Maximum deflection occurs between x = 0 and x = a ymax  0.200  mm

16. The maximum bending stress occurs at x = a, where the moment is a maximum. For c  20 mm

Mmax c
σmax  σmax  31.5 MPa
I

SLOPE, deg. DEFLECTION, mm


0.1 0.1

0.05 0

θ( x) y ( x)
0  0.1
deg mm

 0.05  0.2

 0.1  0.3
0 0.2 0.4 0.6 0.8 1 0 0.2 0.4 0.6 0.8 1
x x
m m

FIGURE 4-26aC
Slope and Deflection Diagrams for Problem 4-26a

© 2011 Pearson Education, Inc., Upper Saddle River, NJ. All rights reserved. This publication is protected by Copyright and written permission should be
MACHINE DESIGN - An Integrated Approach, 4th Ed. 4-27-1
PROBLEM 4-27
Statement: A storage rack is to be designed to hold the paper roll of Problem 4-8 as shown in Figure P4-12.
Determine suitable values for dimensions a and b in the figure. Consider bending, shear, and
bearing stresses. Assume an allowable tensile/compressive stress of 100 MPa and an allowable
shear stress of 50 MPa for both stanchion and mandrel, which are steel. The mandrel is solid
and inserts halfway into the paper roll. Balance the design to use all of the material strength.
Calculate the deflection at the end of the roll.

Given: Paper roll dimensions OD  1.50 m Material properties S y  100  MPa


ID  0.22 m S ys  50 MPa
Lroll  3.23 m E  207  GPa
3
Roll density ρ  984  kg m
Assumptions: 1. The paper roll's weight creates a concentrated load acting at the tip of the mandrel.
2. The mandrel's root in the stanchion experiences a distributed load over its length of
engagement

Solution: See Figures 4-27 and Mathcad file P0427. y W


1. In Problem 3-27, we were concerned only with the
portion of the mandrel outside of the stanchion.
Therefore, we modeled it as a cantilever beam with a x
shear and moment reaction at the stanchion.
Unfortunately, this tells us nothing about the stress or
force distributions in the portion of the mandrel that is M1 Lm
inside the stanchion. To do this we need to modify the R1
model by replacing the concentrated moment (and
FIGURE 4-27A
possibly the concentrated shear force) with a force
Free Body Diagram for Problem 3-27
system that will yield information about the stress
distribution in the mandrel on that portion that is inside y
the stanchion. Figure 4-27A shows the FBD used in
Problem 3-27. Figure 4-27B is a simple model, but is not R1 W
representative of a built-in condition. It would be
appropriate if the hole in the stanchion did not fit tightly
around the mandrel. Figure 4-27C is an improvement
that will do for our analysis. x

2. Determine the weight of the roll and the length of the Lm


mandrel. R2

W 
π
4
 2 2 
 OD  ID  Lroll  ρ  g W  53.9 kN FIGURE 4-27B
Simplified Free Body Diagram, not used
Lm  0.5 Lroll Lm  1.615  m

3. From inspection of Figure 4-27C, write the y W


load function equation
w
q(x) = -w<x>0 + w<x - b>0 + R<x - b>-1 - W<x - b -Lm>-1 a
x
4. Integrate this equation from - to x to obtain shear,
V(x)
b Lm
V(x) = -w<x>1 + w<x - b>1 + R<x - b>0 - W<x - b -Lm>0
R

5. Integrate this equation from - to x to obtain


FIGURE 4-27C
moment, M(x)
Free Body Diagram used in Problem 4-27

© 2011 Pearson Education, Inc., Upper Saddle River, NJ. All rights reserved. This publication is protected by Copyright and written permission should be
MACHINE DESIGN - An Integrated Approach, 4th Ed. 4-27-2

M(x) = -(w/2)<x>2 + (w/2)<x - b>2 + R<x - b>1 - W<x - b -Lm>1

6. Solve for the reactions by evaluating the shear and moment equations at a point just to the right of x = b + Lm,
where both are zero.
At x = (b + Lm)+ , V = M = 0

0 = w  b  Lm  w  Lm  R  W R = W  w b

2  W  Lm
0 =    b  Lm   Lm  R Lm =    b  Lm   Lm  ( W  w b )  Lm
w 2 w 2 w 2 w 2
w=
2 2 2 2 2
b
Note that R is inversely proportional to b and w is inversly proportional to b 2.
7. To see the value of x at which the shear and moment are maximum, let
2  W  Lm
b  400  mm then w  and R  W  w b L  b  Lm
2
b

8. Define the range for x x  0  mm 0.002  L  L

9. For a Mathcad solution, define a step function S. This function will have a value of zero when x is less than z,
and a value of one when it is greater than or equal to z.
S ( x z)  if ( x  z 1 0 )
10. Write the shear and moment equations in Mathcad form, using the function S as a multiplying factor to get
the effect of the singularity functions.

V ( x)  w S ( x 0  mm)  x  w S ( x b )  ( x  b )  R S ( x b )  W  S ( x L)


w 2 w 2
M ( x)   S ( x 0  mm)  x   S ( x b )  ( x  b )  R S ( x b )  ( x  b )  W  S ( x L)  ( x  L)
2 2
11. Plot the shear and moment diagrams.

Shear Diagram Moment Diagram


200 50

0
 200
V ( x) M ( x)
kN kN  m
 400
 50

 600

 800  100
0 400 800 1200 1600 2000 0 400 800 1200 1600 2000
x x
mm mm
FIGURE 4-27D
Shear and Moment Diagram Shapes for Problem 4-27

12. From Figure 4-27D, the maximum internal shear and moment occur at x = b and are
2  W  Lm
Vmax = Mmax  W  Lm Mmax  87.04  kN  m
b
© 2011 Pearson Education, Inc., Upper Saddle River, NJ. All rights reserved. This publication is protected by Copyright and written permission should be
MACHINE DESIGN - An Integrated Approach, 4th Ed. 4-27-3

13. The bending stress will be a maximum at the top or bottom of the mandrel at a section through x = b.

Mmax a 4 32 Mmax


π a
σmax = where I= so, σmax = = Sy
2 I 64 3
1 π a
3
 32 W  Lm 
Solving for a, a   π S  a  206.97 mm
 y 

Round this to a  210  mm

14. Using this value of a and equation 4.15c, solve for the shear stress on the neutral axis at x = b.

4  Vmax 8  W  Lm
τmax = = = S ys
3 A  π a 2 
3   b
 4 
8  W  Lm
Solving for b b  b  134.026  mm
 π a 2 
3    Sys
 4 
Round this to b  134  mm

15. These are minimum values for a and b. Using them, check the bearing stress.

2  W  Lm N
Magnitude of distributed load w  w  9695
2 mm
b
w b
Bearing stress σbear  σbear  46.2 MPa
a b

Since this is less than S y, the design is acceptable for a  210  mm and b  134  mm

16. Assume a cantilever beam loaded at the tip with load W and a mandrel diameter equal to a calculated above.
4
π a 7 4
Moment of inertia I  I  9.547  10  mm
64
3
W  Lm
Deflection at tip (Appendix B) ymax   ymax  3.83 mm
3  E I

This can be accomodated by the 220-mm inside diameter of the paper roll.

© 2011 Pearson Education, Inc., Upper Saddle River, NJ. All rights reserved. This publication is protected by Copyright and written permission should be
MACHINE DESIGN - An Integrated Approach, 4th Ed. 4-28-1
PROBLEM 4-28
Statement: Figure P4-13 shows a forklift truck negotiating a 15 deg ramp to to drive onto a 4-ft-high loading
platform. The truck weighs 5 000 lb and has a 42-in wheelbase. Design two (one for each side)
1-ft-wide ramps of steel to have no more than 1-in deflection in the worst case of loading as the
truck travels up them. Minimize the weight of the ramps by using a sensible cross-sectional
geometry.

Given: Ramp angle θ  15 deg Ramp width w  12 in


Platform height h  4  ft Allowable deflection δmax  1.0 in
6
Truck weight W  5000 lbf Young's modulus E  30 10  psi
Truck wheelbase Lt  42 in

Assumptions: 1. The worst case is when the truck CG is located at the center of the beam's span.
2. Use a coordinate frame that has the x-axis along the long axis of the beam.
3. Ignore traction forces and the weight components along the x-axis of the beam.
4. There are two ramps, one for each side of the forklift.

Solution: See Figure 4-28 and Mathcad file P0428.

L
b
a CG a
y
CG b

R1

Fa Fb x
Wa
Wb
R2

FIGURE 4-28A
Dimensions and Free Body Diagram for Problem 4-28

1. Determine the length of the beam between supports and the distances a and b for the worst-case loading.
h
Length of beam L  L  15.455 ft
sin( θ )

From Problem 3-28, a  5.061  ft b  8.561  ft

2. The load distribution of the wheels on a single ramp is given in Problem 3-28 as
Fa  575.0  lbf Fb  1839.9 lbf

3. From inspection of Figure 4-28A, write the load function equation


q(x) = R1<x - 0>-1 - Fa<x - a>-1 - Fb<x - b>-1 + R2<x - L>-1
© 2011 Pearson Education, Inc., Upper Saddle River, NJ. All rights reserved. This publication is protected by Copyright and written permission should be
MACHINE DESIGN - An Integrated Approach, 4th Ed. 4-28-2

4. Integrate this equation from - to x to obtain shear, V(x)

V(x) = R1<x - 0>0 - Fa<x - a>0 - Fb<x - b>0 + R2<x - L>0

5. Integrate this equation from - to x to obtain moment, M(x)

M(x) = R1<x - 0>1 - Fa<x - a>1 - Fb<x - b>1 + R2<x - L>1

6. The reactions are given in Problem 3-28 as R1  1207.4 lbf R2  1207.4 lbf

7. Integrate the moment function, multiplying by 1/EI, to get the slope.


(x) = [R1<x>2/2 - Fa<x - a>2/2 - Fb<x - b>2/2 + R2<x - L>2/2 + C3]/EI

8. Integrate again to get the deflection.


y(x) = [R1<x>3/6 - Fa<x - a>3/6 - Fb<x - b>3/6 + R2<x-L>3/6 + C3x +C4]/EI

9. Evaluate C3 and C4
At x = 0 and x = L, y = 0, therefore, C4 = 0.

3 3 3
0 = R1 L  Fa ( L  a )  Fb ( L  b )  6  C3 L

 R1 L  Fa ( L  a )  Fb ( L  b )


1 3 3 3 6 2
C3   C3  4.983  10  lbf  in
6 L

8. Define the range for x x  0  m 0.005  L  L

9. For a Mathcad solution, define a step function S. This function will have a value of zero when x is less than z,
and a value of one when it is greater than or equal to z.

S ( x z)  if ( x  z 1 0 )

10. Write the slope and deflection equations in Mathcad form, using the function S as a multiplying factor to get
the effect of the singularity functions. Use an assumed value of I so that the value of x that corresponds to ymax
4
can be found. Let I  10 in

1  R1 2 Fa 2 Fb 2 
θ ( x)    S ( x 0  m)  x   S ( x a )  ( x  a )   S ( x b )  ( x  b ) 
E I 2 2 2
 R 
 2  S ( x L)  ( x  L) 2  C3 
 2 

1  R1 3 Fa 3 Fb 3 
y ( x)    S ( x 0  m)  x   S ( x a )  ( x  a )   S ( x b )  ( x  b ) 
E I 6 6 6
 R 
 2  S( x L)  ( x  L) 3  C3 x 
 6 

11. Plot the shear and moment diagrams using the assumed value of I.

© 2011 Pearson Education, Inc., Upper Saddle River, NJ. All rights reserved. This publication is protected by Copyright and written permission should be
MACHINE DESIGN - An Integrated Approach, 4th Ed. 4-28-3

SLOPE, radians DEFLECTION, in


0.02 0

0.01
 0.5
y ( x)
θ( x) 0
in

1
 0.01

 0.02  1.5
0 4 8 12 16 0 4 8 12 16
x x
ft ft

FIGURE 4-28B
Slope and Deflection Diagrams for Problem 4-28, Using an Assumed Value for I

12. Maximum deflection occurs at x = c, where  = 0 and c < b.

1 R1 2 Fa 2 
θ0 = 
c   ( c  a )  C3 = 0
E I  2 2 
Solving for c,
2
R1 Fa a  Fa
A   B  a  Fa C  C3 
2 2 2
4 6 2
A  316.200  lbf B  3.492  10  lbf  in C  6.043  10  in  lbf

2
B  B  4 A  C
c  c  7.804  ft
2 A

 R  c3 Fa 
  ( c  a )  C3 c = δmax
1 1 3
13. The maximum deflection occurs at x = c and is ymax = 
E I  6 6 
Solving for I
 R  c3 
1 Fa
  ( c  a )  C3 c
1 3 4
I   I  10.159 in
E δmax  6 6 

This is the minimum allowable value of the moment of inertia.

14. Assume a channel section such as that shown in Figure 4-28C. To keep it simple, let the thickness of
the flanges and web be the same. Choose 3/8-in thick plate, which is readily available. Then,
t  0.375  in

15. The cross-sectional area of the ramp is A ( h )  w t  2  t ( h  t)

 w t 2 2
16. The distance to the CG is cg( h ) 
1
 2
 t h  t 
A (h)  2 
© 2011 Pearson Education, Inc., Upper Saddle River, NJ. All rights reserved. This publication is protected by Copyright and written permission should be
MACHINE DESIGN - An Integrated Approach, 4th Ed. 4-28-4

17. The moments of inertia of the web and a flange are


Flange
3 2
w t
 w t  cg( h )  
t
Iweb( h )   Web
12  2

3 2 t
t ( h  t) h  t
Ifl ( h )   h  t  cg( h )  
12  2 

18. Using the known moment of inertia, solve


for the unknown flange height, h. Guess h  1  in h
Given
I = Iweb( h )  2  Ifl ( h )

h  Find ( h ) h  3.988  in w
Round this up to h  4.00 in
FIGURE 4-28C
Channel Section for Problem 4-28

19. Summarizing, the ramp design dimensions are:

Length L  185.5  in Flange height h  4.00 in Shape channel


Width w  12.00  in Thickness t  0.375  in Material steel

© 2011 Pearson Education, Inc., Upper Saddle River, NJ. All rights reserved. This publication is protected by Copyright and written permission should be
MACHINE DESIGN - An Integrated Approach, 4th Ed. 4-29a-1
PROBLEM 4-29a
Statement: Find the spring rate of the beam in Problem 4-23 at the applied concentrated load for row a in
Table P4-2.

Given: Beam length L  1  m L


b
Distance to distributed load a  0.4 m
a
Distance to concentrated load b  0.6 m F
w
1
Distributed load magnitude w  200  N  m
Concentrated load Fb  500  N R1 R2
8 4
Moment of inertia I  2.85 10 m
FIGURE 4-29
Modulus of elasticity E  207  GPa
Free Body Diagram for Problem 4-23

Solution: See Figure 4-29 and Mathcad file P0429a.


1. The deflection equation was found in Problem 4-23. Those results are summarized here.
Load function q(x) = R1<x - 0>-1 - w<x - 0>0 + w<x - a>0 - F<x - b>-1 + R2<x - L>-1
Shear function V(x) = R1<x - 0>0 - w<x - 0>1 + w<x - a>1 - F<x - b>0 + R2<x - L>0
Moment function M(x) = R1<x - 0>1 - w<x - 0>2/2 + w<x - a>2/2 - F<x - b>1 + R2<x - L>1
Slope function (x) = [R1<x>2/2 - w<x>3/6 + w<x - a>3/6 - F<x - b>2/2 + R2<x - L>2/2 + C3]/EI
Deflection function y(x) = [R1<x>3/6 - w<x>4/24 + w<x - a>4/24 - F<x - b>3/6 + R2<x-L>3/6 + C3x +C4]/EI
2. To determine the stiffness under the load F we will need to find the incremental beam deflection due to F
alone. The procedure will be to find the deflection at x = b when F = 0, and then find it when Fb  500  N . The
stiffness will then be the force divided by the incremental deflection.

3. For a Mathcad solution, define a step function S. This function will have a value of zero when x is less than z,
and a value of one when it is greater than or equal to z.

S ( x z)  if ( x  z 1 0 )


4. Write the reactions (from Problem 3-23), integration constant, and deflection (from problem 4-23) equations in
Mathcad form, using the function S as a multiplying factor to get the effect of the singularity functions.
w F w 2
R1( F )  L   ( L  b)   ( L  a)
2 L 2 L
R2( F )  w a  F  R1( F )

1  R1( F ) 3 w 4 w 4 F 3
C3( F )    L  L   ( L  a)   ( L  b) 
L  6 24 24 6 
1  R1( F ) 3 w 4 w 4 
y ( x F )    S ( x 0  in)  x   S ( x 0  in)  x   S ( x a )  ( x  a ) 
E I 6 24 24
 R (F ) 
 2  S( x L)  ( x  L) 3   F  S ( x b )  ( x  b) 3  C3( F )  x 
 6 6 
5. The deflection at x = b for F  0  N is y0  y ( b F ) y0  0.137  mm
6. The deflection at x = b for F  Fb is yF  y ( b F ) yF  1.765  mm

7. The deflection due to F alone is ∆y  yF  y0 ∆y  1.627  mm


F N
8. The stiffness of the beam under the load F at x = b is k  k  307 
∆y mm
© 2011 Pearson Education, Inc., Upper Saddle River, NJ. All rights reserved. This publication is protected by Copyright and written permission should be
MACHINE DESIGN - An Integrated Approach, 4th Ed. 4-30a-1
PROBLEM 4-30a
Statement: Find the spring rate of the beam in Problem 4-24 at the applied concentrated load for row a in
Table P4-2.

Given: Beam length L  1  m L


Distance to distributed load a  0.4 m a
1
Distributed load magnitude w  200  N  m F
w
Concentrated load FL  500  N
8 4
Moment of inertia I  2.85 10 m
M1
Modulus of elasticity E  207  GPa
R1
Solution: See Figure 4-30 and Mathcad file P0430a.
FIGURE 4-30
1. The deflection equation was found in Problem 4-24. Free Body Diagram for Problem 4-24
Those results are summarized here.
Load function q(x) = -M1<x - 0>-2 + R1<x - 0>-1 - w<x - a>0 - F<x - L>-1
Shear function V(x) = -M1<x - 0>-1 + R1<x - 0>0 - w<x - a>1 - F<x - L>0
Moment function M(x) = -M1<x - 0>0 + R1<x - 0>1 - w<x - a>2/2 - F<x - L>1
Slope function (x) = [-M1<x-0>1 + R1<x - 0>2/2 - w<x - a>3/6 - F<x - L>2/2 + C3]/EI
Deflection function y(x) = [-M1<x-0>2/2 + R1<x - 0>3/6 - w<x - a>4/24 - F<x - L>3/6 + C3x +C4]/EI

2. To determine the stiffness under the load F we will need to find the incremental beam deflection due to
F alone. The procedure will be to find the deflection at x = L when F = 0, and then find it when FL  500  N
. The stiffness will then be the force divided by the incremental deflection.

3. For a Mathcad solution, define a step function S. This function will have a value of zero when x is less than z,
and a value of one when it is greater than or equal to z.

S ( x z)  if ( x  z 1 0 )


4. Write the reaction (from Problem 3-24) and deflection (from problem 4-24) equations in Mathcad form, using
the function S as a multiplying factor to get the effect of the singularity functions.

R1( F )  w ( L  a )  F
w 2
M1( F )   ( L  a )  R 1( F )  L
2

1  M1( F ) 2 R1( F ) 3 w 4 
y ( x F )     S ( x 0  in)  x   S ( x 0  in)  x   S ( x a )  ( x  a ) 
E I 2 6 24
 F

   S( x L)  ( x  L) 3 
 6 
5. The deflection at x = L for F  0  N is y0  y ( L F ) y0  3.912  mm

6. The deflection at x = L for F  FL is yF  y ( L F ) yF  32.163 mm

7. The deflection due to F alone is ∆y  yF  y0 ∆y  28.251 mm

F N
8. The stiffness of the beam under the load F at x = L is k  k  17.7
∆y mm

© 2011 Pearson Education, Inc., Upper Saddle River, NJ. All rights reserved. This publication is protected by Copyright and written permission should be
MACHINE DESIGN - An Integrated Approach, 4th Ed. 4-31a-1
PROBLEM 4-31a
Statement: Find the spring rate of the beam in Problem 4-25 at the applied concentrated load for row a in
Table P4-2.

Given: Beam length L  1  m L


b
Distance to distributed load a  0.4 m
a F
Distance to concentrated load b  0.6 m
w
1
Distributed load magnitude w  200  N  m
Concentrated load FL  500  N R1 R2
8 4
Moment of inertia I  2.85 10 m
FIGURE 4-31
Modulus of elasticity E  207  GPa
Free Body Diagram for Problem 4-25

Solution: See Figure 4-31 and Mathcad file P0431a.

1. The deflection equation was found in Problem 4-25. Those results are summarized here.
Load function q(x) = R1<x - 0>-1 - w<x - a>0 + R2<x - b>-1 - F<x - L>-1
Shear function V(x) = R1<x - 0>0 - w<x - a>1 + R2<x - b>0 - F<x - L>0
Moment function M(x) = R1<x - 0>1 - w<x - a>2/2 + R2<x - b>1 - F<x - L>1
Slope function (x) = [R1<x - 0>2/2 - w<x - a>3/6 + R2<x - b>2/2 - F<x - L>2/2 + C3]/EI
Deflection function y(x) = [R1<x - 0>3/6 - w<x - a>4/24 + R2<x-b>3/6 - F<x - L>3/6 + C3x +C4]/EI

2. To determine the stiffness under the load F we will need to find the incremental beam deflection due to
F alone. The procedure will be to find the deflection at x = L when F = 0, and then find it when
FL  500  N . The stiffness will then be the force divided by the incremental deflection.

3. For a Mathcad solution, define a step function S. This function will have a value of zero when x is less than z,
and a value of one when it is greater than or equal to z.

S ( x z)  if ( x  z 1 0 )


4. Write the reactions (from Problem 3-25), integration constant, and deflection (from problem 4-25) equations in
Mathcad form, using the function S as a multiplying factor to get the effect of the singularity functions.

   ( L  a )  F  ( L  b )  w ( L  a )  ( L  b )
1 w 2
R1( F ) 
b 2 
R2( F )  w ( L  a )  F  R1( F )

1  R1( F ) 3 w 4
C3( F )     b   ( b  a) 
b  6 24 

1  R1( F ) 3 w 4 
y ( x F )    S ( x 0  in)  x   S ( x a )  ( x  a )  
E I 6 24
 R (F ) 
 2  S( x b)  ( x  b ) 3   F  S ( x L)  ( x  L) 3  C3( F )  x 
 6 6 
5. The deflection at x = L for F  0  N is y0  y ( L F ) y0  0.288  mm

6. The deflection at x = L for F  FL is yF  y ( L F ) yF  4.808  mm

© 2011 Pearson Education, Inc., Upper Saddle River, NJ. All rights reserved. This publication is protected by Copyright and written permission should be
MACHINE DESIGN - An Integrated Approach, 4th Ed. 4-31a-2

7. The deflection due to F alone is ∆y  yF  y0 ∆y  4.52 mm

F N
8. The stiffness of the beam under the load F at x = L is k  k  111 
∆y mm

© 2011 Pearson Education, Inc., Upper Saddle River, NJ. All rights reserved. This publication is protected by Copyright and written permission should be
MACHINE DESIGN - An Integrated Approach, 4th Ed. 4-32a-1
PROBLEM 4-32a
Statement: Find the spring rate of the beam in Problem 4-26 at the applied concentrated load for row a in
Table P4-2.

Given: Beam length L  1  m L


b
Distance to distributed load a  0.4 m
a F
Distance to concentrated load b  0.6 m
w
1
Distributed load magnitude w  200  N  m
Concentrated load Fa  500  N R1 R2 R3
8 4
Moment of inertia I  2.85 10 m
FIGURE 4-32
Modulus of elasticity E  207  GPa
Free Body Diagram for Problem 4-26
Solution: See Figure 4-32 and Mathcad file P0432a.

1. The deflection equation was found in Problem 4-26. Those results are summarized here.
Load function q(x) = R1<x>-1 - F<x - a>-1 - w<x - a>0 + R2<x - b>-1 - R3<x - L>-1
Shear function V(x) = R1<x>0 - F<x - a>0 - w<x - a>1 + R2<x - b>0 - R3<x - L>0
Moment function M(x) = R1<x>1 - F<x - a>1 - w<x - a>2/2 + R2<x - b>1 - R3<x - L>1
Slope function (x) = [R1<x>2/2 - F<x - a>2/2 - w<x - a>3/6 + R2<x - b>2/2 + R3<x - L>2/2 + C3]/EI
Deflection function y(x) = [R1<x>3/6 - F<x - a>3/6 - w<x - a>4/24 + R2<x - b>3/6 + R3<x - L>3/6 + C3x + C4]/EI

2. To determine the stiffness under the load F we will need to find the incremental beam deflection due to
F alone. The procedure will be to find the deflection at x = a when F = 0, and then find it when Fa  500  N
. The stiffness will then be the force divided by the incremental deflection.

3. For a Mathcad solution, define a step function S. This function will have a value of zero when x is less than z,
and a value of one when it is greater than or equal to z.

S ( x z)  if ( x  z 1 0 )


4. Write the reactions, integration constant, and deflection (from problem 4-26) equations in Mathcad form,
using the function S as a multiplying factor to get the effect of the singularity functions.

F 3 w 4 F 3 w 4
Let f1 ( F )   ( b  a)   ( b  a) f2 ( F )   ( L  a)   ( L  a)
6 24 6 24

w 2
f3 ( F )  F  ( L  a )   ( L  a)
2

then
 L ( L  b)
2 
R1 Fa  112.333  N
3
R1( F )    f1 ( F )  f2 ( F )   f3 ( F )
L b  ( L  b ) b 6 

  f3 ( F )  L R1( F )  R2 Fa  559.167  N


1
R2( F ) 
( L  b)

R3( F )  F  w ( L  a )  R1( F )  R2( F ) R3 Fa  51.500 N


2
1 b
C3( F )   f1 ( F )   R 1( F )
b 6

© 2011 Pearson Education, Inc., Upper Saddle River, NJ. All rights reserved. This publication is protected by Copyright and written permission should be
MACHINE DESIGN - An Integrated Approach, 4th Ed. 4-32a-2

1  R1( F ) 3 F 3 w 4 
y ( x F )    S ( x 0  in)  x   S ( x a )  ( x  a )   S ( x a )  ( x  a ) 
E I 6 6 24
 R (F ) R (F ) 
 2  S( x b)  ( x  b ) 3  3  S ( x L)  ( x  L) 3  C3( F )  x 
 6 6 

5. The deflection at x = a for F  0  N is y0  y ( a F ) y0  0.00126  mm

6. The deflection at x = a for F  Fa is yF  y ( a F ) yF  0.177  mm

7. The deflection due to F alone is ∆y  yF  y0 ∆y  0.176  mm

F N
8. The stiffness of the beam under the load F at x = a is k  k  2844
∆y mm

© 2011 Pearson Education, Inc., Upper Saddle River, NJ. All rights reserved. This publication is protected by Copyright and written permission should be
MACHINE DESIGN - An Integrated Approach, 4th Ed. 4-33a-1
PROBLEM 4-33a
Statement: For the bracket shown in Figure P4-14 and the data in row a of Table P4-3, determine the
bending stress at point A and the shear stress due to transverse loading at point B. Also the
torsional shear stress at both points. Then determine the principal stresses at points A and B.

Given: Tube length L  100  mm F


y
Arm length a  400  mm
A
Arm thickness t  10 mm
B
Arm depth h  20 mm T T x

Applied force F  50 N M


Tube OD OD  20 mm L
R
Tube ID ID  14 mm
Modulus of elasticity E  207  GPa FIGURE 4-33
Free Body Diagram of Tube for Problem 4-33

Solution: See Figure 4-33 and Mathcad file P0433a.


1. Determine the bending stress at point A. From the FBD of the tube in Figure 4-33 we see that
Reaction force R  F R  50.0 N
Reaction moment M  F  L M  5.00 N  m
Distance from NA
to outside of tube ct  0.5 OD ct  10.0 mm

Moment of inertia It 
π
64
 OD  ID
4 4  It  5968 mm
4

Bending stress M  ct
at point A σxA  σxA  8.38 MPa
It

2. Determine the shear stress due to transverse loading at B.

Cross-section area A 
π
4
  OD  ID
2 
2
A  160.2  mm
2

Maximum shear V  R
Maximum shear stress V
(Equation 4.15d) τVmax  2  τVmax  0.624  MPa
A

3. Determine the torsional shear stress at both points. Using equation 4.23b and the FBD above
Torque on tube T  F  a T  20.0 N  m

Polar moment of
inertia J 
π
32
 OD  ID
4 4  J  11936  mm
4

Maximum torsional T  ct
stress at surface τTmax  τTmax  16.76  MPa
J

4. Determine the principal stress at point A.


Stress components σxA  8.378  MPa σzA  0  MPa
τxz  τTmax τxz  16.76  MPa
© 2011 Pearson Education, Inc., Upper Saddle River, NJ. All rights reserved. This publication is protected by Copyright and written permission should be
MACHINE DESIGN - An Integrated Approach, 4th Ed. 4-33a-2

Principal stresses

2
σxA  σzA  σxA  σzA  2
σ1      τxz σ1  21.46  MPa
2  2 

σ2  0  MPa

2
σxA  σzA  σxA  σzA  2
σ3      τxz σ3  13.08  MPa
2  2 
σ1  σ3
τ13  τ13  17.27  MPa
2

5. Determine the principal stress at point B.


Stress components σxB  0  MPa σyB  0  MPa

τxy  τTmax  τVmax τxy  16.13  MPa

Principal stresses

2
σxB  σyB  σxB  σyB  2
σ1      τxy σ1  16.13  MPa
2  2 

σ2  0  MPa

2
σxB  σyB  σxB  σyB  2
σ3      τxy σ3  16.13  MPa
2  2 
σ1  σ3
τ13  τ13  16.13  MPa
2

© 2011 Pearson Education, Inc., Upper Saddle River, NJ. All rights reserved. This publication is protected by Copyright and written permission should be
MACHINE DESIGN - An Integrated Approach, 4th Ed. 4-34a-1
PROBLEM 4-34a
Statement: For the bracket shown in Figure P4-14 and the data in row a of Table P4-3, determine the
deflection at load F.
Given: Tube length L  100  mm Applied force F  50 N
Arm length a  400  mm Tube OD OD  20 mm
Arm thickness t  10 mm Tube ID ID  14 mm
Arm depth h  20 mm Modulus of elasticity E  207  GPa
Modulus of rigidity G  80.8 GPa
Solution: See Figure 4-34 and Mathcad file P0434a.
1. The deflection at load F can be determined by superimposing the rigid-body deflection of the arm due to the
twisting of the tube with the beam deflection of the tube and the arm alone.
2. Determine the rigid-body deflection due to twisting of the tube. Refering to Figure 4-34, the torque in the
tube is
Torque on tube T  F  a T  20.0 N  m

Polar moment of inertia Jt 


π
32
  OD  ID
4

4
Jt  11936  mm
4

TL 3
Tube angle of twist θ  θ  2.07368  10  rad
J t G
θ  0.119  deg
Deflection at F due to  δθ  a  θ δθ  0.829  mm
3. Determine the rigid-body deflection due to bending of the tube.
Jt 4
Moment of inertia It  It  5968 mm
2
Deflection of tube 3
F L
end and arm end δtb  δtb  0.013  mm
(see Appendix B) 3  E It

F F
y y a
A
B x z
T T h
M T

L
R F

FIGURE 4-34
Free Body Diagrams of Tube and Arm for Problem 4-34
4. Determine the beam bending of arm alone.
3
t h 4
Moment of inertia Ia  Ia  6667 mm
12
3
F a
Deflection at F δa  δa  0.773  mm
3  E Ia

5. Determine the total deflection by superposition.


δtot  δθ  δtb  δa δtot  1.616  mm downward
© 2011 Pearson Education, Inc., Upper Saddle River, NJ. All rights reserved. This publication is protected by Copyright and written permission should be
MACHINE DESIGN - An Integrated Approach, 4th Ed. 4-35a-1
PROBLEM 4-35a
Statement: For the bracket shown in Figure P4-14 and the data in row a of Table P4-3, determine the spring
rate of the tube in bending, the spring rate of the arm in bending, and the spring rate of the tube
in torsion. Combine these into an overall spring rate in terms of the force F and the linear
deflection at F.

Given: Tube length L  100  mm Applied force F  50 N


Arm length a  400  mm Tube OD OD  20 mm
Arm thickness t  10 mm Tube ID ID  14 mm
Arm depth h  20 mm Modulus of elasticity E  207  GPa
Modulus of rigidity G  80.8 GPa
Solution: See Figure 4-35 and Mathcad file P0435a.
1. Determine the spring rate due to bending of the tube.

Moment of inertia It 
π
64
  OD  ID
4 4  It  5968 mm
4

Deflection of tube 3
F L
end and arm end δtb  δtb  0.013  mm
(see Appendix B) 3  E It

Spring rate due to F N


bending in tube ktb  ktb  3706
δtb mm

2. Determine the spring rate due to beam bending of arm alone.


3
t h 4
Moment of inertia Ia  Ia  6667 mm
12
3
F a
Deflection at F δa  δa  0.773  mm
3  E Ia

Spring rate due to F N


bending in arm ka  ka  64.7
δa mm

F F
y y a
A
B x z
T T h
M T

L
R F

FIGURE 4-35
Free Body Diagrams of Tube and Arm for Problem 4-35

3. Determine the spring rate of the tube in torsion. Refering to Figure 4-35, the torque in the tube is
Torque on tube T  F  a T  20.0 N  m
Polar moment of inertia Jt 
π
32
  OD  ID
4 4 Jt  11936  mm
4

© 2011 Pearson Education, Inc., Upper Saddle River, NJ. All rights reserved. This publication is protected by Copyright and written permission should be
MACHINE DESIGN - An Integrated Approach, 4th Ed. 4-35a-2

TL 3
Tube angle of twist θ  θ  2.07368  10  rad
J t G
θ  0.119  deg

Deflection at F due to q δθ  a  θ δθ  0.829  mm

Spring rate due to F N


torsion in tube kθ  kθ  60.28 
δθ mm

1 1 1 1
4. Determine the overall spring rate. The springs are in series, thus =  
koa kθ ktb ka

kθ ktb ka N
koa  koa  30.9
ktb ka  kθ ka  kθ ktb mm

F
Checking, δtot  δtot  1.616  mm
koa

which is the same total deflection gotten in Problem 4-34.

© 2011 Pearson Education, Inc., Upper Saddle River, NJ. All rights reserved. This publication is protected by Copyright and written permission should be
MACHINE DESIGN - An Integrated Approach, 4th Ed. 4-36a-1
PROBLEM 4-36a
Statement: For the bracket shown in Figure P4-14 and the data in row a of Table P4-3, redo Problem 4-33
considering the stress concentration at points A and B. Assume a stress concentration factor
of 2.5 in both bending and torsion.

Given: Tube length L  100  mm


F
Arm length a  400  mm y

Arm thickness t  10 mm A


Arm depth h  20 mm B x
T T
Applied force F  50 N
M
Tube OD OD  20 mm
L
Tube ID ID  14 mm R
Modulus of elasticity E  207  GPa
FIGURE 4-36
Stress-concentration Ktb  2.5 Free Body Diagram of Tube for Problem 4-36
factors
Kts  2.5
Solution: See Figure 4-36 and Mathcad file P0436a.
1. Determine the bending stress at point A. From the FBD of the tube in Figure 4-36 we see that
Reaction force R  F R  50.0 N
Reaction moment M  F  L M  5.00 N  m
Distance from NA
to outside of tube ct  0.5 OD ct  10.0 mm

Moment of inertia It 
π
64
 4
 OD  ID
4  It  5968 mm
4

Bending stress M  ct
at point A σxA  Ktb σxA  20.94  MPa
It

2. Determine the shear stress due to transverse loading at B.

Cross-section area A 
π
4
 2
 OD  ID
2  A  160.2  mm
2

Maximum shear V  R
Maximum shear stress V
(Equation 4.15d) τVmax  2  τVmax  0.624  MPa
A

3. Determine the torsional shear stress at both points. Using equation 4.23b and the FBD above
Torque on tube T  F  a T  20.0 N  m

Polar moment of
inertia J 
π
32
 4
 OD  ID
4  J  11936  mm
4

Maximum torsional T  ct
stress at surface τTmax  Kts τTmax  41.89  MPa
J

4. Determine the principal stress at point A.


Stress components σxA  20.944 MPa σzA  0  MPa
© 2011 Pearson Education, Inc., Upper Saddle River, NJ. All rights reserved. This publication is protected by Copyright and written permission should be
MACHINE DESIGN - An Integrated Approach, 4th Ed. 4-36a-2

τxz  τTmax τxz  41.89  MPa


Principal stresses

2
σxA  σzA  σxA  σzA  2
σ1      τxz σ1  53.6 MPa
2  2 

σ2  0  MPa

2
σxA  σzA  σxA  σzA  2
σ3      τxz σ3  32.71  MPa
2  2 
σ1  σ3
τ13  τ13  43.18  MPa
2

5. Determine the principal stress at point B.


Stress components σxB  0  MPa σyB  0  MPa

τxy  τTmax  τVmax τxy  41.26  MPa

Principal stresses

2
σxB  σyB  σxB  σyB  2
σ1      τxy σ1  41.26  MPa
2  2 

σ2  0  MPa

2
σxB  σyB  σxB  σyB  2
σ3      τxy σ3  41.26  MPa
2  2 
σ1  σ3
τ13  τ13  41.26  MPa
2

© 2011 Pearson Education, Inc., Upper Saddle River, NJ. All rights reserved. This publication is protected by Copyright and written permission should be
MACHINE DESIGN - An Integrated Approach, 4th Ed. 4-37-1
PROBLEM 4-37
Statement: A semicircular, curved beam as shown in Figure 4-37 has the dimensions given below. For the
load pair applied along the diameter and given below, find the eccentricity of the neutral axis
and the stress at the inner and outer fibers.
w
Given: Outside diameter od  150  mm
Inside diameter id  100  mm
Width of beam w  25 mm
Load F  14 kN F
od id
Solution: See Figure 4-37 and Mathcad file P0437. F

1. Calculate the section depth, area, inside radius and


outside radus.
od  id (a) Entire Beam
Section depth h  h  25 mm
2
2
Area of section A  h  w A  625  mm
od  id
Centroid radius rc  rc  62.5 mm F
4
M
Inside and outside ri  rc  0.5 h ri  50 mm
F
radii of section rc
ro  rc  0.5 h ro  75 mm
(b) Critical Section
2. The critical section is the one that is along the horizontal
FIGURE 4-37
centerline. There, the bending moment is
Free Body Diagrams for Problem 4-37
Bending moment M  F  rc M  0.875  kN  m

3. Use the equation in the footnote of the text to calculate the radius of the neutral axis.
ro  ri
Radius of neutral axis rn  rn  61.658 mm
 ro 
ln 
 ri 
4. Calculate the eccentricty and the distances from the neutral axis to the extreme fibers.
Eccentricity e  rc  rn e  0.8424 mm

Distances from neutral ci  rn  ri ci  11.66  mm


axis to extreme fibers
co  ro  rn co  13.34  mm

M ci F
Stresses at inner and σi    σi  409.9  MPa
outer radii e A ri A

 M co  F
σo     σo  273.2  MPa
 e A ro  A

© 2011 Pearson Education, Inc., Upper Saddle River, NJ. All rights reserved. This publication is protected by Copyright and written permission should be
MACHINE DESIGN - An Integrated Approach, 4th Ed. 4-38-1
PROBLEM 4-38
Statement: Design a solid, straight, steel torsion bar to have a spring rate of 10 000 in-lb per radian per foot
of length. Compare designs of solid round and solid square cross-sections. Which is more
efficient in terms of material use?
6
Given: Length of rod L  12 in Modulus of rigidity G  11.7 10  psi
in lbf
Spring rate k  10000 
rad

Solution: See Mathcad file P0438.


1. Determine the rod diameter and volume for a round rod.
4
J G π d
Spring rate k= J =
L 32
1
4
 32 L k 
Rod diameter d   π G  d  0.569  in
 
2
π d 3
Volume of rod V  L V  3.046  in
4

2. Determine the rod width and volume for a square rod using equation 4.26b and Table 4-6.
K G 4
Spring rate k= K = 2.25 a
L
1
4
 L k 
Rod half-width a   2.25 G  a  0.260  in 2  a  0.520  in
 
2 3
Volume of rod V  ( 2  a )  L V  3.241  in

3. Even though the square rod width is less than the round rod diameter, it takes slightly more material when a
square rod is used than when a round rod is used. Thus, the round rod is more efficient.

© 2011 Pearson Education, Inc., Upper Saddle River, NJ. All rights reserved. This publication is protected by Copyright and written permission should be
MACHINE DESIGN - An Integrated Approach, 4th Ed. 4-39-1
PROBLEM 4-39
Statement: Design a 1-ft-long steel, end-loaded cantilever spring for a spring rate of 10 000 lb/in. Compare
designs of solid round and solid square cross-sections. Which is more efficient in terms of
material use?
6
Given: Length of rod L  12 in Modulus of rigidity E  30 10  psi
lbf
Spring rate k  10000 
in

Solution: See Figure B-1(a) in Appendix B and Mathcad file P0439.

1. Determine the rod diameter and volume for a round rod.


4
3  E I π d
Spring rate k= I=
3 64
L
1
4
 64 L3 k 
Rod diameter d    d  1.406  in
 3  π E 
2
π d 3
Volume of rod V  L V  18.64  in
4
2. Determine the rod width and volume for a square rod using equation 4.26b and Table 4-6.
4
3  E I a
Spring rate k= I=
3 12
L 1
4
 4 L3 k 
Rod width a    a  1.232  in
 E 
2 3
Volume of rod V  a  L V  18.215 in

3. It takes slightly more material when a round rod is used than when a square rod is used. Thus, the square rod
is more efficient.

© 2011 Pearson Education, Inc., Upper Saddle River, NJ. All rights reserved. This publication is protected by Copyright and written permission should be
MACHINE DESIGN - An Integrated Approach, 4th Ed. 4-40-1
PROBLEM 4-40
Statement: Redesign the roll support of Problem 4-8 to be like that shown in Figure P4-16. The stub
mandrels insert to 10% of the roll length at each end. Choose appropriate dimensions a and
b to fully utilize the mandrel's strength, which is the same as in Problem 4-27. See Problem
4-8 for additional data.

Given: Paper roll dimensions OD  1.50 m Material properties S y  100  MPa


ID  0.22 m S ys  50 MPa
Lroll  3.23 m E  207  GPa
3
Roll density ρ  984  kg m
Assumptions: 1. The paper roll's weight creates a concentrated load acting at the tip of the mandrel.
2. The mandrel's root in the stanchion experiences a distributed load over its length of
engagement
Solution: See Figures 4-40 and Mathcad file P0440.
1. Model the support in such a way that stresses in the
portion of the mandrel that is inside the stanchion can y F
be determined. There are several assumptions that can w
be made about the loads on this portion of the mandrel. a
Figure 4-40A shows the one that will be used for this x
design.

2. Determine the weight of the roll, the load on each b Lm


support, and the length of the mandrel.
R

W 
π
4
 2 2 
 OD  ID  Lroll  ρ  g W  53.9 kN FIGURE 4-40A
Free Body Diagram used in Problem 4-40

F  0.5 W F  26.95  kN
Lm  0.1 Lroll Lm  323  mm

3. From inspection of Figure 4-40A, write the load function equation


q(x) = -w<x>0 + w<x - b>0 + R<x - b>-1 - F<x - b -Lm>-1

4. Integrate this equation from - to x to obtain shear, V(x)


V(x) = -w<x>1 + w<x - b>1 + R<x - b>0 - F<x - b -Lm>0

5. Integrate this equation from - to x to obtain moment, M(x)


M(x) = -(w/2)<x>2 + (w/2)<x - b>2 + R<x - b>1 - F<x - b -Lm>1

6. Solve for the reactions by evaluating the shear and moment equations at a point just to the right of x = b + Lm,
where both are zero.
At x = (b + Lm)+ , V = M = 0

0 = w  b  Lm  w  Lm  R  F R = F  w b

2  F  Lm
0 =    b  Lm   Lm  R Lm =    b  Lm   Lm  ( F  w b )  Lm
w 2 w 2 w 2 w 2
w=
2 2 2 2 2
b
Note that R is inversely proportional to b and w is inversly proportional to b 2.

7. To see the value of x at which the shear and moment are maximum, let
© 2011 Pearson Education, Inc., Upper Saddle River, NJ. All rights reserved. This publication is protected by Copyright and written permission should be
MACHINE DESIGN - An Integrated Approach, 4th Ed. 4-40-2

2  F  Lm
b  200  mm then w  and R  F  w b L  b  Lm
2
b

8. Define the range for x x  0  mm 0.002  L  L

9. For a Mathcad solution, define a step function S. This function will have a value of zero when x is less than z,
and a value of one when it is greater than or equal to z.
S ( x z)  if ( x  z 1 0 )
10. Write the shear and moment equations in Mathcad form, using the function S as a multiplying factor to get the
effect of the singularity functions.

V ( x)  w S ( x 0  mm)  x  w S ( x b )  ( x  b )  R S ( x b )  F  S ( x L)


w 2 w 2
M ( x)   S ( x 0  mm)  x   S ( x b )  ( x  b )  R S ( x b )  ( x  b )  F  S ( x L)  ( x  L)
2 2

11. Plot the shear and moment diagrams.


Shear Diagram Moment Diagram

200 2

100 1

V ( x) M ( x)
0 4
kN kN  m

 100 7

 200  10
0 100 200 300 400 500 600 0 100 200 300 400 500 600
x x
mm mm
FIGURE 4-40B
Shear and Moment Diagram Shapes for Problem 4-40

12. From Figure 4-40B, the maximum internal shear and moment occur at x = b and are

2  F  Lm
Vmax = Mmax  F  Lm Mmax  8.704  kN  m
b

13. The bending stress will be a maximum at the top or bottom of the mandrel at a section through x = b.

Mmax a 4 32 Mmax


π a
σmax = where I= so, σmax = = Sy
2 I 64 3
1 π a
3
 32 W  Lm 
Solving for a, a   π S  a  121.037  mm
 y 

Round this to a  125  mm

© 2011 Pearson Education, Inc., Upper Saddle River, NJ. All rights reserved. This publication is protected by Copyright and written permission should be
MACHINE DESIGN - An Integrated Approach, 4th Ed. 4-40-3

14. Using this value of a and equation 4.15c, solve for the shear stress on the neutral axis at x = b.

4  Vmax 8  F  Lm
τmax = = = S ys
3 A  π a 2 
3   b
 4 
8  F  Lm
Solving for b b  b  37.828 mm
 π a 2 
3    Sys
 4 
Round this to b  38 mm

15. These are minimum values for a and b. Using them, check the bearing stress.

2  F  Lm N
Magnitude of distributed load w  w  12055 
2 mm
b
w b
Bearing stress σbear  σbear  96.4 MPa
a b

Since this is less than S y, the design is acceptable for a  125  mm and
b  38 mm

© 2011 Pearson Education, Inc., Upper Saddle River, NJ. All rights reserved. This publication is protected by Copyright and written permission should be
MACHINE DESIGN - An Integrated Approach, 4th Ed. 4-41-1
PROBLEM 4-41
Statement: A 10-mm ID steel tube carries liquid at 7 MPa. Determine the principal stresses in the wall if its
thickness is: a) 1 mm, b) 5 mm.
Given: Tubing ID ID  10 mm Inside pressure p i  7  MPa

Assumption: The tubing is long therefore the axial stress is zero.


Solution: See Mathcad file P0441.
(a) Wall thickness is t  1  mm
1. Check wall thickness to radius ratio to see if this is a thick or thin wall problem.
t
ratio  ratio  0.2
0.5 ID

Since the ratio is greater than 0.1, this is a thick wall problem.
2. Using equations 4.48a and 4.48b, the stresses are maximum at the inside wall where
Inside radius ri  0.5 ID ri  5  mm

Outside radius ro  ri  t ro  6  mm

2 
ri  p i ro 
2
Tangential stress σt  
 1  σt  38.82  MPa
2 2  2
ro  ri  ri 

2
ri  p i  2
1  
ro
Radial stress σr  σr  7.00 MPa
2  2 2
ro  ri  ri 
3. Determine the principal stresses (since, for this choice of coordinates, the shear stress is zero),
σ1  σt σ1  38.82  MPa

σ2  0  MPa

σ3  σr σ3  7.00 MPa

The maximum shear stress is


σ1  σ3
τmax  τmax  22.91  MPa
2

(b) Wall thickness is t  5  mm

1. Check wall thickness to radius ratio to see if this is a thick or thin wall problem.
t
ratio  ratio  1
0.5 ID

Since the ratio is greater than 0.1, this is a thick wall problem.
2. Using equations 4.48a and 4.48b, the stresses are maximum at the inside wall where

Inside radius ri  0.5 ID ri  5  mm


Outside radius ro  ri  t ro  10 mm
© 2011 Pearson Education, Inc., Upper Saddle River, NJ. All rights reserved. This publication is protected by Copyright and written permission should be
MACHINE DESIGN - An Integrated Approach, 4th Ed. 4-41-2

2 
ri  p i ro 
2
Tangential stress σt  
 1  σt  11.67  MPa
2 2  2
ro  ri  ri 

2 
ri  p i ro 
2
Radial stress σr  
 1  σr  7.00 MPa
2 2  2
ro  ri  ri 

3. Determine the principal stresses (since, for this choice of coordinates, the shear stress is zero),
σ1  σt σ1  11.67  MPa

σ2  0  MPa

σ3  σr σ3  7.00 MPa

The maximum shear stress is


σ1  σ3
τmax  τmax  9.33 MPa
2

© 2011 Pearson Education, Inc., Upper Saddle River, NJ. All rights reserved. This publication is protected by Copyright and written permission should be
MACHINE DESIGN - An Integrated Approach, 4th Ed. 4-42-1
PROBLEM 4-42
Statement: A cylindrical tank with hemispherical ends is required to hold 150 psi of pressurized air at room
temperature. Find the principal stresses in the 1-mm-thick wall if the tank diameter is 0.5 m and
its length is 1 m.

Given: Tank ID ID  500  mm


Wall thickness t  1  mm
Inside pressure p i  150  psi p i  1034 kPa
Solution: See Mathcad file P0442.
1. Check wall thickness to radius ratio to see if this is a thick or thin wall problem.
t 3
ratio  ratio  4  10
0.5 ID

Since the ratio is less than 0.1, this is a thin wall problem.

2. Using equations 4.49a, 4.49b and 4.49c, the stresses are


Radius r  0.5 ID r  250  mm

pi  r
Tangential stress σt  σt  258.55 MPa
t

Radial stress σr  0  MPa σr  0.00 MPa

pi  r
Axial stress σa  σa  129.28 MPa
2 t

3. Determine the principal stresses (since, for this choice of coordinates, the shear stress is zero),

σ1  σt σ1  259  MPa σ1  37.5 ksi

σ2  σa σ2  129  MPa σ2  18.75  ksi

σ3  0  MPa σ3  0.00 MPa σ3  0.00 MPa

The maximum shear stress is

σ1  σ3
τmax  τmax  129  MPa
2

© 2011 Pearson Education, Inc., Upper Saddle River, NJ. All rights reserved. This publication is protected by Copyright and written permission should be
MACHINE DESIGN - An Integrated Approach, 4th Ed. 4-43-1

PROBLEM 4-43
Statement: Figure P4-17 shows an off-loading station at the end of a paper rolling machine. The finished
paper rolls are 0.9-m OD by 0.22-m ID by 3.23-m long and have a density of 984 kg/m3. The
rolls are transfered from the machine conveyor (not shown) to the forklift truck by the
V-linkage of the off-load station, which is rotated through 90 deg by an air cylinder. The paper
then rolls onto the waiting forks of the truck. The forks are 38-mm thick by 100-mm wide by
1.2-m long and are tipped at a 3-deg angle from the horizontal. Find the stresses in the two
forks on the truck when the paper rolls onto it under two different conditions (state all
assumptions):
(a) The two forks are unsupported at their free end.
(b) The two forks are contacting the table at point A.
Given: Paper roll dimensions OD  0.90 m Fork dimensions t  38 mm
ID  0.22 m w  100  mm
Lroll  3.23 m Lfork  1200 mm
3
Roll density ρ  984  kg m θfork  3  deg

Assumptions: 1. The greatest bending moment will occur when the paper roll is at the tip of the fork for case (a)
and when it is midway between supports for case (b).
2. Each fork carries 1/2 the weight of a paper roll.
3. For case (a), each fork acts as a cantilever beam (see Appendix B-1(a)).
4. For case (b), each fork acts as a beam that is built-in at one end and simply-supported at the
other.
Solution: See Figure 4-43 and Mathcad file P0443.

F
L fork
1. Determine the weight of the roll and the load on each
t
fork.

W 
π
4
 2 2 
 OD  ID  Lroll  ρ  g W  18.64  kN
M1
R1
F  0.5 W F  9.32 kN
Case (a), Cantilever Beam
2. The moment of inertia and the distance to the extreme
fiber for a fork are
F
3 0.5 L fork
w t 5 4
I  I  4.573  10  mm t
12
t
c  c  19 mm
2 L fork
M2
R1 R2
Case (a)
Case (b), Fixed-Simply Supported Beam
3. From Figure D-1(a), the moment is a maximum at the
support and is FIGURE 4-43A
Free Body Diagrams used in Problem 4-43
Mmax  F  Lfork Mmax  11.186 kN  m

Mmax c
4. The bending stress is maximum at the support and is σa  σa  464.8  MPa
I
Case (b)
5. This beam is statically indeterminate. However, using singularity functions and the method shown in Example
4-7, we can determine the reactions and find the maximum moment.
6. Calculate the distance from the left support to the load and the distance between supports.
© 2011 Pearson Education, Inc., Upper Saddle River, NJ. All rights reserved. This publication is protected by Copyright and written permission should be
MACHINE DESIGN - An Integrated Approach, 4th Ed. 4-43-2

a  0.5 Lfork a  600  mm


L  Lfork L  1200 mm

7. From inspection of Figure 4-43A, write the load function equation


q(x) = R1<x>-1 - F<x - a>-1 + R2<x - L>-1 + M2<x - L>-2
8. Integrate this equation from - to x to obtain shear, V(x)
V(x) = R1<x>0 - F<x - a>0 + R2<x - L>0 + M2<x - L>-1
9. Integrate this equation from - to x to obtain moment, M(x)
M(x) = R1<x>1 - F<x - a>1 + R2<x - L>1 + M2<x - L>0
10. Integrate the moment function, multiplying by 1/EI, to get the slope.
(x) = [R1<x>2/2 - F<x - a>2/2 + R2<x - L>2/2 + M2<x - L>1 + C3]/EI
11. Integrate again to get the deflection.
y(x) = [R1<x>3/6 - F<x - a>3/6 + R2<x - L>3/6 + M2<x - L>2/2 + C3x + C4]/EI

12. Evaluate R1, R2, M2, C3 and C4


At x = 0 and x = L; y = 0, therefore, C4 = 0. At x = L,  = 0
At x = L+, V = M = 0
2
Guess R1  1  kN R2  1  kN M2  1  kN  m C3  1  kN  m
Given
3 3
R 1 L F  ( L  a) 3
  C3 L = 0  kN  m
6 6

2 2
R 1 L F  ( L  a) 2
  C3 = 0  kN  m
2 2

R1  R2  F = 0  kN

R1 L  F  ( L  a )  M2 = 0  kN  m

 R1 
R 
 2   Find  R R M C 
 M2  1 2 2 3

 
 C3 
2
R1  2.913  kN R2  6.409  kN M2  2.097  kN  m C3  0.419  kN  m

13. Define the range for x x  0  in 0.002  L  L


14. For a Mathcad solution, define a step function S. This function will have a value of zero when x is less than
z, and a value of one when it is greater than or equal to z.

S ( x z)  if ( x  z 1 0 )


© 2011 Pearson Education, Inc., Upper Saddle River, NJ. All rights reserved. This publication is protected by Copyright and written permission should be
MACHINE DESIGN - An Integrated Approach, 4th Ed. 4-43-3

15. Write the shear and moment equations in Mathcad form, using the function S as a multiplying factor to get the
effect of the singularity functions.

V ( x)  R1 S ( x 0  in)  F  S ( x a )  R2 S ( x L)

M ( x)  R1 S ( x 0  in)  x  F  S ( x a )  ( x  a )  R2 S ( x L)  ( x  L)

16. Plot the shear and moment diagrams.


Shear Diagram Moment Diagram

10 2

1
5

0
V ( x) M ( x)
0
kN kN  m
1

5
2

 10 3
0 200 400 600 800 1000 1200 0 200 400 600 800 1000 1200
x x
mm mm
FIGURE 4-43B
Shear and Moment Diagrams for Problem 4-43

17. The maximum moment occurs at x = L, Mmax  M ( L) Mmax  2.097  kN  m

Mmax c
18. The bending stress is maximum at the support and is σa  σa  87.2 MPa
I

© 2011 Pearson Education, Inc., Upper Saddle River, NJ. All rights reserved. This publication is protected by Copyright and written permission should be
MACHINE DESIGN - An Integrated Approach, 4th Ed. 4-44-1
PROBLEM 4-44
Statement: Determine a suitable thickness for the V-links of the off-loading station of Figure P4-17 to limit
their deflections at the tips to 10-mm in any position during their rotation. Two V-links support
the roll, at the 1/4 and 3/4 points along the roll's length, and each of the V arms is 10-cm wide by
1-m long. The V arms are welded to a steel tube that is rotated by the air cylinder. See Problem
4-43 for more information.
Given: Roll OD OD  0.90 m Arm width wa  100  mm
Roll ID ID  0.22 m Arm length La  1000 mm
Roll length Lroll  3.23 m Max tip deflection δtip  10 mm
3
Roll density ρ  984  kg m Mod of elasticity E  207  GPa

Assumptions: 1. The maximum deflection on an arm will occur just after it begins the transfer and just before it
completes it, i.e., when the angle is either zero or 90 deg., but after the tip is no longer
supported by the base unit.
2. At that time the roll is in contact with both arms ("seated" in the V) and will remain in that
state throughout the motion. When the roll is in any other position on an arm the tip will be
supported.
3. The arm can be treated as a cantilever beam with nonend load.
4. A single arm will never carry more than half the weight of a roll.
5. The pipe to which the arms are attached has OD = 160 mm.
Solution: See Figure 4-44 and Mathcad file P0444.

1. Determine the weight of the roll and the load on each 450
V-arm.

W 
π
4
 2 2 
 OD  ID  Lroll  ρ  g W  18.64  kN

F  0.5 W F  9.32 kN

2. From Appendix B, Figure B-1, the tip deflection of a


cantilever beam with a concentrated load located at a
distance a from the support is
2
F a
ymax =  ( a  3  L) 1000 = L
6  E I
370 = a
F
where L is the beam length and I is the
cross-section moment of inertia. In this case

3 M
w a t a
I= F
12
FIGURE 4-44
3. Setting ymax = δtip and a  370  mm Free Body Diagram used in Problem 4-44

substituting for I and solving for ta


1
3
 2 F  a2  3 La  a 
ta  ta  31.889 mm
 E δtip  wa 
 
Let the arm thickness be ta  32 mm

© 2011 Pearson Education, Inc., Upper Saddle River, NJ. All rights reserved. This publication is protected by Copyright and written permission should be
MACHINE DESIGN - An Integrated Approach, 4th Ed. 4-45-1
PROBLEM 4-45
Statement: Determine the critical load on the air cylinder rod in Figure P4-17 if the crank arm that rotates it is
0.3 m long and the rod has a maximum extension of 0.5 m. The 25-mm-dia rod is solid steel with a
yield strength of 400 MPa. State all assumptions.

Given: Rod length L  500  mm Young's modulus E  207  GPa


Rod diameter d  25 mm Yield strength S y  400  MPa
Assumptions: 1. The rod is a fixed-pinned column.
2. Use a conservative value of 1 for the end factor (see Table 4-7 in text).
Solution: See Mathcad file P0445.

1. Calculate the slenderness ratio that divides the unit load vs slenderness ratio graph into Johnson and Euler
regions.

2 E
S rD  π S rD  101.07
Sy

2. Calculate the cross-section area and the moment of inertia.


π 2 2
Area A  d A  490.87 mm
4
π 4 4 4
Moment of inertia I  d I  1.92  10  mm
64

3. Using Table 4-7, calculate the effective column length.


Leff  1  L Leff  500  mm

4. Calculate the slenderness ratio for the column.

I
Radius of gyration k  k  6.25 mm
A

Leff
Slenderness ratio S r  S r  80.00
k

Since the Sr for this column is less than SrD, it is a Johnson column.

5. Calculate the critical load using the Johnson equation.

 2
1  S y S r  

Pcr  A  S y     Pcr  134.8  kN
 E  2 π  

© 2011 Pearson Education, Inc., Upper Saddle River, NJ. All rights reserved. This publication is protected by Copyright and written permission should be
MACHINE DESIGN - An Integrated Approach, 4th Ed. 4-46-1
PROBLEM 4-46
Statement: The V-links of Figure P4-17 are rotated by the crank arm through a shaft that is 60 mm dia by
3.23 m long. Determine the maximum torque applied to this shaft during motion of the
V-linkage and find the maximum stress and deflection for the shaft. See Problem 4-43 for more
information.

Given: Paper roll dimensions OD  900  mm Shaft dims d  60 mm


ID  220  mm Lshaft  3230 mm
Lroll  3230 mm
3
Roll density ρ  984  kg m Modulus of rigidity G  79 GPa

Assumptions: The greatest torque will occur when the link is horizontal and the paper roll is located as shown
in Figure P4-17 or Figure 4-46.
Solution: See Figure 4-46 and Mathcad file P0446.
y

1. Determine the weight of the roll on the V-arms.

W 
π
4
 2 2

 OD  ID  Lroll  ρ  g

W  18.64  kN

2. Summing moments about the shaft center,


OD
T  W T  8.390  kN  m
2

3. Calculate the polar moment of inertia. W

4
π d 6 4 T
J  J  1.272  10  mm
32
4. The maximum torsional stress will be at the outside Ry
diameter of the shaft. The radius of the OD is, 60-mm-dia shaft
450.0
d
r  r  30 mm FIGURE 4-46
2 Free Body Diagram used in Problem 4-46

5. Determine the maximum torsional stress using equation (4.23b).


Tr
τmax  τmax  197.8  MPa
J

6. Use equation (4.24) to determine the angular shaft deflection.


T  Lshaft
θ  θ  15.447 deg
J G

© 2011 Pearson Education, Inc., Upper Saddle River, NJ. All rights reserved. This publication is protected by Copyright and written permission should be
MACHINE DESIGN - An Integrated Approach, 4th Ed. 4-47-1
PROBLEM 4-47
Statement: Determine the maximum forces on the pins at each end of the air cylinder of Figure P4-17.
Determine the stress in these pins if they are 30-mm dia and in single shear.
Given: Paper roll dimensions OD  0.90 m Pin diameter d  30 mm
ID  0.22 m
Lroll  3.23 m
3
Roll density ρ  984  kg m

Assumptions: 1. The maximum force in the cylinder rod occurs when the transfer starts.
2. The cylinder and rod make an angle of 8 deg to the horizontal at the start of transfer.
3. The crank arm is 300 mm long and is 45 deg from vertical at the start of transfer.
4. The cylinder rod is fully retracted at the start of the transfer. At the end of the transfer it will
have extended 500 mm from its initial position.
Solution: See Figure 4-47 and Mathcad file P0447.

1. Determine the weight of the roll on the y


forks.

W 
π
4
 2 2 
 OD  ID  Lroll  ρ  g

W  18.64  kN
2. From the assumptions and
Figure 4-47, the x and y
distances from the origin to
point A are,

Rax  300  cos( 45 deg)  mm


W
Ray  300  sin( 45 deg)  mm
Rx x

Rax  212.132  mm
Ry 212.1
A
Ray  212.132  mm
F 8°
212.1
450.0
3. From Figure 4-47, the x
distance from the origin to
point where W is applied is, FIGURE 4-47
Free Body Diagram at Start of Transfer for V-link of Problem 4-47

OD
Rwx  Rwx  450  mm
2

4. Sum moments about the pivot point and solve for the compressive force in the cylinder rod.

W  Rwx  F  Rax sin( 8  deg)  F  Ray cos( 8  deg) = 0

W  Rwx
F  F  46.469 kN
Ray cos( 8  deg)  Rax sin( 8  deg)

This is the shear force in the pins

© 2011 Pearson Education, Inc., Upper Saddle River, NJ. All rights reserved. This publication is protected by Copyright and written permission should be
MACHINE DESIGN - An Integrated Approach, 4th Ed. 4-47-2

5. Determine the cross-sectional area of the pins and the direct shear stress.
2
π d 2
Shear area A  A  706.858  mm
4

F
Shear stress τ  τ  65.7 MPa
A

© 2011 Pearson Education, Inc., Upper Saddle River, NJ. All rights reserved. This publication is protected by Copyright and written permission should be
MACHINE DESIGN - An Integrated Approach, 4th Ed. 4-48-1
PROBLEM 4-48
Statement: A 100-kg wheelchair marathon racer wants an exerciser that will allow indoor practicing in any
weather. The design shown in Figure P4-18 is proposed. Two free-turning rollers on bearings
support the rear wheels. A platform supports the front wheels. Design the 1-m-long rollers as
hollow tubes of aluminum to minimize the height of the platform and also limit the roller
deflections to 1 mm in the worst case. The wheelchair has 650-mm-dia drive wheels separated
by a 700-mm track width. The flanges shown on the rollers limit the lateral movement of the
chair while exercising and thus the wheels can be anywhere between those flanges. Specify
suitable sized steel axles to support the tubes on bearings. Calculate all significant stresses.

Given: Mass of chair M  100  kg Maximum deflection δ  1  mm


Wheel diameter d w  650  mm Modulus elasticity
Track width T  700  mm Aluminum Ea  71.7 GPa
Roller length Lr  1000 mm Steel Es  207  GPa

Assumptions: 1. The CG of the chair with rider is


sufficiently close to the rear wheel that all of
the weight is taken by the two rear wheels.
2. The small camber angle of the rear wheels
does not significantly affect the magnitude W/2
of the forces on the rollers.
3. Both the aluminum roller and the steel axle
are simply supported. The steel axles that
support the aluminum tube are fixed in the
mounting block and do not rotate. The
aluminum tube is attached to them by two
bearings (one on each end of the tubes, one
for each axle). The bearings' inner race is
fixed, and the outer race rotates with the
aluminum tube. Each steel axle is considered
to be loaded as a simply supported beam. F F
Their diameter must be less than the inner
diameter of the tubes to fit the roller bearings  
between them.

FIGURE 4-48A
Solution: See Figures 4-48 and Mathcad file P0448.
Free Body Diagram of One Wheel
used in Problem 4-48
1. Calculate the weight of the chair with rider.
Weight of chair W  M  g W  980.7  N

2. Calculate the forces exerted by the wheels on the rollers (see Figure 4-48A). From the FBD of a wheel,
summing vertical forces
W
2  F  cos( θ )  =0
2
W
Let θ  20 deg then F  F  260.9  N
4  cos( θ )

3. The worst condition (highest moment at site of a stress concentration) will occur when the chair is all the
way to the left or right. Looking at the plane through the roller that includes the forces exerted by the wheels
(the FBD is shown in Figure 4-48B) the reactions R1 and R2 come from the bearings, which are inside the
hollow roller and are, themselves, supported by the steel axle.

4. Solving for the reactions. Let the distance from R1 to F be a  15 mm

© 2011 Pearson Education, Inc., Upper Saddle River, NJ. All rights reserved. This publication is protected by Copyright and written permission should be
MACHINE DESIGN - An Integrated Approach, 4th Ed. 4-48-2

 M1 R2 Lr  F  ( a  T )  F  a = 0 F 700 F

 Fy R1  2  F  R2 = 0

F  (2 a  T )
R2  R2  190.5  N
Lr 15 R2
R1 1000
R1  2  F  R2 R1  331.3  N
FIGURE 4-48B
Free Body Diagram of One Tube used in Problem 4-48

5. The maximum bending moment will be at the right-hand load and will be
Mrmax  R2 Lr  ( a  T ) Mrmax  54.3 N  m

Note, if the chair were centered on the roller the maximum moment would be
Lr  T
Mc  F  Mc  39.1 N  m
2
and this would be constant along the axle between the two loads, F.

6. Note that the bearing positions are fixed regardless of the position of the chair on the roller.

Because of symmetry,
65 1000
Ra1  R1 Ra1  331.3  N
R1 R2
Ra2  R2 Ra2  190.5  N

7. The maximum bending moment


R a1 R a2
occurs
at R1 and is for b  65 mm 1130
Mamax  Ra1 b
FIGURE 4-48C
Free Body Diagram of One Axle used in Problem 4-48
Mamax  21.5 N  m

8. Determine a suitable axle diameter. Let the factor of safety against yielding in the axle be
Nsa  3
9. Tentatively choose a low-carbon steel for the axle, say AISI 1020, cold rolled with
S y  393  MPa
10. At the top of the axle under the load R1 there is only a bending stress. Set this stress equal to the yield
strength divided by the factor of safety.

32 Mamax Sy
σx = =
3 Nsa
π d a
1
3
 32 Nsa Mamax 
Solving for the axle diameter, d a d a    d a  11.875 mm
 π S y 

Let the axle diameter be d a  15 mm made from cold-rolled AISI 1020 steel.

© 2011 Pearson Education, Inc., Upper Saddle River, NJ. All rights reserved. This publication is protected by Copyright and written permission should be
MACHINE DESIGN - An Integrated Approach, 4th Ed. 4-48-3
11. Suppose that bearing 6302 from Chapter
10, Figure 10-23. It has a bore of 15 mm and an
150 700
OD of 42 mm. Thus, the inside diameter of the F F
roller away from the bearings where the
moment is a maximum will be d i  40 mm.
This will provide a 1-mm shoulder for axial
location of the bearings. F
15
F
12. The maximum deflection of the roller will 1000
occur when the chair is in the center of the
roller. For this case the reactions are both equal
to the loads, F (see Figure 4-48D). The FIGURE 4-48D
maximum deflection is at the center of the roller. Free Body Diagram of Roller with Chair in the Center.

13. Write the load function and then integrate four times to get the deflection function.

q(x) = F<x>-1 - F<x - a>-1 - F<x - b>-1 + F<x - L>-1

y(x) = F[<x>3 - <x - a>3 - <x - b>3 + <x - L>3 + C3x]/(6EI)

 ( L  a )  a  L 
1 3 3 3
where C3 =
L

14. Write the deflection function at x = L/2 for


a  150  mm
 L  3 3 
F
     L  a  1  ( L  a) 3  a3  L3
ymax =
6  Ea I  2 
2 
  2 

15. Set this equation equal to the allowed deflection  and solve for the required moment of inertia, I.

 Lr  3  Lr 
3 
F 1  3
      a    Lr  a   a  Lr 
3 3 4 4
I  I  6.618  10  mm
6  Ea δ  2  2  2 
16. Knowing the inside diameter of the tube, solve for the outside diameter.
1
4
d o    d i 
π  4 64 I
  d o  d i 
4 4
I= d o  44.463 mm
64  π 

Round this up to d o  46 mm

DESIGN SUMMARY

Axles Rollers
Material AISI 1020 steel, cold-rolled Material 2024-T4 aluminum
Diameter d a  15 mm Outside diameter d o  46 mm
Length 1220 mm Inside diameter d i  40 mm
Length 1040 mm
Spacing c   d w  d o  sin( θ )

c  238  mm

© 2011 Pearson Education, Inc., Upper Saddle River, NJ. All rights reserved. This publication is protected by Copyright and written permission should be
MACHINE DESIGN - An Integrated Approach, 4th Ed. 4-49a-1
PROBLEM 4-49a
Statement: A hollow, square column has the dimensions and properties below. Determine if it is a Johnson
or an Euler column and find the critical load:
(a) If its boundary conditions are pinned-pinned.
(b) If its boundary conditions are fixed-pinned.
(c) If its boundary conditions are fixed-fixed.
(d) If its boundary conditions are fixed-free.
Given: Length of column L  100  mm Material Steel
Outside dimension so  4  mm Yield strength S y  300  MPa
Inside dimension si  3  mm Modulus of elasticity E  207  GPa
Solution: See Mathcad file P0449a.
1. Calculate the slenderness ratio that divides the unit load vs slenderness ratio graph into Johnson and Euler
regions.

2 E
S rD  π S rD  116.7
Sy

2. Calculate the cross-section area and the moment of inertia.


2 2 2
Area A  so  si A  7.00 mm

Moment of inertia I 
1
12
 4
 so  si
4 I  14.58  mm
4

(a) pinned-pinned ends

3. Using Table 4-7, calculate the effective column length.


Leff  1  L Leff  100  mm

4. Calculate the slenderness ratio for the column.

I
Radius of gyration k  k  1.443  mm
A

Leff
Slenderness ratio S r  S r  69.28
k

Since the S r for this column is less than S rD, it is a Johnson column.

5. Calculate the critical load using the Johnson equation.

  Sy S r  
2
Pcr  A  S y 
1
  Pcr  1.73 kN
 E  2 π  
(b) fixed-pinned ends

6. Using Table 4-7, calculate the effective column length.

Leff  0.8 L Leff  80 mm

7. Calculate the slenderness ratio for the column.

I
Radius of gyration k  k  1.443  mm
A
© 2011 Pearson Education, Inc., Upper Saddle River, NJ. All rights reserved. This publication is protected by Copyright and written permission should be
MACHINE DESIGN - An Integrated Approach, 4th Ed. 4-49a-2

Leff
Slenderness ratio S r  S r  55.43
k

Since the S r for this column is less than S rD, it is a Johnson column.

8. Calculate the critical load using the Johnson equation.

 2
1  S y S r  

Pcr  A  S y     Pcr  1.86 kN
 E  2 π  

(c) fixed-fixed ends

9. Using Table 4-7, calculate the effective column length.

Leff  0.65 L Leff  65 mm

10. Calculate the slenderness ratio for the column.

I
Radius of gyration k  k  1.443  mm
A
Leff
Slenderness ratio S r  S r  45.03
k

Since the S r for this column is less than S rD, it is a Johnson column.

11. Calculate the critical load using the Johnson equation.

 S y S r  
2

Pcr  A  S y    
1
Pcr  1.94 kN
 E  2 π  

(d) fixed-free ends

12. Using Table 4-7, calculate the effective column length.

Leff  2.1 L Leff  210  mm

13. Calculate the slenderness ratio for the column.

I
Radius of gyration k  k  1.443  mm
A
Leff
Slenderness ratio S r  S r  145.49
k

Since the S r for this column is greater than S rD, it is an Euler column.

14. Calculate the critical load using the Euler equation.

2
π E
Pcr  A  Pcr  676  N
2
Sr

© 2011 Pearson Education, Inc., Upper Saddle River, NJ. All rights reserved. This publication is protected by Copyright and written permission should be
MACHINE DESIGN - An Integrated Approach, 4th Ed. 4-50a-1
PROBLEM 4-50a
Statement: A hollow, round column has the dimensions and properties below. Determine if it is a Johnson
or an Euler column and find the critical load:
(a) If its boundary conditions are pinned-pinned.
(b) If its boundary conditions are fixed-pinned.
(c) If its boundary conditions are fixed-fixed.
(d) If its boundary conditions are fixed-free.

Given: Length of column L  1500 mm Material Steel


Outside diameter od  20 mm Yield strength S y  300  MPa
Inside diameter id  14 mm Modulus of elasticity E  207  GPa
Solution: See Mathcad file P0450a.
1. Calculate the slenderness ratio that divides the unit load vs slenderness ratio graph into Johnson and Euler
regions.

2 E
S rD  π S rD  116.7
Sy

2. Calculate the cross-section area, moment of inertia, and the radius of gyration.

Area A 
π
4
 2
 od  id
2 A  160.22 mm
2

Moment of inertia I 
π
64
 4
 od  id
4 I  5968 mm
4

I
Radius of gyration k  k  6.103  mm
A

3. Define functions to determine column type and critical load.

Type type S r  "Euler" if S r  S rD


"Johnson" otherwise

2
π E
Critical load Pcr S r  return A  if type S r = "Euler"
2
Sr

 2
1  S y S r  

A  Sy     otherwise
 E  2 π  
(a) pinned-pinned ends

4. Using Table 4-7, calculate the effective column length.


Leff  1  L Leff  1500 mm

5. Calculate the slenderness ratio for the column.

Leff
Slenderness ratio S r  S r  245.77
k
6. Determine the type and critical load using the functions defined above.

type S r  "Euler" Pcr S r  5.42 kN


© 2011 Pearson Education, Inc., Upper Saddle River, NJ. All rights reserved. This publication is protected by Copyright and written permission should be
MACHINE DESIGN - An Integrated Approach, 4th Ed. 4-50a-2

(b) fixed-pinned ends

7. Using Table 4-7, calculate the effective column length.

Leff  0.8 L Leff  1200 mm

8. Calculate the slenderness ratio for the column.

Leff
Slenderness ratio S r  S r  196.62
k

9. Determine the type and critical load using the functions defined above.

type S r  "Euler" Pcr S r  8.47 kN

(c) fixed-fixed ends

10. Using Table 4-7, calculate the effective column length.

Leff  0.65 L Leff  975  mm

11. Calculate the slenderness ratio for the column.

Leff
Slenderness ratio S r  S r  159.75
k
12. Determine the type and critical load using the functions defined above.

type S r  "Euler" Pcr S r  12.8 kN

(d) fixed-free ends

13. Using Table 4-7, calculate the effective column length.

Leff  2.1 L Leff  3150 mm

14. Calculate the slenderness ratio for the column.


Leff
Slenderness ratio S r  S r  516.12
k

15. Determine the type and critical load using the functions defined above.

type S r  "Euler" Pcr S r  1.23 kN

© 2011 Pearson Education, Inc., Upper Saddle River, NJ. All rights reserved. This publication is protected by Copyright and written permission should be
MACHINE DESIGN - An Integrated Approach, 4th Ed. 4-51a-1
PROBLEM 4-51a
Statement: A solid, rectangular column has the dimensions and properties below. Determine if it is a
Johnson or an Euler column and find the critical load:
(a) If its boundary conditions are pinned-pinned.
(b) If its boundary conditions are fixed-pinned.
(c) If its boundary conditions are fixed-fixed.
(d) If its boundary conditions are fixed-free.
Given: Length of col. L  100  mm Material Steel
Thickness t  10 mm Yield strength S y  300  MPa
Height h  20 mm Modulus of elasticity E  207  GPa
Solution: See Mathcad file P0451a.
1. Calculate the slenderness ratio that divides the unit load vs slenderness ratio graph into Johnson and Euler
regions.

2 E
S rD  π S rD  116.7
Sy

2. Calculate the cross-section area, moment of inertia, and the radius of gyration.
2
Area A  h  t A  200.00 mm

3
h t 4
Moment of inertia I  I  1667 mm
12

I
Radius of gyration k  k  2.887  mm
A

3. Define functions to determine column type and critical load.

Type type S r  "Euler" if S r  S rD


"Johnson" otherwise

2
π E
Critical load Pcr S r  return A  if type S r = "Euler"
2
Sr

  S y Sr  
2
A  S y 
1
  otherwise
 E  2 π  

(a) pinned-pinned ends

4. Using Table 4-7, calculate the effective column length.


Leff  1  L Leff  100  mm

5. Calculate the slenderness ratio for the column.

Leff
Slenderness ratio S r  S r  34.64
k
6. Determine the type and critical load using the functions defined above.

type S r  "Johnson" Pcr S r  57.36  kN


© 2011 Pearson Education, Inc., Upper Saddle River, NJ. All rights reserved. This publication is protected by Copyright and written permission should be
MACHINE DESIGN - An Integrated Approach, 4th Ed. 4-51a-2

(b) fixed-pinned ends

7. Using Table 4-7, calculate the effective column length.

Leff  0.8 L Leff  80 mm

8. Calculate the slenderness ratio for the column.

Leff
Slenderness ratio S r  S r  27.71
k

9. Determine the type and critical load using the functions defined above.

type S r  "Johnson" Pcr S r  58.31  kN

(c) fixed-fixed ends

10. Using Table 4-7, calculate the effective column length.

Leff  0.65 L Leff  65 mm

11. Calculate the slenderness ratio for the column.


Leff
Slenderness ratio S r  S r  22.52
k

12. Determine the type and critical load using the functions defined above.

type S r  "Johnson" Pcr S r  58.9 kN

(d) fixed-free ends

13. Using Table 4-7, calculate the effective column length.

Leff  2.1 L Leff  210  mm

13. Calculate the slenderness ratio for the column.


Leff
Slenderness ratio S r  S r  72.75
k

14. Determine the type and critical load using the functions defined above.

type S r  "Johnson" Pcr S r  48.34  kN

© 2011 Pearson Education, Inc., Upper Saddle River, NJ. All rights reserved. This publication is protected by Copyright and written permission should be
MACHINE DESIGN - An Integrated Approach, 4th Ed. 4-52a-1
PROBLEM 4-52a

Statement: A solid, circular column, loaded eccentrically, has the dimensions and properties below. Find
the critical load:
(a) If its boundary conditions are pinned-pinned.
(b) If its boundary conditions are fixed-pinned.
(c) If its boundary conditions are fixed-fixed.
(d) If its boundary conditions are fixed-free.
Given: Length of column L  100  mm Material Steel
Outside diameter od  20 mm Yield strength S y  300  MPa
Eccentricity (t) e  10 mm Modulus of elasticity E  207  GPa
Solution: See Mathcad file P0452a.
1. Calculate the cross-section area, distance to extreme fiber, and the moment of inertia.
π 2 2
Area A   od A  314.16 mm
4
Distance to extreme fiber c  0.5 od c  10 mm
π 4 4
Moment of inertia I   od I  7854 mm
64

4. Calculate the radius of gyration and eccentricity ratio for the column.

I
Radius of gyration k  k  5.00 mm
A

e c
Eccentricity ratio Er  Er  4.0
2
k
(a) pinned-pinned ends
3. Using Table 4-7, calculate the effective column length.
Leff  1  L Leff  100  mm

4. Calculate the slenderness ratio for the column.

Leff
Slenderness ratio S r  S r  20.00
k
5. Calculate the critical load using the Secant equation.
Guess P  1  kN
Given
S y A
P=

1  Er sec S r
P 

 4  E A 

Pcr  Find ( P) Pcr  18.63  kN

(b) fixed-pinned ends


6. Using Table 4-7, calculate the effective column length.
Leff  0.8 L Leff  80 mm

7. Calculate the slenderness ratio for the column.


© 2011 Pearson Education, Inc., Upper Saddle River, NJ. All rights reserved. This publication is protected by Copyright and written permission should be
MACHINE DESIGN - An Integrated Approach, 4th Ed. 4-52a-2

Leff
Slenderness ratio S r  S r  16.00
k

8. Calculate the critical load using the Secant equation.


Guess P  1  kN
Given
S y A
P=

1  Er sec S r
P 

 4  E A 

Pcr  Find ( P) Pcr  18.71  kN

(c) fixed-fixed ends

9. Using Table 4-7, calculate the effective column length.


Leff  0.65 L Leff  65 mm

10. Calculate the slenderness ratio for the column.


Leff
Slenderness ratio S r  S r  13.00
k
11. Calculate the critical load using the Secant equation.
Guess P  1  kN
Given
S y A
P=

1  Er sec S r
P 

 4  E A 

Pcr  Find ( P) Pcr  18.76  kN

(d) fixed-free ends


12. Using Table 4-7, calculate the effective column length.
Leff  2.1 L Leff  210  mm

13. Calculate the slenderness ratio for the column.

Leff
Slenderness ratio S r  S r  42
k
14. Calculate the critical load using the Secant equation.
Guess P  1  kN
Given
S y A
P=

1  Er sec S r
P 

 4  E A 

Pcr  Find ( P) Pcr  17.93  kN

© 2011 Pearson Education, Inc., Upper Saddle River, NJ. All rights reserved. This publication is protected by Copyright and written permission should be
MACHINE DESIGN - An Integrated Approach, 4th Ed. 4-53-1
PROBLEM 4-53
Statement: Design an aluminum, hollow, circular column for the conditions given below for (a)
pinned-pinned ends and (b) fixed-free ends.
Given: Length of column L  3  m Factor of safety FS  3
Wall thickness t  5  mm Yield strength S yc  150  MPa
Load supported F  900  N Modulus of elasticity E  71.7 GPa
Solution: See Mathcad file P0453.
1. Start by calculating the slenderness ratio that divides the unit load vs slenderness ratio graph into Johnson
and Euler regions.

2 E
S rD  π S rD  97.136
S yc

2. Using Table 4-7, calculate the effective column length.


Leff  1  L Leff  3000 mm

3. To start the iterative process, assume that the final design will be an Euler column with the critical load equal
to FS*F. From equation 4.38b,
2 2
π  E A  k 2 I
Pcr = and k =
2 A
L
2
π  E I
Substituting for k2 Pcr = = FS  F
2
L
2
Leff  FS  F
Solving for I I 
2
π E
4
The required moment of inertia, assuming an Euler column is I  34339  mm

4. Using the relationships given on the inside cover, solve for the outside diameter of the tube.

Guess D  20 mm
π  4
 D  ( D  2  t) 
4
Given I=
64

D  Find ( D) D  30.64  mm

5. Using this diameter, calculate the slenderness ratio and compare to S rD. If it is greater than S rD the assumption o
an Euler column is correct, if not, recalculate using the Johnson equation.
Inside diameter d  D  2  t d  20.64  mm

Area Ar 
π
4
 2
 D d
2 Ar  402.7  mm
2

I
Radius of gyration kr  kr  9.234  mm
Ar

Leff
Slenderness ratio S r  S r  324.9
kr
© 2011 Pearson Education, Inc., Upper Saddle River, NJ. All rights reserved. This publication is protected by Copyright and written permission should be
MACHINE DESIGN - An Integrated Approach, 4th Ed. 4-53-2

Since this is greater than S rD, the assumption of an Euler column is correct and the minimum outside diameter is

D  30.64  mm

(b) fixed-free ends

6. Using Table 4-7, calculate the effective column length.

Leff  2.1 L Leff  6300 mm

7. To start the iterative process, assume that the final design will be an Euler column with the critical load equal
to FS*F. From equation 4.38b,
2 2
π  E A  k 2 I
Pcr = and k =
2 A
L
2
π  E I
Substituting for k2 Pcr = = FS  F
2
L
2
Leff  FS  F
Solving for I I 
2
π E
5 4
The required moment of inertia, assuming an Euler column is I  2  10  mm

8. Using the relationships given on the inside cover, solve for the outside diameter of the tube.

Guess D  20 mm
π  4
 D  ( D  2  t) 
4
Given I=
64

D  Find ( D) D  47.37  mm

9. Using this diameter, calculate the slenderness ratio and compare to S rD. If it is greater than S rD the
assumption of an Euler column is correct, if not, recalculate using the Johnson equation.

Inside diameter d  D  2  t d  37.37  mm

Area Ar 
π
4
 2
 D d
2 Ar  665.6  mm
2

I
Radius of gyration kr  kr  15.084 mm
Ar

Leff
Slenderness ratio S r  S r  417.7
kr

Since this is greater than S rD, the assumption of an Euler column is correct and the minimum outside diameter
is

D  47.37  mm

© 2011 Pearson Education, Inc., Upper Saddle River, NJ. All rights reserved. This publication is protected by Copyright and written permission should be
MACHINE DESIGN - An Integrated Approach, 4th Ed. 4-54-1

PROBLEM 4-54
Statement: Three round, 1.25-in-dia bars are made of SAE 1030 hot-rolled steel but are of different lengths,
5 in, 30 in, and 60 in, respectively. They are loaded axially in compression. Compare the load
supporting capability of the three bars if the ends are assumed to be:
(a) Pinned-pinned.
(b) Fixed-pinned.
(c) Fixed-fixed.
(d) Fixed-free.
Given: Outside diameter d  1.25 in
Lengths L  5  in L  30 in L  60 in i  1 2  3
1 2 3
Material SAE 1030 Steel Yield strength S y  38 ksi
6
Modulus of elasticity E  30 10  psi
Solution: See Mathcad file P0454.
1. Calculate the slenderness ratio that divides the unit load vs. slenderness ratio graph into Johnson and Euler
regions.

2 E
S rD  π S rD  124.8
Sy

2. Calculate the cross-section area, moment of inertia, and the radius of gyration.
π 2 2
Area A  d A  791.73 mm
4

π 4 4
Moment of inertia I  d I  49882  mm
64

I
Radius of gyration k  k  7.938  mm
A

3. Define functions to determine column type and critical load.

Type type S r  "Euler" if S r  S rD


"Johnson" otherwise

2
π E
Critical load Pcr S r  return A  if type S r = "Euler"
2
Sr

 S y S r  
2

A  S y      otherwise
1
 E  2 π  
(a) pinned-pinned ends
4. Using Table 4-7, calculate the effective column length.

5
Leff  1  L Leff  30   in

 
 60 
5. Calculate the slenderness ratio for the column.

© 2011 Pearson Education, Inc., Upper Saddle River, NJ. All rights reserved. This publication is protected by Copyright and written permission should be
MACHINE DESIGN - An Integrated Approach, 4th Ed. 4-54-2

 16 
S r  96 

Leff
Slenderness ratio S r 
k  
 192 
6. Determine the type and critical load using the functions defined above.

 "Johnson"   
Pcr S r
i
Type  type S r   Type  "Johnson" 

i i i   lbf
 "Euler"  46250
32844
9857

(b) fixed-pinned ends

7. Using Table 4-7, calculate the effective column length.

 4.0 
Leff  0.8 L Leff  24.0   in

 
 48.0 
8. Calculate the slenderness ratio for the column.

 12.8 
S r  76.8 

Leff
Slenderness ratio S r 
k  
 153.6 
9. Determine the type and critical load using the functions defined above.

 "Johnson"   
Pcr S r
i
Type  type S r   Type  "Johnson" 

i i i   lbf
 "Euler"  46388
37808
15401

(c) fixed-fixed ends


10. Using Table 4-7, calculate the effective column length.

 3.3 
Leff  0.65 L Leff  19.5   in

 
 39.0 
11. Calculate the slenderness ratio for the column.

 10.4 
S r  62.4 

Leff
Slenderness ratio S r 
k  
 124.8 

© 2011 Pearson Education, Inc., Upper Saddle River, NJ. All rights reserved. This publication is protected by Copyright and written permission should be
MACHINE DESIGN - An Integrated Approach, 4th Ed. 4-54-3

12. Determine the type and critical load using the functions defined above.

 "Johnson"   
Pcr S r
i
Type  type S r   Type   "Johnson" 
i i i   lbf
 "Johnson"  46471
40807
23329

(d) fixed-free ends


13. Using Table 4-7, calculate the effective column length.

 10.5 
Leff  2.1 L Leff  63.0   in

 
 126.0 
14. Calculate the slenderness ratio for the column.

 33.6 
S r  201.6 

Leff
Slenderness ratio S r 
k  
 403.2 
15. Determine the type and critical load using the functions defined above.

 "Johnson"   
Pcr S r
i
 "Euler" 
Type  type S r
i  
i
Type 
i   lbf
 "Euler"  44944
8940
2235

© 2011 Pearson Education, Inc., Upper Saddle River, NJ. All rights reserved. This publication is protected by Copyright and written permission should be
MACHINE DESIGN - An Integrated Approach, 4th Ed. 4-55-1

PROBLEM 4-55 _____


Statement: Figure P4-19 shows a 1.5-in-dia, 30-in-long steel rod subjected to tensile loads P = 10000 lb
applied at each end of the rod, acting along its longitudinal Y axis and through the centroid of
its circular cross section. Point A is 12 in below the upper end and point B is 8 in below A. For
this bar with its loading, find:
(a) All components of the stress tensor matrix (equation 4.1a) for a point midway between
A and B.
(b) The displacement of point B relative to point A.
(c) The elastic strain in the section between A and B.
(d) The total strain in the section between A and B.
Given: Tensile load P  10000  lbf
6
Diameter d  1.50 in Modulus of elasticity E  30 10  psi
Lengths L  30 in LA  12 in LAB  8  in

Solution: See Mathcad file P0455.


1. Calculate the cross-section area of the rod.
2
π d 2
A  A  1.767  in
4

2. (a) The loading is simple axial tension so all components of the stress tensor are zero except yy, which is
found using equation 4.7.
P
σyy  σyy  5659 psi
A

This stress is uniform across the rod and has the same value at any cross section along the longitudinal axis
except close to the ends where the load is applied.
3. (b) The displacement of point B relative to A can be found using equation 4.8.

P LAB 3
∆sBA  ∆sBA  1.509  10  in
A E

4. (c) The elastic strain in the rod can be found using Hooke's law (equation 2.2)

σyy 4
ε  ε  1.886  10
E

5. (d) Assuming that the yield strength of this steel is greater than yy, the strain calculated in step 4 is the total
strain.

© 2011 Pearson Education, Inc., Upper Saddle River, NJ. All rights reserved. This publication is protected by Copyright and written permission should be
MACHINE DESIGN - An Integrated Approach, 4th Ed. 4-56-1

PROBLEM 4-56 _____


Statement: The rod in Figure P4-19, with the loading of Problem 4-55, is subjected to a reduction of
temperature from 80F to 20F after the load is applied. The coefficient of thermal expansion for
steel is approximately 6 in/in/degF. Find:
(a) All components of the stress tensor matrix (equation 4.1a) for a point midway between
A and B.
(b) The displacement of point B relative to point A.
(c) The elastic strain in the section between A and B.
(d) The total strain in the section between A and B.

Units: Temperature scale F  1


Given: Tensile load P  10000  lbf
6
Diameter d  1.50 in Modulus of elasticity E  30 10  psi
Lengths L  30 in LA  12 in LAB  8  in
Temperatures T1  80 F T2  20 F
6 1
Coefficient of thermal expansion α  6  10 F
Solution: See Mathcad file P0456.
1. Calculate the cross-section area of the rod.
2
π d 2
A  A  1.767  in
4

2. (a) The loading is simple axial tension so all components of the stress tensor are zero except yy, which is
found using equation 4.7.
P
σyy  σyy  5659 psi
A

This stress is uniform across the rod and has the same value at any cross section along the longitudinal axis
except close to the ends where the load is applied. The change in temperature does not affect the stress
since the ends are free.
3. (b) The displacement of point B relative to A can be found by summing equation 4.8 for the elastic portion
and the thermal expansion equation from elementary mechanics of materials for the thermal portion.

P LAB 3
∆sBA   α  T2  T1  LAB ∆sBA  1.371  10  in
A E

4. (c) The elastic strain in the rod can be found using Hooke's law (equation 2.2)

σyy 4
ε  ε  1.886  10
E

5. (d) Assuming that there is no plastic strain in the rod, the total strain is the sum of the elastic strain found
in step 4 plus the thermal strain.

4
ε tot  ε  α  T2  T1 ε tot  1.714  10

© 2011 Pearson Education, Inc., Upper Saddle River, NJ. All rights reserved. This publication is protected by Copyright and written permission should be
MACHINE DESIGN - An Integrated Approach, 4th Ed. 4-57-1

PROBLEM 4-57 _____


Statement: Figure P4-20 shows a steel bar fastened to a rigid ground plane with two 0.25-in-dia hardened
steel dowel pins. For P = 1500 lb, find:
(a) The shear stress in each pin.
(b) The direct bearing stress in each pin and hole.
(c) The minimum value of dimension h to prevent tearout failure if the steel bar has a
shear strength of 32500 psi.
Given: Pin diameter d  0.250  in Applied load P  1500 lbf
Distance between pins a  2.0 in Shear strength of bar S s  32.5 ksi
Thickness of bar t  0.25 in Distance from right pin to load b  4.0 in
Solution: See Mathcad file P0457.

1. Draw a free-body diagram and find the shear forces (reactions) on each pin.

a b
RL

RR

Write equations 3.3b for the bar and solve for the reactions.

 F: RL  RR  P  0  M: RL  a  P b  0

b
RL  P RL  3000 lbf RR  P  RL RR  4500 lbf
a

2. Calculate the cross-section area of a pin.


2
π d 2
A  A  0.0491 in
4
3. (a) Use equation 4.9 to determine the shear stress in each pin.

RL
Left pin τL  τL  61.1 ksi
A
RR
Right pin τR  τR  91.7 ksi
A
4. (b) Calculate the bearing area from equation 4.10 and use it to determine the bearing stress in each pin.
2
Bearing area Abear  d  t Abear  0.0625 in

RL
σL  σL  48.0 ksi
Abear
© 2011 Pearson Education, Inc., Upper Saddle River, NJ. All rights reserved. This publication is protected by Copyright and written permission should be
MACHINE DESIGN - An Integrated Approach, 4th Ed. 4-57-2

RR
σR  σR  72.0 ksi
Abear

h  d
5. (c) The tearout area is Atear  2   
  t , where (h - d)/2 is the distance from the edge of the hole to
 2  
the outside of the bar. Substitute this area in equation 4.9 for the shear area and substitute the shear
strength for xy, solving then for the unknown distance h.

RL
Left pin h L  d h L  0.619  in
S s t

RR
Right pin h R  d h R  0.804  in
S s t

Minimum value of h h min  h R h min  0.804  in

© 2011 Pearson Education, Inc., Upper Saddle River, NJ. All rights reserved. This publication is protected by Copyright and written permission should be
MACHINE DESIGN - An Integrated Approach, 4th Ed. 4-58-1

PROBLEM 4-58 _____


Statement: Figure P4-20 shows a steel bar fastened to a rigid ground plane with two 0.25-in-dia hardened
steel dowel pins. For P = 2200 lb, find:
(a) The shear stress in each pin.
(b) The direct bearing stress in each pin and hole.
(c) The minimum value of dimension h to prevent tearout failure if the steel bar has a shear
strength of 32500 psi.
Given: Pin diameter d  0.250  in Applied load P  2200 lbf
Distance between pins a  2.0 in Shear strength of bar S s  32.5 ksi
Thickness of bar t  0.25 in Distance from right pin to load b  4.0 in
Solution: See Mathcad file P0458.

1. Draw a free-body diagram and find the shear forces (reactions) on each pin.

a b
RL

RR

Write equations 3.3b for the bar and solve for the reactions.

 F: RL  RR  P  0  M: RL  a  P b  0

b
RL  P RL  4400 lbf RR  P  RL RR  6600 lbf
a

2. Calculate the cross-section area of a pin.


2
π d 2
A  A  0.0491 in
4

3. (a) Use equation 4.9 to determine the shear stress in each pin.

RL
Left pin τL  τL  89.6 ksi
A
RR
Right pin τR  τR  134.5  ksi
A

4. (b) Calculate the bearing area from equation 4.10 and use it to determine the bearing stress in each pin.
2
Bearing area Abear  d  t Abear  0.0625 in

© 2011 Pearson Education, Inc., Upper Saddle River, NJ. All rights reserved. This publication is protected by Copyright and written permission should be
MACHINE DESIGN - An Integrated Approach, 4th Ed. 4-58-2

RL
σL  σL  70.4 ksi
Abear
RR
σR  σR  105.6  ksi
Abear

h  d
5. (c) The tearout area is Atear  2   
  t , where (h - d)/2 is the distance from the edge of the hole to
 2  
the outside of the bar. Substitute this area in equation 4.9 for the shear area and substitute the shear
strength for xy, solving then for the unknown distance h.

RL
Left pin h L  d h L  0.792  in
S s t

RR
Right pin h R  d h R  1.062  in
S s t

Minimum value of h h min  h R h min  1.062  in

© 2011 Pearson Education, Inc., Upper Saddle River, NJ. All rights reserved. This publication is protected by Copyright and written permission should be
MACHINE DESIGN - An Integrated Approach, 4th Ed. 4-59-1

PROBLEM 4-59 _____


Statement: Figure P4-21 shows a rectangular section aluminum bar subjected to off-center forces P = 4000
N applied as shown.
(a) Solve for the maximum normal stress in the mid-region of the bar well away from the
eyes where the loads are applied.
(b) Plot the normal stress distribution across the cross section at this mid-region.
(c) Sketch a "reasonable" plot of the normal stress distribution across the cross section at
the ends, close to the applied loads.

Given: Depth of bar h  40 mm Applied loads P  4000 N


Thickness of bar t  10 mm Location of eye d  35 mm (from bottom edge)
Solution: See Mathcad file P0459.
1. Draw a free-body diagram of the bar, cut at any section along the length of the bar.

FACE OF CUT
SURFACE

d h
P
M
0.5h

SECTION CENTROIDAL AXIS

Equilibrium requires that there be a force directed along the centroidal axis of the cross section that is equal
and opposite to the applied force and a bending moment to react the couple formed by the applied force and
the reaction force. Thus, since the reaction moment is clockwise,

M  ( d  0.5 h )  P M  60.000 N  m

2. Calculate the cross-section area, moment of inertia, and distance from the centroid to the outer surface.
2
A  h  t A  400.0  mm
3
t h 4 4
I  I  5.333  10  mm
12
c  0.5 h c  20.000 mm

2. (a) The normal stress on a section well away from the ends is a combination of uniform tension, as given by
equation 4.7, and bending, as given by equation 4.11a.

M y P
σ ( y )   
I A

This will be a maximum at y = c. σmax  σ ( c) σmax  32.5 MPa

3. (b) Plot the normal stress distribution across the cross section at the mid-region of the bar for

y  c c  1  mm  c

© 2011 Pearson Education, Inc., Upper Saddle River, NJ. All rights reserved. This publication is protected by Copyright and written permission should be
MACHINE DESIGN - An Integrated Approach, 4th Ed. 4-59-2

NORMAL STRESS ON SECTION


40

30

20
Stress, MPa

σ( y )
10
MPa

 10

 20
 20  10 0 10 20 30
y
mm
Distance from neutral axis, mm

4. (c) Sketch a "reasonable" plot of the normal stress distribution across the cross section at the ends, close to
the applied loads. Use the "force flow" analogy show in Figures 4-37 and 4-38 as a guide to the stress
distribution. Near the applied load the stress will be highly concentrated. As the distance from the point of
load application increases the stress will become more evenly distributed.

© 2011 Pearson Education, Inc., Upper Saddle River, NJ. All rights reserved. This publication is protected by Copyright and written permission should be
MACHINE DESIGN - An Integrated Approach, 4th Ed. 4-60-1

PROBLEM 4-60 _____


Statement: Figure P4-22 shows a bracket machined from 0.5-in-thick steel flat stock. It is rigidly attached to
a support and loaded with P = 5000 lb at point D. Find:
(a) The magnitude, location, and the plane orientation of the maximum normal stress at
section A-A.
(b) The magnitude, location, and the plane orientation of the maximum shear stress at
section A-A.
(c) The magnitude, location, and the plane orientation of the maximum normal stress at
section B-B.
(d) The magnitude, location, and the plane orientation of the maximum shear stress at
section B-B.
Given: Distance from support to:
Point D d  8  in Section A-A a  10 in Centroid of B-B b  18.5 in
Depth of section h  3  in Thickness of section t  0.5 in
Applied load P  5000 lbf
Assumptions: The bracket remains flat and does not buckle (out-of-plane) under the applied load.

Solution: See Mathcad file P0460.

1. Calculate the cross-section area and moment of inertia at sections A-A and B-B, which are the same.
3
2 t h 4
A  h  t A  1.500  in I  I  1.1250 in
12

2. For parts (a) and (b), draw a free-body diagram of the portion of the bracket that is to the right of section A-A.

a A

M h y

A
x

d D

3. Use the equilibrium equations 3.3a to calculate the shear force and bending moment on section A-A.

 F: V  P  0  M: P ( a  d )  M  0

V  P V  5000 lbf M  P ( a  d ) M  10000  in lbf


4. (a) The maximum normal stress in the bracket at section A-A is determined using equation 4.11b. It is located a
the bottom of the section and is oriented in the positive x direction, i.e., it is tensile.

Distance from neutral axis to extreme fiber c  0.5 h c  1.500  in


© 2011 Pearson Education, Inc., Upper Saddle River, NJ. All rights reserved. This publication is protected by Copyright and written permission should be
MACHINE DESIGN - An Integrated Approach, 4th Ed. 4-60-2

M c
Maximum normal stress σmax  σmax  13.33  ksi
I
5. (b) The maximum shear stress in the bracket at section A-A is either at the neutral axis (due to the transverse
shear, which is a maximum at the NA) or it is at the top or bottom of the section (due to the bending stress at
those points, which is numerically the same).

3 V
At the neutral axis, using equation 4.14b τmax   τmax  5.000  ksi
2 A

At the bottom edge the stress state is: σx  σmax, σy  0  ksi, τxy  0  ksi. Using equation 4.6a, the principal
stresses are

2
σx  σy  σx  σy  2
σ1      τxy σ1  13.333 ksi
2  2 
2
σx  σy  σx  σy  2
σ2      τxy σ2  0.000  ksi
2  2 

σ3  0  ksi

And, from equation 4.6b, the maximum shear stress is

σ1  σ3
τmax  τmax  6.667  ksi
2

As seen from the Mohr's Circle in Figure 4-8, this stress is oriented 45 degrees from the positive x axis.

6. For parts (c) and (d), draw a free-body diagram of the portion of the bracket that is below section B-B.

F y

M
B B x

d D h

7. Use the equilibrium equations 3.3a to calculate the normal force and bending moment on section B-B.

 F: F  P  0  M: P ( b  d )  M  0

© 2011 Pearson Education, Inc., Upper Saddle River, NJ. All rights reserved. This publication is protected by Copyright and written permission should be
MACHINE DESIGN - An Integrated Approach, 4th Ed. 4-60-3

F  P F  5000 lbf M  P ( b  d ) M  52500  in lbf

8. (c) The maximum normal stress in the bracket at section B-B is a combination of uniform tension and bending
and is determined by summing equations 4.7 and 4.11b. It is located at the left edge of the section and is
oriented in the positive y direction, i.e., it is tensile.

Distance from neutral axis to extreme fiber c  0.5 h c  1.500  in

M c F
Maximum normal stress σmax   σmax  73.33  ksi
I A

9. (d) The maximum shear stress in the bracket at section B-B is at the left edge of the section (due to the
combined tensile and bending stresses). Since there is no transverse shear on this section, the shear stress
at the neutral axis is zero.

At the left edge the stress state is: σx  0  ksi, σy  σmax, τxy  0  ksi. Using equation 4.6a, the principal
stresses are

2
σx  σy  σx  σy  2
σ1      τxy σ1  73.333 ksi
2  2 
2
σx  σy  σx  σy  2
σ2      τxy σ2  0.000  ksi
2  2 

σ3  0  ksi

And, from equation 4.6b, the maximum shear stress is

σ1  σ3
τmax  τmax  36.667 ksi
2

As seen from the Mohr's Circle in Figure 4-8, this stress is oriented 45 degrees from the positive x axis.

© 2011 Pearson Education, Inc., Upper Saddle River, NJ. All rights reserved. This publication is protected by Copyright and written permission should be
MACHINE DESIGN - An Integrated Approach, 4th Ed. 4-61-1

PROBLEM 4-61 _____


Statement: For the bracket of Problem 4-60, solve for the deflection of point C.
Given: Distance from support to:
Point D d  8  in Point C a  18.5 in
Depth of section h  3  in Thickness of section t  0.5 in
6
Applied load P  5000 lbf Modulus of elasticity E  30 10  psi

Assumptions: 1. The bracket remains flat and does not buckle (out-of-plane) under the applied load.
2. The bracket can be modeled using its centroidal axis length dimensions.

Solution: See Mathcad file P0461.

1. Calculate the moment of inertia along the segment AC.


3
t h 4
I  I  1.1250 in
12

2. Draw idealized free-body diagrams of the portions of the bracket from the support to point C and from point
C to point D.

MA MC
C x
A

P
d

C
MC

3. Calculate the magnitude of the moments on segment AC using equilibrium equation 3.3a.

MC  P ( a  d ) MC  52500  in lbf

MA  P d MA  40000  in lbf


© 2011 Pearson Education, Inc., Upper Saddle River, NJ. All rights reserved. This publication is protected by Copyright and written permission should be
MACHINE DESIGN - An Integrated Approach, 4th Ed. 4-61-2

4. From inspection of the FBD, write the load function equation


q(x) = -MA<x - 0>-2 + P<x - 0>-1 - P<x - a>-1 - MC<x - a>-2

5. Integrate this equation from - to x to obtain shear, V(x)

V(x) = -MA<x - 0>-1 + P<x - 0>0 - P<x - a>0 - MC<x - a>-1

6. Integrate this equation from - to x to obtain moment, M(x)

M(x) = -MA<x - 0>0 + P<x - 0>1 - P<x - a>1 - MC<x - a>0

7. Integrate the moment function, multiplying by 1/EI, to get the slope.


(x) = [-MA<x-0>1 + P<x - 0>2/2 - P<x - a>2/2 - MC<x-a>1 + C3]/EI

8. Integrate again to get the deflection.


y(x) = [-MA<x-0>2/2 + P<x - 0>3/6 - P<x - a>3/6 - MC<x-a>2/2 + C3x +C4]/EI

9. Evaluate C3 and C4. At x = 0,  = 0 and y = 0, therefore, C3 = 0 and C4 = 0.

10. Evaluate  and y at x = a using the equations in steps 7 and 8, respectively.

  MA a 
1 P 2
θC  a  θC  0.196  deg
E I  2 

1  MA 2 P 3
yC   a  a  yC  0.0465 in
E I  2 6 

© 2011 Pearson Education, Inc., Upper Saddle River, NJ. All rights reserved. This publication is protected by Copyright and written permission should be
MACHINE DESIGN - An Integrated Approach, 4th Ed. 4-62-1

PROBLEM 4-62 _____


Statement: Figure P4-23 shows a 1-in-dia steel bar supported and subjected to the applied load P = 500 lb.
Solve for the deflection at the load and the slope at the roller support.
6
Given: Diameter d  1.00 in Modulus of elasticity E  30 10  psi
Applied load P  500  lbf Dimensions: a  20 in L  40 in
Solution: See Mathcad file P0462.
1. Draw a free-body diagram.

R2

M1

R1 P

2. This is a statically indeterminate beam because there are three unknown reactions, R1, M1, and R2. To solve
for these unknowns, follow the method presented in Example 4-7. First, calculate the moment of inertia for
the round section.
4
π d 4
I  I  0.0491 in
64
3. From inspection of the FBD, write the load function equation

q(x) = -M1<x>-2 + R1<x>-1 - R2<x - a>-1 + P<x - L>-1


4. Integrate this equation from - to x to obtain shear, V(x)
V(x) = -M1<x>-1 + R1<x>0 - R2<x - a>0 + P<x - L>0
5. Integrate this equation from - to x to obtain moment, M(x)
M(x) = -M1<x>0 + R1<x>1 - R2<x - a>1 + P<x - L>1
6. Integrate the moment function, multiplying by 1/EI, to get the slope.
(x) = [ -M1<x>1 + R1<x>2/2 - R2<x - a>2/2 + P<x - L>2/2 + C3]/EI
7. Integrate again to get the deflection.
y(x) = [-M1<x>2/2 + R1<x>3/6 - R2<x - a>3/6 + P<x - L>3/6 + C3x + C4]/EI

8. Evaluate R1, M1, R2, C3 and C4


At x = 0, y = 0 and  = 0, therefore, C3 = 0 and C4 = 0.
At x = a, y = 0
At x = L+, V = M = 0
Guess M1  1000 in lbf R1  500  lbf R2  1000 lbf

M1 2 R1 3 3
Given y(a) = 0:  a   a = 0  lbf  in
2 6

V(L) = 0: R1  R2  P = 0  lbf

© 2011 Pearson Education, Inc., Upper Saddle River, NJ. All rights reserved. This publication is protected by Copyright and written permission should be
MACHINE DESIGN - An Integrated Approach, 4th Ed. 4-62-2

M(L) = 0: M1  R1 L  R2 ( L  a ) = 0  lbf  in

 M1 
 
 R1   Find  M1 R1 R2 M1  5000 in lbf R1  750  lbf
R 
 2 R2  1250 lbf

9. Evaluate y at x = L to get the deflection at the load.

1  M1 2 R1 3 R2 3
yL   L  L   ( L  a)  yL  1.584  in
E I  2 6 6 

10. Evaluate  at x = a to get the slope at the roller support.

1  R1 2
θA    M1 a  a  θA  0.0340 rad
E I  2 

© 2011 Pearson Education, Inc., Upper Saddle River, NJ. All rights reserved. This publication is protected by Copyright and written permission should be
MACHINE DESIGN - An Integrated Approach, 4th Ed. 4-63-1

PROBLEM 4-63 _____


Statement: Figure P4-24 shows a 1.25-in-dia solid steel shaft with several twisting couples applied in the
directions shown. For TA = 10000 lb-in, TB = 20000 lb-in, TC = 30000 lb-in, find:
(a) The magnitude and location of the maximum shear stress in the shaft.
(b) The corresponding principal stresses for the location determined in part (a).
(c) The magnitude and location of the maximum shear strain in the shaft.
6
Given: Shaft diameter d  1.25 in Modulus of rigidity G  11.7 10  psi
Torque magnitudes: TA  10 kip in TB  20 kip in TC  30 kip in
Segment lengths: LAB  18 in LBC  12 in LCD  10 in
Solution: See Mathcad file P0463.
1. Looking at the shaft from the left end (A), TA and TC are clockwise (negative) and TB is counterclockwise
(positive). For the shaft to be in equilibrium, the applied torques must sum to zero. Write the equilibrium
equation and solve for the unknown reaction TD.
TA  TB  TC  TD  0

TD  TA  TB  TC TD  20 kip in

2. The net torque on each shaft segment is now


TAB  TA TAB  10 kip in

TBC  TAB  TB TBC  10 kip in

TCD  TBC  TC TCD  20 kip in

3. Calculate the outside radius and the polar moment of inertia of the shaft.
4
d π d 4
r  r  0.625  in J  J  0.240  in
2 32
4. (a) Since the shaft is uniform in cross-section, the maximum shear stress will occur in that segment that has
the largest absolute value of torque applied to it. In this case, that is segment CD. Use equation 4.23b to
calculate the maximum shear stress in segment CD.

TCD  r
τmax  τmax  52.2 ksi
J

5. (b) Mohr's circle for pure shear is centered at 0,0 and has a radius equal to the shear stress on the stress
element. Thus, for this case, the two nonzero principal stresses are

σ1  τmax σ1  52.2 ksi σ3  τmax σ3  52.2 ksi

The third principal stress is zero, σ2  0  ksi

6. (c) The shear strain in any given segment is proportional to the shear stress so the maximum shear strain
will occur in segment CD, where the shear stress is a maximum. Hooke's law for shear is similar to that
given in equation 2.2.
τmax 3
γmax  γmax  4.46  10  rad
G

© 2011 Pearson Education, Inc., Upper Saddle River, NJ. All rights reserved. This publication is protected by Copyright and written permission should be
MACHINE DESIGN - An Integrated Approach, 4th Ed. 4-64-1

PROBLEM 4-64 _____


Statement: If the shaft of Problem 4-63 were rigidly attached to fixed supports at each end (A and D) and
loaded only by the couples TB and TC, then find:
(a) The reactions TA and TD at each end of the shaft.
(b) The rotation of section B with respect to section C.
(c) The magnitude and location of the maximum shear strain.
6
Given: Shaft diameter d  1.25 in Modulus of rigidity G  11.7 10  psi
Torque magnitudes: TB  20 kip in TC  30 kip in
Segment lengths: LAB  18 in LBC  12 in LCD  10 in

Solution: See Mathcad file P0464.

1. Calculate the outside radius and the polar moment of inertia of the shaft.
4
d π d 4
r  r  0.625  in J  J  0.240  in
2 32
2. (a) Looking at the shaft from the left end (A), TC is clockwise (negative) and TB is counterclockwise
(positive). For the shaft to be in equilibrium, the applied torques must sum to zero. Since there are two
unknown reactions in the equilibrium equation, we cannot solve for them without another equation. An
equation that expresses the fact that the total rotational deflection from A to D is zero is called the
compatibility equation. Write the equilibrium and compatibility equations and solve for the unknown
reactions TA and TD.

TA  TB  TC  TD  0

TA LAB TA  TB LBC TD LCD


θAB  θBC  θCD  0     0
J G J G J G

Guess TA  10 kip in TD  30 kip in

Given TA  TB  TC  TD = 0  kip in

TA LAB TA  TB LBC TD LCD


   = 0  rad
J G J G J G

 TA 
   Find  TA TD TA  3.50 kip in clockwise
 TD 
TD  13.50  kip in counterclockwise

3. The net torque on each shaft segment is now


TAB  TA TAB  3.5 kip in

TBC  TAB  TB TBC  16.5 kip in

TCD  TBC  TC TCD  13.5 kip in

4. (b) Use equation 4.24 to calculate the rotation of section B with respect to C.

© 2011 Pearson Education, Inc., Upper Saddle River, NJ. All rights reserved. This publication is protected by Copyright and written permission should be
MACHINE DESIGN - An Integrated Approach, 4th Ed. 4-64-2

TBC LBC
θBC   θBC  0.0706 rad θBC  4.045  deg
J G

5. (c) Since the shaft is uniform in cross-section, the maximum shear stress will occur in that segment that has the
largest absolute value of torque applied to it. In this case, that is segment BC. Use equation 4.23b to calculate
the maximum shear stress in segment BC.

TBC  r
τmax  τmax  43.0 ksi
J

The shear strain in any given segment is proportional to the shear stress so the maximum shear strain will
occur in segment BC, where the shear stress is a maximum. Hooke's law for shear is similar to that given in
equation 2.2.

τmax 3
γmax  γmax  3.68  10  rad
G

© 2011 Pearson Education, Inc., Upper Saddle River, NJ. All rights reserved. This publication is protected by Copyright and written permission should be
MACHINE DESIGN - An Integrated Approach, 4th Ed. 4-65-1

PROBLEM 4-65 _____


Statement: Figure P4-25 shows a pivot pin that is press-fit into part A and is slip fit in part B. If F = 100 lb
and l = 1.50 in, what pin diameter is needed to limit the maximum stress in the pin to 50 kpsi?
Given: Applied force F  100  lbf Maximum stress σ  50 ksi
Total length, l l  1.50 in Beam length L  0.5 l

Assumptions: 1. Since there is a slip fit between the pin and part B, part B offers no resistance to bending of
the pin and, since the pin is press-fit into part A, it can be modeled as a cantilever beam of
length l/2.
2. Part B distributes the concentrated force F so that, at the pin, it is uniformly distributed over
the exposed length of the pin.

Solution: See Mathcad file P0465.

1. Calculate the intensity of the uniformly distributed load acting over the length of the pin.
F lbf
w  w  133.3 
L in

2. A cantilever beam with uniform loading is shown in Figure B-1(b) in Appendix B. In this case, the dimension
a in the figure is zero. As shown in the figure, when a = 0, the maximum bending moment occurs at the
support and is
2
w L
Mmax  Mmax  37.50  lbf  in
2

3. The bending stress in a beam is given in equation 4.11c, which can be solved for the required section
modulus, Z.

Mmax 4 3
Z  Z  7.500  10  in
σ
3
I π d
where, for a round cross-section z= = Solving for d,
c 32
1
3
 32 Z 
d min   π  d min  0.197  in
 

© 2011 Pearson Education, Inc., Upper Saddle River, NJ. All rights reserved. This publication is protected by Copyright and written permission should be
MACHINE DESIGN - An Integrated Approach, 4th Ed. 4-66-1

PROBLEM 4-66 _____


Statement: Figure P4-25 shows a pivot pin that is press-fit into part A and is slip fit in part B. If F = 100 N
and l = 64 mm, what pin diameter is needed to limit the maximum stress in the pin to 250 MPa?
Given: Applied force F  100  N Maximum stress σ  250  MPa
Total length, l l  64 mm Beam length L  0.5 l
Assumptions: 1. Since there is a slip fit between the pin and part B, part B offers no resistance to bending of
the pin and, since the pin is press-fit into part A, it can be modeled as a cantilever beam of
length l/2.
2. Part B distributes the concentrated force F so that, at the pin, it is uniformly distributed over
the exposed length of the pin.

Solution: See Mathcad file P0466.

1. Calculate the intensity of the uniformly distributed load acting over the length of the pin.
F N
w  w  3.125 
L mm

2. A cantilever beam with uniform loading is shown in Figure B-1(b) in Appendix B. In this case, the dimension
a in the figure is zero. As shown in the figure, when a = 0, the maximum bending moment occurs at the
support and is
2
w L
Mmax  Mmax  1600.0 N  mm
2

3. The bending stress in a beam is given in equation 4.11c, which can be solved for the required section
modulus, Z.

Mmax 3
Z  Z  6.400  mm
σ
3
I π d
where, for a round cross-section z= = Solving for d,
c 32
1
3
 32 Z 
d min   π  d min  4.0 mm
 

© 2011 Pearson Education, Inc., Upper Saddle River, NJ. All rights reserved. This publication is protected by Copyright and written permission should be
MACHINE DESIGN - An Integrated Approach, 4th Ed. 4-67-1
PROBLEM 4-67

Statement: Figure P4-25 shows a pivot pin that is press-fit into part A and is slip fit in part B. Determine
the l/d ratio that will make the pin equally strong in shear and bending if the shear strength is
equal to one-half the bending strength.

Assumptions: 1. Since there is a slip fit between the pin and part B, part B offers no resistance to bending of
the pin and, since the pin is press-fit into part A, it can be modeled as a cantilever beam of
length l/2.
2. Part B distributes the concentrated force F so that, at the pin, it is uniformly distributed
over the exposed length of the pin.

Solution: See Mathcad file P0467.

1. The intensity of the uniformly distributed load acting over the exposed length of the pin is
2 F
w 
l

2. A cantilever beam with uniform loading is shown in Figure B-1(b) in Appendix B. In this case, the
dimension a in the figure is zero. As shown in the figure, when a = 0, the maximum bending moment for
a beam of length L occurs at the support and is

2 2
w L 1 2 F  l  F l
Mmax = =    =
2 2 l 2 4

3. From equation 4.11c, the bending stress is

M  F  l    32  = 8  F  l
σmax = =  4   3
Z    π d  π d 3

4. Figure B-1(b) in Appendix B shows that the maximum shear occurs at the support and, for a = 0, is

 2 F    l  = F
Vmax = w L =  l  2
  
5. From equation 4.15c, the maximum shear stress due to the transverse loading is

4 V 4 4 16 F
τmax =  = F  =
3 A 3 2 2
π d 3  π d

6. For equal shear and bending strength, let the shear stress equal one half the bending stress.

16 F 1 8 F  l
= 
2 2 3
3  π d π d

l 4
Solving for l/d, =
d 3

© 2011 Pearson Education, Inc., Upper Saddle River, NJ. All rights reserved. This publication is protected by Copyright and written permission should be
MACHINE DESIGN - An Integrated Approach, 4th Ed. 4-69-1

PROBLEM 4-69
Statement: Figure P4-26a shows a C-clamp with an elliptical body dimensioned as shown. The clamp has a
T-section with a uniform thickness of 3.2 mm at the throat as shown in Figure P4-26b. Find the
bending stress at the inner and outer fibers of the throat if the clamp force is 2.7 kN.

Given: Clamping force F  2.7 kN


Distance from center of screw to throat ri  63.5 mm
Section dimensions: Flange b  28.4 mm Web h  31.8 mm t  3.2 mm

Solution: See Figure P4-26 and Mathcad file P0469.


1. Determine the location of the CG of the T-section and the distance from the centerline of the screw to the
centroid of the section at the throat.
0.5 t ( b  t)  0.5 ( h  t)  ( h  t) t
yCG  yCG  9.58 mm
b t  ( h  t)  t

rc  ri  yCG rc  73.08  mm

2. Using equation 4.12a and Figure 4-16, calculate the distance to the neutral axis, rn, and the distance from the
centroidal axis to the neutral axis, e.

Distance from the screw centerline to the outside fiber ro  ri  h ro  95.30  mm


2
Cross section area A  b  t  ( h  t)  t A  182.4  mm

A
Distance to neutral axis rn  rn  71.86  mm
ri t ro
 b  t
 dr   dr
 r  r
r r  t
i i

Distance from centroidal to neutral axis e  rc  rn e  1.21 mm

3. Take a section through the throat area and draw a FBD. There will be a vertical axial force through the
section CG (at a distance rc from the screw centerline) which will form a couple of magnitude rc x F. This
couple will be balanced by an internal moment of equal magnitude.
5
Internal moment M  rc F M  1.97  10  N  mm

4. Calculate the distances from the neutral axis to the inner and outer fibers.
ci  rn  ri ci  8.364  mm co  ro  rn co  23.436 mm

5. Using equations 4.12d and 4.12e, calculate the stresses at the inner and outer fibers of the throat section.

M  ci  F
σi    σi  132.2  MPa
e A  ri  A

M  co  F
σo     σo  204.3  MPa
e A  ro  A

© 2011 Pearson Education, Inc., Upper Saddle River, NJ. All rights reserved. This publication is protected by Copyright and written permission should be
MACHINE DESIGN - An Integrated Approach, 4th Ed. 4-70-1

PROBLEM 4-70
Statement: A C-clamp as shown in Figure P4-26a has a rectangular cross section as in Figure P4-26c. Find
the bending stress at the inner and outer fibers of the throat if the clamping force is 1.6 kN.
Given: Clamping force F  1.6 kN
Distance from center of screw to throat ri  63.5 mm
Section dimensions: Width b  6.2 mm Depth h  31.8 mm

Solution: See Figure P4-26 and Mathcad file P0470.

1. Determine the distance from the centerline of the screw to the centroid of the section at the throat.
h
rc  ri  rc  79.40  mm
2

2. Using equation 4.12a and Figure 4-16, calculate the distance to the neutral axis, rn, and the distance from the
centroidal axis to the neutral axis, e.

Distance from the screw centerline to the outside fiber ro  ri  h ro  95.30  mm


2
Cross section area A  b  h A  197.160  mm

A
Distance to neutral axis rn  rn  78.33  mm
ro
 b
 dr
 r
r
i

Distance from centroidal to neutral axis e  rc  rn e  1.07 mm

3. Take a section through the throat area and draw a FBD. There will be a vertical axial force through the
section CG (at a distance rc from the screw centerline) which will form a couple of magnitude rc x F. This
couple will be balanced by an internal moment of equal magnitude.
5
Internal moment M  rc F M  1.27  10  N  mm

4. Calculate the distances from the neutral axis to the inner and outer fibers.
ci  rn  ri ci  14.827 mm co  ro  rn co  16.973 mm

5. Using equations 4.12d and 4.12e, calculate the stresses at the inner and outer fibers of the throat section.

M  ci  F
σi    σi  148.3  MPa
e A  ri  A

M  co  F
σo     σo  98.8 MPa
e A  ro  A

© 2011 Pearson Education, Inc., Upper Saddle River, NJ. All rights reserved. This publication is protected by Copyright and written permission should be
MACHINE DESIGN - An Integrated Approach, 4th Ed. 4-71-1

PROBLEM 4-71
Statement: A C-clamp as shown in Figure P4-26a has an elliptical cross section as in Figure P4-26d.
Dimensions of the major and minor axes of the ellipse are given. Determine the bending stress
at the inner and outer fibers of the throat if the clamping force is 1.6 kN.
Given: Clamping force F  1.6 kN
Distance from center of screw to throat ri  63.5 mm
Section dimensions: Width b  9.6 mm Depth h  31.8 mm

Solution: See Figure P4-26 and Mathcad file P0471.

1. Determine the distance from the centerline of the screw to the centroid of the section at the throat.
h
rc  ri  rc  79.40  mm
2

2. Using equation 4.12a and Figure 4-16, calculate the distance to the neutral axis, rn, and the distance from the
centroidal axis to the neutral axis, e.

Distance from the screw centerline to the outside fiber ro  ri  h ro  95.30  mm


b h 2
Cross section area A  π  A  239.766  mm
2 2

A
Distance to neutral axis rn  rn  78.595 mm
ro
 0.5
  2
 2 b
 1   r  rc 
 4 2 
  h  dr
 r
r
i
Distance from centroidal to neutral axis e  rc  rn e  0.805  mm

3. Take a section through the throat area and draw a FBD. There will be a vertical axial force through the
section CG (at a distance rc from the screw centerline) which will form a couple of magnitude rc x F. This
couple will be balanced by an internal moment of equal magnitude.
5
Internal moment M  rc F M  1.27  10  N  mm

4. Calculate the distances from the neutral axis to the inner and outer fibers.
ci  rn  ri ci  15.095 mm co  ro  rn co  16.705 mm

5. Using equations 4.12d and 4.12e, calculate the stresses at the inner and outer fibers of the throat section.

M  ci  F
σi    σi  163.2  MPa
e A  ri  A

M  co  F
σo     σo  108.7  MPa
e A  ro  A

© 2011 Pearson Education, Inc., Upper Saddle River, NJ. All rights reserved. This publication is protected by Copyright and written permission should be
MACHINE DESIGN - An Integrated Approach, 4th Ed. 4-72-1

PROBLEM 4-72
Statement: A C-clamp as shown in Figure P4-26a has a trapezoidal cross section as in Figure P4-26e.
Determine the bending stress at the inner and outer fibers of the throat if the clamping force is
1.6 kN.
Given: Clamping force F  1.6 kN
Distance from center of screw to throat ri  63.5 mm
Section dimensions: Width b i  9.6 mm b o  3.2 mm Depth h  31.8 mm

Solution: See Figure P4-26 and Mathcad file P0472.

1. Determine the distance from the centerline of the screw to the centroid of the section at the throat.
h bi  2 bo
rc  ri   rc  76.75  mm
3 bi  bo

2. Using equation 4.12a and Figure 4-16, calculate the distance to the neutral axis, rn, and the distance from the
centroidal axis to the neutral axis, e.

Distance from the screw centerline to the outside fiber ro  ri  h ro  95.30  mm


bi  bo 2
Cross section area A  h A  203.520  mm
2

A
Distance to neutral axis rn  rn  75.771 mm
ro

 bi  bo
 bi    r  ri
h
 dr
 r
r
i
Distance from centroidal to neutral axis e  rc  rn e  0.979  mm

3. Take a section through the throat area and draw a FBD. There will be a vertical axial force through the
section CG (at a distance rc from the screw centerline) which will form a couple of magnitude rc x F. This
couple will be balanced by an internal moment of equal magnitude.
5
Internal moment M  rc F M  1.228  10  N  mm

4. Calculate the distances from the neutral axis to the inner and outer fibers.
ci  rn  ri ci  12.271 mm co  ro  rn co  19.529 mm

5. Using equations 4.12d and 4.12e, calculate the stresses at the inner and outer fibers of the throat section.

M  ci  F
σi    σi  126.9  MPa
e A  ri  A

M  co  F
σo     σo  118.4  MPa
e A  ro  A

© 2011 Pearson Education, Inc., Upper Saddle River, NJ. All rights reserved. This publication is protected by Copyright and written permission should be
MACHINE DESIGN - An Integrated Approach, 4th Ed. 4-73-1

PROBLEM 4-73
Statement: We want to design a C-clamp with a T-section similar to the one shown in Figure P4-26. The
depth of the section will be 31.8 mm as shown but the width of the flange (shown as 28.4 mm)
is to be determined. Assuming a uniform thickness of 3.2 mm and a factor of safety against
static yielding of 2, determine a suitable value for the width of the flange if the C-clamp is to be
made from 60-40-18 ductile iron and the maximum design load is 1.6 kN.

Given: Maximum clamping force F  1.6 kN


Distance from center of screw to throat ri  63.5 mm
Section dimensions: Web h  31.8 mm t  3.2 mm
Factor of safety N  2 Yield strength S y  324  MPa

Solution: See Figure P4-26 and Mathcad file P0473.

1. Determine the location of the CG of the T-section and the distance from the centerline of the screw to the
centroid of the section at the throat as functions of the unknown flange width, b.
0.5 t ( b  t)  0.5 ( h  t)  ( h  t) t
yCG ( b ) 
b  t  ( h  t)  t

rc( b )  ri  yCG ( b )

2. Using equation 4.12a and Figure 4-16, calculate the distance to the neutral axis, rn, and the distance from the
centroidal axis to the neutral axis, e,as functions of b.

Distance from the screw centerline to the outside fiber ro  ri  h ro  95.3 mm

Cross section area A ( b )  b  t  ( h  t)  t

A (b)
Distance to neutral axis rn( b ) 
r t r
 i b  o t
 dr   dr
 r  r
r r t
i i

Distance from centroidal to neutral axis e( b )  rc( b )  rn( b )

3. Take a section through the throat area and draw a FBD. There will be a vertical axial force through the
section CG (at a distance rc from the screw centerline) which will form a couple of magnitude rc x F. This
couple will be balanced by an internal moment of equal magnitude.
Internal moment M ( b )  rc( b )  F

4. Calculate the distances from the neutral axis to the inner and outer fibers.
ci( b )  rn( b )  ri co( b )  ro  rn( b )

5. Using equations 4.12d and 4.12e, calculate the stresses at the inner and outer fibers of the throat section.

M (b)  ci( b )  F
σi( b )    
e( b )  A ( b )  ri  A ( b )

6. Set the tensile stress on the inner fiber equal to the yield strength divided by the factor of safety and solve
for the flange width, b.
© 2011 Pearson Education, Inc., Upper Saddle River, NJ. All rights reserved. This publication is protected by Copyright and written permission should be
MACHINE DESIGN - An Integrated Approach, 4th Ed. 4-73-2

Guess b  12 mm

Sy
Given σi( b ) = b  Find ( b ) b  10.13  mm
N

7. Using the calculated value of b, check the stresses at the inner and outer fibers..

M (b)  ci( b )  F
σi( b )     σi( b )  162  MPa
e( b )  A ( b )  ri  A ( b )

M ( b)  c o( b )  F
σo( b )     σo( b )  149.4  MPa
e( b )  A ( b )  ro  A ( b )

A suitable minimum value for the flange width is


b  10.1 mm

© 2011 Pearson Education, Inc., Upper Saddle River, NJ. All rights reserved. This publication is protected by Copyright and written permission should be
MACHINE DESIGN - An Integrated Approach, 4th Ed. 4-74-1

PROBLEM 4-74
Statement: A round steel bar is 10 in long and has a diameter of 1 in.
(a) Calculate the stress in the bar when it is subjected to a 1000-lb force in tension.
(b) Calculate the bending stress in the bar if it is fixed at one end (as a cantilever beam) and
has a 1000-lb transverse load at the other end.
(c) Calculate the transverse shear stress in the bar of part (b).
(d) Calculate the torsional shear stress when the 1000-lb force is displaced 10 inches radially
from the centerline (axis) of the cantilever beam.
(e) Calculate the maximum bending stress in the bar if it is formed into a semicircle with a
centroidal radius of 10/ in and 1000-lb opposing forces are applied at the ends in the plane of
the of the ends. Assume that there is no distortion of the cross section during bending.
(f) Calculate the direct bearing stress that would result on the bar of (a) if it were the pin in a
pin-and-clevis connection that is subjected to a 1000-lb pull if the center part (the eye or
tongue) is 1-in wide.
(g) Determine how short the bar must be when loaded as a cantilever beam for its maximum
flexural bending stress and its maximum transverse shear stress to provide equal tendency to
failure. Find the length as a fraction of the diameter if the failure stress in shear is half the
failure stress in bending.
(h) If the force on the cantilever beam in (f) is eccentric, inducing torsional as well as bending
stress, what fraction of the diameter would the eccentricity need to be in order to give a
torsional stress equal to the transverse shear stress?

Given: Length of bar L  10 in Diameter d  1.00 in


Force F  1000 lbf Load Radius R  10 in

Solution: See Mathcad file P0474.

(a) Use equation 4.7 to calculate the axial stress.


2
d 2
Cross sectional area A  π A  0.785  in
4

F
Axial stress σ  σ  1.27 ksi
A

(b) The beam loading diagram is shown in Appendix Figure B-1a with the concentrated load at a = L. The
maximum bending stress occurs at x = 0 and is given by Equation 4.11b.

Bending moment M  L F M  10000  in lbf


d
Radius of bar c  c  0.5 in
2
4
d 4
Moment of inertia I  π I  0.049  in
64

M c
Maximum bending stress σ  σ  101.9  ksi
I

(c) The maximum transverse shear stress occurs at y = 0 and is given by Equation 4.15c and in Figure 4-20b.

4 F
Maximum transverse shear stress in a solid, round bar τmax   τmax  1.70 ksi
3 A

(d) The maximum torsional shear shear stress occurs at y = d/2 and is given by Equation 4.23b.
© 2011 Pearson Education, Inc., Upper Saddle River, NJ. All rights reserved. This publication is protected by Copyright and written permission should be
MACHINE DESIGN - An Integrated Approach, 4th Ed. 4-74-2

Twisting torque T  F  R T  10000  in lbf


4
d 4
Polar moment of inertia J  π J  0.098  in
32

Td
Max torsional shear stress τ  τ  50.93  ksi
2 J

(e) The maximum bending stress for a curved beam occurs at r = ri and is given by Equation 4.12d.

10
Centroidal radius rc   in rc  3.183  in
π

Inside radius ri  rc  0.5 d ri  2.683  in

Outside radius ro  rc  0.5 d ro  3.683  in


2
d 2
Cross section area A  π A  0.785  in
4

A
Distance to neutral axis rn  rn  3.163  in
r
 o 2
  d   r  r 2
 2   c
 2 dr
 r
r
i

Distance from centroid to neutral axis e  rc  rn e  0.020  in

Internal moment M  rc F M  3183 in lbf


Distances from the neutral axis to the inner and outer fibers
ci  rn  ri ci  0.480  in co  ro  rn co  0.520  in

Stress at the inner fibers of the throat section

M  ci  F
σi    σi  37.9 ksi
e A  ri  A

(f) The direct bearing stress is given in Equations 4.7 and 4.10.

Given length of bearing contact l  1  in


2
Projected area of contact Abearing  l d Abearing  1  in

F
Bearing stress σbearing  σbearing  1.0 ksi
Abearing

(g) Determine how short the bar must be when loaded as a cantilever beam for its maximum flexural bending
stress and its maximum transverse shear stress to provide equal tendency to failure. Find the length as a
fraction of the diameter if the failure stress in shear is half the failure stress in bending.

© 2011 Pearson Education, Inc., Upper Saddle River, NJ. All rights reserved. This publication is protected by Copyright and written permission should be
MACHINE DESIGN - An Integrated Approach, 4th Ed. 4-74-3

M c 32 F  L
Bending stress σ= =
I 3
π d

4 F 16 F
Transverse shear τ=  =
3 A 2
3  π d

32 F  L 32 F
= 
Equating σ = 2 τ 3 3 2
π d π d

d
Solving for L L=
3

(h) If the force on the cantilever beam in (f) is eccentric, inducing torsional as well as bending stress, what
fraction of the diameter would the eccentricity need to be in order to give a torsional stress equal to the
transverse shear stress?

Tc 16 F  e
Torsional shear stress τtor = =
J 3
π d

4 F 16 F
Transverse shear τtrans =  =
3 A 2
3  π d

16 F  e 16 F
Equating τtor = τtrans =
3 2
π d 3  π d

d
Solving for the eccentricity, e e=
3

© 2011 Pearson Education, Inc., Upper Saddle River, NJ. All rights reserved. This publication is protected by Copyright and written permission should be
MACHINE DESIGN - An Integrated Approach, 4th Ed. 4-75a-1

PROBLEM 4-75a
Statement: For a filleted flat bar in tension similar to that shown in Appendix Figure C-9 and the data from
row a from Table P4-4, determine the nominal stress, the geometric stress concentration factor,
and the maximum axial stress in the bar.

Given: Widths D  40 mm d  20 mm


Thickness h  10 mm Radius r  4  mm
Force P  8000 N

Solution: See Appendix Figure C-9 and Mathcad file P0475a.

1. Determine the nominal stress in the bar using equation 4.7.

P
σnom  σnom  40.0 MPa
h d

2. Determine the geometric stress concentration factor using Appendix Figure C-9.

D
Width ratio  2.00
d

From Figure E-9 A  1.0966 b  0.32077


b
Kt  A  
r
SCF  Kt  1.838
d

3. Determine the maximum stress in the bar using equation 4.31.

σmax  Kt σnom σmax  73.5 MPa

© 2011 Pearson Education, Inc., Upper Saddle River, NJ. All rights reserved. This publication is protected by Copyright and written permission should be
MACHINE DESIGN - An Integrated Approach, 4th Ed. 4-76a-1

PROBLEM 4-76a
Statement: For a filleted flat bar in bending similar to that shown in Appendix Figure C-10 and the data from
row a from Table P4-4, determine the nominal stress, the geometric stress concentration factor,
and the maximum bending stress in the bar.

Given: Widths D  40 mm d  20 mm


Thickness h  10 mm Radius r  4  mm
Moment M  80 N  m

Solution: See Appendix Figure C-10 and Mathcad file P0476a.

1. Determine the nominal stress in the bar using equation 4.11b.


3
d h d 3 4
c  c  10 mm I  I  6.667  10  mm
2 12
M c
σnom  σnom  120.0  MPa
I

2. Determine the geometric stress concentration factor using Appendix Figure C-10.

D
Width ratio  2.00
d

From Figure E-9 A  0.93232 b  0.30304


b
Kt  A  
r
SCF  Kt  1.518
d

3. Determine the maximum stress in the bar using equation 4.31.

σmax  Kt σnom σmax  182.2  MPa

© 2011 Pearson Education, Inc., Upper Saddle River, NJ. All rights reserved. This publication is protected by Copyright and written permission should be
MACHINE DESIGN - An Integrated Approach, 4th Ed. 4-77a-1

PROBLEM 4-77a
Statement: For a shaft, with a shoulder fillet, in tension similar to that shown in Appendix Figure C-1 and
the data from row a from Table P4-4, determine the nominal stress, the geometric stress
concentration factor, and the maximum axial stress in the shaft.

Given: Widths D  40 mm d  20 mm


Radius r  4  mm
Force P  8000 N

Solution: See Appendix Figure C-1 and Mathcad file P0477a.

1. Determine the nominal stress in the bar using equation 4.7.

4 P
σnom  σnom  25.5 MPa
2
π d
2. Determine the geometric stress concentration factor using Appendix Figure C-1.

D
Width ratio  2.00
d

From Figure E-1 A  1.01470 b  0.30035


b
Kt  A  
r
SCF  Kt  1.645
d

3. Determine the maximum stress in the bar using equation 4.31.

σmax  Kt σnom σmax  41.9 MPa

© 2011 Pearson Education, Inc., Upper Saddle River, NJ. All rights reserved. This publication is protected by Copyright and written permission should be
MACHINE DESIGN - An Integrated Approach, 4th Ed. 4-78a-1

PROBLEM 4-78a
Statement: For a shaft, with a shoulder fillet, in bending similar to that shown in Appendix Figure C-2 and
the data from row a from Table P4-4, determine the nominal stress, the geometric stress
concentration factor, and the maximum bending stress in the shaft.

Given: Widths D  40 mm d  20 mm


Radius r  4  mm
Moment M  80 N  m

Solution: See Appendix Figure C-2 and Mathcad file P0478a.

1. Determine the nominal stress in the bar using equation 4.11b.


4
d π d 3 4
c  c  10 mm I  I  7.854  10  mm
2 64
M c
σnom  σnom  101.9  MPa
I

2. Determine the geometric stress concentration factor using Appendix Figure C-2.

D
Width ratio  2.00
d

From Figure E-2 A  0.90879 b  0.28598


b
Kt  A  
r
SCF  Kt  1.44
d

3. Determine the maximum stress in the bar using equation 4.31.

σmax  Kt σnom σmax  146.7  MPa

© 2011 Pearson Education, Inc., Upper Saddle River, NJ. All rights reserved. This publication is protected by Copyright and written permission should be
MACHINE DESIGN - An Integrated Approach, 4th Ed. 4-79-1
PROBLEM 4-79
Statement: A differential stress element has a set of applied stresses on it as shown in Figure 4-1. For σx =
850, σy = -200, σz = 300, τxy = 450, τyz = -300, and τzx = 0; find the principal stresses and maximum
shear stress and draw the Mohr's circle diagram for this three-dimensional stress state.

Given: σx  850 σy  200 σz  300


τxy  450 τyz  300 τzx  0
Solution: See Figure 4-1 and Mathcad file P04079.

1. Calculate the coefficients (stress invariants) of equation (4.4c).

C2  σx  σy  σz C2  950.000

 σx τxy   σx τzx   σy τyz  5


C1          C1  2.675  10
 τxy σy   τzx σz   τyz σz 

 σx τxy τzx 
  8
C0   τxy σy τyz  C0  1.882  10
τ τ σ 
 zx yz z 
3 2
2. Find the roots of the triaxial stress equation: σ  C2 σ  C1 σ  C0 = 0

 C0 
   470 
v   C1 
r  polyroots ( v) r  388 

 C2   
   1032 
 1 
 CW
3. Extract the principal stresses from
the vector r by inspection.
 1-3
σ1  r σ1  1032
3

σ2  r σ2  388 500
2
 2-3  1-2
σ3  r σ3  470
1
-500 500 1000 1500
4. Using equations (4.5), evaluate 
3 0 2 1
the principal shear stresses.
σ1  σ3
τ13  τ13  751
2
500
σ1  σ2
τ12  τ12  322
2
σ2  σ3
τ23  τ23  429
2  CCW

5. Draw the three-circle Mohr diagram. FIGURE 4-79


The Three Mohr's Circles for Problem 4-79
© 2011 Pearson Education, Inc., Upper Saddle River, NJ. All rights reserved. This publication is protected by Copyright and written permission should be
MACHINE DESIGN - An Integrated Approach, 4th Ed. 4-80-1

PROBLEM 4 - 80
Statement: Write expressions for the normalized (stress/pressure) tangential stress as a function of the
normalized wall thickness (wall thickness/outside radius) at the inside wall of a thick-wall
cylinder and for a thin-wall cylinder, both with internal pressure only. Plot the ratio of these two
expressions and determine the range of the wall thickness to outside radius-ratio for which the
stress predicted by the thin-wall expression is at least 5% greater than that predicted by the
thick-wall expression.

Solution: See Mathcad file P0480.


1. Let the σt/p ratio be S' and the t/ro ratio be t', then
For the thick-wall cylinder at the inside wall, using equation 4.48a
2
2  2  t'  t'
S'thick ( t') 
2
2  t'  t'

and, for the thin-wall cylinder, using equation 4.49a


1
S'thin ( t') 
t'

2. Choose a range for the normalized thickness ratio, t'  0.01 0.02  0.99

S'thin( t')  S'thick( t') 


3. Plot the difference between the two functions. ∆ ( t') 
S'thick ( t')

25

20

15

∆ ( t')
%

10

0
0 0.1 0.2 0.3 0.4 0.5 0.6 0.7 0.8 0.9 1
t'

© 2011 Pearson Education, Inc., Upper Saddle River, NJ. All rights reserved. This publication is protected by Copyright and written permission should be
MACHINE DESIGN - An Integrated Approach, 4th Ed. 4-80-2

4. Determine the values of t' for which the difference is 5%.

∆ ( 0.10)  5.0 % ∆ ( 0.946 )  5.1 %

5. The range of the normalized thickness for which the thin-wall stress is at least 5% greater than the thick-wall
stress is from 0.10 to 0.946.

© 2011 Pearson Education, Inc., Upper Saddle River, NJ. All rights reserved. This publication is protected by Copyright and written permission should be
MACHINE DESIGN - An Integrated Approach, 4th Ed. 4-81-1

PROBLEM 4 - 81
Statement: A hollow square torsion bar such as that shown in Table 4-3 has dimensions a = 25 mm, t = 3
mm, and l = 300 mm. If it is made of steel with a modulus of rigidity of G = 80.8 GPa, determine
the maximum shear stress in the bar and the angular deflection under a torsional load of 500
N-m.

Given: Dimensions a  25 mm t  3  mm l  300  mm


Modulus G  80.8 GPa Load T  500  N  m
Solution: See Table 4-3 Mathcad file P0481.

1. Calculate the factors K and Q for a hollow square from Table 4-3.

2 4
2  t  ( a  t) 4
K  K  31944  mm
2
2 a t  2 t

2 3
Q  2  t ( a  t) Q  2904 mm

2. Using equation 4.26a, calculate the maximum shear stress.

T
τmax  τmax  172.2  MPa
Q

3. Using equation 4.26b, calculate the angular deflection.

Tl
θ  θ  0.058 radians
K G
θ  3.33 deg

© 2011 Pearson Education, Inc., Upper Saddle River, NJ. All rights reserved. This publication is protected by Copyright and written permission should be
MACHINE DESIGN - An Integrated Approach, 4th Ed. 4-82-1

PROBLEM 4 - 82
Statement: Design a hollow rectangular torsion bar such as that shown in Table 4-3 that has dimensions a
= 45 mm, b = 20 mm, and l = 500 mm. It is made of steel with a shear yield strength of 90 MPa
and has an applied torsional load of 135 N-m. Use a factor of safety against yielding of 2.

Given: Dimensions a  45 mm b  20 mm l  500  mm


Modulus G  80.8 GPa Load T  135  N  m
Shear yield strength S sy  90 MPa Factor of safety N  2

Solution: See Table 4-3 Mathcad file P0482.

1. Calculate the Q-factor for a hollow rectangle from Table 4-3.

Q( t)  2  t ( a  t)  ( b  t)

2. Calculate the maximum shear stress as a function of thickness, t, using equation 4.26a.
T
τ( t) 
Q( t)

3. Define a function that relates the maximum shear stress to the shear strength divided by the factor of safety
and solve for the thickness, t.

Guess a value of t t  3  mm

S sy
Define the design function f ( t)  τ( t) 
N

t  root( f ( t) t) t  1.927  mm

Let t = 2 mm (note that this solution does not check for buckling under the applied load)

© 2011 Pearson Education, Inc., Upper Saddle River, NJ. All rights reserved. This publication is protected by Copyright and written permission should be
MACHINE DESIGN - An Integrated Approach, 4th Ed. 4-83-1

PROBLEM 4 - 83
Statement: A pressure vessel with closed ends has the following dimensions: outside diameter, OD = 450
mm, and wall thickness, t = 6 mm. If the internal pressure is 690 kPa, find the principal stresses
on the inside surface away from the ends. What is the maximum shear stress at the point
analyzed?

Given: Dimensions OD  450  mm t  6  mm


Pressure p  690  kPa
Solution: See Mathcad file P0483.

1. Convert the given dimensions to inside and outside radii.


ro  0.5 OD ro  225  mm
ri  ro  t ri  219  mm

2. Determine whether to use thick-wall or thin-wall theory.


ro
 22.5 mm
10

Since the wall thickness, t  6  mm, is much less than one tenth the outside radius, use thin wall theory.

3. Calculate the principal stresses using equations 4.49.


p  ro
Tangential (y-direction) σt  σt  25.9 MPa
t

Radial (x-direction) σr  0  MPa σr  0.0 MPa

p  ro
Axial (z-direction) σa  σa  12.9 MPa
2 t

The principal stresses are:

σ1  σt σ1  25.9 MPa

σ2  σa σ2  12.9 MPa

σ3  σr σ3  0.0 MPa

4. Using equation 4.6b, calculate the maximum shear stress.

σ1  σ3
τmax  τmax  12.9 MPa
2

© 2011 Pearson Education, Inc., Upper Saddle River, NJ. All rights reserved. This publication is protected by Copyright and written permission should be
MACHINE DESIGN - An Integrated Approach, 4th Ed. 4-84-1
PROBLEM 4-84
Statement: A simply supported steel beam of length, l, with a concentrated load, F, acting at midspan has a
rectangular cross-section with width, b, and depth, h. If the strain energy due to transverse
shear loading is Us and that due to bending loading is Ub, derive an expression for the ratio
Us/Ub and plot it as a function of h/l over the range 0 to 0.10.
Solution: See Mathcad file P0484.
l

3  V
2
1. From equation 4.22e, the strain energy in transverse loading is: Us =   dx
5  G A

0

l

1  M
2
2. From equation 4.22d, the strain energy in bending loading is: Ub =   dx
2  E I

0

l
 2
 V dx

Us 6 E I 0
3. Let U' = , then: U' =  
Ub 5 G A l
 2
 M dx

0
3
b h
4. For a rectangular cross-section: A = b h and I=
12

l
 2
 V dx
2 
E 5 h 0
5. And, for steel: = therefore U' = 
G 2 4 l
 2
 M dx

0
6. For the given loading:

F F x
For x between 0 and l/2, V= and M=
2 2

F F x F l
For x between l/2 and l, V= and M= 
2 2 2

7. Substituting these expressions into the equation for U' and integrating gives:

  0.5 l 
l 
 2  2
   dx
F    dx
F

h 
2   2 2 12.0 12.0 
    
2    
   2 2  2
h  0
 
0.5 l  
 l l 

6.0 h
4  0.5 l l  4 2
  l
   dx 
F  x
2 
 F  x F  l 
2 
  2    2  2  dx

   
   
 0 0.5 l 

© 2011 Pearson Education, Inc., Upper Saddle River, NJ. All rights reserved. This publication is protected by Copyright and written permission should be
MACHINE DESIGN - An Integrated Approach, 4th Ed. 4-84-2

h 2
8. Let h' = then U'( h')  6  h'
l

9. Plotting the strain energy ratio over the range: h'  0 0.001  0.10

STRAIN ENERGY RATIO vs DEPTH TO LENGTH RATIO


6

5
Strain Energy Ratio - Percent

U'( h' )
3
%

0
0 0.02 0.04 0.06 0.08 0.1
h'
Depth to Length Ratio

© 2011 Pearson Education, Inc., Upper Saddle River, NJ. All rights reserved. This publication is protected by Copyright and written permission should be
MACHINE DESIGN - An Integrated Approach, 4th Ed. 4-85a-1
PROBLEM 4-85a
Statement: A beam is supported and loaded as shown in Figure P4-27(a). Find the reactions for the data
given in row a from Table P4-2.
Given: Beam length L  1  m
Distance to R2 a  0.4 m
1
Distributed load magnitude w  200  N  m

a
w

R1 R2 R3

FIGURE 4-85A
Free Body Diagram for Problem 4-85

Solution: See Figure P4-27(a) and Mathcad file P0485a.

1. Consider the reaction force R1 to be redundant and remove it temporarily. The beam will then be statically
determinant and will deflect at x = 0. Now consider the reaction force R1 to be an unkown applied load that
will force the deflection to be zero. Write an equation for the deflection at x = 0 in terms of the force R1 with
the deflection set to zero.
2. Write equation 4.21 for the deflection y1 at the unknown applied load R1 in terms of the strain energy in

the beam at that point: y1 = U
R1

L

 M  
3. Substitute equation 4.22d and differentiate: y1 =   M  dx (a)
E I

  R1 
0

4. Write an expression for the bending moment and its partial derivative with respect toR1 as a function of x.
2
w x 
For x between 0 and a, M = R 1 x  M =x (b)
2 R1
2
w x 
For x between a and l, M = R 1 x   R 2 ( x  a ) M =x (c)
2 R1
5. Substitute equations (b) and (c) into (a), set equal to zero and integrate.

© 2011 Pearson Education, Inc., Upper Saddle River, NJ. All rights reserved. This publication is protected by Copyright and written permission should be
MACHINE DESIGN - An Integrated Approach, 4th Ed. 4-85a-2

a L
 
   
2   2 
  R1 x  w x   x dx   R1 x  w x  R2 ( x  a)  x dx = 0

  2  
  2 
0 a

6. Solving for R1 and R2 and summing forces and moments about x = 0:

 L3 a  L2 a 3 a3  4
  R2  w  L = 0
R1
m  
3
From strain energy   
3 3 2 3 2  8

Summing forces R1  R2  R3  w  L = 0

2
w L
Summing moments R 2 a  R 3 L  =0
2
7. Use these three equations to solve for R1, R2, and R3. Guess R1  65 N R2  70 N R3  65 N

Given
 L3 a  L2 a 3 a3  4
  R2  w  L = 0
R1
m  
3
  
3 3 2 3 2  8

R1  R2  R3  w  L = 0
2
w L
R 2 a  R 3 L  =0
2

 10.714 
R  Find  R1 R2 R3 R  148.81   N

 
 40.476 

R  10.7 N R  148.8  N R  40.5 N


1 2 3

© 2011 Pearson Education, Inc., Upper Saddle River, NJ. All rights reserved. This publication is protected by Copyright and written permission should be
MACHINE DESIGN - An Integrated Approach, 4th Ed. 4-86a-1
PROBLEM 4-86a
Statement: A beam is supported and loaded as shown in Figure P4-27(b). Find the reactions for the data
given in row a from Table P4-2.
Given: Beam length L  1.0 m
Distance to R2 a  0.4 m
1
Distributed load magnitude w  200  N  m
Distance to concentrated load b  0.6 m
Concentrated load F  500  N

L
b
a
F
w

R1 R2 R3

FIGURE 4-86A
Free Body Diagram for Problem 4-86

Solution: See Figure P4-27(b) and Mathcad file P0485a.

1. Consider the reaction force R1 to be redundant and remove it temporarily. The beam will then be statically
determinant and will deflect at x = 0. Now consider the reaction force R1 to be an unkown applied load that
will force the deflection to be zero. Write an equation for the deflection at x = 0 in terms of the force R1 with
the deflection set to zero.
2. Write equation 4.21 for the deflection y1 at the unknown applied load R1 in terms of the strain energy in

the beam at that point: y1 = U
R1

L

 M  
3. Substitute equation 4.22d and differentiate: y1 =   M  dx (a)
E I

  R1 
0

4. Write an expression for the bending moment and its partial derivative with respect toR1 as a function of x.
2
w x 
For x between 0 and a, M = R 1 x  M =x (b)
2 R1

M = R1 x  w a   x 
a 
For x between a and b,   R 2 ( x  a ) M =x (c)
 2 R1

© 2011 Pearson Education, Inc., Upper Saddle River, NJ. All rights reserved. This publication is protected by Copyright and written permission should be
MACHINE DESIGN - An Integrated Approach, 4th Ed. 4-86a-2

M = R1 x  w a   x 
a
For x between b and L,   R 2 ( x  a )  F  ( x  b ) (d)
 2


M =x
R1

5. Substitute equations (b), (c) and (d) into (a), set equal to zero and integrate.
a

   
2 
b
  R1 x  w x   x dx   R  x  w a   x  a 
 1    R2 ( x  a )  x dx  = 0

  2  
   2 
0 a
L

 R  x  w a   x  a 
 1    R2 ( x  a )  F  ( x  b )  x dx

   2 
b

6. Solving for R1 and R2 and summing forces and moments about x = 0:

 L3   3 2 3
From strain energy    R1   L  a  L  a   R2  =0
3 3 2 6 
 a2 L2 a L3 a4   3 2 3
     w   L  b L  b   F
 6 3 24  3 2 6 

Summing forces R1  R2  R3  w  a  F = 0
2
w a
Summing moments R 2 a  R 3 L   F b = 0
2

7. Use these three equations to solve for R1, R2, and R3. Guess R1  100  N R2  400  N R3  200  N

Given
 L3   3 2 3
   R1   L  a  L  a   R2  =0
3 3 2 6 
 a2 L2 a L3 a4   L3 b  L2 b 3 
     w     F
 6 3 24  3 2 6 

R1  R2  R3  w  a  F = 0
2
w a
R 2 a  R 3 L   F b = 0
2

 81.143 
R  Find  R1 R2 R3 R   575.238   N
 
 85.905 

R  81.1 N R  575.2  N R  85.9 N


1 2 3

© 2011 Pearson Education, Inc., Upper Saddle River, NJ. All rights reserved. This publication is protected by Copyright and written permission should be
MACHINE DESIGN - An Integrated Approach, 4th Ed. 5-1a-1
PROBLEM 5-1a
Statement: A differential stress element has a set of applied stresses on it as indicated in row a of Table P5-1.
For row a, draw the stress element showing the applied stresses. Find the principal stresses and
the von Mises stress.

Given: σx  1000 σy  0 σz  0
τxy  500 τyz  0 τzx  0

Solution: See Figure 5-1a and Mathcad file P0501a.

1. Draw the stress element, indicating the x and y axes.


500
2. From Problem 4-1a, the principal stresses are
y
σ1  1207 σ2  0 σ3  207 x 1000

3. Using equatoion 5.7c, the von Mises stress is

2 2 FIGURE 5-1aA
σ'  σ1  σ1 σ3  σ3 σ'  1323 Stress Element for Problem 5-1a

© 2011 Pearson Education, Inc., Upper Saddle River, NJ. All rights reserved. This publication is protected by Copyright and written permission should be
MACHINE DESIGN - An Integrated Approach, 4th Ed. 5-1h-1
PROBLEM 5-1h
Statement: A differential stress element has a set of applied stresses on it as indicated in each row of Table
P5-1. For row h, draw the stress element showing the applied stresses, find the principal stresses
and the von Mises stress.

Given: σx  750 σy  500 σz  250


τxy  500 τyz  0 τzx  0

Solution: See Figure 5-1h and Mathcad file P0501h.

z
1. Draw the stress element (see Figure 5-1h).
250
2. From Problem 4-1h, the principal stresses are

σ1  1140 σ2  250 σ3  110

3. Using equation 5.7, the von Mises stress is

750
  σ1  σ2   σ2  σ3   σ1  σ3
1 2 2 2 500 500 500
σ'  
2 x y

σ'  968
FIGURE 5-1h
Stress Element for Problem 5-1h

© 2011 Pearson Education, Inc., Upper Saddle River, NJ. All rights reserved. This publication is protected by Copyright and written permission should be
MACHINE DESIGN - An Integrated Approach, 4th Ed. 5-2-1
PROBLEM 5-2
Statement: A 400-lb chandelier is to be hung from two 10-ft-long solid, low- carbon steel cables in tension.
Size the cables for a safety factor of 4. State all assumptions.
Given: Weight of chandelier W  400  lbf
Length of cable L  10 ft L  120 in
Design Safety factor Nd  4
Number of cables N  2
6
Young's modulus E  30 10  psi

Assumptions: The material is AISI 1010 hot-rolled steel with S y  26 ksi
Solution: See Mathcad file P0502.

W
1. Determine the load on each cable P  P  200 lbf
N
Sy
2. Using the distortion-energy failure theory, Nd =
σ'

3. In this case, the only stress in the axial direction is the tensile stress. Therefore, this is the principal stress and
also the von Mises stress.
4 P
σ' = σ1 = σ =
2
π d
4. Substitute the equation in step 3 into the design equation in step 2 and solve for the minimum diameter, d.
1
2
 4  P N d 
d    d  0.198 in
 π S y 
5. Round up to an available size (see Table 13-2) and check the actual factor of safety against static failure.

2
π d  S y
d  0.207  in Ns  Ns  4.4
4 P

© 2011 Pearson Education, Inc., Upper Saddle River, NJ. All rights reserved. This publication is protected by Copyright and written permission should be
MACHINE DESIGN - An Integrated Approach, 4th Ed. 5-3-1
PROBLEM 5-3
Statement: For the bicycle pedal-arm assembly in Figure P5-1 with rider-applied force of 1500 N at the pedal,
determine the von Mises stress in the 15-mm-dia pedal arm. The pedal attaches to the arm with a
12-mm thread. Find the von Mises stress in the screw. Find the safety factor against static failure
if the material has S y = 350 MPa.

Given: Distances (see figure) a  170  mm b  60 mm


Rider-applied force Frider  1.5 kN Screw thread diameter d sc  12 mm
Pedal arm diameter d pa  15 mm Material yield strength S y  350  MPa

Solution: See Figures 5-3 and Mathcad file P0503. z

1. From problem 4-3, the maximum principal


stresses in the pedal arm are at point A and
are a
C Tc
σ1  793  MPa σ2  0  MPa Frider
Mc
b
σ3  23 MPa Arm

2. Using equation 5.7c, the von Mises Fc y


stress is Pedal

2 2
σ'  σ1  σ1 σ3  σ3 x

FIGURE 5-3A
σ'  805 MPa Free Body Diagram for Problem 5-3

3. The factor of safety for the pedal arm is


z
Sy
N  N  0.43
σ'

4. From Problem 4-3 solution, the stresses Section C


at the top of the screw where it joins the A
pedal arm are
B
σx  530.5  MPa σz  0  MPa Arm

τzx  0  MPa

5. From this, we see that the principal x


stresses are y
σ1  σx σ2  0  MPa
FIGURE 5-3B
σ3  0  MPa Points A and B at Section C

6. The von Mises stress is σ'  σ1 σ'  530.5 MPa

Sy
7. The factor of safety for the screw is N  N  0.66
σ'

© 2011 Pearson Education, Inc., Upper Saddle River, NJ. All rights reserved. This publication is protected by Copyright and written permission should be
MACHINE DESIGN - An Integrated Approach, 4th Ed. 5-4-1
PROBLEM 5-4
Statement: The trailer hitch shown in Figure P4-2 and Figure 1-1 (p. 12) has loads applied as defined in
Problem 3-4. The tongue weight of 100 kg acts downward and the pull force of 4905 N acts
horizontally. Using the dimensions of the ball bracket shown in Figure 1-5 (p. 15) and S y = 300
MPa ductile steel, determine static safety factors for:
(a) The shank of the ball where it joins the ball bracket.
(b) Bearing failure in the ball bracket hole.
(c) Tearout failure in the ball bracket.
(d) Tensile failure in the 19-mm diameter attachment holes.
(e) Bending failure in the ball bracket as a cantilever.

Given: a  40 mm b  31 mm c  70 mm d  20 mm


Mtongue  100  kg Fpull  4.905  kN d sh  26 mm t  19 mm
S y  300  MPa

Assumptions: 1. The nuts are just snug-tight (no pre-load), which is the worst case.
2. All reactions will be concentrated loads rather than distributed loads or pressures.
Solution: See Figure 5-4 and Mathcad file P0504.

W tongue

70 = c

1 F pull 1

40 = a

2 A A F a1x
B 19 = t

F b1 B
31 = b
C F a1y
20 = d F a2y

D
Fa2x

F b2
Fc2x
C

D
Fd2

F c2y

FIGURE 5-4A
Dimensions and Free Body Diagram for Problem 5-4

1. From Problem 4-4, the principal stresses in the shank of the ball where it joins the ball bracket are:
σ1  114  MPa σ2  0  MPa σ3  0  MPa

2. Since 1 is the only nonzero principal stress, it is also the von Mises stress.The factor of safety against a static
failure at the shank of the ball is
© 2011 Pearson Education, Inc., Upper Saddle River, NJ. All rights reserved. This publication is protected by Copyright and written permission should be
MACHINE DESIGN - An Integrated Approach, 4th Ed. 5-4-2

Sy
σ'  σ1 Na  Na  2.6
σ'
3. From Problem 4-4, the principal stresses at the bearing area in the ball bracket hole are:
σ1  9.93 MPa σ2  0  MPa σ3  0  MPa

4. Since 1 is the only nonzero principal stress, it is also the von Mises stress.The factor of safety against a static
bearing failure in the ball bracket hole is

Sy
σ'  σ1 Nb  Nb  30.2
σ'

5. From Problem 4-4, the shear stress in the tearout area in Tearout length
the ball bracket is:

τ  4.41 MPa

2
6. For pure shear, the von Mises stress is σ'  3  τ
and the factor of safety against a static tearout failure is

Sy
Nc  Nc  39.3
σ'

7. From Problem 4-4, the principal stresses in the attachment


bolts if they are 19-mm diameter are: d R

σx  53.6 MPa σy  0  MPa FIGURE 5-4B


Tearout Diagram for Problem 5-4

τxy  1.7 MPa

8. The von Mises stress and the factor of safety against a static failure in the attachment bolts are:

2 2 2
σ'  σx  σy  σx σy  3  τxy

Sy
σ'  53.7 MPa Nd  Nd  5.6
σ'

9. From Problem 4-4, the principal stresses in the bracket due to bending in the ball bracket as a cantilever are:

σ1  72.8 MPa σ2  0  MPa σ3  0  MPa

10. Since 1 is the only nonzero principal stress, it is also the von Mises stress.The factor of safety against a static
bearing failure in the ball bracket hole is

Sy
σ'  σ1 Ne  Ne  4.1
σ'

© 2011 Pearson Education, Inc., Upper Saddle River, NJ. All rights reserved. This publication is protected by Copyright and written permission should be
MACHINE DESIGN - An Integrated Approach, 4th Ed. 5-5-1
PROBLEM 5-5
Statement: Repeat Problem 5-4 for the loading conditions of Problem 3-5, i.e., determine the horizontal force
that will result on the ball from accelerating a 2000-kg trailer to 60 m/sec in 20 sec. Assume a
constant acceleration. From Problem 3-5, the pull force is 6000 N. Determine static safety factors
for:
(a) The shank of the ball where it joins the ball bracket.
(b) Bearing failure in the ball bracket hole.
(c) Tearout failure in the ball bracket.
(d) Tensile failure in the 19-mm diameter attachment holes.
(e) Bending failure in the ball bracket as a cantilever.
Given: a  40 mm b  31 mm c  70 mm d  20 mm
Mtongue  100  kg Fpull  6  kN d sh  26 mm t  19 mm
S y  300  MPa

Assumptions: 1. The nuts are just snug-tight (no pre-load), which is the worst case.
2. All reactions will be concentrated loads rather than distributed loads or pressures.
Solution: See Figures 5-5 and Mathcad file P0505.

W tongue

70 = c

1 F pull 1

40 = a

2 A A F a1x
B 19 = t

F b1 B
31 = b
C F a1y
20 = d F a2y

D
Fa2x

F b2
Fc2x
C

D
Fd2

F c2y

FIGURE 5-5A
Dimensions and Free Body Diagram for Problem 5-5

1. From Problem 4-5, the principal stresses in the shank of the ball where it joins the ball bracket are:
σ1  139  MPa σ2  0  MPa σ3  0  MPa

2. Since 1 is the only nonzero principal stress, it is also the von Mises stress.The factor of safety against a static
failure at the shank of the ball is
© 2011 Pearson Education, Inc., Upper Saddle River, NJ. All rights reserved. This publication is protected by Copyright and written permission should be
MACHINE DESIGN - An Integrated Approach, 4th Ed. 5-5-2

Sy
σ'  σ1 Na  Na  2.2
σ'
3. From Problem 4-5, the principal stresses at the bearing area in the ball bracket hole are:
σ1  12.15  MPa σ2  0  MPa σ3  0  MPa

4. Since 1 is the only nonzero principal stress, it is also the von Mises stress.The factor of safety against a static
bearing failure in the ball bracket hole is

Sy
σ'  σ1 Nb  Nb  24.7
σ'

5. From Problem 4-5, the shear stress in the tearout area in Tearout length
the ball bracket is:

τ  5.4 MPa

2
6. For pure shear, the von Mises stress is σ'  3  τ
and the factor of safety against a static tearout failure is

Sy
Nc  Nc  32.1
σ'

7. From Problem 4-5, the principal stresses in the attachment


bolts if they are 19-mm diameter are: d R

σx  64.2 MPa σy  0  MPa FIGURE 5-5B


Tearout Diagram for Problem 5-5

τxy  1.7 MPa

8. The von Mises stress and the factor of safety against a static failure in the attachment bolts are:

2 2 2
σ'  σx  σy  σx σy  3  τxy

Sy
σ'  64.3 MPa Nd  Nd  4.7
σ'

9. From Problem 4-5, the principal stresses in the bracket due to bending in the ball bracket as a cantilever are:

σ1  85.1 MPa σ2  0  MPa σ3  0  MPa

10. Since 1 is the only nonzero principal stress, it is also the von Mises stress.The factor of safety against a static
bearing failure in the ball bracket hole is

Sy
σ'  σ1 Ne  Ne  3.5
σ'

© 2011 Pearson Education, Inc., Upper Saddle River, NJ. All rights reserved. This publication is protected by Copyright and written permission should be
MACHINE DESIGN - An Integrated Approach, 4th Ed. 5-6-1
PROBLEM 5-6
Statement: Repeat Problem 5-4 for the loading conditions of Problem 3-6, i.e., determine the horizontal force
that will results from an impact between the ball and the tongue of the 2000-kg trailer if the hitch
deflects 2.8 mm dynamically on impact. The tractor weighs 1000 kg and the velocity at impact is
m/sec. Determine static safety factors for:
(a) The shank of the ball where it joins the ball bracket.
(b) Bearing failure in the ball bracket hole.
(c) Tearout failure in the ball bracket.
(d) Tensile failure in the 19-mm diameter attachment holes.
(e) Bending failure in the ball bracket as a cantilever.
Given: a  40 mm b  31 mm c  70 mm d  20 mm
Mtongue  100  kg Fpull  55.1 kN d sh  26 mm t  19 mm
S y  300  MPa

Assumptions: 1. The nuts are just snug-tight (no pre-load), which is the worst case.
2. All reactions will be concentrated loads rather than distributed loads or pressures.
Solution: See Figures 5-6 and Mathcad file P0506.

W tongue

70 = c

1 F pull 1

40 = a

2 A A F a1x
B 19 = t

F b1 B
31 = b
C F a1y
20 = d F a2y

D
Fa2x

F b2
Fc2x
C

D
Fd2

F c2y

FIGURE 5-6A
Dimensions and Free Body Diagram for Problem 5-6

1. From Problem 4-6, the principal stresses in the shank of the ball where it joins the ball bracket are:
σ1  1277 MPa σ2  0  MPa σ3  0  MPa

2. Since 1 is the only nonzero principal stress, it is also the von Mises stress.The factor of safety against a static
failure at the shank of the ball is
© 2011 Pearson Education, Inc., Upper Saddle River, NJ. All rights reserved. This publication is protected by Copyright and written permission should be
MACHINE DESIGN - An Integrated Approach, 4th Ed. 5-6-2

Sy
σ'  σ1 Na  Na  0.23
σ'
3. From Problem 4-6, the principal stresses at the bearing area in the ball bracket hole are:
σ1  111.5  MPa σ2  0  MPa σ3  0  MPa

4. Since 1 is the only nonzero principal stress, it is also the von Mises stress.The factor of safety against a static
bearing failure in the ball bracket hole is

Sy
σ'  σ1 Nb  Nb  2.7
σ'

5. From Problem 4-6, the shear stress in the tearout area in Tearout length
the ball bracket is:

τ  49.6 MPa

2
6. For pure shear, the von Mises stress is σ'  3  τ
and the factor of safety against a static tearout failure is

Sy
Nc  Nc  3.5
σ'

7. From Problem 4-6, the principal stresses in the attachment


bolts if they are 19-mm diameter are: d R

σx  540.5  MPa σy  0  MPa FIGURE 5-6B


Tearout Diagram for Problem 5-6

τxy  1.7 MPa

8. The von Mises stress and the factor of safety against a static failure in the attachment bolts are:

2 2 2
σ'  σx  σy  σx σy  3  τxy

Sy
σ'  540.5 MPa Nd  Nd  0.56
σ'

9. From Problem 4-6, the principal stresses in the bracket due to bending in the ball bracket as a cantilever are:

σ1  635.5  MPa σ2  0  MPa σ3  0  MPa

10. Since 1 is the only nonzero principal stress, it is also the von Mises stress.The factor of safety against a static
bearing failure in the ball bracket hole is

Sy
σ'  σ1 Ne  Ne  0.47
σ'

© 2011 Pearson Education, Inc., Upper Saddle River, NJ. All rights reserved. This publication is protected by Copyright and written permission should be
MACHINE DESIGN - An Integrated Approach, 4th Ed. 5-7-1
PROBLEM 5-7
Statement: Design the wrist pin of Problem 3-7 for a safety factor of 3 and S y = 100 ksi if the pin is hollow and
loaded in double shear.

Given: Force on wrist pin Fwristpin  12.258 kN Fwristpin  2756 lbf


Yield strength S y  100  ksi
Design safety factor Nd  3

Assumptions: Choose a suitable outside diameter, say od  0.375  in

Solution: See Figure 4-12 in the text and Mathcad file P0507.

Fwristpin
1. The force at each shear plane is F  F  1378 lbf
2

2. With only the direct shear acting on the plane, the Mohr diagram will be a circle with center at the origin and
radius equal to the shear stress. Thus, the principal normal stress is numerically equal to the shear stress, which in
this case is also the principal shear stress, so we have  = 1 = '.

F 4 F
3. The shear stress at each shear plane is τ= = = σ'
A 
π od  id
2 
2

4. Using the distortion-energy failure theory,


Sy  2
π od  id  S y 
2
Nd = =
σ' 4 F

2 4  F  Nd
5. Solving for the inside diameter, id  od  id  0.297 in
π S y

6. Round this down to the decimal equivalent of a common fraction (9/32), id  0.281  in

 2
π od  id  S y
2 
7. The realized factor of safety is, N  N  3.5
4 F

© 2011 Pearson Education, Inc., Upper Saddle River, NJ. All rights reserved. This publication is protected by Copyright and written permission should be
MACHINE DESIGN - An Integrated Approach, 4th Ed. 5-8-1
PROBLEM 5-8
Statement: A paper mill processes rolls of paper having a density of 984 kg/m3. The paper roll is 1.50-m
outside diameter (OD) x 0.22-m inside diameter (ID) x 3.23-m long and is on a simple supported,
hollow, steel shaft with S y = 300 MPa. Find the shaft ID needed to obtain a static safety factor of 5
if the shaft OD is 22 cm.
Given: Paper roll:
kg y
Density ρ  984 
3
m
Outside dia. OD  1500 mm w

Inside dia. ID  220  mm x


Length L  3230 mm
R L R
Shaft:
V
Strength S y  300  MPa
R
Outside dia. od  220  mm L/2 L
0 x
Factor of safety Ns  5
-R
Assumptions: 1. The shaft is stiffer than the paper roll so the
weight of the roll on the shaft can be modelled as
a uniformly distributed load. M
2
wL /8
2. The bearings that support the shaft are close to
the ends of the paper roll and are thin with
respect to the length of the roll so we can
consider the distance between the shaft supports
to be the same as the length of the roll. 0 x
L/2 L

Solution: See Figure 5-8 and Mathcad file P0508.


FIGURE 5-8
Load, Shear, and Moment Diagrams
for Problem 5-8
1. The weight of the paper roll is,
Volume V 
π
4
 2
 OD  ID  L
2 V  5.585  m
3
(1)

Weight W  ρ  g  V W  53.895 kN (2)

2. From Figure 5-8, we see that the bending moment in the shaft is a maximum at the center of the span. First,
determine the magnitude of the distributed load, then find the maximum bending moment using Figure D-2(b) in
Appendix B with a = 0 and x = L/2.

W newton
Distributed load w  w  16.686 (3)
L mm
2
w L 7
Maximum moment Mmax  Mmax  2.176  10  newton  mm (4)
8
3. Using equation 4.11b, find the maximum bending stress as a function of the unkown shaft inside diameter, id.

Bending stress M c 32 Mmax od


σmax = = (5)
π  od  id 
at midspan I 4 4

4. This is the only stress element present at this point on the shaft and there is no shear stress at this point so max
= 1 and 2 = 3 = 0. Furthermore, since 2 and 3 are zero, max = '. Equation 5.8a can be used to find the
unknown id,
© 2011 Pearson Education, Inc., Upper Saddle River, NJ. All rights reserved. This publication is protected by Copyright and written permission should be
MACHINE DESIGN - An Integrated Approach, 4th Ed. 5-8-2

Sy
Factor of safety Ns = (6)
σ'
Substituting equation 5 into 6 and solving for id, we have
1
4
 π Sy od4  32 Ns Mmax od 
Shaft id id  id  198  mm (7)
 π S y 
 

© 2011 Pearson Education, Inc., Upper Saddle River, NJ. All rights reserved. This publication is protected by Copyright and written permission should be
MACHINE DESIGN - An Integrated Approach, 4th Ed. 5-9-1
PROBLEM 5-9
Statement: A ViseGrip plier-wrench is drawn to scale in Figure P5-3, and for which the forces were analyzed in
Problem 3-9 and the stresses in Problem 4-9, find the safety factors for each pin for an assumed
clamping force of P = 4000 N in the position shown. The pins are 8-mm dia, S y = 400 MPa, and are
all in double shear.
Given: Pin stresses as calculated in Problem 4-9:
Pin 1-2 τ12  74.6 MPa
Pin 1-4 τ14  50.7 MPa
Pin 2-3 τ23  50.7 MPa
Pin 3-4 τ34  50.7 MPa
Yield strength S y  400  MPa

Assumptions: Links 3 and 4 are in a toggle position, i.e., the pin that joins links 3 and 4 is in line with the pins th
join 1 with 4 and 2 with 3.
Solution: See Figure 5-9 and Mathcad file P0509.
1. The FBDs of the assembly and each individual link are shown in Figure 5-9. The dimensions, as scaled from
Figure P5-3 in the text, are shown on the link FBDs.

F 4
P
1

3 2
P

55.0 = b 50.0 = a
F14
39.5 = c 22.0 = d
F

129.2° 1 
4

F41 F34
F21 P


28.0 = e


P 2.8 = g


F43

3 F12
21.2 = h
F23 2
F32

26.9 = f
FIGURE 5-9
Free Body Diagrams for Problem 5-9

2. The pins are in pure shear, so the principal stresses are


© 2011 Pearson Education, Inc., Upper Saddle River, NJ. All rights reserved. This publication is protected by Copyright and written permission should be
MACHINE DESIGN - An Integrated Approach, 4th Ed. 5-9-2

Pin joining 1 and 2 σ'12  3  τ12 σ'12  129.211 MPa

All other pins σ'14  3  τ14 σ'14  87.815 MPa

3. Using the distortion-energy failure theory, the factors of safety are

Sy
Pin joining 1 and 2 N12  N12  3.1
σ'12

Sy
All other pins N14  N14  4.6
σ'14

© 2011 Pearson Education, Inc., Upper Saddle River, NJ. All rights reserved. This publication is protected by Copyright and written permission should be
MACHINE DESIGN - An Integrated Approach, 4th Ed. 5-10-1
PROBLEM 5-10
Statement: An over-hung diving board is shown in Figure P5-4a. Assume cross-section dimensions of 305 mm
x 32 mm. Find the largest principal stress in the board when a 100-kg person is standing at the free
end. What is the static safety factor if the material is brittle fiberglass with S ut = 130 MPa in the
longitudinal direction?

Given: Maximum principal stresses due to 2000 = L


bending at R2 from Problem 4-10
R1 P
σ1  24.5 MPa

σ2  0  MPa
R2
σ3  0  MPa
700 = a
Ultimate strength S ut  130  MPa
FIGURE 5-10
Free Body Diagram for Problem 5-10
Solution: See Figure 5-10 and Mathcad file P0510.
1. The diving board will be in tension at the top of the board and compression along the bottom. At the top, over
the right-hand support, the nonzero principal stress is positive and the load line on the 1-3 diagram is along
the 1 axis. Using the Modified-Mohr failure theory, the static safety factor is
S ut
Ns  Ns  5.3
σ1

© 2011 Pearson Education, Inc., Upper Saddle River, NJ. All rights reserved. This publication is protected by Copyright and written permission should be
MACHINE DESIGN - An Integrated Approach, 4th Ed. 5-11-1
PROBLEM 5-11
Statement: Repeat Problem 5-10 assuming the 100-kg person in Problem 5-10 jumps up 25 cm and lands back
on the board. Assume the board weighs 29 kg and deflects 13.1 cm statically when the person
stands on it. What is the static safety factor if the material is brittle fiberglass with S ut = 130 MPa i
the longitudinal direction?

2000 = L
Given: Maximum principal stresses due to
bending at R2 from Problem 4-11 R1 P
σ1  76.3 MPa

σ2  0  MPa R2

σ3  0  MPa 700 = a
Ultimate strength S ut  130  MPa
FIGURE 5-11
Free Body Diagram for Problem 5-11

Solution: See Figure 5-11 and Mathcad file P0511.


1. The diving board will be in tension at the top of the board and compression along the bottom. At the top, ove
the right-hand support, the nonzero principal stress is positive and the load line on the 1-3 diagram is along
the 1 axis. Using the Modified-Mohr failure theory, the static safety factor is
S ut
Ns  Ns  1.7
σ1

© 2011 Pearson Education, Inc., Upper Saddle River, NJ. All rights reserved. This publication is protected by Copyright and written permission should be
MACHINE DESIGN - An Integrated Approach, 4th Ed. 5-12-1
PROBLEM 5-12
Statement: Repeat Problem 5-10 using the cantilevered diving board design in Figure P5-4b.

2000
Given: Maximum principal stresses due to
bending at support from Problem 4-12 1300 = L

σ1  24.5 MPa P

σ2  0  MPa
M1
σ3  0  MPa R1
Ultimate strength S ut  130  MPa
700
Solution: See Figure 5-12 and Mathcad file P0512.
FIGURE 5-12
Free Body Diagram for Problem 5-12

1. The diving board will be in tension at the top of the board and compression along the bottom. At the top, at
the built-in support, the nonzero principal stress is positive and the load line on the 1-3 diagram is along the
1 axis. Using the Modified-Mohr failure theory, the static safety factor is
S ut
Ns  Ns  5.3
σ1

© 2011 Pearson Education, Inc., Upper Saddle River, NJ. All rights reserved. This publication is protected by Copyright and written permission should be
MACHINE DESIGN - An Integrated Approach, 4th Ed. 5-13-1
PROBLEM 5-13
Statement: Repeat Problem 5-11 using the cantilevered diving board design in Figure P5-4b. Assume the
board weighs 19 kg and deflects 8.5 cm statically when the person stands on it.

2000
Given: Maximum principal stresses due to
bending at support from Problem 4-13 1300 = L

σ1  87.1 MPa P

σ2  0  MPa
M1
σ3  0  MPa R1
Ultimate strength S ut  130  MPa
700
Solution: See Figure 5-13 and Mathcad file P0513.
FIGURE 5-13
Free Body Diagram for Problem 5-13

1. The diving board will be in tension at the top of the board and compression along the bottom. At the top, at the
built-in support, the nonzero principal stress is positive and the load line on the 1-3 diagram is along the 1
axis. Using the Modified-Mohr failure theory, the static safety factor is

S ut
Ns  Ns  1.5
σ1

© 2011 Pearson Education, Inc., Upper Saddle River, NJ. All rights reserved. This publication is protected by Copyright and written permission should be
MACHINE DESIGN - An Integrated Approach, 4th Ed. 5-14-1
PROBLEM 5-14
Statement: Figure P4-5 shows a child's toy called a pogo stick. The child stands on the pads, applying half he
weight on each side. She jumps off the ground, holding the pads up against her feet, and bounces
along with the spring cushioning the impact and storing energy to help each rebound. Design the
aluminum cantilever beam sections on which she stands to survive jumping 2 in off the ground
with a safety factor of 2. Use 1100 series aluminum. Define and size the beam shape.

Given: Cold rolled 1100 aluminum:


Yield strength S y  22 ksi
Safety factor Ns  2

Assumptions: The beam will have a rectangular


cross-section with the load applied at a
distance of 5 in from the central support.

L  5  in

Solution: See Figure 5-14 and Mathcad file P0514.


1. From Problem 3-14, the total dynamic force on both
foot supports is
Fi /2 Fi /2
Fi  224  lbf
Therefore, the load on each support is

Fi
P  P  112  lbf
2
2. To give adequate support to the childs foot, let the
width of the support beam be

w  1.5 in
3. From Figure B-1(a) in Appendix B, the maximum P
bending moment at x = 0 is
FIGURE 5-14
M  P L M  560  in lbf
Free Body Diagram for Problem 5-14

4. We can now calculate the minimum required section modulus, Z = I/c. Using the distortion-energy failure theor
the bending stress will also be the only nonzero principal stress, which will also be the von Mises stress.
Sy
Design equation Ns =
σ'

M Sy
Bending stress σ= = σ' =
Z Ns
N s M 3
Solving for Z, Z  Z  834.3  mm
Sy

3 2
w t t w t
5. For a rectangular cross-section, I= and c= so Z=
12 2 6

6 Z
Solving for t, t  t  0.451  in
w

Round this up to the next higher decimal equivalent of a common fraction, t  0.500  in
© 2011 Pearson Education, Inc., Upper Saddle River, NJ. All rights reserved. This publication is protected by Copyright and written permission should be
MACHINE DESIGN - An Integrated Approach, 4th Ed. 5-15-1
PROBLEM 5-15
Statement: What is the safety factor for the shear pin as defined in Problem 4-15?

Solution:
Any part whose stress equals its strength has a safety factor of 1 by definition.

© 2011 Pearson Education, Inc., Upper Saddle River, NJ. All rights reserved. This publication is protected by Copyright and written permission should be
MACHINE DESIGN - An Integrated Approach, 4th Ed. 5-16-1
PROBLEM 5-16
Statement: A track to guide bowling balls is designed with two round rods as shown in Figure P5-6. The rods
are not parallel to one another but have a small angle between them. The balls roll on the rods unt
they fall between them and drop onto another track. The angle between the rods is varied to cause
the ball to drop at different locations. Find the static safety factor for the 1-in dia SAE 1045
normalized steel rods.
(a) Assume rods are simply supported at each end.
(b) Assume rods are fixed at each end.

Given: Yield strength S y  58 ksi a Fball

Solution: See Figure 5-16 and Mathcad file P0516.

1. The maximum bending stress will occur at the outer R1 L R2


fibers of the rod at the section where the maximum
bending moment occurs which, in this case, is at x = a.
The only stress present on the top or bottom surface of FIGURE 5-16A
the rod is the bending stress x. Therefore, on the Free Body Diagram for Problem 5-16(a), taken
bottom surface where the stress is tensile, x is the on a plane through the rod axis and ball center
principal stress 1 . Thus, from Problem 4-16, for a simply
supported rod,
Maximum principal stress σ1  748  psi σ'a  σ1

2. Using the distortion-energy failure theory, the safety factor against a static failure is

Sy
Nsa  Nsa  78
σ'a

a Fball
3. For the built-in case, the maximum bending stress
will occur at the outer fibers of the rod at the section
where the maximum bending moment occurs which, in M1
this case, is at x = L. The only stress present on the R1 L R 2 M2
top or bottom surface of the rod is the bending stress
x. Therefore, on the bottom surface where the stress FIGURE 5-16B
Free Body Diagram for Problem 5-16(b), taken on a
is tensile, x is the principal stress 1 . Thus, from
plane through the rod axis and ball center
Problem 4-16, for a simply supported rod,

Maximum principal stress σ1  577  psi σ'b  σ1

4. Using the distortion-energy failure theory, the safety factor against a static failure is

Sy
Nsb  Nsb  101
σ'b

© 2011 Pearson Education, Inc., Upper Saddle River, NJ. All rights reserved. This publication is protected by Copyright and written permission should be
MACHINE DESIGN - An Integrated Approach, 4th Ed. 5-17-1
PROBLEM 5-17
Statement: A pair of ice tongs is shown in Figure P5-7. The ice weighs 50 lb and is 10 in wide across the
tongs. The distance between the handles is 4 in, and the mean radius r of the tong is 6 in. The
rectangular cross-sectional dimensions are 0.75 x 0.312 in. Find the safety factor for the tongs if
their S y = 30 ksi.
F

C
Given: Yield strength S y  30 ksi FC
3.5 = cy
O
Solution: See Problem 4-17, Figure 5-17, and FO
Mathcad file P0517.
11.0 = ax 2.0 = cx
1. The maximum bending stress in the tong was found
A
in Problem 4-17 at point A. 12.0 = by

Vertical direction σi  8.58 ksi 5.0 = bx

All other components are zero FB


B
2. There are no other stress components present so
W/2
σ1  σi σ2  0  ksi σ3  0  ksi
FIGURE 5-17
and σ'  σ1 σ'  8.58 ksi Free Body Diagram for Problem 5-17

Sy
3. The factor of safety is (using the distortion energy theory) N  N  3.5
σ'

© 2011 Pearson Education, Inc., Upper Saddle River, NJ. All rights reserved. This publication is protected by Copyright and written permission should be
MACHINE DESIGN - An Integrated Approach, 4th Ed. 5-18-1
PROBLEM 5-18
Statement: A pair of ice tongs is shown in Figure P5-7. The ice weighs 50 lb and is 10 in wide across the
tongs. The distance between the handles is 4 in, and the mean radius r of the tong is 6 in. The
rectangular cross-sectional dimensions are 0.75 x 0.312 in. Find the safety factor for the tongs if
they are made from Class 20 gray cast iron.
F
Given: Tensile strength S ut  22 ksi C
Compressive strength S uc  83 ksi FC
3.5 = cy
O
Solution: See Problem 4-18, Figure 5-18, and FO
Mathcad file P0518. 11.0 = ax 2.0 = cx

1. The maximum bending stress in the tong was found A


12.0 = by
in Problem 4-17 at point A.
5.0 = bx
Vertical direction σi  8.58 ksi
FB
All other components are zero
B
2. Therefore, the principal stresses are W/2
σ1  σi σ2  0  ksi σ3  0  ksi
FIGURE 5-18
Free Body Diagram for Problem 5-18
3. The load line on the 1-3 diagram is along the 1 axis.
Using the Modified-Mohr failure theory, the static safety
factor is
S ut
N  N  2.6
σ1

© 2011 Pearson Education, Inc., Upper Saddle River, NJ. All rights reserved. This publication is protected by Copyright and written permission should be
MACHINE DESIGN - An Integrated Approach, 4th Ed. 5-19-1
PROBLEM 5-19
Statement: Determine the size of the clevis pin, shown in Figure P5-8, needed to withstand an applied force
of 130 000 lb. Also determine the required outside radius of the clevis end to not fail in either tear
out or bearing if the clevis flanges are each 2.5 in thick. Use a safety factor of 3 for all modes of
failure. Assume S y = 89.3 ksi for the pin and S y = 35.5 ksi for the clevis.

Given: Applied force P  130  kip Clevis flange thickness t  2.50 in


Clevis strength S yclevis  35.5 ksi Pin strength S ypin  89.3 ksi
Safety factor Ns  3
Solution: See Figures P5-8 in the text and Mathcad file P0519.
1. Determine the force carried by each of the two flanges of the clevis.
F  0.5 P F  65 kip

This force is transmitted through each end of the clevis pin, which is in double shear.

4 3 F
2. The pin is in direct (pure) shear. Therefore, the von Mises stress is σ'pin = 3  τpin =
2
π d

3. Calculate the minimum required clevis pin diameter using the distortion-energy failure theory.
2
S ypin π d  S ypin
Ns = =
σ'pin 4 3 F

4  3  F  Ns
Solving for the pin diameter d  d  2.194 in
π S ypin

Round this up to the next higher decimal equivalent of a common fraction ( 2 1/4) d  2.250  in

4. Check the bearing stress in the clevis due to the pin on one side of the clevis.
2
Bearing stress area Ab  d  t Ab  5.625 in

Bearing force Fb  F Fb  65 kip

Fb
Bearing stress σb  σb  11.6 ksi
Ab
Tearout length
5. Determine the safety factor against a static bearing failure.

S yclevis
Nbear  Nbear  3.1
σb

Since this is greater than 3, the pin diameter is acceptable.

6. Determine the tearout stress in the clevis.

2 2 d R
Shear area (see Figure 5-19) Atear = 2  t R  ( 0.5 d )
FIGURE 5-19
Shear force Tearout Diagram for Problem 5-19

Ftear  F Ftear  65 kip


© 2011 Pearson Education, Inc., Upper Saddle River, NJ. All rights reserved. This publication is protected by Copyright and written permission should be
MACHINE DESIGN - An Integrated Approach, 4th Ed. 5-19-2

Ftear Ftear
Shear stress and distortion-energy equation τtear = =
Atear 2 2
2  t R  ( 0.5 d )

2 2
S yclevis S yclevis 2  t S yclevis R  ( 0.5 d )
Ns = = =
σ'tear 3  τtear 3  Ftear

2
 3  Ftear Ns  2
Solving for the clevis radius, R R     ( 0.5 d ) R  2.211 in
 2  t  S yclevis 

Round this up to the next higher decimal equivalent of a common fraction ( 2 1/4) R  2.250  in

2 2 2
The tearout area for each flange is Atear  2  t R  ( 0.5 d ) Atear  9.743 in

7. Design summary:

Pin diameter d  2.250 in Clevis flange radius R  2.25 in

© 2011 Pearson Education, Inc., Upper Saddle River, NJ. All rights reserved. This publication is protected by Copyright and written permission should be
MACHINE DESIGN - An Integrated Approach, 4th Ed. 5-20-1
PROBLEM 5-20
Statement: A 100 N-m torque is applied to a 1-m-long, solid, round shaft. Design it to limit its angular
deflection to 2 deg and select a steel alloy to have a yielding safety factor of 2.
Given: Applied torque T  100  N  m Shaft length L  1000 mm
Maximum deflection θmax  2  deg Modulus of rigidity G  79 GPa
Safety factor Ns  2
Assumptions: A ductile steel will be chosen.

Solution: See Mathcad file P0520.

1. Using the angular deflection requirement and equation (4.24), determine the required polar moment of inertia an
the minimum diameter.
TL TL 4 4
θ= J  J  3.626  10  mm
J G θmax G
1
4 4
π d  32 J 
J = d   π  d  24.653 mm
32  

Round this up to d  25 mm

2. Determine the shear stress at the outside diameter of the shaft using equation (4.23b).

T  
d

τmax 
2 τmax  34.47  MPa
J

3. For this case of pure shear, use the distortion-energy theory and equations (5.8) and (5.9) to solve for the
minimum required yield strength.

Sy Sy
Ns = = S y  3  τmax Ns S y  119.4  MPa
σ' 3  τmax

4. Using this value of S y, choose a steel from Table A-9 in Appendix A.

Any of the steels listed in Table A-9 will be adequate. The least expensive is AISI 1010, hot rolled.

© 2011 Pearson Education, Inc., Upper Saddle River, NJ. All rights reserved. This publication is protected by Copyright and written permission should be
MACHINE DESIGN - An Integrated Approach, 4th Ed. 5-21-1
PROBLEM 5-21
Statement: Figure P5-9 shows an automobile wheel with two common styles of lug wrench being used to
tighten the wheel nuts, a single-ended wrench in (a), and a double-ended wrench in (b). The
distance between points A and B is 1 ft in both cases and the handle diameter is 0.625 in. What is
the maximum force possible before yielding the handle if the material S y = 45 ksi?

Given: Distance between A and B d AB  1  ft


Wrench diameter d  0.625  in
Yield strength S y  45 ksi

Assumptions: 1. The forces exerted by the user's hands lie in a plane through the wrench that is also parallel to
the plane of the wheel.
2. The applied torque is perpendicular to the plane of the forces.
3. By virtue of 1 and 2 above, this is a planar problem that can be described in a 2D FBD.

Solution: See Figure 5-21 and Mathcad file P0521.


1. From examination of the FBDs, we see that, in 12" = dAB
both cases, the arms are in bending and the stub F
that holds the socket wrench is in pure torsion.
The maximum bending stress in the arm will occur
near the point where the arm transitions to the
stub. The stress state at this transition is very
complicated, but we can find the nominal bending T
stress there by treating the arm as a cantilever
beam, fixed at the transition point. For both cases F
the torque in the stub is the same. (a) Single-ended Wrench

12" = dAB
Case (a)
F
6"
2. The bending moment at the transition is

Ma = Fa d AB
T
3. The tensile stress at this point is found from
F
(b) Double-ended Wrench
Moment of inertia
4
π d 4 FIGURE 5-21
I  I  0.00749 in
64 Free Body Diagrams for Problem 5-21

Dist to extreme fibre c  0.5 d c  0.313 in

Ma c
Stress σx =
I
4. There are no other stress components present at this point, so x is the maximum principle stress here and
σ1 = σx σ2  0  psi σ3  0  psi

Ma c Fa d AB c


5. Since there is only one nonzero principal stress, the von Mises stress is σ' = σ1 = σx = =
I I
6. Using the distortion-energy theory, solve for the maximum applied force.
Sy I Sy I  Sy
Ns = = =1 Fa  Fa  89.882 lbf
σ' Fa d AB c d AB c
© 2011 Pearson Education, Inc., Upper Saddle River, NJ. All rights reserved. This publication is protected by Copyright and written permission should be
MACHINE DESIGN - An Integrated Approach, 4th Ed. 5-21-2

Fa d AB c
7. The von Mises stress in the handle at the transition point is σ'  σ'  45 ksi
I

8. Determine the torque in the stub. T  Fa d AB T  1079 in lbf

9. The shear stress at any point on the outside surface of the stub is found from
4
Polar moment of inertia J  2  I J  0.0150 in
Tc
Shear stress τxy  τxy  22.5 ksi
J

10. There are no other stress components present along the outside surface of the stub, so
σ1  τxy σ1  22.5 ksi σ2  0  psi σ3  σ1

2 2
and σ'  σ1  σ1 σ3  σ3 σ'  39.0 ksi

11. Thus, the maximum von Mises stress for case (a) is on the upper surface of the handle (arm) near the point
where it transitions to the stub, and the maximum force that can be applied to the handle without yielding is

Fa  89.9 lbf

Case (b)
Fb d AB
12. The bending moment at the transition is Mb =
2
Mb c
11. The tensile stress at this point is found from σx =
I

12. There are no other stress components present at this point, so x is the maximum principle stress here and
σ1 = σx σ2  0  psi σ3  0  psi

Mb c Fb d AB c


13. Since there is only one nonzero principal stress, the von Mises stress is σ' = σ1 = σx = =
I 2 I
14. Using the distortion-energy theory, solve for the maximum applied force.

Sy 2 I  Sy 2 I Sy
Ns = = =1 Fb  Fb  179.763 lbf
σ' Fb d AB c d AB c
Fb d AB c
15. The von Mises stress in the handle at the transition point is σ'  σ'  45 ksi
2 I

16. The torque in the stub is T  Fb d AB T  2157 in lbf

14. The shear stress at any point on the outside surface of the stub is found from
Tc
Shear stress τxy  τxy  45 ksi
J

15. There are no other stress components present along the outside surface of the stub, so
σ1  τxy σ1  45.0 ksi σ2  0  psi σ3  σ1
© 2011 Pearson Education, Inc., Upper Saddle River, NJ. All rights reserved. This publication is protected by Copyright and written permission should be
MACHINE DESIGN - An Integrated Approach, 4th Ed. 5-21-3

2 2
and σ'  σ1  σ1 σ3  σ3 σ'  77.9 ksi

16. Since the von Mises stress in the stub due to torsion is greater than the yield strength, the force in the
handle will be limited by the shear stress in the stib and by the bending stress in the handle.

Sy Sy J  Sy J  Sy
Ns = = = = =1
σ' 3  τxy 3 T  c 3  Fb d AB c

J  Sy
Fb  Fb  103.8 lbf
3  d AB c

17. Thus, the maximum von Mises stress for case (b) is on the stub, and the maximum force that can be
applied to the handles without yielding is

Fb  103.8 lbf

© 2011 Pearson Education, Inc., Upper Saddle River, NJ. All rights reserved. This publication is protected by Copyright and written permission should be
MACHINE DESIGN - An Integrated Approach, 4th Ed. 5-22-1
PROBLEM 5-22
Statement: A roller-blade skate is shown in Figure P5-10. The polyurethane wheels are 72 mm dia and
spaced on 104-mm centers. The skate-boot-foot combination weighs 2 kg. The effective "spring
rate" of the person-skate subsystem is 6000 N/m. The axles are 10-mm-dia steel pins in double
shear with S y = 400 MPa. Find the safety factor for the pins when a 100-kg person lands a 0.5-m
jump on one foot.
(a) Assume all 4 wheels land simultaneously.
(b) Assume that one wheel absorbs all the landing force.

Given: Axle pin diameter d  10 mm Yield strength S y  400  MPa

Solution: See Figure P5-10 and Mathcad file P0522.

1. From Problem 4-22, we have the stresses for cases (a) and (b): τa  5.71 MPa τb  22.9 MPa

2. Using the distortion-energy failure theory,

Sy
Case (a) all wheels landing Nsa  Nsa  40.4
3  τa

Sy
Case (b) one wheel landing Nsb  Nsb  10.1
3  τb

© 2011 Pearson Education, Inc., Upper Saddle River, NJ. All rights reserved. This publication is protected by Copyright and written permission should be
MACHINE DESIGN - An Integrated Approach, 4th Ed. 5-23a-1
PROBLEM 5-23a
Statement: A beam is supported and loaded as shown in Figure P5-11a. For the data given in row a from
Table P5-2, find the static safety factor:
(a) If the beam is a ductile material with S y = 300 MPa,
(b) If the beam is a cast-brittle material with S ut = 150 MPa, S uc = 570 MPa.

L
Given: Ductile yield strength S y  300  MPa
b
Brittle ultimate tensile strength
a
S ut  150  MPa F
w

Solution: See Figure 5-23 and Mathcad file P0523a. R1 R2


FIGURE 5-23
Free Body Diagram for Problem 5-23

1. The maximum bending stress occurs under the concentrated load F at x = b. It was determined in Problem 4-23a
as

σx  88.7 MPa

2. Since this is the only stress component present in the given coordinate frame, x is equal to 1 and the other two
principal stresses are zero.
σ1  σx σ2  0  MPa σ3  0  MPa

3. For case (a), use the distortion-energy failure theory. With only one nonzero principal stress, the von Mises
stress is the same as 1.

von Mises stress σ'  σ1 σ'  88.7 MPa

Sy
Safety factor, case (a) Nsa  Nsa  3.4
σ'

4. For case (b), use the Modified Mohr failure theory. The nonzero principal stress is positive and the load line
on the s1-s3 diagram is along the 1 axis.

S ut
Safety factor, case (b) Nsb  Nsb  1.7
σ1

© 2011 Pearson Education, Inc., Upper Saddle River, NJ. All rights reserved. This publication is protected by Copyright and written permission should be
MACHINE DESIGN - An Integrated Approach, 4th Ed. 5-24a-1
PROBLEM 5-24a
Statement: A beam is supported and loaded as shown in Figure P5-11b. For the data given in row a from Tabl
P5-2, find the static safety factor:
(a) If the beam is a ductile material with S y = 300 MPa,
(b) If the beam is a cast-brittle material with S ut = 150 MPa, S uc = 570 MPa.

L
a
Given: Ductile yield strength S y  300  MPa
Brittle ultimate strength F
S ut  150  MPa w

M1
Solution: See Figure 5-24 and Mathcad file P0524a.
R1
FIGURE 5-24
Free Body Diagram for Problem 5-24

1. The maximum bending stress occurs at the support where x = 0. It was determined in Problem 4-24a as

σx  410  MPa
2. Since this is the only stress component present in the given coordinate frame, x is equal to 1 and the other two
principal stresses are zero.
σ1  σx σ2  0  MPa σ3  0  MPa

3. For case (a), use the distortion-energy failure theory. With only one nonzero principal stress, the von Mises
stress is the same as 1.

von Mises stress σ'  σ1 σ'  410 MPa

Sy
Safety factor, case (a) Nsa  Nsa  0.73
σ'

4. For case (b), use the Modified Mohr failure theory. The nonzero principal stress is positive and the load line on
the 1-3 diagram is along the 1 axis.

S ut
Safety factor, case (b) Nsb  Nsb  0.37
σ1

© 2011 Pearson Education, Inc., Upper Saddle River, NJ. All rights reserved. This publication is protected by Copyright and written permission should be
MACHINE DESIGN - An Integrated Approach, 4th Ed. 5-25a-1
PROBLEM 5-25a
Statement: A beam is supported and loaded as shown in Figure P5-11c. For the data given in row a from
Table P5-2, find the static safety factor:
(a) If the beam is a ductile material with S y = 300 MPa,
(b) If the beam is a cast-brittle material with S ut = 150 MPa, S uc = 570 MPa.

L
Given: Ductile yield strength S y  300  MPa b
Brittle ultimate strength a F
S ut  150  MPa w

R1 R2
Solution: See Figure 5-25 and Mathcad file P0525a.
FIGURE 5-25
Free Body Diagram for Problem 5-25
1. The maximum bending stress occurs at the right-hand support where x = b. It was determined in Problem 4-25a
as
σx  151.6  MPa

2. Since this is the only stress component present in the given coordinate frame, x is equal to 1 and the other
two principal stresses are zero.

σ1  σx σ2  0  MPa σ3  0  MPa

3. For case (a), use the distortion-energy failure theory. With only one nonzero principal stress, the von Mises
stress is the same as 1.

von Mises stress σ'  σ1 σ'  151.6 MPa

Sy
Safety factor, case (a) Nsa  Nsa  2.0
σ'

4. For case (b), use the Modified Mohr failure theory. The nonzero principal stress is positive and the load line on
the 1-3 diagram is along the 1 axis.

S ut
Safety factor, case (b) Nsb  Nsb  0.99
σ1

© 2011 Pearson Education, Inc., Upper Saddle River, NJ. All rights reserved. This publication is protected by Copyright and written permission should be
MACHINE DESIGN - An Integrated Approach, 4th Ed. 5-26a-1
PROBLEM 5-26a
Statement: A beam is supported and loaded as shown in Figure P5-11d. For the data given in row a from Tabl
P5-2, find the static safety factor:
(a) If the beam is a ductile material with S y = 300 MPa,
(b) If the beam is a cast-brittle material with S ut = 150 MPa, S uc = 570 MPa.

L
Given: Ductile yield strength S y  300  MPa b
Brittle ultimate strength a F
S ut  150  MPa w

Solution: See Figure 5-26 and Mathcad file P0526a. R1 R2 R3

1. The maximum bending stress occurs under the concen- FIGURE 5-26
trated load F, where x = a. It was determined in Free Body Diagram for Problem 5-26
Problem 4-26a as

σx  31.5 MPa
2. Since this is the only stress component present in the given coordinate frame, x is equal to 1 and the other two
principal stresses are zero.
σ1  σx σ2  0  MPa σ3  0  MPa

3. For case (a), use the distortion-energy failure theory. With only one nonzero principal stress, the von Mises
stress is the same as 1.

von Mises stress σ'  σ1 σ'  31.5 MPa

Sy
Safety factor, case (a) Nsa  Nsa  9.5
σ'

4. For case (b), use the Modified Mohr failure theory. The nonzero principal stress is positive and the load line on
the 1-3 diagram is along the 1 axis.

S ut
Safety factor, case (b) Nsb  Nsb  4.8
σ1

© 2011 Pearson Education, Inc., Upper Saddle River, NJ. All rights reserved. This publication is protected by Copyright and written permission should be
MACHINE DESIGN - An Integrated Approach, 4th Ed. 5-27-1
PROBLEM 5-27
Statement: A storage rack is to be designed to hold the paper roll of Problem 5-8 as shown in Figure P5-12.
Determine suitable values for dimensions a and b in the figure. Make the static factor of safety at
least 1.5. The mandrel is solid and inserts halfway into the paper roll.
(a) The beam is a ductile material with Sy = 300 MPa
(b) The beam is a cast-brittle material with Sut = 150 MPa, S uc = 570 MPa.

3
Given: Paper roll dimensions OD  1.50 m Roll density ρ  984  kg m
ID  0.22 m
Lroll  3.23 m Factor of safety Ns  1.5
Ductile yield strength S y  300  MPa Brittle ultimate strength S ut  150  MPa

Assumptions: The paper roll's weight creates a


concentrated load acting at the tip of the y W
mandrel. The mandrel's root in the
stanchion experiences a distributed load w
over the length of engagement (see the a
solution to Problem 3-27 for further x
discussion of this point). The required
diameter a of the mandrel root section
(over the length b) will be sized to use b Lm
the allowable tensile strength in bending. R
The length b will be sized to use the
allowable transverse shear strength. FIGURE 5-27
Free Body Diagram used in Problem 5-27

Solution: See Figure 5-27 and Mathcad file P0527.


1. Determine the weight of the roll and the length of the mandrel.

Weight W 
π
4
 2 2 
 OD  ID  Lroll  ρ  g W  53.9 kN

Length Lm  0.5 Lroll Lm  1.615 m

2. The maximum internal shear and moment occur at a section where the mandrel root leaves the stanchion. and
are

2  W  Lm
Vmax = Mmax  W  Lm Mmax  87.04 kN  m
b

3. Part (a) - The bending stress will be a maximum at the top or bottom of the mandrel at a section where the
mandrel root leaves the stanchion.

Mmax a 4 32 Mmax


π a
σmax = where I= so, σmax =
2 I 64 3
π a
4. At this point the only nonzero stress component is max therefore

σ1 = σmax σ2  0  MPa σ3  0  MPa


5. All three of the ductile failure theories have the same fail/safe boundary for this condition (slope of load line is
zero)

Sy
Ns = or Ns σ1 = S y
σ1
© 2011 Pearson Education, Inc., Upper Saddle River, NJ. All rights reserved. This publication is protected by Copyright and written permission should be
MACHINE DESIGN - An Integrated Approach, 4th Ed. 5-27-2

1
3
 32 Ns W  Lm 
6. Solving for a, a    a  164.272 mm
 π S y 
Round this to a  166  mm

7. Using this value of a and equation (4.15c), solve for the shear stress on the neutral axis at the same section.

4  Vmax 8  W  Lm
τmax = =
3 A  π a 2 
3   b
 4 
8. At this point, this is the only nonzero stress component therefore, the principal stresses are
σ1 = τmax σ2  0  MPa σ3 = τmax
9 Using the distortion energy theory, the von Mises stress is

Sy Sy
σ' = 3  τmax and Ns = or Ns τmax =
σ' 3
8  Ns W  Lm
Solving for b b  b  92.9 mm
 π a 2  S y
3  
 4  3
Rounding to higher even values, let a  166 mm b  94 mm for case (a).

10. Part (b) - The bending stress will be a maximum at the top or bottom of the mandrel at a section where the
mandrel root leaves the stanchion.

Mmax a 4 32 Mmax


π a
σmax = where I= so, σmax =
2 I 64 3
π a
11. At this point the only nonzero stress component is max therefore

σ1 = σmax σ2  0  MPa σ3  0  MPa


12. All three of the brittle failure theories have the same fail/safe boundary for this condition (slope of load line is
zero)
3
S ut S ut 2  I  S ut π a  S ut
Ns = = = =
σ1 σmax Mmax a 32 Mmax

1
3
 32 Ns Mmax 
13. Solving for a, a    a  206.97 mm
 π Sut 
Round this to a  208  mm

14. Using this value of a and equation (4.15c), solve for the shear stress on the neutral axis at the same section.

© 2011 Pearson Education, Inc., Upper Saddle River, NJ. All rights reserved. This publication is protected by Copyright and written permission should be
MACHINE DESIGN - An Integrated Approach, 4th Ed. 5-27-3

4  Vmax 8  W  Lm
τmax = =
3 A  π a 2 
3   b
 4 
15. At this point, this is the only nonzero stress component therefore, the principal stresses are
σ1 = τmax σ2  0  MPa σ3 = τmax
16. Using the Modified Mohr theory,

 π a 2 
3    b S ut
Ns =
S ut
=
S ut
=
 4 
σ1 τmax 8  W  Lm

8  Ns W  Lm
Solving for b b  b  68.3 mm
 π a 2 
3
   Sut
 4 

Rounding to higher even values, let a  208 mm b  70 mm for case (b).

© 2011 Pearson Education, Inc., Upper Saddle River, NJ. All rights reserved. This publication is protected by Copyright and written permission should be
MACHINE DESIGN - An Integrated Approach, 4th Ed. 5-28-1
PROBLEM 5-28
Statement: Figure P5-13 shows a forklift truck negotiating a 15 deg ramp to to drive onto a 4-ft-high loading
platform. The truck weighs 5 000 lb and has a 42-in wheelbase. Design two (one for each side)
1-ft-wide ramps of steel to have a safety factor of 3 in the worst case of loading as the truck travels
up them. Minimize the weight of the ramps by using a sensible cross-sectional geometry. Choose
an appropriate steel or aluminum alloy.

Given: Ramp angle θ  15 deg Ramp width w  12 in


Platform height h  4  ft Truck weight W  5000 lbf
Truck wheelbase Lt  42 in

Assumptions: 1. The worst case is when the truck CG is located at the center of the beam's span.
2. Use a coordinate frame that has the x-axis along the long axis of the beam.
3. Ignore traction forces and the weight components along the x-axis of the beam.
4. There are two ramps, one for each side of the forklift.
Solution: See Figure 5-28 and Mathcad file P0528.

L
b
a CG a
y
CG b

R1

Fa Fb x
Wa
Wb
R2
FIGURE 5-28A
Dimensions and Free Body Diagram for Problem 5-28

1. From Problem 3-28 the maximum bending moment in the ramp occurs at the rear wheel of the truck and is

Mmax  8324 ft  lbf Mmax  99888 in lbf

2. The bending stress is the only stress component present and is, therefore, also the only nonzero principal stress
and is also the von Mises stress. The governing design equations then are

Mmax Sy
σ' = and Ns =
Z σ'

3. The approach will be to 1) choose a suitable factor of safety, 2) choose a suitable material and determine its yiel
strength, 3) from the equations above determine the required value of the section modulus, 4) choose an
appropriate cross-section for the ramp, and 5) determine the dimensions of the cross-section.

4. The following design choices have been made for this problem:
© 2011 Pearson Education, Inc., Upper Saddle River, NJ. All rights reserved. This publication is protected by Copyright and written permission should be
MACHINE DESIGN - An Integrated Approach, 4th Ed. 5-28-2

Design factor of safety Nsd  3


Material 7075 Aluminum, heat treated
Yield strength S y  73 ksi

5. Solve the design equations for the minimum section modulus, Z.

Nsd  Mmax 3
Z  Z  4.105 in
Sy

This is the minimum allowable value of the section modulus.

6. Assume a channel section such as that shown in Figure 5-28B. To keep it simple, let the thickness of the
flanges and web be the same. Choose 1/2-in thick plate, which is readily available. Then, t  0.50 in

7. The cross-sectional area of the ramp is A ( h )  w t  2  t ( h  t)

 w t 2 2
8. The distance to the CG is cg( h ) 
1
 2
 t h  t 
A (h)  2 

9. The moments of inertia of the web and a flange are


Flange
3 2
w t
 w t  cg( h )  
t Web
Iweb( h )  
12  2

3 2 t
t ( h  t) h  t
Ifl ( h )   h  t  cg( h )  
12  2 

I ( h )  Iweb( h )  2  Ifl ( h )
h
11. The maximum stress will occur in the flange at the top
and is compressive. The distance from the centroid up to
the top of the flange is

c( h )  h  cg( h ) w

12. Using the known section modulus, solve FIGURE 5-28B


for the unknown flange height, h. Guess h  1  in Channel Section for Problem 5-28

Given
I (h)
Z= h  Find ( h ) h  3.843 in Round this up to h  4.00 in
c( h )

13. Summarizing, the ramp design dimensions are:

Width w  12.00 in Flange height h  4.00 in Shape channel


Thickness t  0.5 in Material 7075 aluminum

© 2011 Pearson Education, Inc., Upper Saddle River, NJ. All rights reserved. This publication is protected by Copyright and written permission should be
MACHINE DESIGN - An Integrated Approach, 4th Ed. 5-29-1
PROBLEM 5-29
Statement: A differential element is subected to the stresses given below and a ductile material has the
strengths given below. Calculate the safety factor and draw 1-3 diagrams of each theory
showing the stress state using:
(a) Maximum shear-stress theory, and
(b) Distortion-energy theory.

Given: Principal stresses σ1  10 ksi σ2  0  ksi σ3  20 ksi


Material properties S ut  50 ksi S y  40 ksi S uc  50 ksi

Solution: See Figure 5-29 and Mathcad file P0529.


1. Calculate the slope of load line. (The load line is the line from the origin through the stress point.)
σ3
m  m  2
σ1

2. The safety factor equation for the distortion-enrgy theory is the same regardless of which quadrant the load line
falls in. However, the equation for the maximum shear-stress factor of safety is different for each of the three
quadrants that the load line (1st, 3rd, or 4th) can fall in. In this case, the load line falls in the 4th quadrant. The
factors of safety are:

Sy
(a) Maximum shear-stress theory Na  Na  1.3
σ1  σ3

(b) Distortion energy theory 3


40
(a) Maximum shear
stress boundary
2 2
σ'  σ1  σ1 σ3  σ3 30
(b) Distortion energy
boundary
σ'  26.5 ksi 20
MINIMUM NONZERO PRINCIPAL STRESS, KSI

10
Sy
Nb  Nb  1.5
σ' 0 1
sy

3. Plot the 1-3 diagram showing the safe-fail -10

boundaries, the stress state point (10 ksi, -20 (10,-20)


ksi) and the load line. Note that if 1 > 3 , -20

then only that area on the graph that is to the -30 Stress states at
right of and below the diagonal line can which failure
will occur
contain valid stress points. The factor of -40 -sy
Load Line
safety is the distance along the load line from
the origin to the intersection of the load line -50
with the failure boundary, divided by the
distance from the origin to the stress point. -60
-40 -30 -20 -10 0 10 20 30 40 50
Since the distance from the origin to the
MAXIMUM PRINCIPAL STRESS, KSI
distortion-energy boundary is greater than the
distance to the maximum shear-stress
baoundary, its factor of safety is greater. FIGURE 5-29
1 -  3 Diagram for Problem 5-29

© 2011 Pearson Education, Inc., Upper Saddle River, NJ. All rights reserved. This publication is protected by Copyright and written permission should be
MACHINE DESIGN - An Integrated Approach, 4th Ed. 5-30-1
PROBLEM 5-30
Statement: A differential element is subected to the stresses and strengths given below. Calculate the
safety factor and draw 1-3 diagrams of each theory showing the stress state using:
(a) Coulomb-Mohr theory, and
(b) Modified Mohr theory.

Given: Principal stresses σ1  10 ksi σ2  0  ksi σ3  20 ksi


Material properties S ut  50 ksi S y  40 ksi S uc  90 ksi

Solution: See Figure 5-30 and Mathcad file P0530.


1. Calculate the slope of load line. (The load line is the line from the origin through the stress point.)
σ3
m  m  2
σ1

2. The safety factor equation for both theories is different for each quadrant the load line falls in. The equation for
the modified Mohr factor of safety is different for each of the two regions in the 4th quadrant that the load line can
fall in. In this case, the load line falls in the 4th quadrant, below the -1 slope line.. The factors of safety are:

S uc S ut
(a)Coulomb-Mohr theory Na  Na  2.4
S uc σ1  S ut σ3

3
(b) Modified Mohr theory 50

40
S uc
Nb  30
 S uc  S ut 
 S   σ1  σ3
20
MINIMUM NONZERO PRINCIPAL STRESS, KSI

(a) Coulomb-Mohr

 
boundary
ut 10

0 1
Nb  3.2 -10
(10,-20)
-20

3. Plot the 1-3 diagram showing the safe-fail -30

-40
boundaries, the stress state point (10 ksi, -20
ksi) and the load line. Note that if 1 > 3 , -50 Stress states at
which failure
-S
ut
will occur
then only that area on the graph that is to the -60

right of and below the diagonal line can -70 (b) Modified Mohr
boundary
contain valid stress points. The factor of -80
Load Line
safety is the distance along the load line from
-90 -S
the origin to the intersection of the load line uc
with the failure boundary, divided by the -100
-100 -90 -80 -70 -60 -50 -40 -30 -20 -10 0 10 20 30 40 50
distance from the origin to the stress point.
MAXIMUM PRINCIPAL STRESS, KSI
Since the distance from the origin to the
modified Mohr boundary is greater than the
distance to the Coulomb-Mohr boundary, its FIGURE 5-30
factor of safety is greater. 1 -  3 Diagram for Problem 5-30

© 2011 Pearson Education, Inc., Upper Saddle River, NJ. All rights reserved. This publication is protected by Copyright and written permission should be
MACHINE DESIGN - An Integrated Approach, 4th Ed. 5-31-1
PROBLEM 5-31
Statement: Design a jack-stand in a tripod configuration that will support 2 tons of load with a safety factor of
3. Use SAE 1020 steel and minimize its weight.

Solution: This open-ended design problem has many valid solutions that are left to the student.

© 2011 Pearson Education, Inc., Upper Saddle River, NJ. All rights reserved. This publication is protected by Copyright and written permission should be
MACHINE DESIGN - An Integrated Approach, 4th Ed. 5-32-1
PROBLEM 5-32
Statement: A part has the combined stress state and strengths given below. Choose an appropriate failure
theory based on the given data, find the effective stress and factor of safety against static failure.
Given: Stresses: σx  10 ksi σy  5  ksi τxy  4.5 ksi
Strengths: S y  18 ksi S ut  20 ksi S uc  80 ksi

Solution: See Mathcad file P0532.


1. Because S uc is greater than S ut, this is an uneven material, which is characteristic of a brittle material. Therefore,
use the modified Mohr theory.
2. Find the maximum shear stress and principal stresses that result from this combination of applied stresses using
equations 4.6.

2
 σx  σy  2
Maximum shear stress τmax     τxy τmax  5.148 ksi
 2 
σx  σy
Principal stresses σ1   τmax σ1  12.648 ksi
2
σx  σy
σ2   τmax σ2  2.352 ksi
2

σ3  0  psi

3. Find the Dowling factors C1, C2, C3 using equations 5.12b:

 S uc  2  S ut 
  σ1  σ2
1
C1    σ1  σ2  C1  8.898 ksi
2  S uc 
 S uc  2  S ut 
  σ2  σ3
1
C2    σ2  σ3  C2  1.764 ksi
2  S uc 
 S uc  2  S ut 
  σ3  σ1
1
C3    σ3  σ1  C3  9.486 ksi
2  S uc 
4. Then find the largest of the six stresses C1, C2, C3 , 1, 2, 3:

  1  
C
  C2  
 
C3
σeff  max    σeff  12.6 ksi
  σ1  
 
  σ2  
  σ3  

which is the modified-Mohr effective stress.


S ut
5. The safety factor can now be found using equation 5.12d. N  N  1.6
σeff

© 2011 Pearson Education, Inc., Upper Saddle River, NJ. All rights reserved. This publication is protected by Copyright and written permission should be
MACHINE DESIGN - An Integrated Approach, 4th Ed. 5-33a-1
PROBLEM 5-33a
Statement: For the bracket shown in Figure P5-14 and the data in row a of Table P5-3, determine the von
Mises stresses at points A and B.
Solution: See Mathcad file P0533a.

1. From Problem 4-33a the principal stresses at point A are

σ1  21.46  MPa σ2  0  MPa σ3  13.08  MPa

2. Use equation (5.7c) to find the von Mises stress at point A.

2 2
σ'A  σ1  σ1 σ3  σ3 σ'A  30.2 MPa

3. From Problem 4-33a the principal stresses at point B are

σ1  16.13  MPa σ2  0  MPa σ3  16.13  MPa

4. Use equation (5.7c) to find the von Mises stress at point B.

2 2
σ'B  σ1  σ1 σ3  σ3 σ'B  27.9 MPa

F
y

B x
T T

L
R

FIGURE 5-33
Free Body Diagram of Tube for Problem 5-33

© 2011 Pearson Education, Inc., Upper Saddle River, NJ. All rights reserved. This publication is protected by Copyright and written permission should be
MACHINE DESIGN - An Integrated Approach, 4th Ed. 5-34a-1
PROBLEM 5-34a
Statement: Calculate the safety factor for the bracket in Problem 5-33 using the distortion energy, the
maximum shear stress, and the maximum normal-stress theories. Comment on their
appropriateness. Assume a ductile material strength as given below.

Given: Yield strength S y  400  MPa

Solution: See Mathcad file P0534a.


1. From Problem 4-33a the principal stresses at point A are

σ1A  21.46  MPa σ2A  0  MPa σ3A  13.08  MPa

2. Using the two nonzero stresses, the slope of the load line on a 1-3 graph is

σ3A
m  m  0.61
σ1A

This intersects the failure boundaries in the fourth quadrant.

3. Calculate the von Mises effective stress at point A using equation (5.7c).

2 2
σ'A  σ1A  σ1A σ3A  σ3A σ'A  30.205 MPa

4. Determine the factor of safety at point A

Sy
Distortion energy NADE  NADE  13.2
σ'A

Sy
Maximum shear stress NAMS  NAMS  11.6
σ1A  σ3A

Sy
Maximum normal stress NANS  NANS  18.6
σ1A

5. From Problem 4-33a, the principal stresses at Point B are

σ1B  16.13  MPa σ2B  0  MPa σ3B  16.13  MPa

6. Using the two nonzero stresses, the slope of the load line on a 1-3 graph is

σ3B
m  m  1
σ1B
This intersects the failure boundaries in the fourth quadrant.
7. Calculate the von Mises effective stress at point A using equation (5.7c).

2 2
σ'B  σ1B  σ1B σ3B  σ3B σ'B  27.938 MPa

8. Determine the factor of safety at point B


© 2011 Pearson Education, Inc., Upper Saddle River, NJ. All rights reserved. This publication is protected by Copyright and written permission should be
MACHINE DESIGN - An Integrated Approach, 4th Ed. 5-34a-2

Sy
Distortion energy NBDE  NBDE  14.3
σ'B

Sy
Maximum shear stress NBMS  NBMS  12.4
σ1B  σ3B

Sy
Maximum normal stress NBNS  NBNS  24.8
σ1B

9. Whichever theory is used, the critical point (lowest factor of safety) is point A. The distortion energy theory
should be used because experimental data follows its failure boundary more nearly than the maximum shear
stress in all quadrants. Using the maximum normal stress theory would give an overestimate of the actual safety
factor.

© 2011 Pearson Education, Inc., Upper Saddle River, NJ. All rights reserved. This publication is protected by Copyright and written permission should be
MACHINE DESIGN - An Integrated Approach, 4th Ed. 5-35a-1
PROBLEM 5-35a
Statement: Calculate the safety factor for the bracket in Problem 5-33 using the Coulomb-Mohr and the
modified Mohr effective stress theories. Comment on their appropriateness. Assume a brittle
material strength as given below.

Given: Tensile strength S ut  350  MPa Compressive strength S uc  1000 MPa


Solution: See Mathcad file P0535a.
1. From Problem 4-33a the principal stresses at point A are

σ1A  21.46  MPa σ2A  0  MPa σ3A  13.08  MPa

2. Using the two nonzero stresses, the slope of the load line on a 1-3 graph is

σ3A
m  m  0.61
σ1A

This intersects the failure boundaries in the fourth quadrant. For the Coulomb-Mohr diagram (see Figure 5-9
in the text) there is a single, straight line in this quadrant. For the modified-Mohr theory, the load line will
intersect the boundary at a point similar to B' in Figure 5-11 in the text.

3. Determine the factor of safety at point A

S ut S uc
Coulomb-Mohr NACM  NACM  13.4
S uc σ1A  S ut σ3A

S ut
Modified-Mohr NAMM  NAMM  16.3
σ1A

4. From Problem 4-33a, the principal stresses at Point B are

σ1B  16.13  MPa σ2B  0  MPa σ3B  16.13  MPa

5. Using the two nonzero stresses, the slope of the load line on a 1-3 graph is
σ3B
m  m  1
σ1B
This intersects the failure boundaries in the fourth quadrant. For the Coulomb-Mohr diagram (see Figure 5-9
in the text) there is a single, straight line in this quadrant. For the modified-Mohr theory, the load line will
intersect the boundary at the point (S ut -S ut) Figure 5-11 in the text.

6. Determine the factor of safety at point B

S ut S uc
Coulomb-Mohr NBCM  NBCM  16.1
S uc σ1B  S ut σ3B

S ut
Modified-Mohr NBMM  NBMM  21.7
σ1B

7. Whichever theory is used, the critical point (lowest factor of safety) is point A. The modified-Mohr theory
should be used because experimental data follows its failure boundary more nearly than the Coulomb-Mohr
when the slope of the load line is in the fourth quadrant. Using the Coulomb-Mohr would give an underestimat
of the actual safety factor.

© 2011 Pearson Education, Inc., Upper Saddle River, NJ. All rights reserved. This publication is protected by Copyright and written permission should be
MACHINE DESIGN - An Integrated Approach, 4th Ed. 5-35a-2

8. Calculating factor of safety using Modified Mohr and equations (5.12c, d, and e)

Point A

 S uc  2  S ut 
  σ1A  σ2A
1
C1    σ1A  σ2A  C1  13.9 MPa
2  S uc 

 S uc  2  S ut 
  σ2A  σ3A
1
C2    σ2A  σ3A  C2  4.6 MPa
2  S uc 

 S uc  2  S ut 
  σ3A  σ1A
1
C3    σ3A  σ1A  C3  18.5 MPa
2  S uc 

S ut
1 is maximum so N  N  16.3
σ1A

Point B

 S uc  2  S ut 
  σ1B  σ2B
1
C1    σ1B  σ2B  C1  10.5 MPa
2  S uc 

 S uc  2  S ut 
  σ2B  σ3B
1
C2    σ2B  σ3B  C2  5.6 MPa
2  S uc 

 S uc  2  S ut 
  σ3B  σ1B
1
C3    σ3B  σ1B  C3  16.1 MPa
2  S uc 

S ut
1 is maximum so N  N  21.7
σ1B

© 2011 Pearson Education, Inc., Upper Saddle River, NJ. All rights reserved. This publication is protected by Copyright and written permission should be
MACHINE DESIGN - An Integrated Approach, 4th Ed. 5-36a-1
PROBLEM 5-36a
Statement: For the bracket shown in Figure P5-14 and the data in row a of Table P5-3, redo Problem 5-33
considering the stress concentration at points A and B. Assume a stress concentration factor of
2.5 in both bending and torsion.
Given: Factors of safety:
Bending Kf  2.5 Torsion Kfs  2.5

Solution: See Mathcad file P0536a.


1. From Problem 4-36a the principal stresses at point A are
σ1  53.6 MPa σ2  0  MPa σ3  32.7 MPa
2. Use equation (5.7c) to find the von Mises stress at point A.
2 2
σ'A  σ1  σ1 σ3  σ3 σ'A  75.5 MPa

3. From Problem 4-36a the principal stresses at point B are


σ1  41.3 MPa σ2  0  MPa σ3  41.3 MPa

4. Use equation (5.7c) to find the von Mises stress at point B.


2 2
σ'B  σ1  σ1 σ3  σ3 σ'B  71.5 MPa

F
y

B x
T T

L
R
FIGURE 5-36
Free Body Diagram of Tube for Problem 5-36

© 2011 Pearson Education, Inc., Upper Saddle River, NJ. All rights reserved. This publication is protected by Copyright and written permission should be
MACHINE DESIGN - An Integrated Approach, 4th Ed. 5-37-1
PROBLEM 5-37
Statement: A semicircular, curved beam as shown in Figure 5-37 has the dimensions given below. For a load
pair F = 14 kN applied along the diameter, find the safety factor at the inner and outer fibers:
(a) If the beam is a ductile material with Sy = 700 MPa,
(b) If the beam is a cast-brittle material with Sut = 420 MPa, Suc = 1200 MPa.
w
Given: (a) Yield strength
S y  700  MPa
(b) Tensile strength S ut  420  MPa
Compressive strength S uc  1200 MPa
Solution: See Figure 5-37 and Mathcad file P0537. F

1. From Problem 4-37, the stresses at the inside radius and od id


outside radius are: F

Inside σi  409.9  MPa


Outside σo  273.2  MPa
(a) Entire Beam

These are the only stress components present on their


respective surfaces so they are also principal stresses. Thus,

σ1i  409.9  MPa σ2i  0  MPa σ3i  0  MPa F


σ1o  0  MPa σ2o  0  MPa σ3o  273.2  MPa M
F
Part (a) rc
2. Use the distortion energy theory for the ductile material.
(b) Critical Section
3. Since 1 is the only nonzero principal stress, it is also the
von Mises effective stress, FIGURE 5-37
Free Body Diagrams for Problem 5-37
σ'i  σ1i σ'i  409.9 MPa

σ'o  σ3o σ'o  273.2 MPa

4. The factor of safety against a static failure for this ductile material is

Sy
Inside surface Nai  Nai  1.7
σ'i
Sy
Outside surface Nao  Nao  2.6
σ'o
Part (b)
5. Use the modified-Mohr theory for the brittle material.
6. The load line on the 1-3 graph for the inside surface is along the positive 1 axis. In this case, the factor of
safety equation simplifies to

S ut
Inside surface Nbi  Nbi  1.0
σ1i

7. The load line on the 1-3 graph for the outside surface is along the negative 3 axis. In this case, the factor of
safety equation simplifies to

S uc
Outside surface Nbo  Nbo  4.4
σ3o
© 2011 Pearson Education, Inc., Upper Saddle River, NJ. All rights reserved. This publication is protected by Copyright and written permission should be
MACHINE DESIGN - An Integrated Approach, 4th Ed. 5-38-1
PROBLEM 5-38
Statement: Assume that the curved beam of Problem 5-37 has a crack on its inside surface of half-width a = 2
mm and a fracture toughness of 50 MPa-m0.5. What is its safety factor against sudden fracture?
Given: Outside diameter od  150  mm Inside diameter id  100  mm
Width of section t  25 mm Load F  14 kN
Half crack length a  2  mm Fracture toughness Kc  50 MPa m

Solution: See Figure 5-38 and Mathcad file P0538.

1. From Problem 4-37, the nominal stress at the inside radius is:
Nominal inside stress σi  409.9  MPa

2. Calculate the half-width of the beam. b  0.5 t b  12.5 mm


3. Calculate the geometry and stress intensity factors.
π a 
β  sec   β  1.016
 2 b 
K  β  σi π a K  33.01 MPa m

Kc
4. Determine the factor of safety against sudden fracture failure NFM  NFM  1.5
K

F
od id
F

(a) Entire Beam

M
F
rc

(b) Critical Section


FIGURE 5-38
Free Body Diagrams for Problem 5-38

© 2011 Pearson Education, Inc., Upper Saddle River, NJ. All rights reserved. This publication is protected by Copyright and written permission should be
MACHINE DESIGN - An Integrated Approach, 4th Ed. 5-39-1
PROBLEM 5-39
Statement: Consider the failed 260-in dia by 0.73-in wall rocket case of Figure 5-14. The steel had S y = 240 k
and a fracture toughness Kc = 79.6 ksi-in 0.5. It was designed for an internal pressure of 960 psi but
failed at 542 psi. Failure was attributed to a small crack that precipitated a sudden, brittle,
fracture-mechanics failure. Find the nominal stress in the wall and the yielding safety factor at the
failure conditions and estimate the size of the crack that caused it to explode. Assume b = 1.0.

Given: Case diameter d  260  in Fracture toughness Kc  79.6 ksi in


Wall thickness t  0.73 in Design pressure p d  960  psi
Yield strength S y  240  ksi Failure pressure p f  542  psi

Solution: See Mathcad file P0539.


1. Find the nominal stress in the wall. The ratio of the wall thickness to the radius of the case is such that we can
use thin-wall theory. Thus
Case radius r  0.5 d r  130 in
pd  r
Tangential stress σt  σt  171.0 ksi
t
pd  r
Axial stress σa  σa  85.5 ksi
2 t

Radial stress σr  0  psi

2. Find the yielding safety factor at the failure conditions. Since, for these directions, there are no shear stresses
present, these are the principal stresses. The von Mises stress is

2 2
Von Mises stress σ'  σt  σt σa  σa σ'  148.1 ksi

Factor of safety Sy
against yielding Ns  Ns  1.6
σ'

3. Estimate the size of the crack that caused it to explode.

pf  r
Tangential stress σt  σt  96.5 ksi
t
pf  r
Axial stress σa  σa  48.3 ksi
2 t
(a) Assume that the crack was longitudinal (growing in the axial direction)

Nominal stress σnom  σt σnom  96.5 ksi

Stress intensity
factor K = σnom a  π

Setting the stress intensity factor equal to the fracture toughness of the material and solving for the crack length

 Kc2 
Half-length a    a  0.216 in
 σ 2 π 
 nom 
Crack length 2  a  0.433 in
© 2011 Pearson Education, Inc., Upper Saddle River, NJ. All rights reserved. This publication is protected by Copyright and written permission should be
MACHINE DESIGN - An Integrated Approach, 4th Ed. 5-39-2

(b) Assume that the crack was tangential (growing in the tangential direction)
Nominal stress σnom  σa σnom  48.3 ksi

Stress intensity
factor K = σnom a  π

Setting the stress intensity factor equal to the fracture toughness of the material and solving for the crack length

 Kc2 
Half-length a    a  0.866 in
 σ 2 π 
 nom 

Crack length 2  a  1.732 in

© 2011 Pearson Education, Inc., Upper Saddle River, NJ. All rights reserved. This publication is protected by Copyright and written permission should be
MACHINE DESIGN - An Integrated Approach, 4th Ed. 5-40-1
PROBLEM 5-40
Statement: Redesign the roll support of Problem 5-8 to be like that shown in Figure P5-16. The stub mandrels
insert to 10% of the roll length at each end. Design dimension a for a factor of safety of 2. See
Problem 5-8 for additional data.
(a) The beam is a ductile material with S y = 300 MPa
(b) The beam is a cast-brittle material with S ut = 150 MPa, S uc = 570 MPa.

Given: Paper roll dimensions: OD  1.50 m Material properties:


ID  0.22 m Yield strength S y  300  MPa
Lroll  3.23 m Tensile strength S ut  150  MPa
Comp strength S uc  570  MPa
3
Roll density ρ  984  kg m Factor of safety Ns  2

Assumptions: 1. The paper roll's weight creates a concentrated load acting at the tip of the mandrel.
2. The base of the mandrel (the portion that inserts into the stanchion) is solid and fits tightly into
the stanchion. Therefore, the mandrel can be treated as a cantilever beam.
3. The length of ther mandrel base is b  100  mm.

Solution: See Figure 5-40 and Mathcad file P0540.


1. For the assumptions made, it is not necessary to
F
determine the stress distribution on the mandrel base
inside the stanchion. From Figure 5-40, we see that we y
can determine the diameter a by applying the beam
stress equation at the section where the mandrel
transitions from the base to the full diameter. x
a
2. Determine the weight of the roll, the load on each
support, and the length of the mandrel.
M1 b Lm
W 
π
4
 2 2 
 OD  ID  Lroll  ρ  g W  53.9 kN
R

F  0.5 W F  26.95 kN FIGURE 5-40


Free Body Diagram used in Problem 5-40
Lm  0.1 Lroll Lm  323 mm

3. From Figure 5-40, the maximum internal bending moment occurs at x = 0 and is
Mmax  F  Lm Mmax  8.704 kN  m

4. The bending stress will be a maximum at the top or bottom of the mandrel at a section through x = 0.

Mmax a 4
π a
σmax = where I=
2 I 64

There are no other stress components at this point so σmax = σ1 and

σ2  0  MPa σ3  0  MPa

5. For the ductile material of part (a), the maximum principal stress is also the von Mises stress so

32 Mmax Sy
σmax = σ' = =
3 Ns
π a

© 2011 Pearson Education, Inc., Upper Saddle River, NJ. All rights reserved. This publication is protected by Copyright and written permission should be
MACHINE DESIGN - An Integrated Approach, 4th Ed. 5-40-2

1
3
 32 Ns F  Lm 
Solving for a, a    a  83.922 mm
 π S y 
Round this to a  84 mm for the ductile material of part (a)

5. For the brittle material of part (b), the load line on the 1-3 diagram is along the positive 1 axis where both
brittle material failure theories have the same boundary, which is 1 = S ut. Thus, for the brittle case of part (b),

32 Mmax S ut
σmax = σ1 = =
3 Ns
π a

1
3
 32 Ns F  Lm 
Solving for a, a    a  105.735 mm
 π S ut 
Round this to a  106  mm for the brittle material of part (b)

© 2011 Pearson Education, Inc., Upper Saddle River, NJ. All rights reserved. This publication is protected by Copyright and written permission should be
MACHINE DESIGN - An Integrated Approach, 4th Ed. 5-41-1
PROBLEM 5-41
Statement: A 10-mm ID steel tube carries liquid at 7 MPa. The steel has S y = 400 MPa Determine the safety
factor for the wall if its thickness is: a) 1 mm, b) 5 mm.

Given: Yield strength S y  400  MPa


Assumption: The tubing is long therefore the axial stress is zero.
Solution: See Mathcad file P0541.

(a) Wall thickness is t  1  mm


1. From Problem 4-41, this is a thick wall cylinder and the principal stresses are:

σ1a  38.82  MPa σ2a  0  MPa σ3a  7.00 MPa

2. Calculate the von Mises effective stress using equation (5.7c).

2 2
σ'a  σ1a  σ1a σ3a  σ3a σ'a  42.752 MPa

3. Using the distortion energy theory, the factor of safety is

Sy
Na  Na  9.4
σ'a

(b) Wall thickness is t  5  mm

4. From Problem 4-41, this is a thick wall cylinder and the principal stresses are:

σ1b  11.67  MPa σ2b  0  MPa σ3b  7.00 MPa

5. Calculate the von Mises effective stress using equation (5.7c).

2 2
σ'b  σ1b  σ1b σ3a  σ3b σ'b  16.336 MPa

6. Using the distortion energy theory, the factor of safety is

Sy
Nb  Nb  24.5
σ'b

© 2011 Pearson Education, Inc., Upper Saddle River, NJ. All rights reserved. This publication is protected by Copyright and written permission should be
MACHINE DESIGN - An Integrated Approach, 4th Ed. 5-42-1
PROBLEM 5-42
Statement: A cylindrical tank with hemispherical ends is required to hold 150 psi of pressurized air at room
temperature. The steel has S y = 400 MPa. Determine the safety factor if the tank diameter is 0.5 m
with 1 mm wall thickness, and its length is 1 m.

Given: Yield strength S y  400  MPa

Solution: See Mathcad file P0542.


1. From Problem 4-42, the maximum principal stresses in the wall are

σ1  259  MPa σ2  129  MPa σ3  0  MPa

2 2
2. The von Mises stress is σ'  σ1  σ1 σ2  σ2 σ'  224.301 MPa

3. Using the distortion-energy theory, the factor of safety against a static failure is

Sy
Ns  Ns  1.8
σ'

© 2011 Pearson Education, Inc., Upper Saddle River, NJ. All rights reserved. This publication is protected by Copyright and written permission should be
MACHINE DESIGN - An Integrated Approach, 4th Ed. 5-43-1
PROBLEM 5-43
Statement: The paper rolls in Figure P5-17 are 0.9-m OD by 0.22-m ID by 3.23-m long and have a density of
984 kg/m3. The rolls are transfered from the machine conveyor (not shown) to the forklift truck
by the V-linkage of the off-load station, which is rotated through 90 deg by an air cylinder. The
paper then rolls onto the waiting forks of the truck. The forks are 38-mm thick by 100-mm wide
by 1.2-m long and are tipped at a 3-deg angle from the horizontal and have Sy = 600MPa. Find
the safety factor for the two forks on the truck when the paper rolls onto it under two different
conditions (state all assumptions):
(a) The two forks are unsupported at their free end.
(b) The two forks are contacting the table at point A.

F
L fork
t
Given: Yield strength S y  600  MPa

Assumptions: 1. The greatest bending moment will occur


when the paper roll is at the tip of the fork M1
for case (a) and when it is midway between R1
supports for case (b).
Case (a), Cantilever Beam
2. Each fork carries 1/2 the weight of a
paper roll.
3. For case (a), each fork acts as a cantilever F
beam (see Appendix B-1(a)). 0.5 L fork
4. For case (b), each fork acts as a beam t
that is built-in at one end and
simply-supported at the other.
L fork
Solution: See Figure 5-43 and Mathcad file P0543. M2
R1 R2
1. From Problem 4-43, the maximum stresses in the Case (b), Fixed-Simply Supported Beam
forks are:
FIGURE 5-43
Case (a) σa  464.8  MPa Free Body Diagrams used in Problem 5-43
at the base of the fork.

Case (b) σb  87.2 MPa also at the base of the fork.

Since there are no other stress components present, these are also the maximum principal stresses and the
von Mises stresses. Thus, σ'a  σa and σ'b  σb.

Case (a)
Sy
2. The factor of safety against a static failure is Nsa  Nsa  1.3
σ'a

Case (b)
Sy
3. The factor of safety against a static failure is Nsb  Nsb  6.9
σ'b

© 2011 Pearson Education, Inc., Upper Saddle River, NJ. All rights reserved. This publication is protected by Copyright and written permission should be
MACHINE DESIGN - An Integrated Approach, 4th Ed. 5-44-1
PROBLEM 5-44
Statement: Determine a suitable thickness for the V-links of the off-loading station of Figure P5-17 to limit
their deflections at the tips to 10-mm in any position during their rotation. Two V-links support
the roll, at the 1/4 and 3/4 points along the roll's length, and each of the V arms is 10-cm wide by
1-m long. What is their safety factor against yielding when designed to limit deflection as
above?

Given: Roll OD OD  0.90 m Arm width wa  100  mm


Roll ID ID  0.22 m Arm length La  1000 mm
Roll length Lroll  3.23 m Max tip deflection δtip  10 mm
3
Roll density ρ  984  kg m Mod of elasticity E  207  GPa
Yield strength S y  400  MPa

Assumptions: 1. The maximum deflection on an arm will occur just after it begins the transfer and just before it
completes it, i.e., when the angle is either zero or 90 deg., but after the tip is no longer supported b
the base unit.
2. At that time the roll is in contact with both arms ("seated" in the V) and will remain in that state
throughout the motion. When the roll is in any other position on an arm the tip will be supported.
3. The arm can be treated as a cantilever beam with nonend load.
4. A single arm will never carry more than half the weight of a roll.
5. The pipe to which the arms are attached has OD = 160 mm.
Solution: See Figure 5-44 and Mathcad file P0544.

1. Determine the weight of the roll and the load on each 450
V-arm.

W 
π
4
 2 2 
 OD  ID  Lroll  ρ  g W  18.64  kN

F  0.5 W F  9.32 kN

2. From Appendix B, Figure B-1, the tip deflection of a


cantilever beam with a concentrated load located at a
distance a from the support is
2
F a
ymax =  ( a  3  L) 1000 = L
6  E I
370 = a
where L is the beam length and I is the F
cross-section moment of inertia. In this case
3
w a t a
I= M
12 F

3. Setting ymax = δtip and a  370  mm FIGURE 5-44


Free Body Diagram used in Problem 5-44
substituting for I and solving for ta
1
3
 2 F  a2  3 La  a 
ta  ta  31.889 mm
 E δtip  wa 
 
Let the arm thickness be ta  32 mm

4. The maximum bending stress in the arm will be at its base where it joins the 160-mm-dia pipe. The bending
moment, moment of inertia, and distance to the outside fiber at that point are:
© 2011 Pearson Education, Inc., Upper Saddle River, NJ. All rights reserved. This publication is protected by Copyright and written permission should be
MACHINE DESIGN - An Integrated Approach, 4th Ed. 5-44-2

Bending moment M  a  F M  3449 N  m


3
wa ta 5 4
Moment of inertia I  I  2.731  10  mm
12

Distance to outer fiber c  0.5 t a c  16 mm

5. The bending stress, which is also the von Mises stress, is


M c
σ'  σ'  202.1  MPa
I

6. Using the distortion-energy theory, the factor of safety against a static failure is

Sy
Ns  Ns  2.0
σ'

© 2011 Pearson Education, Inc., Upper Saddle River, NJ. All rights reserved. This publication is protected by Copyright and written permission should be
MACHINE DESIGN - An Integrated Approach, 4th Ed. 5-45-1
PROBLEM 5-45
Statement: Determine the safety factor based on critical load on the air cylinder rod in Figure P5-17 if the
crank arm that rotates it is 0.3 m long and the rod has a maximum extension of 0.5 m. The
25-mm-dia rod is solid steel with a yield strength of 400 MPa. State all assumptions.

Given: Rod length L  500  mm Young's modulus E  207  GPa


Rod diameter d  25 mm Yield strength S y  400  MPa
Assumptions: 1. The rod is a fixed-pinned column.
2. Use a conservative value of 1 for the end factor (see Table 4-7 in text).
Solution: See Problems 4-45, 4-47, and Mathcad file P0545.

1. From Problem 4-45, the critical load on the air cylinder rod is Pcr  134.8  kN

2. From Problem 4-47, the maximum load on the air cylinder rod is F  46.47  kN

Pcr
3. The factor of safety against a buckling failure is Nbuck  Nbuck  2.9
F

© 2011 Pearson Education, Inc., Upper Saddle River, NJ. All rights reserved. This publication is protected by Copyright and written permission should be
MACHINE DESIGN - An Integrated Approach, 4th Ed. 5-46-1
PROBLEM 5-46
Statement: The V-links of Figure P5-17 are rotated by the crank arm through a shaft that is 60 mm dia by
3.23 m long. Determine the maximum torque applied to this shaft during motion of the V-linkage
and find the static safety factor against yielding for the shaft if its S y = 400 MPa. See Problem
5-43 for more information.

Given: Yield strength S y  400  MPa

Assumptions: The greatest torque will occur


when the link is horizontal and the
paper roll is located as shown in
Figure P5-17 or Figure 5-46.

Solution: See Figure 5-46 and Mathcad file


P0546.
1. From Problem 4-46, the maximum torsional stress
in the shaft is W

τmax  197.88 MPa


T

2. Using the distortion-energy theory, the factor of


safety against static yielding is Ry
60-mm-dia shaft
450.0
Sy
Ns  Ns  1.2
3  τmax FIGURE 5-46
Free Body Diagram used in Problem 5-46

© 2011 Pearson Education, Inc., Upper Saddle River, NJ. All rights reserved. This publication is protected by Copyright and written permission should be
MACHINE DESIGN - An Integrated Approach, 4th Ed. 5-47-1
PROBLEM 5-47
Statement: Determine the maximum forces on the pins at each end of the air cylinder of Figure P4-17.
Determine the safety factor for these pins if they are 30-mm dia and in single shear. S y = 400 MPa.

Given: Paper roll dimensions OD  0.90 m Pin diameter d  30 mm


ID  0.22 m Yield strength S y  400  MPa
Lroll  3.23 m
3
Roll density ρ  984  kg m

Assumptions: 1. The maximum force in the cylinder rod occurs when the transfer starts.
2. The cylinder and rod make an angle of 8 deg to the horizontal at the start of transfer.
3. The crank arm is 300 mm long and is 45 deg from vertical at the start of transfer.
4. The cylinder rod is fully retracted at the start of the transfer. At the end of the transfer it will
have extended 500 mm from its initial position.

Solution: See Figure 4-47 and Mathcad file P0447.


1. Determine the weight of the roll on the y
forks.

W 
π
4
 2 2 
 OD  ID  Lroll  ρ  g

W  18.64 kN
2. From the assumptions and
Figure 4-47, the x and y
distances from the origin to
point A are,

Rax  300  cos( 45 deg)  mm


W
Ray  300  sin( 45 deg)  mm
Rx x

Rax  212.132 mm
Ry 212.1
A
Ray  212.132 mm
F 8°
212.1
450.0
3. From Figure 4-47, the x
distance from the origin to
point where W is applied is, FIGURE 4-47
Free Body Diagram at Start of Transfer for V-link of Problem 4-47

OD
Rwx  Rwx  450 mm
2

4. Sum moments about the pivot point and solve for the compressive force in the cylinder rod.

W  Rwx  F  Rax sin( 8  deg)  F  Ray cos( 8  deg) = 0

W  Rwx
F  F  46.469 kN
Ray cos( 8  deg)  Rax sin( 8  deg)

This is the shear force in the pins


5. Determine the cross-sectional area of the pins and the direct shear stress.
© 2011 Pearson Education, Inc., Upper Saddle River, NJ. All rights reserved. This publication is protected by Copyright and written permission should be
MACHINE DESIGN - An Integrated Approach, 4th Ed. 5-47-2

2
π d 2
Shear area A  A  706.858 mm
4

F
Shear stress τ  τ  65.7 MPa
A

6. Using the distortion-energy theory, the factor of safety against a static yielding failure is

Sy
Ns  Ns  3.5
3 τ

© 2011 Pearson Education, Inc., Upper Saddle River, NJ. All rights reserved. This publication is protected by Copyright and written permission should be
MACHINE DESIGN - An Integrated Approach, 4th Ed. 5-48-1
PROBLEM 5-48
Statement: Figure P5-18 shows an exerciser for a 100-kg wheelchair racer. The wheel chair has 65 cm dia
drive wheels separated by a 70-cm track width. Two free-turning rollers on bearings support the
rear wheels. The lateral movement of the chair is limited by the flanges. Design the 1-m-long
rollers as hollow tubes of aluminum (select alloy) to minimize the height of the platform and also
limit the roller deflections to 1 mm in the worst case. Specify suitable sized steel axles to support
the tubes on bearings. Calculate all significant stresses.

Given: Mass of chair M  100  kg Maximum deflection δ  1  mm


Wheel diameter d w  650  mm Modulus elasticity:
Track width T  700  mm Aluminum Ea  71.7 GPa
Roller length Lr  1000 mm Steel Es  207  GPa

Assumptions: 1. The CG of the chair with rider is


sufficiently close to the rear wheel that all of
the weight is taken by the two rear wheels.
2. The small camber angle of the rear wheels W/2
does not significantly affect the magnitude
of the forces on the rollers.
3. Both the aluminum roller and the steel axle
are simply supported. The steel axles that
support the aluminum tube are fixed in the
mounting block and do not rotate. The
aluminum tube is attached to them by two
bearings (one on each end of the tubes, one
for each axle). The bearings' inner race is
fixed, and the outer race rotates with the
F F
aluminum tube. Each steel axle is considered
to be loaded as a simply supported beam.
Their diameter must be less than the inner  
diameter of the tubes to fit the roller
bearings between them.
FIGURE 5-48A
Free Body Diagram of One Wheel
Solution: See Figures 5-48 and Mathcad file P0548.
used in Problem 5-48

1. Calculate the weight of the chair with rider.


Weight of chair W  M  g W  980.7 N

2. Calculate the forces exerted by the wheels on the rollers (see Figure 5-48A). From the FBD of a wheel, summing
vertical forces
W
2  F  cos( θ )  =0
2
W
Let θ  20 deg then F  F  260.9 N
4  cos( θ )

3. The worst condition (highest moment at site of a stress concentration) will occur when the chair is all the way to
the left or right. Looking at the plane through the roller that includes the forces exerted by the wheels (the FBD is
shown in Figure 5-48B) the reactions R1 and R2 come from the bearings, which are inside the hollow roller and are,
themselves, supported by the steel axle.

4. Solving for the reactions. Let the distance from R1 to F be a  15 mm

© 2011 Pearson Education, Inc., Upper Saddle River, NJ. All rights reserved. This publication is protected by Copyright and written permission should be
MACHINE DESIGN - An Integrated Approach, 4th Ed. 5-48-2

700
 M1 R2 Lr  F  ( a  T )  F  a = 0 F F

 Fy R1  2  F  R2 = 0

F  (2 a  T )
R2  R2  190.5 N 15 R2
Lr
R1 1000
R1  2  F  R2 R1  331.3 N
FIGURE 5-48B
5. The maximum bending moment will be Free Body Diagram of One Tube used in Problem 5-48
at the right-hand load and will be

Mrmax  R2 Lr  ( a  T ) Mrmax  54.3 N  m

Note, if the chair were centered on the roller the maximum moment would be
Lr  T
Mc  F  Mc  39.1 N  m
2
and this would be constant along the axle between the two loads, F.

6. Note that the bearing positions are fixed regardless of the position of the chair on the roller.

Because of symmetry,
65 1000
Ra1  R1 Ra1  331.3 N
R1 R2
Ra2  R2 Ra2  190.5 N

7. The maximum bending moment


R a1 R a2
occurs at R1 and is for b  65 mm
1130
Mamax  Ra1 b
FIGURE 5-48C
Free Body Diagram of One Axle used in Problem 5-48
Mamax  21.5 N  m

8. Determine a suitable axle diameter. Let the factor of safety against yielding in the axle be Nsa  3

9. Tentatively choose a low-carbon steel for the axle, say AISI 1020, cold rolled with S y  393  MPa

10. At the top of the axle under the load R1 there is only a bending stress, which is also the von Mises stress. Set th
stress equal to the yield strength divided by the factor of safety.

32 Mamax Sy
σ' = =
3 Nsa
π d a
1
3
 32 Nsa Mamax 
Solving for the axle diameter, d a d a    d a  11.875 mm
 π S y 
Let the axle diameter be d a  15 mm made from cold-rolled AISI 1020 steel.

© 2011 Pearson Education, Inc., Upper Saddle River, NJ. All rights reserved. This publication is protected by Copyright and written permission should be
MACHINE DESIGN - An Integrated Approach, 4th Ed. 5-48-3

11. Suppose that bearing 6302 from Chapter 10, Figure 10-23, page 684 is used. It has a bore of 15 mm and an
OD of 42 mm. Thus, the inside diameter of the roller away from the bearings where the moment is a maximum
will be d i  40 mm. This will provide a 1-mm shoulder for axial location of the bearings.

12. The maximum deflection of the roller will 150 700


occur when the chair is in the center of the F F
roller. For this case the reactions are both equal
to the loads, F (see Figure 4-48D). The
maximum deflection is at the center of the roller.
F
15
F
13. Write the load function and then 1000
integrate four times to get the deflection
function. FIGURE 5-48D
Free Body Diagram of Roller with Chair in the Center.

q(x) = F<x>-1 - F<x - a>-1 - F<x - b>-1 + F<x - L>-1

y(x) = F[<x>3 - <x - a>3 - <x - b>3 + <x - L>3 + C3x]/(6EI)

 ( L  a )  a  L 
1 3 3 3
where C3 =
L

14. Write the deflection function at x = L/2 for a  150  mm

 L 3 L
  1  ( L  a) 3  a3  L3
3
   
F
ymax =  2
6  Ea I  2 
 a 
  2 

15. Set this equation equal to the allowed deflection  and solve for the required moment of inertia, I.

 L 3 3 
F  r   Lr 
  a    Lr  a   a  Lr 
1 3 3 3 4 4
I      I  6.618  10 mm
6  Ea δ  2  2  2 
16. Knowing the inside diameter of the tube, solve for the outside diameter.
1
4
π  4
  d o  d i 
4  64 I 4
I= d o   π  di  d o  44.463 mm
64  

Round this up to d o  46 mm

DESIGN SUMMARY

Axles Rollers
Material AISI 1020 steel, cold-rolled Material 2024-T4 aluminum
Diameter d a  15 mm Outside diameter d o  46 mm
Length 1220 mm Inside diameter d i  40 mm
Length 1040 mm
Spacing c   d w  d o  sin( θ )
c  238 mm
© 2011 Pearson Education, Inc., Upper Saddle River, NJ. All rights reserved. This publication is protected by Copyright and written permission should be
MACHINE DESIGN - An Integrated Approach, 4th Ed. 5-49-1

PROBLEM 5-49 _____


Statement: A part made of ductile steel with Sy = 40 ksi is subjected to a three-dimensional stress state of 1
= -80 ksi, 2 = -80 ksi, 3 = -80 ksi. What is the maximum shear stress? Will the part fail?

Solution: See Mathcad file P0549.

1. This is a case of hydrostatic stress. As explained in Section 5.1, the maximum shear stress is zero. Parts loaded
hydrostatically can withstand stresses well in excess of their yield strength. One example of this is that parts on
the ocean floor such as those retrieved from the Titanic are intact and undistorted even though they are
surrounded by water at great pressure.

© 2011 Pearson Education, Inc., Upper Saddle River, NJ. All rights reserved. This publication is protected by Copyright and written permission should be
MACHINE DESIGN - An Integrated Approach, 4th Ed. 5-50-1

PROBLEM 5-50 _____


Statement: A component in the shape of a large sheet is to be fabricated from 7075-T651 aluminum, which
has a fracture toughness Kc = 24.2 MPa-m0.5 and a tensile yield strength of 495 MPa. Determine
the largest edge crack that could be tolerated in the sheet if the nominal stress does not exceed
one half the yield strength.
0.5
Given: Fracture toughness Kc  24.2 MPa m
Yield strength S y  495  MPa

Solution: Mathcad file P0550.

1. Calculate the nominal stress based on the yield strength and the stress level given in the problem statement.
Sy
σnom  σnom  247.5 MPa
2

2. Determine the value of the geometry factor  from the discussion in Section 5.3 for a plate with an edge crack.

β  1.12

3. Using equation 5.14b, calculate the critical crack length for this material under the given stress condition.

2
1 Kc 
a     a  2.4 mm
π  β  σnom 

© 2011 Pearson Education, Inc., Upper Saddle River, NJ. All rights reserved. This publication is protected by Copyright and written permission should be
MACHINE DESIGN - An Integrated Approach, 4th Ed. 5-51-1

PROBLEM 5-51 _____

Statement: A component in the shape of a large sheet is to be fabricated from 4340 steel, which has a fracture
toughness Kc = 98.9 MPa-m0.5 and a tensile yield strength of 860 MPa. The sheets are inspected
for crack flaws after fabrication, but the inspection device cannot detect flaws smaller than 5 mm.
The part is too heavy as designed. An engineer has suggested that the thickness be reduced and
the material be heat-treated to increase its tensile strength to 1515 MPa, which would result in
decreasing the fracture toughness to 60.4 MPa-m0.5. Assuming that the stress level does not
exceed one half the yield strength, is the suggestion feasible? If not, why not.

0.5 0.5
Given: Fracture toughness Kc1  98.9 MPa m Kc2  60.4 MPa m
Yield strength S y1  860  MPa S y2  1515 MPa

Solution: See Mathcad file P0551.

1. Calculate the nominal stress for the two material conditions based on the yield strength and the stress level
given in the problem statement.
S y1
σnom1  σnom1  430 MPa
2

S y2
σnom2  σnom2  757.5 MPa
2

2. Determine the value of the geometry factor  from the discussion in Section 5.3 for a large plate.

β  1

3. Using equation 5.14b, calculate the critical crack length for each material condition under the given stress
condition.

2
1  Kc1 
a 1    a 1  16.8 mm 2  a 1  33.7 mm
π  β  σnom1 

2
 Kc2 
1
a 2     a 2  2.0 mm 2  a 2  4.0 mm
π  β  σnom2 

4. The suggestion to increase the strength of the material so that its thickness can be decreased to save weight is
not feasible because the critical crack size of the material in the second condition is less than that which can be
detected by the inspection equipment.

© 2011 Pearson Education, Inc., Upper Saddle River, NJ. All rights reserved. This publication is protected by Copyright and written permission should be
MACHINE DESIGN - An Integrated Approach, 4th Ed. 5-52-1

PROBLEM 5-52 _____

Statement: A large plate is subjected to a nominal tensile stress of 350 MPa. The plate has a central crack
that is 15.9 mm long. Calculate the stress intensity factor at the tip of the crack.

Given: Nominal stress σnom  350  MPa


Crack length lcrack  15.9 mm

Solution: See Mathcad file P0552.

1. Calculate the half-width of the crack

a  0.5 l crack a  7.95 mm

2. Determine the value of the geometry factor  from the discussion in Section 5.3 for a plate with an edge crack.
β  1

3. Using equation 5.14b, calculate the stress intensity factor.

0.5
K  β  σnom π a K  55.3 MPa m

© 2011 Pearson Education, Inc., Upper Saddle River, NJ. All rights reserved. This publication is protected by Copyright and written permission should be
MACHINE DESIGN - An Integrated Approach, 4th Ed. 5-53-1

PROBLEM 5-53 _____

Statement: A movie scene calls for a stuntman to hang from a rope that is suspended 3 m above a pit of
poisonous spiders. The rope is attached to a glass sheet that is 3000 mm long by 100 mm wide
and 1.27 mm thick. The stuntman knows that the glass sheet contains a central crack with total
length of 16.2 mm that is oriented parallel to the ground. The fracture toughness of the glass is
0.83 MPa-m0.5. Should he do the stunt? Show all assumptions and calculations in support of
your answer.

0.5
Given: Fracture toughness Kc  0.83 MPa m
Glass dimensions L  3000 mm W  100  mm t  1.27 mm
Total crack length lcrack  16.2 mm

Assumptions: Weight of stuntman Weight  900  N


Desired safety factor NFMd  3

Solution: See Mathcad file P0553.

1. Calculate the nominal stress based on the assumed weight of the stuntman and the glass dimensions.
2
Cross-section area A  W  t A  127 mm

Weight
Nominal stress σnom  σnom  7.087 MPa
A

2. Determine the value of the geometry factor  from equation 5.14c for a plate with a central crack.

Crack half-width a  0.5 l crack a  8.1 mm

Glass half-width b  0.5 W b  50 mm

π a 
β  sec   β  1.017
 2 b 

3. Using equation 5.14b, calculate the stress intensity factor for the given assumptions.

0.5
K  β  σnom π a K  1.149 MPa m

4. Using equation 5.15, calculate the safety factor against sudden failure for the given assumptions.

Kc
NFM  NFM  0.72
K

5. The stuntman should definitely not do the stunt since the factor of safety is not only less than the desired
value, but is less than one.

© 2011 Pearson Education, Inc., Upper Saddle River, NJ. All rights reserved. This publication is protected by Copyright and written permission should be
MACHINE DESIGN - An Integrated Approach, 4th Ed. 5-54-1

PROBLEM 5-54 _____


Statement: A material has a fracture toughness of 50 MPa-m0.5 and a yield strength of 1000 MPa and is to be
made into a large panel. If the panel is stressed to one-half the yield stress, what is the maximum
central crack size that can be tolerated without catastrophic failure?

0.5
Given: Fracture toughness Kc  50 MPa m
Yield strength S y  1000 MPa

Solution: See Mathcad file P0554.

1. Calculate the nominal stress based on the yield strength and the stress level given in the problem statement.

Sy
σnom  σnom  500 MPa
2

2. Determine the value of the geometry factor  from the discussion in Section 5.3 for a large plate with a central
crack.

β  1

3. Using equation 5.14b, calculate the critical crack length for this material under the given stress condition.

2
 Kc 
1
a     a  3.18 mm
π  β  σnom 

lcritical  2  a lcritical  6.4 mm

© 2011 Pearson Education, Inc., Upper Saddle River, NJ. All rights reserved. This publication is protected by Copyright and written permission should be
MACHINE DESIGN - An Integrated Approach, 4th Ed. 5-55-1

PROBLEM 5-55 _____

Statement: A material that has a fracture toughness of 33 MPa-m0.5 is to be made into a large panel that is
2000 mm long by 250 mm wide and 4 mm thick. If the minimum allowable total crack length is 4
mm, what is the maximum tensile load in the long direction that can be applied without
catastrophic failure with a safety factor of 2.5?

0.5
Given: Fracture toughness Kc  33 MPa m
Panel dimensions L  2000 mm W  250  mm t  4  mm
Total allow. crack length lcrack  4  mm
Safety factor NFM  2.5

Solution: See Mathcad file P0555.

1. Calculate the allowable stress intensity factor using equation 5.15.

Kc 0.5
Kallow  Kallow  13.2 MPa m
NFM

2. Determine the value of the geometry factor  from equation 5.14c for a plate with a central crack.

Crack half-width a  0.5 l crack a  2 mm

Panel half-width b  0.5 W b  125 mm

π a 
β  sec   β  1.00
 2 b 

3. Using equation 5.14b, calculate the allowable nominal stress in the panel.

Kallow
σallow  σallow  166.5 MPa
β  π a

4. Calculate the allowable load for the given conditions.

Fallow  σallow W  t Fallow  167 kN

© 2011 Pearson Education, Inc., Upper Saddle River, NJ. All rights reserved. This publication is protected by Copyright and written permission should be
MACHINE DESIGN - An Integrated Approach, 4th Ed. 5-56-1

PROBLEM 5-56 _____


Statement: Figure P5-19 shows an SAE 1020 cold-rolled steel bar fastened to a rigid ground plane with two
0.25-in-dia A8 steel dowel pins, hardened to HRC52. For P = 1500 lb and t = 0.25 in, find:
(a) The safety factor for each pin.
(b) The safety factor for direct bearing stress in each hole.
(c) The safety factor for tearout failure if h = 1 in.

Given: Pin diameter d  0.250  in Depth of section h  1.0 in


Applied load P  1500 lbf Distance from right pin to load b  4.0 in
Distance between pins a  2.0 in Yield strength of bar S yb  57 ksi
Thickness of bar t  0.25 in Yield strength of pin S yp  225  ksi
Solution: See Mathcad file P0556.
1. Draw a free-body diagram and find the shear forces (reactions) on each pin.

a b
RL

RR

Write equations 3.3b for the bar and solve for the reactions.

 F: RL  RR  P  0  M: RL  a  P b  0

b
RL  P RL  3000 lbf RR  P  RL RR  4500 lbf
a

2. Calculate the cross-section area of a pin.


2
π d 2
A  A  0.0491 in
4

3. Use equation 4.9 to determine the shear stress in each pin.

RL
Left pin τL  τL  61.1 ksi
A
RR
Right pin τR  τR  91.7 ksi
A

4. (a) From equations 5.8c and 5.9b, the safety factor against failure in the pins is

0.577  S yp
Left pin NL  NL  2.1
τL
© 2011 Pearson Education, Inc., Upper Saddle River, NJ. All rights reserved. This publication is protected by Copyright and written permission should be
MACHINE DESIGN - An Integrated Approach, 4th Ed. 5-56-2

0.577  S yp
Right pin NR  NR  1.4
τR

5. Calculate the bearing area from equation 4.10 and use it to determine the bearing stress in each hole.
2
Bearing area Abear  d  t Abear  0.0625 in

RL
σL  σL  48.0 ksi
Abear
RR
σR  σR  72.0 ksi
Abear

These are principal stresses 1.

6. (b) Calculate the safety factor for direct bearing from equation 5.8c where 2 and 3 are both zero.

S yb
Left hole NL  NL  1.2
σL

S yb
Right hole NR  NR  0.8
σR

Atear  2  
h  d
7. The tearout area is   t , where (h - d)/2 is the distance from the edge of the hole to the
 2  
outside of the bar. Substitute this area in equation 4.9 for the shear area and solve for the shear strength xy.

h  d
Atear  2    2
  t Atear  0.187 in
 2  
RL
Left hole τL  τL  16.00 ksi
Atear

RR
Right hole τR  τR  24.00 ksi
Atear

8. (c) From equations 5.8c and 5.9b, the safety factor against tearout failure in the holes is

0.577  S yb
Left hole NL  NL  2.1
τL

0.577  S yb
Right hole NR  NR  1.4
τR

© 2011 Pearson Education, Inc., Upper Saddle River, NJ. All rights reserved. This publication is protected by Copyright and written permission should be
MACHINE DESIGN - An Integrated Approach, 4th Ed. 5-57-1

PROBLEM 5-57 _____


Statement: Figure P5-19 shows a class 50 cast iron bar fastened to a rigid ground plane with two 0.25-in-dia
A8 steel dowel pins, hardened to HRC52. For P = 1500 lb and t = 0.25 in, find:
(a) The safety factor for each pin.
(b) The safety factor for direct bearing stress in each hole.
(c) The safety factor for tearout failure if h = 1 in.

Given: Pin diameter d  0.250  in Depth of section h  1.0 in


Applied load P  1500 lbf Distance from right pin to load b  4.0 in
Distance between pins a  2.0 in Tensile strength of bar S utb  52 ksi
Thickness of bar t  0.25 in Yield strength of pin S yp  225  ksi

Solution: See Mathcad file P0557.


1. Draw a free-body diagram and find the shear forces (reactions) on each pin.

a b
RL

RR

Write equations 3.3b for the bar and solve for the reactions.

 F: RL  RR  P  0  M: RL  a  P b  0

b
RL  P RL  3000 lbf RR  P  RL RR  4500 lbf
a

2. Calculate the cross-section area of a pin.


2
π d 2
A  A  0.0491 in
4

3. Use equation 4.9 to determine the shear stress in each pin.

RL
Left pin τL  τL  61.1 ksi
A
RR
Right pin τR  τR  91.7 ksi
A

4. (a) From equations 5.8c and 5.9b, the safety factor against failure in the pins is

0.577  S yp
Left pin NL  NL  2.1
τL
© 2011 Pearson Education, Inc., Upper Saddle River, NJ. All rights reserved. This publication is protected by Copyright and written permission should be
MACHINE DESIGN - An Integrated Approach, 4th Ed. 5-57-2

0.577  S yp
Right pin NR  NR  1.4
τR

5. Calculate the bearing area from equation 4.10 and use it to determine the bearing stress in each hole.
2
Bearing area Abear  d  t Abear  0.0625 in

RL
σL  σL  48.0 ksi
Abear
RR
σR  σR  72.0 ksi
Abear

These are principal stresses 1.

6. (b) Calculate the safety factor for direct bearing from equation 5.12a where 2 and 3 are both zero.

S utb
Left hole NL  NL  1.1
σL

S utb
Right hole NR  NR  0.7
σR

Atear  2  
h  d
7. The tearout area is   t , where (h - d)/2 is the distance from the edge of the hole to the
 2  
outside of the bar. Substitute this area in equation 4.9 for the shear area and solve for the shear strength xy.

h  d
Atear  2    2
  t Atear  0.187 in
 2  
RL
Left hole τL  τL  16.00 ksi
Atear

RR
Right hole τR  τR  24.00 ksi
Atear

8. (c) For pure shear the Mohr circle is centered at 0,0 and has a radius equal to the shear stress. This results in 1
= . Using the Modified-Mohr failure theory and Figure 5-11, we see that we can use equation 5.12a for the
safety factor against tearout.

S utb S utb
NL  NL  3.3 NR  NR  2.2
τL τR

© 2011 Pearson Education, Inc., Upper Saddle River, NJ. All rights reserved. This publication is protected by Copyright and written permission should be
MACHINE DESIGN - An Integrated Approach, 4th Ed. 5-58-1

PROBLEM 5-58 _____


Statement: Figure P5-20 shows a bracket machined from 0.5-in-thick SAE 1045 cold-rolled steel flat stock. It
is rigidly attached to a support and loaded with P = 5000 lb at point D. Find:
(a) The safety factor against static failure at point A.
(b) The safety factor against static failure at point B.

Given: Distance from support to:


Point D d  8  in Points B and C b  17 in
Depth of section h  3  in Thickness of section t  0.5 in
Applied load P  5000 lbf Tensile yield strength S y  77 ksi

Assumptions: The bracket remains flat and does not buckle (out-of-plane) under the applied load.
Solution: See Mathcad file P0558.
1. Calculate the cross-section area and moment of inertia at A, B, and C, which are the same.
3
2 t h 4
A  h  t A  1.500 in I  I  1.1250 in
12
2. For part (a), draw a free-body diagram of the entire bracket.

M h y
B

h x

C
d D h

3. Use the equilibrium equations 3.3a to calculate the shear force and bending moment at the support.
 F: V  P  0  M: P ( d )  M  0
V  P V  5000 lbf MA  P ( d ) MA  40000 in lbf
4. The normal stress in the bracket at point A is determined using equation 4.11b.
Distance from neutral axis to extreme fiber c  0.5 h c  1.500 in
MA c
Normal stress at point A σA   σA  53.33 ksi
I
5. (a) The transverse shear stress in the bracket at point A is zero, therefore A is a principal stress. There are no
stress components in the y or z directions so this is a case of uniaxial stress. Thus, equations 5.7 reduce to
2
σ'  σA σ'  53.3 ksi

Use equation 5.8a to calculate the factor of safety against a static failure at point A.

© 2011 Pearson Education, Inc., Upper Saddle River, NJ. All rights reserved. This publication is protected by Copyright and written permission should be
MACHINE DESIGN - An Integrated Approach, 4th Ed. 5-58-2
Sy
NA  NA  1.4
σ'

6. For part (b), draw a free-body diagram of the portion of the bracket that is below point B.

F
y
M
B
x

d D h

7. Use the equilibrium equations 3.3a to calculate the normal force and bending moment on the section shown.

 F: F  P  0  M: P ( b  0.5 h  d )  M  0
F  P F  5000 lbf MB1  P ( b  0.5 h  d ) MB1  52500 in lbf

8. The normal stress in the bracket at point B in the y direction is a combination of uniform tension and bending
and is determined by summing equations 4.7 and 4.11b.
MB1 c F
Normal stress at B in y direction σBy   σBy  73.33 ksi
I A

9. The normal stress in the bracket at point B in the x direction is bending and is determined from equation 4.11b,
using the FBD from part (a).

MB2  V  b  MA MB2  45000 in lbf

MB2 c
Normal stress at B in x direction σBx  σBx  60.00 ksi
I

10. (b) The transverse shear at B due to the shear force V is zero so Bx and By are the only stress components at B.
Use equations 5.7d and 5.8a to determine the factor of safety against a static failure at B (ignoring the stress
concentration there).

2 2
σ'  σBx  σBy  σBx σBy σ'  67.66 ksi

Sy
NB  NB  1.1
σ'

© 2011 Pearson Education, Inc., Upper Saddle River, NJ. All rights reserved. This publication is protected by Copyright and written permission should be
MACHINE DESIGN - An Integrated Approach, 4th Ed. 5-59-1

PROBLEM 5-59 _____


Statement: Figure P5-20 shows a bracket machined from 1-in-thick class 60 cast iron flat stock. It is rigidly
attached to a support and loaded with P = 5000 lb at point D. Find:
(a) The safety factor against static failure at point A.
(b) The safety factor against static failure at point B.

Given: Distance from support to:


Point D d  8  in Points B and C b  17 in
Depth of section h  3  in Thickness of section t  0.5 in
Applied load P  5000 lbf Ultimate tensile strength S ut  62 ksi
Ultimate comp. strength S uc  187  ksi
Assumptions: The bracket remains flat and does not buckle (out-of-plane) under the applied load.
Solution: See Mathcad file P0559.
1. Calculate the cross-section area and moment of inertia at A, B, and C, which are the same.
3
2 t h 4
A  h  t A  1.500 in I  I  1.1250 in
12
2. For part (a), draw a free-body diagram of the entire bracket.

M h y
B

h x

C
d D h

3. Use the equilibrium equations 3.3a to calculate the shear force and bending moment at the support.
 F: V  P  0  M: P ( d )  M  0
V  P V  5000 lbf MA  P ( d ) MA  40000 in lbf
4. The normal stress in the bracket at point A is determined using equation 4.11b.
Distance from neutral axis to extreme fiber c  0.5 h c  1.500 in
MA c
Normal stress at point A σA   σA  53.33 ksi
I
5. (a) The transverse shear stress in the bracket at point A is zero, therefore A is a principal stress. There are no
stress components in the y or z directions so this is a case of uniaxial stress. Thus, use equation 5.12a (adapted
to a compressive stress state) to calculate the factor of safety against a static failure at point A.

S uc
NA  NA  3.5
σA

© 2011 Pearson Education, Inc., Upper Saddle River, NJ. All rights reserved. This publication is protected by Copyright and written permission should be
MACHINE DESIGN - An Integrated Approach, 4th Ed. 5-59-2

6. For part (b), draw a free-body diagram of the portion of the bracket that is below point B.

F
y
M
B
x

d D h

7. Use the equilibrium equations 3.3a to calculate the normal force and bending moment on the section shown.

 F: F  P  0  M: P ( b  0.5 h  d )  M  0
F  P F  5000 lbf MB1  P ( b  0.5 h  d ) MB1  52500 in lbf

8. The normal stress in the bracket at point B in the y direction is a combination of uniform tension and bending
and is determined by summing equations 4.7 and 4.11b.
MB1 c F
Normal stress at B in y direction σBy   σBy  73.33 ksi
I A

9. The normal stress in the bracket at point B in the x direction is bending and is determined from equation 4.11b,
using the FBD from part (a).

MB2  V  b  MA MB2  45000 in lbf

MB2 c
Normal stress at B in x direction σBx  σBx  60.00 ksi
I
10. (b) The transverse shear at B due to the shear force V is zero so Bx and By are the only stress components at B.
Use equations 4.6 to determine the principal stresses and 5.12a to determine the factor of safety against a static
failure at B (ignoring the stress concentration there).

2
σBx  σBy  σBx  σBy 
σ1     σ1  73.333 ksi
2  2 
2
σBx  σBy  σBx  σBy 
σ2     σ2  60.000 ksi
2  2 
S ut
σ3  0  ksi NB  NB  0.85
σ1

© 2011 Pearson Education, Inc., Upper Saddle River, NJ. All rights reserved. This publication is protected by Copyright and written permission should be
MACHINE DESIGN - An Integrated Approach, 4th Ed. 5-60-1

PROBLEM 5-60 _____


Statement: Figure P5-21 shows a 1-in-dia SAE 1040 hot-rolled, normalized steel bar supported and subjected
to the applied load P = 500 lb. Find the safety factor against static failure.
6
Given: Diameter d  1.00 in Modulus of elasticity E  30 10  psi
Applied load P  500  lbf Yield strength S y  54 ksi
Dimensions: a  20 in L  40 in
Solution: See Mathcad file P0560.
1. Draw a free-body diagram.

R2

M1

R1 P

2. This is a statically indeterminate beam because there are three unknown reactions, R1, M1, and R2. To solve
for these unknowns, follow the method presented in Example 4-7. First, calculate the moment of inertia and
distance to the extreme fiber for the round section.
4
π d 4
I  I  0.0491 in c  0.5 d c  0.500 in
64
3. From inspection of the FBD, write the load function equation
q(x) = -M1<x>-2 + R1<x>-1 - R2<x - a>-1 + P<x - L>-1
4. Integrate this equation from - to x to obtain shear, V(x)
V(x) = -M1<x>-1 + R1<x>0 - R2<x - a>0 + P<x - L>0
5. Integrate this equation from - to x to obtain moment, M(x)
M(x) = -M1<x>0 + R1<x>1 - R2<x - a>1 + P<x - L>1
6. Integrate the moment function, multiplying by 1/EI, to get the slope.
(x) = [ -M1<x>1 + R1<x>2/2 - R2<x - a>2/2 + P<x - L>2/2 + C3]/EI
5. Integrate again to get the deflection.
y(x) = [-M1<x>2/2 + R1<x>3/6 - R2<x - a>3/6 + P<x - L>3/6 + C3x + C4]/EI

7. Evaluate R1, M1, R2, C3 and C4


At x = 0, y = 0 and  = 0, therefore, C3 = 0 and C4 = 0.
At x = a, y = 0
At x = L+, V = M = 0
Guess M1  1000 in lbf R1  500  lbf R2  1000 lbf

M1 2 R1 3 3
Given y(a) = 0:  a   a = 0  lbf  in
2 6

V(L) = 0: R1  R2  P = 0  lbf

© 2011 Pearson Education, Inc., Upper Saddle River, NJ. All rights reserved. This publication is protected by Copyright and written permission should be
MACHINE DESIGN - An Integrated Approach, 4th Ed. 5-60-2

M(L) = 0: M1  R1 L  R2 ( L  a ) = 0  lbf  in

 M1 
 
 R1   Find  M1 R1 R2 M1  5000 in lbf R1  750 lbf
R 
 2 R2  1250 lbf

8. Define the range for x x  0  in 0.02 L  L


9. For a Mathcad solution, define a step function S. This function will have a value of zero when x is less than z,
and a value of one when it is greater than or equal to z.

S ( x z)  if ( x  z 1 0 )


10. Write the moment equation in Mathcad form, using the function S as a multiplying factor to get the effect of the
singularity functions.

M ( x)  M1  R1 S ( x 0  in)  x  R2 S ( x a )  ( x  a )  P S ( x L)  ( x  L)

11. Plot the moment equation and determine the maximum bending moment.

As expected, the maximum bending MOMENT DIAGRAM


moment occurs under the support at 10
x = a.

Mmax  M ( a ) Mmax  10.0 kip in


5
M ( x)
kip  in
0

5
0 10 20 30 40
x
in
12. Use equation 4.11b to calculate the maximum bending stress in the bar.

Mmax c
σmax  σmax  101.9 ksi
I

13. There are no other stress components present (the transverse shear is zero at the extreme fiber) so this is a
principal stress and the other two principal stresses are zero. Thus, this is a case of uniaxial stress. Determine
the safety factor against static failure using equations 5.7c and 5.8a, which reduce to

Sy
N  N  0.53
σmax

© 2011 Pearson Education, Inc., Upper Saddle River, NJ. All rights reserved. This publication is protected by Copyright and written permission should be
MACHINE DESIGN - An Integrated Approach, 4th Ed. 5-61-1

PROBLEM 5-61 _____


Statement: Figure P5-21 shows a 1.5-in-dia class 60 cast iron bar supported and subjected to the applied load
= 500 lb. Find the safety factor against static failure.
6
Given: Diameter d  1.50 in Modulus of elasticity E  30 10  psi
Applied load P  500  lbf Tensile strength S ut  54 ksi
Dimensions: a  20 in L  40 in
Solution: See Mathcad file P0561.
1. Draw a free-body diagram.

R2

M1

R1 P

2. This is a statically indeterminate beam because there are three unknown reactions, R1, M1, and R2. To solve
for these unknowns, follow the method presented in Example 4-7. First, calculate the moment of inertia and
distance to the extreme fiber for the round section.
4
π d 4
I  I  0.2485 in c  0.5 d c  0.750 in
64
3. From inspection of the FBD, write the load function equation
q(x) = -M1<x>-2 + R1<x>-1 - R2<x - a>-1 + P<x - L>-1
4. Integrate this equation from - to x to obtain shear, V(x)
V(x) = -M1<x>-1 + R1<x>0 - R2<x - a>0 + P<x - L>0
5. Integrate this equation from - to x to obtain moment, M(x)
M(x) = -M1<x>0 + R1<x>1 - R2<x - a>1 + P<x - L>1
6. Integrate the moment function, multiplying by 1/EI, to get the slope.
(x) = [ -M1<x>1 + R1<x>2/2 - R2<x - a>2/2 + P<x - L>2/2 + C3]/EI
5. Integrate again to get the deflection.
y(x) = [-M1<x>2/2 + R1<x>3/6 - R2<x - a>3/6 + P<x - L>3/6 + C3x + C4]/EI

7. Evaluate R1, M1, R2, C3 and C4


At x = 0, y = 0 and  = 0, therefore, C3 = 0 and C4 = 0.
At x = a, y = 0
At x = L+, V = M = 0
Guess M1  1000 in lbf R1  500  lbf R2  1000 lbf

M1 2 R1 3 3
Given y(a) = 0:  a   a = 0  lbf  in
2 6

V(L) = 0: R1  R2  P = 0  lbf

© 2011 Pearson Education, Inc., Upper Saddle River, NJ. All rights reserved. This publication is protected by Copyright and written permission should be
MACHINE DESIGN - An Integrated Approach, 4th Ed. 5-61-2

M(L) = 0: M1  R1 L  R2 ( L  a ) = 0  lbf  in

 M1 
 
 R1   Find  M1 R1 R2 M1  5000 in lbf R1  750 lbf
R 
 2 R2  1250 lbf

8. Define the range for x x  0  in 0.02 L  L


9. For a Mathcad solution, define a step function S. This function will have a value of zero when x is less than z,
and a value of one when it is greater than or equal to z.

S ( x z)  if ( x  z 1 0 )


10. Write the moment equation in Mathcad form, using the function S as a multiplying factor to get the effect of the
singularity functions.

M ( x)  M1  R1 S ( x 0  in)  x  R2 S ( x a )  ( x  a )  P S ( x L)  ( x  L)

11. Plot the moment equation and determine the maximum bending moment.

As expected, the maximum bending MOMENT DIAGRAM


moment occurs under the support at 10
x = a.

Mmax  M ( a ) Mmax  10.0 kip in


5
M ( x)
kip  in
0

5
0 10 20 30 40
x
in
12. Use equation 4.11b to calculate the maximum bending stress in the bar.

Mmax c
σmax  σmax  30.2 ksi
I

13. There are no other stress components present (the transverse shear is zero at the extreme fiber) so this is a
principal stress and the other two principal stresses are zero. Thus, this is a case of uniaxial stress. Determine
the safety factor against static failure using equation 5.12a.

S ut
N  N  1.8
σmax

© 2011 Pearson Education, Inc., Upper Saddle River, NJ. All rights reserved. This publication is protected by Copyright and written permission should be
MACHINE DESIGN - An Integrated Approach, 4th Ed. 5-62-1

PROBLEM 5-62 _____


Statement: Figure P5-22 shows a pivot pin that is press-fit into part A and is slip fit in part B. If F = 100 lb,
l = 2 in, and d = 0.5 in, what is the pin's safety factor against yielding when made of SAE 1020
cold-rolled steel?

Given: Applied force F  100  lbf Yield strength S y  57 ksi


Total length, l l  2.00 in Beam length L  0.5 l L  1.000  in
Pin dia d  0.5 in
Assumptions: 1. Since there is a slip fit between the pin and part B, part B offers no resistance to bending of
the pin and, since the pin is press-fit into part A, it can be modeled as a cantilever beam of
length l/2.
2. Part B distributes the concentrated force F so that, at the pin, it is uniformly distributed over
the exposed length of the pin.

Solution: See Mathcad file P0562.


1. Calculate the intensity of the uniformly distributed load acting over the length of the pin.
F lbf
w  w  100.0 
L in

2. A cantilever beam with uniform loading is shown in Figure B-1(b) in Appendix B. In this case, the dimension a
the figure is zero. As shown in the figure, when a = 0, the maximum bending moment occurs at the support and
is
2
w L
Mmax  Mmax  50.00  lbf  in
2

3. Calculate the moment of inertia and distance to the extreme fiber of the pin. The bending stress in the beam is
then found using equation 4.11b.
4
π d 3 4
I  I  3.068  10  in
64

c  0.5 d c  0.250  in

Mmax c
σ  σ  4074 psi
I

4. There are no other stress components present (the transverse shear is zero at the extreme fiber) so this is a
principal stress and the other two principal stresses are zero. Thus, this is a case of uniaxial stress. Determine
the safety factor against static failure using equations 5.7c and 5.8a, which reduce to

Sy
N  N  14
σ

© 2011 Pearson Education, Inc., Upper Saddle River, NJ. All rights reserved. This publication is protected by Copyright and written permission should be
MACHINE DESIGN - An Integrated Approach, 4th Ed. 5-63-1

PROBLEM 5-63 _____


Statement: Figure P5-22 shows a pivot pin that is press-fit into part A and is slip fit in part B. If F = 100 N, l =
50 mm, and d = 16 mm, what is the pin's safety factor against yielding when made of class 50 cast
iron?

Given: Applied force F  100  N Tensile strength S ut  359  MPa


Total length, l l  50 mm Beam length L  0.5 l L  25 mm
Pin dia d  16 mm
Assumptions: 1. Since there is a slip fit between the pin and part B, part B offers no resistance to bending of
the pin and, since the pin is press-fit into part A, it can be modeled as a cantilever beam of length
l/2.
2. Part B distributes the concentrated force F so that, at the pin, it is uniformly distributed over
the exposed length of the pin.

Solution: See Mathcad file P0563.


1. Calculate the intensity of the uniformly distributed load acting over the length of the pin.
F N
w  w  4.0
L mm

2. A cantilever beam with uniform loading is shown in Figure B-1(b) in Appendix B. In this case, the dimension a
the figure is zero. As shown in the figure, when a = 0, the maximum bending moment occurs at the support and
is
2
w L
Mmax  Mmax  1250 N  mm
2

3. Calculate the moment of inertia and distance to the extreme fiber of the pin. The bending stress in the beam is
then found using equation 4.11b.
4
π d 3 4
I  I  3.217  10  mm
64

c  0.5 d c  8.000  mm

Mmax c
σ  σ  3.108  MPa
I

4. There are no other stress components present (the transverse shear is zero at the extreme fiber) so this is a
principal stress and the other two principal stresses are zero. Thus, this is a case of uniaxial stress. Determine
the safety factor against static failure using equation 5.12a.

S ut
N  N  115
σ

© 2011 Pearson Education, Inc., Upper Saddle River, NJ. All rights reserved. This publication is protected by Copyright and written permission should be
MACHINE DESIGN - An Integrated Approach, 4th Ed. 5-64-1

PROBLEM 5-64
Statement: A differential element is subjected to the stresses (in ksi): x = 10, y = -20, xy = -20. The material
is uneven and has strengths (in ksi) of S ut = 50, S y = 40, and S uc = 90. Calculate the safety factor
and draw a a-b diagram showing the boundary for each theory with the stress state and load
line using:
(a) Coulomb-Mohr theory, and
(b) Modified Mohr theory.
Given: Stress components σx  10 ksi σy  20 ksi τxy  20 ksi
Material properties S ut  50 ksi S y  40 ksi S uc  90 ksi

Solution: See Figure 5-62 and Mathcad file P0564.


1. Calculate the nonzero principal stresses using equation 4.6a.

2
σx  σy  σx  σy  2
σa      τxy σa  20 ksi
2  2 
2
σx  σy  σx  σy  2
σb      τxy σb  30 ksi
2  2 
2. Calculate the slope of load line. (The load line is the line from the origin through the stress point.)

σb
m  m  1.5
σa

3. The safety factor equation for both theories is different for each quadrant the load line falls in. The equation
for the modified Mohr factor of safety is different for each of the two regions in the 4th quadrant that the load
line can fall in. In this case, the load line falls in the 4th quadrant, below the -1 slope line.. The factors of
safety are:

S uc S ut
(a) Coulomb-Mohr theory Na  Na  1.4
S uc σa  S ut σb

S uc
(b) Modified Mohr theory Nb  Nb  2
 S uc  S ut 
 S   σa  σb
 ut 

4. Plot the a-b diagram showing the safe-fail boundaries, the stress state point (20 ksi, -30 ksi) and the load line.
Note that if a > b , then only that area on the graph that is to the right of and below the diagonal line can
contain valid stress points. The factor of safety is the distance along the load line from the origin to the
intersection of the load line with the failure boundary, divided by the distance from the origin to the stress poin
Since the distance from the origin to the modified Mohr boundary is greater than the distance to the
Coulomb-Mohr boundary, its factor of safety is greater. See Figure 5-62 on the following page.

© 2011 Pearson Education, Inc., Upper Saddle River, NJ. All rights reserved. This publication is protected by Copyright and written permission should be
MACHINE DESIGN - An Integrated Approach, 4th Ed. 5-64-2

b
50

40

30

20

MINIMUM NONZERO PRINCIPAL STRESS, KSI


(a) Coulomb-Mohr
boundary
10

0 a
-10

-20
(20,-30)
-30

-40

-50 Stress states at -S


which failure ut
-60 will occur

-70 (b) Modified Mohr


boundary
Load Line
-80

-90 -S
uc
-100
-100 -90 -80 -70 -60 -50 -40 -30 -20 -10 0 10 20 30 40 50
MAXIMUM PRINCIPAL STRESS, KSI

FIGURE 5-64
a -  b Diagram for Problem 5-64

© 2011 Pearson Education, Inc., Upper Saddle River, NJ. All rights reserved. This publication is protected by Copyright and written permission should be
MACHINE DESIGN - An Integrated Approach, 4th Ed. 5-65-1

PROBLEM 5-65
Statement: A differential element is subjected to the stresses (in ksi): x = 10, y = -5, xy = 15. The material is
uneven and has strengths (in ksi) of S ut = 50, S y = 40, and S uc = 90. Calculate the safety factor and
draw a a-b diagram showing the boundary for each theory with the stress state and load line
using:
(a) Coulomb-Mohr theory, and
(b) Modified Mohr theory.
Given: Stress components σx  10 ksi σy  5  ksi τxy  15 ksi
Material properties S ut  50 ksi S y  40 ksi S uc  90 ksi

Solution: See Figure 5-65 and Mathcad file P0565.

1. Calculate the nonzero principal stresses using equation 4.6a.

2
σx  σy  σx  σy  2
σa      τxy σa  19.3 ksi
2  2 
2
σx  σy  σx  σy  2
σb      τxy σb  14.3 ksi
2  2 
2. Calculate the slope of load line. (The load line is the line from the origin through the stress point.)

σb
m  m  0.741
σa

3. The safety factor equation for both theories is different for each quadrant the load line falls in. The equation
for the modified Mohr factor of safety is different for each of the two regions in the 4th quadrant that the load
line can fall in. In this case, the load line falls in the 4th quadrant, above the -1 slope line.. The factors of
safety are:

S uc S ut
(a) Coulomb-Mohr theory Na  Na  1.8
S uc σa  S ut σb

S ut
(b) Modified Mohr theory Nb  Nb  2.6
σa

4. Plot the a-b diagram showing the safe-fail boundaries, the stress state point (19.3 ksi, -14.3 ksi) and the load
line. Note that if a > b , then only that area on the graph that is to the right of and below the diagonal line can
contain valid stress points. The factor of safety is the distance along the load line from the origin to the
intersection of the load line with the failure boundary, divided by the distance from the origin to the stress poin
Since the distance from the origin to the modified Mohr boundary is greater than the distance to the
Coulomb-Mohr boundary, its factor of safety is greater. See Figure 5-63 on the following page.

© 2011 Pearson Education, Inc., Upper Saddle River, NJ. All rights reserved. This publication is protected by Copyright and written permission should be
MACHINE DESIGN - An Integrated Approach, 4th Ed. 5-65-2

b
50

40

30

20 (a) Coulomb-Mohr

MINIMUM NONZERO PRINCIPAL STRESS, KSI


boundary
10

0 a
-10 (19.3,-14.3)

-20

-30

-40 Load Line

-50 Stress states at -S


which failure ut
-60 will occur

-70 (b) Modified Mohr


boundary
-80

-90 -S
uc
-100
-100 -90 -80 -70 -60 -50 -40 -30 -20 -10 0 10 20 30 40 50
MAXIMUM PRINCIPAL STRESS, KSI

FIGURE 5-65
a -  b Diagram for Problem 5-65

© 2011 Pearson Education, Inc., Upper Saddle River, NJ. All rights reserved. This publication is protected by Copyright and written permission should be
MACHINE DESIGN - An Integrated Approach, 4th Ed. 5-66-1

PROBLEM 5-66
Statement: A differential element is subjected to the stresses (in ksi): x = -20, y = -15, xy = 15. The material
is uneven and has strengths (in ksi) of S ut = 50, S y = 40, and S uc = 90. Calculate the safety factor
and draw a a-b diagram showing the boundary for each theory with the stress state and load
line using:
(a) Coulomb-Mohr theory, and
(b) Modified Mohr theory.
Given: Stress components σx  20 ksi σy  15 ksi τxy  15 ksi
Material properties S ut  50 ksi S y  40 ksi S uc  90 ksi

Solution: See Figure 5-66 and Mathcad file P0566.

1. Calculate the nonzero principal stresses using equation 4.6a.

2
σx  σy  σx  σy  2
σa      τxy σa  2.29 ksi
2  2 
2
σx  σy  σx  σy  2
σb      τxy σb  32.7 ksi
2  2 
2. Calculate the slope of load line. (The load line is the line from the origin through the stress point.)

σb
m  m  14.263 (third quadrant since both principal
σa stresses are negative)

3. The safety factor equation for both theories is the same when the load line falls in the third quadrant. The
factors of safety are:

S uc
(a) Coulomb-Mohr theory Na   Na  2.8
σb

S uc
(b) Modified Mohr theory Nb   Nb  2.8
σb

4. Plot the a-b diagram showing the safe-fail boundaries, the stress state point (-2.29 ksi, -32.7 ksi) and the load
line. Note that if a > b , then only that area on the graph that is to the right of and below the diagonal line
can contain valid stress points. The factor of safety is the distance along the load line from the origin to the
intersection of the load line with the failure boundary, divided by the distance from the origin to the stress
point. Since the distance from the origin to the modified Mohr boundary is the same as the distance to the
Coulomb-Mohr boundary, its factor of safety is the same. See Figure 5-63 on the following page.

© 2011 Pearson Education, Inc., Upper Saddle River, NJ. All rights reserved. This publication is protected by Copyright and written permission should be
MACHINE DESIGN - An Integrated Approach, 4th Ed. 5-66-2

b
50

40

30

20

MINIMUM NONZERO PRINCIPAL STRESS, KSI


(a) Coulomb-Mohr
boundary
10

0 a
-10

-20

-30
(-2.29,-32.7)
-40

-50 Stress state at -S


which failure ut
-60 will occur for
both theories
-70

-80
(b) Modified Mohr
-90 -S boundary
uc
Load Line
-100
-100 -90 -80 -70 -60 -50 -40 -30 -20 -10 0 10 20 30 40 50
MAXIMUM PRINCIPAL STRESS, KSI

FIGURE 5-66
a -  b Diagram for Problem 5-66

© 2011 Pearson Education, Inc., Upper Saddle River, NJ. All rights reserved. This publication is protected by Copyright and written permission should be
MACHINE DESIGN - An Integrated Approach, 4th Ed. 5-67-1

PROBLEM 5-67 _____

Statement: Derive the von Mises effective stress equation 5.7d for the two-dimensional case.

Solution: See Mathcad file P0567.

1. Start with equation 5.7c, which gives the von Mises stress in terms of the two nonzero principal stresses.

2 2
σ'  σ1  σ1 σ3  σ3 (a)

2. Define 1 and 3 in terms of x, y, and xy using equations 4.6a.

2
σx  σy  σx  σy  2
σ1      τxy (b)
2  2 

2
σx  σy  σx  σy  2
σ3      τxy (c)
2  2 

3. To make the manipulations easier, define:

2
σx  σy  σx  σy  2
σc  and R     τxy (d)
2  2 

4. Substitute equations d into b and c.

σ1  σc  R σ3  σc  R (e)

5. Substitute equations e into a, expand, collect terms and simplify.

σ'  σc  R 2  σc  R  σc  R  σc  R 2

2 2 2 2 2 2
σ'  σc  2  R σc  R  σc  R  σc  2  R σc  R (f)

2 2
σ'  σc  3  R

6. Substitute equations d into f, expand, collect terms and simplify to obtain the derived equation.

2 2
 σx  σy   σx  σy  2
σ'     3    3  τxy
 2   2 

2 2 2
σ'  σx  σy  σx σy  3  τxy (5.7d)

© 2011 Pearson Education, Inc., Upper Saddle River, NJ. All rights reserved. This publication is protected by Copyright and written permission should be
MACHINE DESIGN - An Integrated Approach, 4th Ed. 5-68-1

PROBLEM 5-68
Statement: Figure P5-23 shows an oil-field pump jack. The crank drive shaft at O2 is loaded in torsion and
bending with maximum values of 6500 in-lb and 9800 in-lb, respectively. The point on the shaft
with maximum stress is located away from the key that connects the shaft to the crank. Using a
factor of safety of 2 against static yielding, determine a suitable diameter for the shaft if it is to be
made of SAE 1040 cold-rolled steel.

Given: Yield strength S y  71 ksi Factor of safty Ns  2

Torque T  6500 in lbf Bending moment M  9800 in lbf

Solution: See Figure P5-23 and Mathcad file P0568.

1. Express the torsional and bending stresses as a functions of the unknown shaft diameter, d

32 M
Bending stress σx( d ) 
3
π d

16 T
Torsional stress τxy( d ) 
3
π d

2. Use these two stresses in an expression for the von Mises effective stress, equation 5.7d withy = 0.

2 2
von Mises effective stress σ'( d )  σx( d )  3  τxy( d )

3. Use equation 5.8a as a design relationship to solve for the diameter, d.

2 2 Sy
Design equation σx( d )  3  τxy( d ) =
Ns

2 2 2
 32 M   3  16 T  =  S y 
 3  3   Ns 
 π d   π d   

Solving for d
1
6
 ( 32 M ) 2  3  ( 16 T ) 2
d    d  1.480 in
 2  Sy 
2 
 π   
  Ns  

A suitable diameter for the given design requirements is d  1.500  in

© 2011 Pearson Education, Inc., Upper Saddle River, NJ. All rights reserved. This publication is protected by Copyright and written permission should be
MACHINE DESIGN - An Integrated Approach, 4th Ed. 5-69-1

PROBLEM 5-69
Statement: Figure P5-24a shows a C-clamp with an elliptical body dimensioned as shown. The clamp has a
T-section with a uniform thickness of 3.2 mm at the throat as shown in Figure P5-24b. Find the
static factor of safety if the clamping force is 2.7 kN and the material is class 40 gray cast iron.

Given: Clamping force F  2.7 kN


Distance from center of screw to throat ri  63.5 mm
Section dimensions: Flange b  28.4 mm Web h  31.8 mm
t  3.2 mm
Material properties S ut  290  MPa S uc  965  MPa
Solution: See Figure P4-26 and Mathcad file P0569.

1. Determine the location of the CG of the T-section and the distance from the centerline of the screw to the
centroid of the section at the throat.
0.5 t ( b  t)  0.5 ( h  t)  ( h  t) t
yCG  yCG  9.578  mm
b t  ( h  t)  t

rc  ri  yCG rc  73.078 mm

2. Using equation 4.12a and Figure 4-16, calculate the distance to the neutral axis, rn, and the distance from the
centroidal axis to the neutral axis, e.
Distance from the screw centerline to the outside fiber ro  ri  h ro  95.3 mm
2
Cross section area A  b  t  ( h  t)  t A  182.4  mm

A
Distance to neutral axis rn  rn  71.864 mm
ri t ro
 b  t
 dr   dr
 r  r
r r  t
i i

Distance from centroidal to neutral axis e  rc  rn e  1.214  mm

3. Take a section through the throat area and draw a FBD. There will be a vertical axial force through the section
CG (at a distance rc from the screw centerline) which will form a couple of magnitude rc x F. This couple will be
balanced by an internal moment of equal magnitude.
Internal moment M  rc F M  197  N  m

4. Calculate the distances from the neutral axis to the inner and outer fibers.
ci  rn  ri ci  8.364  mm co  ro  rn co  23.436 mm

5. Using equations 4.12d and 4.12e, calculate the stresses at the inner and outer fibers of the throat section.

M  ci  F
Inner radius σi    σi  132.2  MPa
e A  ri  A

M  co  F
Outer radius σo     σo  204.3  MPa
e A  ro  A

6. These are the principal stresses so,

© 2011 Pearson Education, Inc., Upper Saddle River, NJ. All rights reserved. This publication is protected by Copyright and written permission should be
MACHINE DESIGN - An Integrated Approach, 4th Ed. 5-69-2

Inner radius σ1i  σi σ2i  0  MPa σ3i  0  MPa

Outer radius σ1o  0  MPa σ2o  0  MPa σ3o  σo

7. Calculate the factor of safety using equations 5.12c, 5.12d, and 5.12e.
Inner radius

 S uc  2  S ut 
  σ1i  σ2i
1
C1i    σ1i  σ2i  C1i  92.46  MPa
2  S uc 

 S uc  2  S ut 
  σ2i  σ3i
1
C2i    σ2i  σ3i  C2i  0.00 MPa
2  S uc 

 S uc  2  S ut 
  σ3i  σ1i
1
C3i    σ3i  σ1i  C3i  92.46  MPa
2  S uc 

σeff  max C1i C2i C3i σ1i σ2i σ3i σeff  132.182  MPa

S ut
Ni  Ni  2.2
σeff

Outer radius

 S uc  2  S ut 
  σ1o  σ2o
1
C1o    σ1o  σ2o  C1o  0.00 MPa
2  S uc 

 S uc  2  S ut 
  σ2o  σ3o
1
C2o    σ2o  σ3o  C2o  61.41  MPa
2  S uc 

 S uc  2  S ut 
  σ3o  σ1o
1
C3o    σ3o  σ1o  C3o  61.41  MPa
2  S uc 

σeff  max C1o C2o C3o σ1o σ2o σ3o σeff  61.41  MPa

S ut
No  No  4.7
σeff

© 2011 Pearson Education, Inc., Upper Saddle River, NJ. All rights reserved. This publication is protected by Copyright and written permission should be
MACHINE DESIGN - An Integrated Approach, 4th Ed. 5-70-1

PROBLEM 5-70
Statement: A C-clamp as shown in Figure P5-24a has a rectangular cross section as in Figure P5-24c. Find the
static factor of safety if the clamping force is 1.6 kN and the material is class 50 gray cast iron.
Given: Clamping force F  1.6 kN
Distance from center of screw to throat ri  63.5 mm
Section dimensions: Width b  6.4 mm Depth h  31.8 mm
Material properties S ut  359  MPa S uc  1131 MPa

Solution: See Figure P5-24 and Mathcad file P0570.

1. Determine the distance from the centerline of the screw to the centroid of the section at the throat.
h
rc  ri  rc  79.4 mm
2

2. Using equation 4.12a and Figure 4-16, calculate the distance to the neutral axis, rn, and the distance from the
centroidal axis to the neutral axis, e.

Distance from the screw centerline to the outside fiber ro  ri  h ro  95.300 mm


2
Cross section area A  b  h A  203.520 mm

A
Distance to neutral axis rn  rn  78.327 mm
ro
 b
 dr
 r
r
i

Distance from centroidal to neutral axis e  rc  rn e  1.073 mm

3. Take a section through the throat area and draw a FBD. There will be a vertical axial force through the section
CG (at a distance rc from the screw centerline) which will form a couple of magnitude rc x F. This couple will be
balanced by an internal moment of equal magnitude.
Internal moment M  rc F M  127 N  m

4. Calculate the distances from the neutral axis to the inner and outer fibers.
ci  rn  ri ci  14.827 mm co  ro  rn co  16.973 mm

5. Using equations 4.12d and 4.12e, calculate the stresses at the inner and outer fibers of the throat section.

M  ci  F
σi    σi  143.7 MPa
e A  ri  A

M  co  F
σo     σo  95.8 MPa
e A  ro  A

6. These are the principal stresses so,

Inner radius σ1i  σi σ2i  0  MPa σ3i  0  MPa

Outer radius σ1o  0  MPa σ2o  0  MPa σ3o  σo


© 2011 Pearson Education, Inc., Upper Saddle River, NJ. All rights reserved. This publication is protected by Copyright and written permission should be
MACHINE DESIGN - An Integrated Approach, 4th Ed. 5-70-2

7. Calculate the factor of safety using equations 5.12c, 5.12d, and 5.12e.
Inner radius

 S uc  2  S ut 
  σ1i  σ2i
1
C1i    σ1i  σ2i  C1i  98.09 MPa
2  S uc 

 S uc  2  S ut 
  σ2i  σ3i
1
C2i    σ2i  σ3i  C2i  0.00 MPa
2  S uc 

 S uc  2  S ut 
  σ3i  σ1i
1
C3i    σ3i  σ1i  C3i  98.09 MPa
2  S uc 

σeff  max C1i C2i C3i σ1i σ2i σ3i σeff  143.707 MPa

S ut
Ni  Ni  2.5
σeff

Outer radius

 S uc  2  S ut 
  σ1o  σ2o
1
C1o    σ1o  σ2o  C1o  0.00 MPa
2  S uc 

 S uc  2  S ut 
  σ2o  σ3o
1
C2o    σ2o  σ3o  C2o  30.39 MPa
2  S uc 

 S uc  2  S ut 
  σ3o  σ1o
1
C3o    σ3o  σ1o  C3o  30.39 MPa
2  S uc 
σeff  max C1o C2o C3o σ1o σ2o σ3o σeff  30.394 MPa

S ut
No  No  11.8
σeff

© 2011 Pearson Education, Inc., Upper Saddle River, NJ. All rights reserved. This publication is protected by Copyright and written permission should be
MACHINE DESIGN - An Integrated Approach, 4th Ed. 5-71-1

PROBLEM 5-71
Statement: A C-clamp as shown in Figure P5-24a has an elliptical cross section as in Figure P5-24d.
Dimensions of the major and minor axes of the ellipse are given. Find the static factor of safety if
the clamping force is 1.6 kN and the material is class 60 gray cast iron.

Given: Clamping force F  1.6 kN


Distance from center of screw to throat ri  63.5 mm
Section dimensions: Width b  9.6 mm Depth h  31.8 mm
Material properties S ut  427  MPa S uc  1289 MPa

Solution: See Figure P5-24 and Mathcad file P0571.

1. Determine the distance from the centerline of the screw to the centroid of the section at the throat.
h
rc  ri  rc  79.4 mm
2

2. Using equation 4.12a and Figure 4-16, calculate the distance to the neutral axis, rn, and the distance from the
centroidal axis to the neutral axis, e.

Distance from the screw centerline to the outside fiber ro  ri  h ro  95.300 mm


b h 2
Cross section area A  π  A  239.766 mm
2 2

A
Distance to neutral axis rn  rn  78.595 mm
ro
 0.5
  2
 2 b
 1   r  rc 
 4 2 
  h  dr
 r
r
i
Distance from centroidal to neutral axis e  rc  rn e  0.805 mm

3. Take a section through the throat area and draw a FBD. There will be a vertical axial force through the section
CG (at a distance rc from the screw centerline) which will form a couple of magnitude rc x F. This couple will be
balanced by an internal moment of equal magnitude.
Internal moment M  rc F M  127 N  m

4. Calculate the distances from the neutral axis to the inner and outer fibers.
ci  rn  ri ci  15.095 mm co  ro  rn co  16.705 mm

5. Using equations 4.12d and 4.12e, calculate the stresses at the inner and outer fibers of the throat section.

M  ci  F
σi    σi  163.2 MPa
e A  ri  A

M  co  F
σo     σo  108.7 MPa
e A  ro  A

6. These are the principal stresses so,


© 2011 Pearson Education, Inc., Upper Saddle River, NJ. All rights reserved. This publication is protected by Copyright and written permission should be
MACHINE DESIGN - An Integrated Approach, 4th Ed. 5-71-2

Inner radius σ1i  σi σ2i  0  MPa σ3i  0  MPa

Outer radius σ1o  0  MPa σ2o  0  MPa σ3o  σo

7. Calculate the factor of safety using equations 5.12c, 5.12d, and 5.12e.
Inner radius

 S uc  2  S ut 
  σ1i  σ2i
1
C1i    σ1i  σ2i  C1i  109.12 MPa
2  S uc 

 S uc  2  S ut 
  σ2i  σ3i
1
C2i    σ2i  σ3i  C2i  0.00 MPa
2  S uc 

 S uc  2  S ut 
  σ3i  σ1i
1
C3i    σ3i  σ1i  C3i  109.12 MPa
2  S uc 

σeff  max C1i C2i C3i σ1i σ2i σ3i σeff  163.169 MPa

S ut
Ni  Ni  2.6
σeff

Outer radius

 S uc  2  S ut 
  σ1o  σ2o
1
C1o    σ1o  σ2o  C1o  0.00 MPa
2  S uc 

 S uc  2  S ut 
  σ2o  σ3o
1
C2o    σ2o  σ3o  C2o  36.02 MPa
2  S uc 

 S uc  2  S ut 
  σ3o  σ1o
1
C3o    σ3o  σ1o  C3o  36.02 MPa
2  S uc 

σeff  max C1o C2o C3o σ1o σ2o σ3o σeff  36.016 MPa

S ut
No  No  11.9
σeff

© 2011 Pearson Education, Inc., Upper Saddle River, NJ. All rights reserved. This publication is protected by Copyright and written permission should be
MACHINE DESIGN - An Integrated Approach, 4th Ed. 5-72-1

PROBLEM 5-72
Statement: A C-clamp as shown in Figure P5-24a has a trapezoidal cross section as in Figure P5-24e. Find the
static factor of safety if the clamping force is 350 lb and the material is class 40 gray cast iron.
Given: Clamping force F  1.6 kN
Distance from center of screw to throat ri  63.5 mm
Section dimensions: Width b i  9.6 mm b o  3.2 mm Depth h  31.8 mm
Material properties S ut  290  MPa S uc  965  MPa

Solution: See Figure P5-24 and Mathcad file P0572.

1. Determine the distance from the centerline of the screw to the centroid of the section at the throat.
h bi  2 bo
rc  ri   rc  76.75 mm
3 bi  bo

2. Using equation 4.12a and Figure 4-16, calculate the distance to the neutral axis, rn, and the distance from the
centroidal axis to the neutral axis, e.

Distance from the screw centerline to the outside fiber ro  ri  h ro  95.300 mm


bi  bo 2
Cross section area A  h A  203.520 mm
2

A
Distance to neutral axis rn  rn  75.771 mm
ro

 bi  bo
 bi    r  ri
h
 dr
 r
r
i
Distance from centroidal to neutral axis e  rc  rn e  0.979 mm

3. Take a section through the throat area and draw a FBD. There will be a vertical axial force through the section
CG (at a distance rc from the screw centerline) which will form a couple of magnitude rc x F. This couple will be
balanced by an internal moment of equal magnitude.
Internal moment M  rc F M  123 N  m

4. Calculate the distances from the neutral axis to the inner and outer fibers.
ci  rn  ri ci  12.271 mm co  ro  rn co  19.529 mm

5. Using equations 4.12d and 4.12e, calculate the stresses at the inner and outer fibers of the throat section.

M  ci  F
σi    σi  126.9 MPa
e A  ri  A

M  co  F
σo     σo  118.4 MPa
e A  ro  A

6. These are the principal stresses so,

Inner radius σ1i  σi σ2i  0  MPa σ3i  0  MPa

Outer radius σ1o  0  MPa σ2o  0  MPa σ3o  σo


© 2011 Pearson Education, Inc., Upper Saddle River, NJ. All rights reserved. This publication is protected by Copyright and written permission should be
MACHINE DESIGN - An Integrated Approach, 4th Ed. 5-72-2

7. Calculate the factor of safety using equations 5.12c, 5.12d, and 5.12e.
Inner radius

 S uc  2  S ut 
  σ1i  σ2i
1
C1i    σ1i  σ2i  C1i  88.77 MPa
2  S uc 

 S uc  2  S ut 
  σ2i  σ3i
1
C2i    σ2i  σ3i  C2i  0.00 MPa
2  S uc 

 S uc  2  S ut 
  σ3i  σ1i
1
C3i    σ3i  σ1i  C3i  88.77 MPa
2  S uc 

σeff  max C1i C2i C3i σ1i σ2i σ3i σeff  126.907 MPa

S ut
Ni  Ni  2.3
σeff

Outer radius

 S uc  2  S ut 
  σ1o  σ2o
1
C1o    σ1o  σ2o  C1o  0.00 MPa
2  S uc 

 S uc  2  S ut 
  σ2o  σ3o
1
C2o    σ2o  σ3o  C2o  35.58 MPa
2  S uc 

 S uc  2  S ut 
  σ3o  σ1o
1
C3o    σ3o  σ1o  C3o  35.58 MPa
2  S uc 
σeff  max C1o C2o C3o σ1o σ2o σ3o σeff  35.577 MPa

S ut
No  No  8.2
σeff

© 2011 Pearson Education, Inc., Upper Saddle River, NJ. All rights reserved. This publication is protected by Copyright and written permission should be
MACHINE DESIGN - An Integrated Approach, 4th Ed. 5-73-1

PROBLEM 5-73
Statement: The connecting rod (3) on the oil-field pump jack shown in Figure P5-23 is, in fact, made up of
two rods, one connecting on each side of the walking beam (4). Determine a suitable width of
1/2-inch-thick SAE 1020 cold-rolled bar stock to use if the maximum tensile load on the bars is
3500 lb each. Use a factor of safety of 4 against static yielding.

Given: Yield strength S y  57 ksi Factor of safty Ns  4

Thickness t  0.50 in Tensile force F  3500 lbf

Solution: See Figure P5-23 and Mathcad file P0573.

1. Express the tensile stress as a functions of the unknown width, w.

F
Tensile stress σx( w) 
w t

2. The tensile stress is the only stress present so it is also the von Mises effective stress.

F
von Mises effective stress σ'( w) 
w t

3. Use equation 5.8a as a design relationship to solve for the diameter, d.

F Sy
Design equation =
w t Ns

N s F
Solving for w w  w  0.491 in
t Sy

A suitable size for the given design requirements is w  0.500  in

© 2011 Pearson Education, Inc., Upper Saddle River, NJ. All rights reserved. This publication is protected by Copyright and written permission should be
MACHINE DESIGN - An Integrated Approach, 4th Ed. 5-74-1

PROBLEM 5-74
Statement: A work platform is elevated on the end of a boom that has the ability to extend its length and
vary its angle with respect to ground. The platform width is large compared to the boom
diameter so that it is possible to load the boom eccentrically resulting in a combination of
bending, torsion and direct compression in the base of the boom. At the base the boom is a
hollow tube with an outside diameter of 8 in and a wall thickness of 0.75 in. It is made from SAE
1030 CR steel. Determine the factor of safety against static failure if the loading at a point at the
base of the boom is: M = 600 kip-in, T = 76 kip-in, and an axial compression of 4800 lb.

Given: Yield strength SAE 1030 CR steel S y  64 ksi


Boom dimensions D  8.00 in twall  0.75 in
Loading M  600  kip in T  76 kip in F  4800 lbf

Solution: See Mathcad file P0574.

1. Calculate the bending stress at the point of interest.


Inside diameter d  D  2  twall d  6.500 in

Moment of inertia I 
π
64
  D d
4 
4
I  113.438 in
4

Distance to outer fiber c  0.5 D c  4.000 in

M c
Bending stress σbend  σbend  21.157 ksi
I

2. Calculate the axial stress due to the compressive load at the point of interest.

Cross-section area A 
π
4
  D d
2 
2
A  17.082 in
2

F
Axial stress σaxial  σaxial  0.281 ksi
A

3. Combine the bending and axial stresses to get the maximum normal stress on the compressive side of the boom.

Max. normal stress σx  σbend  σaxial σx  21.438 ksi

4. Calculate the torsional stress at the point of interest.


4
Polar moment J  2  I J  226.876 in
Tc
Torsional stress τxy  τxy  1.34 ksi
J
5. Calculate the von Mises effective stress using equation 5.7d.
2 2
von Mises stress σ'  σx  3  τxy σ'  21.563 ksi

6. Calculate the factor of safety using equation 5.8a.


Sy
Factor of safety N  N  2.97
σ'

© 2011 Pearson Education, Inc., Upper Saddle River, NJ. All rights reserved. This publication is protected by Copyright and written permission should be
MACHINE DESIGN - An Integrated Approach, 4th Ed. 5-75-1

PROBLEM 5-75
Statement: Repeat Problem 5-74 for a boom that is made from class 20 gray cast iron. At the base the boom is
hollow tube with an outside diameter of 10 in and a wall thickness of 1.00 in.
Given: Strength Class 20 gray cast iron S ut  22 ksi S uc  83 ksi
Boom dimensions D  10.00  in twall  1.00 in
Loading M  600  kip in T  76 kip in F  4800 lbf

Solution: See Mathcad file P0575.

1. Calculate the bending stress at the point of interest.


Inside diameter d  D  2  twall d  8.000 in

Moment of inertia I 
π
64
  D d
4 4 I  289.812 in
4

Distance to outer fiber c  0.5 D c  5.000 in

M c
Bending stress σbend  σbend  10.352 ksi
I

2. Calculate the axial stress due to the compressive load at the point of interest.

Cross-section area A 
π
4
  D d
2 2 A  28.274 in
2

F
Axial stress σaxial  σaxial  0.17 ksi
A

3. Combine the bending and axial stresses to get the maximum normal stress on the tensile and compressive sides
of the boom.
Max compressive σxc  σbend  σaxial σxc  10.521 ksi
Max tensile σxt  σbend  σaxial σxt  10.182 ksi

4. Calculate the torsional stress at the point of interest.


4
Polar moment J  2  I J  579.624 in
Tc
Torsional stress τxy  τxy  0.656 ksi
J
5. Calculate the principal stresses on the tensile and compressive sides of the boom.

2
 σxc  2
Compressive side τmaxc     τxy τmaxc  5.301 ksi
 2 
σxc
σ1c   τmaxc σ1c  0.041 ksi
2
σ2c  0  ksi

σxc
σ3c   τmaxc σ3c  10.562 ksi
2

© 2011 Pearson Education, Inc., Upper Saddle River, NJ. All rights reserved. This publication is protected by Copyright and written permission should be
MACHINE DESIGN - An Integrated Approach, 4th Ed. 5-75-2

2
 σxt  2
Tensile side τmaxt     τxy τmaxt  5.133 ksi
 2

σxt
σ1t   τmaxc σ1t  10.392 ksi
2
σ2t  0  ksi

σxt
σ3t   τmaxc σ3t  0.210 ksi
2

6. Calculate the factor of safety using equations 5.12c, 5.12d, and 5.12e.
Compressive side

 S uc  2  S ut 
  σ1c  σ2c
1
C1c    σ1c  σ2c  C1c  0.03 ksi
2  S uc 

 S uc  2  S ut 
  σ2c  σ3c
1
C2c    σ2c  σ3c  C2c  2.80 ksi
2  S uc 

 S uc  2  S ut 
  σ3c  σ1c
1
C3c    σ3c  σ1c  C3c  2.83 ksi
2  S uc 

σeff  max C1c C2c C3c σ1c σ2c σ3c σeff  2.829 ksi

S ut
Nc  Nc  7.8
σeff

Tensile side

 S uc  2  S ut 
  σ1t  σ2t
1
C1t    σ1t  σ2t  C1t  7.64 ksi
2  S uc 

 S uc  2  S ut 
  σ2t  σ3t
1
C2t    σ2t  σ3t  C2t  0.06 ksi
2  S uc 

 S uc  2  S ut 
  σ3t  σ1t
1
C3t    σ3t  σ1t  C3t  7.69 ksi
2  S uc 

σeff  max C1t C2t C3t σ1t σ2t σ3t σeff  10.392 ksi

S ut
Nt  Nt  2.1
σeff

© 2011 Pearson Education, Inc., Upper Saddle River, NJ. All rights reserved. This publication is protected by Copyright and written permission should be
MACHINE DESIGN - An Integrated Approach, 4th Ed. 5-76-1
PROBLEM 5-76
Statement: Assume that the curved beam of Problem 5-70 has a crack on its inside surface of half-width a =
1.5 mm and a fracture toughness of 35 MPa-m0.5. What is its safety factor against sudden
fracture?
Given: Width of section t  31.8 mm
Half crack length a  1.5 mm Fracture toughness Kc  35 MPa m

Solution: See Figure 5-38 and Mathcad file P0538.

1. From Problem 5-70, the nominal stress at the inside radius is:
Nominal inside stress σi  143.7  MPa

2. Calculate the half-width of the beam. b  0.5 t b  15.9 mm


3. Calculate the geometry and stress intensity factors.

β  sec 
π a 
 β  1.006
 2 b 
K  β  σi π a K  9.92 MPa m

Kc
4. Determine the factor of safety against sudden fracture failure NFM  NFM  3.5
K

© 2011 Pearson Education, Inc., Upper Saddle River, NJ. All rights reserved. This publication is protected by Copyright and written permission should be
MACHINE DESIGN - An Integrated Approach, 4th Ed. 5-77-1

PROBLEM 5-77 _____


Statement: A large aircraft panel is to be made from 7075-T651 aluminum bar. From test data it is found that
the nominal tensile stress in the panel is 200 MPa. What is the average maximum central crack
size that can be tolerated without catastrophic failure?

0.5 0.5
Given: Fracture toughness Kc  22 ksi in Kc  24.2 MPa m
Nominal stress σnom  200  MPa

Solution: See Mathcad file P0577.

1. Determine the value of the geometry factor  from the discussion in Section 5.3 for a large plate with a central
crack.

β  1

2. Using equation 5.14b, calculate the critical crack length for this material under the given stress condition.

2
 Kc 
1
a     a  4.65 mm
π  β  σnom 

lcritical  2  a lcritical  9.3 mm

© 2011 Pearson Education, Inc., Upper Saddle River, NJ. All rights reserved. This publication is protected by Copyright and written permission should be
MACHINE DESIGN - An Integrated Approach, 4th Ed. 5-78-1
PROBLEM 5-78
Statement: Design the connecting rod (link 3) of Problem 3-50 for a safety factor of 4 if the link is made from
SAE 1010 hot-rolled steel sheet, the pin hole diameter at each end is 6 mm, and the maximum
applied tensile load is 2000 N. There are two links carrying the load.

Given: Force on links Ftotal  2000 N


Yield strength S y  179  MPa
Design safety factor Nd  4
Pin hole diameter d  6  mm

Assumptions: Choose a suitable width, say w  3  d w  18 mm

Solution: See Figure P3-22 and Mathcad file P0577.

Ftotal
1. The force on each link is F  F  1000 N
2

2. With only a tensile force acting on the link, the tensile stress will be the principal stress and it will also be the
von Mises effective stress, so we have σx = 1 = '.

F F
3. The tensile stress on each link is σx = = = σ'
A t w

Sy t w Sy
4. Using the distortion-energy failure theory, Nd = =
σ' F

F  Nd
5. Solving for the thickness,t, t  t  1.241  mm
w S y

6. Round this up to the next higher integer value, t  2  mm

t  w S y
7. The realized factor of safety N  N  6.4
against tensile failure is, F

8. Check the factor of safety against bearing failure in the pin holes.
2
Bearing area Abear  w t Abear  36.0 mm

F
σbear  σbear  27.8 MPa
Abear

This is the principal stresses 1.

9. Calculate the safety factor for direct bearing from equation 5.8c where 2 and 3 are both zero.

Sy
Pin hole Nbear  Nbear  6.4
σbear

2 2
10. The tearout area is Atear = 2  t R  ( 0.5 d ) , where R  0.5 w (see figure below). Substitute this area in
equation 4.9 for the shear area and solve for the shear strength xy.

© 2011 Pearson Education, Inc., Upper Saddle River, NJ. All rights reserved. This publication is protected by Copyright and written permission should be
MACHINE DESIGN - An Integrated Approach, 4th Ed. 5-78-2

Tearout length
2 2
Shear area Atear  2  t R  ( 0.5 d )
2
Atear  33.941 mm

F
Shear stress τxy 
Atear

τxy  29.46  MPa


d R

8. From equations 5.8c and 5.9b, the safety factor against tearout failure in the holes is

0.577  S y
Ntear  Ntear  3.5
τxy

This is slightly less than the design FS of Nd  4 so, choose t = 2.5 mm or increase w to 4*d.

© 2011 Pearson Education, Inc., Upper Saddle River, NJ. All rights reserved. This publication is protected by Copyright and written permission should be
MACHINE DESIGN - An Integrated Approach, 4th Ed. 5-79-1

PROBLEM 5-79
Statement: Design the compacting ram (link 4) of Problem 3-50 for a safety factor of 4 if the ram is made
from SAE 1010 hot-rolled steel bar, the pin hole diameter at the joint where link 3 attaches is 6
mm, and the applied load Fcom = 2000 N. The piston has a diameter of 35 mm.

Given: Force at point P Fcom  2000 N


Yield strength S y  179  MPa
Design safety factor Nd  4
Pin hole diameter d  6  mm

Assumptions: The points of maximum stress on a plane through point D are sufficiently removed from point D
that there is no stress concentration at those points.

Solution: See Figure P3-22 and Mathcad file P0579.

1. From Problem 3-51 the forces and reactions on the ram


are:
42.5
F34x  553  N F34y  2000 N E
F14E
F14E  357  N F14F  196  N D
F34x
2. The maximum bending moment is at point D and is: 120.0
F34y 77.5
M  42.5 mm F14E M  15172.5 N  mm
The section modulus and area for the ram are F14F
F
3 2
π D π D
Z ( D)  A ( D) 
32 4 P
3. Between points D and P there is a compressive force of
Fcom  2000 N. Thus, there is a compressive stress due Fcom
to this force in addition to the bending stress at point D.
Compacting Ram (4)
On the left side of the ram at the section through point D

M Fcom
σbL ( D)  σa( D)  σL ( D)  σbL ( D)  σa( D)
Z ( D) A ( D)

On the right side of the ram at the section through point D

M 4  Fcom
σbR( D)   σa( D)  σR( D)  σbR( D)  σa( D)
Z ( D) 2
π D
The compressive stress on the right side will be numerically greater than that on the left side.

4. Since the shear stress due to bending is zero at these points, the axial stress will be the principal stress and it
will also be the von Mises effective stress, so we have σx = 1 = '.

Sy Sy
5. Using the distortion-energy failure theory, Nd = =
σ' σR( D)

6. Solving for the diameter, D,


© 2011 Pearson Education, Inc., Upper Saddle River, NJ. All rights reserved. This publication is protected by Copyright and written permission should be
MACHINE DESIGN - An Integrated Approach, 4th Ed. 5-79-2

Guess D  10 mm

f ( D)  Nd  σR( D)  S y D  root( f ( D) D) D  16.368 mm

7. Round this up to the next higher even integer value, say D  18 mm

Sy
8. The realized factor of safety N  N  5.2
against axial yeilding is, σR( D)

9. The axial stress on each side of the ram on a section through D is:

σL ( D)  18.6 MPa σR( D)  34.4 MPa

© 2011 Pearson Education, Inc., Upper Saddle River, NJ. All rights reserved. This publication is protected by Copyright and written permission should be
MACHINE DESIGN - An Integrated Approach, 4th Ed. 5-80-1
PROBLEM 5-80
Statement: A differential element is subected to the stresses given below and a ductile material has the
strengths given below. Calculate the safety factor and draw 1-3 diagrams of each theory
showing the stress state using:
(a) Maximum shear-stress theory, and
(b) Distortion-energy theory.

Given: Principal stresses σ1  70 MPa σ2  0  ksi σ3  140  MPa


Material properties S ut  350  MPa S y  280  MPa S uc  350  MPa

Solution: See Figure 5-80 and Mathcad file P0580.


1. Calculate the slope of load line. (The load line is the line from the origin through the stress point.)
σ3
m  m  2
σ1

2. The safety factor equation for the distortion-enrgy theory is the same regardless of which quadrant the load line
falls in. However, the equation for the maximum shear-stress factor of safety is different for each of the three
quadrants that the load line (1st, 3rd, or 4th) can fall in. In this case, the load line falls in the 4th quadrant. The
factors of safety are:

Sy
(a) Maximum shear-stress theory Na  Na  1.3
σ1  σ3

(b) Distortion energy theory 3


280
(a) Maximum shear
2 2 stress boundary
σ'  σ1  σ1 σ3  σ3 210
(b) Distortion energy
σ'  26.9 ksi 140
boundary
MINIMUM NONZERO PRINCIPAL STRESS, MPa

Sy
Nb  Nb  1.5 70
σ'
0 1
3. Plot the 1-3 diagram showing the sy

safe-fail boundaries, the stress state point -70


(70MPa, -140 MPa) and the load line.
Note that if 1 > 3 , then only that area -140
(70,-140)

on the graph that is to the right of and


below the diagonal line can contain valid -210 Stress states at
stress points. The factor of safety is the which failure
distance along the load line from the will occur
-280 -s y
origin to the intersection of the load line Load Line
with the failure boundary, divided by the
-350
distance from the origin to the stress
point. Since the distance from the origin
-420
to the distortion-energy boundary is
-280 -210 -140 -70 0 70 140 210 280 350
greater than the distance to the maximum
MAXIMUM PRINCIPAL STRESS, MPa
shear-stress baoundary, its factor of
safety is greater.
FIGURE 5-80
1 -  3 Diagram for Problem 5-80

© 2011 Pearson Education, Inc., Upper Saddle River, NJ. All rights reserved. This publication is protected by Copyright and written permission should be
MACHINE DESIGN - An Integrated Approach, 4th Ed. 5-81-1
PROBLEM 5-81
Statement: A part has the combined stress state and strengths given below. Using the Distortion-Energy
failure theory, find the von Mises effective stress and factor of safety against static failure.
Given: Stresses: σx  70 MPa σy  35 MPa τxy  31.5 MPa
Strengths: S y  126  MPa S ut  140  MPa S uc  140  MPa

Solution: See Mathcad file P0581.


1. Find the maximum shear stress and principal stresses that result from this combination of applied stresses
using equations 4.6.

2
 σx  σy  2
Maximum shear stress τmax     τxy τmax  36.0 MPa
 2 
σx  σy
Principal stresses σ1   τmax σ1  88.5 MPa
2
σx  σy
σ2   τmax σ2  16.5 MPa
2

σ3  0  psi

2. Find the von Mises effective stress using equation 5.7d:

2 2 2
σ'  σx  σx σy  σy  3  τxy σ'  81.6 MPa

Sy
3. The safety factor can now be found using equation 5.8a. N  N  1.5
σ'

© 2011 Pearson Education, Inc., Upper Saddle River, NJ. All rights reserved. This publication is protected by Copyright and written permission should be
MACHINE DESIGN - An Integrated Approach, 4th Ed. 5-82-1
PROBLEM 5-82
Statement: Repeat Problem 5-78 for the connecting rod made from class 20 cast iron.

Given: Force on links Ftotal  2000 N


Strength S ut  152  MPa S uc  572  MPa
Design safety factor Nd  4
Pin hole diameter d  6  mm
Assumptions: Choose a suitable width, say w  4  d w  24 mm

Solution: See Figure P3-22 and Mathcad file P0582.

Ftotal
1. The force on each link is F  F  1000 N
2

2. With only a tensile force acting on the link, the tensile stress will be the principal stress so we have σx = 1.

F F
3. The tensile stress on each link is σx = = = σ1
A t w

S ut t  w S ut
4. Using the modified-Mohr failure theory, Nd = =
σ1 F

F  Nd
5. Solving for the thickness,t, t  t  1.096  mm
w S ut

6. Round this up to the next higher integer value, t  2  mm

t  w S ut
7. The realized factor of safety N  N  7.3
against tensile failure is, F

8. Check the factor of safety against bearing failure in the pin holes.
2
Bearing area Abear  w t Abear  48.0 mm

F
σbear  σbear  20.8 MPa
Abear

This is the principal stresses 1.

9. Calculate the safety factor for direct bearing from equation 5.8c where 2 and 3 are both zero.

S uc
Pin hole Nbear  Nbear  27.5
σbear

2 2
10. The tearout area is Atear = 2  t R  ( 0.5 d ) , where R  0.5 w (see figure below). Substitute this area in
equation 4.9 for the shear area and solve for the shear strength xy.

© 2011 Pearson Education, Inc., Upper Saddle River, NJ. All rights reserved. This publication is protected by Copyright and written permission should be
MACHINE DESIGN - An Integrated Approach, 4th Ed. 5-82-2

Tearout length
2 2
Shear area Atear  2  t R  ( 0.5 d )
2
Atear  46.476 mm

F
Shear stress τxy 
Atear

τxy  21.52  MPa


d R
Principal stress σ1  τxy

8. For pure shear the Mohr circle is centered at 0,0 and has a radius equal to the shear stress. This results in
1 = . Using the Modified-Mohr failure theory and Figure 5-11, we see that we can use equation 5.12a for
the safety factor against tearout.

S ut
Ntear  Ntear  7.1
σ1

© 2011 Pearson Education, Inc., Upper Saddle River, NJ. All rights reserved. This publication is protected by Copyright and written permission should be
MACHINE DESIGN - An Integrated Approach, 4th Ed. 5-83-1

PROBLEM 5-83
Statement: Repeat Problem 5-79 for the part made from class 20 cast iron.

Given: Force at point P Fcom  2000 N


Tensile strength S ut  152  MPa
Design safety factor Nd  4
Pin hole diameter d  6  mm

Assumptions: The points of maximum stress on a plane through point D are sufficiently removed from point D
that there is no stress concentration at those points.

Solution: See Figure P3-22 and Mathcad files P0579 and P0583.

1. From Problem 3-51 the forces and reactions on the ram


are:
42.5
F34x  553  N F34y  2000 N E
F14E
F14E  357  N F14F  196  N D
F34x
2. The maximum bending moment is at point D and is: 120.0
F34y 77.5
M  42.5 mm F14E M  15172.5 N  mm
The section modulus and area for the ram are F14F
F
3 2
π D π D
Z ( D)  A ( D) 
32 4 P
3. Between points D and P there is a compressive force of
Fcom  2000 N. Thus, there is a compressive stress due Fcom
to this force in addition to the bending stress at point D.
Compacting Ram (4)
On the left side of the ram at the section through point D

M Fcom
σbL ( D)  σa( D)  σL ( D)  σbL ( D)  σa( D)
Z ( D) A ( D)

On the right side of the ram at the section through point D

M 4  Fcom
σbR( D)   σa( D)  σR( D)  σbR( D)  σa( D)
Z ( D) 2
π D
The tensile stress on the left side will be critical for an uneven, brittle material.
4. With only a tensile stress acting on the ram at this point, it will be the principal stress so we have σL = 1.

S ut S ut
5. Using the modified-Mohr failure theory, Nd = =
σ1 σL

6. Solving for the diameter, D,


Guess D  10 mm

f ( D)  Nd  σL ( D)  S ut D  root( f ( D) D) D  14.567 mm

© 2011 Pearson Education, Inc., Upper Saddle River, NJ. All rights reserved. This publication is protected by Copyright and written permission should be
MACHINE DESIGN - An Integrated Approach, 4th Ed. 5-83-2

7. Round this up so that D  3  d D  18 mm

S ut
8. The realized factor of safety N  N  8.2
against axial failure is, σL ( D)

9. The axial stress on each side of the ram on a section through D is:

σL ( D)  18.6 MPa σR( D)  34.4 MPa

© 2011 Pearson Education, Inc., Upper Saddle River, NJ. All rights reserved. This publication is protected by Copyright and written permission should be
MACHINE DESIGN - An Integrated Approach, 4th Ed. 5-84-1
PROBLEM 5-84
Statement: A differential element is subected to the stresses and strengths given below. Calculate the
safety factor and draw 1-3 diagrams of each theory showing the stress state using:
(a) Coulomb-Mohr theory, and
(b) Modified Mohr theory.

Given: Principal stresses σ1  70 MPa σ2  0  MPa σ3  140  MPa


Material properties S ut  350  MPa S uc  630  MPa

Solution: See Figure 5-84 and Mathcad file P05384.


1. Calculate the slope of load line. (The load line is the line from the origin through the stress point.)
σ3
m  m  2
σ1

2. The safety factor equation for both theories is different for each quadrant the load line falls in. The equation for
the modified Mohr factor of safety is different for each of the two regions in the 4th quadrant that the load line can
fall in. In this case, the load line falls in the 4th quadrant, below the -1 slope line.. The factors of safety are:

S uc S ut
(a)Coulomb-Mohr theory Na  Na  2.4
S uc σ1  S ut σ3

(b) Modified Mohr theory 3


350

280
S uc
Nb  210
 S uc  S ut 
 S   σ1  σ3 140 (a) Coulomb-Mohr
 
MINIMUM NONZERO PRINCIPAL STRESS, MPa

ut boundary
70

0 1
Nb  3.2
-70
(70,-140)
-140

3. Plot the 1-3 diagram showing the -210

safe-fail boundaries, the stress state point -280


(70 MPa,-140 MPa) and the load line. Note Stress states at -S
-350
that if 1 > 3 , then only that area on the which failure ut
-420 will occur
graph that is to the right of and below the
-490 (b) Modified Mohr
diagonal line can contain valid stress boundary
Load Line
points. The factor of safety is the distance -560
along the load line from the origin to the -630 -S
uc
intersection of the load line with the failure
-700
boundary, divided by the distance from -700 -630 -560 -490 -420 -350 -280 -210 -140 -70 0 70 140 210 280 350
the origin to the stress point. Since the MAXIMUM PRINCIPAL STRESS, MPa
distance from the origin to the modified
Mohr boundary is greater than the
distance to the Coulomb-Mohr boundary, FIGURE 5-84
its factor of safety is greater. 1 -  3 Diagram for Problem 5-84

© 2011 Pearson Education, Inc., Upper Saddle River, NJ. All rights reserved. This publication is protected by Copyright and written permission should be
MACHINE DESIGN - An Integrated Approach, 4th Ed. 5-85-1
PROBLEM 5-85
Statement: A part has the combined stress state and strengths given below. Using the Modified-Mohr failure
theory, find the effective stress and factor of safety against static failure.
Given: Stresses: σx  70 MPa σy  35 MPa τxy  31.5 MPa
Strengths: S y  126  MPa S ut  140  MPa S uc  560  MPa

Solution: See Mathcad file P0585.


1. Because S uc is greater than S ut, this is an uneven material, which is characteristic of a brittle material.
2. Find the maximum shear stress and principal stresses that result from this combination of applied stresses using
equations 4.6.

2
 σx  σy  2
Maximum shear stress τmax     τxy τmax  36.0 MPa
 2 
σx  σy
Principal stresses σ1   τmax σ1  88.5 MPa
2
σx  σy
σ2   τmax σ2  16.5 MPa
2

σ3  0  psi

3. Find the Dowling factors C1, C2, C3 using equations 5.12b:

 S uc  2  S ut 
  σ1  σ2
1
C1    σ1  σ2  C1  62.3 MPa
2  S uc 
 S uc  2  S ut 
  σ2  σ3
1
C2    σ2  σ3  C2  12.3 MPa
2  S uc 
 S uc  2  S ut 
  σ3  σ1
1
C3    σ3  σ1  C3  66.4 MPa
2  S uc 
4. Then find the largest of the six stresses C1, C2, C3 , 1, 2, 3:

  1  
C
  C2  
 C 
σeff  max   
3
σeff  88.5 MPa
  σ1  
 
  σ2  
  σ3  

which is the modified-Mohr effective stress.


S ut
5. The safety factor can now be found using equation 5.12d. N  N  1.6
σeff

© 2011 Pearson Education, Inc., Upper Saddle River, NJ. All rights reserved. This publication is protected by Copyright and written permission should be
MACHINE DESIGN - An Integrated Approach, 4th Ed. 6-1a-1
PROBLEM 6-1a
Statement: For the data in row a in Table P6-1, find the stress range, alternating stress component, mean stress
component, stress ratio, and amplitude ratio.
Given: σmax  1000 σmin  0

Solution: See Mathcad file P0601a.

1. Use equations (6.1) to calculate the required quantities.

Stress range ∆σ  σmax  σmin ∆σ  1000

σmax  σmin
Alternating stress σa  σa  500
2

σmax  σmin
Mean stress σm  σm  500
2

σmin
Stress ratio R  R0
σmax

σa
Amplitude ratio A  A1
σm

© 2011 Pearson Education, Inc., Upper Saddle River, NJ. All rights reserved. This publication is protected by Copyright and written permission should be
MACHINE DESIGN - An Integrated Approach, 4th Ed. 6-2a-1
PROBLEM 6-2a
Statement: For the strength data in row a in Table P6-2, calculate the uncorrected endurance limit and draw th
strength-life (S-N) diagram for the material, assuming it to be steel.
Given: Tensile strength S ut  90 ksi
Solution: See Mathcad file P0602a.

1. Using equation (6-5a), calculate the uncorrected endurance limit.


S'e  return 0.5 S ut if S ut  200  ksi S'e  45.0 ksi
100  ksi otherwise

2. Using equation (6.9), calculate the fatigue strength at N = 10 3 cycles. S m  0.9 S ut S m  81.0 ksi

b
3. The equation for the S-N curve in the HCF region is given by equation (6.10a): S'f = a  N

4. Determine the constants a and b from equations (6.10c) and (6.10a). From Table 6-5, for N = 10 6 , z  3.000

1  Sm 
b   log  b  0.0851
z  S'e 
Sm
a  a  145.8 ksi
103
b

5. To draw the S-N graph over the range 10 3 <= N <= 10 8, define a piecewise continuous function.

b 6
S'f ( N )  return a  N if N  10
S'e otherwise

3 5 8
6. Plot the S-N curve over the range N  10 10  10

100

S' f ( N )
ksi

10
3 4 5 6 7 8
1 10 1 10 1 10 1 10 1 10 1 10
N
FIGURE 6-2a
S-N Diagram for Steel for Problem 6-2a

© 2011 Pearson Education, Inc., Upper Saddle River, NJ. All rights reserved. This publication is protected by Copyright and written permission should be
MACHINE DESIGN - An Integrated Approach,4th Ed. 6-3-1
PROBLEM 6-3
Statement: For the bicycle pedal-arm assembly in Figure P6-1 assume a rider-applied force that ranges from 0
1500 N at the pedal each cycle. Determine the fluctuating stresses in the 15-mm-dia pedal arm. Fi
the fatigue safety factor if S ut = 500 MPa.

Given: Material yield strength S y  350  MPa S ut  500  MPa


Applied load Fmax  1500 N Fmin  0  N
Pedal arm diameter d  15 mm

Solution: See Figures 6-3 and Mathcad file P0603.

1. From problem 4-3, the maximum principal stresses in the pedal arm due to Fmax are at point A and are

σ1max  793  MPa σ2max  0  MPa σ3max  23 MPa


2. Using equation 5.7c, the maximum von Mises stress is

2 2
σ'max  σ1max  σ1max σ3max  σ3max σ'max  804.7 MPa
3. The minimum von Mises stress is zero. σ'min  0  MPa

4. The alternating and mean components of the von Mises stress are:

σ'max  σ'min
σ'a  σ'a  402.4 MPa
2
σ'max  σ'min
σ'm  σ'm  402.4 MPa
2

5. Calculate the unmodified endurance limit. S'e  0.5 S ut S'e  250 MPa

z z

a Section C
C Tc A
Frider
Mc
b Arm
B
Arm

Fc y
Pedal
x
x
y

FIGURE 6-3A FIGURE 6-3B


Free Body Diagram for Problem 6-3 Points A and B at Section C

6. Calculate the endurance limit modification factors for a nonrotating round beam.

Load Cload  1
© 2011 Pearson Education, Inc., Upper Saddle River, NJ. All rights reserved. This publication is protected by Copyright and written permission should be
MACHINE DESIGN - An Integrated Approach,4th Ed. 6-3-2

2 2
Size A95  0.010462 d A95  2.354 mm

A95
d equiv  d equiv  5.544 mm
0.0766
 0.097
 d equiv 
Csize  1.189    Csize  1.007 Csize  1
 mm 

Surface A  4.51 b  0.265 (machined)

b
 Sut 
Csurf  A    Csurf  0.869
 MPa 
Temperature Ctemp  1

Reliability Creliab  0.753 (R = 99.9%)

7. Calculate the modified endurance limit.


S e  Cload  Csize Csurf  Ctemp Creliab S'e S e  163.56 MPa

8. Assuming a Case 3 load line, use equation (6.18e) to determine the factor of safety.

S e S ut
Nf  Nf  0.31
σ'a S ut  σ'm S e

© 2011 Pearson Education, Inc., Upper Saddle River, NJ. All rights reserved. This publication is protected by Copyright and written permission should be
MACHINE DESIGN - An Integrated Approach 6-4a-1
PROBLEM 6-4a
Statement: For the strength data in row a in Table P6-2, calculate the uncorrected fatigue strength at 5E8
cycles and draw the strength-life (S-N) diagram for the material, assuming it to be an aluminum
alloy.

Given: Tensile strength S ut  90 ksi

Solution: See Mathcad file P0604a.


1. Using equation (6-5c), calculate the uncorrected fatigue strength at 5E8 cycles.
S'f5E8  return 0.4 S ut if S ut  48 ksi S'f5E8  19.0 ksi
19 ksi otherwise

2. Using equation (6.9), calculate the fatigue strength at N = 10 3 cycles. S m  0.9 S ut S m  81.0 ksi

b
3. The equation for the S-N curve in the HCF region is given by equation (6.10a): S'f = a  N

4. Determine the constants a and b from equations (6.10c) and (6.10a). From Table 6-5, for N = 5E8 , z  5.699

1  Sm 
b   log  b  0.1105
z  S'f5E8 
Sm
a  a  173.772 ksi
103
b

b
5. To draw the S-N graph over the range 10 3 <= N <= 10 8, S'f ( N )  a  N

3 5 8
6. Plot the S-N curve over the range N  10 1.01 10  10

100

S' f ( N )
ksi

10
3 4 5 6 7 8
1 10 1 10 1 10 1 10 1 10 1 10
N
FIGURE 6-4a
S-N Diagram for Aluminum for Problem 6-4a

© 2011 Pearson Education, Inc., Upper Saddle River, NJ. All rights reserved. This publication is protected by Copyright and written permission should be
MACHINE DESIGN - An Integrated Approach, 4th Ed. 6-5a-1
PROBLEM 6-5a
Statement: For the data in row a in Table P6-3, find the corrected endurance strength (or limit), create
equations for the S-N line, and draw the S-N diagram.
Given: Material steel Surface finish surface  "ground"
Tensile strength S ut  110  ksi Loading load  "torsion"
Shape round Temperature T  72
Size (diameter) d  2  in Reliability R  0.999

Solution: See Mathcad file P0605a.


1. Using equation (6-5a), calculate the uncorrected endurance limit.
S'e  return 0.5 S ut if S ut  200  ksi S'e  55.0 ksi
100  ksi otherwise

2. Calculate the endurance limit modification factors for a nonrotating round rod.

Load Cload  return 1 if load = "bending" Cload  1


return 1 if load = "torsion"
return 0.7 if load = "axial"

Size d equiv  d

 0.097
 d equiv 
Csize  0.869    Csize  0.812
 in 
Surface A  return 1.34 if surface = "ground" A  1.34
return 2.70 if surface = "machined"
return 2.70 if surface = "cold_rolled"
return 14.4 if surface = "hot_rolled"
return 39.9 if surface = "forged"

b  return 0.085 if surface = "ground" b  0.085


return 0.265 if surface = "machined"
return 0.265 if surface = "cold_rolled"
return 0.718 if surface = "hot_rolled"
return 0.995 if surface = "forged"

b
 S ut 
Csurf  A    Csurf  0.899
 ksi 
Temperature Ctemp  return 1 if T  840 Ctemp  1
1  0.0032 ( T  840 ) otherwise

© 2011 Pearson Education, Inc., Upper Saddle River, NJ. All rights reserved. This publication is protected by Copyright and written permission should be
MACHINE DESIGN - An Integrated Approach, 4th Ed. 6-5a-2

Reliability Creliab  return 1.000 if R = 0.50 Creliab  0.753


return 0.897 if R = 0.90
return 0.814 if R = 0.99
return 0.753 if R = 0.999
return 0.702 if R = 0.9999
return 0.659 if R = 0.99999

3. Calculate the modified endurance limit.


S e  Cload  Csize Csurf  Ctemp Creliab S'e S e  30.2 ksi

4. Using equation (6.9), calculate the fatigue strength at N = 10 3 cycles.

S m  return 0.75 S ut if load = "axial" S m  99.0 ksi


0.9 S ut otherwise

b
5. The equation for the S-N curve in the HCF region is given by equation (6.10a): Sf = a N
6. Determine the constants a and b from equations (6.10c) and (6.10a). From Table 6-5, for N = 10 6 , z  3.000

1  Sm 
b   log  b  0.1717
z  Se 
Sm
a  a  324.120 ksi
10 
b
3

7. To draw the S-N graph over the range 10 3 <= N <= 10 8, define a piecewise continuous function.

b 6
S f ( N )  return a  N if N  10
S e otherwise

3 5 8
8. Plot the S-N curve over the range N  10 10  10

100

Sf ( N )
ksi

10
3 4 5 6 7 8
1 10 1 10 1 10 1 10 1 10 1 10
N
FIGURE 6-5a
S-N Diagram for Problem 6-5a
© 2011 Pearson Education, Inc., Upper Saddle River, NJ. All rights reserved. This publication is protected by Copyright and written permission should be
MACHINE DESIGN - An Integrated Approach, 4th Ed. 6-6-1
PROBLEM 6-6
Statement: For the trailer hitch from Problem 3-6 on p. 169 (also see Figures P6-2 and 1-5), find the infinite-lif
fatigue safety factors for all modes of failure assuming that the horizontal impact force of the traile
on the ball is fully reversed. Use steel with S ut = 600 MPa and S y = 450 MPa. Determine safety
factors for:
(a) The shank of the ball where it joins the ball bracket.
(b) Bearing failure in the ball bracket hole.
(c) Tearout failure in the ball bracket.
(d) Tensile failure in the 19-mm diameter attachment holes.
(e) Bending failure in the ball bracket as a cantilever.
Given: a  40 mm b  31 mm c  70 mm d  20 mm
Mtongue  100  kg Fpull  55.1 kN d sh  26 mm t  19 mm
S y  300  MPa S ut  600  MPa w  64 mm R  32 mm
Assumptions: 1. The nuts are just snug-tight (no pre-load), which is the worst case.
2. All reactions will be concentrated loads rather than distributed loads or pressures.
Solution: See Figures 6-6 and Mathcad file P0606.

W tongue

70 = c

1 F pull 1

40 = a

2 A A F a1x
B 19 = t

F b1 B
31 = b
C F a1y
20 = d F a2y

D
Fa2x

F b2
Fc2x
C

D
Fd2

F c2y

FIGURE 6-6A
Dimensions and Free Body Diagram for Problem 6-6

1. The dynamic loading in this problem is fully reversed so the mean stresses are zero and the alternating stresses
are the same as those calculated in Problem 4-6. From Problem 4-6, the alternating components of the principal
stresses in the shank of the ball where it joins the ball bracket are:

σa1  1277 MPa σa2  0  MPa σa3  0  MPa

2. Since 1 is the only nonzero principal stress, it is also the von Mises stress.
© 2011 Pearson Education, Inc., Upper Saddle River, NJ. All rights reserved. This publication is protected by Copyright and written permission should be
MACHINE DESIGN - An Integrated Approach, 4th Ed. 6-6-2

σ'a  σa1 σ'a  1277 MPa

3. Calculate the unmodified endurance limit. S'e  0.5 S ut S'e  300 MPa

4. Calculate the endurance limit modification factors for a nonrotating round beam.

Load Cload  1 (bending load)


2 2
Size A95  0.010462 d sh A95  7.072 mm

A95
d equiv  d equiv  9.609 mm
0.0766
 0.097
 d equiv 
Csize  1.189    Csize  0.955
 mm 

Surface A  4.51 b  0.265 (machined)

b
 Sut 
Csurf  A    Csurf  0.828
 MPa 
Temperature Ctemp  1

Reliability Creliab  0.753 (R = 99.9%)

5. Calculate the modified endurance limit.


S e  Cload  Csize Csurf  Ctemp Creliab S'e S e  178.54 MPa

Se
6. Calculate the factor of safety for the ball shank. Na  Na  0.14
σ'a

7. From Problem 4-6, the alternating components of the principal stresses at the bearing area in the ball bracket ho
are:

σa1  111.5  MPa σa2  0  MPa σa3  0  MPa

8. Since 1 is the only nonzero principal stress, it is also the von Mises stress.
σ'a  σa1 σ'a  111.5 MPa

9. Calculate the endurance limit modification factors that are different from those in step 4.

Load Cload  0.7 (axial load)

Size Csize  1 (axial load)

10. Calculate the modified endurance limit.


S e  Cload  Csize Csurf  Ctemp Creliab S'e S e  130.91 MPa

Se
11. Calculate the factor of safety for the bearing. Nb  Nb  1.17
σ'a
© 2011 Pearson Education, Inc., Upper Saddle River, NJ. All rights reserved. This publication is protected by Copyright and written permission should be
MACHINE DESIGN - An Integrated Approach, 4th Ed. 6-6-3

12. From Problem 4-6, the alternating components of the von Mises stress at the tearout shear area in the ball
bracket hole is:

σ'a  85.91  MPa

13. Calculate the endurance limit modification factors that are different from those in step 9.

Load Cload  1 (simulated bending load)

A95  2  t ( 32 mm)   0.5 d sh


2 2 2
Size A95  1111 mm

A95
d equiv  d equiv  120.439 mm
0.0766
 0.097
 d equiv 
Csize  1.189    Csize  0.747
 mm 

14. Calculate the modified endurance limit.


S e  Cload  Csize Csurf  Ctemp Creliab S'e S e  139.71 MPa

Se
15. Calculate the factor of safety against tearout. Nc  Nc  1.6
σ'a

16. From Problem 4-6, the alternating components of the von Mises stress in the attachment bolts is:
σ'a  540.5  MPa

17. Calculate the endurance limit modification factors that are different from those in step 9.

Load Cload  0.7 (axial load)

Size Csize  1 (axial load)

18. Calculate the modified endurance limit.


S e  Cload  Csize Csurf  Ctemp Creliab S'e S e  130.91 MPa

Se
19. Calculate the factor of safety against bolt tensile failure. Nd  Nd  0.24
σ'a

20. From Problem 4-6, the alternating components of the principal stresses in the cantilever beam are:

σa1  635.5  MPa σa2  0  MPa σa3  0  MPa

21. Since 1 is the only nonzero principal stress, it is also the von Mises stress.
σ'a  σa1 σ'a  635.5 MPa

22. Calculate the endurance limit modification factors that are different from those in step 17.
Load Cload  1 (bending load)
2
Size A95  0.05 w t A95  60.8 mm

© 2011 Pearson Education, Inc., Upper Saddle River, NJ. All rights reserved. This publication is protected by Copyright and written permission should be
MACHINE DESIGN - An Integrated Approach, 4th Ed. 6-6-4

A95
d equiv  d equiv  28.173 mm
0.0766
 0.097
 d equiv 
Csize  1.189    Csize  0.86
 mm 

23. Calculate the modified endurance limit.


S e  Cload  Csize Csurf  Ctemp Creliab S'e S e  160.85 MPa

Se
24. Calculate the factor of safety for the cantilever beam. Ne  Ne  0.25
σ'a

© 2011 Pearson Education, Inc., Upper Saddle River, NJ. All rights reserved. This publication is protected by Copyright and written permission should be
MACHINE DESIGN - An Integrated Approach, 4th Ed. 6-7-1
PROBLEM 6-7
Statement: Design the wrist pin of Problem 3-7 for infinite life with a safety factor of 1.5 if the 2500-g
acceleration is fully reveresed and S ut = 130 ksi.
Given: Force on wrist pin Fwristpin  12.258 kN Fwristpin  2756 lbf
Tensile strength S ut  130  ksi
Design safety factor Nd  1.5

Assumptions: Choose a suitable outside diameter, say od  0.375  in


Solution: See Figure 4-12 in the text and Mathcad file P0607.

Fwristpin
1. The force at each shear plane is F  F  1378 lbf
2
2. With only the direct shear acting on the plane, the Mohr diagram will be a circle with center at the origin and
radius equal to the shear stress. Thus, the principal normal stress is numerically equal to the shear stress,
which in this case is also the principal shear stress, so we have  = 1 = '.
F 4 F
3. The shear stress at each shear plane is τ= = = σ'
A  2
π od  id
2
π  od  id   S
2 2
Se e
4. For fully reversed loading the factor of safety is, Nd = =
σ' 4 F

5. Calculate the unmodified endurance limit. S'e  0.5 S ut S'e  65 ksi


6. Calculate the endurance limit modification factors for a nonrotating round pin (uniformly stressed).

Load Cload  1

Size A95 ( id) 



π od  id
2 2  d equiv( id) 
A95 ( id)
4 0.0766
 0.097
 d equiv( id) 
Csize( id)  0.869   
 in 

Surface A  1.34 b  0.085 (ground)

b
 S ut 
Csurf  A    Csurf  0.886
 ksi 
Temperature Ctemp  1

Reliability Creliab  1.000 (R = 50%)

7. Calculate the modified endurance limit.


S e( id)  Cload  Csize( id)  Csurf  Ctemp Creliab S'e

8. Solving for the inside diameter, guess id  0.2 in

Given
 2 
2
π od  id  S e( id)
Nd =
4 F
© 2011 Pearson Education, Inc., Upper Saddle River, NJ. All rights reserved. This publication is protected by Copyright and written permission should be
MACHINE DESIGN - An Integrated Approach, 4th Ed. 6-7-2

id  Find ( id) id  0.299 in

9. Round this down to the decimal equivalent of a common fraction (9/32), id  0.281  in

10. The realized factor of safety is,

 2 
2
π od  id  S e( id)
Nf  Nf  1.8
4 F

© 2011 Pearson Education, Inc., Upper Saddle River, NJ. All rights reserved. This publication is protected by Copyright and written permission should be
MACHINE DESIGN - An Integrated Approach, 4th Ed. 6-8-1
PROBLEM 6-8
Statement: A paper machine processes rolls of paper having a density of 984 kg/m3. The paper roll is 1.50-m
OD x 0.22-m ID x 3.23 m long and is on a simply supported, hollow, steel shaft with S ut = 400 MPa
Find the shaft ID needed to obtain a dynamic safety factor of 2 for a 10-year life if the shaft OD is
22 cm and the roll turns at 50 rpm.
y
Given: Paper roll:
kg
Density ρ  984  w
3
m
x
Outside dia. OD  1500 mm
Inside dia. ID  220  mm R L R
Length L  3230 mm
Shaft: V
Strength S ut  400  MPa R
Outside dia. od  220  mm L/2 L
0 x
Design safety factor Nfd  2
Design life Life  10 yr -R
Shaft speed ω  50 rpm
M
2
wL /8
Assumptions: 1. The shaft is stiffer than the paper roll so the
weight of the roll on the shaft can be modelled as a
uniformly distributed load.
2. The bearings that support the shaft are close to
0 x
the ends of the paper roll and are thin with respect L/2 L
to the length of the roll so we can consider the
distance between the shaft supports to be the FIGURE 6-8
same as the length of the roll.
Load, Shear, and Moment Diagrams
3. There are no stress concentrations near the
for Problem 6-8
point of maximum moment on the shaft.
4. The paper mill operates 3 shifts/day, 365 days/year.
5. The shaft is machined and the material reliabilty is 99.9%.
Solution: See Figure 6-8 and Mathcad file P0608.
1. This is a case of fully reversed bending. The FBD for this loading case is shown in Appendix B, Figure B-2b,
with the dimension a equal to 0. That is, the distributed load starts at the left support and ends at the right
support.
2. Calculate the number of stress cycles to see if we will design for finite or infinite life.
9
Nlife  Life ω Nlife  1.652  10 cycles

This is well beyond 10 6 cycles, so we will design for infinite life.

3. Determine the weight of the paper roll and the magnitude of the distributed load on the shaft.

Roll volume V 
π
4
 2
 OD  ID  L
2 V  5.585  10  mm
9 3

Roll weight W  V  ρ  g W  53.895 kN

Distributed W N
load on shaft w  w  16.686 (a
L mm
4. Figure D-2b shows that the maximum bending moment occurs at the center of the shaft and is
2
w L 7
Mmax  Mmax  2.176  10  N  mm
8
© 2011 Pearson Education, Inc., Upper Saddle River, NJ. All rights reserved. This publication is protected by Copyright and written permission should be
MACHINE DESIGN - An Integrated Approach, 4th Ed. 6-8-2

This is fully reversed bending so Ma  Mmax and Mm  0  N  mm

5. The stress in the shaft at the point of maximum bending moment will depend upon the, as yet, unknown id. Tha
is,

Area moment of inertia I ( id) 


π
64
 4
 od  id
4 (b)

Ma od
Alternating stress σa( id)  (c)
2  I ( id)

6. Calculate the modified endurance strength of the shaft.


Unmodified endurance limit S'e  0.5 S ut S'e  200  MPa
Modification factors:

Load Cload  1

 0.097
Csize  1.189   
od
Size  Csize  0.705
 mm 
 0.265
 Sut 
Surface Csurf  4.51   Csurf  0.922
 MPa 

Temperature Ctemp  1

Reliability Creliab  0.753

Modified endurance limit


S e  Cload  Csize Csurf  Ctemp Creliab S'e S e  97.8 MPa
(d)
7. Use the factor of safety equation as a design equation to solve for the unknown id. For fully reversed bending,
the factor of safety is

Se
Nf = (e)
σa

Substituting equations b and c into e and solving for id,


1
4
 4 32 Nfd  Ma od 
id   od   id  191.526  mm
 π S e 
Rounding this, let the shaft ID be id  190  mm

1
This gives a wall thickness of t   ( od  id) t  15 mm
2

© 2011 Pearson Education, Inc., Upper Saddle River, NJ. All rights reserved. This publication is protected by Copyright and written permission should be
MACHINE DESIGN - An Integrated Approach, 4th Ed. 6-9-1

PROBLEM 6-9
Statement: For the Vise Grip plier-wrench is drawn to scale in Figure P6-3, and for which the forces were
analyzed in Problem 3-9 and the stresses in Problem 4-9, find the safety factors for each pin for an
assumed clamping force of P = 4000 N in the position shown. The pins are 8-mm dia, S y = 400 MP
S ut = 520 MPa, and are all in double shear. Assume a desired finite life of 5E4 cycles.
Given: Pin stresses as calculated in Problem 4-9:
Pin 1-2 τ12  74.6 MPa Yield strength S y  400  MPa
Pin 1-4 τ14  50.7 MPa Tensile strength S ut  520  MPa
Pin 2-3 τ23  50.7 MPa Pin diameter d  8  mm
4
Pin 3-4 τ34  50.7 MPa Desired life Nlife  5  10

Assumptions: 1. Links 3 and 4 are in a toggle position, i.e., the pin that joins links 3 and 4 is in line with the pins
that join 1 with 4 and 2 with 3.
Solution: See Figure 6-9 and Mathcad file P0609.
1. The FBDs of the assembly and each individual link are shown in Figure 6-9. The dimensions, as scaled from
Figure P5-3 in the text, are shown on the link FBDs.

F 4
P
1

3 2
P

55.0 = b 50.0 = a
F14
39.5 = c 22.0 = d
F

129.2° 1 
4

F41 F34
F21 P


28.0 = e


P 2.8 = g


F43

3 F12
21.2 = h
F23 2
F32

26.9 = f

FIGURE 6-9
Free Body Diagrams for Problem 6-9

2. The pins are in pure shear, so the principal stresses are


© 2011 Pearson Education, Inc., Upper Saddle River, NJ. All rights reserved. This publication is protected by Copyright and written permission should be
MACHINE DESIGN - An Integrated Approach, 4th Ed. 6-9-2

Pin joining 1 and 2 σ'12  3  τ12 σ'12  129.211 MPa

All other pins σ'14  3  τ14 σ'14  87.815 MPa

3. This is a case of repeated fatigue loading. The alternating and mean von Mises stress components are:

Pin joining 1 and 2 σ'12a  0.5 σ'12 σ'12m  σ'12a

All other pins σ'14a  0.5 σ'14 σ'14m  σ'14a

4. Calculate the unmodified endurance limit. S'e  0.5 S ut S'e  260 MPa

5. Calculate the endurance limit modification factors for a non rotating round pin (uniformly stressed).

Load Cload  1
2
π d 2
Size A95  A95  50.265 mm
4

A95
d equiv  d equiv  25.617 mm
0.0766
 0.097
 d equiv 
Csize  1.189    Csize  0.868
 mm 

Surface A  4.51 b  0.265 (machined)

b
 Sut 
Csurf  A    Csurf  0.86
 MPa 
Temperature Ctemp  1

Reliability Creliab  1.000 (R = 50%)

6. Calculate the modified endurance limit.


S e  Cload  Csize Csurf  Ctemp Creliab S'e S e  194.07 MPa

7. Using equation (6.9), calculate the fatigue strength at N = 10 3 cycles. S m  0.9 S ut S m  468 MPa

b
8. The equation for the S-N curve in the HCF region is given by equation (6.10a): Sf = a N
9. Determine the constants a and b from equations (6.10c) and (6.10a). From Table 6-5, for N = 10 6 , z  3.000

1  Sm 
b   log  b  0.0851
z  S'e 
Sm
a  a  842.4 MPa
10 
b
3

© 2011 Pearson Education, Inc., Upper Saddle River, NJ. All rights reserved. This publication is protected by Copyright and written permission should be
MACHINE DESIGN - An Integrated Approach, 4th Ed. 6-9-3

4 b
10. Calculate the corrected fatigue strength at Nlife  5  10 cycles. S f  a  Nlife S f  335.49 MPa

11. Assuming a Case 3 load line, use equation (6.18e) to determine the factor of safety.

S f  S ut
Pin joining 1 and 2 Nf  Nf  3.2
σ'12a S ut  σ'12m S f

S f  S ut
All other pins Nf  Nf  4.6
σ'14a S ut  σ'14m S f

© 2011 Pearson Education, Inc., Upper Saddle River, NJ. All rights reserved. This publication is protected by Copyright and written permission should be
MACHINE DESIGN - An Integrated Approach, 4th Ed. 6-10-1
PROBLEM 6-10
Statement: An overhung diving board is shown in Figure P6-4a. A 100-kg person is standing on the free end.
Assume cross-sectional dimensions of 305 mm x 32 mm. What is the fatigue safety factor for
infinite life if the material is brittle fiberglass with S f = 39 MPa @ N = 5E8 cycles and S ut = 130 MP
in the longitudinal direction?
2000 = L

Given: Mass of person M  100  kg R1 P


Board dimensions w  305  mm
t  32 mm
Load dimensions b  700  mm
R2
L  2000 mm
Material properties S ut  130  MPa
S f5E8  39 MPa 700 = b
FIGURE 6-10
Free Body Diagram for Problem 6-10
Assumptions: 1. The given fatigue strength is fully corrected.
2. There are no stress-concentrations near the point of maximum moment on the diving board.
Solution: See Figure 6-10 and Mathcad file P0610.
1. This is a case of repeated bending. The FBD for this loading case is shown in Appendix B, Figure B-3a, with th
dimension a equal to L. That is, the concentrated force F is at the end of the overhung beam.
2. Determine the weight of the person on the end of the board.
Weight W  M  g W  980.7  N (a)

3. Figure B-3a in Appendix B shows that the maximum bending moment occurs at the right-hand support and is
6
Mmax  W  ( L  b ) Mmax  1.275  10  N  mm
This is repeated bending so

Mmax
Ma  and Mm  Ma
2
4. The stress in the board at the point of maximum bending moment is
3
w t 5 4
Area moment of inertia I  I  8.329  10  mm (b)
12
Ma t
Alternating stress σa  σa  12.2 MPa (c)
2 I
Mm t
Mean stress σm  σm  12.2 MPa (d)
2 I

5. For repeated (fluctuating) bending, the factor of safety for Case 3 loading is

S f5E8 S ut
Nf  Nf  2.4 (e)
σa S ut  σm S f5E8

© 2011 Pearson Education, Inc., Upper Saddle River, NJ. All rights reserved. This publication is protected by Copyright and written permission should be
MACHINE DESIGN - An Integrated Approach, 4th Ed. 6-11-1

PROBLEM 6-11
Statement: Repeat Problem 6-10 assuming the 100-kg person in Problem 6-10 jumps up 25 cm and lands back
on the board. Assume the board weighs 29 kg and deflects 13.1 cm statically when the person
stands on it. What is the fatigue safety factor for finite life if the material is brittle fiberglass with S
= 39 MPa @ N = 5E8 cycles and S ut = 130 MPa in the longitudinal direction?

Given: Maximum principal stresses due to 2000 = L


bending at R2 from Problem 4-11
σ1max  76.3 MPa R1 P
σ2max  0  MPa
σ3max  0  MPa
Ultimate strength S ut  130  MPa R2

Fatigue strength S f  39 MPa


8
Fatigue life Ncycle  5  10 700 = b

FIGURE 6-11
Solution: See Figure 6-11 and Mathcad file P0611. Free Body Diagram for Problem 6-11
1. The dynamic loading in this case is repeated, i.e., the stresses go from zero to the maximum values given above.
Thus, the minimum and maximum von Mises stresses are:

2 2
σ'max  σ1max  σ1max σ3max  σ3max

σ'max  76.3 MPa σ'min  0  MPa

2. The alternating and mean components of the von Mises stress are:

σ'max  σ'min
σ'a  σ'a  38.1 MPa
2
σ'max  σ'min
σ'm  σ'm  38.1 MPa
2

3. Assuming a Case 3 load line, use equation (6.18e) to calculate the factor of safety.

S f  S ut
Nf  Nf  0.79
σ'a S ut  σ'm S f

© 2011 Pearson Education, Inc., Upper Saddle River, NJ. All rights reserved. This publication is protected by Copyright and written permission should be
MACHINE DESIGN - An Integrated Approach, 4th Ed. 6-12-1

PROBLEM 6-12
Statement: Repeat Problem 6-10 using the cantilevered diving board design in Figure P6-4b.

2000
Given: Mass of person M  100  kg
1300 = L
Board dimensions w  305  mm
t  32 mm P
Load dimensions L  1300 mm
Material properties S ut  130  MPa
S f5E8  39 MPa M1
R1

Assumptions: 1. The given fatigue strength is fully corrected. 700


2. There are no stress-concentrations near the
point of maximum moment on the diving board. FIGURE 6-12
Free Body Diagram for Problem 6-12
Solution: See Figure 6-12 and Mathcad file P0612.
1. This is a case of repeated bending. The FBD for this loading case is shown in Appendix B, Figure B-1a, with
the dimension a equal to L. That is, the concentrated force F is at the end of the cantilever beam.
2. Determine the weight of the person on the end of the board.
Weight W  M  g W  980.7  N (a)

3. Figure B-1a in Appendix B shows that the maximum bending moment occurs at the support and is
6
Mmax  W  L Mmax  1.275  10  N  mm
This is repeated bending so

Mmax
Ma  and Mm  Ma
2
4. The stress in the board at the point of maximum bending moment is
3
w t 5 4
Area moment of inertia I  I  8.329  10  mm (b)
12
Ma t
Alternating stress σa  σa  12.2 MPa (c)
2 I
Mm t
Mean stress σm  σm  12.2 MPa (d)
2 I

5. For repeated (fluctuating) bending, the factor of safety for Case 3 loading is

S f5E8 S ut
Nf  Nf  2.4 (e)
σa S ut  σm S f5E8

© 2011 Pearson Education, Inc., Upper Saddle River, NJ. All rights reserved. This publication is protected by Copyright and written permission should be
MACHINE DESIGN - An Integrated Approach, 4th Ed. 6-13-1

PROBLEM 6-13
Statement: Repeat Problem 6-11 using the cantilevered diving board design in Figure P6-4b. Assume the
board weighs 19 kg and deflects 8.5 cm statically when the person stands on it.

2000
Given: Maximum principal stresses due to
bending at support from Problem 4-13 1300 = L
σ1max  87.1 MPa
P
σ2max  0  MPa
σ3max  0  MPa
Ultimate strength S ut  130  MPa M1
R1
Fatigue strength S f  39 MPa
8 700
Fatigue life Ncycle  5  10
FIGURE 6-13
Solution: See Figure 6-13 and Mathcad file P0613. Free Body Diagram for Problem 6-13
1. The dynamic loading in this case is repeated, i.e., the stresses go from zero to the maximum values given above.
Thus, the minimum and maximum von Mises stresses are:

2 2
σ'max  σ1max  σ1max σ3max  σ3max

σ'max  87.1 MPa σ'min  0  MPa

2. The alternating and mean components of the von Mises stress are:

σ'max  σ'min
σ'a  σ'a  43.5 MPa
2
σ'max  σ'min
σ'm  σ'm  43.5 MPa
2

3. Assuming a Case 3 load line, use equation (6.18e) to calculate the factor of safety.

S f  S ut
Nf  Nf  0.69
σ'a S ut  σ'm S f

© 2011 Pearson Education, Inc., Upper Saddle River, NJ. All rights reserved. This publication is protected by Copyright and written permission should be
MACHINE DESIGN - An Integrated Approach, 4th Ed. 6-14-1
PROBLEM 6-14
Statement: Figure P6-5 shows a child's toy called a pogo stick. The child stands on the pads, applying half
her weight on each side. She jumps off the ground, holding the pads up against her feet, and
bounces along with the spring cushioning the impact and storing energy to help each rebound.
Design the aluminum cantilever beam sections on which she stands to survive jumping 2 in off
the ground with a dynamic safety factor of 2 for a finite life of 5E4 cycles. Use 2000 series
aluminum. Define and size the beam shape.

Given: Heat treated 2024 aluminum:


Tensile strength S ut  64 ksi
Design safety factor Nfd  2
4
Design life Nlife  5  10

Assumptions: The beam will have a rectangular cross-


section with the load applied at a distance
of 5 in from the central support.
L  5  in

Solution: See Figure 6-14 and Mathcad file P0614.


1. From Problem 3-14, the total dynamic force on both Fi /2 Fi /2
foot supports is

Fimax  224  lbf Fimin  0  lbf


Therefore, the load on each support is
Fimax
Pmax  Pmax  112  lbf
2
Fimin
Pmin  Pmin  0  lbf
2

2. To give adequate support to the childs foot, let the P


width of the support beam be w  1.5 in
FIGURE 5-14
3. From Figure B-1(a) in Appendix B, the maximum Free Body Diagram for Problem 5-14
bending moment at x = 0 is

Mmax  Pmax L Mmax  560  in lbf Mmin  0  in lbf

4. Calculate the alternating and mean components of the bending moment.


Mmax  Mmin
Ma  Ma  280  in lbf
2
Mmax  Mmin
Mm  Mm  280  in lbf
2

5. Determine the unmodified endurance limit. S'e  19 ksi @ 5E8 cycles

6. Calculate the endurance limit modification factors for a nonrotating rectangular beam.
Load Cload  1

A95 ( t)
Size A95 ( t)  0.05 w t d equiv( t) 
0.0766
© 2011 Pearson Education, Inc., Upper Saddle River, NJ. All rights reserved. This publication is protected by Copyright and written permission should be
MACHINE DESIGN - An Integrated Approach, 4th Ed. 6-14-2

 0.097
 d equiv( t) 
Csize( t)  0.869   
 in 

Surface A  2.7 b  0.265 (machined)

b
 S ut 
Csurf  A    Csurf  0.897
 ksi 
Temperature Ctemp  1

Reliability Creliab  1.000 (R = 50%)

7. Calculate the modified endurance limit.


S e( t)  Cload  Csize( t)  Csurf  Ctemp Creliab S'e

8. Using equation (6.9), calculate the fatigue strength at N = 10 3 cycles. S m  0.9 S ut S m  57.6 ksi

b
9. The equation for the S-N curve in the HCF region is given by equation (6.10a): Sf = a N
10. Determine the constants a and b from equations (6.10c) and (6.10a). From Table 6-5, for N = 5E8 , z  5.699

1  Sm  Sm
b ( t)   log  a ( t) 
 Se( t) 
103
z b( t )

4 b( t )
11. Determine the corrected fatigue strength at Nlife  5  10 cycles. S f ( t)  a ( t)  Nlife

12. We can now determine the minimum required section depth, t. Using the distortion-energy failure theory
with the modified Goodman diagram, the bending stress will also be the only nonzero principal stress,
which will also be the von Mises stress. Assuming a Case 3 load line, use equation (6.18e) to determine the
factor of safety. Guess t  10 mm.

M c t 12 6 M
Bending stress σ= = M  =
I 2 3 2
w t w t
Given
2 S f ( t)  S ut
w t
Nfd = 
6 Ma S ut  Mm S f ( t)

t  Find ( t) t  0.304  in

Round this up to the next higher decimal equivalent of a common fraction, t  0.375  in
Using this value of t, the values of the functions of t are:

Csize( t)  0.912 S e( t)  15.545 ksi S f ( t)  38.981 ksi

The realized safety factor is


2 S f ( t)  S ut
w t
Nf   Nf  3.0
6 Ma S ut  Mm S f ( t)

© 2011 Pearson Education, Inc., Upper Saddle River, NJ. All rights reserved. This publication is protected by Copyright and written permission should be
MACHINE DESIGN - An Integrated Approach, 4th Ed. 6-15a-1
PROBLEM 6-15a
Statement: For a notched part having a notch dimension r, geometric stress concentration factor Kt, and
material strength S ut as shown in row a of Table P6-4, find the Neuber factor a, the material's notch
sensitivity q, and the fatigue stress-concentration factor Kf.

Given: Ultimate tensile strength S ut  100  ksi Notch radius r  0.25 in


Geometric stress-concentration factor Kt  3.3 Material is steel
Loading  "Bending"

Solution: See Mathcad file P0615a.

1
2 2
1. From Table 6-6, the Neuber constant for S ut  100 ksi is a  0.062  in a  0.062 in

1
2. Using equation 6-13, the notch sensitivity is q  q  0.890
a
1
r

3. The fatigue stress-concentration factor, from equation 6.11b, is


Kf  1  q   Kt  1  Kf  3.05

© 2011 Pearson Education, Inc., Upper Saddle River, NJ. All rights reserved. This publication is protected by Copyright and written permission should be
MACHINE DESIGN - An Integrated Approach, 4th Ed. 6-16-1
PROBLEM 6-16
Statement: A track to guide bowling balls is designed with two round rods as shown in Figure P6-6. The
rods are not parallel to one another but have a small angle between them. The balls roll on the
rods until they fall between them and drop onto another track. The angle between the rods is
varied to cause the ball to drop at different locations. Find the infinite-life safety factor for the
1-in dia SAE 1010 cold-rolled steel rods.
(a) Assume rods are simply supported at each end.
(b) Assume rods are fixed at each end.

Given: Tensile strength S ut  53 ksi


Rod diameter d  1.00 in a Fball

Solution: See Figure 6-16 and Mathcad file P0616.


R1 L R2
1. The maximum bending stress will occur at the outer
fibers of the rod at the section where the maximum
bending moment occurs which, in this case, is at x = a. FIGURE 6-16A
The only stress present on the top or bottom surface Free Body Diagram for Problem 6-16(a), taken
of the rod is the bending stress x. Therefore, on the on a plane through the rod axis and ball center

bottom surface where the stress is tensile, sx is the


principal stress 1 . Thus, from Problem 4-16, for a
simply supported rod,
Maximum principal stress σ1  748  psi

2. The dynamic loading is repeated from 0 to 1 for each ball that rolls down the track. The alternating and mean
components of the von Mises stress are:

Alternating von Mises stress σ'a  0.5 σ1 σ'a  374 psi

Mean von Mises stress σ'm  0.5 σ1 σ'm  374 psi

3. Calculate the unmodified endurance limit. S'e  0.5 S ut S'e  26.5 ksi

4. Calculate the endurance limit modification factors for a nonrotating round beam.

Load Cload  1

2 A95
Size A95  0.010462 d d equiv 
0.0766
 0.097
 d equiv 
Csize  0.869    Csize  0.957
 in 

Surface A  2.70 b  0.265 (machined)

b
 S ut 
Csurf  A    Csurf  0.943
 ksi 
Temperature Ctemp  1

Reliability Creliab  0.659 (R = 99.999%)

© 2011 Pearson Education, Inc., Upper Saddle River, NJ. All rights reserved. This publication is protected by Copyright and written permission should be
MACHINE DESIGN - An Integrated Approach, 4th Ed. 6-16-2

5. Calculate the modified endurance limit.


S e  Cload  Csize Csurf  Ctemp Creliab S'e S e  15.759 ksi

6. Assuming a Case 3 load line, use equation (6.18e) to determine the factor of safety.

S e S ut
Case a Nfa  Nfa  32
σ'a S ut  σ'm S e

7. For the built-in case, the maximum bending stress a Fball


will occur at the outer fibers of the rod at the
section where the maximum bending moment
occurs which, in this case, is at x = L. The only M1
R1 L R 2 M2
stress present on the top or bottom surface of the
rod is the bending stress x. Therefore, on the FIGURE 6-16B
bottom surface where the stress is tensile, sx is the Free Body Diagram for Problem 6-16(b), taken on a
principal stress 1 . Thus, from Problem 4-16, for a plane through the rod axis and ball center
simply supported rod,
Maximum principal stress σ1  577  psi

8. The dynamic loading is repeated from 0 to 1 for each ball that rolls down the track. The alternating and mean
components of the von Mises stress are:

Alternating von Mises stress σ'a  0.5 σ1 σ'a  288.5 psi

Mean von Mises stress σ'm  0.5 σ1 σ'm  288.5 psi

9. Assuming a Case 3 load line, use equation (6.18e) to determine the factor of safety.

S e S ut
Case b Nfb  Nfb  42
σ'a S ut  σ'm S e

© 2011 Pearson Education, Inc., Upper Saddle River, NJ. All rights reserved. This publication is protected by Copyright and written permission should be
MACHINE DESIGN - An Integrated Approach, 4th Ed. 6-17-1

PROBLEM 6-17
Statement: A pair of ice tongs is shown in Figure P6-7. The ice weighs 50 lb and is 10 in wide across the
tongs. The distance between the handles is 4 in, and the mean radius r of the tong is 6 in. The
rectangular cross-sectional dimensions are 0.75 x 0.312 in. Find the safety factor for the tongs for
5E5 cycles if their S ut = 50 ksi.
F
Given: Tensile strength S ut  50 ksi C
Cross-section: FC
Width w  0.312  in O 3.5 = cy

Depth h  0.75 in FO
5
Life Nf  5  10 11.0 = ax 2.0 = cx

A
12.0 = by
Assumptions: The tongs are forged. Use 99.99% reliability.
Operating temperature is between 32F and 70F. 5.0 = bx
FB
Solution: See Problem 4-17, Figure 6-17, and
B
Mathcad file P0617.
1. The maximum bending stress in the tong was found W/2
in Problem 4-17 at point A.
FIGURE 6-17
Free Body Diagram for Problem 6-17
Vertical direction σi  8.58 ksi
All other components are zero.
2. There are no other stress components present so
σ1max  σi σ1max  8.58 ksi σ2max  0  ksi σ3max  0  ksi

3. The dynamic loading in this case is repeated, thus


σ1min  0  ksi σ2min  0  ksi σ3min  0  ksi

4. Even though this is a brittle material, for HCF analysis, determine the von Mises effective stresses. Since there i
only one nonzero stress,

σ'max  σ1max σ'max  8.58 ksi

σ'min  σ1min σ'min  0 ksi

σ'max  σ'min
σ'a  σ'a  4.29 ksi
2
σ'max  σ'min
σ'm  σ'm  4.29 ksi
2

5. Calculate the unmodified endurance limit. S'e  0.5 S ut S'e  25 ksi


6. Calculate the endurance limit modification factors for a nonrotating rectangular beam.
Load Cload  1
2
Size A95  0.05 w h A95  7.548 mm

A95
d equiv  d equiv  9.927 mm
0.0766
© 2011 Pearson Education, Inc., Upper Saddle River, NJ. All rights reserved. This publication is protected by Copyright and written permission should be
MACHINE DESIGN - An Integrated Approach, 4th Ed. 6-17-2

 0.097
 d equiv 
Csize  1.189    Csize  0.952
 mm 

Surface A  39.9 b  0.995 (forged)

b
 S ut 
Csurf  A    Csurf  0.814
 ksi 
Temperature Ctemp  1

Reliability Creliab  0.702 (R = 99.99%)

7. Calculate the modified endurance limit.


S e  Cload  Csize Csurf  Ctemp Creliab S'e S e  13.59 ksi
8. Using equation (6.9), calculate the fatigue strength at N = 10 3 cycles. S m  0.9 S ut S m  45 ksi

b
9. The equation for the S-N curve in the HCF region is given by equation (6.10a): Sf = a N

10. Determine the constants a and b from equations (6.10c) and (6.10a). From Table 6-5, for N = 10 6 , z  3.000

1  Sm 
b   log  b  0.1733
z  Se 
Sm
a  a  148.991 ksi
10 
b
3

b
11. Using equation (6.10a), determine the fatigue strength. S f5E5  a  Nf S f5E5  15.326 ksi

12. Assuming a Case 3 load line, use equation (6.18e) to calculate the factor of safety.

S f5E5 S ut
Nf5E5  Nf5E5  2.7
σ'a S ut  σ'm S f5E5

© 2011 Pearson Education, Inc., Upper Saddle River, NJ. All rights reserved. This publication is protected by Copyright and written permission should be
MACHINE DESIGN - An Integrated Approach, 4th Ed. 6-18-1

PROBLEM 6-18
Statement: A pair of ice tongs is shown in Figure P6-7. The ice weighs 50 lb and is 10 in wide across the
tongs. The distance between the handles is 4 in, and the mean radius r of the tong is 6 in. The
rectangular cross-sectional dimensions are 0.75 x 0.312 in. Find the safety factor for the tongs
for 5E5 cycles if they are made of Class 40 gray cast iron.

Given: Tensile strength S ut  42 ksi F


Cross-section:
Width w  0.312  in C
FC
Depth h  0.75 in 3.5 = cy
O
5 FO
Life Nf  5  10
11.0 = ax 2.0 = cx
Assumptions: The tongs are as-cast. Use 99.99%
A
reliability. Operating temperature is 12.0 = by
between 32F and 70F. Set Csurf to 1 for a
5.0 = bx
cast finish, which does not need a
surface factor. FB
Solution: See Problem 4-18, Figure 6-18, and B
Mathcad file P0618.
W/2
1. The maximum bending stress in the tong was found in
Problem 4-17 at point A.
FIGURE 6-18
Vertical direction σi  8.58 ksi Free Body Diagram for Problem 6-18
All other components are zero.
2. There are no other stress components present so
σ1max  σi σ1max  8.58 ksi σ2max  0  ksi σ3max  0  ksi

3. The dynamic loading in this case is repeated, thus


σ1min  0  ksi σ2min  0  ksi σ3min  0  ksi

4. Even though this is a brittle material, for HCF analysis, determine the von Mises effective stresses. Since
there is only one nonzero stress,

σ'max  σ1max σ'max  8.58 ksi

σ'min  σ1min σ'min  0 ksi

σ'max  σ'min
σ'a  σ'a  4.29 ksi
2
σ'max  σ'min
σ'm  σ'm  4.29 ksi
2

5. Calculate the unmodified endurance limit. S'e  0.4 S ut S'e  16.8 ksi
6. Calculate the endurance limit modification factors for a nonrotating rectangular beam.
Load Cload  1
2
Size A95  0.05 w h A95  7.548 mm

© 2011 Pearson Education, Inc., Upper Saddle River, NJ. All rights reserved. This publication is protected by Copyright and written permission should be
MACHINE DESIGN - An Integrated Approach, 4th Ed. 6-18-2

A95
d equiv  d equiv  9.927 mm
0.0766
 0.097
 d equiv 
Csize  1.189    Csize  0.952
 mm 

Surface Csurf  1

Temperature Ctemp  1

Reliability Creliab  0.702 (R = 99.99%)

7. Calculate the modified endurance limit.


S e  Cload  Csize Csurf  Ctemp Creliab S'e S e  11.22 ksi

8. Using equation (6.9), calculate the fatigue strength at N = 10 3 cycles. S m  0.9 S ut S m  37.8 ksi

b
9. The equation for the S-N curve in the HCF region is given by equation (6.10a): Sf = a N

10. Determine the constants a and b from equations (6.10c) and (6.10a). From Table 6-5, for N = 10 6 , z  3.000

1  Sm 
b   log  b  0.1758
z  Se 
Sm
a  a  127.305 ksi
10 
b
3

b
11. Using equation (6.10a), determine the fatigue strength. S f5E5  a  Nf S f5E5  12.678 ksi

12. Assuming a Case 3 load line, use equation (6.18e) to calculate the factor of safety.

S f5E5 S ut
Nf5E5  Nf5E5  2.3
σ'a S ut  σ'm S f5E5

© 2011 Pearson Education, Inc., Upper Saddle River, NJ. All rights reserved. This publication is protected by Copyright and written permission should be
MACHINE DESIGN - An Integrated Approach, 4th Ed. 6-19-1

PROBLEM 6-19
Statement: Determine the size of the clevis pin shown in Figure P6-8 needed to withstand an applied repeated
force of 0 to 130000 lb for infinite life. Also determine the required outside radius of the clevis
end to not fail in either tearout or bearing if the clevis flanges are each 2.5 in thick. Use a safety
factor of 3. Assume S ut = 140 ksi for the pin and S ut = 80 ksi for the clevis.

Given: Minimum force Pmin  0  lbf Material strength:


Maximum force Pmax  130  kip Pin S utp  140  ksi
Flange thickness t  2.5 in Clevis S utc  80 ksi
Factor of safety against fatigue failure Nf  3
Assumptions: The parts are machined. Use 90% reliability and room temperature.

Solution: See Figure 6-19 and Mathcad file P0619.


1. Calculate the alternating and mean components of the forces on the clevis and link.

Pmax  Pmin
Pa  Pa  65 kip
2
Pmax  Pmin
Pm  Pm  65 kip
2
Stress in Pin
2. The pin is in double shear and there is no stress-concentration. The alternating and mean loads at one section
on the pin are
1 Pa 1 Pm
τa =  τm =  (1)
2 Apin 2 Apin

2
π d
3. The cross-section area of the pin is Apin ( d )  (2)
4

4. The alternating and mean shear stresses and von Mises stresses are

Pa Pm
τa( d )  τm( d )  (3)
2  Apin ( d ) 2  Apin ( d )

σ'a( d )  3  τa( d ) σ'm( d )  3  τm( d )


(4)
Pin Strength
5. Calculate the unmodified endurance limit. S'ep  0.5 S utp S'ep  70 ksi
6. Calculate the endurance limit modification factors for a nonrotating round pin in bending.

Load Cload  1
2 A95 ( d )
Size A95 ( d )  0.010462 d d equiv( d ) 
0.0766
 0.097
 dequiv( d) 
Csize( d )  0.869   
 in 
Surface A  2.70 b  0.265 (machined)

© 2011 Pearson Education, Inc., Upper Saddle River, NJ. All rights reserved. This publication is protected by Copyright and written permission should be
MACHINE DESIGN - An Integrated Approach, 4th Ed. 6-19-2

b
 S utp 
Csurf  A    Csurf  0.729
 ksi 
Temperature Ctemp  1

Reliability Creliab  0.897 (R = 90%)

7. Calculate the modified endurance limit.


S ep( d )  Cload  Csize( d )  Csurf  Ctemp Creliab S'ep (5)

Design Equation

8. Using the modified-Goodman failure criterion and a case 3 load line, the factor of safety is given by equation
6-18e as
S e S ut
Nf = (6)
σ'a S ut  σ'm S e

9. Substituting equations 4 and 5 into 6 and solving for d yields Guess d  2.0 in

Given
S ep( d )  S utp
Nf =
σ'a( d )  S utp  σ'm( d )  S ep( d )

d  Find ( d ) d  2.632 in

Rounding to the next higher eighth of an inch, let d  2.750  in

With this value of d, we have

σ'a( d )  9.5 ksi σ'm( d )  9.5 ksi S ep( d )  39.71 ksi

and the realized factor of safety against fatigue failure in the pin is

S ep( d )  S utp
Nf  Nf  3.3
σ'a( d )  S utp  σ'm( d )  S ep( d )
Tearout length

Clevis Tearout (See Figure 6-19)

10. Let the outside radius of the clevis be R. Then the tearout
area is

2 2
Atear ( R)  2  t  R  ( 0.5 d )

11. The alternating and mean shear stresses and von Mises
stresses are d R

Pa Pm
τa( R)  τm( R)  (7) FIGURE 6-19
2  Atear ( R) 2  Atear ( R) Tearout Diagram for Problem 6-19

© 2011 Pearson Education, Inc., Upper Saddle River, NJ. All rights reserved. This publication is protected by Copyright and written permission should be
MACHINE DESIGN - An Integrated Approach, 4th Ed. 6-19-3

σ'a( R)  3  τa( R) σ'm( R)  3  τm( R) (8)

Clevis Strength
12. Calculate the unmodified endurance limit. S'ec  0.5 S utc S'ec  40 ksi

13. Calculate the endurance limit modification factors for a nonrotating rectangular shear area (uniformly stressed).

Load Cload  1

A95 ( R)
Size A95 ( R)  Atear ( R) d equiv( R) 
0.0766
 0.097
 d equiv( R) 
Csize( R)  0.869   
 in 

Surface A  2.70 b  0.265 (machined)

b
 S utc 
Csurf  A    Csurf  0.845
 ksi 
Temperature Ctemp  1

Reliability Creliab  0.897 (R = 90%)

14. Calculate the modified endurance limit.


S ec( R)  Cload  Csize( R)  Csurf  Ctemp Creliab S'ec (9)

Design Equation

15. Using the modified-Goodman failure criterion and a case 3 load line, the factor of safety is given by equation
6-18e as
S e S ut
Nf = (10)
σ'a S ut  σ'm S e

16. Substituting equations 8 and 9 into 10 and solving for R yields Guess R  2  in
Given
S ec( R)  S utc
Nf =
σ'a( R)  S utc  σ'm( R)  S ec( R)

R  Find ( R) R  2.624 in R  2.625  in

Bearing Stress
The maximum bearing stress in the hole in each flange is

 Pa  Pm
σmaxbear  σmaxbear  9.5 ksi
2 d t

This is small compared to the ultimate strength of the clevis.


© 2011 Pearson Education, Inc., Upper Saddle River, NJ. All rights reserved. This publication is protected by Copyright and written permission should be
MACHINE DESIGN - An Integrated Approach, 4th Ed. 6-20-1
PROBLEM 6-20
Statement: A ±100 N-m torque is applied to a 1-m-long, solid round steel shaft. Design it to limit its angular
deflection to 2 deg and select a steel alloy to have a fatigue safety factor of 2 for infinite life.
Given: Applied torque Ta  100  N  m Tm  0  N  m
Shaft length L  1000 mm
Max deflection θmax  2  deg
Design safety factor Nfd  2
Modulus of rigidity G  80.8 GPa

Assumptions: There are no stress-concentrations anywhere on the shaft. The shaft is machined, reliability is
99.9%, and the it is at room temperature.
Solution: See Mathcad file P0620.
1. This is a case of fully reversed torsion. We will use the von Mises effective stress so the load factor will be 1.

The maximum torque is Tmax  Ta  Tm Tmax  100 N  m

2. The diameter of the shaft can be found from equations 4.24 and 4.25 with  = max.

Tmax L 32 Tmax L


θmax = =
J G 4
π d  G
1
4
 32 Tmax L 
Solving for d, d    d  24.514 mm
 π θmax G 
Rounding, let d  24.5 mm

3. Now, we can solve for the stress in the shaft.

Polar moment π 4 4 4
of inertia J  d J  3.537  10 mm
32

Ta d
Torsional stress τa  τa  34.632 MPa
2 J

The corresponding von Mises normal stress is

von Mises stress σ'a  3  τa σ'a  59.984 MPa

4. Using the factor of safety equation for reversed loading, calculate the required endurance limit

Se
Nf = S e  Nfd  σ'a S e  119.967 MPa
σ'a

5. This endurance limit is a function of the unknown ultimate tensile strength. Use the endurance limit modificati
equation to determine the required S ut.

S e = Cload  Csize Csurf  Ctemp Creliab S'e

6. Calculate the endurance limit modification factors for a solid, round steel shaft.

© 2011 Pearson Education, Inc., Upper Saddle River, NJ. All rights reserved. This publication is protected by Copyright and written permission should be
MACHINE DESIGN - An Integrated Approach, 4th Ed. 6-20-2

Load Cload  1
 0.097
Csize  1.189   
d
Size  Csize  0.872
 mm 

Surface A  4.51 b  0.265 (machined)

b
 S ut 
Csurf = A  
 MPa 
Temperature Ctemp  1

Reliability Creliab  0.753 (R = 99.9%)

Uncorrected
endurance strength S'e = 0.5 S ut

7. Substituting these into the equation above and solving for S ut,
1
b 1
 Se 
S ut     MPa S ut  395 MPa
 0.5 A  Csize Creliab MPa 
Based on this requirement, choose AISI 1020 cold-rolled steel that will be machined to size.

8. Check the actual factor of safety based on the material chosen. For this material, S ut  469  MPa

b
 Sut 
Surface factor Csurf  A    Csurf  0.884
 MPa 
Uncorrected
endurance strength S'e  0.5 S ut S'e  234.5 MPa

Corrected
endurance strength S e  Cload  Csize Csurf  Ctemp Creliab S'e

S e  136.0 MPa

Se
Factor of safety Nf  Nf  2.3
σ'a

© 2011 Pearson Education, Inc., Upper Saddle River, NJ. All rights reserved. This publication is protected by Copyright and written permission should be
MACHINE DESIGN - An Integrated Approach, 4th Ed. 6-21-1
PROBLEM 6-21
Statement: Figure P6-9 shows an automobile wheel with two common styles of lug wrench being used to
tighten the wheel nuts, a single-ended wrench in (a), and a double-ended wrench in (b). The
distance between points A and B is 1 ft in both cases and the handle diameter is 0.625 in. How
many cycles of tightening can be expected before a fatigue failure if the average tightening torque
is 100 ft-lb and the material S ut = 60 ksi?

Given: Distance between A and B d AB  1  ft Minimum torque Tmin  0  ft  lbf


Wrench diameter d  0.625  in Maximum torque Tmax  100  ft  lbf
Tensile strength S ut  60 ksi

Assumptions: 1. The forces exerted by the user's hands lie in a plane through the wrench that is also parallel to
the plane of the wheel.
2. The applied torque is perpendicular to the plane of the forces.
3. By virtue of 1 and 2 above, this is a planar problem that can be described in a 2D FBD.
4. The surface is as-forged, the reliability is 50%, and the wrench will not be used in extremely ho
or cold environments.

Solution: See Figure 6-21 and Mathcad file P0621.


12" = dAB
1. From examination of the FBDs, we see that, in F
both cases, the arms are in bending and the
stub that holds the socket wrench is in pure
torsion. The maximum bending stress in the
arm will occur near the point where the arm
transitions to the stub. The stress state at
this transition is very complicated, but we can T
find the nominal bending stress there by F
treating the arm as a cantilever beam, fixed at (a) Single-ended Wrench
the transition point. For both cases the
torque in the stub is the same.

Case (a) 12" = dAB


F
6"
2. The bending moment at the transition is

M = F  d AB = T

Tmax  Tmin T
Ma 
2 F
(b) Double-ended Wrench
Ma  600 in lbf
FIGURE 6-21
Mm  Ma Free Body Diagrams for Problem 6-21

3. The alternating and mean components of the bending stress at this point are found from
4
Moment of inertia 4 I  in
π d
I 
64
Dist to extreme fibre c  0.5 d c  0.313 in

Ma c
Alternating stress σxa  σxa  25.033 ksi
I
Mm c
Mean stress σxm  σxm  25.033 ksi
I
© 2011 Pearson Education, Inc., Upper Saddle River, NJ. All rights reserved. This publication is protected by Copyright and written permission should be
MACHINE DESIGN - An Integrated Approach, 4th Ed. 6-21-2

4. There are no other stress components present at this point, so x is the maximum principle stress here and
σ1 = σx σ2  0  psi σ3  0  psi
5. Since there is only one nonzero principal stress, the von Mises stress is σ' = σ1 = σx and

σ'a  σxa σ'm  σxm

6. Assuming a Case 3 load line, use equation (6.18e) to solve for the fatigue strength at which the wrench will fail
(safety factor of 1).

S f  S ut σ'a S ut
Nf = =1 S f  S f  42.954 ksi
σ'a S ut  σ'm S f S ut  σ'm

7. Using equation (6-5a), calculate the uncorrected endurance limit. S'e  0.5 S ut S'e  30 ksi
8. Calculate the endurance limit modification factors for a nonrotating round beam.

Load Cload  1

2 A95
Size A95  0.010462 d d equiv 
0.0766
 0.097
 d equiv 
Csize  0.869    Csize  1.002
 in 

Surface A  39.9 b  0.995 (as forged)

b
 S ut 
Csurf  A    Csurf  0.679
 ksi 
Temperature Ctemp  1

Reliability Creliab  1.000 (R = 50%)

9. Calculate the modified endurance limit.


S e  Cload  Csize Csurf  Ctemp Creliab S'e S e  20.398 ksi

10. Using equation (6.9), calculate the fatigue strength at N = 10 3 cycles. S m  0.9 S ut S m  54 ksi
b
11. The equation for the S-N curve in the HCF region is given by equation (6.10a): Sf = a N

12. Determine the constants a and b from equations (6.10c) and (6.10a). From Table 6-5, for N = 10 6 , z  3.000

1  Sm 
b   log  b  0.1409
z  Se 
Sm
a  a  142.955 ksi
10 
b
3

© 2011 Pearson Education, Inc., Upper Saddle River, NJ. All rights reserved. This publication is protected by Copyright and written permission should be
MACHINE DESIGN - An Integrated Approach, 4th Ed. 6-21-3

1
b
 Sf  3
13. Calculate the number of cycles to failure using equation (6.10a) Na    Na  5.1  10
a
Case (b)
F  d AB T
14. The bending moment at the transition is M= =
2 2

M c Tc
15. The tensile stress at this point is found from σx = =
I 2 I

16. The bending stress in the handle for case (b) is one half that of case (a). However, the torque in the stub is the
same in both cases. The shear stress at any point on the outside surface of the stub is found from

4
Polar moment of inertia J  2  I J  0.0150 in

Tmax c
Maximum shear stress τxymax  τxymax  25.03 ksi
J
Tmin c
Minimum shear stress τxymin  τxymin  0 ksi
J
τxymax  τxymin
Alternating shear stress τa  τa  12.52 ksi
2
τxymax  τxymin
Mean shear stress τm  τm  12.52 ksi
2

17. There are no other stress components present along the outside surface of the stub, so
σ1a  τa σ1a  12.5 ksi σ2a  0  psi σ3a  σ1a

2 2
and σ'a  σ1a  σ1a σ3a  σ3a σ'a  21.7 ksi

σ'm  σ'a σ'm  21.7 ksi

18. Assuming a Case 3 load line, use equation (6.18e) to solve for the fatigue strength at which the wrench will fail
(safety factor of 1).

S f  S ut σ'a S ut
Nf = =1 S f  S f  33.944 ksi
σ'a S ut  σ'm S f S ut  σ'm

1
b
 Sf  4
19. Calculate the number of cycles to failure using equation (6.10a) Nb    Nb  2.7  10
a

© 2011 Pearson Education, Inc., Upper Saddle River, NJ. All rights reserved. This publication is protected by Copyright and written permission should be
MACHINE DESIGN - An Integrated Approach, 4th Ed. 6-22-1
PROBLEM 6-22
Statement: A roller-blade skate is shown in Figure P6-10. The polyurethane wheels are 72 mm dia and spaced
on 104-mm centers. The skate-boot-foot combination weighs 2 kg. The effective "spring rate" of
the person-skate subsystem is 6000 N/m. The axles are 10-mm-dia steel pins in double shear with
S ut = 550 MPa. Find the fatigue safety factor for the pins when a 100-kg person lands a 0.5-m jump
on one foot assuming infinite life.
(a) Assume all 4 wheels land simultaneously.
(b) Assume that one wheel absorbs all the landing force.

Given: Axle pin diameter d  10 mm Tensile strength S ut  550  MPa


Assumptions: Pins are machined and reliability is 99.999%.
Solution: See Figure P6-10 and Mathcad file P0622.
1. From Problem 4-22, we have the stresses for cases (a) and (b): τa  5.71 MPa τb  22.9 MPa
2. The dynamic loading in this case is repeated so the stresses given in step 1 are the maximum and the minimum
stresses are zero. Determine the minimum, maximum, alternating, and mean von Mises stresses.

Part (a) σ'maxa  3  τa σ'maxa  9.89 MPa σ'mina  0  MPa

σ'maxa  σ'mina
σ'aa  σ'aa  4.945 MPa
2
σ'maxa  σ'mina
σ'ma  σ'ma  4.945 MPa
2

Part (b) σ'maxb  3  τb σ'maxb  39.664 MPa σ'minb  0  MPa

σ'maxb  σ'minb
σ'ab  σ'ab  19.832 MPa
2
σ'maxb  σ'minb
σ'mb  σ'mb  19.832 MPa
2

3. Calculate the unmodified endurance limit. S'e  0.5 S ut S'e  275 MPa
4. Calculate the endurance limit modification factors for a nonrotating round pin.

Load Cload  1
2
π d 2
Size A95  A95  78.54 mm
4

A95
d equiv  d equiv  32.021 mm
0.0766
 0.097
 d equiv 
Csize  1.189    Csize  0.849
 mm 

Surface A  4.51 b  0.265 (machined)

b
 Sut 
Csurf  A    Csurf  0.847
 MPa 
© 2011 Pearson Education, Inc., Upper Saddle River, NJ. All rights reserved. This publication is protected by Copyright and written permission should be
MACHINE DESIGN - An Integrated Approach, 4th Ed. 6-22-2

Temperature Ctemp  1

Reliability Creliab  0.659 (R = 99.999%)

5. Calculate the modified endurance limit.


S e  Cload  Csize Csurf  Ctemp Creliab S'e S e  130.42 MPa

6. Assuming a Case 3 load line, use equation (6.18e) to determine the factor of safety.

S e S ut
Part (a) Nfa  Nfa  21.3
σ'aa S ut  σ'ma S e

S e S ut
Part (b) Nfb  Nfb  5.3
σ'ab S ut  σ'mb S e

© 2011 Pearson Education, Inc., Upper Saddle River, NJ. All rights reserved. This publication is protected by Copyright and written permission should be
MACHINE DESIGN - An Integrated Approach, 4th Ed. 6-23a-1

PROBLEM 6-23a
Statement: The beam in Figure P6-11a is subjected to a sinusoidal force-time function with Fmax = F and Fmin
= -F/2, where F and the beam's other data are given in row a of Table P6-5. Find the stress state in
the beam due to this loading and choose a material specification that will give a safety factor of 3
for N = 5E8 cycles.

L
Given: Beam length L  1  m
b
Distance to concen. load b  0.6 m
Concentrated load F  500  N F
Moment of inertia
8 4
I  2.85 10 m
2 R1 R2
Distance to extreme fiber c  2.00 10 m
Design safety factor Nd  3
FIGURE 6-23
8 Free Body Diagram for Problem 6-23
Cycle life Nf  5  10
Solution: See Figure 6-23 and Mathcad file P0623a.
1. The minimum, maximum, alternating, and mean components of the loads are:
F
Fmax  F Fmax  500  N Fmin  Fmin  250  N
2
Fmax  Fmin Fmax  Fmin
Fa  Fa  375  N Fm  Fm  125  N
2 2

2. Calculate the alternating and mean components of the maximum bending moment on the beam using the equati
in Figure B-2(a) in Appendix B.

Ma  Fa b   1 
b
 Ma  90 N  m
 L

Mm  Fm b   1 
b
 Mm  30 N  m
 L

3. Calculate the alternating and mean components of the maximum bending stress in the beam using equation
(4.11b). These are principal stresses and also von Mises stresses.
M a c
σ'a  σ'a  63.158 MPa
I

Mm c
σ'm  σ'm  21.053 MPa
I

4. Calculate the beam cross-section dimensions from I and c.

Beam depth h  2  c h  40 mm

12 I
Beam width w  w  5.344  mm
3
h

5. Calculate the endurance limit modification factors for a nonrotating rectangular beam.

Load Cload  1
© 2011 Pearson Education, Inc., Upper Saddle River, NJ. All rights reserved. This publication is protected by Copyright and written permission should be
MACHINE DESIGN - An Integrated Approach, 4th Ed. 6-23a-2

2
Size A95  0.05 w h A95  10.688 mm

A95
d equiv  d equiv  11.812 mm
0.0766
 0.097
 d equiv 
Csize  1.189    Csize  0.936
 mm 

Surface Csurf  1

Temperature Ctemp  1

Reliability Creliab  1 (R = 50%)

6. Determine the modified endurance limit as a function of the unknown endurance limit.

S e S ut  Cload  Csize Csurf  Ctemp Creliab 0.5 S ut

7. Assuming a Case 3 load line, use equation (6.18e) as the design equation.

S e S ut  S ut
Nd =
σ'a S ut  σ'm S e S ut

8. Solve the equations in steps 6 and 7 simultaneously for the desired S ut.

Nd   2  σ'a  Cload  Csize Csurf  Ctemp Creliab σ'm


S ut 
Cload  Csize Csurf  Ctemp Creliab

S ut  468  MPa

9. Choose AISI 1020, cold-rolled steel (see Appendix A, Table A-9).

© 2011 Pearson Education, Inc., Upper Saddle River, NJ. All rights reserved. This publication is protected by Copyright and written permission should be
MACHINE DESIGN - An Integrated Approach, 4th Ed. 6-24a-1
PROBLEM 6-24a
Statement: The beam in Figure P6-11b is subjected to a sinusoidal force-time function with Fmax = F and
Fmin = F/2, where F and the beam's other data are given in row a of Table P6-5. Find the stress
state in the beam due to this loading and choose a material specification that will give a safety
factor of 1.5 for N = 5E8 cycles.
Given: Beam length L  1  m
Concentrated load F  500  N L
Moment of inertia
8 4
I  2.85 10 m
F
2
Distance to extreme fiber c  2.00 10 m
Design safety factor Nd  1.5
8 M1
Cycle life Nf  5  10
R1
Solution: See Figure 6-24 and Mathcad file P0624a.
FIGURE 6-24
1. The minimum, maximum, alternating, and mean components Free Body Diagram for Problem 6-24
of the loads are:
F
Fmax  F Fmax  500  N Fmin  Fmin  250  N
2
Fmax  Fmin Fmax  Fmin
Fa  Fa  125  N Fm  Fm  375  N
2 2

2. Calculate the alternating and mean components of the maximum bending moment on the beam using the equati
in Figure B-1(a) in Appendix B.

Ma  Fa L Ma  125  N  m

Mm  Fm L Mm  375  N  m

3. Calculate the alternating and mean components of the maximum bending stress in the beam using equation
(4.11b). These are principal stresses and also von Mises stresses.
M a c
σ'a  σ'a  87.719 MPa
I

Mm c
σ'm  σ'm  263.158  MPa
I

4. Calculate the beam cross-section dimensions from I and c.

Beam depth h  2  c h  40 mm

12 I
Beam width w  w  5.344  mm
3
h

5. Calculate the endurance limit modification factors for a nonrotating rectangular beam.
Load Cload  1
2
Size A95  0.05 w h A95  10.688 mm
© 2011 Pearson Education, Inc., Upper Saddle River, NJ. All rights reserved. This publication is protected by Copyright and written permission should be
MACHINE DESIGN - An Integrated Approach, 4th Ed. 6-24a-2

A95
d equiv  d equiv  11.812 mm
0.0766
 0.097
 d equiv 
Csize  1.189    Csize  0.936
 mm 

Surface Csurf  1

Temperature Ctemp  1

Reliability Creliab  1 (R = 50%)

6. Determine the modified endurance limit as a function of the unknown endurance limit assuming the material is
steel.

S e S ut  Cload  Csize Csurf  Ctemp Creliab 0.5 S ut

7. Assuming a Case 3 load line, use equation (6.18e) as the design equation.

S e S ut  S ut
Nd =
σ'a S ut  σ'm S e S ut

8. Solve the equations in steps 6 and 7 simultaneously for the desired S ut.

Nd   2  σ'a  Cload  Csize Csurf  Ctemp Creliab σ'm


S ut 
Cload  Csize Csurf  Ctemp Creliab

S ut  676  MPa

9. Choose AISI 1060 hot-rolled steel (see Appendix A, Table C-9).

© 2011 Pearson Education, Inc., Upper Saddle River, NJ. All rights reserved. This publication is protected by Copyright and written permission should be
MACHINE DESIGN - An Integrated Approach, 4th Ed. 6-25a-1
PROBLEM 6-25a
Statement: The beam in Figure P6-11c is subjected to a sinusoidal force-time function with Fmax = F and
Fmin = 0, where F and the beam's other data are given in row a of Table P6-5. Find the stress
state in the beam due to this loading and choose a material specification that will give a safety
factor of 2.5 for N = 5E8 cycles.

Given: Beam length L  1  m


Distance between supports b  0.6 m L
Concentrated load F  500  N b
Moment of inertia
F
8 4
I  2.85 10 m
2
Distance to extreme fiber c  2.00 10 m
Design safety factor Nd  2.5 R1 R2

8 FIGURE 6-25
Cycle life Nf  5  10
Free Body Diagram for Problem 6-25
Solution: See Figure 6-25 and Mathcad file P0625a.
1. The minimum, maximum, alternating, and mean components of the loads are:

Fmax  F Fmax  500  N Fmin  0  N Fmin  0  N

Fmax  Fmin Fmax  Fmin


Fa  Fa  250  N Fm  Fm  250  N
2 2

2. Calculate the alternating and mean components of the maximum bending moment on the beam using the equati
in Figure B-1(a) in Appendix B.

Ma  Fa ( b  L) Ma  100  N  m

Mm  Fm ( b  L) Mm  100  N  m

3. Calculate the alternating and mean components of the maximum bending stress in the beam using equation
(4.11b). These are principal stresses and also von Mises stresses.
M a c
σ'a  σ'a  70.175 MPa
I

Mm c
σ'm  σ'm  70.175 MPa
I

4. Calculate the beam cross-section dimensions from I and c.

Beam depth h  2  c h  40 mm

12 I
Beam width w  w  5.344  mm
3
h
5. Calculate the endurance limit modification factors for a nonrotating rectangular beam.

Load Cload  1
© 2011 Pearson Education, Inc., Upper Saddle River, NJ. All rights reserved. This publication is protected by Copyright and written permission should be
MACHINE DESIGN - An Integrated Approach, 4th Ed. 6-25a-2

2
Size A95  0.05 w h A95  10.688 mm

A95
d equiv  d equiv  11.812 mm
0.0766
 0.097
 d equiv 
Csize  1.189    Csize  0.936
 mm 

Surface Csurf  1

Temperature Ctemp  1

Reliability Creliab  1 (R = 50%)

6. Determine the modified endurance limit as a function of the unknown endurance limit assuming the material is
steel.

S e S ut  Cload  Csize Csurf  Ctemp Creliab 0.5 S ut

7. Assuming a Case 3 load line, use equation (6.18e) as the design equation.

S e S ut  S ut
Nd =
σ'a S ut  σ'm S e S ut

8. Solve the equations in steps 6 and 7 simultaneously for the desired S ut.

Nd   2  σ'a  Cload  Csize Csurf  Ctemp Creliab σ'm


S ut 
Cload  Csize Csurf  Ctemp Creliab

S ut  550  MPa

9. Choose AISI 1035 cold-rolled steel (see Appendix A, Table A-9).

© 2011 Pearson Education, Inc., Upper Saddle River, NJ. All rights reserved. This publication is protected by Copyright and written permission should be
MACHINE DESIGN - An Integrated Approach, 4th Ed. 6-26a-1
PROBLEM 6-26a
Statement: The beam in Figure P6-11d is subjected to a sinusoidal force-time function with Fmax = F and Fmin
= -F, where F and the beam's other data are given in row a of Table P6-5. Find the stress state in
the beam due to this loading and choose a material specification that will give a safety factor of 6
for N = 5E8 cycles.

Given: Beam length L  1  m L


Distance to concentrated load a  0.4 m b
Distance to 2nd support b  0.6 m a F
Concentrated load F  500  N
Moment of inertia
8 4
I  2.85 10 m R1 R2 R3
2
Distance to extreme fiber c  2.00 10 m
Design safety factor Nfd  6 FIGURE 6-26A
Free Body Diagram for Problem 6-26
8
Cycle life Nlife  5  10

Solution: See Figures 6-26 and Mathcad file P0626a.


1. To determine the stresses, we must first get the maximum bending moment. From inspection of Figure P6-26,
write the load function equation
q(x) = R1<x>-1 - F<x - a>-1 + R2<x - b>-1 - R3<x - L>-1
2. Integrate this equation from - to x to obtain shear, V(x)
V(x) = R1<x>0 - F<x - a>0 + R2<x - b>0 - R3<x - L>0
3. Integrate this equation from - to x to obtain moment, M(x)
M(x) = R1<x>1 - F<x - a>1 + R2<x - b>1 - R3<x - L>1
4. Integrate the moment function, multiplying by 1/EI, to get the slope.
(x) = [R1<x>2/2 - F<x - a>2/2 + R2<x - b>2/2 + R3<x - L>2/2 + C3]/EI
5. Integrate again to get the deflection.
y(x) = [R1<x>3/6 - F<x - a>3/6 + R2<x - b>3/6 + R3<x - L>3/6 + C3x + C4]/EI
6. Evaluate R1, R2, R3, C3 and C4
At x = 0, x = b, and x = L; y = 0, therefore, C4 = 0.
At x = L+, V = M = 0
2
Guess R1  100  N R2  100  N R3  100  N C3  5  N  m
Given

R1 3 F 3 3
 b   ( b  a )  C 3 b = 0  N  m
6 6

R1 3 F 3 R2 3 3
 L   ( L  a)   ( L  b )  C3 L = 0  N  m
6 6 6

R1  F  R2  R3 = 0  N

R1 L  F  ( L  a )  R2 ( L  b ) = 0  N  m

© 2011 Pearson Education, Inc., Upper Saddle River, NJ. All rights reserved. This publication is protected by Copyright and written permission should be
MACHINE DESIGN - An Integrated Approach, 4th Ed. 6-26a-2

 R1 
 
 R2   Find  R R R C 
 R3  1 2 3 3

 
 C3 
2
R1  111.11 N R2  472.22 N R3  83.33  N C3  5.556  N  m

7. Define the range for x x  0  in 0.002  L  L


8. For a Mathcad solution, define a step function S. This function will have a value of zero when x is less than z,
and a value of one when it is greater than or equal to z.
S ( x z)  if ( x  z 1 0 )
9. Write the shear and moment equations in Mathcad form, using the function S as a multiplying factor to get the
effect of the singularity functions.

V ( x)   R1 S ( x 0  in)  F  S ( x a )   R2 S ( x b )  R3 S ( x L)

M ( x)  R1 S ( x 0  in)  x  F  S ( x a )  ( x  a )  R2 S ( x b )  ( x  b )

10. Plot the shear and moment diagrams.


Shear Diagram Moment Diagram

200 60

0 35

V ( x) M ( x)
 200 10
N Nm

 400  15

 600  40
3 3
0 200 400 600 800 1 10 0 200 400 600 800 1 10
x x
mm mm
FIGURE 6-26aB
Shear and Moment Diagrams for Problem 6-26a

11. From Figure 6-26aB, the maximum moment occurs at x = a. The maximum, minimum, alternating and mean
bending moments at x = a are:

Mmax  M ( a ) Mmax  44.4 N  m Mmin  Mmax

Mmax  Mmin
Ma  Ma  44.444 N  m
2
Mmax  Mmin
Mm  Mm  0  N  m
2
© 2011 Pearson Education, Inc., Upper Saddle River, NJ. All rights reserved. This publication is protected by Copyright and written permission should be
MACHINE DESIGN - An Integrated Approach, 4th Ed. 6-26a-3

12. Calculate the alternating and mean components of the maximum bending stress in the beam using equation
(4.11b). These are principal stresses and also von Mises stresses.
M a c
σ'a  σ'a  31.189 MPa
I

Mm c
σ'm  σ'm  0  MPa
I

13. Calculate the beam cross-section dimensions from I and c.

Beam depth h  2  c h  40 mm

12 I
Beam width w  w  5.344  mm
3
h

14. Calculate the endurance limit modification factors for a nonrotating rectangular beam.

Load Cload  1
2
Size A95  0.05 w h A95  10.688 mm

A95
d equiv  d equiv  11.812 mm
0.0766
 0.097
 d equiv 
Csize  1.189    Csize  0.936
 mm 
b
 S ut 
Surface A  4.51 b  0.265 Csurf  S ut  A   
 MPa 
Temperature Ctemp  1

Reliability Creliab  1 (R = 50%)

15. Determine the modified endurance limit as a function of the unknown endurance limit.

S e S ut  Cload  Csize Csurf  S ut  Ctemp Creliab 0.5 S ut

16. Assuming a Case 3 load line, use equation (6.18e) as the design equation and solve for S ut.

Guess S ut  100  MPa


Given

S e S ut  S ut
Nfd = S ut  Find  S ut S ut  447  MPa
σ'a S ut  σ'm S e S ut

Csurf  S ut  0.895

18. Choose AISI 1020, cold-rolled steel (see Appendix A, Table A-9).
© 2011 Pearson Education, Inc., Upper Saddle River, NJ. All rights reserved. This publication is protected by Copyright and written permission should be
MACHINE DESIGN - An Integrated Approach, 4th Ed. 6-27-1
PROBLEM 6-27
Statement: A storage rack is to be designed to hold the paper roll of Problem 6-8 as shown in Figure P6-12.
Determine a suitable value for dimension a in the figure for an infinite-life fatigue safety factor of
2. Assume dimension b = 100 mm and that the mandrel is solid and inserts halfway into the
paper roll.
(a) The beam is a ductile material with S ut = 600 MPa.
(b) The beam is a cast-brittle material with S ut = 300 MPa.

3
Given: Paper roll dimensions OD  1.50 m Roll density ρ  984  kg m
ID  0.22 m Design safety factor Nfd  2
Lroll  3.23 m Brittle tensile strength S utb  300  MPa
Ductile tensile strength S uta  600  MPa

Assumptions: The paper roll's weight creates a


concentrated load acting at the tip of
the mandrel. The mandrel's root fits a
tightly in the stanchion so it can be
modeled as a cantilever beam. The
M1 Lm
mandrel is machined, reliability is 90%,
R1
and it operates at room temperature.
FIGURE 6-27
Solution: See Figure 6-27 and Mathcad file P0627.
Free Body Diagram used in Problem 6-27
1. Determine the weight of the roll and the length of the mandrel.

Weight W 
π
 2 2 
 OD  ID  Lroll  ρ  g W  53.9 kN
4
Length Lm  0.5 Lroll Lm  1.615 m

2. The maximum moment occurs at a section where the mandrel root leaves the stanchion and is
Mmax  W  Lm Mmax  87.04 kN  m

3. The dynamic loading is repeated from 0 to Mmax on each stress cycle, thus Mmin  0  kN  m

4. Part (a) - Calculate the alternating and mean components of the bending moment.

Mmax  Mmin
Ma  Ma  43520 N  m
2
Mmax  Mmin
Mm  Mm  43520 N  m
2
5. Determine the unmodified endurance limit. S'e  0.5 S uta S'e  300 MPa

6. Calculate the endurance limit modification factors for a nonrotating rectangular beam.
Load Cload  1

2 A95 ( a )
Size A95 ( a )  0.010462 a d equiv( a ) 
0.0766
 0.097
 dequiv( a) 
Csize( a )  1.189   
 mm 
© 2011 Pearson Education, Inc., Upper Saddle River, NJ. All rights reserved. This publication is protected by Copyright and written permission should be
MACHINE DESIGN - An Integrated Approach, 4th Ed. 6-27-2

Surface A  4.51 b  0.265 (machined)

b
 S uta 
Csurf  A    Csurf  0.828
 MPa 
Temperature Ctemp  1

Reliability Creliab  0.897 (R = 90%)

7. Calculate the modified endurance limit.


S e( a )  Cload  Csize( a )  Csurf  Ctemp Creliab S'e

8. We can now determine the minimum required diameter, a. Using the distortion-energy failure theory with
the modified Goodman diagram, the bending stress will also be the only nonzero principal stress, which will
also be the von Mises stress. Assuming a Case 3 load line, use equation (6.18e) to determine the factor of
safety. Guess a  100  mm.

M c a 64 32 M
Bending stress σ= = M  =
I 2 4 3
π a π a
Given
3 S e( a )  S uta
π a
Nfd = 
32 Ma S uta  Mm S e( a )

a  Find ( a ) a  186.864 mm

Round this up to the next higher even value a  190  mm


Using this value of a, the values of the functions of a are:

Csize( a )  0.787 S e( a )  175.371 MPa

The realized safety factor is


3 S e( a )  S uta
π a
Nfa   Nfa  2.1
32 Ma S uta  Mm S e( a )

9. Part (b) - Determine the unmodified endurance limit. S'e  0.4 S utb S'e  120 MPa

10. Calculate the endurance limit size modification factor for a nonrotating rectangular beam.

2 A95 ( a )
Size A95 ( a )  0.010462 a d equiv( a ) 
0.0766
 0.097
 dequiv( a) 
Csize( a )  1.189   
 mm 
11. Calculate the modified endurance limit.
S e( a )  Cload  Csize( a )  Csurf  Ctemp Creliab S'e

© 2011 Pearson Education, Inc., Upper Saddle River, NJ. All rights reserved. This publication is protected by Copyright and written permission should be
MACHINE DESIGN - An Integrated Approach, 4th Ed. 6-27-3

12. We can now determine the minimum required diameter, a. Using the distortion-energy failure theory with the
modified Goodman diagram, the bending stress will also be the only nonzero principal stress, which will also
be the von Mises stress. Assuming a Case 3 load line, use equation (6.18e) to determine the factor of safety.
Guess a  100  mm.

M c a 64 32 M
Bending stress σ= = M  =
I 2 4 3
π a π a
Given
3 S e( a )  S utb
π a
Nfd = 
32 Ma S utb  Mm S e( a )

a  Find ( a ) a  251.687 mm

Round this up to the next higher even value a  252  mm


Using this value of a, the values of the functions of a are:

Csize( a )  0.766 S e( a )  68.253 MPa

The realized safety factor is


3 S e( a )  S utb
π a
Nfb   Nfb  2.0
32 Ma S utb  Mm S e( a )

© 2011 Pearson Education, Inc., Upper Saddle River, NJ. All rights reserved. This publication is protected by Copyright and written permission should be
MACHINE DESIGN - An Integrated Approach, 4th Ed. 6-28-1
PROBLEM 6-28
Statement: Figure P6-13 shows a forklift truck negotiating a 15 deg ramp to to drive onto a 4-ft-high loading
platform. The truck weighs 5 000 lb and has a 42-in wheelbase. Design two (one for each side)
1-ft-wide ramps of steel to have a safety factor of 2 for infinite life in the worst case of loading as
the truck travels up them. Minimize the weight of the ramps by using a sensible cross-sectional
geometry. Choose an appropriate steel or aluminum alloy.

Given: Ramp angle θ  15 deg Ramp width w  12 in


Platform height h  4  ft Truck weight W  5000 lbf
Truck wheelbase Lt  42 in

Assumptions: 1. The worst case is when the truck CG is located at the center of the beam's span.
2. Use a coordinate frame that has the x-axis along the long axis of the beam.
3. Ignore traction forces and the weight components along the x-axis of the beam.
4. There are two ramps, one for each side of the forklift.
Solution: See Figures 6-28 and Mathcad file P0628.

L
b
a CG a
y
CG b

R1

Fa Fb x
Wa
Wb
R2

FIGURE 6-28A
Dimensions and Free Body Diagram for Problem 6-28

1. From Problem 3-28 the maximum bending moment in the ramp occurs at the rear wheel of the truck and is
Mmax  8324 ft  lbf Mmax  99888 in lbf Mmin  0  in lbf

The alternating and mean components of the bending moment are:

Mmax  Mmin
Ma  Ma  49944 in lbf
2

Mmax  Mmin
Mm  Mm  49944 in lbf
2

2. The bending stress is the only stress component present and is, therefore, also the only nonzero principal
stress and is also the von Mises stress. The governing design equations then are
© 2011 Pearson Education, Inc., Upper Saddle River, NJ. All rights reserved. This publication is protected by Copyright and written permission should be
MACHINE DESIGN - An Integrated Approach, 4th Ed. 6-28-2

Ma Mm S e S ut
σ'a = σ'm = Nfd =
Z Z σ'a S ut  σ'm S e

Z  S e S ut
Combining these into a single equation Nfd =
Ma S ut  Mm S e

3. The approach will be to 1) choose a suitable factor of safety, 2) choose a suitable material and determine its
tensile strength and endurance limit, 3) from the equation above determine the required value of the section
modulus, 4) choose an appropriate cross-section for the ramp, and 5) determine the dimensions of the
cross-section.

4. The following design choices have been made for this problem:
Design factor of safety Nfd  2
Material AISI 1095 steel, hot-rolled
Tensile strength S ut  120  ksi

5. Determine the unmodified endurance limit. S'e  0.5 S ut S'e  60 ksi

6. Calculate the endurance limit modification factors for a nonrotating rectangular beam.
Load Cload  1

Size Csize  0.732 (initially guessed, and then found by iteration)

Surface A  14.4 b  0.718 (hot-rolled)

b
 S ut 
Csurf  A    Csurf  0.463
 ksi 
Temperature Ctemp  1

Reliability Creliab  0.897 (R = 90%)

7. Calculate the modified endurance limit.


S e  Cload  Csize Csurf  Ctemp Creliab S'e S e  18.237 ksi

8. Solve the design equation for the minimum section modulus, Z.

Ma Sut  Mm Se 3


Z  Nfd  Z  6.309 in
S e S ut

This is the minimum allowable value of the section modulus.


9. Assume a channel section such as that shown in Figure 6-28B. To keep it simple, let the thickness of the
flanges and web be the same. Choose 5/8-in thick plate, which is readily available. Then, t  0.625  in

10. The cross-sectional area of the ramp is A ( h )  w t  2  t ( h  t)

 w t 2 2
11. The distance to the CG is cg( h ) 
1
 2
 t h  t 
A (h)  2 

© 2011 Pearson Education, Inc., Upper Saddle River, NJ. All rights reserved. This publication is protected by Copyright and written permission should be
MACHINE DESIGN - An Integrated Approach, 4th Ed. 6-28-3

12. The moments of inertia of the web and a flange are


Flange
3 2
w t
 w t  cg( h )  
t Web
Iweb( h )  
12  2

3 2 t
t ( h  t) h
 h  t  cg( h ) 
t
Ifl ( h )  
12  2 

I ( h )  Iweb( h )  2  Ifl ( h )
h
13. The maximum stress will occur in the flange at the
top and is compressive. The distance from the
centroid up to the top of the flange is

c( h )  h  cg( h ) w

14. Using the known section modulus, solve for the FIGURE 6-28B
unknown flange height, h. Guess h  1  in Channel Section for Problem 6-28

I (h)
Given Z= h  Find ( h ) h  4.304 in Round this to h  4.25 in
c( h )

15. Check the size modification factor.


2
A95  0.05 w cg( h )  t ( h  cg( h ) ) A95  2.628 in
A95
d equiv  d equiv  5.858 in
0.0766
 0.097
 d equiv 
Csize  0.869    Csize  0.732
 in 

16. Summarizing, the ramp design dimensions are:

Width w  12.00 in Flange height h  4.25 in Shape channel


Thickness t  0.625 in Material 1095 steel

© 2011 Pearson Education, Inc., Upper Saddle River, NJ. All rights reserved. This publication is protected by Copyright and written permission should be
MACHINE DESIGN - An Integrated Approach, 4th Ed. 6-29-1
PROBLEM 6-29
Statement: A bar, 22 mm x 30 mm in cross-section, is loaded axially in tension with Fmin = -8 kN and Fmax =
+8 kN. A 10-mm hole passes through the center of the 30-mm side. Find the safety factor for
infinite life if the material has S ut = 500 MPa.

Given: Bar width w  30 mm Maximum load Fmax  8  kN


Bar thickness h  22 mm Minimum load Fmin  8  kN
Hole diameter d  10 mm Infinite life
Tensile strength S ut  500  MPa

Assumptions: Machined surfaces, temperature of 37C, and reliability F


of 99.999%.
Solution: See Figure 6-29 and Mathcad file P0629.
1. For completely reversed loading, the factor of safety is
Se
N =
Kf  σ'athe uniform axial stress is the only stress component present,
Since 30

Se
σ'a = σa and N =
Kf  σa
ø10
2. Calculate the unmodified endurance limit.
S'e  0.5 S ut S'e  250  MPa

3. Calculate the endurance limit modification factors for an axial bar.


Load Cload  0.7 (axial loading) 22

Size Csize  1 (axial loading)

Surface A  4.51 b  0.265

b
 Sut 
Csurf  A    Csurf  0.869
 MPa  F
Temperature Ctemp  1
FIGURE 6-29
Free Body Diagram used in
Reliability Creliab  0.659 (R = 99.999%) Problem 6-29

4. Calculate the modified endurance limit.


S e  Cload  Csize Csurf  Ctemp Creliab S'e S e  100.2  MPa

5. Determine the nominal (not increased by a stress concentration factor) alternating component of stress at the
hole.

2
Area A  ( w  d )  h A  440  mm

Fmax  Fmin
Alternating load Fa  Fa  8  kN
2
Fa
Alternating stress σa  σa  18.182 MPa
A
© 2011 Pearson Education, Inc., Upper Saddle River, NJ. All rights reserved. This publication is protected by Copyright and written permission should be
MACHINE DESIGN - An Integrated Approach, 4th Ed. 6-29-2

6. Determine the geometric stress concentration factor from Appendix C, Figure C-13.
2 3
 7.9735 
d
 9.2659   
d d
Kt  3.0039  3.753  
w  w w
4 5
 1.8145    2.9684  
d d Kt  2.33
w  w
7. Determine the notch sensitivity of the material. Note from Figure 6-35 that the Neuber constant for steel in
tension is slightly lower that for torsional loading. However, comparison of values of a 1/2 obtained from the
dashed red curve with those in Table 6-6 indicates that, for tension as well as torsion, a value of 20 ksi should be
added to S ut to obtain a 1/2 from Table 6-6.

Lookup value of S ut S'ut  S ut  20 ksi S'ut  93 ksi


2 0.5
Neuber constant a  0.068  in a  0.068  in

Notch radius r  0.5 d r  5  mm


1
Notch sensitivity q  q  0.867
a
1
r

8. Determine the fatigue stress concentration factor from equation (6.11b).


Kf  1  q   Kt  1  Kf  2.153

9. Determine the factor of safety against fatigue failure for the assumptions made.

Se
Nf  Nf  2.6
Kf  σa

© 2011 Pearson Education, Inc., Upper Saddle River, NJ. All rights reserved. This publication is protected by Copyright and written permission should be
MACHINE DESIGN - An Integrated Approach, 4th Ed. 6-30-1
PROBLEM 6-30
Statement: A bar, 22 mm x 30 mm in cross-section, is loaded axially in tension with Fmin = 0 kN and Fmax =
16 kN. A 10-mm hole passes through the center of the 30-mm side. Find the safety factor for
infinite life if the material has S ut = 500 MPa.

Given: Bar width w  30 mm Maximum load Fmax  16 kN


Bar thickness h  22 mm Minimum load Fmin  0  kN
Hole diameter d  10 mm Infinite life
Tensile strength S ut  500  MPa

Assumptions: Machined surfaces, temperature of 37C, and reliability


of 99.999%.
Solution: See Figure 6-29 and Mathcad file P0629.
F
1. For fluctuating loading, the factor of safety is

S e S ut
N =
Kf  σ'a S ut  Kfm σ'm S e

30
Since the uniform axial stress is the only stress component present,

σ'a = σa and σ'm = σm ø10


2. Calculate the unmodified endurance limit.
S'e  0.5 S ut S'e  250  MPa

3. Calculate the endurance limit modification factors for an axial bar.


Load Cload  0.7 (axial loading) 22

Size Csize  1 (axial loading)

Surface A  4.51 b  0.265

b
 Sut 
Csurf  A    Csurf  0.869
 MPa  F
Temperature Ctemp  1
FIGURE 6-30
Free Body Diagram used in
Reliability Creliab  0.659 (R = 99.999%) Problem 6-30

4. Calculate the modified endurance limit.


S e  Cload  Csize Csurf  Ctemp Creliab S'e S e  100.2  MPa

5. Determine the nominal (not increased by a stress concentration factor) alternating and mean components of
stress at the hole.

2
Area A  ( w  d )  h A  440  mm

© 2011 Pearson Education, Inc., Upper Saddle River, NJ. All rights reserved. This publication is protected by Copyright and written permission should be
MACHINE DESIGN - An Integrated Approach, 4th Ed. 6-30-2

Fmax  Fmin
Alternating load Fa  Fa  8  kN
2
Fmax  Fmin
Mean load Fm  Fm  8  kN
2
Fa
Alternating stress σa  σa  18.182 MPa
A

Fm
Mean stress σm  σm  18.182 MPa
A

6. Determine the geometric stress concentration factor from Appendix C, Figure C-13.
2 3
 7.9735 
d
 9.2659   
d d
Kt  3.0039  3.753   Kt  2.33
w  w w
4 5
 1.8145    2.9684  
d d
w  w
7. Determine the notch sensitivity of the material. Note from Figure 6-35 that the Neuber constant for steel in
tension is slightly lower that for torsional loading. However, comparison of values of a 1/2 obtained from the
dashed red curve with those in Table 6-6 indicates that, for tension as well as torsion, a value of 20 ksi
should be added to S ut to obtain a 1/2 from Table 6-6.

Lookup value of S ut S'ut  S ut  20 ksi S'ut  93 ksi


2 0.5
Neuber constant a  0.068  in a  0.068  in

Notch radius r  0.5 d r  5  mm


1
Notch sensitivity q  q  0.867
a
1
r

8. Determine the fatigue stress concentration factors from equations (6.11b) and (6.17).
Kf  1  q   Kt  1  Kf  2.153 Kf   σm  σa  78 MPa

Assuming the yield strength for this material is about 400 MPa, we can use the first of equations (6.17) and
Kfm  Kf Kfm  2.153

9. Determine the factor of safety against fatigue failure for the assumptions made.

S e S ut
Nf  Nf  2.1
Kf  σa S ut  Kfm σm S e

© 2011 Pearson Education, Inc., Upper Saddle River, NJ. All rights reserved. This publication is protected by Copyright and written permission should be
MACHINE DESIGN - An Integrated Approach, 4th Ed. 6-31-1
PROBLEM 6-31
Statement: A bar, 22 mm x 30 mm in cross-section, is loaded axially in tension with Fmin = 8 kN and Fmax =
24 kN. A 10-mm hole passes through the center of the 30-mm side. Find the safety factor for
infinite life if the material has S ut = 500 MPa.

Given: Bar width w  30 mm Maximum load Fmax  24 kN


Bar thickness h  22 mm Minimum load Fmin  8  kN
Hole diameter d  10 mm Infinite life
Tensile strength S ut  500  MPa

Assumptions: Machined surfaces, temperature of 37C, and reliability


of 99.999%.
Solution: See Figure 6-29 and Mathcad file P0629. F

1. For fluctuating loading, the factor of safety is

S e S ut
N =
Kf  σ'a S ut  Kfm σ'm S e
30

Since the uniform axial stress is the only stress component present,

ø10
σ'a = σa and σ'm = σm

2. Calculate the unmodified endurance limit.


S'e  0.5 S ut S'e  250  MPa

3. Calculate the endurance limit modification factors for an axial bar.


22
Load Cload  0.7 (axial loading)

Size Csize  1 (axial loading)

Surface A  4.51 b  0.265

b
 Sut 
Csurf  A    Csurf  0.869 F
 MPa 
FIGURE 6-31
Temperature Ctemp  1
Free Body Diagram used in
Problem 6-31
Reliability Creliab  0.659 (R = 99.999%)

4. Calculate the modified endurance limit.


S e  Cload  Csize Csurf  Ctemp Creliab S'e S e  100.2  MPa
5. Determine the nominal (not increased by a stress concentration factor) alternating and mean components of
stress at the hole.

2
Area A  ( w  d )  h A  440  mm

Fmax  Fmin
Alternating load Fa  Fa  8  kN
2
© 2011 Pearson Education, Inc., Upper Saddle River, NJ. All rights reserved. This publication is protected by Copyright and written permission should be
MACHINE DESIGN - An Integrated Approach, 4th Ed. 6-31-2

Fmax  Fmin
Mean load Fm  Fm  16 kN
2
Fa
Alternating stress σa  σa  18.182 MPa
A

Fm
Mean stress σm  σm  36.364 MPa
A

6. Determine the geometric stress concentration factor from Appendix C, Figure C-13.
2 3
 7.9735 
d
 9.2659 
d d
Kt  3.0039  3.753     Kt  2.33
w  w w
4 5
 1.8145    2.9684  
d d
w  w
7. Determine the notch sensitivity of the material. Note from Figure 6-35 that the Neuber constant for steel in
tension is slightly lower that for torsional loading. However, comparison of values of a 1/2 obtained from the
dashed red curve with those in Table 6-6 indicates that, for tension as well as torsion, a value of 20 ksi should
be added to S ut to obtain a 1/2 from Table 6-6.

Lookup value of S ut S'ut  S ut  20 ksi S'ut  93 ksi


2 0.5
Neuber constant a  0.068  in a  0.068  in

Notch radius r  0.5 d r  5  mm


1
Notch sensitivity q  q  0.867
a
1
r

8. Determine the fatigue stress concentration factors from equations (6.11b) and (6.17).
Kf  1  q   Kt  1  Kf  2.153 Kf   σm  σa  117  MPa

Assuming the yield strength for this material is about 400 MPa, we can use the first of equations (6.17) and
Kfm  Kf Kfm  2.153

9. Determine the factor of safety against fatigue failure for the assumptions made.

S e S ut
Nf  Nf  1.8
Kf  σa S ut  Kfm σm S e

© 2011 Pearson Education, Inc., Upper Saddle River, NJ. All rights reserved. This publication is protected by Copyright and written permission should be
MACHINE DESIGN - An Integrated Approach, 4th Ed. 6-32-1
PROBLEM 6-32
Statement: A bar, 22 mm x 30 mm in cross-section, is loaded axially in tension with Fmin = -4 kN and Fmax =
12 kN. A 10-mm hole passes through the center of the 30-mm side. Find the safety factor for
infinite life if the material has S ut = 500 MPa.
Given: Bar width w  30 mm Maximum load Fmax  12 kN
Bar thickness h  22 mm Minimum load Fmin  4  kN
Hole diameter d  10 mm Infinite life
Tensile strength S ut  500  MPa

Assumptions: Machined surfaces, temperature of 37C, and reliability


of 99.999%.
F
Solution: See Figure 6-29 and Mathcad file P0629.
1. For fluctuating loading, the factor of safety is

S e S ut
N =
Kf  σ'a S ut  Kfm σ'm S e
30

Since the uniform axial stress is the only stress component present,

ø10
σ'a = σa and σ'm = σm

2. Calculate the unmodified endurance limit.


S'e  0.5 S ut S'e  250  MPa

3. Calculate the endurance limit modification factors for an axial bar.


22
Load Cload  0.7 (axial loading)
Size Csize  1 (axial loading)

Surface A  4.51 b  0.265

b
 Sut 
Csurf  A    Csurf  0.869
 MPa  F
Temperature Ctemp  1
FIGURE 6-32
Free Body Diagram used in
Reliability Creliab  0.659 (R = 99.999%) Problem 6-32

4. Calculate the modified endurance limit.


S e  Cload  Csize Csurf  Ctemp Creliab S'e S e  100.2  MPa

5. Determine the nominal (not increased by a stress concentration factor) alternating and mean components of
stress at the hole.

2
Area A  ( w  d )  h A  440  mm

Fmax  Fmin
Alternating load Fa  Fa  8  kN
2
© 2011 Pearson Education, Inc., Upper Saddle River, NJ. All rights reserved. This publication is protected by Copyright and written permission should be
MACHINE DESIGN - An Integrated Approach, 4th Ed. 6-32-2

Fmax  Fmin
Mean load Fm  Fm  4  kN
2
Fa
Alternating stress σa  σa  18.182 MPa
A

Fm
Mean stress σm  σm  9.091  MPa
A

6. Determine the geometric stress concentration factor from Appendix C, Figure C-13.
2 3
 7.9735 
d
 9.2659   
d d
Kt  3.0039  3.753   Kt  2.33
w  w w
4 5
 1.8145    2.9684  
d d
w  w
7. Determine the notch sensitivity of the material. Note from Figure 6-35 that the Neuber constant for steel in
tension is slightly lower that for torsional loading. However, comparison of values of a 1/2 obtained from the
dashed red curve with those in Table 6-6 indicates that, for tension as well as torsion, a value of 20 ksi should be
added to S ut to obtain a 1/2 from Table 6-6.

Lookup value of S ut S'ut  S ut  20 ksi S'ut  93 ksi


2 0.5
Neuber constant a  0.068  in a  0.068  in

Notch radius r  0.5 d r  5  mm


1
Notch sensitivity q  q  0.867
a
1
r

8. Determine the fatigue stress concentration factors from equations (6.11b) and (6.17).
Kf  1  q   Kt  1  Kf  2.153 Kf   σm  σa  59 MPa

Assuming the yield strength for this material is about 400 MPa, we can use the first of equations (6.17) and
Kfm  Kf Kfm  2.153

9. Determine the factor of safety against fatigue failure for the assumptions made.

S e S ut
Nf  Nf  2.3
Kf  σa S ut  Kfm σm S e

© 2011 Pearson Education, Inc., Upper Saddle River, NJ. All rights reserved. This publication is protected by Copyright and written permission should be
MACHINE DESIGN - An Integrated Approach, 4th Ed. 6-33a-1
PROBLEM 6-33a
Statement: For the bracket shown in Figure P6-14 subjected to a sinusoidal force-time function with Fmax = F
and Fmin = -F, where F and the beam's other data are given in row a of Table P6-6. Find the stress
states at points A and B due to this fully reversed loading and choose a ductile steel material
specification that will give a safety factor of 2 for infinite life. Assume a geometric
stress-concentration factor of 2.5 in bending and 2.8 in torsion.

Given: Outside diameter od  20 mm F


y
Geometric stress Kt  2.5
A
concentration factors Kts  2.8
Design safety factor Nd  2 T
B
T x

Assumptions: The finish is machined, reliability is 50%, M


and the bracket operates at room
temperature. The notch sensitivity q = 1 L
so that Kf = Kt. R

Solution: See Figure 6-33 and Mathcad file P0633a. FIGURE 6-33
Free Body Diagram of Tube for Problem 6-33
1. From Problem 4-33a the stress components at point A are σx  8.38 MPa τzx  16.76  MPa

2. Calculate fatigue stresses and principal stresses.

Fatigue stresses σb  Kt σx σb  20.95 MPa

τs  Kts τzx τs  46.93 MPa

2
 σb 
σb 2
Principal stresses σ1      τs σ1  58.56 MPa
2 2

σ2  0  MPa

2
σb  σb  2
σ3      τs σ3  37.61 MPa
2 2

3. Calculate the alternating von Mises effective stress (the mean component is zero).

2 2
σ'a  σ1  σ1 σ3  σ3 σ'a  83.94 MPa

4. Calculate the unmodified endurance limit S'e S ut  0.5 S ut

5. Determine the endurance limit modification factors


Load Cload  1
2
Size A95  0.010462 od (nonrotating round section)

A95
d eq  d eq  7.391 mm
0.0766

© 2011 Pearson Education, Inc., Upper Saddle River, NJ. All rights reserved. This publication is protected by Copyright and written permission should be
MACHINE DESIGN - An Integrated Approach, 4th Ed. 6-33a-2

 0.097
 deq 
Csize  1.189    Csize  0.979
 mm 
Surface A  4.51 b  0.265 (cold-drawn tubing)

b
 S ut 
Cs S ut  A   
 MPa 
Csurf  S ut  if  Cs S ut  1 1 Cs S ut 

Temperature Ctemp  1

Reliability Creliab  1.0 (R = 50%)

6. Calculate the modified endurance limit

S e S ut  Cload  Csize Csurf  S ut  Ctemp Creliab S'e S ut

7. Use the equation for the factor of safety for fully reversed loading to solve for S ut. Guess S ut  100  MPa

S e S ut
Given Nd = S ut  Find  S ut S ut  362 MPa
σ'a

8. The varables that depend on S ut are: Csurf  S ut  0.946 S e S ut  167.88 MPa

© 2011 Pearson Education, Inc., Upper Saddle River, NJ. All rights reserved. This publication is protected by Copyright and written permission should be
MACHINE DESIGN - An Integrated Approach, 4th Ed. 6-34a-1
PROBLEM 6-34a
Statement: For the bracket shown in Figure P6-14 subjected to a sinusoidal force-time function with Fmax = F
and Fmin = 0, where F and the beam's other data are given in row a of Table P6-6. Find the stress
states at points A and B due to this repeated loading and choose a ductile steel material
specification that will give a safety factor of 2 for infinite life. Assume a geometric
stress-concentration factor of 2.8 in bending and 3.2 in torsion.

Given: Outside diameter od  20 mm F


y
Geometric stress Kt  2.8
A
concentration factors Kts  3.2
Design safety factor Nd  2 T
B
T x

Assumptions: The finish is machined, reliability is M


50%, and the bracket operates at room
temperature. The notch sensitivity q = L
1 so that Kf = Kt. R

Solution: See Figure 6-33 and Mathcad file P0633a. FIGURE 6-34
Free Body Diagram of Tube for Problem 6-34
1. From Problem 4-33a the stress components at point A are
σxmax  8.38 MPa τzxmax  16.76  MPa
σxmin  0  MPa τzxmin  0  MPa

2. Calculate the alternating and mean stress components.


σxmax  σxmin
σxa  σxa  4.19 MPa
2
σxmax  σxmin
σxm  σxm  4.19 MPa
2
τzxmax  τzxmin
τzxa  τzxa  8.38 MPa
2
τzxmax  τzxmin
τzxm  τzxm  8.38 MPa
2

3. Calculate fatigue stresses and principal stresses.

Fatigue stresses σba  Kt σxa σba  11.73 MPa

τsa  Kts τzxa τsa  26.82 MPa

σbm  Kt σxa σbm  11.73 MPa

τsm  Kts τzxa τsm  26.82 MPa

Principal stresses

2
 σba 
σba 2
σ1a      τsa σ1a  33.32 MPa
2  2 
σ2a  0  MPa

© 2011 Pearson Education, Inc., Upper Saddle River, NJ. All rights reserved. This publication is protected by Copyright and written permission should be
MACHINE DESIGN - An Integrated Approach, 4th Ed. 6-34a-2

2
 σba 
σba 2
σ3a      τsa σ3a  21.58 MPa
2  2

2
 σbm 
σbm 2
σ1m      τsm σ1m  33.32 MPa
2  2 
σ2m  0  MPa

2
 σbm 
σbm 2
σ3m      τsm σ3m  21.58 MPa
2  2 
4. Calculate the alternating and mean von Mises effective stress components.

2 2
σ'a  σ1a  σ1a σ3a  σ3a σ'a  47.91 MPa

2 2
σ'm  σ1m  σ1m σ3m  σ3m σ'm  47.91 MPa

5. Calculate the unmodified endurance limit S'e S ut  0.5 S ut

6. Determine the endurance limit modification factors


Load Cload  1
2
Size A95  0.010462 od (nonrotating round section)

A95
d eq  d eq  7.391 mm
0.0766
 0.097
 deq 
Csize  1.189    Csize  0.979
 mm 
Surface A  4.51 b  0.265 (machined tubing)

b
 S ut 
Cs S ut  A   
 MPa 
Csurf  S ut  if  Cs S ut  1 1 Cs S ut 

Temperature Ctemp  1

Reliability Creliab  1.0 (R = 50%)

7. Calculate the modified endurance limit

S e S ut  Cload  Csize Csurf  S ut  Ctemp Creliab S'e S ut

8. Use the equation for the factor of safety for repeated loading assuming a Case 3 load line and using equation
(6.18e). Guess S ut  100  MPa
© 2011 Pearson Education, Inc., Upper Saddle River, NJ. All rights reserved. This publication is protected by Copyright and written permission should be
MACHINE DESIGN - An Integrated Approach, 4th Ed. 6-34a-3

S e S ut  S ut
Given Nd =
σ'a S ut  σ'm S e S ut

S ut  Find  S ut S ut  291 MPa

9. The varables that depend on S ut are: Csurf  S ut  1 S e S ut  142.72 MPa

© 2011 Pearson Education, Inc., Upper Saddle River, NJ. All rights reserved. This publication is protected by Copyright and written permission should be
MACHINE DESIGN - An Integrated Approach, 4th Ed. 6-35a-1
PROBLEM 6-35a
Statement: For the bracket shown in Figure P6-14 subjected to a sinusoidal force-time function with Fmax = F
and Fmin = -F, where F and the beam's other data are given in row a of Table P6-6. Find the stress
states at points A and B due to this fully reversed loading and choose a cast iron material
specification that will give a safety factor of 2 for infinite life. Assume a geometric
stress-concentration factor of 2.5 in bending and 2.8 in torsion.

Given: Outside diameter od  20 mm F


Geometric stress Kt  2.5 y

concentration factors Kts  2.8 A


Design safety factor Nd  2 B
T T x
Assumptions: The finish is as-cast, reliability is 50%,
and the bracket operates at room M
temperature. There ia a fillet at the wall L
with radius r  2  mm. However, set the R
stress concentration factors and Csurf to 1
since cast iron's internal flaws mask these FIGURE 6-35
effects. Free Body Diagram of Tube for Problem 6-35

Solution: See Figure 6-35 and Mathcad file P0635a.


1 From Problem 4-33a the stress components at point A are σx  8.38 MPa τzx  16.76  MPa

2. Calculate fatigue stress concentration factors and principal stresses. Let Kf  1 Kfs  1

Fatigue stresses σb  Kf  σx σb  8.38 MPa

τs  Kfs τzx τs  16.76 MPa

Principal stresses

2
σb  σb  2
σ1      τs σ1  21.47 MPa
2 2

2
 σb 
σb 2
σ3      τs σ3  13.09 MPa
2 2
σ2  0  MPa

3. Calculate the alternating von Mises effective stress (the mean component is zero).

2 2
σ'a  σ1  σ1 σ3  σ3 σ'a  30.21 MPa

4. Calculate the unmodified endurance limit S'e S ut  0.4 S ut

5. Determine the endurance limit modification factors


Load Cload  1
2
Size A95  0.010462 od (nonrotating round section)

© 2011 Pearson Education, Inc., Upper Saddle River, NJ. All rights reserved. This publication is protected by Copyright and written permission should be
MACHINE DESIGN - An Integrated Approach, 4th Ed. 6-35a-2

A95
d eq  d eq  7.391 mm
0.0766
 0.097
 deq 
Csize  1.189    Csize  0.979
 mm 
Surface Csurf  1 ( cast iron)

Temperature Ctemp  1

Reliability Creliab  1.0 (R = 50%)

6. Calculate the modified endurance limit


S e S ut  Cload  Csize Csurf  Ctemp Creliab S'e S ut

7. Use the equation for the factor of safety for fully reversed loading to solve for S ut. Guess S ut  100  MPa

S e S ut
Given Nd = S ut  Find  S ut S ut  154 MPa
σ'a

8. The varables that depend on S ut are: S e S ut  60.43 MPa

© 2011 Pearson Education, Inc., Upper Saddle River, NJ. All rights reserved. This publication is protected by Copyright and written permission should be
MACHINE DESIGN - An Integrated Approach, 4th Ed. 6-36a-1
PROBLEM 6-36a
Statement: For the bracket shown in Figure P6-14 subjected to a sinusoidal force-time function with Fmax = F
and Fmin = 0, where F and the beam's other data are given in row a of Table P6-6. Find the stress
states at points A and B due to this repeated loading and choose a cast iron material specification
that will give a safety factor of 2 for infinite life. Assume a geometric stress-concentration factor o
2.8 in bending and 3.2 in torsion.

Given: Outside diameter od  20 mm


Geometric stress Kt  2.8
concentration factors Kts  3.2
Design safety factor Nd  2

Assumptions: The finish is as-cast, reliability is 50%,


and the bracket operates at room
temperature. There ia a fillet at the
wall with radius r  2  mm. However,
set the stress concentration factors and
Csurf to 1 since cast iron's internal flaws FIGURE 6-36
Free Body Diagram of Tube for Problem 6-36
mask these effects.
Solution: See Figure 6-36 and Mathcad file P0636a.
1. From Problem 4-33a the stress components at point A are
σxmax  8.38 MPa τzxmax  16.76  MPa

σxmin  0  MPa τzxmin  0  MPa

2. Calculate the alternating and mean stress components.


σxmax  σxmin
σxa  σxa  4.19 MPa
2
σxmax  σxmin
σxm  σxm  4.19 MPa
2
τzxmax  τzxmin
τzxa  τzxa  8.38 MPa
2
τzxmax  τzxmin
τzxm  τzxm  8.38 MPa
2

3. Calculate fatigue stress concentration factors (set 1 for cast iron) and principal stresses
Fatigue stress Kf  1 Kfs  1
concentration
factors Kfm  1 Kfsm  1

Fatigue stresses σba  Kf  σxa σba  4.19 MPa

τsa  Kfs τzxa τsa  8.38 MPa

σbm  Kfm σxa σbm  4.19 MPa

τsm  Kfsm τzxa τsm  8.38 MPa

© 2011 Pearson Education, Inc., Upper Saddle River, NJ. All rights reserved. This publication is protected by Copyright and written permission should be
MACHINE DESIGN - An Integrated Approach, 4th Ed. 6-36a-2

Principal stresses

2
 σba 
σba 2
σ1a      τsa σ1a  10.73 MPa
2  2
σ2a  0  MPa

2
 σba 
σba 2
σ3a      τsa σ3a  6.54 MPa
2  2

2
 σbm 
σbm 2
σ1m      τsm σ1m  10.73 MPa
2  2 
σ2m  0  MPa

2
 σbm 
σbm 2
σ3m      τsm σ3m  6.54 MPa
2  2 
4. Calculate the alternating von Mises effective stress (the mean component is zero).

2 2
σ'a  σ1a  σ1a σ3a  σ3a σ'a  15.11 MPa

2 2
σ'm  σ1m  σ1m σ3m  σ3m σ'm  15.11 MPa

5. Calculate the unmodified endurance limit S'e S ut  0.4 S ut


6. Determine the endurance limit modification factors
Load Cload  1
2
Size A95  0.010462 od (nonrotating round section)

A95
d eq  d eq  7.391 mm
0.0766
 0.097
 deq 
Csize  1.189    Csize  0.979
 mm 
Surface Csurf  1 (cast iron)
Temperature Ctemp  1
Reliability Creliab  1.0 (R = 50%)
7. Calculate the modified endurance limit
S e S ut  Cload  Csize Csurf  Ctemp Creliab S'e S ut

8. Use the equation for the factor of safety for repeated loading assuming a Case 3 load line and using equation
(6.18e). Guess S ut  100  MPa

S e S ut  S ut
Given Nd =
σ'a S ut  σ'm S e S ut

S ut  Find  S ut S ut  107 MPa


© 2011 Pearson Education, Inc., Upper Saddle River, NJ. All rights reserved. This publication is protected by Copyright and written permission should be
MACHINE DESIGN - An Integrated Approach, 4th Ed. 6-37-1
PROBLEM 6-37
Statement: A semicircular, curved beam as shown in Figure 5-37 has the dimensions given below. For a load
pair F = ±3 kN applied along the diameter, find the safety factor at the inner and outer fibers:
(a) If the beam is steel with S ut = 700 MPa,
(b) If the beam is cast-iron with S ut = 420 MPa.

Given: (a) Tensile strength S uta  700  MPa


(b) Tensile strength S utb  420  MPa
Maximum load Fmax  3  kN
Minimum load Fmin  3  kN
Solution: See Figure 6-37 and Mathcad file P0637.

1. From Problem 4-37, the stresses at the inside radius are:


Inside σi  409.9  MPa
These are based on a load of 14 kN. Since the stress in
a curved beam is directly proportional to the applied
load, we can determine the stresses at the inside surface
for this problem by applying the ratio 3/14 to this stress.
Thus,

3
σmax   σi σmax  87.836 MPa
14
3
σmin   σi σmin  87.836 MPa
14
FIGURE 6-37
These are the only stress components present on their Free Body Diagrams for Problem 6-37
respective surfaces so they are also von Mises stresses.
2. The dynamic loading in this problem is fully reversed. Determine the alternating stress component.

σmax  σmin
σ'a  σ'a  87.836 MPa
2
Part (a)
3. Calculate the unmodified endurance limit. S'ea  0.5 S uta S'ea  350 MPa

4. Calculate the endurance limit modification factors for a nonrotating rectangular beam.

Load Cload  0.7 (combined axial and bending loads)

Size Section dims w  25 mm h  25 mm


2
A95  0.05 w h A95  31.25 mm

A95
d equiv  d equiv  20.198 mm
0.0766
 0.097
 d equiv 
Csize  1.189    Csize  0.888
 mm 
Surface A  4.51 b  0.265 (machined)
© 2011 Pearson Education, Inc., Upper Saddle River, NJ. All rights reserved. This publication is protected by Copyright and written permission should be
MACHINE DESIGN - An Integrated Approach, 4th Ed. 6-37-2

b
 S uta 
Csurf  A    Csurf  0.795
 MPa 
Temperature Ctemp  1

Reliability Creliab  0.897 (R = 90%)

5. Calculate the modified endurance limit.


S ea  Cload  Csize Csurf  Ctemp Creliab S'ea S ea  155.15 MPa

6. Assuming no stress concentration, the fatigue factor of safety at the inner fiber for fully reversed loading is

S ea
Nfa  Nfa  1.8
σ'a

Part (b)

7. Calculate the unmodified endurance limit using equation (6.5b). S'eb  160  MPa

8. Calculate the endurance limit modification factors for a nonrotating rectangular beam.

Load Cload  0.7 (combined axial and bending loads)

Size Section dims w  25 mm h  25 mm


2
A95  0.05 w h A95  31.25 mm

A95
d equiv  d equiv  20.198 mm
0.0766
 0.097
 d equiv 
Csize  1.189    Csize  0.888
 mm 
Surface A  4.51 b  0.265 (machined)

b
 S utb 
Csurf  A    Csurf  0.910
 MPa 
Temperature Ctemp  1

Reliability Creliab  0.897 (R = 90%)

9. Calculate the modified endurance limit.


S eb  Cload  Csize Csurf  Ctemp Creliab S'eb S eb  81.21 MPa

10. Assuming no stress concentration, the fatigue factor of safety at the inner fiber for fully reversed loading is

S eb
Nfb  Nfb  0.92
σ'a
© 2011 Pearson Education, Inc., Upper Saddle River, NJ. All rights reserved. This publication is protected by Copyright and written permission should be
MACHINE DESIGN - An Integrated Approach, 4th Ed. 6-38-1
PROBLEM 6-38
Statement: A 42-mm-dia steel shaft with a 19-mm transverse hole is subjected to a sinusoidal combined
loading of  = ±100 MPa bending stress and steady torsion of 110 MPa. Find the safety factor
for infinite life if S ut = 1000 MPa.
Given: Shaft diameter D  42 mm Max bending stress σmax  100  MPa
Hole diameter d  19 mm Min bending stress σmin  100  MPa
Tensile strength S ut  1000 MPa Steady torsion τm  110  MPa
Infinite life

Assumptions: Stresses given include stress concentration effects. Ground surfaces, temperature of 37C, and
reliability of 50%.
Solution: See Mathcad file P0638.
1. Calculate the alternating and mean von Mises stress components.

σmax  σmin
σ'a  σ'a  100 MPa
2

σ'm  3  τm σ'm  190.526 MPa

2. Calculate the unmodified endurance limit.


S'e  0.5 S ut S'e  500 MPa

3. Calculate the endurance limit modification factors for a rotating, round shaft.
Load Cload  1 (combined bending and torsion)
 0.097
Csize  1.189   
D
Size  Csize  0.827
 mm 
Surface A  1.58 b  0.085 (ground)

b
 Sut 
Csurf  A    Csurf  0.878
 MPa 
Temperature Ctemp  1

Reliability Creliab  1 (R = 50%)

4. Calculate the modified endurance limit.


S e  Cload  Csize Csurf  Ctemp Creliab S'e S e  363.37 MPa

5. Assuming a Case 3 load line, determine the factor of safety against fatigue failure.

S e S ut
Nf  Nf  2.1
σ'a S ut  σ'm S e

© 2011 Pearson Education, Inc., Upper Saddle River, NJ. All rights reserved. This publication is protected by Copyright and written permission should be
MACHINE DESIGN - An Integrated Approach, 4th Ed. 6-39-1
PROBLEM 6-39
Statement: A 42-mm-dia steel shaft with a 19-mm transverse hole is subjected to a combined loading of  =
±100 MPa bending stress and an alternating torsion of ±110 MPa, which are 90 deg out of phase.
Find the safety factor for infinite life if S ut = 1000 MPa.

Given: Shaft diameter D  42 mm Max bending stress σmax  100  MPa
Hole diameter d  19 mm Min bending stress σmin  100  MPa
Tensile strength S ut  1000 MPa Max torsional stress τmax  110  MPa
Phase angle ϕ  90 deg Min torsional stress τmin  110  MPa

Assumptions: Stresses given include stress concentration effects. Ground surfaces, temperature of 37C, and
reliability of 50%.
Solution: See Mathcad file P0639.
1. The dynamic loading is fully reversed so both mean stresses are zero. Calculate the alternating SEQA stress
component using equation (6.23).
τmax
Stress ratio Q  2  Q  2.2
σmax
1
2
σmax  3 2 3 2 9 4
SEQAa  1  Q  1  Q  cos( 2  ϕ)  Q  SEQAa  190.526 MPa
2  4 2 16 

2. Calculate the unmodified endurance limit.


S'e  0.5 S ut S'e  500 MPa

3. Calculate the endurance limit modification factors for a rotating, round shaft.
Load Cload  1 (combined bending and torsion)
 0.097
Csize  1.189   
D
Size  Csize  0.827
 mm 
Surface A  1.58 b  0.085 (ground)

b
 Sut 
Csurf  A    Csurf  0.878
 MPa 
Temperature Ctemp  1

Reliability Creliab  1 (R = 50%)

4. Calculate the modified endurance limit.


S e  Cload  Csize Csurf  Ctemp Creliab S'e S e  363.37 MPa

5. Assuming a Case 3 load line, determine the factor of safety against fatigue failure.

Se
Nf  Nf  1.9
SEQAa

© 2011 Pearson Education, Inc., Upper Saddle River, NJ. All rights reserved. This publication is protected by Copyright and written permission should be
MACHINE DESIGN - An Integrated Approach, 4th Ed. 6-40-1
PROBLEM 6-40
Statement: Redesign the roll support of Problem 6-8 to be like that shown in Figure P6-16. The stub mandrels
insert to 10% of the roll length at each end. Design dimension a for an infinite-life factor of safety
of 2. See Problem 6-8 for additional data.
(a) The beam is a ductile material with S y = 450 MPa, S ut = 600 MPa
(b) The beam is a cast-brittle material with S ut = 300 MPa
3
Given: Paper roll dimensions OD  1.50 m Roll density ρ  984  kg m
ID  0.22 m Design safety factor Nfd  2
Lroll  3.23 m
Ductile tensile strength S uta  600  MPa Brittle tensile strength S utb  300  MPa

Assumptions: The paper roll's weight creates a concen-


trated load acting at the tip of the mandrel.
The mandrel's root fits tightly in the
stanchion so it can be modeled as a
cantilever beam. The mandrel is machined,
reliability is 99.999%, and it operates at
room temperature.

Solution: See Figure 6-40 and Mathcad file P0640.

1. Determine the weight of the roll, the load on each


support, and the length of the mandrel.
Weight of paper
FIGURE 6-40
W 
π
4
 2 2 
 OD  ID  Lroll  ρ  g
Free Body Diagram used in Problem 6-40

W  53.9 kN
Load on one mandrel F  0.5 W F  26.95 kN
Length of mandrel Lm  0.1 Lroll Lm  0.323 m

2. The maximum moment occurs at a section where the mandrel root leaves the stanchion and is
Mmax  F  Lm Mmax  8.704 kN  m

3. The dynamic loading is repeated from 0 to Mmax on each stress cycle, thus Mmin  0  kN  m
4. Part (a) - Calculate the alternating and mean components of the bending moment.

Mmax  Mmin
Ma  Ma  4352 N  m
2
Mmax  Mmin
Mm  Mm  4352 N  m
2

5. Determine the unmodified endurance limit. S'e  0.5 S uta S'e  300 MPa

6. Calculate the endurance limit modification factors for a nonrotating rectangular beam.

Load Cload  1

2 A95 ( a )
Size A95 ( a )  0.010462 a d equiv( a ) 
0.0766

© 2011 Pearson Education, Inc., Upper Saddle River, NJ. All rights reserved. This publication is protected by Copyright and written permission should be
MACHINE DESIGN - An Integrated Approach, 4th Ed. 6-40-2

 0.097
 dequiv( a) 
Csize( a )  1.189   
 mm 
Surface A  4.51 b  0.265 (machined)

b
 S uta 
Csurf  A    Csurf  0.828
 MPa 
Temperature Ctemp  1

Reliability Creliab  0.659 (R = 99.999%)

7. Calculate the modified endurance limit.


S e( a )  Cload  Csize( a )  Csurf  Ctemp Creliab S'e

8. We can now determine the minimum required diameter, a. Using the distortion-energy failure theory with the
modified Goodman diagram, the bending stress will also be the only nonzero principal stress, which will also
be the von Mises stress. Assuming a Case 3 load line, use equation (6.18e) to determine the factor of safety.
Guess a  100  mm.

M c a 64 32 M
Bending stress σ= = M  =
I 2 4 3
π a π a
Given
3 S e( a )  S uta
π a
Nfd = 
32 Ma S uta  Mm S e( a )

a  Find ( a ) a  92.421 mm

Round this up to the next higher even value a  94 mm


Using this value of a, the values of the functions of a are:

Csize( a )  0.843 S e( a )  137.942 MPa

The realized safety factor is


3 S e( a )  S uta
π a
Nfa   Nfa  2.1
32 Ma S uta  Mm S e( a )

9. Part (b) - Determine the unmodified endurance limit. S'e  0.4 S utb S'e  120 MPa

10. Calculate the endurance limit size modification factor for a nonrotating rectangular beam.

2 A95 ( a )
Size A95 ( a )  0.010462 a d equiv( a ) 
0.0766
 0.097
 dequiv( a) 
Csize( a )  1.189   
 mm 
11. Calculate the modified endurance limit.
S e( a )  Cload  Csize( a )  Csurf  Ctemp Creliab S'e
© 2011 Pearson Education, Inc., Upper Saddle River, NJ. All rights reserved. This publication is protected by Copyright and written permission should be
MACHINE DESIGN - An Integrated Approach, 4th Ed. 6-40-3

12. We can now determine the minimum required diameter, a. Using the distortion-energy failure theory with
the modified Goodman diagram, the bending stress will also be the only nonzero principal stress, which will
also be the von Mises stress. Assuming a Case 3 load line, use equation (6.18e) to determine the factor of
safety. Guess a  100  mm.

M c a 64 32 M
Bending stress σ= = M  =
I 2 4 3
π a π a
Given
3 S e( a )  S utb
π a
Nfd = 
32 Ma S utb  Mm S e( a )

a  Find ( a ) a  124.874 mm

Round this up to the next higher even value a  125  mm


Using this value of a, the values of the functions of a are:

Csize( a )  0.82 S e( a )  53.672 MPa

The realized safety factor is


3 S e( a )  S utb
π a
Nfb   Nfb  2.0
32 Ma S utb  Mm S e( a )

© 2011 Pearson Education, Inc., Upper Saddle River, NJ. All rights reserved. This publication is protected by Copyright and written permission should be
MACHINE DESIGN - An Integrated Approach, 4th Ed. 6-41-1
PROBLEM 6-41
Statement: A 10-mm ID steel tube carries liquid at 7 MPa. The pressure varies periodically from zero to
maximum. The steel has S ut = 400 MPa Determine the infinite-life fatigue safety factor for the
wall if its thickness is: a) 1 mm, b) 5 mm.
Given: Tensile strength S ut  400  MPa

Assumption: The tubing is long therefore the axial stress is zero. The finish is machined, reliability is 99.999%
and the tubing is at room temperature.
Solution: See Mathcad file P0641.

(a) Wall thickness is t  1  mm


1. From Problem 4-41, this is a thick wall cylinder and the maximum principal stresses are:

σ1maxa  38.82  MPa σ2maxa  0  MPa σ3maxa  7.00 MPa

2. Calculate the minimum, maximum, alternating, and mean von Mises effective stress using equation (5.7c).

σ'min  0  MPa

2 2
σ'maxa  σ1maxa  σ1maxa σ3maxa  σ3maxa σ'maxa  42.752 MPa

σ'maxa  σ'min
σ'aa  σ'aa  21.376 MPa
2
σ'maxa  σ'min
σ'ma  σ'ma  21.376 MPa
2

3. Calculate the unmodified endurance limit. S'e  0.5 S ut S'e  200 MPa

4. Calculate the endurance limit modification factors for axial loading.


Load Cload  0.7 (axial loading)
Size Csize  1 (axial loading)

Surface A  4.51 b  0.265 ( machined )

b
 Sut 
Csurf  A    Csurf  0.922
 MPa 
Temperature Ctemp  1

Reliability Creliab  0.659 (R = 99.999%)

5. Calculate the modified endurance limit.

S e  Cload  Csize Csurf  Ctemp Creliab S'e S e  85.04 MPa

6. Assuming a Case 3 load line, the factor of safety from equation (6.18e) is

S e S ut
Na  Na  3.3
σ'aa S ut  σ'ma S e
© 2011 Pearson Education, Inc., Upper Saddle River, NJ. All rights reserved. This publication is protected by Copyright and written permission should be
MACHINE DESIGN - An Integrated Approach, 4th Ed. 6-41-2

(b) Wall thickness is t  5  mm

7. From Problem 4-41, this is a thick wall cylinder and the principal stresses are:

σ1maxb  11.67  MPa σ2maxb  0  MPa σ3maxb  7.00 MPa

8. Calculate the minimum, maximum, alternating, and mean von Mises effective stress using equation (5.7c).

2 2
σ'maxb  σ1maxb  σ1maxb σ3maxb  σ3maxb σ'maxb  16.336 MPa

σ'maxb  σ'min
σ'ab  σ'ab  8.168 MPa
2

σ'maxb  σ'min
σ'mb  σ'mb  8.168 MPa
2

9. The endurance limit does not change from part a to b. Assuming a Case 3 load line, the factor of safety from
equation (6.18e) is

S e S ut
Nb  Nb  8.6
σ'ab S ut  σ'mb S e

© 2011 Pearson Education, Inc., Upper Saddle River, NJ. All rights reserved. This publication is protected by Copyright and written permission should be
MACHINE DESIGN - An Integrated Approach, 4th Ed. 6-42-1
PROBLEM 6-42
Statement: A cylindrical tank with hemispherical ends is required to hold 150 psi of pressurized air at room
temperature. The pressure cycles from zero to maximum. The steel has S ut = 500 MPa. Determine
the infinite-life fatigue safety factor if the tank diameter is 0.5 m with 1 mm wall thickness, and its
length is 1 m.
Given: Tensile strength S ut  500  MPa

Assumption: The finish is machined, reliability is 99.999% and the tank is at room temperature.
Solution: See Mathcad file P0642.
1. From Problem 4-42, this is a thin wall cylinder and the maximum principal stresses are:
σ1max  259  MPa σ2max  129  MPa σ3max  0  MPa

2. Calculate the minimum, maximum, alternating, and mean von Mises effective stress using equation (5.7c).

σ'min  0  MPa

2 2
σ'max  σ1max  σ1max σ2max  σ2max σ'max  224.301 MPa

σ'max  σ'min
σ'a  σ'a  112.151 MPa
2

σ'max  σ'min
σ'm  σ'm  112.151 MPa
2

3. Calculate the unmodified endurance limit. S'e  0.5 S ut S'e  250 MPa
4. Calculate the endurance limit modification factors for axial loading.
Load Cload  0.7 (axial loading)
Size Csize  1 (axial loading)

Surface A  4.51 b  0.265 ( machined )

b
 Sut 
Csurf  A    Csurf  0.869
 MPa 
Temperature Ctemp  1

Reliability Creliab  0.659 (R = 99.999%)

5. Calculate the modified endurance limit.


S e  Cload  Csize Csurf  Ctemp Creliab S'e S e  100.2 MPa

6. Assuming a Case 3 load line, the factor of safety from equation (6.18e) is

S e S ut
Nf  Nf  0.74
σ'a S ut  σ'm S e

© 2011 Pearson Education, Inc., Upper Saddle River, NJ. All rights reserved. This publication is protected by Copyright and written permission should be
MACHINE DESIGN - An Integrated Approach, 4th Ed. 6-43-1
PROBLEM 6-43
Statement: The paper rolls in Figure P6-17 are 0.9-m OD by 0.22-m ID by 3.23-m long and have a density of
984 kg/m3. The rolls are transfered from the machine conveyor (not shown) to the forklift truck
by the V-linkage of the off-load station, which is rotated through 90 deg by an air cylinder. The
paper then rolls onto the waiting forks of the truck. The forks are 38-mm thick by 100-mm wide
by 1.2-m long and are tipped at a 3-deg angle from the horizontal and have S ut = 600MPa. Find
the infinite-life safety factor for the two forks on the truck when the paper rolls onto it under two
different conditions (state all assumptions):
(a) The two forks are unsupported at their free end.
(b) The two forks are contacting the table at point A.
F
L fork
Given: Tensile strength S ut  600  MPa
t
Fork width w  100  mm
Fork thickness t  38 mm
Assumptions: 1. The greatest bending moment will occur M1
when the paper roll is at the tip of the fork R1
for case (a) and when it is midway between Case (a), Cantilever Beam
supports for case (b).
2. Each fork carries 1/2 the weight of a
paper roll. F
3. For case (a), each fork acts as a cantilever 0.5 L fork
beam (see Appendix B-1(a)). t
4. For case (b), each fork acts as a beam
that is built-in at one end and
simply-supported at the other. L fork
5. The forks are machined, the reliability is M2
R1 R2
90%, and they operate at room temperature.
Case (b), Fixed-Simply Supported Beam
Solution: See Figure 6-43 and Mathcad file P0643.
FIGURE 6-43
1. From Problem 4-43, the maximum stresses in the forks are: Free Body Diagrams used in Problem 6-43

Case (a) σmaxa  464.8  MPa at the base of the fork.

Case (b) σmaxb  87.2 MPa also at the base of the fork.

Both cases σmin  0  MPa


Since there are no other stress components present, these are also the maximum principal stresses and the von
Mises stresses. This is a repeated load problem.

Case (a)
2. The dynamic loading is repeated from 0 to 1 for each paper roll that is transfered. The alternating and mean
components of the von Mises stress are:

Alternating von Mises stress σ'a  0.5  σmaxa  σmin σ'a  232.4  MPa

Mean von Mises stress σ'm  0.5  σmaxa  σmin σ'm  232.4  MPa

3. Calculate the unmodified endurance limit. S'e  0.5 S ut S'e  300  MPa

4. Calculate the endurance limit modification factors for a nonrotating rectangular beam.
Load Cload  1

© 2011 Pearson Education, Inc., Upper Saddle River, NJ. All rights reserved. This publication is protected by Copyright and written permission should be
MACHINE DESIGN - An Integrated Approach, 4th Ed. 6-43-2

A95
Size A95  0.05 w t d equiv 
0.0766
 0.097
 d equiv 
Csize  0.869    Csize  0.814
 in 

Surface A  4.51 b  0.265 (machined)

b
 Sut 
Csurf  A    Csurf  0.828
 MPa 
Temperature Ctemp  1

Reliability Creliab  0.897 (R = 90%)

5. Calculate the modified endurance limit.


S e  Cload  Csize Csurf  Ctemp Creliab S'e S e  181.357  MPa

6. Assuming a Case 3 load line, use equation (6.18e) to determine the factor of safety.

S e S ut
Case a Nfa  Nfa  0.60
σ'a S ut  σ'm S e

Case (b)
7. The dynamic loading is repeated from 0 to 1 for each paper roll that is transfered. The alternating and mean
components of the von Mises stress are:

Alternating von Mises stress σ'a  0.5  σmaxb  σmin σ'a  43.6 MPa

Mean von Mises stress σ'm  0.5  σmaxb  σmin σ'm  43.6 MPa

8. Assuming a Case 3 load line, use equation (6.18e) to determine the factor of safety.

S e S ut
Case a Nfb  Nfb  3.2
σ'a S ut  σ'm S e

© 2011 Pearson Education, Inc., Upper Saddle River, NJ. All rights reserved. This publication is protected by Copyright and written permission should be
MACHINE DESIGN - An Integrated Approach, 4th Ed. 6-44-1
PROBLEM 6-44
Statement: Determine a suitable thickness for the V-links of the off-loading station of Figure P6-17 to limit th
deflections at the tips to 10 mm in any position during their rotation. Two V-links support the roll
at the 1/4 and 3/4 points along the roll's length and that each of the V-links is 10 cm wide by 1 m
long. What is the infinite-life safety factor when designed to limit deflection as above? S ut = 600
MPa. See Problem 4-43 for more information.
Given: Roll OD OD  0.90 m Arm width wa  100  mm
Roll ID ID  0.22 m Arm length La  1000 mm
Roll length Lroll  3.23 m Max tip deflection δtip  10 mm
3
Roll density ρ  984  kg m Mod of elasticity E  207  GPa
Tensile strength S ut  600  MPa

Assumptions: 1. The maximum deflection on an arm will occur just after it begins the transfer and just before it
completes it, i.e., when the angle is either zero or 90 deg., but after the tip is no longer supported b
the base unit.
2. At that time the roll is in contact with both arms ("seated" in the V) and will remain in that state
throughout the motion. When the roll is in any other position on an arm the tip will be supported.
3. The arm can be treated as a cantilever beam with nonend load.
4. A single arm will never carry more than half the weight of a roll.
5. The pipe to which the arms are attached has OD = 160 mm.
6. The V-links are machined, reliability is 90%, and they operate at room temperature.

Solution: See Figure 6-44 and Mathcad file P0644.


1. Determine the weight of the roll and the load on each
V-arm.

W 
π
4
 2 2 
 OD  ID  Lroll  ρ  g W  18.64  kN

F  0.5 W F  9.32 kN

2. From Appendix B, Figure B-1, the tip deflection of


a cantilever beam with a concentrated load
located at a distance a from the support is

2
F a
ymax =  ( a  3  L)
6  E I
where L is the beam length and I is the
cross-section moment of inertia. In this case
3
w a t a
I=
12

3. Setting ymax = δtip and a  370  mm, FIGURE 6-44


substituting for I and solving for ta Free Body Diagram used in Problem 6-44

1
3
 2 F  a2  3 La  a 
ta  ta  31.889 mm
 E δtip  wa 
 
Let the arm thickness be ta  32 mm
© 2011 Pearson Education, Inc., Upper Saddle River, NJ. All rights reserved. This publication is protected by Copyright and written permission should be
MACHINE DESIGN - An Integrated Approach, 4th Ed. 6-44-2

4. The maximum bending stress in the arm will be at its base where it joins the 160-mm-dia pipe. The bending
moment, moment of inertia, and distance to the outside fiber at that point are:
Bending moment Mmax  F  a Mmax  3.449  kN  m

Mmin  0  kN  m

  Mmax  Mmin
1
Ma  Ma  1.725  kN  m
2

  Mmax  Mmin
1
Mm  Mm  1.725  kN  m
2

Distance to n.a. c  0.5 t a c  16 mm

3
wa ta 5 4
Moment of inertia I  I  2.731  10  mm
12
M a c
Nom tensile stress σanom  σanom  101  MPa
I
Mm c
σmnom  σmnom  101  MPa
I

5. Determine the stress concentration factors. Figure E-10 comes the closest to our situation. Assuming that the
effective D/d-ratio is 2 and r/d is about 0.25, Kt  1.4. For a material with
3 2
ksi  10  psi S ut  87 ksi a  0.073  in

and, for the assumed value of r/d, r  0.25 ta r  8  mm

1
The notch sensitivity factor is q  q  0.885
a
1
r

and the fatigue stress concentration factor is


Kf  1  q   Kt  1  Kf  1.35

6. Assuming that Kfm  Kf , the actual alternating and mean components of stress at the point where the V-link
meets the central hub are
σa  Kf  σanom σa  136.8  MPa

σm  Kfm σmnom σm  136.8  MPa


7. Since there are no other nonzero stress components at this point on the top of the arm, the von Mises stresses
are
σ'a  σa and σ'm  σm

8. Determine the modified material strength.


Unmodified endurance limit S'e  0.5 S ut S'e  300  MPa

Load Cload  1
2
Size A95  0.05 wa t a A95  160  mm
© 2011 Pearson Education, Inc., Upper Saddle River, NJ. All rights reserved. This publication is protected by Copyright and written permission should be
MACHINE DESIGN - An Integrated Approach, 4th Ed. 6-44-3

A95
d eq  d eq  45.703 mm
0.0766
 0.097
 deq 
Csize  1.189    Csize  0.821
 mm 
 0.265
 Sut 
Surface Csurf  4.51   Csurf  0.828
 MPa 
Ctemp  1
Temperature
Creliab  0.897 (R = 90%)
Reliability

Endurance limit S e  Cload  Csize Csurf  Ctemp Creliab S'e S e  182.8  MPa

9. Calculate the factor of safety. Using the distortion energy theory and the modified Goodman theory, the
fatigue factor of safety for a V-link thickness of ta  32 mm is

S ut S e
Nf  Nf  1.0
σ'a S ut  σ'm S e

© 2011 Pearson Education, Inc., Upper Saddle River, NJ. All rights reserved. This publication is protected by Copyright and written permission should be
MACHINE DESIGN - An Integrated Approach, 4th Ed. 6-45-1
PROBLEM 6-45
Statement: Determine the infinite-life fatigue safety factor based on the tension load on the air-cylinder rod
in Figure P6-17. The tension load cycles from zero to maximum (compression loads below the
critical buckling load will not affect the fatigue life). The crank arm that it rotates is 0.3 m long
and the rod has a maximum extension of 0.5 m. The 25-mm-dia rod is solid steel with S ut = 600
MPa. State all assumptions.
Given: Paper roll dimensions OD  0.90 m Rod diameter d  25 mm
ID  0.22 m Tensile strength S ut  600  MPa
Lroll  3.23 m
3
Roll density ρ  984  kg m

Assumptions: 1. The maximum force in the cylinder rod occurs when the transfer starts.
2. The cylinder and rod make an angle of 5.5 deg to the horizontal at the end of transfer.
3. The crank arm is 300 mm long and is 45 deg from vertical at the end of transfer.
4. The finish is machined, reliability is 90%, and the cylinder operates at room temperature.
5. The cylinder rod is fully retracted at the start of the transfer. At the end of the transfer
it will have extended 500 mm from its initial position.

Solution: See Figure 6-45 and Mathcad file P0645.


1. Determine the weight of the roll on the y
forks.

W 
π
4
 2 2 
 OD  ID  Lroll  ρ  g

W  18.64 kN
2. From the assumptions and Figure 6-45,
the x and y distances from the origin to
point A are,

Rax  300  cos( 45 deg)  mm


450.0

Ray  300  sin( 45 deg)  mm W


Rx
Rax  212.132 mm x

212.1 Ry
Ray  212.132 mm A

F 5.5°
3. From Figure 6-45, the x distance from the
origin to point where W is applied is,
212.1
OD
Rwx  Rwx  450 mm
2
FIGURE 6-45
4. Sum moments about the pivot point and
Free Body Diagram at End of Transfer for V-link of Problem 6-45
solve for the tensile force in the cylinder
rod.

W  Rwx  Fmax Rax sin( 5.5 deg)  Fmax Ray cos( 5.5 deg) = 0

W  Rwx
Fmax  Fmax  35.017 kN tension
Ray cos( 8  deg)  Rax sin( 8  deg)

5. Assume that the dynamic load is repeated so Fmin  0  kN

© 2011 Pearson Education, Inc., Upper Saddle River, NJ. All rights reserved. This publication is protected by Copyright and written permission should be
MACHINE DESIGN - An Integrated Approach, 4th Ed. 6-45-2

6. Determine the alternating and mean components of axial stress in the rod.
2
π d 2
Area A  A  490.874 mm
4
Fmax  Fmin
Alternating load Fa  Fa  17.508 kN
2
Fmax  Fmin
Mean load Fm  Fm  17.508 kN
2
Fa
Alternating stress σa  σa  35.668 MPa
A

Fm
Mean stress σm  σm  35.668 MPa
A

S e S ut
7. For fluctuating loading, the factor of safety is Nf =
σ'a S ut  σ'm S e

8. Since the uniform axial stress is the only stress component present,
σ'a = σa and σ'm = σm

9. Calculate the unmodified endurance limit. S'e  0.5 S ut S'e  300 MPa
10. Calculate the endurance limit modification factors for an axial bar.
Load Cload  0.7 (axial loading)
Size Csize  1 (axial loading)

Surface A  4.51 b  0.265 (machined)

b
 Sut 
Csurf  A    Csurf  0.828
 MPa 
Temperature Ctemp  1

Reliability Creliab  0.897 (R = 90%)

11. Calculate the modified endurance limit.


S e  Cload  Csize Csurf  Ctemp Creliab S'e S e  155.95 MPa

12. Determine the factor of safety against fatigue failure for the assumptions made.

S e S ut
Nf  Nf  3.5
σa S ut  σm S e

© 2011 Pearson Education, Inc., Upper Saddle River, NJ. All rights reserved. This publication is protected by Copyright and written permission should be
MACHINE DESIGN - An Integrated Approach, 4th Ed. 6-46-1
PROBLEM 6-46
Statement: The V-links of Figure P6-17 are rotated by the crank arm through a shaft that is 60 mm dia by 3.23
m long. Determine the maximum torque applied to this shaft during motion of the V-linkage and
find the infinite-life fatigue safety factor for the shaft if its S ut = 600 MPa. See Problem 6-43 for
more information.

Given: Tensile strength S ut  600  MPa


Shaft diameter d  60 mm

Assumptions: 1. The greatest torque will occur


when the link is horizontal and the
paper roll is located as shown in
Figure P6-17 or Figure 6-46.
2. The V-links are machined, use a
reliability of 90%, and operate at
room temperature.

Solution: See Figure 6-46 and Mathcad file


P0646. W

1. From Problem 4-46, the maximum torsional


T
stress in the shaft is

τmax  197.88 MPa Ry


60-mm-dia shaft
2. Although not exactly true, assume that the 450.0
load is fully reversed, then the minimum
torque is FIGURE 6-46
Free Body Diagram used in Problem 6-46
τmin  197.88 MPa
3. Calculate the alternating component of the torsional stress in the shaft.
τmax  τmin
Alternating stress τa  τa  197.88 MPa
2

4. Convert this to the corresponding component of the von Mises stress.

Alternating stress σ'a  3  τa σ'a  342.738 MPa

5. Calculate the unmodified endurance limit using equation (6.5a). S'e  0.5 S ut S'e  300 MPa

6. Calculate the endurance limit modification factors for a solid, round steel shaft.

Load Cload  1
 0.097
Csize  1.189   
d
Size  Csize  0.799
 mm 

Surface A  4.51 b  0.265 (machined)

b
 Sut 
Csurf  A    Csurf  0.828
 MPa 
Temperature Ctemp  1

Reliability Creliab  0.897 (R = 90%)


© 2011 Pearson Education, Inc., Upper Saddle River, NJ. All rights reserved. This publication is protected by Copyright and written permission should be
MACHINE DESIGN - An Integrated Approach, 4th Ed. 6-46-2

7. Calculate the modified endurance limit.


S e  Cload  Csize Csurf  Ctemp Creliab S'e S e  178.07 MPa

8. Calculate the factor of safety for the shaft.

Se
Nf  Nf  0.52
σ'a

© 2011 Pearson Education, Inc., Upper Saddle River, NJ. All rights reserved. This publication is protected by Copyright and written permission should be
MACHINE DESIGN - An Integrated Approach, 4th Ed. 6-47-1
PROBLEM 6-47
Statement: Determine the maximum forces on the pins at each end of the air cylinder of Figure P6-17.
Determine the infinite-life fatigue safety factor in these pins if they are 30-mm dia and in single
shear. S ut = 600 MPa. See Problem 6-43 for more information.

Given: Pin diameter d  30 mm Tensile strength S ut  600  MPa


Assumptions: 1. The maximum force in the cylinder rod occurs when the transfer starts.
2. The dynamic loading is fully reversed.
3. The finish is machined, reliability is 90%, and the pins are at room temperature.

Solution: See Figure 6-47 and Mathcad file P0647.

W
Rx x

Ry 212.1
A

F 8°
212.1
450.0

FIGURE 6-47
Free Body Diagram at Start of Transfer for V-link of Problem 6-47

1. From Problem 4-47 the maximum shear stress on the pins is τmax  65.7 MPa
2. This is the only stress com- ponent so the alternating von Mises stress is σ'a  3  τmax σ'a  113.796 MPa

3. Calculate the unmodified endurance limit. S'e  0.5 S ut S'e  300 MPa
4. Calculate the endurance limit modification factors for a nonrotating direct shear.
2
π d 2
Size A95  A95  706.858 mm
4
Load Cload  1
A95
d equiv  d equiv  96.062 mm
0.0766
 0.097
 d equiv 
Csize  1.189    Csize  0.764
 mm 
Surface A  4.51 b  0.265 (machined)
© 2011 Pearson Education, Inc., Upper Saddle River, NJ. All rights reserved. This publication is protected by Copyright and written permission should be
MACHINE DESIGN - An Integrated Approach, 4th Ed. 6-47-2

b
 Sut 
Csurf  A    Csurf  0.828
 MPa 
Temperature Ctemp  1

Reliability Creliab  0.897 (R = 90%)

5. Calculate the modified endurance limit.


S e  Cload  Csize Csurf  Ctemp Creliab S'e S e  170.12 MPa

6. Assuming a Case 3 load line, use equation (6.14) to determine the factor of safety.

Se
Nf  Nf  1.5
σ'a

© 2011 Pearson Education, Inc., Upper Saddle River, NJ. All rights reserved. This publication is protected by Copyright and written permission should be
MACHINE DESIGN - An Integrated Approach, 4th Ed. 6-48-1
PROBLEM 6-48
Statement: Figure P6-18 shows an exerciser for a 100-kg wheelchair racer. The wheelchair has 65-cm-dia
drive wheels separated by a 70-cm track width. Two free-turning rollers on bearings support the
rear wheels. The lateral movement of the chair is limited by the flanges. Design the 1-m-lomg
rollers as hollow tubes of aluminum (select alloy) to minimize the height of the platform and also
limit the roller deflections to 1 mm in the worst case. Specify suitable sized steel axles to support
the tubes on bearings. Calculate the fatigue safety factors at a life of 5E8 cycles.

Given: Mass of chair M  100  kg Maximum deflection δ  1  mm


Wheel diameter d w  650  mm Modulus elasticity
Track width T  700  mm Aluminum Ea  71.7 GPa
Roller length Lr  1000 mm Steel Es  207  GPa

Assumptions: 1. The CG of the chair with rider is


sufficiently close to the rear wheel that all
of the weight is taken by the two rear
wheels.
2. The small camber angle of the rear W/2
wheels does not significantly affect the
magnitude of the forces on the rollers.
3. Both the aluminum roller and the steel
axle are simply supported. The steel axles
that support the aluminum tube are fixed
in the mounting block and do not rotate.
The aluminum tube is attached to them by
two bearings (one on each end of the
tubes, one for each axle). The bearings'
inner race is fixed, and the outer race F F
rotates with the aluminum tube. Each
steel axle is considered to be loaded as a
simply supported beam. Their diameter  
must be less than the inner diameter of the
tubes to fit the roller bearings between FIGURE 6-48A
them. Free Body Diagram of One Wheel
4. All surfaces are machined, reliability is used in Problem 6-48
90%, and parts are at room temperature.

Solution: See Figures 6-48 and Mathcad file P0648.


1. Calculate the weight of the chair with rider. Weight of chair W  M  g W  980.7  N
2. Calculate the forces exerted by the wheels on the rollers (see Figure 5-48A). From the FBD of a wheel,
summing vertical forces
W
2  F  cos( θ )  =0
2
W
Let θ  20 deg then F  F  260.9  N
4  cos( θ )

3. The worst condition (highest moment at site of a stress concentration) will occur when the chair is all the way
to the left or right. Looking at the plane through the roller that includes the forces exerted by the wheels (the
FBD is shown in Figure 6-48B) the reactions R1 and R2 come from the bearings, which are inside the hollow
roller and are, themselves, supported by the steel axle.

4. Solving for the reactions. Let the distance from R1 to F be a  15 mm

© 2011 Pearson Education, Inc., Upper Saddle River, NJ. All rights reserved. This publication is protected by Copyright and written permission should be
MACHINE DESIGN - An Integrated Approach, 4th Ed. 6-48-2

 M1 R2 Lr  F  ( a  T )  F  a = 0 700
F F
 Fy R1  2  F  R2 = 0

F  (2 a  T )
R2  R2  190.5  N
Lr
15 R2
R1 1000
R1  2  F  R2 R1  331.3  N

5. The maximum bending moment will be at FIGURE 6-48B


the right-hand load and will be Free Body Diagram of One Tube used in Problem 6-48

Mrmax  R2 Lr  ( a  T ) Mrmax  54.3 N  m

Note, if the chair were centered on the roller the maximum moment would be
Lr  T
Mc  F  Mc  39.1 N  m
2
and this would be constant along the axle between the two loads, F.

6. Note that the bearing positions are fixed regardless of the position of the chair on the roller.

Because of symmetry, 1000


65
Ra1  R1 Ra1  331.3  N R1 R2

Ra2  R2 Ra2  190.5  N

7. The maximum bending moment R a1 R a2


occurs at R1 and is for 1130
b  65 mm
FIGURE 6-48C
Mamax  Ra1 b
Free Body Diagram of One Axle used in Problem 6-48

Mamax  21.5 N  m

8. Determine a suitable axle diameter. Let the factor of safety against yielding in the axle be Nsa  3
9. Tentatively choose a low-carbon steel for the axle, say AISI 1020, cold rolled with S y  393  MPa
10. At the top of the axle under the load R1 there is only a bending stress, which is also the von Mises stress. Set
this stress equal to the yield strength divided by the factor of safety.

32 Mamax Sy
σ' = =
3 Nsa
π d a
1
3
 32 Nsa Mamax 
Solving for the axle diameter, d a d a    d a  11.875 mm
 π S y 
Let the axle diameter be d a  15 mm made from cold-rolled AISI 1020 steel.

11. Suppose that bearing 6302 from Chapter 10, Figure 10-23, page 684 is used. It has a bore of 15 mm and an OD
of 42 mm. Thus, the inside diameter of the roller away from the bearings where the moment is a maximum will
be d i  40 mm. This will provide a 1-mm shoulder for axial location of the bearings.
© 2011 Pearson Education, Inc., Upper Saddle River, NJ. All rights reserved. This publication is protected by Copyright and written permission should be
MACHINE DESIGN - An Integrated Approach, 4th Ed. 6-48-3

12. Design for a factor of safety of


Nra  3. Tentatively choose 150 700
F F
2024-T4 aluminum with

S ut  440  MPa
and F
S'e5E8  138  MPa 15
F
1000
13. A point on the outside diameter of the
roller will see completely reversed FIGURE 6-48D
bending, which will also be the only Free Body Diagram of Roller with Chair in the Center.
nonzero principal stress. Thus,

Kf  Mrmax Se
σx = = σ' =
Z Nfr

where Kf is the fatigue stress concentration at the shoulder and S e is the modified endurance limit.
14. Tentatively choose (these values arrived at by iteration):
Outside diameter d o  45 mm
Shoulder diameter D  54 mm
Fillet radius r  5  mm
15. Determine the fatigue stress concentration factor. From Figure E-2 and Table 6-6 for
r D
 0.111  1.2
do do

 0.21796
Kt  0.97098   r
 Kt  1.57
 o
d

4
S ut  6.38  10  psi

1
q  q  0.813
0.102
1
r
in

Kf  1  q   Kt  1  Kf  1.46

16. Calculate the alternating von Mises stress component.


π  4
  d o  d i 
4 4 4
I  I  7.563  10  mm
64

c  0.5 d o c  22.5 mm

Kf  Mrmax  c
σ'a  σ'a  23.6 MPa
I

© 2011 Pearson Education, Inc., Upper Saddle River, NJ. All rights reserved. This publication is protected by Copyright and written permission should be
MACHINE DESIGN - An Integrated Approach, 4th Ed. 6-48-4

17. Destermine the endurance limit at 5E8 cycles

Load factor Cload  1

 0.097
 do 
Size factor Csize  1.189    Csize  0.822
 mm 
 0.265
Surface factor  Sut 
as machined Csurf  4.51   Csurf  0.899
 MPa 

Reliability
at 90% Creliab  0.897

Modified endurance limit


S e5E8  Cload  Csize Csurf  Creliab S'e5E8 S e5E8  91.44  MPa

18. Determine the factor of safety for repeated loading.

S e5E8
Nfr  Nfr  3.87
σ'a

19. The maximum deflection of the roller will occur when the chair is in the center of the roller. For this case the
reactions are both equal to the loads, F. Using Figure B-2(a) in Appendix B, the maximum deflection is at the
center of the roller and is for

a  150  mm x  0.5 Lr x  500  mm


E  71.7 GPa

  1 
F a 3 3 
ymax  2    x  ( x  a )  ymax  0.875  mm
6  E I  
 a
L r

   a  3 a  Lr  2  Lr   x
 2 2

 rL 

This design meets the deflection requirement and has a reasonable factor of safety against fatigue failure while
allowing sufficient space for the bearings.

DESIGN SUMMARY
Axle Roller
Material AISI 1020 steel, cold-rolled Material 2024-T4 aluminum
Diameter d a  15 mm Outside diameter d o  45 mm
Length 1220 mm Inside diameter d i  40 mm
Shoulder dia D  54 mm
Fillet radius r  5  mm
Length 1040 mm
Center line spacing c   d w  d o  sin( θ )

c  238  mm

© 2011 Pearson Education, Inc., Upper Saddle River, NJ. All rights reserved. This publication is protected by Copyright and written permission should be
MACHINE DESIGN - An Integrated Approach, 4th Ed. 6-49-1

PROBLEM 6-49 _____


Statement: Figure P6-19 shows a machined pivot pin that is press-fit into part A and is slip fit in part B. If F
= 100 lb, l = 2 in, and d = 0.5 in, what is the pin's safety factor against fatigue when made of SAE
1020 cold-rolled steel? The loading is fully reversed and a reliability of 90% is desired. There is
a bending stress concentration factor Kt = 1.8 at the section where the pin leaves part A on the
right-hand side.
Given: Applied force F  100  lbf Material strength S y  57 ksi S ut  68 ksi
Total length, l l  2.00 in Beam length L  0.5 l
Pin dia d  0.5 in
Assumptions: 1. Since there is a slip fit between the pin and part B, part B offers no resistance to bending of
the pin and, since the pin is press-fit into part A, it can be modeled as a cantilever beam of
length l/2.
2. Part B distributes the concentrated force F so that, at the pin, it is uniformly distributed over
the exposed length of the pin.
Solution: See Mathcad file P0649.
1. Calculate the intensity of the uniformly distributed load acting over the length of the pin.
F lbf
w  w  100.0 
L in

2. A cantilever beam with uniform loading is shown in Figure B-1(b) in Appendix B. In this case, the dimension
a in the figure is zero. As shown in the figure, when a = 0, the maximum bending moment occurs at the
support and is

2
w L
Mmax  Mmax  50.00  lbf  in
2
3. Calculate the moment of inertia and distance to the extreme fiber of the pin. The nominal alternating bending
stress in the beam is then found using equation 4.11b.
4
π d 3 4
I  I  3.068  10  in
64
c  0.5 d c  0.250  in

Mmax c
σanom  σanom  4074 psi
I

4. From Table 6-6, the Neuber constant for S ut  68 ksi is


1
2 2
a  0.096  in a  0.096  in

5. Using equation 6-13, the notch sensitivity for r  0.5 d is

1
q  q  0.839
a
1
r

6. The fatigue stress-concentration factor for Kt  1.8, from equation 6.11b, is

Kf  1  q   Kt  1  Kf  1.67

© 2011 Pearson Education, Inc., Upper Saddle River, NJ. All rights reserved. This publication is protected by Copyright and written permission should be
MACHINE DESIGN - An Integrated Approach, 4th Ed. 6-49-2

7. Because the stress state in the pin is simple, uniaxial stress, the alternating principal stress is equal to the
alternating tensile stress and is also equal to the alternating von Mises stress. Thus,

σa  Kf  σanom σa  6.81 ksi

σ'  σa

8. Calculate the unmodified endurance limit. S'e  0.5 S ut S'e  34.0 ksi

9. Calculate the endurance limit modification factors for a non rotating round pin in bending.
Load Cload  1

2 3 2
Size A95  0.010462 d A95  2.615  10  in

A95
d equiv  d equiv  0.185  in
0.0766

This is less than the lower limit in equation 6.7b, so use


Csize  1
Surface A  2.7 b  0.265 (machined)

b
 S ut 
Csurf  A    Csurf  0.883
 ksi 
Temperature Ctemp  1

Reliability Creliab  0.897 (R = 90%)

10. Calculate the modified endurance limit.


S e  Cload  Csize Csurf  Ctemp Creliab S'e S e  26.9 ksi

11. Using equation 6.14, calculate the factor of safety against a fatigue failure for this case of fully reversed
bending.

Se
Nf  Nf  4.0
σ'

© 2011 Pearson Education, Inc., Upper Saddle River, NJ. All rights reserved. This publication is protected by Copyright and written permission should be
MACHINE DESIGN - An Integrated Approach, 4th Ed. 6-50-1

PROBLEM 6-50 _____


Statement: Figure P6-19 shows a machined pivot pin that is press-fit into part A and is slip fit in part B. If F
= 100 N, l = 50 mm, and d = 16 mm, what is the pin's safety factor against fatigue when made of
class 50 cast iron? The loading is fully reversed and a reliability of 90% is desired. There is a
bending stress concentration factor Kt = 1.8 at the section where the pin leaves part A on the
right-hand side.

Given: Applied force F  100  N Tensile strength S ut  359  MPa


Total length, l l  50 mm Beam length L  0.5 l Pin dia d  16 mm

Assumptions: 1. Since there is a slip fit between the pin and part B, part B offers no resistance to bending of
the pin and, since the pin is press-fit into part A, it can be modeled as a cantilever beam of
length l/2.
2. Part B distributes the concentrated force F so that, at the pin, it is uniformly distributed over
the exposed length of the pin.

Solution: See Mathcad file P0650.


1. Calculate the intensity of the uniformly distributed load acting over the length of the pin.
F N
w  w  4.0
L mm

2. A cantilever beam with uniform loading is shown in Figure B-1(b) in Appendix B. In this case, the dimension
a in the figure is zero. As shown in the figure, when a = 0, the maximum bending moment occurs at the
support and is
2
w L
Mmax  Mmax  1250 N  mm
2

3. Calculate the moment of inertia and distance to the extreme fiber of the pin. The nominal alternating bending
stress in the beam is then found using equation 4.11b.
4
π d 3 4
I  I  3.217  10  mm
64

c  0.5 d c  8.000  mm

Mmax c
σanom  σanom  3.108  MPa
I

4. Since this is a brittle material, so the full value of the geometric stress concentration factor Kt  1.8 will be
applied to the nominal stress using equation 4.31.

5. Because the stress state in the pin is simple, uniaxial stress, the alternating principal stress is equal to the
alternating tensile stress and is also equal to the alternating von Mises stress. Thus,

σa  Kt σanom σa  5.60 MPa

σ'  σa

6. Calculate the unmodified endurance limit. S'e  0.5 S ut S'e  179.5  MPa

7. Calculate the endurance limit modification factors for a non rotating round pin in bending.
Load Cload  1
© 2011 Pearson Education, Inc., Upper Saddle River, NJ. All rights reserved. This publication is protected by Copyright and written permission should be
MACHINE DESIGN - An Integrated Approach, 4th Ed. 6-50-2

2 2
Size A95  0.010462 d A95  2.678  mm

A95
d equiv  d equiv  5.913  mm
0.0766

This is less than the lower limit in equation 6.7b, so use


Csize  1
Surface A  4.51 b  0.265 (machined)

b
 Sut 
Csurf  A    Csurf  0.949
 MPa 
Temperature Ctemp  1

Reliability Creliab  0.897 (R = 90%)

8. Calculate the modified endurance limit.


S e  Cload  Csize Csurf  Ctemp Creliab S'e S e  152.7  MPa

9. Using equation 6.14, calculate the factor of safety against a fatigue failure for this case of fully reversed bending

Se
Nf  Nf  27
σ'

© 2011 Pearson Education, Inc., Upper Saddle River, NJ. All rights reserved. This publication is protected by Copyright and written permission should be
MACHINE DESIGN - An Integrated Approach, 4th Ed. 6-51-1

PROBLEM 6-51 _____


Statement: A component in the shape of a large sheet is to be fabricated from 7075-T651 aluminum, which
has a fracture toughness Kc = 24.2 MPa-m0.5 and a tensile yield strength of 495 MPa. Determine
the number of loading cycles that can be endured if the nominal stress varies from 0 to one half
the yield strength and the initial crack had a total length of 1.2 mm. The values of the coefficient
and exponent in equation 6.4 for this material are A = 5 x 10 -11 (mm/cyc) and n = 4.

Units: cycle  1
0.5
Given: Fracture toughness Kc  24.2 MPa m
Yield strength S y  495  MPa
Initial crack length lo  1.2 mm
 11 1
Coeff. and exponent A  5  10  mm cycle n  4
Solution: See Mathcad file P0651.
1. Calculate the minimum and maximum nominal stresses based on the yield strength and the stress level given in
the problem statement.

σmin  0  MPa

Sy
σmax  σmax  247.5 MPa
2

2. Determine the value of the geometry factor  from the discussion in Section 5.3 for a plate with a central crack.

β  1
3. Using equation 5.14b, calculate the critical crack length for this material at the maximum stress condition.

2
1 Kc 
a c     a c  3.0 mm
π  β  σmax 

4. Calculate the initial crack half-length.

a o  0.5 l o a o  0.60 mm

5. Using equations 6.3, write the stress intensity factor range as a function of crack half-length.

∆K ( a )  β  π a   σmax  σmin

6. Integrate equation 6.4 to find the number of cycles to failure.

ac
1 
Nc   
1 5
da Nc  7.2  10 cycle
A  4
  ∆K ( a ) 
  0.5 
  MPa m 
 ao

© 2011 Pearson Education, Inc., Upper Saddle River, NJ. All rights reserved. This publication is protected by Copyright and written permission should be
MACHINE DESIGN - An Integrated Approach, 4th Ed. 6-52-1

PROBLEM 6-52 _____


Statement: A component in the shape of a large sheet is to be fabricated from 4340 steel, which has a fracture
toughness Kc = 98.9 MPa-m0.5. The sheets are inspected for crack flaws after fabrication, but the
inspection device cannot detect flaws smaller than 5 mm. Determine the minimum thickness
required for the sheet to have a minimum cycle life of 10 6 cycles (using fracture-mechanics criteria
if its width is 400 mm and the load normal to the crack varies from 20 to 170 kN. The values of th
coefficient and exponent in equation 6.4 for this material are A = 4 x 10 -9 (mm/cyc) and n = 3.

Units: cycle  1
0.5
Given: Fracture toughness Kc  98.9 MPa m
Width of sheet W  400  mm
Load Fmin  20 kN Fmax  170  kN
9 1
Coeff. and exponent A  4  10  mm cycle n  3
6
Cycles to failure Nf  10  cycle

Assumption: The initial total crack length is lo  5  mm


Solution: See Mathcad file P0652.
1. Write equations for the minimum and maximum nominal stresses as a function of the unknown thickness.

Fmin Fmax
σmin( t)  σmax( t) 
W t W t

2. Determine the value of the geometry factor  from the discussion in Section 5.3 for a plate with a central crack.
β  1

3. Using equation 5.14b, write an equation for the critical crack length as a function of t for this material at the
maximum stress condition.

2
1  Kc 
a c( t)    
π  β  σmax( t) 

4. Calculate the initial crack half-length.


a o  0.5 l o a o  2.50 mm
5. Using equations 6.3, write the stress intensity factor range as a function of crack half-length and sheet thickness

∆K ( a t)  β  π a   σmax( t)  σmin( t) 

6. Use equation 6.4 to find the minimum sheet thickness. First, guess a value: t  4  mm

ac( t )

Nf  A = 
1
Given da t  Find ( t) t  3.2 mm
 n
  ∆K ( a t) 
  0.5 
  MPa m 

ao

© 2011 Pearson Education, Inc., Upper Saddle River, NJ. All rights reserved. This publication is protected by Copyright and written permission should be
MACHINE DESIGN - An Integrated Approach, 4th Ed. 6-53-1

PROBLEM 6-53 _____

Statement: A closed, thin-wall cylinder is made from an aluminum alloy that has a fracture toughness of 38
MPa-m0.5 and has the following dimensions: length = 200 mm, OD = 84 mm, and ID = 70 mm. A
2.8-mm-deep semicircular crack is discovered on the inner diameter away from the ends, oriented
along a line parallel to the cylinder axis. If the cylinder is repeatedly pressurized from 0 to 75
MPa, how many pressure cycles can it withstand? The values of the coefficient and exponent in
equation 6.4 for this material are A = 5 x 10 -12 (mm/cyc) and n = 4. (Hint: the value of the
geometry factor for a semicircular surface flaw is  = 2/and the crack grows in the radial
direction).

Units: cycle  1
0.5
Given: Fracture toughness Kc  38 MPa m
Initial crack depth a o  2.8 mm
Cylinder dimensions L  200  mm OD  84 mm ID  70 mm
Internal pressure p min  0  MPa p max  75 MPa
 12 1
Coeff. and exponent A  5  10  mm cycle n  4
Geometry factor β  0.6367
Solution: See Mathcad file P0653.

1. Calculate the nominal hoop stress (tangential direction, normal to cylinder axis) using equation 4.49a based on
the pressure levels given in the problem statement.

r  0.5 ID r  35.0 mm t  0.5 ( OD  ID) t  7.0 mm


r
σmin  p min  σmin  0.0 MPa
t
r
σmax  p max  σmax  375.0 MPa
t

2. Using equation 5.14b, calculate the critical crack length for this material at the maximum stress condition.
2
1  Kc 
a c     a c  8.1 mm
π  β  σmax 

However, since this is larger than the wall thickness, failure will occur when the crack reaches the OD so
a c  t a c  7.0 mm

3. Using equations 6.3, write the stress intensity factor range as a function of crack depth.

∆K ( a )  β  π a   σmax  σmin

4. Integrate equation 6.4 to find the number of cycles to failure.

ac
1 
Nc   
1 6
da Nc  1.34  10 cycle
A  4
  ∆K ( a ) 
  0.5 
  MPa m 
 ao

© 2011 Pearson Education, Inc., Upper Saddle River, NJ. All rights reserved. This publication is protected by Copyright and written permission should be
MACHINE DESIGN - An Integrated Approach, 4th Ed. 6-54-1

PROBLEM 6-54 _____

Statement: A non rotating, hot-rolled, steel beam has a channel section with h = 64 mm and b = 127 mm. It is
loaded in repeated bending with the neutral axis through the web. Determine its corrected fatigue
strength with 90% reliability if it is used in an environment that has a temperature that is below
450C and has an ultimate tensile strength of 320 MPa.

Given: Ultimate tensile strength S ut  320  MPa


Reliability R  0.90
Dimensions h  64 mm b  127  mm

Solution: See Mathcad file P0654.

1. Calculate the uncorrected endurance limit using equation 6.5a.


S'e  0.5 S ut S'e  160.0 MPa

2. Determine the loading factor from equation 6.7a.

Cload  1

3. Determine the size factor from equations 6.7b and 6.7d, and Figure 6-25.
2
Area stressed above 95% of max A95  0.05 b  h A95  406.4 mm

A95
Equivalent diameter d equiv  d equiv  72.8 mm
0.0766
 0.097
 d equiv 
Size factor Csize  1.189    Csize  0.784
 mm 

4. Determine the surface factor from equation 6.7e and Table 6-3.
From Table 6-3 A  57.7 b  0.718

b
 Sut 
Surface factor Csurf  A    Csurf  0.917
 MPa 

5. Determine the temperature factor from equation 6.7f. Since T < 450C, Ctemp  1.

6. Determine the reliability factor from Table 6-4, Creliab  0.897.

7. Using equation 6.6, calculate the corrected endurance limit.


S e  Cload  Csize Csurf  Ctemp Creliab S'e S e  103.3 MPa

© 2011 Pearson Education, Inc., Upper Saddle River, NJ. All rights reserved. This publication is protected by Copyright and written permission should be
MACHINE DESIGN - An Integrated Approach, 4th Ed. 6-55-1

PROBLEM 6-55 _____

Statement: A non rotating, machined, steel rod has a round section with d = 50 mm. It is loaded with a
fluctuating axial force. Determine its corrected fatigue strength with 99% reliability if it is used in
an environment that has a temperature below 450C and has an ultimate tensile strength of 480
MPa.

Given: Ultimate tensile strength S ut  480  MPa


Reliability R  0.99
Dimensions d  50 mm

Solution: See Mathcad file P0655.

1. Calculate the uncorrected endurance limit using equation 6.5a.


S'e  0.5 S ut S'e  240.0 MPa

2. Determine the loading factor from equation 6.7a.

Cload  0.70

3. The size factor for an axially loaded member is Csize  1

4. Determine the surface factor from equation 6.7e and Table 6-3.
From Table 6-3 A  4.51 b  0.265

b
 Sut 
Surface factor Csurf  A    Csurf  0.878
 MPa 

5. Determine the temperature factor from equation 6.7f. Since T < 450C, Ctemp  1.

6. Determine the reliability factor from Table 6-4, Creliab  0.814.

7. Using equation 6.6, calculate the corrected endurance limit.


S e  Cload  Csize Csurf  Ctemp Creliab S'e S e  120.1 MPa

© 2011 Pearson Education, Inc., Upper Saddle River, NJ. All rights reserved. This publication is protected by Copyright and written permission should be
MACHINE DESIGN - An Integrated Approach, 4th Ed. 6-56-1

PROBLEM 6-56 _____

Statement: A non rotating, cold-drawn, steel rod has a round section with d = 76 mm. It is loaded in
repeated torsion. Determine its corrected fatigue strength with 99% reliability if it is used in an
environment that has a temperature of 500C and has an ultimate tensile strength of 855 MPa.

Given: Ultimate tensile strength S ut  855  MPa


Reliability R  0.99
Dimensions d  76 mm
Temperature T  500  C

Solution: See Mathcad file P0656.

1. Calculate the uncorrected endurance limit using equation 6.5a.


S'e  0.5 S ut S'e  427.5  MPa

2. Determine the loading factor from equation 6.7a.

Cload  1

3. Determine the size factor from equation 6.7b.


 0.097
Csize  1.189   
d
Size factor  Csize  0.781
 mm 

4. Determine the surface factor from equation 6.7e and Table 6-3.
From Table 6-3 A  4.51 b  0.265

b
 Sut 
Surface factor Csurf  A    Csurf  0.754
 MPa 

5. Determine the temperature factor from equation 6.7f.


T  450  C 
Ctemp  1  0.0058   Ctemp  0.710
 C 

6. Determine the reliability factor from Table 6-4, Creliab  0.814.

7. Using equation 6.6, calculate the corrected endurance limit.


S e  Cload  Csize Csurf  Ctemp Creliab S'e S e  145.5  MPa

© 2011 Pearson Education, Inc., Upper Saddle River, NJ. All rights reserved. This publication is protected by Copyright and written permission should be
MACHINE DESIGN - An Integrated Approach, 4th Ed. 6-57-1

PROBLEM 6-57 _____

Statement: A non rotating, ground, steel rod has a rectangular section with h = 60 mm and b = 40 mm. It is
loaded in repeated bending. Determine its corrected fatigue strength with 99.9% reliability if it is
used in an environment that has a temperature that is below 450C and has an ultimate tensile
strength of 1550 MPa.

Given: Ultimate tensile strength S ut  1550 MPa


Reliability R  0.999
Dimensions h  60 mm b  40 mm

Solution: See Mathcad file P0657.

1. Calculate the uncorrected endurance limit using equation 6.5a (S ut exceeds 1400 MPa).

S'e  700  MPa

2. Determine the loading factor from equation 6.7a.

Cload  1

3. Determine the size factor from equations 6.7b and 6.7d, and Figure 6-25.
2
Area stressed above 95% of max A95  0.05 b  h A95  120.0 mm

A95
Equivalent diameter d equiv  d equiv  39.6 mm
0.0766
 0.097
 d equiv 
Size factor Csize  1.189    Csize  0.832
 mm 

4. Determine the surface factor from equation 6.7e and Table 6-3.
From Table 6-3 A  1.58 b  0.085

b
 Sut 
Surface factor Csurf  A    Csurf  0.846
 MPa 

5. Determine the temperature factor from equation 6.7f. Since T < 450C, Ctemp  1.

6. Determine the reliability factor from Table 6-4, Creliab  0.753.

7. Using equation 6.6, calculate the corrected endurance limit.


S e  Cload  Csize Csurf  Ctemp Creliab S'e S e  371.2 MPa

© 2011 Pearson Education, Inc., Upper Saddle River, NJ. All rights reserved. This publication is protected by Copyright and written permission should be
MACHINE DESIGN - An Integrated Approach, 4th Ed. 6-58-1

PROBLEM 6-58 _____


Statement: A steel, grooved shaft similar to that shown in Figure C-5 (Appendix C) is to be loaded in bending
Its dimensions are: D = 57 mm, d = 38 mm, r = 3 mm. Determine the fatigue stress- concentration
factor if the material S ut = 1130 MPa.

Given: Dimensions: D  57 mm d  38 mm r  3  mm


Tensile strength S ut  1130 MPa

Solution: See Figure C-5 and Mathcad file P0658.

1. The geometric stress-concentration factor is found from the equation in Figure C-5.
D
For  1.500 A  0.93894 b  0.32380 and
d
b
Kt  A  
r
 Kt  2.14
d

2. The Neuber constant is found by linear interpolation of the values in Table 6-6.
S ut  163.9  ksi

2 2
a 1  0.031  in S 1  160  ksi a 2  0.024  in S 2  180  ksi

S ut  S 2
  a 1  a 2  a 2
0.5
a  a  0.030  in
S1  S2

3. Calculate the notch sensitivity using equation 6.13.

1
q  q  0.920
a
1
r

4. The fatigue stress-concentration factor can now be found from equation 6.11b.

Kf  1  q   Kt  1  Kf  2.05

© 2011 Pearson Education, Inc., Upper Saddle River, NJ. All rights reserved. This publication is protected by Copyright and written permission should be
MACHINE DESIGN - An Integrated Approach, 4th Ed. 6-59-1

PROBLEM 6-59 _____


Statement: A steel shaft with a transverse hole similar to that shown in Figure C-8 (Appendix C) is to be
loaded in torsion. Its dimensions are: D = 32 mm, d = 3 mm. Determine the fatigue stress-
concentration factor if the material S ut = 808 MPa.

Given: Dimensions: D  32 mm d  3  mm


Tensile strength S ut  808  MPa

Solution: See Figure C-8 and Mathcad file P0659.

1. The geometric stress-concentration factor is found from an equation in Figure C-8. Although the maximum
torsional stress is on the surface, it will be almost a maximum just below the surface so it will be conservative
to use curve B.

2 3
 27.159 
d
 30.231   
d d
Kt  3.9702  9.292   Kt  3.34
D  D  D
4 5 6
 393.19    650.39    15.451  
d d d
 D  D  D
2. The Neuber constant is found by linear interpolation of the values in Table 6-6. However, since the loading is
torsional, 20 ksi must be added to the value of S ut that is used in the table (see the text in Figure 6-36, Part 1).

S utt  S ut  20 ksi S utt  137.2  ksi


2 2
a 1  0.044  in S 1  130  ksi a 2  0.039  in S 2  140  ksi

S utt  S 2
  a 1  a 2  a 2
0.5
a  a  0.040  in
S1  S2

3. Calculate the notch sensitivity using equation 6.13. Let r  0.5 d

1
q  q  0.857
a
1
r

4. The fatigue stress-concentration factor can now be found from equation 6.11b.

Kf  1  q   Kt  1  Kf  3.00

© 2011 Pearson Education, Inc., Upper Saddle River, NJ. All rights reserved. This publication is protected by Copyright and written permission should be
MACHINE DESIGN - An Integrated Approach, 4th Ed. 6-60-1

PROBLEM 6-60 _____

Statement: A hardened aluminum filleted flat bar similar to that shown in Figure C-9 (Appendix C) is to be
loaded axially. Its dimensions are: D = 1.20 in, d = 1.00 in, r = 0.10 in. Determine the fatigue
stress-concentration factor if the material S ut = 76 ksi.

Given: Dimensions: D  1.20 in d  1.00 in r  0.100  in


Tensile strength S ut  76 ksi

Solution: See Figure C-9 and Mathcad file P0660.

1. The geometric stress-concentration factor is found from the equation in Figure C-9.
D
For  1.200 A  1.03510 b  0.25084 and
d
b
Kt  A  
r
 Kt  1.84
d

2. The Neuber constant is found by linear interpolation of the values in Table 6-8.

2 2
a 1  0.144  in S 1  70 ksi a 2  0.131  in S 2  80 ksi

S ut  S 2
  a 1  a 2  a 2
0.5
a  a  0.136  in
S1  S2

3. Calculate the notch sensitivity using equation 6.13.

1
q  q  0.699
a
1
r

4. The fatigue stress-concentration factor can now be found from equation 6.11b.

Kf  1  q   Kt  1  Kf  1.59

© 2011 Pearson Education, Inc., Upper Saddle River, NJ. All rights reserved. This publication is protected by Copyright and written permission should be
MACHINE DESIGN - An Integrated Approach, 4th Ed. 6-61-1

PROBLEM 6-61
Statement: A rotating shaft with a shoulder fillet seated in the inner race of a rolling contact bearing with the
shoulder against the edge of the bearing is shown in Figure P6-20. The bearing has a slight
eccentricity that induces a fully reversed bending moment in the shaft as it rotates.
Measurements indicate that the resulting alternating stress amplitude due to bending is a = 57
MPa. The torque on the shaft fluctuates from a high of 90 N-m to a low of 12 N-m and is in
phase with the bending stress. The shaft is ground and its dimensions are: D = 23 mm, d = 19
mm, and r = 1.6 mm. The shaft material is SAE 1040 cold-rolled steel. Determine the infinite-life
fatigue safety factor for the shaft for a reliability of 99%.

Given: Strength SAE 1040 CR S ut  586  MPa Alternating bending stress σxa  57 MPa
Fluctuating torque Tmax  90 N  m Tmin  12 N  m
Shaft dimensions D  23 mm d  19 mm r  1.6 mm

Solution: See Figure P6-20 and Mathcad file P0661.

1. Determine the mean and alternating components of the fluctuating torsional stress.
d
Distance to outside fiber c  c  9.5 mm
2
4
π d 4 4
Polar moment of inertia J  J  1.279  10  mm
32

Tmax c
Torsional stress τxymax  τxymax  66.827 MPa
J
Tmin c
τxymin  τxymin  8.91 MPa
J
τxymax  τxymin
τxym  τxym  37.869 MPa
2
τxymax  τxymin
τxya  τxya  28.958 MPa
2

2. Using Appendix C, determine the geometric stress concentration factors for the bending and torsional stresses.
D r
Bending (Fig. C-2): For  1.211  0.084 A  0.97098 b  0.21796
d d

b
Kt  A  
r
 Kt  1.665
d
D r
Torsion (Fig. C-3): For  1.211  0.084 A  0.83425 b  0.21649
d d

b
Kts  A  
r
 Kts  1.425
d

3. Calculate the notch sensitivity of the material for bending and torsion using Table 6-6.

© 2011 Pearson Education, Inc., Upper Saddle River, NJ. All rights reserved. This publication is protected by Copyright and written permission should be
MACHINE DESIGN - An Integrated Approach, 4th Ed. 6-61-2

Bending:
2
Neuber constant (for S ut  586.0  MPa) a  0.075  in
1
Notch sensitivity q b  q b  0.77
a
1
r
Torsion:
2
Neuber constant (for S ut  20 MPa  606.0  MPa) a  0.0585  in
1
Notch sensitivity q s  q s  0.811
a
1
r

4. Calculate the fatigue stress concentration factors for bending and torsion using equation 6.11b.

Bending Kf  1  q b  Kt  1  Kf  1.512

Torsion Kfs  1  q s  Kts  1  Kfs  1.345

5. Determine what, if any, fatigue stress concentration factor should be applied to the mean torsional stress.

Yield strength SAE 1040 CR S y  490  MPa

Evaluate Kfs  2  τxymax  179.8  MPa which is less than S y so

Kfsm  Kfs

6. Calculate the mean and alternating components of the stresses increased by the appropriate fatigue stress
concentration factors.

Bending σm  0  MPa σa  Kf  σxa σa  86.188 MPa

Torsion τm  Kfsm τxym τm  50.934 MPa

τa  Kfs τxya τa  38.949 MPa

7. Find the mean and alternating von Mises stresses using equations 6.22b (with y = 0).

2
Mean σ'm  3  τm σ'm  88.22  MPa

2 2
Alternating σ'a  σa  3  τa σ'a  109.451  MPa

8. Calculate the unmodified endurance limit.


S'e  0.5 S ut S'e  293  MPa

9. Calculate the endurance limit modification factors for a rotating, round shaft.
Load Cload  1 (combined bending and torsion)

© 2011 Pearson Education, Inc., Upper Saddle River, NJ. All rights reserved. This publication is protected by Copyright and written permission should be
MACHINE DESIGN - An Integrated Approach, 4th Ed. 6-61-3

 0.097
Csize  1.189   d
Size  Csize  0.894
 
mm

Surface A  1.58 b  0.085 (ground)

b
 Sut 
Csurf  A    Csurf  0.919
 MPa 
Temperature Ctemp  1

Reliability Creliab  0.814 (R = 99%)

10. Calculate the modified endurance limit.


S e  Cload  Csize Csurf  Ctemp Creliab S'e S e  196  MPa

11. Assuming a Case 2 load line, determine the factor of safety against fatigue failure.

Se  σ'm 
Nf  1   Nf  1.5
σ'a  S ut 

© 2011 Pearson Education, Inc., Upper Saddle River, NJ. All rights reserved. This publication is protected by Copyright and written permission should be
MACHINE DESIGN - An Integrated Approach, 4th Ed. 6-62-1

PROBLEM 6-62
Statement: A tension member in a machine is filleted as shown in Figure P6-21. The member has a
manufacturing defect that causes the fluctuating tension load to be applied eccentrically
resulting in a fluctuating bending load as well. Measurements indicate that the maximum
bending stress is 16.4 MPa and the minimum is 4.1 MPa. The tensile load fluctuates from a high
of 3.6 kN to a low of 0.90 kN and is in phase with the bending stress. The member is machined
and its dimensions are: D = 33 mm, d = 25 mm, h = 3 mm and r = 3 mm. The material is SAE 1020
cold-rolled steel. Determine the infinite-life fatigue safety factor for the member for a reliability
of 90%.

Given: Strength SAE 1020 CR S ut  380  MPa σxmax  16.4 MPa σxmin  4.1 MPa
Fluctuating tension Fmax  3.6 kN Fmin  0.90 kN
Dimensions D  33 mm d  25 mm h  3  mm r  3  mm

Solution: See Figure P6-21 and Mathcad file P0662.

1. Determine the mean and alternating components of the fluctuating stresses.

σxmax  σxmin
Bending σmb  σmb  10.250 MPa
2
σxmax  σxmin
σab  σab  6.150  MPa
2
Fmax
Tension σxtmax  σxtmax  48.000 MPa
h d

Fmin
σxtmin  σxtmin  12.000 MPa
h d
σxtmax  σxtmin
σmt  σmt  30.000 MPa
2
σxtmax  σxtmin
σat  σat  18.000 MPa
2

2. Using Appendix C, determine the geometric stress concentration factors for the bending and tensile stresses.
D r
Bending (Fig. C-10): For  1.32  0.12 A  0.95880 b  0.27269
d d

b
Ktb  A  
r
 Ktb  1.709
d
D r
Tension (Fig. C-9): For  1.32  0.12 A  1.05440 b  0.27021
d d

b
Ktt  A  
r
 Ktt  1.87
d

3. Calculate the notch sensitivity of the material for bending and tension using Table 6-6.
© 2011 Pearson Education, Inc., Upper Saddle River, NJ. All rights reserved. This publication is protected by Copyright and written permission should be
MACHINE DESIGN - An Integrated Approach, 4th Ed. 6-62-2

Bending and tension:


2
Neuber constant (for S ut  55.1 ksi) a  0.118  in
1
Notch sensitivity q  q  0.744
a
1
r

4. Calculate the fatigue stress concentration factors for bending and tension using equation 6.11b.

Bending Kfb  1  q   Ktb  1  Kfb  1.528

Tension Kft  1  q   Ktt  1  Kft  1.648

5. Determine what, if any, fatigue stress concentration factor should be applied to the mean stresses.

Yield strength SAE 1040 CR S y  207  MPa

Evaluate Kft   σxmax  σxtmax  106.1  MPa which is less than S y so

Kfbm  Kfb Kftm  Kft

6. Calculate the mean and alternating components of the stresses increased by the appropriate fatigue stress
concentration factors.

Bending σmb  Kfb σmb σmb  15.662 MPa

σab  Kfb σab σab  9.397  MPa

Tension σmt  Kftm σmt σmt  49.427 MPa

σat  Kft  σat σat  29.656 MPa

7. Find the mean and alternating von Mises stresses using equations 6.22b (with y = 0).

Mean σ'm  σmb  σmt σ'm  65.089 MPa

Alternating σ'a  σab  σmb σ'a  25.06  MPa

8. Calculate the unmodified endurance limit.


S'e  0.5 S ut S'e  190  MPa

9. Calculate the endurance limit modification factors for a rotating, round shaft.
Load Cload  0.70

Size Csize  1

Surface A  4.51 b  0.265 (machined)

b
 Sut 
Csurf  A    Csurf  0.934
 MPa 
© 2011 Pearson Education, Inc., Upper Saddle River, NJ. All rights reserved. This publication is protected by Copyright and written permission should be
MACHINE DESIGN - An Integrated Approach, 4th Ed. 6-62-3

Temperature Ctemp  1

Reliability Creliab  0.897 (R = 90%)

10. Calculate the modified endurance limit.


S e  Cload  Csize Csurf  Ctemp Creliab S'e S e  111.48 MPa

11. Assuming a Case 3 load line, determine the factor of safety against fatigue failure.

S e S ut
Nf  Nf  2.5
σ'a S ut  σ'm S e

© 2011 Pearson Education, Inc., Upper Saddle River, NJ. All rights reserved. This publication is protected by Copyright and written permission should be
MACHINE DESIGN - An Integrated Approach, 4th Ed. 6-63a-1

PROBLEM 6-63a
Statement: For a filleted flat bar in tension similar to that shown in Appendix Figure C-9 and the data from
row a from Table P6-7, determine the alternating and mean axial stresses as modified by the
appropriate stress concentration factors in the bar.

Given: Strength SAE 1020 CR S ut  469  MPa


Widths D  40 mm d  20 mm
Thickness h  10 mm Radius r  4  mm
Force Pmin  8000 N Pmax  32000  N

Solution: See Table P6-7 and Mathcad file P0663a.

1. Determine the nominal mean and alternating components of the fluctuating stresses.

Pmax
σxmax  σxmax  160.0  MPa
h d
Pmin
σxmin  σxmin  40.0 MPa
h d

σxmax  σxmin
σxm  σxm  100.0  MPa
2
σxmax  σxmin
σxa  σxa  60.0 MPa
2

2. Using Appendix C-9, determine the geometric stress concentration factor.

D r
For 2  0.2 A  1.09960 b  0.32077
d d

b
Kt  A  
r
 Kt  1.843
d
3. Calculate the notch sensitivity of the material using Table 6-6.

2
Neuber constant (for S ut  68 ksi) a  0.096  in
1
Notch sensitivity q  q  0.805
a
1
r

4. Calculate the fatigue stress concentration factor using equation 6.11b.

Kf  1  q   Kt  1  Kf  1.679

5. Determine what, if any, fatigue stress concentration factor should be applied to the mean stress.

Yield strength SAE 1020 CR S y  393  MPa

Evaluate

© 2011 Pearson Education, Inc., Upper Saddle River, NJ. All rights reserved. This publication is protected by Copyright and written permission should be
MACHINE DESIGN - An Integrated Approach, 4th Ed. 6-63a-2

Kfm  S 1  Kf  σxmax
S 2  Kf  σxmax  σxmin
return Kf if S 1  S y
S y  Kf  σxa
return if  S 1  S y  S 2  2  S y
σxm
0 otherwise

Kfm  1.679

6. Calculate the mean and alternating components of the stresses increased by the appropriate fatigue stress
concentration factors.

σm  Kfm σxm σm  167.9  MPa

σa  Kf  σxa σa  100.7  MPa

© 2011 Pearson Education, Inc., Upper Saddle River, NJ. All rights reserved. This publication is protected by Copyright and written permission should be
MACHINE DESIGN - An Integrated Approach, 4th Ed. 6-64a-1

PROBLEM 6-64a
Statement: For a filleted flat bar in bending similar to that shown in Appendix Figure C-10 and the data from
row a from Table P6-7, determine the alternating and mean bending stresses as modified by the
appropriate stress concentration factors in the bar.

Given: Strength SAE 1020 CR S ut  469  MPa


Widths D  40 mm d  20 mm
Thickness h  10 mm Radius r  4  mm
Moment Mmin  80 N  m Mmax  320  N  m

Solution: See Table P6-7 and Mathcad file P0664a.

1. Determine the nominal mean and alternating components of the fluctuating stresses.
3
d h d 3 4
c  c  10 mm I  I  6.667  10  mm
2 12
Mmin c
σxmin  σxmin  120  MPa
I
Mmax c
σxmax  σxmax  480  MPa
I

σxmax  σxmin
σxm  σxm  300.0  MPa
2
σxmax  σxmin
σxa  σxa  180.0  MPa
2

2. Using Appendix C-10, determine the geometric stress concentration factor.

D r
For 2  0.2 A  0.93232 b  0.30304
d d

b
Kt  A  
r
 Kt  1.518
d
3. Calculate the notch sensitivity of the material using Table 6-6.

2
Neuber constant (for S ut  68 ksi) a  0.096  in
1
Notch sensitivity q  q  0.805
a
1
r

4. Calculate the fatigue stress concentration factor using equation 6.11b.

Kf  1  q   Kt  1  Kf  1.417

5. Determine what, if any, fatigue stress concentration factor should be applied to the mean stresses.

Yield strength SAE 1020 CR S y  393  MPa

© 2011 Pearson Education, Inc., Upper Saddle River, NJ. All rights reserved. This publication is protected by Copyright and written permission should be
MACHINE DESIGN - An Integrated Approach, 4th Ed. 6-64a-2

Evaluate
Kfm  S 1  Kf  σxmax
S 2  Kf  σxmax  σxmin
return Kf if S 1  S y
S y  Kf  σxa
return if  S 1  S y  S 2  2  S y
σxm
0 otherwise

Kfm  0.460

6. Calculate the mean and alternating components of the stresses increased by the appropriate fatigue stress
concentration factors.

σm  Kfm σxm σm  137.9  MPa

σa  Kf  σxa σa  255.1  MPa

© 2011 Pearson Education, Inc., Upper Saddle River, NJ. All rights reserved. This publication is protected by Copyright and written permission should be
MACHINE DESIGN - An Integrated Approach, 4th Ed. 6-65a-1

PROBLEM 6-65a
Statement: For a shaft, with a shoulder fillet, in tension similar to that shown in Appendix Figure C-1 and the
data from row a from Table P6-7, determine the alternating and mean axial stresses as modified by
the appropriate stress concentration factors in the shaft.

Given: Strength SAE 1020 CR S ut  469  MPa


Widths D  40 mm d  20 mm
Thickness h  10 mm Radius r  4  mm
Force Pmin  8000 N Pmax  32000  N

Solution: See Table P6-7 and Mathcad file P0663a.

1. Determine the nominal mean and alternating components of the fluctuating stresses.

4  Pmax
σxmax  σxmax  101.9  MPa
2
π d
4  Pmin
σxmin  σxmin  25.5 MPa
2
π d
σxmax  σxmin
σxm  σxm  63.7 MPa
2
σxmax  σxmin
σxa  σxa  38.2 MPa
2

2. Using Appendix C-1, determine the geometric stress concentration factor.

D r
For 2  0.2 A  1.01470 b  0.30035
d d

b
Kt  A  
r
 Kt  1.645
d
3. Calculate the notch sensitivity of the material using Table 6-6.

2
Neuber constant (for S ut  68 ksi) a  0.096  in
1
Notch sensitivity q  q  0.805
a
1
r

4. Calculate the fatigue stress concentration factor using equation 6.11b.

Kf  1  q   Kt  1  Kf  1.52

5. Determine what, if any, fatigue stress concentration factor should be applied to the mean stress.

Yield strength SAE 1020 CR S y  393  MPa

© 2011 Pearson Education, Inc., Upper Saddle River, NJ. All rights reserved. This publication is protected by Copyright and written permission should be
MACHINE DESIGN - An Integrated Approach, 4th Ed. 6-65a-2

Evaluate Kfm  S 1  Kf  σxmax


S 2  Kf  σxmax  σxmin
return Kf if S 1  S y
S y  Kf  σxa
return if  S 1  S y  S 2  2  S y
σxm
0 otherwise

Kfm  1.520

6. Calculate the mean and alternating components of the stresses increased by the appropriate fatigue stress
concentration factors.

σm  Kfm σxm σm  96.7 MPa

σa  Kf  σxa σa  58.0 MPa

© 2011 Pearson Education, Inc., Upper Saddle River, NJ. All rights reserved. This publication is protected by Copyright and written permission should be
MACHINE DESIGN - An Integrated Approach, 4th Ed. 6-66a-1

PROBLEM 6-66a
Statement: For a shaft, with a shoulder fillet, in bending similar to that shown in Appendix Figure C-2 and
the data from row a from Table P6-7, determine the alternating and mean bending stresses as
modified by the appropriate stress concentration factors in the shaft.

Given: Strength SAE 1020 CR S ut  469  MPa


Widths D  40 mm d  20 mm
Thickness h  10 mm Radius r  4  mm
Moment Mmin  80 N  m Mmax  320  N  m

Solution: See Table P6-7 and Mathcad file P0666a.

1. Determine the nominal mean and alternating components of the fluctuating stresses.
4
d π d 3 4
c  c  10 mm I  I  7.854  10  mm
2 64
Mmin c
σxmin  σxmin  101.859  MPa
I
Mmax c
σxmax  σxmax  407.437  MPa
I

σxmax  σxmin
σxm  σxm  254.6  MPa
2
σxmax  σxmin
σxa  σxa  152.8  MPa
2

2. Using Appendix C-2, determine the geometric stress concentration factor.

D r
For 2  0.2 A  0.90879 b  0.28598
d d

b
Kt  A  
r
 Kt  1.44
d
3. Calculate the notch sensitivity of the material using Table 6-6.

2
Neuber constant (for S ut  68 ksi) a  0.096  in
1
Notch sensitivity q  q  0.805
a
1
r

4. Calculate the fatigue stress concentration factor using equation 6.11b.

Kf  1  q   Kt  1  Kf  1.354

5. Determine what, if any, fatigue stress concentration factor should be applied to the mean stress.

Yield strength SAE 1020 CR S y  393  MPa

© 2011 Pearson Education, Inc., Upper Saddle River, NJ. All rights reserved. This publication is protected by Copyright and written permission should be
MACHINE DESIGN - An Integrated Approach, 4th Ed. 6-66a-2

Evaluate Kfm  S 1  Kf  σxmax


S 2  Kf  σxmax  σxmin
return Kf if S 1  S y
S y  Kf  σxa
return if  S 1  S y  S 2  2  S y
σxm
0 otherwise

Kfm  0.731

6. Calculate the mean and alternating components of the stresses increased by the appropriate fatigue stress
concentration factors.

σm  Kfm σxm σm  186.1  MPa

σa  Kf  σxa σa  206.9  MPa

© 2011 Pearson Education, Inc., Upper Saddle River, NJ. All rights reserved. This publication is protected by Copyright and written permission should be
MACHINE DESIGN - An Integrated Approach, 4th Ed. 6-67-1
PROBLEM 6-67
Statement: A machine part is subjected to fluctuating, simple, multiaxial stresses. The fully corrected
nonzero stress ranges are: σxmin = 50 MPa, σxmax = 200 MPa, σymin = 80 MPa, σymax = 320 MPa,
τxymin = 120 MPa, τxymax = 480 MPa. The material properties are: S e = 525 MPa and S ut = 1200
MPa. Using a Case 3 load line, calculate and compare the infinite-life safety factors given by the
Sines and von Mises Methods.
Given: Material properties: S ut  1200 MPa S e  525  MPa
Stresses: σxmin  50 MPa σxmax  200  MPa
σymin  80 MPa σymax  320  MPa
τxymin  120  MPa τxymax  480  MPa

Solution: See Mathcad file P0667.


1. Calculate the alternating, and mean components of the given stresses.

σxmax  σxmin
σxa  σxa  75 MPa
2
σxmax  σxmin
σxm  σxm  125 MPa
2

σymax  σymin
σya  σya  120 MPa
2
σymax  σymin
σym  σym  200 MPa
2
τxymax  τxymin
τxya  τxya  180 MPa
2
τxymax  τxymin
τxym  τxym  300 MPa
2

(a) Sines Method


2. Calculate the equivalent alternating and mean stresses using equations 6.21b.

2 2 2
σ'a  σxa  σya  σxa σya  3  τxya σ'a  329.0 MPa

σ'm  σxm  σym σ'm  325.0 MPa

3. The factor of safety for the Sines Method, using equation (6.18e) is

S e S ut
Nfs  Nfs  1.11
σ'a S ut  σ'm S e

(b) von Mises Method


4. Calculate the equivalent alternating and mean stresses using equations 6.22b.

2 2 2
σ'a  σxa  σya  σxa σya  3  τxya σ'a  329.0 MPa

© 2011 Pearson Education, Inc., Upper Saddle River, NJ. All rights reserved. This publication is protected by Copyright and written permission should be
MACHINE DESIGN - An Integrated Approach, 4th Ed. 6-67-2

2 2 2
σ'm  σxm  σym  σxm σym  3  τxym σ'm  548.3 MPa

5. The factor of safety for the von Mises Method, using equation (6.18e) is

S e S ut
Nfvm  Nfvm  0.92
σ'a S ut  σ'm S e

6. This example shows that the von Mises method is more conservative when the endurance limit is modified by
such factors as surface finish and when there is a high mean shear stress.

© 2011 Pearson Education, Inc., Upper Saddle River, NJ. All rights reserved. This publication is protected by Copyright and written permission should be
MACHINE DESIGN - An Integrated Approach, 4th Ed. 6-68-1
PROBLEM 6-68
Statement: A cylindrical tank with hemispherical ends has been built. It was made from hot rolled steel that
has S ut = 380 MPa. The tank outside diameter is 300 mm with 20 mm wall thickness. The
pressure may fluctuate from 0 to an unknown maximum. For an infinite-life fatigue safety factor
of 4 with 99.99% reliability, what is the maximum pressure to which the tank may be subjected?
Given: Ultimate strength S ut  380  MPa
Tank dimensions OD  300  mm t  20 mm
Reliability & FS R  0.9999 Nf  4

Solution: See Mathcad file P0668.


1. Determine the maximum principal stresses as functions of the unkown pressure, which occur at the inside wall,
for this thick-wall cylinder:
Outside radius ro  0.5 OD ro  150 mm
Inside radius ri  ro  t ri  130 mm

2 
ri  p
2
Tangential σt( p )   1  ro 
2 2 2
ro  ri  ri 

 2
ri  p ro 
2
Radial σr( p )   1 
2 2 2
ro  ri  ri 

2
ri  p
Axial σa( p ) 
2 2
ro  ri

These are principal stresses so, using equation 5.7a:

2 2 2
σ'( p )  σt( p )  σr( p )  σa( p )  σt( p )  σa( p )  σa( p )  σr( p )  σt( p )  σr( p )

σ'min  0  MPa

2. Determine the alternating, and mean von Mises effective stress using equations 6.1.

σ'( p )  σ'min σ'( p )  σ'min


σ'a( p )  σ'm( p ) 
2 2

3. Calculate the unmodified endurance limit. S'e  0.5 S ut S'e  190  MPa
4. Calculate the endurance limit modification factors for axial loading.
Load Cload  0.7 (axial loading)
Size Csize  1 (axial loading)

Surface A  57.7 b  0.718 (hot rolled)

b
 Sut 
Csurf  A    Csurf  0.811
 MPa 
Temperature Ctemp  1
© 2011 Pearson Education, Inc., Upper Saddle River, NJ. All rights reserved. This publication is protected by Copyright and written permission should be
MACHINE DESIGN - An Integrated Approach, 4th Ed. 6-68-2

Reliability Creliab  0.702 (R = 99.99%)

5. Calculate the modified endurance limit.


S e  Cload  Csize Csurf  Ctemp Creliab S'e S e  75.7 MPa

6. Assuming a Case 3 load line, solve equation (6.18e) for p max

Guess p  1  MPa

Given

S e S ut
Nf = Nf  4.00
σ'a( p )  S ut  σ'm( p )  S e

p max  Find ( p ) p max  4.54 MPa

7. The principal stresses at p max are, respectively:

σ1  σt p max  σ2  σa p max  σ3  σr p max 

σ1  31.9 MPa σ2  13.7 MPa σ3  4.5 MPa

© 2011 Pearson Education, Inc., Upper Saddle River, NJ. All rights reserved. This publication is protected by Copyright and written permission should be
MACHINE DESIGN - An Integrated Approach, 4th Ed. 6-69-1

PROBLEM 6-69
Statement: A rotating shaft has been designed and fabricated from SAE 1040 HR steel. It is made from
tubing that has an outside diameter of 60 mm and a wall thickness of 5 mm. Strain gage
measurements indicate that there is a fully reversed axial stress of 68 MPa and a torsional stress
that fluctuates from 12 MPa to 52 MPa in phase with the axial stress at the critical point on the
shaft. Determine the infinite-life fatigue safety factor for the shaft for a reliability of 99%.

Given: Strength SAE 1040 HR S ut  524  MPa Alternating bending stress σxa  68 MPa
Torsional stress τxymax  52 MPa τxymin  12 MPa
Shaft dimensions OD  60 mm t  5  mm

Solution: See Mathcad file P0669.


1. Calculate the alternating, and mean components of the given stresses.
σxm  0  MPa σxa  68 MPa
τxymax  τxymin
τxya  τxya  20 MPa
2
τxymax  τxymin
τxym  τxym  32 MPa
2
2. Find the mean and alternating von Mises stresses using equations 6.22b (with y = 0).

2
Mean σ'm  3  τxym σ'm  55.426 MPa

2 2
Alternating σ'a  σxa  3  τxya σ'a  76.315 MPa

3. Calculate the unmodified endurance limit.


S'e  0.5 S ut S'e  262  MPa
4. Calculate the endurance limit modification factors for a rotating, round shaft.
Load Cload  1 (combined bending and torsion)

 0.097
Csize  1.189  
OD 
Size  Csize  0.799
 mm 
Surface A  57.7 b  0.718 (hot rolled)

b
 Sut 
Csurf  A    Csurf  0.644
 MPa 
Temperature Ctemp  1

Reliability Creliab  0.814 (R = 99%)

5. Calculate the modified endurance limit.


S e  Cload  Csize Csurf  Ctemp Creliab S'e S e  110  MPa

6. Assuming a Case 3 load line, determine the infinite-life fatigue safety factor.
S e S ut
Nf  Nf  1.25
σ'a S ut  σ'm S e
© 2011 Pearson Education, Inc., Upper Saddle River, NJ. All rights reserved. This publication is protected by Copyright and written permission should be
MACHINE DESIGN - An Integrated Approach, 4th Ed. 6-70a-1
PROBLEM 6-70a
Statement: For the data in row a in Table P6-8, find the safety factor for each of the four loading cases based
on the Modified-Goodman diagram if S e = 100, S y = 150, and S ut = 200.

Given: S e  100 S y  150 S ut  200


σ'm  50 σ'a  30

Solution: See Table 6-8 and Mathcad file P0670a.

1. Calculate the coordinates of point D in Figure 6-46 using equations (6.16).

S ut  S y  S e
Dm  Dm  100
S ut  S e

Da  S y  Dm Da  50

1. Use equations (6.18) and (6.16) to calculate the required quantities.

Case 1:

Sy  σ'a 
Nf1  1   if σ'a  Da Nf1  2.40
σ'm   Sy
S ut  σ'a 
1   otherwise
σ'm  Se 

Case 2:

Se  σ'm 
Nf2  1   if σ'm  Dm Nf2  2.50
σ'a  S ut 
S y  σ'm
otherwise
σ'a

Case 3:

S e S ut Se
Nf3  if  Dm Nf3  1.82
σ'a S ut  σ'm S e σ'a Se

σ'm S ut
S y  σ'a
otherwise
σ'm

Case 4:
S ut  S e  S e σ'a  S ut σ'm
2
σ'mSut 
2 2
S e  S ut
Se
σ'aSut  S e   σ'mSut
S ut

2 2
OZ  σ'a  σ'm

© 2011 Pearson Education, Inc., Upper Saddle River, NJ. All rights reserved. This publication is protected by Copyright and written permission should be
MACHINE DESIGN - An Integrated Approach, 4th Ed. 6-70a-2

Se
ZSut   σ'm  σ'mSut 2  σ'a  σ'aSut2 if σ'm 
σ'a Se

σ'm S ut

 σ'm  σ'mSut2   σ'a  σ'aSut 2 otherwise

S y  σ'a  σ'm
σ'mSy 
2

σ'aSy  σ'mSy  σ'a  σ'm

Sy
ZSy   σ'm  σ'mSy2   σ'a  σ'aSy 2 if σ'm 
σ'a
1
σ'm

 σ'm  σ'mSy 2  σ'a  σ'aSy2 otherwise

OZ  ZSut
Nf4  if σ'mSut  Dm Nf4  1.69
OZ
OZ  ZSy
otherwise
OZ

© 2011 Pearson Education, Inc., Upper Saddle River, NJ. All rights reserved. This publication is protected by Copyright and written permission should be
MACHINE DESIGN - An Integrated Approach, 4th Ed. 6-70h-1
PROBLEM 6-70h
Statement: For the data in row h in Table P6-8, find the safety factor for each of the four loading cases based
on the Modified-Goodman diagram if S e = 100, S y = 150, and S ut = 200.

Given: S e  100 S y  150 S ut  200


σ'm  80 σ'a  80

Solution: See Table 6-8 and Mathcad file P0670h.

1. Calculate the coordinates of point D in Figure 6-46 using equations (6.16).

S ut  S y  S e
Dm  Dm  100
S ut  S e

Da  S y  Dm Da  50

1. Use equations (6.18) and (6.16) to calculate the required quantities.

Case 1:

Sy  σ'a 
Nf1  1   if σ'a  Da Nf1  0.50
σ'm   Sy
S ut  σ'a 
1   otherwise
σ'm  Se 

Case 2:

Se  σ'm 
Nf2  1   if σ'm  Dm Nf2  0.75
σ'a  S ut 
S y  σ'm
otherwise
σ'a

Case 3:

S e S ut Se
Nf3  if  Dm Nf3  0.83
σ'a S ut  σ'm S e σ'a Se

σ'm S ut
S y  σ'a
otherwise
σ'm

Case 4:
S ut  S e  S e σ'a  S ut σ'm
2
σ'mSut 
2 2
S e  S ut
Se
σ'aSut  S e   σ'mSut
S ut

2 2
OZ  σ'a  σ'm

© 2011 Pearson Education, Inc., Upper Saddle River, NJ. All rights reserved. This publication is protected by Copyright and written permission should be
MACHINE DESIGN - An Integrated Approach, 4th Ed. 6-70h-2

Se
ZSut   σ'm  σ'mSut 2  σ'a  σ'aSut2 if σ'm 
σ'a Se

σ'm S ut

 σ'm  σ'mSut2   σ'a  σ'aSut 2 otherwise

S y  σ'a  σ'm
σ'mSy 
2

σ'aSy  σ'mSy  σ'a  σ'm

Sy
ZSy   σ'm  σ'mSy2   σ'a  σ'aSy 2 if σ'm 
σ'a
1
σ'm

 σ'm  σ'mSy 2  σ'a  σ'aSy2 otherwise

OZ  ZSut
Nf4  if σ'mSut  Dm Nf4  0.84
OZ
OZ  ZSy
otherwise
OZ

© 2011 Pearson Education, Inc., Upper Saddle River, NJ. All rights reserved. This publication is protected by Copyright and written permission should be
MACHINE DESIGN - An Integrated Approach, 4th Ed. 6-71-1

PROBLEM 6-71
Statement: A rotating shaft with a shoulder fillet, in torsion similar to that shown in Appendix Figure C-3 is
made from SAE 1020 CR steel and has dimensions D = 40 mm, d = 20 mm, and r = 4 mm. The
shaft is ground and is subjected to a fully reversed torque of +/- 80 N-m. Determine the infinite
life safety factor for the shaft for a reliability of 99.9%.

Given: Strength SAE 1020 CR S ut  469  MPa


Dimensions D  40 mm d  20 mm r  4  mm
Torque Ta  80 N  m Tm  0  N  m

Solution: See Mathcad file P0671.

1. Determine the nominal mean and alternating components of the stresses.

τxym  0  MPa

16 Ta
τxya  τxya  50.9 MPa
3
π d

2. Using Appendix C-3, determine the geometric stress concentration factor.

D r
For 2  0.2 A  0.86331 b  0.23865
d d
b
Kt  A  
r
 Kt  1.268
d

3. Calculate the notch sensitivity of the material using Table 6-6 adding 20 ksi to S ut because of the torsional load
2
Neuber constant (for S ut  20 ksi  88 ksi) a  0.072  in

1
Notch sensitivity q  q  0.846
a
1
r

4. Calculate the fatigue stress concentration factor using equation 6.11b.

Kf  1  q   Kt  1  Kf  1.226

5. Calculate the mean and alternating components of the stresses increased by the appropriate fatigue stress
concentration factors.

τm  τxym τm  0  MPa

τa  Kf  τxya τa  62.46  MPa

6. Calculate the von Mises normal stress.

von Mises stress σ'a  3  τa σ'a  108.19 MPa

7. Calculate the endurance limit modification factors for a rotating, solid, round steel shaft.

Load Cload  1 (pure torsion)


© 2011 Pearson Education, Inc., Upper Saddle River, NJ. All rights reserved. This publication is protected by Copyright and written permission should be
MACHINE DESIGN - An Integrated Approach, 4th Ed. 6-71-2

 0.097
Csize  1.189   
d
Size  Csize  0.889
 mm 

Surface A  1.58 b  0.085 (ground)

b
 Sut 
Csurf  A    Csurf  0.937
 MPa 
Temperature Ctemp  1

Reliability Creliab  0.753 (R = 99.9%)

Uncorrected endurance strength S'e  0.5 S ut

S'e  234.5  MPa

8. Calculate the endurance limit.

S e  Cload  Csize Csurf  Ctemp Creliab S'e S e  147.1  MPa

9. Using equation 6.14, calculate the infinite-life factor of safety.

Se
Nf  Nf  1.4
σ'a

© 2011 Pearson Education, Inc., Upper Saddle River, NJ. All rights reserved. This publication is protected by Copyright and written permission should be
MACHINE DESIGN - An Integrated Approach, 4th Ed. 7-1-1
PROBLEM 7-1
Statement: Two 3 x 5 cm blocks of steel with machined finish Ra = 0.6 mm are rubbed together with a normal
force of 400 N. Estimate the true area of contact between them if their S y = 400 MPa.
Given: Length of block L  5  cm Normal force F  400  N
Width of block w  3  cm Yield strength S y  400  MPa

Assumptions: The compressive yield strength is the same as the tensile yield strength. Then, S yc  S y.

Solution: See Mathcad file P0701.

1. Using equation 7.1, the true area of contact is estimated as


F 3 2
Ar  Ar  3.33333  10 cm
3  S yc
2
2. The apparent area of contact is Aa  w L Aa  15 cm

Aa
3. The ratio of apparent area to true area is  4500
Ar

© 2011 Pearson Education, Inc., Upper Saddle River, NJ. All rights reserved. This publication is protected by Copyright and written permission should be
MACHINE DESIGN - An Integrated Approach, 4th Ed. 7-2-1
PROBLEM 7-2
Statement: Estimate the dry coefficient of friction between the two pieces in Problem 7-1 if their S ut = 600
MPa.

Given: Length of block L  5  cm Yield strength S y  400  MPa


Width of block w  3  cm Ultimate strength S ut  600  MPa
Normal force F  400  N

Assumptions: The compressive yield strength is the same as the tensile yield strength. Then, S yc  S y.

Solution: See Mathcad file P0702.

Using equations 7.3 and 7.4, the coefficient of friction is estimated as

S us  0.80 S ut S us  480 MPa

S us
μ  μ  0.40
3  S yc

© 2011 Pearson Education, Inc., Upper Saddle River, NJ. All rights reserved. This publication is protected by Copyright and written permission should be
MACHINE DESIGN - An Integrated Approach, 4th Ed. 7-3-1
PROBLEM 7-3
Statement: For the bicycle pedal-arm assembly in Figure P7-1 assume a rider-applied force that ranges from
0 to 400 N at the pedal each cycle. Determine the maximum contact stresses at one sprocket
tooth-chain roller interface. Assume that the one tooth takes all the applied torque, that the
chain roller is 8-mm dia, the sprocket has a nominal (pitch) dia of 100 mm, and that the sprocket
tooth is essentially flat at the point of contact. The roller and sprocket are made of SAE X1340
steel, induction hardened to HRC 45-58. The roller and sprocket contact over a length of 8 mm.
Assuming rolling plus 9% sliding, estimate the number of cycles to failure for this particular
tooth-roller combination.

Given: Roller radius R1  4  mm The parts are steel. Therefore:


Sprocket radius R2  ∞ mm E  207  GPa
Sprocket width w  8  mm ν  0.28
Pedal force Frider  400  N Sprocket pitch dia d p  100  mm
Pedal arm length len  170  mm

Assumptions: The coefficient of friction is μ  0.33


Solution: See Figure P7-1 and Mathcad file P0703.
1. Determine the maximum contact force.
Torque on sprocket Tmax  Frider len Tmax  68 N  m

2  Tmax
Contact force Fcmax  Fcmax  1.36 kN
dp

2. Find the material constants from equation 7.9a.


2
1ν 6 1
Material constants m1  m1  4.452  10
E MPa
m2  m1

  
1 1 1 1
Geometry constant B    B  125 m
2  R1 R2 

1
Contact patch 2
 2 m1  m2 Fcmax 
half-width a      a  0.0878 mm
π B w 

3. The average and maximum contact pressure can now be found from equations 7.14b and c.
Fcmax
Average pressure p avg  p avg  968.1 MPa
2 a w
2  Fcmax
Maximum pressure p max  p max  1233 MPa
π a  w
Tangential fmax  μ  p max fmax  406.8 MPa
pressure

4. With m = 0.33, the principal stresses in the contact zone will be maximal on the surface (z = 0) at x = 0.3a from th
centerline as shown in Figures 7-20 and 7-22. The applied stress components are found from equation 7.23a for
the normal force and equation 7.23b for the tangential force.

For x  0.3 a
© 2011 Pearson Education, Inc., Upper Saddle River, NJ. All rights reserved. This publication is protected by Copyright and written permission should be
MACHINE DESIGN - An Integrated Approach, 4th Ed. 7-3-2

2
x
σxn  p max  1  σxn  1176 MPa
2
a

x
σxt  2  fmax  σxt  244.05 MPa
a

2
x
σzn  p max  1  σzn  1176 MPa
2
a

σzt  0  MPa τxzn  0  MPa

2
x
τxzt  fmax  1  τxzt  388.0 MPa
2
a

5. Equations 7.24a and 7.24b can now be solved for the total applied stresses along the x, y, and z axes.

σx  σxn  σxt σx  1420 MPa

σz  σzn  σzt σz  1176 MPa

τxz  τxzn  τxzt τxz  388.019 MPa

6. Assuming the rollers are short, we expect a plane stress condition to exist. The stress in the third dimension is
then:

σy  0  MPa also, τxy  0  MPa τyz  0  MPa

7. Unlike the pure-rolling case, these stresses are not principal because of the applied shear stress. The principal
stresses are found from equation 4.4 using a cubic root finding solution.
σx  σy  σz 3
C2  C2  2.596  10
MPa

 σx τxy   σx τxz   σy τyz 


     
C1   MPa MPa 
  MPa MPa 
  MPa MPa 
C1  1.519  10
6
 τxy σy   τxz σz   τyz σz 
 MPa MPa   MPa MPa   MPa MPa 
     

 σx τxy τxz 
 MPa MPa MPa 
 τxy σy τyz 
C0    C0  0
 MPa MPa MPa 
 τxz τyz σz 
 MPa MPa MPa 
 
3 2
f ( σ )  σ  C2 σ  C1 σ  C0

© 2011 Pearson Education, Inc., Upper Saddle River, NJ. All rights reserved. This publication is protected by Copyright and written permission should be
MACHINE DESIGN - An Integrated Approach, 4th Ed. 7-3-3

 C0 
   1704.6 
v   C1 
r  polyroots ( v)  MPa r  891.1  MPa

 C2   
   0.0 
 1 
Principal stresses: σ1  r σ1  0 MPa
3

σ2  r σ2  891.1 MPa
2

σ3  r σ3  1704.6 MPa
1

8. The maximum normal stress calculated in step 7 is 1705 MPa, compressive. Its K-factor can be calculated from
equation 7.25d.

K  π  m1  m2  σ3
2
K-factor K  81.3 MPa

9. From Table 7-7, Part 1, Line 4 the slope and intercept factors of this cast iron for rolling with 9% sliding are

λ  8.51 ζ  41.31

10. These are used in equation 7.26 along with the value of K from above to find the number of cycles that can be
expected at this load before pitting begins.

ζ  log  Nlife
log ( K) =
λ

ζ λ log
K

Nlife  10  psi  Nlife  4.6  10
6
cycles

© 2011 Pearson Education, Inc., Upper Saddle River, NJ. All rights reserved. This publication is protected by Copyright and written permission should be
MACHINE DESIGN - An Integrated Approach, 4th Ed. 7-4-1
PROBLEM 7-4
Statement: For the trailer hitch from Problem 3-4 on p. 169, determine the contact stresses in the ball and ball
cup. Assume that the ball is 2-in dia and the ill-fitting cup that surrounds the it is an internal
spherical surface 10% larger in diameter than the ball.

F
Given: Ball diameter d  2  in
Cup diameter D  2.2 in
Pull force Fpull  4.905  kN
Tongue weight Wtong  0.981  kN
Poisson's ratio ν  0.28
6
Modulus of elasticity E  30.0 10  psi

Solution: See Figure 7-4 and Mathcad file P0704.

2 2
Total force F  Fpull  Wtong FIGURE 7-4
Diagram Showing Contact Force
F  1125 lbf for Problem 7-4

Ball radius R1  0.5 d R1  1.000 in


Cup radius R2  0.5 D R2  1.100 in

  
1 1 1 1
Geometry constant B    B  0.045 in
2  R1 R2 

2
1ν 8 1
Material constants m1  m1  3.072  10
E psi
m2  m1
1
Contact patch 3
 3 m1  m2 
radius a    F a  0.0829 in
8 B 
2 2
Contact area A  π a A  0.022 in
F
Average pressure p avg  p avg  52.1 ksi
A

3
Maximum pressure p max   p avg p max  78.1 ksi
2

Stresses
Axial σzmax  p max σzmax  78.1 ksi

1  2 ν
In-plane σxmax    p max σxmax  60.9 ksi
2

σymax  σxmax

© 2011 Pearson Education, Inc., Upper Saddle River, NJ. All rights reserved. This publication is protected by Copyright and written permission should be
MACHINE DESIGN - An Integrated Approach, 4th Ed. 7-4-2

1  2 ν
   ( 1  ν)  2  ( 1  ν)
p max 2
Max shear stress τyzmax  
2  2 9 

τyzmax  26.4 ksi

Depth at max 2  2 ν
shear stress zτmax  a  zτmax  0.05228 in
7  2 ν

© 2011 Pearson Education, Inc., Upper Saddle River, NJ. All rights reserved. This publication is protected by Copyright and written permission should be
MACHINE DESIGN - An Integrated Approach, 4th Ed. 7-5-1
PROBLEM 7-5
Statement: For the trailer hitch from Problem 3-5 on p. 169, determine the contact stresses in the ball and ball
cup. Assume that the ball is 2-in dia and the ill-fitting cup that surrounds the it is an internal
spherical surface 10% larger in diameter than the ball.

Given: Ball diameter d  2  in


Cup diameter D  2.2 in
Trailer mass mtrail  2000 kg
F
1
Terminal velocity v  60 m sec
Time to reach vel. t  20 sec
Tongue weight Wtong  0.981  kN
Poisson's ratio ν  0.28
6
Modulus of elasticity E  30.0 10  psi

Solution: See Figure 7-5 and Mathcad file P0705.

v m
Acceleration a  a  3.00
t 2
sec
FIGURE 7-5
Pull force Fpull  mtrail a
Diagram Showing Contact Force
for Problem 7-5
Fpull  6.00 kN

2 2
Total force F  Fpull  Wtong F  1367 lbf
Ball radius R1  0.5 d R1  1.000 in
Cup radius R2  0.5 D R2  1.100 in

  
1 1 1 1
Geometry constant B    B  0.045 in
2  R1 R2 

2
1ν 8 1
Material constants m1  m1  3.072  10
E psi
m2  m1
1
Contact patch 3
 3 m1  m2 
radius a    F a  0.0885 in
8 B 
2 2
Contact area A  π a A  0.025 in
F
Average pressure p avg  p avg  55.6 ksi
A

3
Maximum pressure p max   p avg p max  83.3 ksi
2

Stresses
Axial σzmax  p max σzmax  83.3 ksi

© 2011 Pearson Education, Inc., Upper Saddle River, NJ. All rights reserved. This publication is protected by Copyright and written permission should be
MACHINE DESIGN - An Integrated Approach, 4th Ed. 7-5-2

1  2 ν
In-plane σxmax    p max σxmax  65.0 ksi
2

σymax  σxmax

1  2 ν
   ( 1  ν)  2  ( 1  ν)
p max 2
Max shear stress τyzmax  
2  2 9 

τyzmax  28.1 ksi

Depth at max 2  2 ν
shear stress zτmax  a  zτmax  0.05579 in
7  2 ν

© 2011 Pearson Education, Inc., Upper Saddle River, NJ. All rights reserved. This publication is protected by Copyright and written permission should be
MACHINE DESIGN - An Integrated Approach, 4th Ed. 7-6-1
PROBLEM 7-6
Statement: For the trailer hitch from Problem 3-6 on p. 169, determine the contact stresses in the ball and ball
cup. Assume that the ball is 2-in dia and the ill-fitting cup that surrounds the it is an internal
spherical surface 10% larger in diameter than the ball.

Given: Ball diameter d  2  in F


Cup diameter D  2.2 in
Trailer mass mtrail  2000 kg
Tongue weight Wtong  0.981  kN
Poisson's ratio ν  0.28
6
Modulus of elasticity E  30.0 10  psi

Solution: See Figure 7-6 and Mathcad file P0706.


From Problem 3-6, the impact (pull) force is
Pull force Fpull  55.1 kN

2 2 FIGURE 7-6
Total force F  Fpull  Wtong
Diagram Showing Contact Force
for Problem 7-6
F  12389 lbf
Ball radius R1  0.5 d R1  1.000 in
Cup radius R2  0.5 D R2  1.100 in

  
1 1 1 1
Geometry constant B    B  0.045 in
2  R1 R2 

2
1ν 8 1
Material constants m1  m1  3.072  10
E psi
m2  m1
1
Contact patch 3
 3 m1  m2 
radius a    F a  0.1845 in
8 B 
2 2
Contact area A  π a A  0.107 in
F
Average pressure p avg  p avg  115.9 ksi
A

3
Maximum pressure p max   p avg p max  173.8 ksi
2

Stresses
Axial σzmax  p max σzmax  173.8 ksi

1  2 ν
In-plane σxmax    p max σxmax  135.6 ksi
2
σymax  σxmax
© 2011 Pearson Education, Inc., Upper Saddle River, NJ. All rights reserved. This publication is protected by Copyright and written permission should be
MACHINE DESIGN - An Integrated Approach, 4th Ed. 7-6-2

1  2 ν
   ( 1  ν)  2  ( 1  ν)
p max 2
Max shear stress τyzmax  
2  2 9 

τyzmax  58.7 ksi

Depth at max 2  2 ν
shear stress zτmax  a  zτmax  0.11632 in
7  2 ν

© 2011 Pearson Education, Inc., Upper Saddle River, NJ. All rights reserved. This publication is protected by Copyright and written permission should be
MACHINE DESIGN - An Integrated Approach, 4th Ed. 7-7-1
PROBLEM 7-7
Statement: For the 12-mm dia steel wrist pin of Problem 3-7, find the maximum contact stress if the 2500 g
acceleration is fully reversed. The aluminum piston has a hole for the wrist pin that is 2% larger
than the pin and an engagement length of 20 mm.

Given: Wrist pin dia d  12 mm Material properties:


Piston hole multiplier k  1.02 Steel wrist pin ν1  0.28
Acceleration a  2500 g E1  206.8  GPa
Piston mass M  0.5 kg Aluminum piston ν2  0.34
Length of contact L  2  cm E2  71.7 GPa
Solution: See Mathcad file P0707.

Contact force F  M  a F  12.26 kN


Pin radius R1  0.5 d R1  6 mm
Hole radius R2  k R1 R2  6.12 mm

  
1 1 1 3 1
Geometry constant B    B  1.63399  10 mm
2  R1 R2 
2
1  ν1 6 1
Material constants m1  m1  4.456  10
E1 MPa
2
1  ν2 5 1
m2  m2  1.233  10
E2 MPa
1
Contact patch 2
 2 m1  m2 F 
half-width a      a  2.002 mm
π B L
2
Contact area A  2  L a A  80.10 mm
F
Average pressure p avg  p avg  153.0 MPa
A
2 F
Maximum pressure p max  p max  194.9 MPa
π a  L

Wrist pin
1. The maximum normal stresses in the center of the contact patch at the surface of the steel wrist pin are found
using equations 7.17a.
Axial σzmax  p max σzmax  194.9 MPa

In-plane σy1max  2  ν1 p max σy1max  109.1 MPa

2. The maximum shear stress and its location under the surface are found from equations 7.17b.
Max shear stress τ13max  0.304  p max τ13max  59.2 MPa
Depth at max
shear stress zτmax  0.786  a zτmax  1.57 mm

© 2011 Pearson Education, Inc., Upper Saddle River, NJ. All rights reserved. This publication is protected by Copyright and written permission should be
MACHINE DESIGN - An Integrated Approach, 4th Ed. 7-7-2

Piston hole

3. The maximum normal stresses in the center of the contact patch at the surface of the aluminum piston hole are
found using equations 7.17a.
Axial σzmax  p max σzmax  194.9 MPa
In-plane σy2max  2  ν2 p max σy2max  132.5 MPa

4. The maximum shear stress and its location under the surface are found from equations 7.17b.
Max shear stress τ13max  0.304  p max τ13max  59.2 MPa
Depth at max
shear stress zτmax  0.786  a zτmax  1.57 mm

© 2011 Pearson Education, Inc., Upper Saddle River, NJ. All rights reserved. This publication is protected by Copyright and written permission should be
MACHINE DESIGN - An Integrated Approach, 4th Ed. 7-8-1
PROBLEM 7-8
Statement: A paper mill processes rolls of paper having a density of 984 kg/m3. The paper roll is 1.50 m
outside dia (OD) by 0.22 m inside dia (ID) by 3.23 m long and has an effective modulus of
elasticity in compression of 14 MPa and n = 0.3. Determine the width of its contact patch when
it sits on a flat steel surface, loaded by its own weight.
3
Given: Paper properties ρ1  984  kg m Roll dimensions:
E1  14 MPa Outside diameter OD  1.50 m
ν1  0.3 Inside diameter ID  0.22 m
Steel properties E2  207  GPa Lemgth L  3.23 m
ν2  0.28

Solution: See Mathcad file P0708.

1. The weight of the paper roll is equal to its volume times the paper density times g.

Wroll 
π
4
 2 2 
 OD  ID  L ρ1  g Wroll  53.895 kN

2. Determine the radii of the contacting bodies.


Roll R1  0.5 OD R1  750 mm

Plate R2  ∞ mm

3. Determine the geometry and material constants.

  
1 1 1 1
Geometry constant B    B  0.667 m
2  R1 R2 

2
1  ν1 1
Material constants m1  m1  0.065
E1 MPa
2
1  ν2 6 1
m2  m2  4.452  10
E2 MPa

4. Calculate the contact patch hakf-width and width.


1
Contact patch 2
 2 m1  m2 Wroll 
half-width a      a  32.1832 mm
π B L 

Contact patch
width width  2  a width  64.4 mm

© 2011 Pearson Education, Inc., Upper Saddle River, NJ. All rights reserved. This publication is protected by Copyright and written permission should be
MACHINE DESIGN - An Integrated Approach, 4th Ed. 7-9-1
PROBLEM 7-9
Statement: For the ViseGrip plier-wrench for which the forces were analyzed in Problem 3-9, find the force in
link 4 needed to create a 0.25-mm-wide flat on each side of a 2-mm dia aluminum pin squeezed in
its 5-mm-wide jaws.

Given: Pin radius R1  1  mm Material properties:


Anvil curvature R2  ∞ mm Steel anvil ν2  0.28
Contact patch a  0.125  mm E2  206.8  GPa
Contact length L  5  mm Aluminum pin ν1  0.34
E1  71.7 GPa

Solution: See Mathcad file P0709.


1. Determine the geometry and material constants.

  
1 1 1 1
Geometry constant B    B  500 m
2  R1 R2 

2
1  ν1 5 1
Material constants m1  m1  1.233  10
E1 MPa
2
1  ν2 6 1
m2  m2  4.456  10
E2 MPa

2
π  a  B L
2. Calculate the contact force F  F  3.654 kN
2   m1  m2

3. Get the geometry from Problem 3-9 and calculate the pin force.

α  21.0 deg β  129.2  deg

F
F4 
cos( 180  deg  α)
sin( 180  deg  α)   sin( β  180  deg)
cos( β  180  deg)

F4  4.65 kN

© 2011 Pearson Education, Inc., Upper Saddle River, NJ. All rights reserved. This publication is protected by Copyright and written permission should be
MACHINE DESIGN - An Integrated Approach, 4th Ed. 7-10-1
PROBLEM 7-10
Statement: An overhung diving board is shown in Figure P7-4a. A 100-kg person is standing on the free end.
The board sits on a fulcrum that has a cylindrical contact surface of 5-mm radius. What is the size
of the contact patch between the board and the fulcrum if the board material is fiberglass with E =
10.3 GPa and n = 0.3?

Given: Fulcrum radius R1  5  mm


Board curvature R2  ∞ mm 2000 = L
Mass of person M  100  kg R1 P
Material properties:
Aluminum fulcrum ν1  0.34
E1  71.7 GPa
R2
Fiberglass board ν2  0.30
E2  10.3 GPa 700 = a
Board dimensions:
Width (Prob 4-10) w  305  mm FIGURE 7-10
Distance to right support a'  0.7 m Free Body Diagram for Problem 7-10
Contact length L  2  m

Solution: See Figure 7-10 and Mathcad file P0710.

Weight of person P  M  g P  0.981 kN


Summing moments about the support on the left end of the board, P L  F  b = 0
L
Fulcrum reaction F  P F  2.802 kN
a'

  
1 1 1 1
Geometry constant B    B  0.100 mm
2  R1 R2 

2
1  ν1 5 1
Material constants m1  m1  1.233  10
E1 MPa

2
1  ν2 5 1
m2  m2  8.835  10
E2 MPa
1
Contact patch 2
 2 m1  m2 F 
half-width a      a  0.0767 mm
π B w
Contact patch
width a2  2  a a2  0.153 mm

© 2011 Pearson Education, Inc., Upper Saddle River, NJ. All rights reserved. This publication is protected by Copyright and written permission should be
MACHINE DESIGN - An Integrated Approach, 4th Ed. 7-11-1
PROBLEM 7-11
Statement: Repeat Problem 7-10 assuming the 100-kg person in Problem 7-10 jumps up 25 cm and lands back
on the board. Assume the board weighs 29 kg and deflects 13.1 cm statically when the person
stands on it. What is the size of the contact patch between the board and the 5-mm-radius
aluminum fulcrum if the board material is fiberglass with E = 10.3 GPa and n = 0.3?

Given: Fulcrum radius R1  5  mm


Board curvature R2  ∞ mm 2000 = L
Mass of person M  100  kg R1 P
Material properties:
Aluminum fulcrum ν1  0.34
E1  71.7 GPa
R2
Fiberglass board ν2  0.30
E2  10.3 GPa 700 = a
Board dimensions:
Width (Prob 4-10) w  305  mm FIGURE 7-11
Distance to right support a'  0.7 m Free Body Diagram for Problem 7-11
Contact length L  2  m

Solution: See Figure 7-11 and Mathcad file P0711.

1. From Problem 3-11, the dynamic load resulting from the impact of the person with the board is P  3.056  kN

Summing moments about the support on the left end of the board, P L  F  b = 0
L
Fulcrum reaction F  P F  8.731 kN
a'

  
1 1 1 1
Geometry constant B    B  0.100 mm
2  R1 R2 

2
1  ν1 5 1
Material constants m1  m1  1.233  10
E1 MPa

2
1  ν2 5 1
m2  m2  8.835  10
E2 MPa
1
Contact patch 2
 2 m1  m2 F 
half-width a      a  0.1355 mm
π B w
Contact patch
width a2  2  a a2  0.271 mm

© 2011 Pearson Education, Inc., Upper Saddle River, NJ. All rights reserved. This publication is protected by Copyright and written permission should be
MACHINE DESIGN - An Integrated Approach, 4th Ed. 7-12-1
PROBLEM 7-12
Statement: Estimate the volume of adhesive wear to expect from an HB270 steel shaft of 40-mm diameter
rotating at 250 rpm for 10 years in a plain bronze bushing if the transverse load is 1000 N.
(a) For conditions of poor lubrication.
(b) For conditions of good lubrication.

1
Units: rpm  2  π rad min kilo  kg g yr  260  day rev  2  π rad

Given: Journal diameter d  40 mm Load F  1000 N


Journal speed n  250  rpm Life Life  10 yr
kilo
Journal hardness HB  270 
2
mm
Assumptions: The bushing is softer than the journal (shaft) so the wear will take place predominately on the
bushing.
Solution: See Mathcad file P0712.
1. From Figure 7-6 in the text we see that iron (the principal ingredient of steel) is metalurgically compatible with
copper and tin, the principal ingredients of bronze. From Figure 7-7 we have the following values of adhesive
wear coefficient for metalurgically compatible and poor and good lubrication, respectively.

4
Adhesive wear Ka  10 poor lubrication
coefficients
5
Kb  10 good lubrication

The length of π d 8
sliding is L   n  Life L  1.176  10 m
rev

2. Using equation 7.7a we can estimate the volume of wear for each lubrication condition
F L 6 3
(a) poor lubrication Va  Ka Va  4.4  10 mm
HB

F L 5 3
(b) good lubrication Vb  Kb Vb  4.4  10 mm
HB

3. These numbers are entirely too large. The bushing will fail long before the ten years have gone by.

© 2011 Pearson Education, Inc., Upper Saddle River, NJ. All rights reserved. This publication is protected by Copyright and written permission should be
MACHINE DESIGN - An Integrated Approach, 4th Ed. 7-13-1
PROBLEM 7-13
Statement: Estimate how long it will take to file 1 mm off a 2-cm cube of HB150 steel if the machinist applies
100 N over a 10-cm stroke at 60 strokes per minute.
(a) If done dry.
(b) If done lubricated.

Units: kilo  kg g


1
Given: Cube dimension a  2  cm Stroke rate n  60 min
Depth of wear d  1  mm Stroke s  10 cm
kilo
Force on file F  100  N Steel hardness HB  150 
2
mm

Assumptions: Only one face will be filed.

Solution: See Mathcad file P0713.


1. This is a two-body abrasion problem. From Table 7-2, the wear coefficients for dry and lubricated abrasion
using a file are

2
Wear coefficients Ka  5  10 dry
1
Kb  1  10 lubricated

2. The length of sliding is L = s n  t where t is the time required.

2 F L
3. The area of a face is Aa = a , and the depth is d = K
HB Aa

4. Combining these three equations and solving for the time, t

2
d  HB a
(a) dry ta  ta  20 min
Ka F  s n

strokesa  ta n strokesa  1177

2
d  HB a
(b) lubricated tb  tb  10 min
Kb F  s n

strokesb  tb n strokesb  588

© 2011 Pearson Education, Inc., Upper Saddle River, NJ. All rights reserved. This publication is protected by Copyright and written permission should be
MACHINE DESIGN - An Integrated Approach, 4th Ed. 7-14-1
PROBLEM 7-14
Statement: Figure P7-5 shows a child's toy called a pogo stick. The child stands on the pads, applying half
her weight on each side. She jumps off the ground, holding the pads up against her feet, and
bounces along with the spring cushioning the impact and storing energy to help each rebound.
Estimate the abrasivbe wear rate for the tip, which impacts the ground assuming a condition of
dry, loose abrasive grains (sand). Express the wear rate in number of jumps to remove 0.02 in
from the 1-in-dia aluminum tip if its S ut = 50 ksi.

Units: kilo  kg g

Given: Tip diameter d  1.00 in


Wear depth a  0.02 in
Tensile strength S ut  50 ksi

Assumptions: The tip slides a distance of one tip diameter per


jump.
Solution: See Figure 7-14 and Mathcad file P0714.
1. From Problem 3-14, the impact force for each jump is P  224  lbf
2. Use equation (2.10) to estimate the Brinell hardness of the steel tip. Fi /2 Fi /2
1  Sut  kilo kilo
HB    HB  94
530  psi  2 2
mm mm
3. This is a two-body abrasion problem. From Table 7-2, the wear
coefficient for dry abrasion using loose abrasive grains is
3
K  1  10

4. The length of sliding (see assumption) is L = d  Njumps

where d is the tip diameter and Njumps is the number of jumps. P

2
π d 2 FIGURE 7-14
5. The area of the tip is Aa  Aa  0.785 in
4 Free Body Diagram for Problem 7-14

P L
and the wear depth is a = K
HB Aa
a  HB Aa
6. Substituting for L and solving for the number of jumps, Njumps  Njumps  9410
K d P

© 2011 Pearson Education, Inc., Upper Saddle River, NJ. All rights reserved. This publication is protected by Copyright and written permission should be
MACHINE DESIGN - An Integrated Approach, 4th Ed. 7-15-1
PROBLEM 7-15
Statement: Create a table of acceptable materials to run against a steel shaft based on their metallurical
compatibility. Rank them as to suitability.

Solution: See Figure 7-6 in text.

© 2011 Pearson Education, Inc., Upper Saddle River, NJ. All rights reserved. This publication is protected by Copyright and written permission should be
MACHINE DESIGN - An Integrated Approach, 4th Ed. 7-16-1
PROBLEM 7-16
Statement: Determine the size of the contact patch and the maximum contact stresses for a 20-mm-dia steel
ball rolled against a flat aluminum plate with 1 kN.

Given: Ball radius R1  10 mm Material properties


Plate curvature R2  ∞ mm Steel ball ν1  0.28
Load F  1  kN E1  206.8  GPa
Aluminum plate ν2  0.34
E2  71.7 GPa

Solution: See Mathcad file P0716.


1. Calculate geometry and material constants, contact patch dimension, and pressures.

  
1 1 1 1
Geometry constant B    B  0.05 mm
2  R1 R2 

2
1  ν1 6 1
Material constants m1  m1  4.456  10
E1 MPa
2
1  ν2 5 1
m2  m2  1.233  10
E2 MPa
1
Contact patch 3
 3 m1  m2 
radius a    F a  0.501 mm
8 B 
2 2
Contact area A  π a A  0.789 mm

F
Average pressure p avg  p avg  1267 MPa
A

3
Maximum pressure p max   p avg p max  1900 MPa
2

2. Determine the stresses in the ball at the surface


Axial σzmax  p max σzmax  1900 MPa

1  2  ν1
In-plane σxmax1    p max σxmax1  1482 MPa
2

σymax1  σxmax1

3. Determine the stresses in the ball below the surface

 1  2 ν1 2
p max 
Max shear τyzmax1      1  ν1  2   1  ν1
stress 2  2 9 
τyzmax1  641.5 MPa

© 2011 Pearson Education, Inc., Upper Saddle River, NJ. All rights reserved. This publication is protected by Copyright and written permission should be
MACHINE DESIGN - An Integrated Approach, 4th Ed. 7-16-2

Depth at max 2  2  ν1
shear stress zτmax1  a  zτmax1  0.316 mm
7  2  ν1

4. Determine the stresses in the plate at the surface


Axial σzmax  p max σzmax  1900 MPa

1  2  ν2
In-plane σxmax2    p max σxmax2  1596 MPa
2

σymax2  σxmax2

5. Determine the stresses in the plate below the surface

 1  2 ν2 2
p max 
Max shear τyzmax2      1  ν2  2   1  ν2
stress 2  2 9 
τyzmax2  615.2 MPa

Depth at max 2  2  ν2
shear stress zτmax2  a  zτmax2  0.326 mm
7  2  ν2

© 2011 Pearson Education, Inc., Upper Saddle River, NJ. All rights reserved. This publication is protected by Copyright and written permission should be
MACHINE DESIGN - An Integrated Approach, 4th Ed. 7-17-1
PROBLEM 7-17
Statement: Determine the size of the contact patch and the maximum contact stresses for a 20-mm-dia steel
ball rolled against a 30-mm diameter aluminum ball with 800 N.

Given: Steel ball radius R1  10 mm Material properties:


Aluminum ball radius R2  15 mm Steel ball ν1  0.28
Load F  0.8 kN E1  206.8  GPa
Aluminum ball ν2  0.34
E2  71.7 GPa

Solution: See Mathcad file P0717.


1. Calculate geometry and material constants, contact patch dimension, and pressures.

  
1 1 1 1
Geometry constant B    B  0.083 mm
2  R1 R2 

2
1  ν1 6 1
Material constants m1  m1  4.456  10
E1 MPa
2
1  ν2 5 1
m2  m2  1.233  10
E2 MPa
1
Contact patch 3
 3 m1  m2 
radius a    F a  0.392 mm
8 B 
2 2
Contact area A  π a A  0.484 mm

F
Average pressure p avg  p avg  1653 MPa
A

3
Maximum pressure p max   p avg p max  2480 MPa
2

2. Determine the stresses in the steel ball at the surface


Axial σzmax  p max σzmax  2480 MPa

1  2  ν1
In-plane σxmax1    p max σxmax1  1934 MPa
2

σymax1  σxmax1

3. Determine the stresses in the steel ball below the surface

 1  2 ν1 2
p max 
Max shear τyzmax1      1  ν1  2   1  ν1
stress 2  2 9 
τyzmax1  837.1 MPa

© 2011 Pearson Education, Inc., Upper Saddle River, NJ. All rights reserved. This publication is protected by Copyright and written permission should be
MACHINE DESIGN - An Integrated Approach, 4th Ed. 7-17-2

Depth at max 2  2  ν1
shear stress zτmax1  a  zτmax1  0.247 mm
7  2  ν1

4. Determine the stresses in the aluminum ball at the surface


Axial σzmax  p max σzmax  2480 MPa

1  2  ν2
In-plane σxmax2    p max σxmax2  2083 MPa
2

σymax2  σxmax2

5. Determine the stresses in the aluminum ball below the surface

 1  2 ν2 2
p max 
Max shear τyzmax2      1  ν2  2   1  ν2
stress 2  2 9 
τyzmax2  802.9 MPa

Depth at max 2  2  ν2
shear stress zτmax2  a  zτmax2  0.256 mm
7  2  ν2

© 2011 Pearson Education, Inc., Upper Saddle River, NJ. All rights reserved. This publication is protected by Copyright and written permission should be
MACHINE DESIGN - An Integrated Approach, 4th Ed. 7-18-1
PROBLEM 7-18
Statement: Determine the size of the contact patch and the maximum contact stresses for a 40-mm-dia steel
cylinder, 25-cm long, rolled against a flat aluminum plate with 4 kN.
Given: Cylinder radius R1  20 mm Material properties:
Plate curvature R2  ∞ mm Steel cylinder ν1  0.28
Load F  4  kN E1  206.8  GPa
Contact length L  250  mm Aluminum plate ν2  0.34
E2  71.7 GPa

Solution: See Mathcad file P0718.


1. Calculate geometry and material constants, contact patch dimension, and pressures.

  
1 1 1 1
Geometry constant B    B  0.025 mm
2  R1 R2 

2
1  ν1 6 1
Material constants m1  m1  4.456  10
E1 MPa
2
1  ν2 5 1
m2  m2  1.233  10
E2 MPa
1
Contact patch 2
 2 m1  m2 F 
half-width a      a  0.0827 mm
π B L

2
Contact area A  2  L a A  41.36 mm

F
Average pressure p avg  p avg  96.7 MPa
A

2 F
Maximum pressure p max  p max  123.1 MPa
π a  L

2. Determine the stresses in the cylinder at the surface. The maximum normal stresses in the center of the contact
patch at the surface of the steel cylinder are found using equations 7.17a.
Axial σzmax  p max σzmax  123 MPa

In-plane σy1max  2  ν1 p max σy1max  69.0 MPa

3. Determine the stresses in the cylinder below the surface. The maximum shear stress and its location under the
surface are found from equations 7.17b.
Max shear stress τ13max  0.304  p max τ13max  37.4 MPa

Depth at max
shear stress zτmax  0.786  a zτmax  0.0650 mm

4. Determine the stresses in the plate at the surface. The maximum normal stresses in the center of the contact
patch at the surface of the aluminum plate are found using equations 7.17a.

Axial σzmax  p max σzmax  123.1 MPa


© 2011 Pearson Education, Inc., Upper Saddle River, NJ. All rights reserved. This publication is protected by Copyright and written permission should be
MACHINE DESIGN - An Integrated Approach, 4th Ed. 7-18-2

In-plane σy2max  2  ν2 p max σy2max  83.7 MPa

5. Determine the stresses in the plate below the surface. The maximum shear stress and its location under the
surface are found from equations 7.17b.

Max shear stress τ13max  0.304  p max τ13max  37.4 MPa


Depth at max
shear stress zτmax  0.786  a zτmax  0.0650 mm

© 2011 Pearson Education, Inc., Upper Saddle River, NJ. All rights reserved. This publication is protected by Copyright and written permission should be
MACHINE DESIGN - An Integrated Approach, 4th Ed. 7-19-1
PROBLEM 7-19
Statement: Determine the size of the contact patch and the maximum contact stresses for a 40-mm-dia steel
cylinder, 25-cm long, rolled against a parallel 50-mm-dia steel cylinder with 10 kN.
Given: Cylinder radius R1  20 mm Material properties:
Cylinder radius R2  25 mm Steel cylinder ν1  0.28
Load F  10 kN E1  206.8  GPa
Contact length L  250  mm Steel cylinder ν2  0.28
E2  206.8  GPa

Solution: See Mathcad file P0719.


1. Calculate geometry and material constants, contact patch dimension, and pressures.

  
1 1 1 1
Geometry constant B    B  0.045 mm
2  R1 R2 

2
1  ν1 6 1
Material constants m1  m1  4.456  10
E1 MPa
2
1  ν2 6 1
m2  m2  4.456  10
E2 MPa
1
Contact patch 2
 2 m1  m2 F 
half-width a      a  0.0710 mm
π B L

2
Contact area A  2  L a A  35.51 mm

F
Average pressure p avg  p avg  281.6 MPa
A

2 F
Maximum pressure p max  p max  358.6 MPa
π a  L

2. Determine the stresses in either cylinder at the surface. The maximum normal stresses in the center of the
contact patch at the surface of the steel cylinder are found using equations 7.17a.
Axial σzmax  p max σzmax  358.6 MPa

In-plane σy1max  2  ν1 p max σy1max  200.8 MPa

3. Determine the stresses in either cylinder below the surface. The maximum shear stress and its location
under the surface are found from equations 7.17b.
Max shear stress τ13max  0.304  p max τ13max  109.0 MPa

Depth at max
shear stress zτmax  0.786  a zτmax  0.0558 mm

© 2011 Pearson Education, Inc., Upper Saddle River, NJ. All rights reserved. This publication is protected by Copyright and written permission should be
MACHINE DESIGN - An Integrated Approach, 4th Ed. 7-20-1
PROBLEM 7-20
Statement: Determine the size of the contact patch and the maximum contact stresses for a 20-mm-dia steel
ball rolled against a 40-mm-dia steel cylinder, 25-mm long with 10 kN force.
Given: Ball radii R1  10.00  mm The parts are steel. Therefore:
R'1  10.00  mm E  206.8  GPa ν  0.28
Cylinder radii R2  20.00  mm (radial)
R'2  ∞ mm (axial)
Radial load F  10 kN (normal to contact plane)
Angle between planes of R1 and R2 θ  0  deg

Assumptions: The relative motion is rolling with < 1% sliding.


Solution: See Mathcad file P0720.
1. Find the material constants from equation 7.9b.
2
1ν 6 1
Material constants m1  m1  4.456  10 
E MPa
m2  m1
2. Two geometry constants are needed from equations 7.19a.

  
Geometry 1 1 1 1 1 1
constants A      A  0.1250 mm
2  R1 R'1 R2 R'2 
1
2
 1 2 2 
B   
1 1   1  1    1
  R  B  0.0250 mm
2  R1 R'1   2 R'2  
 1 1  1 1  
 2  R  R'    R  R'   cos( 2  θ ) 
  1 1  2 2 

ϕ  acos  
B 180
Angle  ϕ  78.46
A π
Factors from  0.86215
equations 7.19e ka  50.192 ϕ ka  1.167

2
kb  0.0045333  0.043581 ϕ  0.0017292  ϕ 
5 3 7 4
 3.7374 10  ϕ  3.7418 10  ϕ 
9 5
 1.4207 10 ϕ
kb  0.875
3. Determine the contact patch dimensions using the material and geometry constants in equations 7.19d.
1
Major axis 3
 3 m1  m2 
half-width a  ka   F a  0.947  mm
4 A 
1
Minor axis 3
 3 m1  m2 
half-width b  kb   F b  0.710  mm
4 A 

© 2011 Pearson Education, Inc., Upper Saddle River, NJ. All rights reserved. This publication is protected by Copyright and written permission should be
MACHINE DESIGN - An Integrated Approach, 4th Ed. 7-20-2

2
Contact area A  π a  b A  2.11354  mm
4. The average and maximum contact pressure can now be found from equations 7.18b and c.
F
Average pressure p avg  p avg  4731 MPa
A
3
Maximum pressure p max   p avg p max  7097 MPa
2
5. The maximum normal stresses in the center of the contact patch at the surface are then found using equations
7.21a.

σx  2  ν  ( 1  2  ν)  
b
In-plane   pmax σx  5312 MPa
 a  b

σy  2  ν  ( 1  2  ν)  
a
  pmax σy  5759 MPa
 a  b

Axial σz  p max σz  7097 MPa

These stresses are principal:


σ1  σx σ2  σy σ3  σz

The maximum shear stress associated with them at the surface is

σ1  σ3
τ13  τ13  892  MPa
2

6. The maximum shear stress under the surface on the z-axis is approximately
Max shear stress τ13max  0.34 p max τ13max  2413 MPa

7. All of the stresses found so far exist on the centerline of the patch. At the edge of the patch, at the surface,
there will also be a shear stress. Two constants are found from equation 7.21b for this calculation.
b
k3  k3  0.75
a

1 2 2
k4   a b k4  0.662
a

8. These constants are used in equations 7.21c and d to find the shear stresses on the surface at the ends of the
major and minor axes.

   atanh k4  1  p max


k3 1
Major axis τxy  ( 1  2  ν)  τxy  1084 MPa
k4
2  k4 

k3  k3  k4  
Minor axis τxy  ( 1  2  ν)  1   atan    pmax τxy  966  MPa
k4
2  k4  k3  

© 2011 Pearson Education, Inc., Upper Saddle River, NJ. All rights reserved. This publication is protected by Copyright and written permission should be
MACHINE DESIGN - An Integrated Approach, 4th Ed. 7-21-1
PROBLEM 7-21
Statement: A cam-follower system has a dynamic load of 0 to 2 kN. The cam is cylindrical with a minimum
radius of curvature of 20 mm. The roller follower is crowned with radii of 15 mm in one direction
and 150 mm in the other. Find the contact stresses if the follower is steel and the cam is nodular
iron.

Given: Roller radius R1  15.00  mm


Crown radius R'1  150.00 mm(90 deg to roller rad)
Cam curvature R2  20.00  mm (radial)
Cam curvature R'2  ∞ mm (axial)
Radial load F  2  kN (normal to contact plane)
Angle between planes of R1 and R2 θ  0  deg
Material properties E1  206.8  GPa ν1  0.28 steel

E2  172.4  GPa ν2  0.30 cast iron

Assumptions: The relative motion is rolling with < 1% sliding.

Solution: See Mathcad file P0721.


1. Find the material constants from equation 7.9b.
2
1  ν1 3 1
Material constants m1  m1  4.456  10 
E1 GPa
2
1  ν2 3 1
m2  m2  5.278  10 
E2 GPa
2. Two geometry constants are needed from equations 7.19a.

  
Geometry 1 1 1 1 1 1
constants A      A  0.0617 mm
2  R1 R'1 R2 R'2 
1
2
 1 2 2 
B   
1 1   1  1    1
  R  B  0.0550 mm
2  R1 R'1   2 R'2  
 1 1  1 1  
 2  R  R'    R  R'   cos( 2  θ ) 
  1 1  2 2 

ϕ  acos  
B 180
Angle  ϕ  26.89
A π

Factors from  0.86215


equations 7.19e ka  50.192 ϕ ka  2.939

2
kb  0.0045333  0.043581 ϕ  0.0017292  ϕ 
5 3 7 4
 3.7374 10  ϕ  3.7418 10  ϕ 
9 5
 1.4207 10 ϕ

kb  0.477

© 2011 Pearson Education, Inc., Upper Saddle River, NJ. All rights reserved. This publication is protected by Copyright and written permission should be
MACHINE DESIGN - An Integrated Approach, 4th Ed. 7-21-2

3. Determine the contact patch dimensions using the material and geometry constants in equations 7.19d.

1
Major axis 3
 3 m1  m2 
half-width a  ka   F a  1.818  mm
4 A 
1
Minor axis 3
 3 m1  m2 
half-width b  kb   F b  0.295  mm
4 A 

2
Contact area A  π a  b A  1.68582  mm

4. The average and maximum contact pressure can now be found from equations 7.18b and c.
F
Average pressure p avg  p avg  1186 MPa
A

3
Maximum pressure p max   p avg p max  1780 MPa
2

5. The maximum normal stresses in the steel follower at the center of the contact patch at the surface are then found
using equations 7.21a.

σx  2  ν1   1  2  ν1   p
b
In-plane  max σx  1106 MPa
 a  b

σy  2  ν1   1  2  ν1   p
a
 max σy  1670 MPa
 a  b

Axial σz  p max σz  1780 MPa

These stresses are principal:

σ1  σx σ2  σy σ3  σz

The maximum shear stress associated with them at the surface is

σ1  σ3
τ13  τ13  337  MPa
2

6. The maximum shear stress under the surface on the z-axis is approximately
Max shear stress τ13max  0.34 p max τ13max  605  MPa

7. All of the stresses found so far exist on the centerline of the patch. At the edge of the patch, at the surface,
there will also be a shear stress. Two constants are found from equation 7.21b for this calculation.
b
k3  k3  0.162
a

1 2 2
k4   a b k4  0.987
a
© 2011 Pearson Education, Inc., Upper Saddle River, NJ. All rights reserved. This publication is protected by Copyright and written permission should be
MACHINE DESIGN - An Integrated Approach, 4th Ed. 7-21-3

8. These constants are used in equations 7.21c and d to find the shear stresses on the surface at the ends of the
major and minor axes.

   atanh k4  1  p max


k3
τxy   1  2  ν1 
1
Major axis τxy  201  MPa
k4
2  k4 

k3  k3  k4  
Minor axis τxy   1  2  ν1  1   atan    p max τxy  100  MPa
k4
2  k4  k3  

9. The maximum normal stresses in the nodular iron cam at the center of the contact patch at the surface are then
found using equations 7.21a.

σx  2  ν2   1  2  ν2   p
b
In-plane  max σx  1167 MPa
 a  b

σy  2  ν2   1  2  ν2   p
a
 max σy  1680 MPa
 a  b

Axial σz  p max σz  1780 MPa

These stresses are principal:

σ1  σx σ2  σy σ3  σz

The maximum shear stress associated with them at the surface is

σ1  σ3
τ13  τ13  306  MPa
2

10. The maximum shear stress under the surface on the z-axis is approximately
Max shear stress τ13max  0.34 p max τ13max  605  MPa

11. All of the stresses found so far exist on the centerline of the patch. At the edge of the patch, at the surface,
there will also be a shear stress. Two constants are found from equation 7.21b for this calculation.
b
k3  k3  0.162
a

1 2 2
k4   a b k4  0.987
a

12. These constants are used in equations 7.21c and d to find the shear stresses on the surface at the ends of the
major and minor axes.

   atanh k4  1  p max


k3
τxy   1  2  ν2 
1
Major axis τxy  183  MPa
k4
2  k4 

k3  k3  k4  
Minor axis τxy   1  2  ν2  1   atan    p max τxy  91 MPa
k4
2  k4  k3  

© 2011 Pearson Education, Inc., Upper Saddle River, NJ. All rights reserved. This publication is protected by Copyright and written permission should be
MACHINE DESIGN - An Integrated Approach, 4th Ed. 7-22-1
PROBLEM 7-22
Statement: An "inline" skate is shown in Figure P7-10. The polyurethane wheels are 72 mm dia. by 12-mm
thick with a 6-mm crown radius and are spaced on 104- mm centers. The skate-boot-foot
combination weighs 2 kg. The effective "spring rate" of the person-skate subsystem is 6000
N/m. Find the contact stresses in the wheels when a 100-kg person lands a 0.5-m jump on one
foot on concrete. Assume the urethane wheels and concrete have the properties below.
(a) Assume that all 4 wheels land simultaneously.
(b) Assume that one wheel absorbs all the landing force.

Given: Force per wheel (from Problem 3-22):


Case (a) Fa  897  N Case (b) Fb  3.59 kN
Material properties:
Urethane E1  600  MPa ν1  0.4
Concrete E2  21 GPa ν2  0.2
Wheel dimensions R1  36 mm R'1  6  mm
Concrete dimensions R2  ∞ mm R'2  ∞ mm
Contact angle θ  0  deg

Assumptions: The relative motion is rolling with < 1% sliding.


Solution: See Mathcad file P0722.
1. Find the material constants from equation 7.9b.
2
1  ν1 1
Material constants m1  m1  1.400 
E1 GPa
2
1  ν2 1
m2  m2  0.0457
E2 GPa

2. Two geometry constants are needed from equations 7.19a.

  
Geometry 1 1 1 1 1 1
constants A      A  0.0972 mm
2  R1 R'1 R2 R'2 
1
2
 1 2 2 
B   
1 1   1  1    1
  R  B  0.0694 mm
2  R1 R'1   2 R'2  
 1 1  1 1  
 2  R  R'    R  R'   cos( 2  θ ) 
  1 1  2 2 

ϕ  acos  
B 180
Angle  ϕ  44.42
A π

Factors from  0.86215


equations 7.19e ka  50.192 ϕ ka  1.906

2
kb  0.0045333  0.043581 ϕ  0.0017292  ϕ 
5 3 7 4
 3.7374 10  ϕ  3.7418 10  ϕ 
9 5
 1.4207 10 ϕ
kb  0.593

© 2011 Pearson Education, Inc., Upper Saddle River, NJ. All rights reserved. This publication is protected by Copyright and written permission should be
MACHINE DESIGN - An Integrated Approach, 4th Ed. 7-22-2

Part (a)
3. Determine the contact patch dimensions using the material and geometry constants in equations 7.19d.
1
Major axis 3
 3 m1  m2 
half-width a  ka    Fa a  4.108  mm
4 A 
1
Minor axis 3
 3 m1  m2 
half-width b  kb    Fa b  1.278  mm
4 A 

2
Contact area Ac  π a  b Ac  16.49024 mm

4. The average and maximum contact pressure can now be found from equations 7.18b and c.
Fa
Average pressure p avg  p avg  54.4 MPa
Ac

3
Maximum pressure p max   p avg p max  81.6 MPa
2

5. The maximum normal stresses in the center of the contact patch at the surface are then found using equations
7.21a.

σx  2  ν1   1  2  ν1   p
b
In-plane  max σx  69.1 MPa
 a  b

σy  2  ν1   1  2  ν1   p
a
 max σy  77.7 MPa
 a  b

Axial σz  p max σz  81.6 MPa

These stresses are principal:

σ1  σx σ2  σy σ3  σz

The maximum shear stress associated with them at the surface is

σ1  σ3
τ13  τ13  6.22 MPa
2

6. The maximum shear stress under the surface on the z-axis is approximately
Max shear stress τ13max  0.34 p max τ13max  27.7 MPa

7. All of the stresses found so far exist on the centerline of the patch. At the edge of the patch, at the surface,
there will also be a shear stress. Two constants are found from equation 7.21b for this calculation.

b
k3  k3  0.311
a

© 2011 Pearson Education, Inc., Upper Saddle River, NJ. All rights reserved. This publication is protected by Copyright and written permission should be
MACHINE DESIGN - An Integrated Approach, 4th Ed. 7-22-3

1 2 2
k4   a b k4  0.95
a

8. These constants are used in equations 7.21c and d to find the shear stresses on the surface at the ends of the
major and minor axes.

   atanh k4  1  p max


k3
τxy   1  2  ν1 
1
Major axis τxy  5.24 MPa
k4
2  k4 

k3  k3  k4  
Minor axis τxy   1  2  ν1  1   atan    p max τxy  3.31 MPa
k4
2  k4  k3  

Part (b)
9. Determine the contact patch dimensions using the material and geometry constants in equations 7.19d.
1
Major axis 3
 3 m1  m2 
half-width a  ka    Fb a  6.522  mm
4 A 
1
Minor axis 3
 3 m1  m2 
half-width b  kb    Fb b  2.029  mm
4 A 

2
Contact area Ac  π a  b Ac  41.56824 mm

10. The average and maximum contact pressure can now be found from equations 7.18b and c.
Fb
Average pressure p avg  p avg  86.4 MPa
Ac

3
Maximum pressure p max   p avg p max  129.5  MPa
2

11. The maximum normal stresses in the center of the contact patch at the surface are then found using equations
7.21a.

σx  2  ν1   1  2  ν1   p
b
In-plane  max σx  109.8  MPa
 a  b

σy  2  ν1   1  2  ν1   p
a
 max σy  123.4  MPa
 a  b

Axial σz  p max σz  129.5  MPa

These stresses are principal:


σ1  σx σ2  σy σ3  σz

The maximum shear stress associated with them at the surface is

© 2011 Pearson Education, Inc., Upper Saddle River, NJ. All rights reserved. This publication is protected by Copyright and written permission should be
MACHINE DESIGN - An Integrated Approach, 4th Ed. 7-22-4

σ1  σ3
τ13  τ13  9.88 MPa
2

12. The maximum shear stress under the surface on the z-axis is approximately
Max shear stress τ13max  0.34 p max τ13max  44.0 MPa

13. All of the stresses found so far exist on the centerline of the patch. At the edge of the patch, at the surface,
there will also be a shear stress. Two constants are found from equation 7.21b for this calculation.
b
k3  k3  0.311
a

1 2 2
k4   a b k4  0.95
a

14. These constants are used in equations 7.21c and d to find the shear stresses on the surface at the ends of the
major and minor axes.

   atanh k4  1  p max


k3
τxy   1  2  ν1 
1
Major axis τxy  8.31 MPa
k4
2  k4 

k3  k3  k4  
Minor axis τxy   1  2  ν1  1   atan    p max τxy  5.26 MPa
k4
2  k4  k3  

© 2011 Pearson Education, Inc., Upper Saddle River, NJ. All rights reserved. This publication is protected by Copyright and written permission should be
MACHINE DESIGN - An Integrated Approach, 4th Ed. 7-23-1
PROBLEM 7-23
Statement: A pair of 12-in dia cylindrical steel rolls run together with 9% slip. Find their contact stresses
for a radial contact forces of 1000 lb/in of length.
Given: Roller radii R1  6.00 in The parts are hardened steel. Therefore:
6
R2  6.00 in E  30 10  psi ν  0.28
1
Radial load/in F'  1000 lbf  in
Assumptions: The coefficient of friction is μ  0.33.

Solution: See Mathcad file P0723.


1. Find the material constants from equation 7.9a.
2
1ν 8 1
Material constants m1  m1  3.072  10
E psi
m2  m1

  
1 1 1 1
Geometry constant B    B  0.167 in
2  R1 R2 

1
Contact patch 2
 2 m1  m2 
half-width a     F' a  0.0153 in
π B 
2. The average and maximum contact pressure can now be found from equations 7.14b and c.
F'
Average pressure p avg  p avg  32.6 ksi
2 a

2  F'
Maximum pressure p max  p max  41.6 ksi
π a

Tangential fmax  μ  p max fmax  13.7 ksi


pressure
3. With m = 0.33, the principal stresses in the contact zone will be maximal on the surface (z = 0) at x = 0.3a from
the centerline as shown in Figures 7-20 and 7-22. The applied stress components are found from equation
7.23a for the normal force and equation 7.23b for the tangential force.

For x  0.3 a

2
x
σxn  p max  1  σxn  39.6 ksi
2
a

x
σxt  2  fmax  σxt  8.23 ksi
a

2
x
σzn  p max  1  σzn  39.6 ksi
2
a

σzt  0  ksi τxzn  0  ksi

© 2011 Pearson Education, Inc., Upper Saddle River, NJ. All rights reserved. This publication is protected by Copyright and written permission should be
MACHINE DESIGN - An Integrated Approach, 4th Ed. 7-23-2

2
x
τxzt  fmax  1  τxzt  13.1 ksi
2
a

4. Equations 7.24a and 7.24b can now be solved for the total applied stresses along the x, y, and z axes.
σx  σxn  σxt σx  47.871 ksi
σz  σzn  σzt σz  39.642 ksi
τxz  τxzn  τxzt τxz  13.082 ksi

5. Assuming the rollers are long, we expect a plane strain condition to exist. The stress in the third dimension is
found from equation 7.23b:

σy  ν  σx  σz σy  24.504 ksi

6. Unlike the pure-rolling case, these stresses are not principal because of the applied shear stress. The principal
stresses are found from equation 4.4 using a cubic root finding solution.
σx σy σz
σx  σy  σz 
ksi ksi ksi
τxz
τxy  0 τyz  0 τxz 
ksi

C2  σx  σy  σz C2  112.017

 σx τxy   σx τxz   σy τyz  3


C1          C1  3.871  10
 τxy σy   τxz σz   τyz σz 

 σx τxy τxz 
  4
C0   τxy σy τyz  C0  4.231  10
τ τ σ 
 xz yz z 
3 2
f ( σ )  σ  C2 σ  C1 σ  C0

 C0 
   57.5 
v   C1 
s  polyroots ( v)  ksi s  30.0  ksi

 C2   
   24.5 
 1 
Principal stresses:
σ1  s σ1  24.5 ksi
3
σ2  s σ2  30.0 ksi
2
σ3  s σ3  57.5 ksi
1
Maximum shear stress

σ1  σ3
τ13  τ13  16.5 ksi
2

7. The principal stresses are maximum at the surface as seen in Figures 7-20 and 7-22 in the text.
© 2011 Pearson Education, Inc., Upper Saddle River, NJ. All rights reserved. This publication is protected by Copyright and written permission should be
MACHINE DESIGN - An Integrated Approach, 4th Ed. 7-24-1

PROBLEM 7-24
Statement: Estimate the cycle life of the rolls in Problem 7-23 if they are made of class 30 gray cast iron,
austempered to HB270.
Given: Roller radii R1  6.00 in The parts are gray cast iron. Therefore:
6
R2  6.00 in E  15 10  psi ν  0.28
1
Radial load/in F'  1000 lbf  in
Assumptions: The coefficient of friction is μ  0.33.
Solution: See Mathcad file P0724.
1. Find the material constants from equation 7.9a.
2
1ν 8 1
Material constants m1  m1  6.144  10
E psi
m2  m1

  
1 1 1 1
Geometry constant B    B  0.167 in
2  R1 R2 

1
Contact patch 2
 2 m1  m2 
half-width a     F' a  0.0217 in
π B 
2. The average and maximum contact pressure can now be found from equations 7.14b and c.
F'
Average pressure p avg  p avg  23.1 ksi
2 a

2  F'
Maximum pressure p max  p max  29.38 ksi
π a

Tangential fmax  μ  p max fmax  9.70 ksi


pressure
3. With m = 0.33, the principal stresses in the contact zone will be maximal on the surface (z = 0) at x = 0.3a from
the centerline as shown in Figures 7-20 and 7-22. The applied stress components are found from equation
7.23a for the normal force and equation 7.23b for the tangential force.

For x  0.3 a

2
x
σxn  p max  1  σxn  28.0 ksi
2
a

x
σxt  2  fmax  σxt  5.82 ksi
a

2
x
σzn  p max  1  σzn  28.0 ksi
2
a

σzt  0  ksi τxzn  0  ksi

© 2011 Pearson Education, Inc., Upper Saddle River, NJ. All rights reserved. This publication is protected by Copyright and written permission should be
MACHINE DESIGN - An Integrated Approach, 4th Ed. 7-24-2

2
x
τxzt  fmax  1  τxzt  9.25 ksi
2
a

4. Equations 7.24a and 7.24b can now be solved for the total applied stresses along the x, y, and z axes.
σx  σxn  σxt σx  33.850 ksi

σz  σzn  σzt σz  28.031 ksi

τxz  τxzn  τxzt τxz  9.250 ksi

5. Assuming the rollers are long, we expect a plane strain condition to exist. The stress in the third dimension is
found from equation 7.23b:

σy  ν  σx  σz σy  17.327 ksi

6. Unlike the pure-rolling case, these stresses are not principal because of the applied shear stress. The principal
stresses are found from equation 4.4 using a cubic root finding solution.
σx σy σz
σx  σy  σz 
ksi ksi ksi
τxz
τxy  0 τyz  0 τxz 
ksi

C2  σx  σy  σz C2  79.208

 σx τxy   σx τxz   σy τyz  3


C1          C1  1.935  10
 τxy σy   τxz σz   τyz σz 

 σx τxy τxz 
  4
C0   τxy σy τyz  C0  1.496  10
τ τ σ 
 xz yz z 
3 2
f ( σ )  σ  C2 σ  C1 σ  C0

 C0 
   40.6 
v   C1 
s  polyroots ( v)  ksi s  21.2  ksi

 C2   
   17.3 
 1 
Principal stresses:
σ1  s σ1  17.3 ksi
3
σ2  s σ2  21.2 ksi
2
σ3  s σ3  40.6 ksi
1
7. The maximum normal stress calculated in step 6 is 40.9 ksi, compressive. Its K-factor can be calculated from
equation 7.25d.

K  π  m1  m2  σ3
2
K-factor K  637.5 psi
© 2011 Pearson Education, Inc., Upper Saddle River, NJ. All rights reserved. This publication is protected by Copyright and written permission should be
MACHINE DESIGN - An Integrated Approach, 4th Ed. 7-24-3

8. From Table 7-7, Part 2, Line 33 the slope and intercept factors of this cast iron for rolling with 9% sliding are
λ  7.87 ζ  35.90
9. These are used in equation 7.26 along with the value of K from above to find the number of cycles that can be
expected at this load before pitting begins.

ζ  log  Nlife
log ( K) =
λ

ζ λ log
K

Nlife  10  psi  Nlife  6.7  10
13
cycles

© 2011 Pearson Education, Inc., Upper Saddle River, NJ. All rights reserved. This publication is protected by Copyright and written permission should be
MACHINE DESIGN - An Integrated Approach, 4th Ed. 7-25-1
PROBLEM 7-25
Statement: A 12-mm-dia, class 30 gray cast iron pad is supported by a steel bar made from SAE 4340 steel,
quenched and tempered at 800F. The force on the pad is 3.8 kN. Estimate the real area of contact
and the ratio of the real area to the apparent area of contact.

Given: Pad diameter d  12 mm


Compressive yield strengths: Steel S ycs  1469 MPa
Cast iron S ycci  752  MPa
Contact force F  3.8 kN

Assumptions: The compressive yield strength of the cast iron is approximately the same as the compressive
strength.

Solution: See Mathcad file P0725.

1. Calculate the apparent area of contact.


2
π d 2
Aa  Aa  113.097 mm
4

2. Use equation 7.1 and the cast iron strength, which is the weaker of the two, to estimate the real area of contact.

F 2
Ar  Ar  1.6844 mm
3  S ycci

3. The ratio of real to apparent area of contact is:

Ar
ratio  ratio  1.5 %
Aa

© 2011 Pearson Education, Inc., Upper Saddle River, NJ. All rights reserved. This publication is protected by Copyright and written permission should be
MACHINE DESIGN - An Integrated Approach 4th Ed. 7-26-1
PROBLEM 7-26
Statement: Estimate the dry coefficient of friction between the two materials in Problem 7-35 if the shear
strength of the cast iron is S us = 310 MPa. How does this compare to the value given in Table
7-1?

Given: Pad diameter d  12 mm


Shear strength: Cast iron S us  310  MPa
Compressive yield strength S yc  752  MPa
Assumptions: The compressive yield strength of the cast iron is approximately the same as the compressive
strength.
Solution: See Table 7-1 and Mathcad file P0726.

1. Using equation 7.3, the coefficient of friction is approximately:

S us
μ  μ  0.14
3  S yc

From Table 7-1, the coefficient of friction for mild steel on cast iron (lubricated) is 0.183.

© 2011 Pearson Education, Inc., Upper Saddle River, NJ. All rights reserved. This publication is protected by Copyright and written permission should be
MACHINE DESIGN - An Integrated Approach, 4th Ed. 7-27-1

PROBLEM 7-27 _____

Statement: Two 0.5-in x 1-in 1040 hot-rolled steel pads are in contact with a force of 900 lb. Estimate the
real area of contact and the ratio of the real area to the apparent area of contact.

Given: Pad dimensions L  1.00 in w  0.5 in


Compressive yield strength: S yc  42 ksi
Contact force F  900  lbf
Assumptions: The compressive yield strength of the steel is approximately the same as the tensile yield
strength.
Solution: See Mathcad file P0727.

1. Calculate the apparent area of contact.

2
Aa  L w Aa  0.500 in

2. Use equation 7.1 and the compressive yield strength to estimate the real area of contact.

F 2
Ar  Ar  0.0071 in
3  S yc

3. The ratio of real to apparent area of contact is:

Ar
ratio  ratio  1.43 %
Aa

© 2011 Pearson Education, Inc., Upper Saddle River, NJ. All rights reserved. This publication is protected by Copyright and written permission should be
MACHINE DESIGN - An Integrated Approach, 4th Ed. 7-28-1

PROBLEM 7-28 _____

Statement: Estimate the dry coefficient of friction between the two materials in Problem 7-27. How does this
compare to the value given in Table 7-1?

Given: Pad dimensions L  1.0 in w  0.50 in


Ultimate tensile strength: S ut  76 ksi
Compressive yield strength: S yc  42 ksi
Assumptions: The compressive yield strength of the steel is approximately the same as the tensile yield
strength.

Solution: See Mathcad file P0728.

1. Estimate the ultimate shear strength using equation 7.4 for steel.
S us  0.8 S ut S us  60.8 ksi

2. Using equation 7.3, the coefficient of friction is approximately:

S us
μ  μ  0.48
3  S yc

© 2011 Pearson Education, Inc., Upper Saddle River, NJ. All rights reserved. This publication is protected by Copyright and written permission should be
MACHINE DESIGN - An Integrated Approach, 4th Ed. 7-29-1

PROBLEM 7-29 _____

Statement: Two materials have been tested to determine the amount of adhesive wear that takes place when
they are run together. The average depth of wear results are given in Table P7-1 along with the
test parameters for a total of 350 tests. What is the average wear coefficient for the materials
tested if the Brinell hardness of the softer of the two is HB 277.
2
Given: Hardness HB  277  kgf  mm
2
Data from Table P7-1: i  1  4 A  10 mm
i
F  l  NT  d 
i i i i

100  N 5000 m 100 0.180  mm


200  N 5000 m 75 0.372  mm
200  N 7500 m 75 0.550  mm
400  N 10000  m 100 1.47 mm

Solution: See Table P7-1 and Mathcad file P0729.

1. The total number of tests is:


Total 
 NT i Total  350
i
2. The average force, apparent contact area, length of sliding, and depth of wear are:

 F iNT i
1
Favg   Favg  228.571 N
Total
i

Total   i
A  NT 
1 2
Aavg   Aavg  10.000 mm
i
i

Total   i
l  NT 
1 3
lavg   lavg  6.964  10 m
i
i

 diNT i
1
d avg   d avg  0.669 mm
Total
i
3. Calculate the average adhesive wear coefficient using equation 7.7b and the values calculated above.

HB Aavg  d avg 5


Kavg  Kavg  1.14  10
Favg lavg

© 2011 Pearson Education, Inc., Upper Saddle River, NJ. All rights reserved. This publication is protected by Copyright and written permission should be
MACHINE DESIGN - An Integrated Approach, 4th Ed. 7-30-1

PROBLEM 7-30 _____

Statement: A piece of mild steel with HB = 280 has its thickness reduced in an abrasive grinder. Both the
grinding wheel and the steel part have the same width, which is 20 mm. On each pass through
the grinder 0.1 mm is removed. If the abrasive wear coefficient for this operation is 5E-1, what is
the approximate normal force on the grinding wheel?

2
Given: Hardness H  280  kgf  mm
Depth of wear per stroke d  0.1 mm Width of wear w  20 mm
1
Wear coefficient K  5  10

Solution: See Mathcad file P0730.

1. From equation 7.7a, the volume of wear per stroke is


F l
V  K
H

2. Solving for the normal force F,

V H
F  where V  d  w l
K l

3. Substituting for V and canceling the l-terms,

d  w H
F  F  11.0 kN
K

© 2011 Pearson Education, Inc., Upper Saddle River, NJ. All rights reserved. This publication is protected by Copyright and written permission should be
MACHINE DESIGN - An Integrated Approach, 4th Ed. 7-31-1

PROBLEM 7-31 _____


Statement: Two steel gears with involute tooth profiles are in mesh. At the line of contact between the gears
they can be modeled as two cylinders in contact. When the contact is away from the pitch point
there is a combination of rolling and sliding. Determine and plot the dynamic contact principal
stresses on the surface of the teeth for the following gears if the contact force is 500 lb and the
coefficient of friction is 0.15:
R1 = 2.000 in, R2 = 6.000 in. The thickness (face width) of both gears is 0.5 in.
Also find the value of x/a for which the principal stresses have an extreme value.

6
Given: The parts are hardened steel. Therefore: E  30 10  psi ν  0.28
Radii of curvature R1  2.00 in R2  6.00 in
Contact force W  500  lbf
Gear tooth thickness F  0.5 in (face width)
Coefficient of friction μ  0.15

Solution: See Mathcad file P0731.


1. Calculate the force per unit of length along the line of contact.
W 1
F'  F'  1000 lbf  in
F
2. Find the material constants from equation 7.9a.
2
1ν 8 1
Material constants m1  m1  3.072  10
E psi
m2  m1

  
1 1 1 1
Geometry constant B    B  0.333 in
2  R1 R2 

1
Contact patch 2
 2 m1  m2 
half-width a     F' a  0.0108 in
π B 

3. The average and maximum contact pressure can now be found from equations 7.14b and c.
F'
Average pressure p avg  p avg  46.2 ksi
2 a

2  F'
Maximum pressure p max  p max  58.8 ksi
π a

Tangential fmax  μ  p max fmax  8.8 ksi


pressure

4. The applied stress components are found from equation 7.23a for the normal force and equation 7.23b for the
tangential force.

 2 
σxn( x)  if  x  a p max  1  0  psi 
x
σzn( x)  σxn( x)
 2 
 a 
© 2011 Pearson Education, Inc., Upper Saddle River, NJ. All rights reserved. This publication is protected by Copyright and written permission should be
MACHINE DESIGN - An Integrated Approach, 4th Ed. 7-31-2

τxzn  0  ksi

x 2 
return 2  fmax    1 if x  a
x
σxt( x)  
a 2 
 a 
x 2 
return 2  fmax     1 if x  a
x σzt  0  ksi
a 2 
 a 
x
2  fmax  otherwise
a

 2 
τxzt( x)  if  x  a fmax  1  0  ksi
x
 2 
 a 

5. Equations 7.24a can now be written for the total applied stresses along the x and z axes.

σx( x)  σxn( x)  σxt( x)

σz( x)  σzn( x)  σzt

τxz( x)  τxzn  τxzt( x)

6. The face width of the gears is short with respect to the other dimensions, therefore we expect a plane stress
condition to exist. The stress in the third dimension is

σy  0  ksi

7. Unlike the pure-rolling case, these stresses are not principal because of the applied shear stress. The principal
stresses are found from equation 4.6a for the plane stress case.

2
σx( x)  σz( x)  σx( x)  σz( x)  2
σ1( x)      τxz( x)
2  2 

2
σx( x)  σz( x)  σx( x)  σz( x)  2
σ2( x)      τxz( x)
2  2 

σ3  0  ksi

8. Plot the two nonzero principal stresses over the range x  3  a 2.999  a  3  a

© 2011 Pearson Education, Inc., Upper Saddle River, NJ. All rights reserved. This publication is protected by Copyright and written permission should be
MACHINE DESIGN - An Integrated Approach, 4th Ed. 7-31-3

PRINCIPAL CONTACT STRESSES


0.5
0.18

σ1( x)
Normalized stress

pmax
 0.5
σ2( x)
pmax

1

 1.5
2 1 0 1 2
x
a
Normalized width, x/a

9. Note that for  = 0.15, the extreme values of the principal stresses occur at approximately x/a = 0.18.

© 2011 Pearson Education, Inc., Upper Saddle River, NJ. All rights reserved. This publication is protected by Copyright and written permission should be
MACHINE DESIGN - An Integrated Approach, 4th Ed. 7-32-1

PROBLEM 7-32 _____

Statement: Two steel gears with involute tooth profiles are in mesh. At the line of contact between the gears
they can be modeled as two cylinders in contact. When the contact is away from the pitch point
there is a combination of rolling and sliding. Determine the dynamic contact stresses on the
surface of the teeth for the following gears if the contact force is 1500 lb and the coefficient of
friction is 0.33:
R1 = 2.500 in, R2 = 5.000 in. The thickness (face width) of both gears is 0.625 in.

6
Given: The parts are hardened steel. Therefore E  30 10  psi ν  0.28
Radii of curvature R1  2.500  in R2  5.000  in
Contact force W  1500 lbf
Gear tooth thickness F  0.625  in (face width)
Coefficient of friction μ  0.33
Solution: See Mathcad file P0732.
1. Calculate the force per unit of length along the line of contact.
W 1
F'  F'  2400 lbf  in
F
2. Find the material constants from equation 7.9a.
2
1ν 8 1
Material constants m1  m1  3.072  10
E psi
m2  m1

  
1 1 1 1
Geometry constant B    B  0.300 in
2  R1 R2 

1
Contact patch 2
 2 m1  m2 
half-width a     F' a  0.0177 in
π B 

3. The average and maximum contact pressure can now be found from equations 7.14b and c.
F'
Average pressure p avg  p avg  67.8 ksi
2 a

2  F'
Maximum pressure p max  p max  86.4 ksi
π a

Tangential fmax  μ  p max fmax  28.5 ksi


pressure
4. With  = 0.33, the principal stresses in the contact zone will be maximal on the surface (z = 0) at x = 0.3a from th
centerline as shown in Figures 7-20 and 7-22. The applied stress components are found from equation 7.23a for
the normal force and equation 7.23b for the tangential force.

For x  0.3 a

2
x
σxn  p max  1  σxn  82.4 ksi
2
a
© 2011 Pearson Education, Inc., Upper Saddle River, NJ. All rights reserved. This publication is protected by Copyright and written permission should be
MACHINE DESIGN - An Integrated Approach, 4th Ed. 7-32-2

x
σxt  2  fmax  σxt  17.10 ksi
a

2
x
σzn  p max  1  σzn  82.4 ksi
2
a

σzt  0  ksi τxzn  0  ksi

2
x
τxzt  fmax  1  τxzt  27.2 ksi
2
a

5. Equations 7.24a can now be written for the total applied stresses along the x and z axes.

σx  σxn  σxt σx  99.497 ksi

σz  σzn  σzt σz  82.395 ksi

τxz  τxzn  τxzt τxz  27.190 ksi

6. The face width of the gears is short with respect to the other dimensions, therefore we expect a plane stress
condition to exist. The stress in the third dimension is

σy  0  ksi

7. Unlike the pure-rolling case, these stresses are not principal because of the applied shear stress. The principal
stresses are found from equation 4.6a for the plane stress case.

σ1  0  ksi

2
σx  σz  σx  σz  2
σ2      τxz σ2  62.443 ksi
2  2 

2
σx  σz  σx  σz  2
σ3      τxz σ3  119.449 ksi
2  2 

Maximum shear stress

σ1  σ3
τ13  τ13  59.7 ksi
2

© 2011 Pearson Education, Inc., Upper Saddle River, NJ. All rights reserved. This publication is protected by Copyright and written permission should be
MACHINE DESIGN - An Integrated Approach, 4th Ed. 7-33-1

PROBLEM 7-33 _____


Statement: Two contacting rollers are needed for a machine application. They run together with a
combination of rolling and 9% sliding. Both are to be made from 1144 CD steel. The radial
contact force is 1200 N and the coefficient of friction is 0.33. The rollers are to have the same
radii and are both 10 mm long. If the design life is 8E08 cycles, determine a suitable radius for the
rollers.
6
Given: The parts are hardened steel. Therefore E  30 10  psi ν  0.28
Contact force W  1200 N
Roller length w  10 mm
8
Coefficient of friction μ  0.33 Design life Nd  8  10
Material data from Table 7-7 λ  4.10 ζ  21.79

Solution: See Mathcad file P0733.


1. Calculate the force per unit of length along the line of contact.
W 1
F'  F'  120.00 N  mm
w
2. Find the material constants from equation 7.9a.
2
1ν 6 1
Material constants m1  m1  4.456  10 
E MPa
m2  m1

 
1 1 1
Geometry constant B( R)   
2 R R
1
Contact patch 2
 2 m1  m2 
half-width a ( R)     F'
 π B( R) 

3. The average and maximum contact pressure can now be found from equations 7.14b and c.
F'
Average pressure p avg ( R) 
2  a ( R)

2  F'
Maximum pressure p max ( R) 
π  a ( R)

Tangential fmax ( R)  μ  p max ( R)


pressure
4. With  = 0.33, the principal stresses in the contact zone will be maximal on the surface (z = 0) at x = 0.3a from th
centerline as shown in Figures 7-20 and 7-22. The applied stress components are found from equation 7.23a for
the normal force and equation 7.23b for the tangential force.

For x( R)  0.3 a ( R)

2 2
σxn( R)  p max ( R)  1  0.3 σzn( R)  p max ( R)  1  0.3

τxzn  0  MPa
© 2011 Pearson Education, Inc., Upper Saddle River, NJ. All rights reserved. This publication is protected by Copyright and written permission should be
MACHINE DESIGN - An Integrated Approach, 4th Ed. 7-33-2

σxt( R)  2  fmax ( R)  0.3 σzt  0  MPa

2
τxzt( R)  fmax ( R)  1  0.3

5. Equations 7.24a can now be written for the total applied stresses along the x and z axes.

σx( R)  σxn( R)  σxt( R)

σz( R)  σzn( R)  σzt

τxz( R)  τxzn  τxzt( R)

6. The face width of the rollers is short with respect to the other dimensions, therefore we expect a plane stress
condition to exist. The stress in the third dimension is

σy  0  MPa

7. Unlike the pure-rolling case, these stresses are not principal because of the applied shear stress. The principal
stresses are found from equation 4.6a for the plane stress case.

σ1  0  MPa

2
σx( R)  σz( R)  σx( R)  σz( R)  2
σ2( R)      τxz( R)
2  2 

2
σx( R)  σz( R)  σx( R)  σz( R)  2
σ3( R)      τxz( R)
2  2 

8. Calculate the K-factor for this material and the number of cycles before the onset of pitting for this design using
equation 7.26.

ζ  log Nd 
λ
K  10  psi K  1390 psi

9. Use equation 7.25e to solve for the roller radius. First, guess R  40 mm

K = π  m1  m2  σ3( R)
2
Given

R  Find ( R) R  47.9 mm

© 2011 Pearson Education, Inc., Upper Saddle River, NJ. All rights reserved. This publication is protected by Copyright and written permission should be
MACHINE DESIGN - An Integrated Approach, 4th Ed. 7-34-1

PROBLEM 7-34 _____


Statement: Two contacting rollers are needed for a machine application. They run together with a
combination of rolling and 9% sliding. Both are to be made from Meehanite. The rollers have
the same radii (30 mm) and are both 45 mm long. If the design life is 1E08 cycles, determine the
allowable load that may be applied to these rollers.

Given: Material data from Table 7-7 K  1450 psi


Roller radii R1  30 mm R2  30 mm
Roller length L  45 mm

Solution: See Mathcad file P0734.

1. Find the geometry constant from equation 7.9a.

  
1 1 1 1
Geometry constant B    B  0.033
2  R1 R2  mm

2. Calculate the allowable force on the rollers using equation 7.25d.

K L
Allowable force F  F  6.75 kN
2 B

© 2011 Pearson Education, Inc., Upper Saddle River, NJ. All rights reserved. This publication is protected by Copyright and written permission should be
MACHINE DESIGN - An Integrated Approach, 4th Ed. 7-35-1

PROBLEM 7-35
Statement: A 25-mm-dia, Class 20 gray cast iron pad is supported by a steel bar made from SAE 4130 steel,
quenched and tempered at 800F. The force on the pad is 2800 N. Estimate the real area of
contact and the ratio of the real area to the apparent area of contact.

Given: Pad diameter d  25 mm


Compressive yield strengths: Pad S ycp  572  MPa
Bar S ycb  1193 MPa
Contact force F  2800 N

Assumptions: The compressive yield strength of the cast iron is approximately the same as its compressive
strength.

Solution: See Mathcad file P0735.

1. Calculate the apparent area of contact.


2
π d 2
Aa  Aa  490.9 mm
4

2. Use equation 7.1 and the pad strength, which is the weaker of the two, to estimate the real area of contact.

F 2
Ar  Ar  1.632 mm
3  S ycp

3. The ratio of real to apparent area of contact is:

Ar
ratio  ratio  0.3 %
Aa

© 2011 Pearson Education, Inc., Upper Saddle River, NJ. All rights reserved. This publication is protected by Copyright and written permission should be
MACHINE DESIGN - An Integrated Approach, 4th Ed. 7-36-1

PROBLEM 7-36
Statement: Estimate the dry coefficient of friction between the two materials in Problem 7-35 if the shear
strength of the cast iron is S us = 310 Mpa. How does this compare to the value given in Table
7-1?
Given: Pad diameter d  25 mm
Shear strength: Cast iron S us  310  MPa
Compressive yield strength S yc  572  MPa
Assumptions: The compressive yield strength of the cast iron is approximately the same as the compressive
strength.
Solution: See Mathcad file P0736.

1. Using equation 7.3, the coefficient of friction is approximately:

S us
μ  μ  0.18
3  S yc

From Table 7-1, the coefficient of friction for mild steel on cast iron (lubricated) is 0.183.

© 2011 Pearson Education, Inc., Upper Saddle River, NJ. All rights reserved. This publication is protected by Copyright and written permission should be
MACHINE DESIGN - An Integrated Approach, 4th Ed. 7-37-1

PROBLEM 7-37
Statement: Two 25-mm x 40-mm SAE 1020 hot-rolled steel pads are in contact with a force of 9 kN. Estimate
the real area of contact and the ratio of the real area to the apparent area of contact.
Given: Pad dimensions w  25 mm L  40 mm
Compressive yield strength: S yc  379  MPa
Contact force F  9000 N

Assumptions: The compressive yield strength of the steel is approximately the same as its tensile yield strength.

Solution: See Mathcad file P0737.


1. Calculate the apparent area of contact.

2
Aa  L w Aa  1000.0 mm

2. Use equation 7.1 and the pad strength, which is the weaker of the two, to estimate the real area of contact.

F 2
Ar  Ar  7.916 mm
3  S yc

3. The ratio of real to apparent area of contact is:

Ar
ratio  ratio  0.79 %
Aa

© 2011 Pearson Education, Inc., Upper Saddle River, NJ. All rights reserved. This publication is protected by Copyright and written permission should be
MACHINE DESIGN - An Integrated Approach, 4th Ed. 7-38-1

PROBLEM 7-38
Statement: Estimate the dry coefficient of friction between the two materials in Problem 7-37. How does this
compare to the value given in Table 7-1?
Given: Pad dimensions w  25 mm L  40 mm
Ultimate tensile strength: S ut  379  MPa
Compressive yield strength: S yc  379  MPa
Assumptions: The compressive yield strength of the steel is approximately the same as the tensile yield strength.

Solution: See Table 7-1 and Mathcad file P0738.

1. Estimate the ultimate shear strength equation 7.4 for steel.


S us  0.8 S ut S us  303.2 MPa

2. Using equation 7.3, the coefficient of friction is approximately:


S us
μ  μ  0.27
3  S yc

From Table 7-1, the coefficient of friction for mild steel on mild steel is 0.74.

© 2011 Pearson Education, Inc., Upper Saddle River, NJ. All rights reserved. This publication is protected by Copyright and written permission should be
MACHINE DESIGN - An Integrated Approach, 4th Ed. 7-39-1

PROBLEM 7-39
Statement: A 25-mm-diameter steel shaft of hardness HB420 rotates at 700 rpm in a 40-mm-long plain bronze
bushing with an average radial load of 500 N. Estimate the time it would take to remove 0.05 mm
of bushing material by adhesive wear if the lubrication were suddenly lost assuming a uniform
wear rate around the bushing.

1
Units: kilo  kg g rev  2  π rad rpm  rev  min

Given: Journal diameter d  25 mm Load F  500  N


Journal speed n  700  rpm Bushing length lbush  40 mm
kilo
Journal hardness HB  420  Depth of wear d  0.05 mm
2
mm

Solution: See Mathcad file P0739.

1. From Figure 7-6 in the text we see that iron (the principal ingredient of steel) is metalurgically compatible with
copper and tin, the principal ingredients of bronze. From Figure 7-7 we have the following value of the
adhesive wear coefficient for metalurgically compatible materials with no lubrication.

4
Wear coefficient K  8  10

2. Using equation 7.7b, estimate the time for the bushing to wear d  0.05 mm

Apparent area of contact Aa  π d  lbush

π d
Length of contact in time t lcont( t)   n t
rev
d  HB Aa
Solving equation 7.7b for l lcont =
K F
d  HB l bush rev
Substituting and solving for t t  t  29.4 min
n K F

4
In a time of t  29.4 min the shaft will turn θ  n  t θ  2  10 rev

© 2011 Pearson Education, Inc., Upper Saddle River, NJ. All rights reserved. This publication is protected by Copyright and written permission should be
MACHINE DESIGN - An Integrated Approach, 4th Ed. 7-40-1

PROBLEM 7-40
Statement: A machine has a tripod base that utilizes 15-mm-dia Nylon 11 balls as support pads at its feet.
The tripod rests on a flat steel plate. The 360N weight of the machine is distributed equally to
the three legs of the tripod. Determine the size of the contact patch and the contact stresses in
the nylon balls. Assume that Poisson's ratio for Nylon is 0.25.

Given: Ball radius R1  7.5 mm Material properties


Plate curvature R2  ∞ mm Nylon ball ν1  0.25
Load F  120  N E1  1.3 GPa
Steel plate ν2  0.28
E2  207  GPa

Solution: See Mathcad file P0740.


1. Calculate geometry and material constants, contact patch dimension, and pressures.

  
1 1 1 1
Geometry constant B    B  0.067 mm
2  R1 R2 
2
1  ν1 4 1
Material constants m1  m1  7.212  10
E1 MPa
2
1  ν2 6 1
m2  m2  4.452  10
E2 MPa
1
Contact patch 3
 3 m1  m2 
radius a    F a  0.788 mm
8 B 
2 2
Contact area A  π a A  1.952 mm
F
Average pressure p avg  p avg  61 MPa
A

3
Maximum pressure p max   p avg p max  92 MPa
2
2. Determine the stresses in the ball at the surface
Axial σzmax  p max σzmax  92 MPa

1  2  ν1
In-plane σxmax1    p max σxmax1  69 MPa
2
σymax1  σxmax1

3. Determine the stresses in the ball below the surface


 1  2 ν1 2
p max 
Max shear τyzmax1      1  ν1  2   1  ν1
stress 2  2 9 
τyzmax1  31.8 MPa

Depth at max 2  2  ν1
shear stress zτmax1  a  zτmax1  0.489 mm
7  2  ν1

© 2011 Pearson Education, Inc., Upper Saddle River, NJ. All rights reserved. This publication is protected by Copyright and written permission should be
MACHINE DESIGN - An Integrated Approach, 4th Ed. 7-41-1

PROBLEM 7-41
Statement: A ball bearing consists of a number of balls (separated by a ball cage) and two rings with
raceways as shown in Figure P7-7. The raceways have compound curvature. In a plane
containing the axis of the bearing the curvature is concave and conforms closely to the ball
radius. In a plane perpendicular to the axis the curvature is convex for the inner raceway and is
related to the bore size of the bearing. Determine the size of the contact patch and the maximum
contact stresses between a ball and the inner raceway with a radial load of 5200 N in a steel
bearing with the following dimensions: ball dia = 8 mm, raceway radius for concave surface =
4.05 mm, raceway radius for convex surface = 13 mm.
Given: Ball radii R1  4.00 mm The parts are steel. Therefore
R'1  4.00 mm E  206.8  GPa ν  0.28
Raceway radii R2  13.0 mm (radial)
R'2  4.05 mm (axial)
Radial load F  5200 N (normal to contact plane)
Angle between planes of R1 & R2 θ  0  deg

Solution: See Mathcad file P0741.

1. Find the material constants from equation 7.9b.


2
1ν 6 1
Material constants m1  m1  4.456  10 
E MPa
m2  m1

2. Two geometry constants are needed from equations 7.19a.

  
Geometry 1 1 1 1 1 1
constants A      A  0.1650 mm
2  R1 R'1 R2 R'2 
1
2
 1 2 2 
B   
1 1   1  1    1
  R  B  0.1619 mm
2  R1 R'1   2 R'2  
 1 1  1 1  
 2  R  R'    R  R'   cos( 2  θ ) 
  1 1  2 2 

ϕ  acos  
B 180
Angle  ϕ  11.10
A π

Factors from  0.86215


equations 7.19e ka  50.192 ϕ ka  6.301

2
kb  0.0045333  0.043581 ϕ  0.0017292  ϕ 
5 3 7 4
 3.7374 10  ϕ  3.7418 10  ϕ 
9 5
 1.4207 10 ϕ

kb  0.321

© 2011 Pearson Education, Inc., Upper Saddle River, NJ. All rights reserved. This publication is protected by Copyright and written permission should be
MACHINE DESIGN - An Integrated Approach, 4th Ed. 7-41-2

3. Determine the contact patch dimensions using the material and geometry constants in equations 7.19d.

1
Major axis 3
 3 m1  m2 
half-width a  ka   F a  3.749  mm
4 A 
1
Minor axis 3
 3 m1  m2 
half-width b  kb   F b  0.191  mm
4 A 
2
Contact area A  π a  b A  2.24904  mm
4. The average and maximum contact pressure can now be found from equations 7.18b and c.
F
Average pressure p avg  p avg  2312 MPa
A

3
Maximum pressure p max   p avg p max  3468 MPa
2

5. The maximum normal stresses in the center of the contact patch at the surface are then found using equations
7.21a.

σx  2  ν  ( 1  2  ν)  
b
In-plane   pmax σx  2016 MPa
 a  b

σy  2  ν  ( 1  2  ν)  
a
  pmax σy  3394 MPa
 a  b

Axial σz  p max σz  3468 MPa

These stresses are principal:

σ1  σx σ2  σy σ3  σz

The maximum shear stress associated with them at the surface is

σ1  σ3
τ13  τ13  726  MPa
2

6. The maximum shear stress under the surface on the z-axis is approximately
Max shear stress τ13max  0.34 p max τ13max  1179 MPa

7. All of the stresses found so far exist on the centerline of the patch. At the edge of the patch, at the surface,
there will also be a shear stress. Two constants are found from equation 7.21b for this calculation.
b
k3  k3  0.051
a

1 2 2
k4   a b k4  0.999
a
8. These constants are used in equations 7.21c and d to find the shear stresses on the surface at the ends of the
major and minor axes.
© 2011 Pearson Education, Inc., Upper Saddle River, NJ. All rights reserved. This publication is protected by Copyright and written permission should be
MACHINE DESIGN - An Integrated Approach, 4th Ed. 7-41-3

   atanh k4  1  p max


k3 1
Major axis τxy  ( 1  2  ν)  τxy  208  MPa
k4
2  k4 

k3  k3  k4  
Minor axis τxy  ( 1  2  ν)  1   atan    pmax τxy  72 MPa
k4
2  k4  k3  

© 2011 Pearson Education, Inc., Upper Saddle River, NJ. All rights reserved. This publication is protected by Copyright and written permission should be
MACHINE DESIGN - An Integrated Approach, 4th Ed. 7-42-1

PROBLEM 7-42
Statement: A pair of steel rollers used in a manufacturing process roll together with a combination of rolling
and sliding. One roller has a diameter of 75 mm and the other has a diameter of 50 mm. They are
both 200-mm long. The contact force, which is normal to the contact plane, is 18500 N. Assuming
that the coefficient of friction between the rollers is 0.33, determine the maximum tensile,
compressive, and shear stresses in the rollers.

Given: Roller radii R1  25 mm The parts are hardened steel. Therefore
R2  37.5 mm E  206.8  GPa ν  0.28
Roller length L  200  mm
Radial load F  18500  N Coefficient of friction μ  0.33

Solution: See Mathcad file P0742.

1. Find the material constants from equation 7.9a.


2
1ν 6 1
Material constants m1  m1  4.456  10
E MPa
m2  m1

  
1 1 1 1
Geometry constant B    B  0.033 mm
2  R1 R2 

1
Contact patch 2
 2 m1  m2 F 
half-width a      a  0.1255 mm
π B L

2. The average and maximum contact pressure can now be found from equations 7.14b and c.
F
Average pressure p avg  p avg  368.6 MPa
2 a L

2 F
Maximum pressure p max  p max  469.3 MPa
π a  L

Tangential fmax  μ  p max fmax  154.9 MPa


unit force

3. With = 0.33, the principal stresses in the contact zone will be maximal on the surface (z = 0) at x = 0.3a from th
centerline as shown in Figures 7-20 and 7-22. The applied stress components are found from equation 7.23a for
the normal force and equation 7.23b for the tangential force.

For x  0.3 a

2
x
σxn  p max  1  σxn  447.7 MPa
2
a

x
σxt  2  fmax  σxt  92.92 MPa
a

2
x
σzn  p max  1  σzn  447.7 MPa
2
a
© 2011 Pearson Education, Inc., Upper Saddle River, NJ. All rights reserved. This publication is protected by Copyright and written permission should be
MACHINE DESIGN - An Integrated Approach, 4th Ed. 7-42-2

σzt  0  MPa τxzn  0  MPa

2
x
τxzt  fmax  1  τxzt  147.7 MPa
2
a

4. Equations 7.24a and 7.24b can now be solved for the total applied stresses along the x, y, and z axes.
σx  σxn  σxt σx  540.591 MPa

σz  σzn  σzt σz  447.672 MPa

τxz  τxzn  τxzt τxz  147.732 MPa

5. Since the rollers are long, we expect a plane strain condition to exist. The stress in the third dimension is found
from equation 7.23b:

σy  ν  σx  σz σy  276.714 MPa

6. Unlike the pure-rolling case, these stresses are not principal because of the applied shear stress. The principal
stresses are found from equation 4.4 using a cubic root finding solution.
σx σy σz
σx  σy  σz 
MPa MPa MPa
τxz
τxy  0 τyz  0 τxz 
MPa
3
C2  σx  σy  σz C2  1.265  10

 σx τxy   σx τxz   σy τyz  5


C1          C1  4.936  10
 τxy σy   τxz σz   τyz σz 

 σx τxy τxz 
  7
C0   τxy σy τyz  C0  6.093  10
τ τ σ 
 xz yz z 
3 2
f ( σ )  σ  C2 σ  C1 σ  C0

 C0 
   649.0 
v   C1 
s  polyroots ( v)  MPa s  339.3  MPa

 C2   
   276.7 
 1 
Principal stresses:
σ1  s σ1  276.7 MPa
3
σ2  s σ2  339.3 MPa
2
σ3  s σ3  649.0 MPa
1

© 2011 Pearson Education, Inc., Upper Saddle River, NJ. All rights reserved. This publication is protected by Copyright and written permission should be
MACHINE DESIGN - An Integrated Approach, 4th Ed. 7-42-3

Maximum shear stress

σ1  σ3
τ13  τ13  186.1 MPa
2

7. The principal stresses are maximum at the surface as seen in Figures 7-20 and 7-22 in the text.

© 2011 Pearson Education, Inc., Upper Saddle River, NJ. All rights reserved. This publication is protected by Copyright and written permission should be
MACHINE DESIGN - An Integrated Approach, 4th Ed. 7-43-1

PROBLEM 7-43
Statement: Repeat Problem 7-41 for contact beween a ball and outer raceway. The outer raceway radius for
concave surface = 4.05 mm and the outer raceway radius for concave surface = 17.02 mm.
Given: Ball radii R1  4.00 mm The parts are steel. Therefore
R'1  4.00 mm E  206.8  GPa ν  0.28
Raceway radii R2  17.02  mm (radial)
R'2  4.05 mm (axial)
Radial load F  5200 N (normal to contact plane)
Angle between planes of R1 & R2 θ  0  deg

Solution: See Figure P7-7 and Mathcad file P0743.


1. Find the material constants from equation 7.9b.
2
1ν 6 1
Material constants m1  m1  4.456  10 
E MPa
m2  m1

2. Two geometry constants are needed from equations 7.19a.

  
Geometry 1 1 1 1 1 1
constants A      A  0.0972 mm
2  R1 R'1 R2 R'2 
1
2
 1 2 2 
B   
1 1   1  1    1
  R  B  0.0941 mm
2  R1 R'1   2 R'2  
 1 1  1 1  
 2  R  R'    R  R'   cos( 2  θ ) 
  1 1  2 2 

ϕ  acos  
B 180
Angle  ϕ  14.48
A π

Factors from  0.86215


equations 7.19e ka  50.192 ϕ ka  5.011

2
kb  0.0045333  0.043581 ϕ  0.0017292  ϕ 
5 3 7 4
 3.7374 10  ϕ  3.7418 10  ϕ 
9 5
 1.4207 10 ϕ
kb  0.371
3. Determine the contact patch dimensions using the material and geometry constants in equations 7.19d.
1
Major axis 3
 3 m1  m2 
half-width a  ka   F a  3.557  mm
4 A 
1
Minor axis 3
 3 m1  m2 
half-width b  kb   F b  0.263  mm
4 A 
© 2011 Pearson Education, Inc., Upper Saddle River, NJ. All rights reserved. This publication is protected by Copyright and written permission should be
MACHINE DESIGN - An Integrated Approach, 4th Ed. 7-43-2

2
Contact area A  π a  b A  2.94254  mm
4. The average and maximum contact pressure can now be found from equations 7.18b and c.
F
Average pressure p avg  p avg  1767 MPa
A

3
Maximum pressure p max   p avg p max  2651 MPa
2

5. The maximum normal stresses in the center of the contact patch at the surface are then found using equations
7.21a.

σx  2  ν  ( 1  2  ν)  
b
In-plane   pmax σx  1565 MPa
 a  b

σy  2  ν  ( 1  2  ν)  
a
  pmax σy  2570 MPa
 a  b

Axial σz  p max σz  2651 MPa

These stresses are principal:


σ1  σx σ2  σy σ3  σz
The maximum shear stress associated with them at the surface is

σ1  σ3
τ13  τ13  543  MPa
2

6. The maximum shear stress under the surface on the z-axis is approximately
Max shear stress τ13max  0.34 p max τ13max  901  MPa

7. All of the stresses found so far exist on the centerline of the patch. At the edge of the patch, at the surface,
there will also be a shear stress. Two constants are found from equation 7.21b for this calculation.
b
k3  k3  0.074
a

1 2 2
k4   a b k4  0.997
a
8. These constants are used in equations 7.21c and d to find the shear stresses on the surface at the ends of the
major and minor axes.

   atanh k4  1  p max


k3 1
Major axis τxy  ( 1  2  ν)  τxy  200  MPa
k4
2  k4 

k3  k3  k4  
Minor axis τxy  ( 1  2  ν)  1   atan    pmax τxy  77 MPa
k4
2  k4  k3  

© 2011 Pearson Education, Inc., Upper Saddle River, NJ. All rights reserved. This publication is protected by Copyright and written permission should be
MACHINE DESIGN - An Integrated Approach, 4th Ed. 7-44-1
PROBLEM 7-44
Statement: A machine has two crowned, cylindrical rollers rolling against each other with a dynamic load of
0 to 3.5 kN. The first roller has a major radius of 14 mm with a crown radius of 80 mm. The
second roller has a major radius of 75 mm and a crown radius of 100 mm. The two axes of
rotation have a 30 degree angle between them. Find the contact stresses if both rollers are
steel.

Given: Roller 1 radius R1  14.00  mm


Crown radius R'1  80.0 mm (90 deg to roller rad)
Roller 2 radius R2  75.00  mm (radial)
Crown radius R'2  100.0  mm (axial)
Radial load F  3.5 kN (normal to contact plane)
Angle between planes of R1 and R2 θ  30 deg
Material properties E1  206.8  GPa ν1  0.28 steel

Assumptions: The relative motion is rolling with < 1% sliding.

Solution: See Mathcad file P0744.


1. Find the material constants from equation 7.9b.
2
1  ν1 3 1
Material constants m1  m1  4.456  10 
E1 GPa

m2  m1

2. Two geometry constants are needed from equations 7.19a.

  
Geometry 1 1 1 1 1 1
constants A      A  0.0536 mm
2  R1 R'1 R2 R'2 
1
2
 1 2 2 
B   
1 1   1  1    1
  R  B  0.0303 mm
2  R1 R'1   2 R'2  
 1 1  1 1  
 2  R  R'    R  R'   cos( 2  θ ) 
  1 1  2 2 

ϕ  acos  
B 180
Angle  ϕ  55.56
A π
Factors from  0.86215
equations 7.19e ka  50.192 ϕ ka  1.572

2
kb  0.0045333  0.043581 ϕ  0.0017292  ϕ 
5 3 7 4
 3.7374 10  ϕ  3.7418 10  ϕ 
9 5
 1.4207 10 ϕ

kb  0.685

© 2011 Pearson Education, Inc., Upper Saddle River, NJ. All rights reserved. This publication is protected by Copyright and written permission should be
MACHINE DESIGN - An Integrated Approach, 4th Ed. 7-44-2

3. Determine the contact patch dimensions using the material and geometry constants in equations 7.19d.

1
Major axis 3
 3 m1  m2 
half-width a  ka   F a  1.192  mm
4 A 
1
Minor axis 3
 3 m1  m2 
half-width b  kb   F b  0.519  mm
4 A 

2
Contact area A  π a  b A  1.94438  mm

4. The average and maximum contact pressure can now be found from equations 7.18b and c.
F
Average pressure p avg  p avg  1800 MPa
A

3
Maximum pressure p max   p avg p max  2700 MPa
2

5. The maximum normal stresses in the steel follower at the center of the contact patch at the surface are then found
using equations 7.21a.

σx  2  ν1   1  2  ν1   p
b
In-plane  max σx  1872 MPa
 a  b

σy  2  ν1   1  2  ν1   p
a
 max σy  2340 MPa
 a  b

Axial σz  p max σz  2700 MPa

These stresses are principal:

σ1  σx σ2  σy σ3  σz

The maximum shear stress associated with them at the surface is

σ1  σ3
τ13  τ13  414  MPa
2

6. The maximum shear stress under the surface on the z-axis is approximately
Max shear stress τ13max  0.34 p max τ13max  918  MPa

7. All of the stresses found so far exist on the centerline of the patch. At the edge of the patch, at the surface,
there will also be a shear stress. Two constants are found from equation 7.21b for this calculation.
b
k3  k3  0.436
a

1 2 2
k4   a b k4  0.9
a

© 2011 Pearson Education, Inc., Upper Saddle River, NJ. All rights reserved. This publication is protected by Copyright and written permission should be
MACHINE DESIGN - An Integrated Approach, 4th Ed. 7-44-3

8. These constants are used in equations 7.21c and d to find the shear stresses on the surface at the ends of the
major and minor axes.

   atanh k4  1  p max


k3
τxy   1  2  ν1 
1
Major axis τxy  406  MPa
2
k4  k4 

k3  k3  k4  
Minor axis τxy   1  2  ν1  1   atan    p max τxy  292  MPa
2
k4  k4  k3  

© 2011 Pearson Education, Inc., Upper Saddle River, NJ. All rights reserved. This publication is protected by Copyright and written permission should be
MACHINE DESIGN - An Integrated Approach, 4th Ed. 7-45-1

PROBLEM 7-45
Statement: A cam-follower system has a motion with a combination of rolling and sliding. The cam is
cylindrical with a minimum radius of curvature of 80 mm. The roller follower is also cylindrical
with a radius of 14 mm. They are both 18-mm long. The maximum contact force, which is normal
to the contact plane, is 3200 N. Both the cam and roller are made from hardened steel. Assuming
that the coefficient of friction between the cam and roller follower is 0.33, determine the maximum
tensile, compressive, and shear stresses in the cam.

Given: Roller radius R1  14 mm The parts are hardened steel. Therefore
Cam radius R2  80 mm E  206.8  GPa ν  0.28
Roller length L  18 mm
Radial load F  3200 N Coefficient of friction μ  0.33

Solution: See Mathcad file P0742.


1. Find the material constants from equation 7.9a.
2
1ν 6 1
Material constants m1  m1  4.456  10 
E MPa
m2  m1

  
1 1 1 1
Geometry constant B    B  0.042  mm
2  R1 R2 

1
Contact patch 2
 2 m1  m2 F 
half-width a      a  0.1550 mm
π B L

2. The average and maximum contact pressure can now be found from equations 7.14b and c.
F
Average pressure p avg  p avg  573.3  MPa
2 a L

2 F
Maximum pressure p max  p max  730.0  MPa
π a  L

Tangential fmax  μ  p max fmax  240.9  MPa


unit force

3. With = 0.33, the principal stresses in the contact zone will be maximal on the surface (z = 0) at x = 0.3a from th
centerline as shown in Figures 7-20 and 7-22. The applied stress components are found from equation 7.23a for
the normal force and equation 7.23b for the tangential force.

For x  0.3 a

2
x
σxn  p max  1  σxn  696.4  MPa
2
a

x
σxt  2  fmax  σxt  144.53 MPa
a

2
x
σzn  p max  1  σzn  696.4  MPa
2
a
© 2011 Pearson Education, Inc., Upper Saddle River, NJ. All rights reserved. This publication is protected by Copyright and written permission should be
MACHINE DESIGN - An Integrated Approach, 4th Ed. 7-45-2

σzt  0  MPa τxzn  0  MPa

2
x
τxzt  fmax  1  τxzt  229.8  MPa
2
a

4. Equations 7.24a and 7.24b can now be solved for the total applied stresses along the x, y, and z axes.
σx  σxn  σxt σx  840.886  MPa

σz  σzn  σzt σz  696.351  MPa

τxz  τxzn  τxzt τxz  229.796  MPa

5. Since the contact patch is short, we expect a plane stress condition to exist. The stress in the third dimension is:

σy  0  MPa σy  0.000  MPa

6. Unlike the pure-rolling case, these stresses are not principal because of the applied shear stress. The principal
stresses are found from equation 4.4 using a cubic root finding solution.
σx σy σz
σx  σy  σz 
MPa MPa MPa
τxz
τxy  0 τyz  0 τxz 
MPa
3
C2  σx  σy  σz C2  1.537  10

 σx τxy   σx τxz   σy τyz  5


C1          C1  5.327  10
 τxy σy   τxz σz   τyz σz 

 σx τxy τxz 
 
C0   τxy σy τyz  C0  0
τ τ σ 
 xz yz z 
3 2
f ( σ )  σ  C2 σ  C1 σ  C0

 C0 
   1009.5 
v   C1 
s  polyroots ( v)  MPa s  527.7   MPa

 C2   
   0.0 
 1 
Principal stresses:
σ1  s σ1  0  MPa
3
σ2  s σ2  527.7  MPa
2
σ3  s σ3  1009.5 MPa
1

© 2011 Pearson Education, Inc., Upper Saddle River, NJ. All rights reserved. This publication is protected by Copyright and written permission should be
MACHINE DESIGN - An Integrated Approach, 4th Ed. 7-45-3

Maximum shear stress

σ1  σ3
τ13  τ13  504.8  MPa
2

7. The principal stresses are maximum at the surface as seen in Figures 7-20 and 7-22 in the text.

© 2011 Pearson Education, Inc., Upper Saddle River, NJ. All rights reserved. This publication is protected by Copyright and written permission should be
MACHINE DESIGN - An Integrated Approach, 4th Ed. 7-46-1
PROBLEM 7-46
Statement: Estimate how long it will take to remove 2 μm of material from the 5000 mm2 surface of a block
of HB110 steel if a coarse polishing machine applies 80 N over a 400-mm stroke at 120 strokes
per minute.
(a) If done dry.
(b) If done lubricated.

Units: kilo  kg g


2 1
Given: Block surface Aa  5000 mm Stroke rate n  120  min
Depth of wear d  2  μm Stroke s  400  mm
kilo
Force on file F  80 N Steel hardness HB  110 
2
mm
Assumptions: Only one face will be polished.
Solution: See Mathcad file P0746.
1. This is a two-body abrasion problem. From Table 7-2, the wear coefficients for dry and lubricated abrasion
in a coarse polishing operation are

4
Wear coefficients Ka  1  10 dry
4
Kb  2  10 lubricated

2. The length of sliding is L = s n  t where t is the time required.

F L
3. The depth of material removed is d = K
HB Aa

4. Combining these two equations and solving for the time, t

d  HB Aa
(a) dry ta  ta  28 min
Ka F  s n

strokesa  ta n strokesa  3371

d  HB Aa
(b) lubricated tb  tb  14 min
Kb F  s n

strokesb  tb n strokesb  1686

© 2011 Pearson Education, Inc., Upper Saddle River, NJ. All rights reserved. This publication is protected by Copyright and written permission should be
MACHINE DESIGN - An Integrated Approach, 4th Ed. 7-47-1

PROBLEM 7-47 _____

Statement: Loose abrasive grains are introduced in error into the lubricating system of a flat bronze thrust
bearing that has a hardness of 60HB and a surface area of 500 mm2. If a hardened steel part
exerts a force of 50 N on the bearing while oscillating across it at 200 strokes/min with a stroke
of 30 mm, what depth of wear will occur in an 8-hour shift?
Units: stroke  1
2
Given: Hardness H  60 kgf  mm
2
Shift time t  8  hr Area of wear Aa  500  mm
3
Wear coefficient K  2  10 Stroke s  30 mm
1
Stroke rate n  200  stroke min
Contact force F  50 N

Solution: See Mathcad file P0747.

1. From equation 7.7b, the depth of wear is

F l
d = K
H  Aa
2. The length of wear is,

l = s n  t where t is the elapsed time

3. Substituting for l ,

F  s n  t
d  K d  1.0 mm
H  Aa

© 2011 Pearson Education, Inc., Upper Saddle River, NJ. All rights reserved. This publication is protected by Copyright and written permission should be
MACHINE DESIGN - An Integrated Approach, 4th Ed. 7-48-1
PROBLEM 7-48
Statement: Two rollers are in contact with a 9% sliding combined with rolling and the resulting maximum
compressive principal stress in the contact zone is 15500 psi. Both rollers are made from 6061-T6
hard anodized aluminum. The design life of the rollers is 4 years of 2-shift operation at 260
days/year and they each turn at 200 rpm. What is the expected safety factor against pitting for
the roller pair?
6
Given: Roller material properties E  10.4 10  psi ν  0.34
1
Roller speed n  200  rpm shift  2  day hrspershift  8  hr
1
Max stress σmax  15500  psi daysperyr  260  day yr
Design life life d  4  yr

Solution: See Mathcad file P0748.

1. Calculate the required cycle life.


9
cycles  n  shift hrspershift daysperyr life d cycles  1.255  10
2. Calculate the material constants.
2
1ν 8 1
m1  m1  8.504  10 m2  m1
E psi

3. The maximum normal stress is σmax  15.5 ksi, compressive. Its K-factor can be calculated from equation 7.25d.

K  π  m1  m2  σmax
2
K-factor K  128.4  psi

4. From Table 7-7, Part 2, Line 36 the slope and intercept factors of this aluminum for rolling with 9% sliding are

λ  5.02 ζ  20.12

5. These are used in equation 7.26 along with the value of K from above to find the number of cycles that can be
expected at this load before pitting begins.

ζ  log  Nlife
log ( K) =
λ

ζ λ log 
 K 
Nlife  10  psi  Nlife  3.4  10
9
cycles

6. Calculate the factor of safety against pitting.

Nlife
Nf  Nf  2.7
cycles

© 2011 Pearson Education, Inc., Upper Saddle River, NJ. All rights reserved. This publication is protected by Copyright and written permission should be
MACHINE DESIGN - An Integrated Approach, 4th Ed. 7-49-1

PROBLEM 7-49 _____


Statement: Two contacting rollers run together in pure rolling. Both are made from Class 20 Gray iron, HB
130-180. One roller has a diameter of 2.75 in and the other has a diameter of 3.25 in. Both are 10
in long. The applied load is 5500 lbf. If the design life is 1E08 cycles, determine the factor of
safety against pitting failure.

Given: Material data from Table 7-7 K  960  psi


Roller radii R1  2.75 in R2  3.25 in
Roller length L  10.0 in
Applied force F  5500lbf

Solution: See Mathcad file P0749.

1. Find the geometry constant from equation 7.9a.

  
1 1 1 1
Geometry constant B    B  0.336 
2  R1 R2  in

2. Calculate the allowable force on the rollers using equation 7.25d.

K L
Allowable force Fallow  Fallow  14300  lbf
2 B

3. Calculate the factor of safety using equation f in Example 7-5.

Fallow
Safety factor Nf  Nf  2.6
F

© 2011 Pearson Education, Inc., Upper Saddle River, NJ. All rights reserved. This publication is protected by Copyright and written permission should be
MACHINE DESIGN - An Integrated Approach, 4th Ed. 10-1a-1
PROBLEM 10-1a
Statement: A simply supported shaft is shown in Figure P10-1. A constant magnitude transverse load P is
applied as the shaft rotates subject to a time-varying torque that varies from Tmin to Tmax. For
the data in row a of Table P10-1, find the diameter of shaft required to obtain a safety factor of 2
in fatigue loading if the shaft is steel of S ut = 108 ksi and S y = 62 ksi. The dimensions are in
inches, the force in pounds, and the torque is in lb-in. Assume no stress concentrations are
present.

Given: Distance between bearings a  16 in Distance to P b  18 in


Applied load P  1000 lbf Tensile strength S ut  108  ksi
Minimum torque Tmin  0  lbf  in Yield strength S y  62 ksi
Maximum torque Tmax  2000 lbf  in Design safety factor Nd  2

Assumptions: The finish is machined, reliability is 99%, and the shaft is at room temperature.
Solution: See Figure P10-1 and Mathcad file P1001a.
1. The maximum moment in the shaft occurs at the right bearing as seen in the moment diagram in Figure B-3(a)
in Appendix B (note that in the figure a is the distance to the load and b is the distance between bearings).
Using the equation given in the figure, calculate the alternate bending moment (the mean is zero).

Ma  P ( a  b ) Ma  2000 in lbf

2. Calculate the mean and alternating components of torque.

Tmax  Tmin
Tm  Tm  1000 in lbf
2
Tmax  Tmin
Ta  Ta  1000 in lbf
2

3. Calculate the unmodified endurance limit. S'e  0.5 S ut S'e  54 ksi

4. Determine the endurance limit modification factors for a rotating round shaft.

Load Cload  1
 0.097
Csize( d )  0.869   
d
Size 
 in 

Surface A  2.70 b  0.265 (machined)

b
 S ut 
Csurf  A    Csurf  0.781
 ksi 
Temperature Ctemp  1

Reliability Creliab  0.814 (R = 99%)

5. Determine the modified endurance limit as a function of the unknown diameter, d.


S e( d )  Cload  Csize( d )  Csurf  Ctemp Creliab S'e

6. Use equation (10.8) with unity for all stress concentration factors as a design equation to find d.

© 2011 Pearson Education, Inc., Upper Saddle River, NJ. All rights reserved. This publication is protected by Copyright and written permission should be
MACHINE DESIGN - An Integrated Approach, 4th Ed. 10-1a-2

Guess d  1  in
Given
1
3
  2 3 2 
 32 Nd  Ma  4  Ta 3 Tm

d=    
 π  S e( d ) 4 S ut 

d  Find ( d ) d  1.188  in

Using this value for d, the size modification factor and endurance limit are:

Size modification factor Csize( d )  0.855

Endurance limit S e( d )  29.3 ksi

© 2011 Pearson Education, Inc., Upper Saddle River, NJ. All rights reserved. This publication is protected by Copyright and written permission should be
MACHINE DESIGN - An Integrated Approach, 4th Ed. 10-2a-1
PROBLEM 10-2a
Statement: A simply supported shaft is shown in Figure P10-2. A constant magnitude distributed load p is
applied as the shaft rotates subject to a time-varying torque that varies from Tmin to Tmax. For the
data in row a of Table P10-1, find the diameter of shaft required to obtain a safety factor of 2 in
fatigue loading if the shaft is steel of S ut = 745 MPa and Sy = 427 MPa. The dimensions are in cm,
the distributed force in N/cm, and the torque is in N-m. Assume no stress concentrations are
present.

Given: Dist. between bearings L  20 cm Distance to p a  16 cm


Distance to end of p b  18 cm Tensile strength S ut  745  MPa
1
Applied distributed load p  1000 N  cm Yield strength S y  427  MPa
Minimum torque Tmin  0  N  m Design safety factor Nfd  2
Maximum torque Tmax  2000 N  m

Assumptions: The finish is machined, reliability is 99%, and the shaft is at room temperature.
Solution: See Figure P10-2 and Mathcad file P01002a.
1. The maximum moment in the shaft occurs between a and b. See the appendix to this problem below for the
determination of Ma.

Ma  48.45  N  m
2. Calculate the mean and alternating components of torque.

Tmax  Tmin
Tm  Tm  1000 N  m
2
Tmax  Tmin
Ta  Ta  1000 N  m
2

3. Calculate the unmodified endurance limit. S'e  0.5 S ut S'e  372.5  MPa

4. Determine the endurance limit modification factors for a rotating round shaft.

Load Cload  1
 0.097
Csize( d )  1.189   
d
Size 
 mm 

Surface A  4.51 b'  0.265 (machined)

b'
 Sut 
Csurf  A    Csurf  0.782
 MPa 
Temperature Ctemp  1

Reliability Creliab  0.814 (R = 99%)

5. Determine the modified endurance limit as a function of the unknown diameter, d.

S e( d )  Cload  Csize( d )  Csurf  Ctemp Creliab S'e

6. Use equation (10.8) with unity for all stress concentration factors as a design equation to find d.
© 2011 Pearson Education, Inc., Upper Saddle River, NJ. All rights reserved. This publication is protected by Copyright and written permission should be
MACHINE DESIGN - An Integrated Approach, 4th Ed. 10-2a-2

Guess d  1  in
Given
1
3
  2 3 2 
 32 Nfd  Ma  4  Ta 3 Tm

d=    
 π  S e( d ) 4 S ut 

d  Find ( d ) d  48.6 mm

Using this value for d, the size modification factor and endurance limit are:

Size modification factor Csize( d )  0.816

Endurance limit S e( d )  193.4  MPa

APPENDIX - Maximum bending moment


1. From inspection of Figure P10-2, write the load function equation

q(x) = R1<x - 0>-1 - p<x - a>0 + p<x - b>0 + R2<x - L>-1

2. Integrate this equation from - to x to obtain shear, V(x)

V(x) = R1<x - 0>0 - p<x - a>1 + p<x - b>1 + R2<x - L>0

3. Integrate this equation from - to x to obtain moment, M(x)

M(x) = R1<x - 0>1 - p<x - a>2/2 + p<x - b>2/2 + R2<x - L>1

4. Solve for the reactions by evaluating the shear and moment equations at a point just to the right of x = L,
where both are zero.
At x = L+, V = M = 0

V = R1  p  ( L  a )  p  ( L  b )  R2 = 0

p 2 p 2
M = R 1 L   ( L  a)   ( L  b) = 0
2 2

 ( L  a )  ( L  b )
p 2 2
R1   R1  300  N
2 L

R2  p  ( b  a )  R1 R2  1700 N

5. Define the range for x x  0  m 0.005  L  L

6. For a Mathcad solution, define a step function S. This function will have a value of zero when x is less than
z, and a value of one when it is greater than or equal to z.

S ( x z)  if ( x  z 1 0 )


7. Write the shear and moment equations in Mathcad form, using the function S as a multiplying factor to get the
effect of the singularity functions.

© 2011 Pearson Education, Inc., Upper Saddle River, NJ. All rights reserved. This publication is protected by Copyright and written permission should be
MACHINE DESIGN - An Integrated Approach, 4th Ed. 10-2a-3

V ( x)  R1 S ( x 0  m)  p  S ( x a )  ( x  a )  p  S ( x b )  ( x  b )  R2 S ( x L)


p 2 p 2
M ( x)  R1 S ( x 0  m)  x   S ( x a )  ( x  a )   S ( x b )  ( x  b )
2 2

8. Plot the shear and moment diagrams.

Shear Diagram
500

V ( x)  500
N
 1000

 1500

 2000
0 5 10 15 20
x
cm
Moment Diagram
50

35

M ( x)
20
Nm

 10
0 5 10 15 20
x
cm
FIGURE 10-2a
Shear and Moment Diagrams for Problem 10-2a

9. Determine the maximum maximum moment from inspection of the diagrams. Maximum moment occurs where
zero, which is x = c. From the shear diagram,

ca bc a  R2  b  R1
= c  c  16.300 cm
R1 R2 R1  R2

Mmax  M ( c) Mmax  48.45  N  m

© 2011 Pearson Education, Inc., Upper Saddle River, NJ. All rights reserved. This publication is protected by Copyright and written permission should be
MACHINE DESIGN - An Integrated Approach, 4th Ed. 10-3-1
PROBLEM 10-3
Statement: For the bicycle pedal-arm assembly in Figure P6-1 assume a rider-applied force that ranges from
0 to 1500 N at the pedal each cycle. Design a suitable shaft to connect the two pedal arms. Use
a fatigue safety factor of 2 and a material with S ut = 500 MPa. The shaft has a square extension
on each end where it inserts into the pedal arms.
Given: Material tensile strength S ut  500  MPa Design safety factor Nfd  2
Applied load by rider Fridermax  1500 N Fridermin  0  N

Assumptions: 1. The configuration is similar to that shown in Figure 10-3 with the dimensions a, b, and c below.
2. The moments in the shaft are greater when the right pedal is loaded.
3. The torque is transmitted from the sprocket to the shaft through the square shaft extension.
4. The shaft is machined, reliability is 50%, and the bicycle is not operated in extreme
temperatures.
Shaft dimensions: a  10 mm b  22 mm c  80 mm
Solution: See Figure 10-3 and Mathcad file P1003.

1. From problems 3-3 and


Sprocket Frame
7-3, the moment, torque, Fchain Bearing (2 Places)
and chain force Fchain
R 1z
are: Pedal Arm (Both Ends)
Mmax  90 N  m
T
Mmin  90 N  m x
T
Tmax  255  N  m

Tmin  0  N  m R 2z
z
d sprocket  100  mm Horizontal Section (Top View)
y
2  Tmax
Fchainmax  c
d sprocket a
R 2y
Frider
Fchainmax  5100 N

2. Note that in Problem 3-3 the d M x


pedal-arm torque becomes a
concentrated bending
moment on this shaft and
the pedal-arm bending R 1y
moment becomes the b
sprocket torque, which is
carried along the shaft from Vertical Section (Front View)
the y-axis to the centerplane
FIGURE 10-3A
of the sprocket, the distance
a (see Figure 10-3A). Free Body Diagram for Problem 10-3

3. Find the moment distribution along the shaft from x = 0 to x = c. This will be done by finding the moment paral
to the z-axis by looking at the forces and moments in the x-y plane, then finding the moment parrallel to the y-ax
by looking at the forces in the x-z plane. These two moment distributions will be added vectorially to get the
total moment distribution.

4. Determine the bearing reactions R1y and R2y in the x-y plane.
 Fy: Frider  R1y  R2y = 0
© 2011 Pearson Education, Inc., Upper Saddle River, NJ. All rights reserved. This publication is protected by Copyright and written permission should be
MACHINE DESIGN - An Integrated Approach, 4th Ed. 10-3-2

 Mz: M  b  Frider  ( c  b )  R2y = 0

Mmax  b  Fridermax
R2ymax  R2ymax  2121 N
cb

R1ymax  Fridermax  R2ymax R1ymax  3621 N

5. Use singularity functions to determine the moment distribution of the forces in the x-y plane.
q xy = -M<x>-2 - Frider<x>-1 + R1y<x - b>-1 - R2y<x - c>-1

Vxy = -M<x>-1 - Frider<x>0 + R1y<x - b>0 - R2y<x - c>0

Mxy = -M<x>0 - Frider<x>1 + R1y<x - b>1 - R2y<x - c>1

6. Define the range for x x  0  mm 0.005  c  c

7. For a Mathcad solution, define a step function S. This function will have a value of zero when x is less than z,
and a value of one when it is greater than or equal to z.

S ( x u )  if ( x  u 1 0 )


8. Write the moment equation in Mathcad form, using the function S as a multiplying factor to get the effect of the
singularity functions.

Mxy( x)  Mmax  Fridermax S ( x 0  mm)  x  R1ymax S ( x b )  ( x  b )  R2ymax S ( x c)  ( x  c)

9. Plot the moment diagram (see Figure 10-3B).


0
10. Determine the bearing reactions R1z and
R2z in the x-z plane.
 50
 Fz : Fchain  R1z  R2z = 0
Mxy ( x)
 Mz: b  Fchain  ( c  b )  R2z = 0 Nm
 100
b  Fchainmax
R2zmax 
cb

R2zmax  1934 N  150


0 20 40 60 80
x
R1zmax  Fchainmax  R2zmax
mm
FIGURE 10-3B
R1zmax  7034 N
Moment Diagram for x-y Plane for Problem 10-3

11. Use singularity functions to determine the moment distribution of the forces in the x-z plane.
q xz = - Fchain<x>-1 + R1z<x - b>-1 - R2z<x - c>-1

Vxz = - Fchain<x>0 + R1z<x - b>0 - R2z<x - c>0

Mxz = - Fchain<x>1 + R1z<x - b>1 - R2z<x - c>1

© 2011 Pearson Education, Inc., Upper Saddle River, NJ. All rights reserved. This publication is protected by Copyright and written permission should be
MACHINE DESIGN - An Integrated Approach, 4th Ed. 10-3-3

12. Write the moment equation in Mathcad form, using the function S as a multiplying factor to get the effect of the
singularity functions.

Mxz( x)  Fchainmax S ( x 0  mm)  x  R1zmax  S ( x b )  ( x  b )  R2zmax  S ( x c)  ( x  c)

13. Plot the moment diagram (see Figure 10-3C).

2 2
14. Determine the total moment distribution. Mtot ( x)  Mxy( x)  Mxz( x) This is plotted in Figure 10-3D.

0 200

150
 50
Mxz( x) Mtot ( x)
100
Nm Nm
 100
50

 150 0
0 20 40 60 80 0 20 40 60 80
x x
mm mm
FIGURE 10-3C FIGURE 10-3D
Moment Diagram for x-z Plane for Problem 10-3 Total Moment Diagram for Problem 10-3

15. From Figure 10-3D, we see that the moment is a maximum at x = b. However, the torque is carried only from x =
to x = a so we will investigate the section just to the right of x = a where there is a stress concentration due to
the shoulder.

Moment at x = a Mamax  Mtot ( a ) Mamax  116.73 N  m

Moment at x = b Mbmax  Mtot ( b ) Mbmax  166.487  N  m


16. The bending moment is fully reversed while the torque is repeated from zero to Tmax. Therefore, we have the
following alternating and mean components of moment and torque at x = a.

Alternating moment Ma  Mamax Ma  116.73 N  m

Mean moment Mm  0  N  m Mm  0  N  m

Alternating torque Ta  0.5 Tmax Ta  127.5  N  m

Mean torque Tm  0.5 Tmax Tm  127.5  N  m

16. There will be a stress concentration due to the step just to the right of the sprocket and also because of the
transition from square to round at the shoulder. Estimate this to be Kf  2. Also, let

Kfm  Kf Kfs  Kf Kfsm  Kfs

17. Calculate the unmodified endurance limit. S'e  0.5 S ut S'e  250  MPa

© 2011 Pearson Education, Inc., Upper Saddle River, NJ. All rights reserved. This publication is protected by Copyright and written permission should be
MACHINE DESIGN - An Integrated Approach, 4th Ed. 10-3-4

18. Calculate the endurance limit modification factors for a rotating round beam.

Load Cload  1
 0.097
Csize( d )  1.189    d
Size 
 mm 

Surface A  4.51 b  0.265 (machined)

b
 Sut 
Csurf  A    Csurf  0.869
 MPa 
Temperature Ctemp  1

Reliability Creliab  1.0 (R = 50%)


19. Calculate the modified endurance limit.
S e( d )  Cload  Csize( d )  Csurf  Ctemp Creliab S'e

20. Use equation (10.8) to solve for the diameter at x = a. Guess d  12 mm

Given
1
3
  2
 32 Nfd   Kf  Ma  4   Kfs Ta Kfm Mm   Kfsm Tm
2 3 2 2 3
 
4
d=   
 π  S e( d ) S ut 

d  Find ( d ) d  35.526 mm

This is too big for a practical design. A stronger material should be chosen to bring this down to about 16 mm.

© 2011 Pearson Education, Inc., Upper Saddle River, NJ. All rights reserved. This publication is protected by Copyright and written permission should be
MACHINE DESIGN - An Integrated Approach, 4th Ed. 10-4a-1
PROBLEM 10-4a
Statement: Determine the maximum deflections in torsion and in bending of the shaft shown in Figure P10-1
for the data in row a in Table P10-1 if the steel shaft diameter is 1.75 in.
Given: Shaft length L  20 in Distance between bearings a  16 in
Distance to P b  18 in Shaft diameter d  1.750  in
6
Applied load P  1000 lbf Young's modulus E  30 10  psi
6
Maximum torque Tmax  2000 lbf  in Modulus of rigidity G  11.5 10  psi

Solution: See Figure P10-1 and Mathcad file P1004a.

1. Calculate the area moment of inertia and the polar moment of inertia.
4
π d 4
Area moment of inertia I  I  0.460  in
64
4
π d 4
Polar moment of inertia J  J  0.921  in
32

2. Use the equation in Figure B-3(a) in Appendix B for this overhung beam with concentrated load to determine
the maximum deflection in bending. Note that in the figure a is the distance to the load and b is the distance
between bearings. This deflection occurs at the right end of the shaft, under the load P.

ab
   ( L  a )  ( L  b )  a  ( b  a )  L
P 3 b 3 3
ymax  L  ymax  0.00357  in
6  E I  a a 
3. Calculate the torsional deflection using equation (10.9a).

Tmax L
θmax  θmax  0.216  deg
G J

© 2011 Pearson Education, Inc., Upper Saddle River, NJ. All rights reserved. This publication is protected by Copyright and written permission should be
MACHINE DESIGN - An Integrated Approach, 4th Ed. 10-5a-1
PROBLEM 10-5a
Statement: Determine the maximum deflections in torsion and in bending of the shaft shown in Figure P10-2
for the data in row a in Table P10-1 if the steel shaft diameter is 40 mm.

Given: Distance between bearings L  20 cm Distance to p a  16 cm


Distance to end of p b  18 cm Shaft diameter d  40 mm
1
Applied distributed load p  1000 N  cm Young's modulus E  206.8  GPa
Minimum torque Tmin  0  N  m Mod of rigidity G  80.8 GPa
Maximum torque Tmax  2000 N  m

Solution: See Figure P10-2 and Mathcad file P1005a.


1. Calculate the area moment of inertia and the polar moment of inertia.
4
π d 4
Area moment of inertia I  I  125664 mm
64
4
π d 4
Polar moment of inertia J  J  251327 mm
32

2. Calculate the torsional deflection using equation (10.9a).

Tmax L
θmax  θmax  1.129  deg
G J

3. From inspection of Figure P10-2, write the load function equation and integrate it twice to get the shear and
moment equations.
q(x) = R1<x - 0>-1 - p<x - a>0 + p<x - b>0 + R2<x - L>-1
V(x) = R1<x - 0>0 - p<x - a>1 + p<x - b>1 + R2<x - L>0
M(x) = R1<x - 0>1 - p<x - a>2/2 + p<x - b>2/2 + R2<x - L>1

4. Solve for the reactions by evaluating the shear and moment equations at a point just to the right of x = L,
where both are zero.
At x = L+, V = M = 0
V = R1  p  ( L  a )  p  ( L  b )  R2 = 0

p 2 p 2
M = R 1 L   ( L  a)   ( L  b) = 0
2 2

 ( L  a )  ( L  b )
p 2 2
R1   R1  300  N
2 L

R2  p  ( b  a )  R1 R2  1700 N

5. Integrate twice more to get the slope and deflection equations.


(x) = [R1<x - 0>2/2 - p<x - a>3/6 + p<x - b>3/6 + R2<x - L>2/2 + C3]/EI

y(x) = [R1<x - 0>3/6 - p<x - a>4/24 + p<x - b>4/24 + R2<x - L>3/6 + C3x + C4]/EI
6. Solve for the constants of integration with the boundary conditions y = 0 at x = 0 and x = L.
At x = 0 C4 = 0

© 2011 Pearson Education, Inc., Upper Saddle River, NJ. All rights reserved. This publication is protected by Copyright and written permission should be
MACHINE DESIGN - An Integrated Approach, 4th Ed. 10-5a-2

R1 3 p 4 p 4
At x = L 0= L   ( L  a)   ( L  b )  C 3 L
6 24 24

1 p 4 p 4 R1 3 2
C3    ( L  a)   ( L  b)  L  C3  1.950  N  m
L  24 24 6 

7. Define the range for x x  0  m 0.005  L  L

8. For a Mathcad solution, define a step function S. This function will have a value of zero when x is less than z,
and a value of one when it is greater than or equal to z.

S ( x z)  if ( x  z 1 0 )


9. Write the slope and deflection equations in Mathcad form, using the function S as a multiplying factor to get
the effect of the singularity functions.

1  R1 2 p 3 p 3 
θ ( x)    S ( x 0  mm)  x   S ( x a )  ( x  a )   S ( x b )  ( x  b )  C3
E I  2 6 6 

1  R1 3 p 4 p 4 
y ( x)    S ( x 0  mm)  x   S ( x a )  ( x  a )   S ( x b )  ( x  b )  C3 x
E I  6 24 24 
10. Plot the slope and deflection diagrams (see Figure 10-5).

11. The maximum deflection occurs at the value of x for which  is zero. Let this be x = c, then guess c  10 cm
Given θ ( c) = 0 c  Find ( c) c  11.402 cm

12. The maximum deflection occurs at x = c and is ymax  y ( c) ymax  0.00570  mm

SLOPE DEFLECTION
0.15 0

0.1
3
 2 10
0.05
θ( x) y ( x)
3 mm
10
0
3
 4 10

 0.05

3
 0.1  6 10
0 5 10 15 20 0 5 10 15 20
x x
FIGURE 10-5 cm cm
Slope and Deflection Diagrams for Problem 10-5a

© 2011 Pearson Education, Inc., Upper Saddle River, NJ. All rights reserved. This publication is protected by Copyright and written permission should be
MACHINE DESIGN - An Integrated Approach, 4th Ed. 10-6a-1

PROBLEM 10-6a
Statement: Determine the size of key necessary to give a safety factor of at least 2 against both shear and
bearing failure for the design shown in Figure P10-3 using the data from row a in Table P10-1 and
the data given below. Assume a shaft diameter of 1.75 in.

Given: Shaft diameter d  1.750  in Shaft properties: S ut1  108  ksi


Maximum torque Tmax  2000 lbf  in S y1  62 ksi
Minimum torque Tmin  0  lbf  in Key properties: S ut2  88 ksi
Design safety factor Nd  2 S y2  52 ksi

Assumptions: 1. The finish is machined, reliability is 90%, and the shaft is at room temperature.
2. Note that the given magnitudes of the radial forces shown acting on gear(s) in this problem
are not necessarily consistent with a load associated with the given torque for any real gear of
reasonable pressure angle. Since gears are taken up in a later chapter, these shaft design
problems ignore the real gear loadings and use an arbitrary value to provide a shaft design
exercise.
Solution: See Figure P10-3 and Mathcad file P1006a.

1. As recommended in Table 10-2, for a shaft diameter of d  1.75 in, use a square key of width
Key width w  0.375  in
2. Determine the alternating and mean key shear force components.

Mean and alternating 1 Tmax  Tmin


force components Fa   Fa  1143 lbf
2 0.5 d

1 Tmax  Tmin
Fm   Fm  1143 lbf
2 0.5 d
3. Write the equations for the mean and alternating components of the shear stress and the effective von Mises
stress.
Key shear area as
function of length Ashear = w L

Mean and alternating Fa Fm


shear stresses τa = τm =
w L w L

Mean and alternating Fa Fm


von Mises stresses σ'a = 3  τa = 3  σ'm = 3  τm = 3 
w L w L

4. Using the modified Goodman failure criterion, the design equation is

S e S ut S e S ut
Nd = =
S ut σ'a  S e σ'm  3 
   S ut Fa  S e Fm
 w L 

3  Nd   S ut Fa  S e Fm
Solving for L L=
w S ut S e

5. Determine the key material endurance limit.

© 2011 Pearson Education, Inc., Upper Saddle River, NJ. All rights reserved. This publication is protected by Copyright and written permission should be
MACHINE DESIGN - An Integrated Approach, 4th Ed. 10-6a-2

Uncorrected endurance
strength S'e  0.5 S ut2 S'e  44 ksi

Correction factors:
Load Cload  1
w L
Size A95 = w L d eq =
0.0766

 0.097
Csize = 0.869  d eq

 0.5 0.097
Cs  0.869   
w
 Cs  0.805
 0.0766  in 
 0.0485
Csize = 0.805  L
 0.265
 Sut2 
Surface (machined) Csurf  2.7   Csurf  0.824
 ksi 
Temperature Ctemp  1

Reliability Creliab  0.897 (R = 90%)

 0.0485  0.0485
S e = Cload  Csize Csurf  Ctemp Creliab S'e = 1  Cs L  Csurf  1  Crelaib S'e = Ce L

where Ce  Cs Csurf  Creliab S'e


Ce  26.175 ksi
6. Substitute S e into the design equation.

3  Nd   S ut Fa  Ce L  Fm


 0.0485
L=
 0.0485
w S ut Ce L

Solving by iteration, let

  0.0485 
  L
3  Nd  S ut2 Fa  Ce    Fm
RHS( L) 
  in  
 0.0485
w S ut2 Ce  
L
 in 
Guess L  0.5 in RHS( L)  0.510  in
L  RHS( L) RHS( L)  0.510  in

Tentatively, let L  0.625  in (5/8 in)


7. Check the realised factor of safety against fatigue failure.
 0.0485
S e  Ce 
L
Endurance limit  S e  26.778 ksi
 in 
© 2011 Pearson Education, Inc., Upper Saddle River, NJ. All rights reserved. This publication is protected by Copyright and written permission should be
MACHINE DESIGN - An Integrated Approach, 4th Ed. 10-6a-3

S e S ut2
Realised factor Nf  Nf  2.4
 3 
  S ut2 Fa  Se Fm
of safety

 w L 
8. Check worst-case static bearing stress.
1 2
Bearing area Abear   w L Abear  0.117  in
2

Maximum force Fmax  Fa  Fm Fmax  2286 lbf

Maximum bearing stress Fmax


on key σmax  σmax  19.50  ksi
Abear

Factor of safety against S y2


static bearing yield Ns  Ns  2.7
σmax

9. Design Summary

Key width w  0.375  in (3/8 x 3/8)


Key length L  0.625  in (5/8 in)

© 2011 Pearson Education, Inc., Upper Saddle River, NJ. All rights reserved. This publication is protected by Copyright and written permission should be
MACHINE DESIGN - An Integrated Approach, 4th Ed. 10-7a-1

PROBLEM 10-7a
Statement: Determine the size of key necessary to give a safety factor of at least 2 against both shear and
bearing failure for the design shown in Figure P10-4 using the data from row a in Table P10-1 and
the data given below. Assume a shaft diameter of 40 mm.

Given: Shaft diameter d  40 mm Shaft properties: S ut1  745  MPa


Maximum torque Tmax  2000 N  m S y1  427  MPa
Minimum torque Tmin  0  N  m Key properties: S ut2  600  MPa
Design safety factor Nd  2 S y2  360  MPa
Assumptions: 1. The finish is machined, reliability is 90%, and the shaft is at room temperature.
2. Note that the given magnitudes of the radial forces shown acting on gear(s) in this problem
are not necessarily consistent with a load associated with the given torque for any real gear of
reasonable pressure angle. Since gears are taken up in a later chapter, these shaft design
problems ignore the real gear loadings and use an arbitrary value to provide a shaft design
exercise.
Solution: See Figure P10-4 and Mathcad file P1007a.

1. As recommended in Table 10-2, for a shaft diameter of d  40 mm, use a square key of width
Key width w  10 mm
2. Determine the alternating and mean key shear force components.

Mean and alternating 1 Tmax  Tmin


force components Fa   Fa  50.0 kN
2 0.5 d

1 Tmax  Tmin
Fm   Fm  50.0 kN
2 0.5 d
3. Write the equations for the mean and alternating components of the shear stress and the effective von Mises
stress.
Key shear area as
function of length Ashear = w L

Mean and alternating Fa Fm


shear stresses τa = τm =
w L w L

Mean and alternating Fa Fm


von Mises stresses σ'a = 3  τa = 3  σ'm = 3  τm = 3 
w L w L

4. Using the modified Goodman failure criterion, the design equation is

S e S ut S e S ut
Nd = =
S ut σ'a  S e σ'm  3 
   S ut Fa  S e Fm
 w L 

3  Nd   S ut Fa  S e Fm
Solving for L L=
w S ut S e

5. Determine the key material endurance limit.


Uncorrected endurance
strength S'e  0.5 S ut2 S'e  300  MPa
© 2011 Pearson Education, Inc., Upper Saddle River, NJ. All rights reserved. This publication is protected by Copyright and written permission should be
MACHINE DESIGN - An Integrated Approach, 4th Ed. 10-7a-2

Correction factors:
Load Cload  1
w L
Size A95 = w L d eq =
0.0766

 0.097
Csize = 1.189  d eq

 0.5 0.097
Cs  1.189   
w
 Cs  0.939
 0.0766  mm 
 0.0485
Csize = Cs L
 0.265
 S ut2 
Surface (machined) Csurf  4.51   Csurf  0.828
 MPa 
Temperature Ctemp  1

Reliability Creliab  0.897 (R = 90%)

 0.0485  0.0485
S e = Cload  Csize Csurf  Ctemp Creliab S'e = 1  Cs L  Csurf  1  Creliab S'e = Ce L

where Ce  Cs Csurf  Creliab S'e Ce  209.145  MPa

6. Substitute S e into the design equation.

3  Nd   S ut Fa  Ce L  Fm


 0.0485
L=
 0.0485
w S ut Ce L

Solving by iteration, let

  0.0485 
 
3  Nd  S ut2 Fa  Ce 
L 
 Fm

RHS( L) 
  mm  
 0.0485
w S ut2 Ce 
L 

 mm 
Guess L  10 mm RHS( L)  121.468  mm
L  RHS( L) RHS( L)  133.39 mm
L  RHS( L) RHS( L)  133.866  mm

Tentatively, let L  134  mm (both keys, divide by 2 for one)

7. Check the realised factor of safety against fatigue failure.

 0.0485
S e  Ce  L
Endurance limit  S e  164.923  MPa
 mm 
© 2011 Pearson Education, Inc., Upper Saddle River, NJ. All rights reserved. This publication is protected by Copyright and written permission should be
MACHINE DESIGN - An Integrated Approach, 4th Ed. 10-7a-3

S e S ut2
Realised factor Nf  Nf  2.0
 3 
  S ut2 Fa  Se Fm
of safety

 w L 
8. Check worst-case static bearing stress.
1 2
Bearing area Abear   w L Abear  670  mm
2

Maximum force Fmax  Fa  Fm Fmax  100  kN

Maximum bearing stress Fmax


on key σmax  σmax  149.25 MPa
Abear

Factor of safety against S y2


static bearing yield Ns  Ns  2.4
σmax

9. Design Summary

Key width w  10 mm


Key length L1  0.5 L L1  67 mm

© 2011 Pearson Education, Inc., Upper Saddle River, NJ. All rights reserved. This publication is protected by Copyright and written permission should be
MACHINE DESIGN - An Integrated Approach, 4th Ed. 10-8-1
PROBLEM 10-8
Statement: A paper mill processes rolls of paper having a density of 984 kg/m3. The paper roll is 1.50-m
outside diameter (OD) x 0.22-m inside diameter (ID) x 3.23-m long and is on a simple supported,
hollow, steel shaft with S ut = 400 MPa. Find the shaft ID needed to obtain a dynamic safety
factor of 2 for a 10-yr life if the shaft OD is 22 cm and the roll turns at 50 rpm with 1.2 hp
absorbed.
Given: Paper roll: y
kg
Density ρ  984 
3
m w
Outside dia. OD  1500 mm x
Inside dia. ID  220  mm
Length L  3230 mm R L R
Shaft:
Strength S ut  400  MPa V
Outside dia. od  220  mm R
Factor of safety Nf  2 L/2 L
0 x
Power HP  1.2 hp
Speed ω  50 rpm -R

Assumptions: 1. The shaft is stiffer than the paper roll so M


2
the weight of the roll on the shaft can be wL /8
modelled as a uniformly distributed load.
2. The bearings that support the shaft are
close to the ends of the paper roll and are
thin with respect to the length of the roll so 0 x
we can consider the distance between the L/2 L
shaft supports to be the same as the length
of the roll. FIGURE 10-8
3. The shaft is machined, reliability is 99.9%, Load, Shear, and Moment Diagrams
and it is at room temperature. for Problem 10-8
4. The paper machine operates 3 shifts/day.

Solution: See Figure 10-8 and Mathcad file P1008.


1. The weight of the paper roll is,
Volume V 
π
4
 2
 OD  ID  L 
2
V  5.585  m
3

Weight W  ρ  g  V W  53.895 kN

2. From Figure 10-8, we see that the bending moment in the shaft is a maximum at the center of the span. First,
determine the magnitude of the distributed load, then find the maximum bending moment using Figure B-2(b)
in Appendix B with a = 0 and x = L/2.

W newton
Distributed load w  w  16.686
L mm
2
w L 7
Maximum moment Mmax  Mmax  2.176  10  newton  mm
8
3. Using equation 4.11b, find the maximum bending stress as a function of the unkown shaft inside diameter, id.
This is the only alternating stress element present at this point on the shaft and there is no alternating shear
stress at this point so max = 1 and 2 = 3 = 0. Furthermore, since 2 and 3 are zero, max = 'a.

Bending stress M c 32 Mmax od


σmax = = = σ'a
at midspan I  4
π od  id
4 
© 2011 Pearson Education, Inc., Upper Saddle River, NJ. All rights reserved. This publication is protected by Copyright and written permission should be
MACHINE DESIGN - An Integrated Approach, 4th Ed. 10-8-2

4. Calculate the steady shaft torque using equation (10.1a) and the mean von Mises stress using equation (5.7d).
HP
Mean torque Tm  Tm  171  N  m
ω

Mean torsional 16 Tm od


τm =
stress
 4
π od  id 
4

Mean von Mises


stress σ'm = 3  τm

2080 hr
5. Determine the number of stress cycles. Let Nshifts  3 shift_hours  Lcycle  10 yr
yr

Stress cycles Ncycles  ω shift_hours  Nshifts Lcycle

9
Ncycles  1.176  10

Since this is greater than 10 6, design for infinite life.

6. Calculate the unmodified endurance limit. S'e  0.5 S ut S'e  200  MPa

7. Determine the endurance limit modification factors for a rotating round shaft.

Load Cload  1
 0.097
Csize  1.189   
od
Size  Csize  0.705
 
mm

Surface A  4.51 b  0.265 (machined)

b
 Sut 
Csurf  A    Csurf  0.922
 MPa 
Temperature Ctemp  1

Reliability Creliab  0.753 (R = 99.9%)

8. Determine the modified endurance limit as a function of the unknown diameter, d.


S e  Cload  Csize Csurf  Ctemp Creliab S'e S e  97.819 MPa

9. Use equation (10.7b) with unity for all stress concentration factors as a design equation to find id.

S e S ut
Nf =
σ'a S ut  σ'm S e

Substituting the expressions for 'a and 'm found above,

© 2011 Pearson Education, Inc., Upper Saddle River, NJ. All rights reserved. This publication is protected by Copyright and written permission should be
MACHINE DESIGN - An Integrated Approach, 4th Ed. 10-8-3

S e S ut
Nf =
32 Mmax od 16 3  Tm od
 S ut   Se
 4
π od  id
4  
π od  id
4 4
Solving for the unknown inside diameter,

 4 16 Nf  od  2  Mmax Sut  3 Tm S e


4
id  od   id  191.467  mm
 π S e S ut 
Round this down (for a thicker wall) to id  190  mm

The wall thickness will be t  0.5 ( od  id) t  15 mm

© 2011 Pearson Education, Inc., Upper Saddle River, NJ. All rights reserved. This publication is protected by Copyright and written permission should be
MACHINE DESIGN - An Integrated Approach, 4th Ed. 10-9a-1

PROBLEM 10-9a
Statement: Repeat Problem 10-1 taking the stress concentration at the keyway shown in Figure P10-3 into
account.
Given: Distance between bearings a  16 in Distance to P b  18 in
Applied load P  1000 lbf Tensile strength S ut  108  ksi
Minimum torque Tmin  0  lbf  in Yield strength S y  62 ksi
Maximum torque Tmax  2000 lbf  in Design safety factor Nd  2
Assumptions: 1. The finish is machined, reliability is 99%, and the shaft is at room temperature.
2. The notch radius in the keyway is r  0.015  in.
3. Note that the given magnitudes of the radial forces shown acting on gear(s) in this problem
are not necessarily consistent with a load associated with the given torque for any real gear of
reasonable pressure angle. Since gears are taken up in a later chapter, these shaft design
problems ignore the real gear loadings and use an arbitrary value to provide a shaft design
exercise.

Solution: See Figures P10-1 and P10-3, and Mathcad file P1009a.
1. There is no stress concentration at the point on the shaft where the bending moment is a maximum. On the
other hand, at the gear where there is a stress concentration, the bending moment is zero. The question is:
which point requires the larger diameter in order to meet the safety factor requirement? To answer this
question, find the diameter required at each point and choose the larger. Start with the point where the
bending moment is maximum.
2. The maximum moment in the shaft occurs at the right bearing as seen in the moment diagram in Figure B-3(a)
in Appendix B (note that in the figure a is the distance to the load and b is the distance between bearings).
Using the equation given in the figure, calculate the alternating bending moment (the mean is zero).

Ma  P ( a  b ) Ma  2000 in lbf

3. Calculate the mean and alternating components of torque.

Tmax  Tmin
Tm  Tm  1000 in lbf
2
Tmax  Tmin
Ta  Ta  1000 in lbf
2
4. Calculate the unmodified endurance limit. S'e  0.5 S ut S'e  54 ksi

5. Determine the endurance limit modification factors for a rotating round shaft.
Load Cload  1
 0.097
Csize( d )  0.869   
d
Size 
 
in

Surface A  2.70 b  0.265 (machined)

b
 S ut 
Csurf  A    Csurf  0.781
 ksi 
Temperature Ctemp  1

Reliability Creliab  0.814 (R = 99%)

© 2011 Pearson Education, Inc., Upper Saddle River, NJ. All rights reserved. This publication is protected by Copyright and written permission should be
MACHINE DESIGN - An Integrated Approach, 4th Ed. 10-9a-2

6. Determine the modified endurance limit as a function of the unknown diameter, d.


S e( d )  Cload  Csize( d )  Csurf  Ctemp Creliab S'e

7. Use equation (10.8) with unity for all stress concentration factors as a design equation to find d.

Guess d  1  in
Given
1
3
  2 3 2 
 32 Nd  Ma  4  Ta 3 m 
T
d=    
 π  S e( d ) 4 S ut 

Diameter required at point of maximum moment: d 1  Find ( d ) d 1  1.188  in

Using this value for d, the size modification factor and endurance limit are:

Size modification factor Csize d 1  0.855

Endurance limit S e d 1  29.3 ksi

8. At the gear the bending moment is zero but the alternating and mean torques are the same as above. Also, there
is a stress concentration due to the keyway that must be applied to the alternating and mean shear stresses
present at the gear. First, set the moment equal to zero: Ma  0  lbf  in, then determine the values of the
alternating and mean fatigue stress concentration factors.

9. Determine the geometric stress concentration factor from the upper curve in Figure 10-16 using a curve-fit
equation.
 0.230
Kts( d )  1.251  
r

d
10. Determine the notch sensitivity of the material. Note from Figure 6-36, part 1, that a value of 20 ksi should be
added to S ut to obtain a 1/2 from Table 6-6. Using a curve-fit to Table 6-6,

 2 3 
a ( S )  8.69657  10  
  3.10030  10 7  S    in
2 3 S 5 S
 2.75956  10  
 3.94116  10  
 ksi  ksi   ksi  
 4 5 6 
 1.38322  10     3.28018  10     3.21209  10   
9 S  12 S  15 S

  ksi   ksi   ksi  

a  a  S ut  20 ksi
0.5
Neuber constant a  0.045  in

Notch radius r  0.015  in


1
Notch sensitivity q  q  0.732
a
1
r

© 2011 Pearson Education, Inc., Upper Saddle River, NJ. All rights reserved. This publication is protected by Copyright and written permission should be
MACHINE DESIGN - An Integrated Approach, 4th Ed. 10-9a-3

11. Determine the fatigue stress concentration factor from equation (6.11b).
Kfs( d )  1  q   Kts( d )  1 
Alternating factor

Mean factor Kfsm( d )  Kfs( d )

12. Repeat step 7 using these stress concentration factors with the torsional stresses.

Given
1
3
  2 3 2 
 32 Nd  Ma  4  Kfs( d )  Ta 3 T m

d=    Kfsm( d )  
 π  S e( d ) 4 S ut 

Diameter required at point of gear attachment: d 2  Find ( d ) d 2  1.129  in

 d1 
Putting the two calculated diameters into a vector trial_dia   
 d2 

13. The larger of the two must be used as the minimum shaft diameter. Thus, d  max( trial_dia ). d  1.188  in

14. The factor of safety at the point of maximum bending moment is Nd  2 . The factor of safety at the gear is
equal to the ratio of the two diameters cubed times Nd. Thus,

3
 d1 
Nfgear     Nd Nfgear  2.3
 d2 

© 2011 Pearson Education, Inc., Upper Saddle River, NJ. All rights reserved. This publication is protected by Copyright and written permission should be
MACHINE DESIGN - An Integrated Approach, 4th Ed. 10-10a-1

PROBLEM 10-10a
Statement: Repeat Problem 10-2 taking the stress concentration at the keyway shown in Figure P10-4 into
account.
Given: Distance between bearings L  20 cm Distance to p a  16 cm
Distance to end of p b  18 cm Tensile strength S ut  745  MPa
1
Applied distributed load p  1000 N  cm Yield strength S y  427  MPa
Minimum torque Tmin  0  N  m Design safety factor Nfd  2
Maximum torque Tmax  2000 N  m
Assumptions: 1. The finish is machined, reliability is 99%, and the shaft is at room temperature.
2. The ratio of notch radius to shaft diameter in the keyway is roverd  0.021.
3. The stress concentration factors for the mean stresses are Kfm  1 and Kfsm  1.

Solution: See Figures P10-2 and P10-4, and Mathcad file P1010a.
1. There is no stress concentration at the point on the shaft where the bending moment is a maximum, which is
at x = 16.3 cm (see the appendix to this problem, below). The keys are at a  16 cm and b  18 cm. On the
other hand, the key at a  16 cm looks as if it extends into the section where the moment is a maximum so,
use the maximum moment as the alternating bending moment.
2. The maximum moment in the shaft occurs between a and b. See the appendix to this problem below for the
determination of Ma.

Ma  48.45  N  m
3. Calculate the mean and alternating components of torque.

Tmax  Tmin
Tm  Tm  1000 N  m
2
Tmax  Tmin
Ta  Ta  1000 N  m
2

4. Determine the geometric stress concentration factors from Figure 10-16 using curve-fit equations.
 0.4115
Bending Kt  0.4521 ( roverd) Kt  2.2
 0.230
Torsion Kts  1.251  ( roverd) Kts  3.0

5. Determine the notch sensitivity of the material for bending and torsion. Note from Figure 6-36, part 1, that a
value of 20 ksi should be added to S ut to obtain a 1/2 from Table 6-6 for the torsional Neuber constant.

Bending Lookup value of S ut S ut  108  ksi


2 0.5
Neuber constant a N  0.056  in a N  0.056  in

1
Notch sensitivity q ( d ) 
aN
1
roverd d

Torsion Lookup value of S ut S'ut  S ut  20 ksi S'ut  128  ksi

2 0.5
Neuber constant a N  0.045  in a N  0.045  in

© 2011 Pearson Education, Inc., Upper Saddle River, NJ. All rights reserved. This publication is protected by Copyright and written permission should be
MACHINE DESIGN - An Integrated Approach, 4th Ed. 10-10a-2

1
Notch sensitivity q s( d ) 
aN
1
roverd d

6. Determine the fatigue stress concentration factors from equation (6.11b).

Bending factor Kf ( d )  1  q ( d )   Kt  1 

Torsion factor Kfs( d )  1  q s( d )   Kts  1 

7. Calculate the unmodified endurance limit. S'e  0.5 S ut S'e  372.5  MPa

8. Determine the endurance limit modification factors for a rotating round shaft.

Load Cload  1
 0.097
Csize( d )  1.189    d
Size 
 
mm

Surface A  4.51 b'  0.265 (machined)

b'
 Sut 
Csurf  A    Csurf  0.782
 MPa 
Temperature Ctemp  1

Reliability Creliab  0.814 (R = 99%)

9. Determine the modified endurance limit as a function of the unknown diameter, d.

S e( d )  Cload  Csize( d )  Csurf  Ctemp Creliab S'e

10. Use equation (10.8) as a design equation to find d.

Guess d  25 mm
Given
1
3
  
 32 Nfd   Kf ( d)  Ma  4   Kfs( d)  Ta
2 3 2
3 fsm m
K  T
d=    
 π  S e( d ) 4 S ut 

d  Find ( d ) d  65.3 mm

Using this value for d, the size modification factor, stress concentration factors, and endurance limit are:
Size modification factor Csize( d )  0.793
Endurance limit S e( d )  187.9  MPa
Stress concentration factors Kf ( d )  2.0 Kfs( d )  2.7

© 2011 Pearson Education, Inc., Upper Saddle River, NJ. All rights reserved. This publication is protected by Copyright and written permission should be
MACHINE DESIGN - An Integrated Approach, 4th Ed. 10-10a-3

APPENDIX - Maximum bending moment


1. From inspection of Figure P10-2, write the load function
equation
q(x) = R1<x - 0>-1 - p<x - a>0 + p<x - b>0 + R2<x - L>-1

2. Integrate this equation from - to x to obtain shear, V(x)

V(x) = R1<x - 0>0 - p<x - a>1 + p<x - b>1 + R2<x - L>0

3. Integrate this equation from - to x to obtain moment, M(x)

M(x) = R1<x - 0>1 - p<x - a>2/2 + p<x - b>2/2 + R2<x - L>1

4. Solve for the reactions by evaluating the shear and moment equations at a point just to the right of x = L, where
both are zero.
At x = L+, V = M = 0
V = R1  p  ( L  a )  p  ( L  b )  R2 = 0

p 2 p 2
M = R 1 L   ( L  a)   ( L  b) = 0
2 2

 ( L  a )  ( L  b )
p 2 2
R1   R1  300  N
2 L

R2  p  ( b  a )  R1 R2  1700 N

5. Define the range for x x  0  m 0.005  L  L

6. For a Mathcad solution, define a step function S. This function will have a value of zero when x is less than z,
and a value of one when it is greater than or equal to z.

S ( x z)  if ( x  z 1 0 )


7. Write the shear and moment equations in Mathcad form, using the function S as a multiplying factor to get the
effect of the singularity functions.

V ( x)  R1 S ( x 0  m)  p  S ( x a )  ( x  a )  p  S ( x b )  ( x  b )  R2 S ( x L)


p 2 p 2
M ( x)  R1 S ( x 0  m)  x   S ( x a )  ( x  a )   S ( x b )  ( x  b )
2 2
8. Plot the shear and moment diagrams.

Shear 500
Diagram
0

V ( x)  500
N
 1000

 1500

 2000
0 5 10 15 20
x
cm
© 2011 Pearson Education, Inc., Upper Saddle River, NJ. All rights reserved. This publication is protected by Copyright and written permission should be
MACHINE DESIGN - An Integrated Approach, 4th Ed. 10-10a-4

Moment 50
Diagram
40

30
M ( x)
20
Nm

10

 10
0 5 10 15 20
x
cm

FIGURE 10-10a
Shear and Moment Diagrams for Problem 10-10a

9. Determine the maximum maximum moment from inspection of the diagrams. Maximum moment occurs where
zero, which is x = c. From the shear diagram,

ca bc a  R2  b  R1
= c  c  16.300 cm
R1 R2 R1  R2

Mmax  M ( c) Mmax  48.45  N  m

Moment at x = b M ( b )  34 N  m

© 2011 Pearson Education, Inc., Upper Saddle River, NJ. All rights reserved. This publication is protected by Copyright and written permission should be
MACHINE DESIGN - An Integrated Approach, 4th Ed. 10-11a-1
PROBLEM 10-11a
Statement: Determine the amount of diametral interference needed to provide a suitable interference fit for
the 6-in diameter by 1-in thick gear of Figure P10-3 using a shaft diameter of 1.75 in, such that the
stresses in the hub and shaft will be safe and the torque from row a in Table P10-1 can be
transmitted through the interference fit. Assume that both parts are steel with the properties
given below.

Given: Gear hub diameter d hub  6.00 in Shaft diameter: d shaft  1.75 in
6
Gear hub length L  1.00 in Young's modulus E  30 10  psi
Peak shaft torque Tp  2000 lbf  in Poisson's ratio ν  0.28
Assumptions: 1. Material properties are: S ut  108  ksi, and S y  62 ksi.
2. Note that the given magnitudes of the radial forces shown acting on gear(s) in this problem
are not necessarily consistent with a load associated with the given torque for any real gear of
reasonable pressure angle. Since gears are taken up in a later chapter, these shaft design
problems ignore the real gear loadings and use an arbitrary value to provide a shaft design
exercise.
3. The coefficient of friction between the hub and shaft is μ  0.15.
Solution: See Figure P10-3 and Mathcad file P1011a.

1. As a design choice, let the design factor of safety on torque capacity and hub failure be Nd  2

2. Calculate hub and shaft radii.


Hub radius ro  0.5 d hub ro  3  in
Nominal interface radius r  0.5 d shaft r  0.875  in
Shaft inside diameter ri  0  in

3. The minimum interference is determine by the desired torque capacity. Use equation 10.14c to solve for the
minimum diametral interference.

π L r μ  δmin E  ro  r 
2 2
Torque capacity T= 
Nd 2
2  ro
2
2  ro  Nd  Tp
Solving for min δmin  δmin  0.00071  in
π L r μ  E  ro  r 
2 2

Let the minimum diametral interference be δmin  0.0007 in

Note that it is not practical to specify a dimension in inches that has more than 4 decimal places.

4. Find the von Mises stress in the hub as a function of the unknown maximum diametral interference .

δ E  ro  r 
2 2
Interference pressure p ( δ) 
2
4  r ro
Stress in shaft
Tangential σti ( δ)  p ( δ)

Radial σri( δ)  p ( δ)

© 2011 Pearson Education, Inc., Upper Saddle River, NJ. All rights reserved. This publication is protected by Copyright and written permission should be
MACHINE DESIGN - An Integrated Approach, 4th Ed. 10-11a-2

Stress in hub

2 2
ro  r
Tangential σto( δ)  p ( δ) 
2 2
ro  r
Radial σro( δ)  p ( δ)
These are principal stresses. The tangential stress is 1 and the radial is 3. Use equation (5.7c) to find the von
Mises stress.
0.5
δ E
  3  ro  r
4 4
von Mises σ'o( δ)  
2
4  r ro

5. There is no bending stress in the shaft at the gear, therefore Kt  1

6. The safety factors against failure (yielding in the shaft and hub) during press fit can now be used to find the
maximum diametral interference:
Guess δ  δmin

S y
Shaft Given Nd =
Kt σti ( δ)

δsmax  Find ( δ) δsmax  0.0040 in

Sy
Hub Given Nd =
Kt σ'o( δ)

δhmax  Find( δ) δhmax  0.0021 in

Using the smaller of the two, δmax  δhmax δmax  0.0021 in

7. The torque capacity of the joint (with an assumed coefficient of friction of μ  0.15 ) is about 2 times the
peak transmitted torque and the safety factors against failure of the shaft and hub exceed 2 therefore, the
minimum and maximum diametral interference below is acceptable.
Minimum diametral interference δmin  0.0007 in

Minimum diametral interference δmax  0.0021 in


8. If we divide the tolerance on the shaft and hub equally and use the basic hole system, the shaft and hub bore
specifications are:

Tolerance on shaft or hub t  0.5  δmax  δmin t  0.0007 in

Hub bore diameter: Dmin  d shaft Dmin  1.7500 in

Dmax  Dmin  t Dmax  1.7507 in

Shaft diameter: d max  d shaft  δmax d max  1.7521 in

d min  d max  t d min  1.7514 in

Maximum interference δmax  d max  Dmin δmax  0.0021 in


© 2011 Pearson Education, Inc., Upper Saddle River, NJ. All rights reserved. This publication is protected by Copyright and written permission should be
MACHINE DESIGN - An Integrated Approach, 4th Ed. 10-11a-3

Minimum interference δmin  d min  Dmax δmin  0.0007 in

9. Check the safety factors against torque capacity and static failure of the hub.

π L r μ  δmin E  ro  r 
2 2
Torque Ntorque   Ntorque  2.0
Tp 2
2  ro

Sy
Hub strength Nhub  Nhub  2.0
Kt σ'o δmax

© 2011 Pearson Education, Inc., Upper Saddle River, NJ. All rights reserved. This publication is protected by Copyright and written permission should be
MACHINE DESIGN - An Integrated Approach, 4th Ed. 10-12a-1
PROBLEM 10-12a
Statement: Assume that the device shown keyed to the shaft of Figure P10-3 is a Class 50, cast iron flywheel
of 20-in outside diameter and 1-in thickness. The hub is 4-in dia and 3-in thick. Determine the
maximum speed at which it can safely be run using a safety factor of 2. Use dimensions and
other appropriate data from Problem 10-6 and row a in Table P10-1. Consider the transverse
force P to be zero in this case.

Given: Flywheel : odfw  20 in Safety factor Nos  2


thickfw  1  in Tensile strength S ut  52.5 ksi
3
Hub: odhub  4  in Specific weight γ  0.26 lbf  in
thickhub  3  in Poisson's ratio ν  0.26
Shaft diameter d shaft  1.75 in

Assumptions: The hub is thicker (in the axial direction) than the flywheel disk. This has the effect of
strengthening the flywheel from the shaft out to the od of the hub. This effect will be ignored
and the flywheel analyzed as if the hub had the same thickness as the disk portion.

Solution: See Mathcad file P1012a.

1. The stress is maximum at the inside radius of a flywheel, which in this case is at the shaft diameter. At this radiu
the radial stress is zero so that the tangential stress is the only nonzero stress and is 1. With the other principal
stresses equal to zero, the load line is along the s1 axis on the 1-3 diagram. For that case, the factor of safety
equation is the same for all brittle material failure theories and is
S ut
Ns =
σ1

2. Use equation (10.23a) with t replaced by S ut (Ns = 1) to determine the speed at which the flywheel will fail.

Inside radius ri  0.5 d shaft ri  0.875  in

Outside radius ro  0.5 odfw ro  10 in

Critical radius r  ri r  0.875  in


1
2
 g  S ut 
Failure speed ωfail   
  3  ν   2 
2 2
2 ri  ro 1  3  ν 2 
 γ  8    ri  ro  2 
3ν
r 

  r 

ωfail  9332 rpm


ωfail
3. The factor of safety on the operating speed (see Example 10-7) is defined as Nos =
ωos

ωfail
Maximum safe operating speed ωos  ωos  4666 rpm
Nos

Using this definition results in the static strength factor of safety, Ns, being equal to the square of Nos.

© 2011 Pearson Education, Inc., Upper Saddle River, NJ. All rights reserved. This publication is protected by Copyright and written permission should be
MACHINE DESIGN - An Integrated Approach, 4th Ed. 10-13a-1
PROBLEM 10-13a
Statement: Determine the critical frequency of shaft whirl for the assembly shown in Figure P10-3 using
dimensions from row a of Table P10-1 and a steel shaft diameter of 2 in. Use the flywheel
dimensions of Problem 10-12.
2
lbf  sec
Units: blob 
in
Given: Flywheel : odfw  20 in Shaft:
thickfw  1  in Distance between bearings a  16 in
Hub: odhub  4  in Distance to flywheel b  18 in
thickhub  3  in Shaft diameter d shaft  2.00 in
3 6
Specific weight γ  0.26 lbf  in Young's modulus E  30 10  psi

Solution: See Mathcad file P1013a.

1. Determine the shaft stiffness at the flywheel using Figure B-3(a) in Appendix B. Note that in Figure B-3(a)
the dimensions a and b are the dimensions b and a, respectively in Figure P10-3.

The deflection under the load (x=a in Figure B-3(a)) is


ba
   ( a  b )  b  ( a  b )  a
F 3 a 3
y = a 
6  E I  b b 
To use the symbols of Figure P10-3, change a to b and b to a.

ab
   ( b  a )  a  ( b  a )  b
F 3 b 3
y = b 
6  E I  a a 
F
The stiffness of the beam at the point of the applied load is k = . Solve the above equation for F/y = k
y
while substituting d 4/64 for I.

4
3  π d shaft  E 5 lbf
k  k  9.817  10 
ab
32   ( b  a )  a  ( b  a )  b
3 b 3 in
b 
 a a 

2. Calculate the mass of the flywheel and hub.


π 
  odhub  d shaft   thickhub
2 2 3
Volume of hub Vhub  Vhub  28.274 in
4

π 
  odfw  odhub   thickfw
2 2 3
Volume of flywheel Vfw  Vfw  301.593  in
4

  Vhub  Vfw
γ
Total mass mtot  mtot  0.222  blob
g

3. Calculate the critical frequency.


k rad
ωn  ωn  2102
mtot sec

ωn  20075  rpm
© 2011 Pearson Education, Inc., Upper Saddle River, NJ. All rights reserved. This publication is protected by Copyright and written permission should be
MACHINE DESIGN - An Integrated Approach, 4th Ed. 10-13a-2

ωn
fn  fn  335  Hz
2 π

mtot  g 5
4. Deflection at the flywheel due to its own weight. δ  δ  8.736  10  in
k

© 2011 Pearson Education, Inc., Upper Saddle River, NJ. All rights reserved. This publication is protected by Copyright and written permission should be
MACHINE DESIGN - An Integrated Approach, 4th Ed. 10-14a-1
PROBLEM 10-14a
Statement: Determine the critical frequency of shaft whirl for the assembly shown in Figure P10-4 using
dimensions from row a of Table P10-1 and a steel shaft diameter of 40 mm. The cast-iron roller
diameter is 3 times the shaft diameter.

Given: Distance between bearings L  20 cm Shaft Young's modulus E  206.8  GPa
3
Distance to start of roller a  16 cm Roller specific weight γ  70580  N  m
Distance to end of roller b  18 cm Roller diameter d roller  3  d shaft
Shaft diameter d shaft  40 mm d roller  120  mm

Solution: See Figure P10-4 and Mathcad file P1014a.

1. Calculate the area moment of inertia of the shaft.


4
π d shaft 5 4
Area moment of inertia I  I  1.257  10  mm
64

2. Calculate the weight of the roller.


π  2 2
Vroller   d  d shaft   ( b  a)
4 
Volume of roller roller

5 3
Vroller  2.011  10  mm

Weight of roller Wroller  γ Vroller Wroller  14.191 N

Wroller
Mass of roller mroller  mroller  1.447 kg
g
Wroller N
Distributed load on shaft due to weight of roller p  p  709.548 
ba m

3. From inspection of Figure P10-4, write the load function equation and integrate it twice to get the shear and
moment equations.
q(x) = R1<x - 0>-1 - p<x - a>0 + p<x - b>0 + R2<x - L>-1
V(x) = R1<x - 0>0 - p<x - a>1 + p<x - b>1 + R2<x - L>0
M(x) = R1<x - 0>1 - p<x - a>2/2 + p<x - b>2/2 + R2<x - L>1

4. Solve for the reactions by evaluating the shear and moment equations at a point just to the right of x = L, where
both are zero.
At x = L+, V = M = 0

V = R1  p  ( L  a )  p  ( L  b )  R2 = 0

p 2 p 2
M = R 1 L   ( L  a)   ( L  b) = 0
2 2

 ( L  a )  ( L  b )
p 2 2
R1   R1  2.129  N
2 L

R2  p  ( b  a )  R1 R2  12.06  N

5. Integrate twice more to get the slope and deflection equations.


© 2011 Pearson Education, Inc., Upper Saddle River, NJ. All rights reserved. This publication is protected by Copyright and written permission should be
MACHINE DESIGN - An Integrated Approach, 4th Ed. 10-14a-2

(x) = [R1<x - 0>2/2 - p<x - a>3/6 + p<x - b>3/6 + R2<x - L>2/2 + C3]/EI

y(x) = [R1<x - 0>3/6 - p<x - a>4/24 + p<x - b>4/24 + R2<x - L>3/6 + C3x + C4]/EI
6. Solve for the constants of integration with the boundary conditions y = 0 at x = 0 and x = L.
At x = 0 C4 = 0

R1 3 p 4 p 4
At x = L 0= L   ( L  a)   ( L  b )  C 3 L
6 24 24

1 p 4 p 4 R1 3 2
C3    ( L  a)   ( L  b)  L  C3  0.014  N  m
L  24 24 6 

7. For a Mathcad solution, define a step function S. This function will have a value of zero when x is less than z,
and a value of one when it is greater than or equal to z.

S ( x z)  if ( x  z 1 0 )


8. Write the deflection equation in Mathcad form, using the function S as a multiplying factor to get the effect of
the singularity functions.

1  R1 3 p 4 p 4 
y ( x)    S ( x 0  mm)  x   S ( x a )  ( x  a )   S ( x b )  ( x  b )  C3 x
E I  6 24 24 
9. Determine the approximate stiffness of the shaft by dividing the weight of the roller by the deflection of the sha
due to the roller weight at the mid-point of the roller.
ab
Roller midpoint c  c  170  mm
2
5
Deflection at x = c δc  y ( c) δc  2.345  10  mm

Wroller 8 N
Stiffness k  k  6.051  10 
δc m

10. Calculate the critical frequency.

k rad
ωn  ωn  20449 
mroller sec

ωn  195275 rpm

ωn
fn  fn  3255 Hz
2 π

© 2011 Pearson Education, Inc., Upper Saddle River, NJ. All rights reserved. This publication is protected by Copyright and written permission should be
MACHINE DESIGN - An Integrated Approach, 4th Ed. 10-15a-1
PROBLEM 10-15a
Statement: What are the maximum, minimum, and average power values for the shaft shown in Figure P10-1
for the data in row a of Table P10-1 if the shaft speed is 750 rpm?

rad
Given: Shaft speed ω  750  rpm ω  78.54 
sec
Minimum torque Tmin  0  in lbf
Maximum torque Tmax  2000 in lbf

Solution: See Mathcad file P1015a.

1. Use equations (10.1) to calculate the power values.

Tmax  Tmin
Average torque Tavg  Tavg  1000 in lbf
2

Maximum power Pmax  Tmax ω Pmax  23.8 hp

Minimum power Pmin  Tmin ω Pmin  0  hp

Average power Pavg  Tavg ω Pavg  11.9 hp

2. Note, if doing these calculations by hand, that suitable unit conversion factors must be used.

© 2011 Pearson Education, Inc., Upper Saddle River, NJ. All rights reserved. This publication is protected by Copyright and written permission should be
MACHINE DESIGN - An Integrated Approach, 4th Ed. 10-16a-1
PROBLEM 10-16a
Statement: What are the maximum, minimum, and average power values for the shaft shown in Figure P10-2
for the data in row a of Table P10-1 if the shaft speed is 50 rpm?

rad
Given: Shaft speed ω  50 rpm ω  5.236 
sec
Minimum torque Tmin  0  N  m
Maximum torque Tmax  2000 N  m

Solution: See Mathcad file P1016a.

1. Use equations (10.1) to calculate the power values.

Tmax  Tmin
Average torque Tavg  Tavg  1000 N  m
2

Maximum power Pmax  Tmax ω Pmax  10.5 kW

Minimum power Pmin  Tmin ω Pmin  0  kW

Average power Pavg  Tavg ω Pavg  5.24 kW

2. Note, if doing these calculations by hand, that suitable unit conversion factors must be used.

© 2011 Pearson Education, Inc., Upper Saddle River, NJ. All rights reserved. This publication is protected by Copyright and written permission should be
MACHINE DESIGN - An Integrated Approach, 4th Ed. 10-17a-1

PROBLEM 10-17a
Statement: Figure P10-5 shows a roller assembly driven by a gear. The roller extends over 80% of the length a
and is centered in that dimension. The roller occupies 95% of the exposed shaft length between
the bearing faces. The shaft is steel of S ut = 745 MPa and S y = 427 MPa. For the data in row a of
Table P10-1, find:
(a) The safety factor against fatigue failure for a shaft diameter of 40 mm.
(b) The maximum torsional deflection between gear and roller.
(c) The torsional natural frequency of the shaft.

Given: Distance to end of shaft L  20 cm Distance between bearings a  16 cm


Distance to gear b  18 cm Tensile strength S ut  745  MPa
1
Applied distributed load p  1000 N  cm Yield strength S y  427  MPa
Minimum torque Tmin  0  N  m Shaft diameter d  40 mm
Maximum torque Tmax  2000 N  m Concentrated load P  1000 N

Assumptions: 1. The finish is machined, reliability is 90%, and the shaft is at room temperature.
2. Use SI units with Table P10-1 consistent with Problem 10-2.
3. The ratio of notch radius to shaft diameter in the keyway is roverd  0.021.
4. The roller stiffens the shaft torsionally so that the torsional deflection will occur largely
between the right end of the roller (at the second or middle key) and the gear (right key).
5. Note that the given magnitudes of the radial forces shown acting on gear(s) in this problem
are not necessarily consistent with a load associated with the given torque for any real gear of
reasonable pressure angle. Since gears are taken up in a later chapter, these shaft design
problems ignore the real gear loadings and use an arbitrary value to provide a shaft design
exercise.

Solution: See Figure P10-5 and Mathcad file P1017a.


Part (a)
1. The maximum moment in the shaft occurs at approximately the middle of the roller. See the appendix to this
problem below for the determination of Mmax. The maximum moment is also the alternating moment. Thus,

Mmax  297.3  N  m Ma  Mmax Ma  297.3  N  m


2. Calculate the mean and alternating components of torque, assuming that one half is taken out at the right-hand
key and the other half at the left-hand key. At the middle of the roller only one half of the torque is present.

1  Tmax  Tmin 
Tm    Tm  500  N  m
2  2 

1  Tmax  Tmin 
Ta    Ta  500  N  m
2  2 

3. Calculate the unmodified endurance limit. S'e  0.5 S ut S'e  372.5  MPa
4. Determine the endurance limit modification factors for a rotating round shaft.
Load Cload  1
 0.097
Csize  1.189   d
Size  Csize  0.831
 
mm

Surface A  4.51 b'  0.265 (machined)

© 2011 Pearson Education, Inc., Upper Saddle River, NJ. All rights reserved. This publication is protected by Copyright and written permission should be
MACHINE DESIGN - An Integrated Approach, 4th Ed. 10-17a-2

b'
 Sut 
Csurf  A    Csurf  0.782
 MPa 
Temperature Ctemp  1

Reliability Creliab  0.897 (R = 90%)

5. Determine the modified endurance limit as a function of the unknown diameter, d.


S e  Cload  Csize Csurf  Ctemp Creliab S'e S e  217.148  MPa

6. Use equation (10.8) with unity for all stress concentration factors to find the factor of safety at the point in the
shaft where the bending moment is maximum.

1
 2 3 2 3 
3  Ma   Ta  Tm 
π d 4 4
Nfc     Nfc  2.1
32  Se S ut 

7. Although the bending moment is lower at the ends of the roller where the keys are, there is a stress
concentration there that is not present at the point where the bending moment is a maximum. Of the two key
locations, the one on the right is critical (even though the moment is lower there) because the full torque is
present in the shaft at this point. The bending moment and torque at the right keyway (x = f) are:

Mf  84.4 N  m Ma  Mf Ma  84.4 N  m

 Tmax  Tmin 
Tm    Tm  1000 N  m
 2 

 Tmax  Tmin 
Ta    Ta  1000 N  m
 2 
8. Determine the geometric stress concentration factors from Figure 10-16 using the assumed r/d.
For an r/d ratio of roverd  0.021 Kt  2.2 Kts  3.0

9. Determine the notch sensitivity of the material for bending and torsion. Note from Figure 6-36, part 1, that a
value of 20 ksi should be added to S ut to obtain a 1/2 from Table 6-6 for the torsional Neuber constant.

Notch radius r  roverd d r  0.84 mm

Bending Lookup value of S ut S ut  108  ksi


2 0.5
Neuber constant a N  0.056  in a N  0.056  in

1
Notch sensitivity q  q  0.765
aN
1
r

Torsion Lookup value of S ut S'ut  S ut  20 ksi S'ut  128  ksi


2 0.5
Neuber constant a N  0.045  in a N  0.045  in
© 2011 Pearson Education, Inc., Upper Saddle River, NJ. All rights reserved. This publication is protected by Copyright and written permission should be
MACHINE DESIGN - An Integrated Approach, 4th Ed. 10-17a-3

1
Notch sensitivity q s  q s  0.802
aN
1
r

10. Determine the fatigue stress concentration factors from equation (6.11b).

Bending factor Kf  1  q   Kt  1  Kf  1.917

Torsion factor Kfs  1  q s  Kts  1  Kfs  2.603

Maximum and minimum nominal stress

16 Ta 16 Tm
τanom  τanom  79.6 MPa τmnom  τmnom  79.6 MPa
3 3
π d π d

τmaxnom  τmnom  τanom τmaxnom  159.155  MPa

τminnom  τmnom  τanom τminnom  0  MPa

Using equations 6.17, the mean-stress fatigue-concentration factors are

Kfm τmax τmin K S   return K if K τmax  S


S  K τanom
return if K τmax  S
τmnom
return 0 if K τmax  τmin  2  S

Kfsm  Kfm τmaxnom τminnom Kfs 0.577  S y Kfsm  0.49

11. Use equation (10.8) with the stress concentration factors above to find the factor of safety at the point in the
shaft where the right-hand roller key is located.

1
 
3   Kf  Ma    K fs Ta
2 3 2 3
 Kfsm Tm
π d 4 4
Nff     Nff  0.57
32  Se S ut 
So, in this case, the critical section on the shaft is not at the point of maximum bending moment. This is for two
reasons, 1) there is no stress concentration at this point but there is elsewhere, and 2) the torque is maximum at
the point of a stress concentration.

Part (b)
12. For the assumptions made, the length of the shaft that will deflect torsionally is Lbf  b  f Lbf  36 mm

4
π d
13. Calculate the polar moment of inertia of the shaft. J 
32
14. 5 4
J  Using
2.513 equation
10  mm(10.9a), calculate the angular deflection between the gear and the right-hand key for the maximu
applied torque.
© 2011 Pearson Education, Inc., Upper Saddle River, NJ. All rights reserved. This publication is protected by Copyright and written permission should be
MACHINE DESIGN - An Integrated Approach, 4th Ed. 10-17a-4

Modulus of rigidity G  80.8 GPa

Tmax Lbf
Angular deflection θ  θ  0.203  deg
J G
Part (c)
15. Use equations (10.27) to determine the shaft spring constant, mass moment of inertia, and natural frequency.
kg
Density of steel ρ  76500 
3
m
2
π d
Mass of shaft mshaft   L ρ mshaft  19.227 kg
4

2
 
Mass moment of inertia mshaft d 3 2
Im   Im  3.845  10 m  kg
2 2
G J N m
Spring constant kt  kt  564090
Lbf rad

kt rad
Natural frequency ωn  ωn  12112 
Im sec

ωn
fn  fn  1928 Hz
2 π

APPENDIX - Maximum bending moment


1. Determine the distance from the origin to the left and right ends of the roller.
Distance to left end e  0.1 a e  16 mm
Distance to right end f  0.9 a f  144  mm
2. From inspection of Figure P10-5, write the load function
equation
q(x) = R1<x - 0>-1 - p<x - e>0 + p<x - f>0 + R2<x - a>-1 - P<x - b>-1
2. Integrate this equation from - to x to obtain shear, V(x)

V(x) = R1<x - 0>0 - p<x - e>1 + p<x - f>1 + R2<x - a>0 - P<x - b>0
3. Integrate this equation from - to x to obtain moment, M(x)

M(x) = R1<x - 0>1 - p<x - e>2/2 + p<x - f>2/2 + R2<x - a>1 - P<x - b>1
4. Solve for the reactions by evaluating the shear and moment equations at a point just to the right of x = L, where
both are zero.
At x = b +, V = M = 0
V = R1  p  ( b  e )  p  ( b  f )  R 2  P = 0
p 2 p 2
M = R 1 b   ( b  e)   ( b  f )  R 2 ( b  a ) = 0
2 2
 e2  f 2 
 f  e  p  
b a
R1   P R1  6275 N
 2 a   a 
© 2011 Pearson Education, Inc., Upper Saddle River, NJ. All rights reserved. This publication is protected by Copyright and written permission should be
MACHINE DESIGN - An Integrated Approach, 4th Ed. 10-17a-5

R2  p  ( f  e)  R1  P R2  7525 N
5. Define the range for x x  0  m 0.005  b  b

6. For a Mathcad solution, define a step function S. This function will have a value of zero when x is less than z,
and a value of one when it is greater than or equal to z.

S ( x z)  if ( x  z 1 0 )


7. Write the shear and moment equations in Mathcad form, using the function S as a multiplying factor to get the
effect of the singularity functions.

V ( x)  R1 S ( x 0  m)  p  S ( x e)  ( x  e)  p  S ( x f )  ( x  f )  R2 S ( x a )  P S ( x b )

p 2 p 2
M ( x)  R1 S ( x 0  m)  x   S ( x e)  ( x  e)   S ( x f )  ( x  f )  R2 S ( x a )  ( x  a )
2 2

8. Plot the shear and moment diagrams.

SHEAR MOMENT
10 300

5 200

V ( x) M ( x)
0 100
kN Nm

5 0

 10  100
0 5 10 15 20 0 5 10 15 20
x x
cm cm
FIGURE 10-17a
Shear and Moment Diagrams for Problem 10-17a

9. Determine the maximum maximum moment from inspection of the diagrams. Maximum moment occurs where V
zero, which is x = c. From the shear diagram,

ce f c f  R1  e  R2  e  P
= c  c  7.875  cm
R1 R2  P R1  R2  P

Mmax  M ( c) Mmax  297.3  N  m

The moment at the left edge of the roller (at key) is Me  M ( e) Me  100.4  N  m
The moment at the right edge of the roller (at key) is Mf  M ( f ) Mf  84.4 N  m
The moment at the right-hand bearing is Ma  M ( a ) Ma  20 N  m

© 2011 Pearson Education, Inc., Upper Saddle River, NJ. All rights reserved. This publication is protected by Copyright and written permission should be
MACHINE DESIGN - An Integrated Approach, 4th Ed. 10-18a-1
PROBLEM 10-18a
Statement: Figure P10-5 shows a roller assembly driven by a gear. The roller extends over 80% of the length a
and is centered in that dimension. The roller occupies 95% of the exposed shaft length between
the bearing faces. For the data in row a of Table P10-1, find the maximum bending deflection of th
40-mm-diameter shaft.

Given: Distance to end of shaft L  20 cm Distance between bearings a  16 cm


Distance to gear b  18 cm Shaft diameter d  40 mm
1
Applied distributed load p  1000 N  cm Concentrated load P  1000 N
Maximum torque Tmax  2000 N  m Young's modulus E  206.8  GPa
Minimum torque Tmin  0  N  m

Assumptions: 1. The roller does not stiffen the shaft in bending but it does transmit the distributed load
directly to the shaft.
2. Note that the given magnitudes of the radial forces shown acting on gear(s) in this problem
are not necessarily consistent with a load associated with the given torque for any real gear of
reasonable pressure angle. Since gears are taken up in a later chapter, these shaft design
problems ignore the real gear loadings and use an arbitrary value to provide a shaft design
exercise.
Solution: See Figure P10-5 and Mathcad file P1018a.
1. Determine the distance from the origin to the left and right ends of the roller and the area moment of inertia.
Distance to left end e  0.1 a e  16 mm
Distance to right end f  0.9 a f  144  mm
4
π d 5 4
Moment of inertia I  I  1.257  10  mm
64
2. From inspection of Figure P10-5, write the load function equation and integrate it twice to get the shear and
moment equations.
q(x) = R1<x - 0>-1 - p<x - e>0 + p<x - f>0 + R2<x - a>-1 - P<x - b>-1

V(x) = R1<x - 0>0 - p<x - e>1 + p<x - f>1 + R2<x - a>0 - P<x - b>0

M(x) = R1<x - 0>1 - p<x - e>2/2 + p<x - f>2/2 + R2<x - a>1 - P<x - b>1

3. Solve for the reactions by evaluating the shear and moment equations at a point just to the right of x = L,
where both are zero.
At x = b +, V = M = 0
V = R1  p  ( b  e )  p  ( b  f )  R 2  P = 0
p 2 p 2
M = R 1 b   ( b  e)   ( b  f )  R 2 ( b  a ) = 0
2 2
 e2  f 2  b  a
R1    f  e  p   P R1  6275 N
 2  a   a 
R2  p  ( f  e)  R1  P R2  7525 N

4. Integrate twice more to get the slope and deflection equations.


(x) = [R1<x - 0>2/2 - p<x - e>3/6 + p<x - f>3/6 + R2<x - a>2/2 - P<x - b>2/2 + C3]/EI

y(x) = [R1<x - 0>3/6 - p<x - e>4/24 + p<x - f>4/24 + R2<x - a>3/6 - P<x - b>3/6 + C3x + C4]/EI

© 2011 Pearson Education, Inc., Upper Saddle River, NJ. All rights reserved. This publication is protected by Copyright and written permission should be
MACHINE DESIGN - An Integrated Approach, 4th Ed. 10-18a-2

5. Solve for the constants of integration with the boundary conditions y = 0 at x = 0 and x = a.
At x = 0 C4 = 0

R1 3 p 4 p 4
At x = L 0= a   ( a  e)   ( a  f )  C 3 a
6 24 24

1 p 4 p 4 R1 3 2
C3    ( a  e)  (a  f )  a  C3  15.578 N  m
a  24 24 6 

6. Define the range for x x  0  m 0.005  L  L


7. For a Mathcad solution, define a step function S. This function will have a value of zero when x is less than z,
and a value of one when it is greater than or equal to z.

S ( x z)  if ( x  z 1 0 )

8. Write the slope and deflection equations in Mathcad form, using the function S as a multiplying factor to get
the effect of the singularity functions.

1  R1 2 p 3 p 3 
θ ( x)    S ( x 0  mm)  x   S ( x e)  ( x  e)   S ( x f )  ( x  f ) 
E I 2 6 6
 R 
 2  S ( x a )  ( x  a) 2  P  S( x b)  ( x  b ) 2  C3 
 2 2 

1  R1 3 p 4 p 4 
y ( x)    S ( x 0  mm)  x   S ( x e)  ( x  e)   S ( x f )  ( x  f ) 
E I 6 24 24
 R 
 2  S( x a)  ( x  a ) 3  P  S ( x b )  ( x  b) 3  C3 x 
 6 6 

9. Plot the slope and deflection diagrams (see Figure 10-18).

SLOPE DEFLECTION
1 0.04

0.5 0.02

θ( x) y ( x)
3
0 0
10 mm

 0.5  0.02

1  0.04
0 5 10 15 20 0 5 10 15 20
x x
FIGURE 10-18 cm cm
Slope and Deflection Diagrams for Problem 10-18a

© 2011 Pearson Education, Inc., Upper Saddle River, NJ. All rights reserved. This publication is protected by Copyright and written permission should be
MACHINE DESIGN - An Integrated Approach, 4th Ed. 10-18a-3

10. The maximum deflection occurs at the value of x for which  is zero. Let this be x = c, then guess c  8  cm
Given θ ( c) = 0 c  Find ( c) c  7.955  cm

11. The maximum deflection occurs at x = c and is

ymax  y ( c) ymax  0.0300 mm

12. The deflection at the end of the shaft is

yL  y ( L) yL  0.0229 mm

© 2011 Pearson Education, Inc., Upper Saddle River, NJ. All rights reserved. This publication is protected by Copyright and written permission should be
MACHINE DESIGN - An Integrated Approach, 4th Ed. 10-19a-1

PROBLEM 10-19a
Statement: Figure P10-6 shows two gears on a common shaft. Assume that the constant radial force P1 is
40% of P2. For the data in row a from Table P10-1, find the diameter of the shaft required to
obtain a safety factor of 2 in fatigue loading if the shaft is steel with properties given below.

Given: Maximum torque Tmax  2000 lbf  in Minimum torque Tmin  0  lbf  in
Radial load P2  1000 lbf Load factor n  0.4
Shaft dimensions: a  16 in Tensile strength
S ut  108  ksi
b  18 in Yield strength S y  62 ksi
L  20 in Design safety factor Nd  2
Assumptions: There are no applied axial loads. Steel will be used for infinite life. The shaft is machined,
reliability is 90%, and the shaft is at room temperature. The ratio of the fillet radius in the
keyway to shaft diameter is roverd  0.02.

Solution: See Mathcad file P1019a.


1. Calculate the radial load on gear 1:
P1  n  P2 P1  400  lbf

2. Using Figures D-2(a) and D-3(a), the reaction and moment equations are

R1  P1  1 
a  L
  P2  1   R1  67 lbf
 b  b

R2  P1  P2  R1
R2  1467 lbf

M(z) = R1<z>1 - P1<z - a>1 + R2<z-b>1 - P2<z - L>1

3. Define the range for z z  0  in 0.005  L  L

4. For a Mathcad solution, define a step function S. This function will have a value of zero when z is less than u,
and a value of one when it is greater than or equal to u.

S ( z u )  if ( z  u 1 0 )


5. Write the shear and moment in Mathcad form, using the function S as a multiplying factor to get the effect of
the singularity functions.

V ( z)  R1 S ( z 0  in)  P1 S ( z a )  R2 S ( z b )  P2 S ( z L)

M ( z)  R1 S ( z 0  mm)  ( z  0  mm)  P1 S ( z a )  ( z  a ) 


 R2 S ( z b )  ( z  b )  P2 S ( z L)  ( z  L)

6. Plot the shear and moment diagrams (see Figure 10-19a). The maximum moment occurs at the right-hand
bearing and is MB  M ( b ) . The moment at the left-hand gear is MA  M ( a ) . The moment at the
right-hand gear is zero so the two points of interest are A and B. At A the bending moment is lower than at B
but there is a stress concentration due to the key seat. At B the moment is a maximum but there is no stress
concentration. We will investigate both points.

MA  1067 in lbf MB  2000 in lbf

© 2011 Pearson Education, Inc., Upper Saddle River, NJ. All rights reserved. This publication is protected by Copyright and written permission should be
MACHINE DESIGN - An Integrated Approach, 4th Ed. 10-19a-2

Shear Diagram Moment Diagram


1000 0

 500
500
V ( z) M ( z)
 1000
lbf lbf  in
0
 1500

 500  2000
0 5 10 15 20 0 5 10 15 20
z z
FIGURE 10-19a in in
Shear and Moment Diagrams for Problem 10-19a

7. Calculate the mean and alternating components of torque.

Tmax  Tmin
Tm  Tm  1000 in lbf
2
Tmax  Tmin
Ta  Ta  1000 in lbf
2

Right-hand bearing (point B)


8. Calculate the unmodified endurance limit. S'e  0.5 S ut S'e  54 ksi
9. Determine the endurance limit modification factors for a rotating round shaft.

Load Cload  1
 0.097
Csize( d )  0.869   
d
Size 
 in 
Surface A  2.70 b  0.265 (machined)

b
 S ut 
Csurf  A    Csurf  0.781
 ksi 
Temperature Ctemp  1

Reliability Creliab  0.897 (R = 90%)

10. Determine the modified endurance limit as a function of the unknown diameter, d.
S e( d )  Cload  Csize( d )  Csurf  Ctemp Creliab S'e

11. Use equation (10.8) with unity for all stress concentration factors as a design equation to find d.
Guess d  1  in

© 2011 Pearson Education, Inc., Upper Saddle River, NJ. All rights reserved. This publication is protected by Copyright and written permission should be
MACHINE DESIGN - An Integrated Approach, 4th Ed. 10-19a-3

Given
1
3
  2 3 2 
 32 Nd  MB  4  Ta 3 m 
T
d=    
 π  S e( d ) 4 S ut 

d B  Find ( d ) d B  1.153  in

Using this value for d, the size modification factor and endurance limit are:
Size modification factor Csize d B  0.857

Endurance limit S e d B  32.4 ksi


Left-hand gear (point A)
12. Determine the geometric stress concentration factors from Figure 10-16 using a curve-fit equation.
 0.4115
Bending Kt  0.452  ( roverd) Kt  2.3
 0.230
Torsion Kts  1.251  ( roverd) Kts  3.1

13. Determine the notch sensitivity of the material in bending and torsion. Note from Figure 6-36, part 1, that a
value of 20 ksi should be added to S ut to obtain a 1/2 from Table 6-6 when the loading is torsional.

Bending:
Lookup value of S ut S'ut  S ut S'ut  108  ksi
2 0.5
Neuber constant a  0.057  in a  0.057  in

Notch radius r( d )  roverd d


1
Notch sensitivity q ( d ) 
a
1
r( d )

Torsion:

Lookup value of S ut S'ut  S ut  20 ksi S'ut  128  ksi


2 0.5
Neuber constant a s  0.045  in a s  0.045  in

Notch radius r( d )  roverd d


1
Notch sensitivity q s( d ) 
as
1
r( d )

14. Determine the fatigue stress concentration factors from equation 6.11b and 6.17.

Kf ( d )  1  q ( d )   Kt  1 
Bending
Torsion Kfs( d )  1  q s( d )   Kts  1 

© 2011 Pearson Education, Inc., Upper Saddle River, NJ. All rights reserved. This publication is protected by Copyright and written permission should be
MACHINE DESIGN - An Integrated Approach, 4th Ed. 10-19a-4

Mean factor Kfsm( d )  Kfs( d )

15. Repeat step 11 using these stress concentration factors with the mean and alternating stresses.
Given 1
3
  
 32 Nd   Kf ( d )  MA  4   Kfs( d)  Ta
2 3 2
3 Tm 
d=    Kfsm( d )  
 π  S e( d ) 4 S ut

Diameter required at point A: d A  Find ( d ) d A  1.337  in

16. Check the assumption that Kfsm = Kfs using equation 6.17.

  Tm  Ta
16
τmaxnom  τmaxnom  10.2 ksi
3
π d

Kfs( d )  τmaxnom  26.2 ksi 0.577  S y  35.774 ksi

Since Kfsmax is less than S sy, the assumption was valid.

17. Since d A is larger than d B, it must be used as the minimum shaft diameter. Thus, d  d A. d  1.337  in

18. The factor of safety at A is Nd  2 . The safety factor at B is

1
 2 3 2 
3  MB   Ta
π d 4 3 m
T
NB      NB  3.1
32  S e( d ) 4 S ut 

© 2011 Pearson Education, Inc., Upper Saddle River, NJ. All rights reserved. This publication is protected by Copyright and written permission should be
MACHINE DESIGN - An Integrated Approach, 4th Ed. 10-20-1
PROBLEM 10-20
Statement: A 300-mm-long, solid, straight shaft is supported in self-aligning bearings at each end. A gear
is attached at the middle of the shaft with a 10-mm square, steel key in a slot. The geometric
stress-concentration factor in the keyslot is 2.5 and its corner radius is 0.5 mm. The gear drives
a fluctuating load that creates a bending moment that varies from +10 N-m to +100 N-m and a
torque that varies from -35 N-m to +170 N-m each cycle. The material chosen is cold-drawn 4140
steel, hardened and tempered to RC 45 (S ut = 1250 MPa). Design the shaft for infinite life and
determine the diameter needed for a safety factor of 1.5.

Given: Minimum moment Mmin  10 N  m Stress conc. factor Kt  2.5


Maximum moment Mmax  100  N  m Notch radius r  0.5 mm
Minimum torque Tmin  35 N  m Tensile strength S ut  1250 MPa
Maximum torque Tmax  170  N  m Design safety factor Nfd  1.5

Assumptions: 1. The finish is machined, reliability is 90%, and the shaft is at room temperature.
2. The geometric stress concentration factors are the same for bending and torsion.
3. Note that the given magnitudes of the radial forces shown acting on gear(s) in this problem
are not necessarily consistent with a load associated with the given torque for any real gear of
reasonable pressure angle. Since gears are taken up in a later chapter, these shaft design
problems ignore the real gear loadings and use an arbitrary value to provide a shaft design
exercise.
Solution: See Mathcad file P1020.
1. Calculate the mean and alternating components of bending moment and torque.

Mmax  Mmin
Mm  Mm  55 N  m
2
Mmax  Mmin
Ma  Ma  45 N  m
2
Tmax  Tmin
Tm  Tm  68 N  m
2
Tmax  Tmin
Ta  Ta  103  N  m
2

2. Determine the notch sensitivity of the material for bending and torsion. Note from Figure 6-36, part 1, that a
value of 20 ksi should be added to S ut to obtain a 1/2 from Table 6-6 for the torsional Neuber constant.

Bending Lookup value of S ut S ut  181  ksi


2 0.5
Neuber constant a N  0.024  in a N  0.024  in

1
Notch sensitivity q  q  0.854
aN
1
r

Torsion Lookup value of S ut S'ut  S ut  20 ksi S'ut  201  ksi


2 0.5
Neuber constant a N  0.018  in a N  0.018  in
1
Notch sensitivity q s  q s  0.886
aN
1
r
© 2011 Pearson Education, Inc., Upper Saddle River, NJ. All rights reserved. This publication is protected by Copyright and written permission should be
MACHINE DESIGN - An Integrated Approach, 4th Ed. 10-20-2

3. Determine the fatigue stress concentration factors from equation (6.11b).

Bending factor Kf  1  q   Kt  1  Kf  2.281

Torsion factor Kfs  1  q s  Kt  1  Kfs  2.329

Assume Kfm  Kf Kfsm  Kfs

4. Calculate the unmodified endurance limit. S'e  0.5 S ut S'e  625  MPa
5. Determine the endurance limit modification factors for a rotating round shaft.

Load Cload  1
 0.097
Csize( d )  1.189   
d
Size 
 mm 

Surface A  4.51 b'  0.265 (machined)

b'
 Sut 
Csurf  A    Csurf  0.682
 MPa 
Temperature Ctemp  1

Reliability Creliab  0.897 (R = 90%)

6. Determine the modified endurance limit as a function of the unknown diameter, d.

S e( d )  Cload  Csize( d )  Csurf  Ctemp Creliab S'e

7. Use equation (10.8) as a design equation to find d.

Guess d  20 mm
Given
1
3
  2
 32 Nfd   Kf  Ma  4   Kfs Ta Kfm Mm   Kfsm Tm
2 3 2 2 3
 
4
d=   
 π  S e( d ) S ut 

d  Find ( d ) d  23.395 mm Round this to d  22 mm

Using this value for d, the size modification factor and endurance limit are:

Size modification factor Csize( d )  0.881

Endurance limit S e( d )  337  MPa

© 2011 Pearson Education, Inc., Upper Saddle River, NJ. All rights reserved. This publication is protected by Copyright and written permission should be
MACHINE DESIGN - An Integrated Approach, 4th Ed. 10-21-1

PROBLEM 10-21 _____


Statement: Figure P10-7 shows a dial assembly. The dial is 500-mm-dia by 16-mm-thick, solid steel and is
bolted to the top flange of the shaft. The shaft is 70 mm dia over length L1, 40 mm dia over
length L2, and is carried on tapered roller bearings. All shaft fillet radii are 0.5 mm. A 5 kW
motor drives the shaft through a 20:1 reduction gearbox and a commercial bellows coupling.
The maximum rated torque from the motor is 17.75 N-m and its stall torque is 3x its rated torque.
Find the maximum stress and angular deflection in the shaft under conditions of dial locked and
motor at stall.
Given: Rated motor torque TMR  17.75  N  m Stall torque factor fstall  3
Gear reduction ratio mG  20 Modulus of rigidity G  80.8 GPa
Shaft dimensions d 1  70 mm L1  144  mm d 2  40 mm L2  24 mm
Solution: See Mathcad file P1021.

1. For a locked dial the motor will be stalled. The torque on the shaft at the gearbox output will be
Tss  TMR fstall  mG Tss  1065 N  m

2. Calculate the polar moment of inertia and outside radius of each section of the shaft.
4
π d 1 6 4
J1  J1  2.357  10  mm r1  0.5 d 1 r1  35 mm
32
4
π d 2 5 4
J2  J2  2.513  10  mm r2  0.5 d 2 r2  20 mm
32

3. Calculate the stress in each section of the shaft when the dial is locked.

Tss r1 Tss r2
τ1  τ1  15.8 MPa τ2  τ2  84.8 MPa
J1 J2

The maximum stress for a locked dial is in the smaller diameter shaft and is
τmax  τ2 τmax  84.8 MPa

4. Calculate the deflection of the shaft under the stall torque.

Tss L1 Tss L2
θstall   θstall  0.118  deg
J 1 G J2 G

© 2011 Pearson Education, Inc., Upper Saddle River, NJ. All rights reserved. This publication is protected by Copyright and written permission should be
MACHINE DESIGN - An Integrated Approach, 4th Ed. 10-22-1

PROBLEM 10-22 _____

Statement: For the dial assembly of Problem 10-21 and Figure P10-7, find the first torsional natural frequency
the shaft-dial assembly as felt at the motor shaft. The gearbox has a torsional stiffness of 1.56E5
N-m/rad and the coupling has a torsional stiffness of 5.1E5 N-m/rad. Assume the dial is empty of
tooling and that ground support is infinitely stiff.

Given: Dial dimensions: D  500  mm t  16 mm


3
Gear reduction ratio mG  20 Density of steel ρ  7800 kg m
Shaft dimensions d 1  70 mm L1  144  mm d 2  40 mm L2  24 mm
5 5
Gearbox stiffness kG  1.56 10  N  m Coupling stiffness kC  5.1 10  N  m

Solution: See Mathcad file P1022.

1. Calculate the mass of the dial (the other masses will be neglected).

 π D2 
mD     t ρ mD  24.504 kg
 4 
2. Calculate the mass moment of inertia of the dial.
2
 mD 
1 D 2.000
ID   ID  0.766 m  kg
2 2

3. Calculate the stiffness of the shaft using equations 10.27. For steel, G  80.8 GPa
4 4
π d 1 6 4 π d 2 5 4
J1  J1  2.357  10  mm J2  J2  2.513  10  mm
32 32

J 1 G 6 N m J 2 G 5 N m
k1  k1  1.323  10  k2  k2  8.461  10 
L1 rad L2 rad

1
5 N m
kshaft  
1 1
  kshaft  5.160  10 
 k1 k2  rad

4. Calculate the stiffness of the shaft-coupling-gearbox assembly.


1
4 N m
ktotal  
1 1 1 
   ktotal  9.700  10 
 kshaft kC kG  rad

5. Using equation 10.27a, calculate the torsional natural frequency.

ktotal rad ωn
ωn  ωn  356  fn  fn  56.6 Hz
ID sec 2 π

© 2011 Pearson Education, Inc., Upper Saddle River, NJ. All rights reserved. This publication is protected by Copyright and written permission should be
MACHINE DESIGN - An Integrated Approach, 4th Ed. 10-23-1

PROBLEM 10-23 _____


Statement: For the dial assembly of Problem 10-21 and Figure P10-7, find the first torsional natural frequency
the shaft-dial-tool assembly as felt at the motor shaft. The gearbox has a torsional stiffness of
1.56E5 N-m/rad and the coupling has a torsional stiffness of 5.1E5 N-m/rad. The dial is fitted with
20 sets of tooling bolted to its top surface, equispaced on a 416-mm bolt circle. Each tooling
assembly weighs 75.62 N. Assume all ground support is infinitely stiff.

Given: Dial dimensions: D  500  mm t  16 mm


3
Gear reduction ratio mG  20 Density of steel ρ  7800 kg m
Shaft dimensions d 1  70 mm L1  144  mm d 2  40 mm L2  24 mm
5 5
Gearbox stiffness kG  1.56 10  N  m Coupling stiffness kC  5.1 10  N  m
Tooling: Bolt circle dia Dt  416  mm Weight Wt  75.62  N Nt  20

Solution: See Mathcad file P1023.


1. Calculate the mass of the dial and the tools.

 π D2  Wt
mD     t ρ mD  24.504 kg mt  Nt mt  154.222 kg
 4  g

2. Calculate the mass moment of inertia of the dial and the tools.
2 2
 Dt 
ID   mD  
1 D 2.000 2.000
ID  0.766 m  kg It  mt   It  6.672 m  kg
2 2 2
2.000
Itotal  ID  It Itotal  7.438 m  kg

3. Calculate the stiffness of the shaft using equations 10.27. For steel, G  80.8 GPa
4 4
π d 1 6 4 π d 2 5 4
J1  J1  2.357  10  mm J2  J2  2.513  10  mm
32 32

J 1 G 6 N m J 2 G 5 N m
k1  k1  1.323  10  k2  k2  8.461  10 
L1 rad L2 rad

1
5 N m
kshaft  
1 1
  kshaft  5.160  10 
 k1 k2  rad

4. Calculate the stiffness of the shaft-coupling-gearbox assembly.


1
4 N m
ktotal  
1 1 1 
   ktotal  9.700  10 
 kshaft kC kG  rad

5. Using equation 10.27a, calculate the torsional natural frequency.

ktotal rad ωn
ωn  ωn  114  fn  fn  18.2 Hz
Itotal sec 2 π

© 2011 Pearson Education, Inc., Upper Saddle River, NJ. All rights reserved. This publication is protected by Copyright and written permission should be
MACHINE DESIGN - An Integrated Approach, 4th Ed. 10-24-1

PROBLEM 10-24 _____

Statement: For the dial assembly of Problem 10-21 and Figure P10-7, find the first torsional natural frequency
the shaft-dial assembly as felt at the motor shaft. The gearbox has a torsional stiffness of 1.56E5
N-m/rad, the coupling has a torsional stiffness of 5.1E5 N-m/rad, and the motor mount has a
torsional stiffness of 2.44E5 N-m. Assume the dial is empty of tooling.

Given: Dial dimensions: D  500  mm t  16 mm


3
Gear reduction ratio mG  20 Density of steel ρ  7800 kg m
Shaft dimensions d 1  70 mm L1  144  mm d 2  40 mm L2  24 mm
5 5
Gearbox stiffness kG  1.56 10  N  m Coupling stiffness kC  5.1 10  N  m
5
Motor mount stiffness km  2.44 10  N  m

Solution: See Mathcad file P1024.

1. Calculate the mass of the dial (the other masses will be neglected).

 π D2 
mD     t ρ mD  24.504 kg
 4 
2. Calculate the mass moment of inertia of the dial.
2
 mD 
1 D 2.000
ID   ID  0.766 m  kg
2 2

3. Calculate the stiffness of the shaft using equations 10.27. For steel, G  80.8 GPa
4 4
π d 1 6 4 π d 2 5 4
J1  J1  2.357  10  mm J2  J2  2.513  10  mm
32 32

J 1 G 6 N m J 2 G 5 N m
k1  k1  1.323  10  k2  k2  8.461  10 
L1 rad L2 rad

1
1  1 5 N m
kshaft  k  kshaft  5.160  10 
 1 k2  rad

4. Calculate the stiffness of the shaft-coupling-gearbox-motor-mount assembly.


1
 1  1  1  1 4 N m
ktotal  k  ktotal  6.941  10 
 shaft kC kG km  rad

5. Using equation 10.27a, calculate the torsional natural frequency.

ktotal rad ωn
ωn  ωn  301  fn  fn  47.9 Hz
ID sec 2 π

© 2011 Pearson Education, Inc., Upper Saddle River, NJ. All rights reserved. This publication is protected by Copyright and written permission should be
MACHINE DESIGN - An Integrated Approach, 4th Ed. 10-25-1

PROBLEM 10-25 _____


Statement: The tooling on the dial assembly of Problem 10-21 and Figure P10-7 imparts a time varying torque
to the dial that ranges from a peak of 30% of the motor's rated torque (at the motor) to zero, 20 time
per dial revolution. If the motor speed is 600 rpm, find the worst-case stress-time and
deflection-time functions for the shaft. Choose a shaft material to give a safety factor of at least 3
against failure. Disregard the response of the system to the forced vibration.

Given: Rated motor torque TMR  17.75  N  m Stall torque factor fstall  3
Gear reduction ratio mG  20 Modulus of rigidity G  80.8 GPa
Shaft dimensions d 1  70 mm L1  144  mm d 2  40 mm L2  24 mm
Motor speed n m  600  rpm Tool torque factor ftool  0.3
Design safety factor Nd  3 Number of tools Nt  20

Solution: See Mathcad file P1025.


1. Calculate the mean and alternating torque components on the dial shaft.
Maximum torque Tmax  ftool  TMR mG Tmax  106.5  N  m

Minimum torque Tmin  0  N  m

Tmax  Tmin
Mean torque Tm  Tm  53.25  N  m
2
Tmax  Tmin
Alternating torque Ta  Ta  53.25  N  m
2

2. Calculate the polar moment of inertia and outside radius of each section of the shaft.
4
π d 1 6 4
J1  J1  2.357  10  mm r1  0.5 d 1 r1  35 mm
32
4
π d 2 5 4
J2  J2  2.513  10  mm r2  0.5 d 2 r2  20 mm
32

3. Calculate the mean and alternating stress components in the smaller diameter shaft.

Tm r2 Ta r2
τ2m  τ2m  4.24 MPa τ2a  τ2a  4.24 MPa
J2 J2

σ'm  3  τ2m σ'm  7.34 MPa σ'a  3  τ2a σ'a  7.34 MPa

4. Calculate the mean and alternating deflection of the shaft.

Tm L1 Tm L2 3
θm   θm  5.913  10  deg
J 1 G J 2 G

Ta L1 Ta L2 3
θa   θa  5.913  10  deg
J1 G J2 G

5. Calculate the endurance limit correction factors using equations 6.7 and the modified endurance limit, as a
function of S ut, using equations 6.5a and 6.6.

© 2011 Pearson Education, Inc., Upper Saddle River, NJ. All rights reserved. This publication is protected by Copyright and written permission should be
MACHINE DESIGN - An Integrated Approach, 4th Ed. 10-25-2

 0.097
 d2 
Cload  1 Csize  1.189    Csize  0.831
 mm 
 0.265
 S ut 
Csurf  S ut  4.51   Ctemp  1 Creliab  0.897 (R = 0.90)
 MPa 

S e S ut  Cload  Csize Csurf  S ut  Ctemp Creliab 0.5 S ut

6. Calculate the fatigue stress-concentration factors at the step where shafts 1 and 2 join for a fillet radius of
r  0.5 mm. Use Figure C-3 in appendix C to calculate the geometric stress-concentration factor and
equations 6.11 and 6.17 to calculate the fatigue factors.

d1
Doverd  Doverd  1.750
d2

2  Doverd
A   ( 0.84897  0.86331 )  0.86331 A  0.8580
2  1.33
2  Doverd
b   ( 0.23161  0.23865 )  0.23865 b  0.2360
2  1.33
b
Kts  A   
r
Kts  2.41
 d2 
Neuber constant function:
 2 3 
a ( S )  8.69657  10  
  3.10030  10 7  S    in
2 3 S 5 S
 2.75956  10  
 3.94116  10  ksi 
 ksi  ksi    
 4 5 6 
 1.38322  10     3.28018  10     3.21209  10   
9 S  12 S  15 S

  ksi   ksi   ksi  

q  S ut 
1
Notch sensitivity
a  S ut  20 ksi
1
r

Alternating factor Kfs S ut  1  q  S ut   Kts  1 

Mean factor Kfsm S ut  Kfs S ut

7. Using equation 10.8, solve for the minimum required ultimate tensile strength. Guess S ut  200  MPa

1
3
  3 
 32 Nd  4  Kfs Sut  Ta  Kfsm S ut  Tm 
3
4
Given d2 =    
 π  S e S ut S ut 

S ut  Find  S ut S ut  112  MPa

© 2011 Pearson Education, Inc., Upper Saddle River, NJ. All rights reserved. This publication is protected by Copyright and written permission should be
MACHINE DESIGN - An Integrated Approach, 4th Ed. 10-26-1

PROBLEM 10-26 _____


Statement: Combine the data from Problems 10-23 and 10-25 and
(a) Find the ratio between the torsional forcing frequency and the first torsional natural
frequency of the dial assembly.
(b) Using the frequency ratio calculated in part (a), repeat Problem 10-25 taking into account
the forced vibration response of the system if the damping ratio  = 0.20.

Given: Dial dimensions: D  500  mm t  16 mm


3
Gear reduction ratio mG  20 Density of steel ρ  7800 kg m
Shaft dimensions d 1  70 mm L1  144  mm d 2  40 mm L2  24 mm
5 5
Gearbox stiffness kG  1.56 10  N  m Coupling stiffness kC  5.1 10  N  m
Tooling: Bolt circle dia Dt  416  mm Weight Wt  75.62  N Nt  20
Motor speed n m  600  rpm Tool torque factor ftool  0.3
Rated motor torque TMR  17.75  N  m Design safety factor Nd  3
Solution: See Mathcad file P1026.
1. Calculate the mass of the dial and the tools.

 π D2  Wt
mD     t ρ mD  24.504 kg mt  Nt mt  154.222 kg
 4  g

2. Calculate the mass moment of inertia of the dial and the tools.
2 2
 Dt 
ID   mD  
1 D 2.000 2.000
ID  0.766 m  kg It  mt   It  6.672 m  kg
2 2 2
2.000
Itotal  ID  It Itotal  7.438 m  kg

3. Calculate the stiffness of the shaft using equations 10.27. For steel, G  80.8 GPa
4 4
π d 1 6 4 π d 2 5 4
J1  J1  2.357  10  mm J2  J2  2.513  10  mm
32 32

J 1 G 6 N m J 2 G 5 N m
k1  k1  1.323  10  k2  k2  8.461  10 
L1 rad L2 rad

1
5 N m
kshaft  
1 1
  kshaft  5.160  10 
 k1 k2  rad

4. Calculate the stiffness of the shaft-coupling-gearbox assembly.


1
 1  1  1 4 N m
ktotal  k  ktotal  9.700  10 
 shaft kC kG  rad

5. Using equation 10.27a, calculate the torsional natural frequency.

ktotal rad ωn
ωn  ωn  114  fn  fn  18.2 Hz
Itotal sec 2 π

6. Determine the forcing frequency.

© 2011 Pearson Education, Inc., Upper Saddle River, NJ. All rights reserved. This publication is protected by Copyright and written permission should be
MACHINE DESIGN - An Integrated Approach, 4th Ed. 10-26-2

nm
Dial speed n D  n D  30.0 rpm
mG

rad ωf
ωf  Nt n D ωf  62.8 ff  ff  10.0 Hz
sec 2 π

7. (a) Calculate the ratio of forcing frequency to natural frequency.

ωf
Fratio  Fratio  0.55
ωn

8. Determine the amplitude ratio of the dial-tooling-shaft-coupling-gearbox system, when subjected to the
externally forced vibration, using the graph in Figure 10-26(a). For the frequency ratio Fratio  0.55 , and
damping ratio  = 0.20, the amplitude (and torque) magnification is Y  1.46 .

9. Calculate the mean and alternating torque response components on the dial shaft.
Maximum torque Tmax  Y  ftool  TMR mG Tmax  155.5  N  m

Minimum torque Tmin  0  N  m

Tmax  Tmin
Mean torque Tm  Tm  77.75  N  m
2
Tmax  Tmin
Alternating torque Ta  Ta  77.75  N  m
2

10. Calculate the polar moment of inertia and outside radius of each section of the shaft.
4
π d 1 6 4
J1  J1  2.357  10  mm r1  0.5 d 1 r1  35 mm
32
4
π d 2 5 4
J2  J2  2.513  10  mm r2  0.5 d 2 r2  20 mm
32

11. Calculate the mean and alternating stress components in the smaller diameter shaft.

Tm r2 Ta r2
τ2m  τ2m  6.19 MPa τ2a  τ2a  6.19 MPa
J2 J2

σ'm  3  τ2m σ'm  10.72  MPa σ'a  3  τ2a σ'a  10.72  MPa

12. Calculate the mean and alternating deflection of the shaft.

Tm L1 Tm L2 3
θm   θm  8.632  10  deg
J 1 G J 2 G
Ta L1 Ta L2 3
θa   θa  8.632  10  deg
J1 G J2 G
13. Calculate the endurance limit correction factors using equations 6.7 and the modified endurance limit, as a
function of S ut, using equations 6.5a and 6.6.
 0.097
 d2 
Cload  1 Csize  1.189    Csize  0.831
 mm 
© 2011 Pearson Education, Inc., Upper Saddle River, NJ. All rights reserved. This publication is protected by Copyright and written permission should be
MACHINE DESIGN - An Integrated Approach, 4th Ed. 10-26-3

 0.265
 S ut 
Csurf  S ut  4.51   Ctemp  1 Creliab  0.897 (R = 0.90)
 MPa 

S e S ut  Cload  Csize Csurf  S ut  Ctemp Creliab 0.5 S ut

14. Calculate the fatigue stress-concentration factors at the step where shafts 1 and 2 join for a fillet radius of
r  0.5 mm. Use Figure C-3 in appendix C to calculate the geometric stress-concentration factor and
equations 6.11 and 6.17 to calculate the fatigue factors.

d1
Doverd  Doverd  1.750
d2

2  Doverd
A   ( 0.84897  0.86331 )  0.86331 A  0.8580
2  1.33
2  Doverd
b   ( 0.23161  0.23865 )  0.23865 b  0.2360
2  1.33
b
Kts  A   r
 Kts  2.41
 2
d

Neuber constant function:

 2 3 
a ( S )  8.69657  10  
  3.10030  10 7  S    in
2 3 S 5 S
 2.75956  10  
 3.94116  10  ksi 
 ksi  ksi    
 4 5 6 
 1.38322  10     3.28018  10     3.21209  10   
9 S  12 S  15 S

  ksi   ksi   ksi  

q  S ut 
1
Notch sensitivity
a  S ut  20 ksi
1
r

Alternating factor Kfs S ut  1  q  S ut   Kts  1 

Mean factor Kfsm S ut  Kfs S ut

15. (b) Using equation 10.8, solve for the minimum required ultimate tensile strength. Guess S ut  200  MPa

1
3
  3 
 32 Nd  4  Kfs Sut  Ta  Kfsm S ut  Tm 
3
4
Given d2 =    
 π  S e S ut S ut 

S ut  Find  S ut S ut  187  MPa

© 2011 Pearson Education, Inc., Upper Saddle River, NJ. All rights reserved. This publication is protected by Copyright and written permission should be
MACHINE DESIGN - An Integrated Approach, 4th Ed. 10-27-1

PROBLEM 10-27 _____


Statement: An impulsive impact load of 500 N is applied tangentially to the dial rim 20 times per dial
revolution. If the motor speed is 30 rpm, find the worst-case stress-time and deflection-time
functions for the shaft. Choose a shaft material to give a safety factor of at least 1.5 against
failure.

Given: Dial diameter D  500  mm


Gear reduction ratio mG  20 Modulus of rigidity G  80.8 GPa
Shaft dimensions d 1  70 mm L1  144  mm d 2  40 mm L2  24 mm
Motor speed n m  30 rpm Impact load F  500  N
Design safety factor Nd  1.5 Number of tools Nt  20

Solution: See Mathcad file P1027.


1. Calculate the mean and alternating torque components on the dial shaft. Assume that the impact load causes
a reaction in the dial shaft that is 3 times the applied load. (See discussion of force impact loads in Section
3.8).

D
Maximum torque Tmax  3  F  Tmax  375.0  N  m
2

Minimum torque Tmin  0  N  m

Tmax  Tmin
Mean torque Tm  Tm  187.50 N  m
2
Tmax  Tmin
Alternating torque Ta  Ta  187.50 N  m
2

2. Calculate the polar moment of inertia and outside radius of each section of the shaft.
4
π d 1 6 4
J1  J1  2.357  10  mm r1  0.5 d 1 r1  35 mm
32
4
π d 2 5 4
J2  J2  2.513  10  mm r2  0.5 d 2 r2  20 mm
32

3. Calculate the mean and alternating stress components in the smaller diameter shaft.

Tm r2 Ta r2
τ2m  τ2m  14.92  MPa τ2a  τ2a  14.92  MPa
J2 J2

σ'm  3  τ2m σ'm  25.84  MPa σ'a  3  τ2a σ'a  25.84  MPa

4. Calculate the mean and alternating deflection of the shaft.

Tm L1 Tm L2
θm   θm  0.021  deg
J 1 G J 2 G

Ta L1 Ta L2
θa   θa  0.021  deg
J1 G J2 G

5. Calculate the endurance limit correction factors using equations 6.7 and the modified endurance limit, as a
function of S ut, using equations 6.5a and 6.6.
© 2011 Pearson Education, Inc., Upper Saddle River, NJ. All rights reserved. This publication is protected by Copyright and written permission should be
MACHINE DESIGN - An Integrated Approach, 4th Ed. 10-27-2

 0.097
 d2 
Cload  1 Csize  1.189    Csize  0.831
 mm 
 0.265
 S ut 
Csurf  S ut  4.51   Ctemp  1 Creliab  0.897 (R = 0.90)
 MPa 

S e S ut  Cload  Csize Csurf  S ut  Ctemp Creliab 0.5 S ut

6. Calculate the fatigue stress-concentration factors at the step where shafts 1 and 2 join for a fillet radius of
r  0.5 mm. Use Figure C-3 in appendix C to calculate the geometric stress-concentration factor and
equations 6.11 and 6.17 to calculate the fatigue factors.

d1
Doverd  Doverd  1.750
d2

2  Doverd
A   ( 0.84897  0.86331 )  0.86331 A  0.8580
2  1.33
2  Doverd
b   ( 0.23161  0.23865 )  0.23865 b  0.2360
2  1.33
b
Kts  A   
r
Kts  2.41
 d2 
Neuber constant function:

 2 3 
a ( S )  8.69657  10  
  3.10030  10 7  S    in
2 3 S 5 S
 2.75956  10  
 3.94116  10  
 ksi  ksi   ksi  
 4 5 6 
 1.38322  10     3.28018  10     3.21209  10   
9 S  12 S  15 S

  ksi   ksi   ksi  

q  S ut 
1
Notch sensitivity
a  S ut  20 ksi
1
r

Alternating factor Kfs S ut  1  q  S ut   Kts  1 

Mean factor Kfsm S ut  Kfs S ut

7. Using equation 10.8, solve for the minimum required ultimate tensile strength. Guess S ut  200  MPa

1
3
  3 
 32 Nd  4  Kfs Sut  Ta  Kfsm S ut  Tm 
3
4
Given d2 =    
 π  S e S ut S ut 

S ut  Find  S ut S ut  244  MPa

© 2011 Pearson Education, Inc., Upper Saddle River, NJ. All rights reserved. This publication is protected by Copyright and written permission should be
MACHINE DESIGN - An Integrated Approach, 4th Ed. 10-28-1

PROBLEM 10-28 _____


Statement: For the dial assembly of Problem 10-21 and Figure P10-7, size a square key to couple the 40-mm-d
dial shaft to the coupling.
Given: Shaft stall torque Tss  1065 N  m Shaft diameter d  40 mm

Design choices:
Shaft material: SAE 1050, normalized steel S y1  427  MPa S ut1  745  MPa
Key material: SAE 1020, cold-rolled steel S y2  393  MPa S ut2  469  MPa
Design safety factor Nd  1, let the key shear at stall torque to protect the
motor.
Key is machined, reliability is 90%, and the dial operates at room temperature.
Maximum torque Tmax  Tss

Minimum torque Tmin  0  N  m

Solution: See Mathcad file P1028.

1. As recommended in Table 10-2, for a shaft diameter of d  40.0 mm, use a square key of width
Key width w  12 mm
2. Determine the alternating and mean key shear force components.

Mean and alternating 1 Tmax  Tmin


force components Fa   Fa  26625  N
2 0.5 d

1 Tmax  Tmin
Fm   Fm  26625  N
2 0.5 d

3. Write the equations for the mean and alternating components of the shear stress and the effective von Mises
stress.
Key shear area as
function of length Ashear = w L

Mean and alternating Fa Fm


shear stresses τa = τm =
w L w L

Mean and alternating Fa Fm


von Mises stresses σ'a = 3  τa = 3  σ'm = 3  τm = 3 
w L w L

4. Using the modified Goodman failure criterion, the design equation is

S e S ut S e S ut
Nd = =
S ut σ'a  S e σ'm  3 
   S ut Fa  S e Fm
 w L 

3  Nd   S ut Fa  S e Fm
Solving for L L=
w S ut S e

5. Determine the key material endurance limit.

© 2011 Pearson Education, Inc., Upper Saddle River, NJ. All rights reserved. This publication is protected by Copyright and written permission should be
MACHINE DESIGN - An Integrated Approach, 4th Ed. 10-28-2

Uncorrected endurance
strength S'e  0.5 S ut2 S'e  235  MPa

Correction factors:
Load Cload  1
w L
Size A95 = w L d eq =
0.0766

 0.097
Csize = 1.189  d eq

 0.5 0.097
Cs  1.189   
w
 Cs  0.931
 0.0766  mm 
 0.0485
Csize = 0.931  L
 0.265
 S ut2 
Surface (machined) Csurf  4.51   Csurf  0.884
 MPa 
Temperature Ctemp  1

Reliability Creliab  0.897 (R = 90%)

 0.0485  0.0485
S e = Cload  Csize Csurf  Ctemp Creliab S'e = 1  Cs L  Csurf  1  Crelaib S'e = Ce L

where Ce  Cs Csurf  Creliab S'e Ce  172.973  MPa

6. Substitute S e into the design equation.

3  Nd   S ut Fa  Ce L  Fm


 0.0485
L=
 0.0485
w S ut Ce L

Solving by iteration, let

  0.0485 
 
3  Nd  S ut2 Fa  Ce 
L 
 Fm

RHS( L) 
  mm  
 0.0485
w S ut2 Ce 
L 

 mm 
Guess L  20 mm RHS( L)  33.886 mm
L  RHS( L) RHS( L)  34.551 mm
L  RHS( L) RHS( L)  34.576 mm

Tentatively, let L  35 mm


7. Check the realized factor of safety against fatigue failure.
 0.0485
S e  Ce  L
Endurance limit  S e  145.6  MPa
 mm 
© 2011 Pearson Education, Inc., Upper Saddle River, NJ. All rights reserved. This publication is protected by Copyright and written permission should be
MACHINE DESIGN - An Integrated Approach, 4th Ed. 10-28-3

S e S ut2
Realized factor Nf  Nf  1.0
 3 
  S ut2 Fa  Se Fm
of safety

 w L 

8. Check worst-case static bearing stress.


1 2
Bearing area Abear   w L Abear  210  mm
2

Maximum force Fmax  Fa  Fm Fmax  53.3 kN

Maximum bearing stress Fmax


on key σmax  σmax  254  MPa
Abear

Factor of safety against S y2


static bearing yield Ns  Ns  1.5
σmax

9. Design Summary

Key width w  12 mm (12 x 12 mm)


Key length L  35 mm

© 2011 Pearson Education, Inc., Upper Saddle River, NJ. All rights reserved. This publication is protected by Copyright and written permission should be
MACHINE DESIGN - An Integrated Approach, 4th Ed. 10-29-1

PROBLEM 10-29 _____


Statement: For the dial assembly of Problem 10-25 and Figure P10-7, size a square key to couple the 40-mm-d
dial shaft to the coupling.
Given: Shaft torque Tdial  106.5  N  m Shaft diameter d  40 mm

Design choices:
Shaft material: SAE 1050, normalized steel S y1  427  MPa S ut1  745  MPa
Key material: SAE 1020, cold-rolled steel S y2  393  MPa S ut2  469  MPa
Design safety factor Nd  3

Key is machined, reliability is 90%, and the dial operates at room temperature.
Maximum torque Tmax  Tdial

Minimum torque Tmin  0  N  m

Solution: See Mathcad file P1029.

1. As recommended in Table 10-2, for a shaft diameter of d  40.0 mm, use a square key of width
Key width w  12 mm
2. Determine the alternating and mean key shear force components.

Mean and alternating 1 Tmax  Tmin


force components Fa   Fa  2663 N
2 0.5 d

1 Tmax  Tmin
Fm   Fm  2663 N
2 0.5 d

3. Write the equations for the mean and alternating components of the shear stress and the effective von Mises
stress.
Key shear area as
function of length Ashear = w L

Mean and alternating Fa Fm


shear stresses τa = τm =
w L w L

Mean and alternating Fa Fm


von Mises stresses σ'a = 3  τa = 3  σ'm = 3  τm = 3 
w L w L

4. Using the modified Goodman failure criterion, the design equation is

S e S ut S e S ut
Nd = =
S ut σ'a  S e σ'm  3 
   S ut Fa  S e Fm
 w L 
3  Nd   S ut Fa  S e Fm
Solving for L L=
w S ut S e
5. Determine the key material endurance limit.
Uncorrected endurance
strength S'e  0.5 S ut2 S'e  235  MPa
© 2011 Pearson Education, Inc., Upper Saddle River, NJ. All rights reserved. This publication is protected by Copyright and written permission should be
MACHINE DESIGN - An Integrated Approach, 4th Ed. 10-29-2

Correction factors:
Load Cload  1
w L
Size A95 = w L d eq =
0.0766

 0.097
Csize = 1.189  d eq

 0.5 0.097
Cs  1.189   
w
 Cs  0.931
 0.0766  mm 
 0.0485
Csize = 0.931  L
 0.265
 S ut2 
Surface (machined) Csurf  4.51   Csurf  0.884
 MPa 
Temperature Ctemp  1

Reliability Creliab  0.897 (R = 90%)

 0.0485  0.0485
S e = Cload  Csize Csurf  Ctemp Creliab S'e = 1  Cs L  Csurf  1  Crelaib S'e = Ce L

where Ce  Cs Csurf  Creliab S'e Ce  172.973  MPa

6. Substitute S e into the design equation.

3  Nd   S ut Fa  Ce L  Fm


 0.0485
L=
 0.0485
w S ut Ce L

Solving by iteration, let

  0.0485 
 
3  Nd  S ut2 Fa  Ce 
L 
 Fm

RHS( L) 
  mm  
 0.0485
w S ut2 Ce 
L 

 mm 
Guess L  20 mm RHS( L)  10.166 mm
L  RHS( L) RHS( L)  9.917  mm
L  RHS( L) RHS( L)  9.908  mm

Tentatively, let L  10 mm

7. Check the realized factor of safety against fatigue failure.

 0.0485
S e  Ce  L
Endurance limit  S e  154.7  MPa
 mm 
© 2011 Pearson Education, Inc., Upper Saddle River, NJ. All rights reserved. This publication is protected by Copyright and written permission should be
MACHINE DESIGN - An Integrated Approach, 4th Ed. 10-29-3

S e S ut2
Realized factor Nf  Nf  3.0
 3 
  S ut2 Fa  Se Fm
of safety

 w L 

8. Check worst-case static bearing stress.


1 2
Bearing area Abear   w L Abear  60 mm
2

Maximum force Fmax  Fa  Fm Fmax  5.3 kN

Maximum bearing stress Fmax


on key σmax  σmax  89 MPa
Abear

Factor of safety against S y2


static bearing yield Ns  Ns  4.4
σmax

9. Design Summary

Key width w  12 mm (12 x 12 mm)


Key length L  10 mm

© 2011 Pearson Education, Inc., Upper Saddle River, NJ. All rights reserved. This publication is protected by Copyright and written permission should be
MACHINE DESIGN - An Integrated Approach, 4th Ed. 10-30-1

PROBLEM 10-30 _____


Statement: For the dial assembly of Problem 10-27 and Figure P10-7, size a square key to couple the 40-mm-d
dial shaft to the coupling.
Given: Shaft torque Tdial  375  N  m Shaft diameter d  40 mm

Design choices:
Shaft material: SAE 1050, normalized steel S y1  427  MPa S ut1  745  MPa
Key material: SAE 1020, cold-rolled steel S y2  393  MPa S ut2  469  MPa
Design safety factor Nd  3

Key is machined, reliability is 90%, and the dial operates at room temperature.
Maximum torque Tmax  Tdial

Minimum torque Tmin  0  N  m

Solution: See Mathcad file P1030.


1. As recommended in Table 10-2, for a shaft diameter of d  40.0 mm, use a square key of width
Key width w  12 mm
2. Determine the alternating and mean key shear force components.

Mean and alternating 1 Tmax  Tmin


force components Fa   Fa  9375 N
2 0.5 d

1 Tmax  Tmin
Fm   Fm  9375 N
2 0.5 d

3. Write the equations for the mean and alternating components of the shear stress and the effective von Mises
stress.
Key shear area as
function of length Ashear = w L

Mean and alternating Fa Fm


shear stresses τa = τm =
w L w L

Mean and alternating Fa Fm


von Mises stresses σ'a = 3  τa = 3  σ'm = 3  τm = 3 
w L w L

4. Using the modified Goodman failure criterion, the design equation is

S e S ut S e S ut
Nd = =
S ut σ'a  S e σ'm  3 
   S ut Fa  S e Fm
 w L 
3  Nd   S ut Fa  S e Fm
Solving for L L=
w S ut S e
5. Determine the key material endurance limit.
Uncorrected endurance
strength S'e  0.5 S ut2 S'e  235  MPa

© 2011 Pearson Education, Inc., Upper Saddle River, NJ. All rights reserved. This publication is protected by Copyright and written permission should be
MACHINE DESIGN - An Integrated Approach, 4th Ed. 10-30-2

Correction factors:
Load Cload  1
w L
Size A95 = w L d eq =
0.0766

 0.097
Csize = 1.189  d eq

 0.5 0.097
Cs  1.189   
w
 Cs  0.931
 0.0766 mm 
 0.0485
Csize = 0.931  L
 0.265
 S ut2 
Surface (machined) Csurf  4.51   Csurf  0.884
 MPa 
Temperature Ctemp  1

Reliability Creliab  0.897 (R = 90%)

 0.0485  0.0485
S e = Cload  Csize Csurf  Ctemp Creliab S'e = 1  Cs L  Csurf  1  Crelaib S'e = Ce L

where Ce  Cs Csurf  Creliab S'e Ce  172.973  MPa

6. Substitute S e into the design equation.

3  Nd   S ut Fa  Ce L  Fm


 0.0485
L=
 0.0485
w S ut Ce L

Solving by iteration, let

  0.0485 
3  Nd  S ut2 Fa  Ce  L
  Fm
RHS( L) 
  
mm 
 0.0485
w S ut2 Ce  
L

 mm 
Guess L  20 mm RHS( L)  35.795 mm
L  RHS( L) RHS( L)  36.572 mm
L  RHS( L) RHS( L)  36.601 mm

Tentatively, let L  38 mm

7. Check the realized factor of safety against fatigue failure.

 0.0485
S e  Ce  
L
Endurance limit  S e  145.0  MPa
 
mm

© 2011 Pearson Education, Inc., Upper Saddle River, NJ. All rights reserved. This publication is protected by Copyright and written permission should be
MACHINE DESIGN - An Integrated Approach, 4th Ed. 10-30-3

S e S ut2
Realized factor Nf  Nf  3.1
 3 
  S ut2 Fa  Se Fm
of safety

 w L 

8. Check worst-case static bearing stress.


1 2
Bearing area Abear   w L Abear  228  mm
2

Maximum force Fmax  Fa  Fm Fmax  18.8 kN

Maximum bearing stress Fmax


on key σmax  σmax  82 MPa
Abear

Factor of safety against S y2


static bearing yield Ns  Ns  4.8
σmax

9. Design Summary

Key width w  12 mm (12 x 12 mm)


Key length L  38 mm

© 2011 Pearson Education, Inc., Upper Saddle River, NJ. All rights reserved. This publication is protected by Copyright and written permission should be
MACHINE DESIGN - An Integrated Approach, 4th Ed. 10-31a-1

PROBLEM 10-31a _____


Statement: A simply supported shaft with overhanging load is shown in Figure P10-1. A constant
magnitude transverse force P is applied as the shaft rotates. The shaft is also subject to a
steady torque of Tmax. For the data in the row(s) assigned from Table P10-1 (ignoring Tmin), find
the diameter of shaft required to obtain a safety factor of 2.5 in fatigue loading if the shaft is
steel of S ut = 118 kpsi and S y = 102 kpsi. The dimensions are in inches, the force in pounds, and
the torque is in lb-in. Assume no stress concentrations are present, the shaft is machined, the
required reliability is 90%, and the shaft operates at room temperature.
Given: Distance between bearings a  16 in Distance to P b  18 in
Applied load P  1000 lbf Tensile strength S ut  118  ksi
Maximum torque Tmax  2000 lbf  in Yield strength S y  102  ksi
Design safety factor Nd  2.5

Solution: See Figure P10-1, Table P10-1, and Mathcad file P1031a.
1. The maximum moment in the shaft occurs at the right bearing as seen in the moment diagram in Figure B-3(a) in
Appendix B (note that in the figure a is the distance to the load and b is the distance between bearings). Using
the equation given in the figure, calculate the alternating bending moment (the mean is zero).

Ma  P ( a  b ) Ma  2000 in lbf

2. Calculate the unmodified endurance limit.


S'e  0.5 S ut S'e  59.0 ksi

3. Determine the endurance limit modification factors for a rotating round shaft.
Load Cload  1
 0.097
Csize( d )  0.869   
d
Size 
 in 
Surface A  2.70 b  0.265 (machined)
b
 S ut 
Csurf  A    Csurf  0.763
 ksi 
Temperature Ctemp  1

Reliability Creliab  0.897 (R = 90%)

4. Determine the modified endurance limit as a function of the unknown diameter, d.


S e( d )  Cload  Csize( d )  Csurf  Ctemp Creliab S'e

5. Use equation 10.6a with unity for all stress concentration factors as a design equation to find d.
Guess d  1.00 in 1
3
 1
 2
 32 Nd  Ma  2 3  Tmax  2 
d=     4  S    d  Find ( d )
Given

 π  S e( d )   y  

d  1.153  in

© 2011 Pearson Education, Inc., Upper Saddle River, NJ. All rights reserved. This publication is protected by Copyright and written permission should be
MACHINE DESIGN - An Integrated Approach, 4th Ed. 10-32a-1

PROBLEM 10-32a _____


Statement: Repeat Problem 10-31 taking the stress concentration at the keyway shown in Figure P10-3 into
account.
Given: Distance between bearings a  16 in Distance to P b  18 in
Applied load P  1000 lbf Tensile strength S ut  118  ksi
Maximum torque Tmax  2000 lbf  in Yield strength S y  102  ksi
Design safety factor Nd  2.5
Assumptions: 1. The finish is machined, reliability is 90%, and the shaft is at room temperature.
2. The notch radius in the keyway is r  0.015  in
3. Note that the given magnitudes of the radial forces shown acting on gear(s) in this problem
are not necessarily consistent with a load associated with the given torque for any real gear of
reasonable pressure angle. Since gears are taken up in a later chapter, these shaft design
problems ignore the real gear loadings and use an arbitrary value to provide a shaft design
exercise.
Solution: See Figure P10-3, Table P10-1, and Mathcad file P1032a.
1. There is no stress concentration at the point on the shaft where the bending moment is a maximum. On the
other hand, at the gear where there is a stress concentration, the bending moment is zero. The question is:
which point requires the larger diameter in order to meet the safety factor requirement? To answer this
question, find the diameter required at each point and choose the larger. Start with the point where the
bending moment is maximum.
2. The maximum moment in the shaft occurs at the right bearing as seen in the moment diagram in Figure B-3(a)
in Appendix B (note that in the figure a is the distance to the load and b is the distance between bearings).
Using the equation given in the figure, calculate the alternating bending moment (the mean is zero).

Ma  P ( a  b ) Ma  2000 in lbf

3. Calculate the unmodified endurance limit.


S'e  0.5 S ut S'e  59.0 ksi

4. Determine the endurance limit modification factors for a rotating round shaft.
Load Cload  1
 0.097
Csize( d )  0.869   
d
Size 
 in 

Surface A  2.70 b  0.265 (machined)

b
 S ut 
Csurf  A    Csurf  0.763
 ksi 
Temperature Ctemp  1

Reliability Creliab  0.897 (R = 90%)

5. Determine the modified endurance limit as a function of the unknown diameter, d.


S e( d )  Cload  Csize( d )  Csurf  Ctemp Creliab S'e

6. Use equation 10.6a with unity for all stress concentration factors as a design equation to find d.
Guess d  1.00 in

© 2011 Pearson Education, Inc., Upper Saddle River, NJ. All rights reserved. This publication is protected by Copyright and written permission should be
MACHINE DESIGN - An Integrated Approach, 4th Ed. 10-32a-2

1
3
 1
 2
 32 Nd  Ma  2 3  Tmax  2 
d=     4  S   
Given

 π  S e( d )   y  

d 1  Find ( d ) d 1  1.153  in

7. At the gear the bending moment is zero but the steady torque is the same as above. Also, there is a stress
concentration due to the keyway that must be applied to the mean shear stress present at the gear. First, set the
moment equal to zero: Ma  0  lbf  in, then determine the values of the alternating and mean fatigue stress
concentration factors.

8. Determine the geometric stress concentration factor from the upper curve in Figure 10-16 using a curve-fit
equation.
 0.230
Kts( d )  1.251  
r

d
9. Determine the notch sensitivity of the material. Note from Figure 6-36, part 1, that a value of 20 ksi should be
added to S ut to obtain a 1/2 from Table 6-6. Using a curve-fit to Table 6-6,

 2 3 
a ( S )  8.69657  10  
  3.10030  10 7  S    in
2 3 S 5 S
 2.75956  10  
 3.94116  10  ksi 
 ksi  ksi    
 4 5 6 
 1.38322  10     3.28018  10     3.21209  10   
9 S  12 S  15 S

  ksi   ksi   ksi  

a  a  S ut  20 ksi
0.5
Neuber constant a  0.040  in

Notch radius r  0.015  in


1
Notch sensitivity q  q  0.754
a
1
r

10. Determine the fatigue stress concentration factor from equation (6.11b).

Alternating factor Kfs( d )  1  q   Kts( d )  1 

Mean factor Kfsm( d )  Kfs( d )

11. Repeat step 6 using these stress concentration factors with the torsional stresses.

1
Given
3
 1
 2
 32 Nd  Ma  2 3  2 
Tmax  
d=       Kfsm( d )   
 S y   
 π  S e( d )  4 

© 2011 Pearson Education, Inc., Upper Saddle River, NJ. All rights reserved. This publication is protected by Copyright and written permission should be
MACHINE DESIGN - An Integrated Approach, 4th Ed. 10-32a-3

Diameter required at point of gear attachment: d 2  Find ( d ) d 2  1.061  in

 d1 
Putting the two calculated diameters into a vector trial_dia   
 d2 
12. The larger of the two must be used as the minimum shaft diameter. Thus, d  max( trial_dia ). d  1.153  in

13. The factor of safety at the point of maximum bending moment is Nd  2.500 . The factor of safety at the gear
is equal to the ratio of the two diameters cubed times Nd. Thus,

3
 d1 
Nfgear     Nd Nfgear  3.2
 d2 

© 2011 Pearson Education, Inc., Upper Saddle River, NJ. All rights reserved. This publication is protected by Copyright and written permission should be
MACHINE DESIGN - An Integrated Approach, 4th Ed. 10-33a-1

PROBLEM 10-33a _____


Statement: A simply supported shaft is shown in Figure P10-2. A constant magnitude distributed unit load
p is applied as the shaft rotates. The shaft is also subject to a steady torque of Tmax. For the
data in the row(s) assigned from Table P10-1 (ignoring Tmin), find the diameter of shaft required
to obtain a safety factor of 2.5 in fatigue loading if the shaft is steel of S ut = 814 MPa and S y =
703 MPa. The dimensions are in cm, the distributed force in N/cm, and the torque is in N-m.
Assume no stress concentrations are present, the shaft is machined, the required reliability is
90%, and the shaft operates at room temperature.
Given: Distance between bearings L  20 cm Distance to start of p a  16 cm
Distance to end of p b  18 cm Tensile strength S ut  814  MPa
1
Applied distributed load p  1000 N  cm Yield strength S y  703  MPa
Maximum torque Tmax  2000 N  m Design safety factor Nfd  2.5
Solution: See Figure P10-2, Table P10-1, and Mathcad file P1033a.
1. The maximum moment in the shaft occurs between a and b. See the appendix to this problem below for the
determination of Ma.

Ma  48.45  N  m
2. Calculate the unmodified endurance limit.
S'e  0.5 S ut S'e  407.0  MPa
3. Determine the endurance limit modification factors for a rotating round shaft.
Load Cload  1
 0.097
Csize( d )  1.189   
d
Size 
 
mm

Surface A  4.51 b'  0.265 (machined)

b'
 Sut 
Csurf  A    Csurf  0.764
 MPa 
Temperature Ctemp  1
Reliability Creliab  0.897 (R = 90%)

4. Determine the modified endurance limit as a function of the unknown diameter, d.


S e( d )  Cload  Csize( d )  Csurf  Ctemp Creliab S'e

5. Use equation 10.6a with unity for all stress concentration factors as a design equation to find d.
Guess d  20 mm
1
3
 1
 2
 32 Nfd  Ma  2 3  Tmax  2 
d=     4  S    d  Find ( d ) d  39.8 mm
Given

 π  Se( d)   y  
APPENDIX - Maximum bending moment
1. From inspection of Figure P10-2, write the load function equation
q(x) = R1<x - 0>-1 - p<x - a>0 + p<x - b>0 + R2<x - L>-1
© 2011 Pearson Education, Inc., Upper Saddle River, NJ. All rights reserved. This publication is protected by Copyright and written permission should be
MACHINE DESIGN - An Integrated Approach, 4th Ed. 10-33a-2

2. Integrate this equation from - to x to obtain shear, V(x)


V(x) = R1<x - 0>0 - p<x - a>1 + p<x - b>1 + R2<x - L>0

3. Integrate this equation from - to x to obtain moment, M(x)


M(x) = R1<x - 0>1 - p<x - a>2/2 + p<x - b>2/2 + R2<x - L>1

4. Solve for the reactions by evaluating the shear and moment equations at a point just to the right of x = L,
where both are zero.
At x = L+, V = M = 0
V = R1  p  ( L  a )  p  ( L  b )  R2 = 0
p 2 p 2
M = R 1 L   ( L  a)   ( L  b) = 0
2 2

 ( L  a )  ( L  b )
p 2 2
R1   R1  300.000  N
2 L
R2  p  ( b  a )  R1 R2  1700 N

5. Define the range for x x  0  m 0.005  L  L


6. For a Mathcad solution, define a step function S. This function will have a value of zero when x is less than z,
and a value of one when it is greater than or equal to z.

S ( x z)  if ( x  z 1 0 )


7. Write the shear and moment equations in Mathcad form, using the function S as a multiplying factor to get the
effect of the singularity functions.

V ( x)  R1 S ( x 0  m)  p  S ( x a )  ( x  a )  p  S ( x b )  ( x  b )  R2 S ( x L)


p 2 p 2
M ( x)  R1 S ( x 0  m)  x   S ( x a )  ( x  a )   S ( x b )  ( x  b )
2 2
8. Plot the shear and moment diagrams.

Shear Diagram Moment Diagram


500 50

0
35

V ( x)  500 M ( x)
N 20
 1000 Nm

 1500 5

 2000
0 5 10 15 20  10
0 5 10 15 20
x
x
cm
cm
FIGURE 10-33a
Shear and Moment Diagrams for Problem 10-33a

© 2011 Pearson Education, Inc., Upper Saddle River, NJ. All rights reserved. This publication is protected by Copyright and written permission should be
MACHINE DESIGN - An Integrated Approach, 4th Ed. 10-33a-3

9. Determine the maximum maximum moment from inspection of the diagrams. Maximum moment occurs where
V is zero, which is x = c. From the shear diagram,

ca bc a  R2  b  R1
= c  c  16.300 cm
R1 R2 R1  R2

Mmax  M ( c) Mmax  48.450 N  m

© 2011 Pearson Education, Inc., Upper Saddle River, NJ. All rights reserved. This publication is protected by Copyright and written permission should be
MACHINE DESIGN - An Integrated Approach, 4th Ed. 10-34a-1

PROBLEM 10-34a _____


Statement: Repeat Problem 10-33 taking the stress concentration at the keyways shown in Figure P10-4 into
account.
Given: Distance between bearings L  20 cm Distance to start of p a  16 cm
Distance to end of p b  18 cm Tensile strength S ut  814  MPa
1
Applied distributed load p  1000 N  cm Yield strength S y  703  MPa
Maximum torque Tmax  2000 N  m Design safety factor Nfd  2.5

Assumptions: 1. The finish is machined, reliability is 90%, and the shaft is at room temperature.
2. The ratio of notch radius to shaft diameter in the keyway is roverd  0.021
3. Note that the given magnitudes of the radial forces shown acting on gear(s) in this problem
are not necessarily consistent with a load associated with the given torque for any real gear of
reasonable pressure angle. Since gears are taken up in a later chapter, these shaft design
problems ignore the real gear loadings and use an arbitrary value to provide a shaft design
exercise.
Solution: See Figure P10-4, Table P10-1, and Mathcad file P1034a.
1. There is no stress concentration at the point on the shaft where the bending moment is a maximum, which is
at x = 16.3 cm (see the appendix to this problem, below). The keys are at a  16 cm and b  18 cm. On the
other hand, the key at a  16 cm looks as if it extends into the section where the moment is a maximum so,
use the maximum moment as the alternating bending moment.

2. The maximum moment in the shaft occurs between a and b. See the appendix to this problem below for the
determination of Ma.

Ma  48.45  N  m
3. Calculate the unmodified endurance limit.
S'e  0.5 S ut S'e  407.0  MPa

4. Determine the endurance limit modification factors for a rotating round shaft.
Load Cload  1

 0.097
Csize( d )  1.189   
d
Size 
 mm 
Surface A  4.51 b'  0.265 (machined)

b'
 Sut 
Csurf  A    Csurf  0.764
 MPa 
Temperature Ctemp  1

Reliability Creliab  0.897 (R = 90%)

5. Determine the modified endurance limit as a function of the unknown diameter, d.


S e( d )  Cload  Csize( d )  Csurf  Ctemp Creliab S'e

6. Determine the geometric stress concentration factors from Figure 10-16 using curve-fit equations.
 0.4115
Bending Kt  0.4521 ( roverd) Kt  2.2
 0.230
Torsion Kts  1.251  ( roverd) Kts  3.0
© 2011 Pearson Education, Inc., Upper Saddle River, NJ. All rights reserved. This publication is protected by Copyright and written permission should be
MACHINE DESIGN - An Integrated Approach, 4th Ed. 10-34a-2

7. Determine the notch sensitivity of the material for bending and torsion. Note from Figure 6-36, part 1, that a
value of 20 ksi should be added to S ut to obtain a 1/2 from Table 6-6 for the torsional Neuber constant.

Bending Lookup value of S ut S ut  118  ksi

   3.10030  10 7  S    in


2 3
aN ( S )  8.69657  10  
2 3 S 5 S
 2.75956  10   3.94116  10
  ksi 
 ksi  ksi    
 4 5 6 
 1.38322  10     3.28018  10     3.21209  10   
9 S  12 S  15 S

  ksi   ksi   ksi  

a N  aN  S ut
0.5
Neuber constant a N  0.050  in

1
Notch sensitivity q ( d ) 
aN
1
roverd d

Torsion Lookup value of S ut S'ut  S ut  20 ksi S'ut  138  ksi

a N  aN  S ut  20 ksi
0.5
Neuber constant a N  0.040  in

1
Notch sensitivity q s( d ) 
aN
1
roverd d

8. Determine the fatigue stress concentration factors from equation 6.11b.

Kf ( d )  1  q ( d )   Kt  1  Kfm( d )  Kf ( d )

Kfs( d )  1  q ( d )   Kts  1  Kfsm( d )  Kfs( d )

9. Use equation 10.6a as a design equation to find d.


Guess d  1.00 in
1
3
 1
 2
Given
 32 Nfd  Kf ( d )  Ma  2 3  Kfsm( d)  Tmax  2 
d=     4   

 π  S e( d )   Sy  

d 1  Find ( d ) d 1  55.0 mm

APPENDIX - Maximum bending moment


1. From inspection of Figure P10-2, write the load function equation
q(x) = R1<x - 0>-1 - p<x - a>0 + p<x - b>0 + R2<x - L>-1
© 2011 Pearson Education, Inc., Upper Saddle River, NJ. All rights reserved. This publication is protected by Copyright and written permission should be
MACHINE DESIGN - An Integrated Approach, 4th Ed. 10-34a-3

2. Integrate this equation from - to x to obtain shear, V(x)


V(x) = R1<x - 0>0 - p<x - a>1 + p<x - b>1 + R2<x - L>0

3. Integrate this equation from - to x to obtain moment, M(x)


M(x) = R1<x - 0>1 - p<x - a>2/2 + p<x - b>2/2 + R2<x - L>1

4. Solve for the reactions by evaluating the shear and moment equations at a point just to the right of x = L,
where both are zero.
At x = L+, V = M = 0
V = R1  p  ( L  a )  p  ( L  b )  R2 = 0
p 2 p 2
M = R 1 L   ( L  a)   ( L  b) = 0
2 2

 ( L  a )  ( L  b )
p 2 2
R1   R1  300.000  N
2 L
R2  p  ( b  a )  R1 R2  1700 N

5. Define the range for x x  0  m 0.005  L  L


6. For a Mathcad solution, define a step function S. This function will have a value of zero when x is less
than z, and a value of one when it is greater than or equal to z.

S ( x z)  if ( x  z 1 0 )


7. Write the shear and moment equations in Mathcad form, using the function S as a multiplying factor to get the
effect of the singularity functions.

V ( x)  R1 S ( x 0  m)  p  S ( x a )  ( x  a )  p  S ( x b )  ( x  b )  R2 S ( x L)


p 2 p 2
M ( x)  R1 S ( x 0  m)  x   S ( x a )  ( x  a )   S ( x b )  ( x  b )
2 2
8. Plot the shear and moment diagrams.

Shear Diagram Moment Diagram


500 50

0 40

30
V ( x)  500 M ( x)
N 20
 1000 Nm
10
 1500
0
 2000
0 5 10 15 20  10
0 5 10 15 20
x
x
cm
cm
FIGURE 10-34a
Shear and Moment Diagrams for Problem 10-34a

© 2011 Pearson Education, Inc., Upper Saddle River, NJ. All rights reserved. This publication is protected by Copyright and written permission should be
MACHINE DESIGN - An Integrated Approach, 4th Ed. 10-34a-4

9. Determine the maximum maximum moment from inspection of the diagrams. Maximum moment occurs where
zero, which is x = c. From the shear diagram,

ca bc a  R2  b  R1
= c  c  16.300 cm
R1 R2 R1  R2

Mmax  M ( c) Mmax  48.450 N  m

© 2011 Pearson Education, Inc., Upper Saddle River, NJ. All rights reserved. This publication is protected by Copyright and written permission should be
MACHINE DESIGN - An Integrated Approach, 4th Ed. 10-35-1

PROBLEM 10-35
Statement: Figure P10-8 shows the last stage in a gearbox with dual output. The gear is manufactured
integrally with the shaft. The shaft is supported by two self-aligning ball bearings. Crank arms
are connected to each end of the shaft. The load on the cranks produces equal fluctuating
transverse forces on the shaft ends as well as equal fluctuating torques. The torque is
transmitted through end-milled keyways in the crank and shaft and a parallel key that fits snugly
in each keyway. The crank is located axially by a shoulder that is 50-mm from the plane in which
the transverse load acts. The fillet radius to shaft diameter is r/d = 0.05 and the shoulder to shaft
diameter ratio is D/d = 1.2. The shaft/gear material is SAE 4130 steel Q&T @ 1200F. The
transverse force fluctuates from 8 kN to 16.5 kN and the torque fluctuates from 1.1 kN-m to 2.2
kN-m. For a factor of safety of 2.5 against an infinite-life fatigue failure, determine a suitable
shaft diameter, d.

Given: Tensile strength S ut  814  MPa Ratios: Doverd  1.2 roverd  0.05
Loads: Pmin  8  kN Pmax  16.5 kN Tmin  1.1 kN  m Tmax  2.2 kN  m
Distance from center of crank to shoulder l  50 mm Safety factor Nf  2.5

Solution: See Figure P10-8 and Mathcad file P1035.

1. Calculate the mean and alternating components of the loads.

Pmin  Pmax
Mean Pm  Pm  12.3 kN
2
Tmin  Tmax
Tm  Tm  1.65 kN  m
2
Pmax  Pmin
Alternating Pa  Pa  4.3 kN
2
Tmax  Tmin
Ta  Ta  0.55 kN  m
2

2. Calculate the mean and alternating bending moment at the shoulder.


Mean Mm  Pm l Mm  0.61 kN  m

Alternating Ma  Pa l Ma  0.21 kN  m

3. Using Appendix C, determine the geometric stress concentration factors for the bending and torsional stresses
at the shoulder.

Bending (Fig. C-2): For Doverd  1.2 roverd  0.05 A  0.97098 b  0.21796
b
Kt  A  ( roverd) Kt  1.865

Torsion (Fig. C-3): A  0.83425 b  0.21649

b
Kts  A  ( roverd) Kts  1.596

4. Calculate the notch sensitivity of the material for bending and torsion using Table 6-6. Assume a value for the
notch radius and iterate the solution if necessary. Assume d  50 mm r  roverd d r  2.500  mm

© 2011 Pearson Education, Inc., Upper Saddle River, NJ. All rights reserved. This publication is protected by Copyright and written permission should be
MACHINE DESIGN - An Integrated Approach, 4th Ed. 10-35-2

Bending:
2
Neuber constant (for S ut  814  MPa) a  0.050  in
1
Notch sensitivity q b  q b  0.863
a
1
r
Torsion:
2
Neuber constant (for S ut  20 MPa  834  MPa) a  0.040  in
1
Notch sensitivity q s  q s  0.887
a
1
r
5. Calculate the fatigue stress concentration factors for bending and torsion using equation 6.11b.

Bending Kf  1  q b  Kt  1  Kf  1.746

Torsion Kfs  1  q s  Kts  1  Kfs  1.528

6. Assume that the stresses are such that the conditions of equation 6.17 will result in

Kfm  Kf and Kfsm  Kfs

7. Calculate the unmodified endurance limit (equation 6.5a). S'e  0.5 S ut S'e  407  MPa

8. Calculate the endurance limit modification factors for a rotating round beam.
Load Cload  1 Combined bending and torsion

 0.097
Csize( d )  1.189    d
Size 
 
mm

Surface A  4.51 b  0.265 (machined)

b
 Sut 
Csurf  A    Csurf  0.764
 MPa 
Temperature Ctemp  1

Reliability Creliab  1.0 (R = 50%)


9. Calculate the modified endurance limit.
S e( d )  Cload  Csize( d )  Csurf  Ctemp Creliab S'e

10. Use equation (10.8) to solve for the diameter at the shoulder.

Given
1
3
  2
 32 Nf   Kf  Ma  4   Kfs Ta Kfm Mm   Kfsm Tm
2 3 2 2 3
 
4
d=   
 π  S e( d ) S ut 
© 2011 Pearson Education, Inc., Upper Saddle River, NJ. All rights reserved. This publication is protected by Copyright and written permission should be
MACHINE DESIGN - An Integrated Approach, 4th Ed. 10-35-3

d  Find ( d ) d  54.177 mm Say d  56 mm

10. Calculate the minimum, maximum, mean, and alternating bending stresses to check the assumption regarding th
mean stress concentration factors.

32 Pmin l
σxmin  σxmin  23.2 MPa
3
π d
32 Pmax l
σxmax  σxmax  47.851 MPa
3
π d

32 Pm l
σxm  σxm  35.526 MPa
3
π d
32 Pa l
σxa  σxa  12.325 MPa
3
π d

Yield strength SAE 4130 Q&T @ 1200F S y  703  MPa

Evaluate Kfm  S 1  Kf  σxmax


S 2  Kf  σxmax  σxmin
return Kf if S 1  S y
S y  Kf  σxa
return if  S 1  S y  S 2  2  S y
σxm
0 otherwise

Kfm  1.746 Kf  1.746

Which checks with the assumption made in step 6 above.

11. DESIGN SUMMARY

Shaft diameter d  56 mm

Shoulder dia D  Doverd d D  67.2 mm

Fillet radius r  roverd d r  2.8 mm

© 2011 Pearson Education, Inc., Upper Saddle River, NJ. All rights reserved. This publication is protected by Copyright and written permission should be
MACHINE DESIGN - An Integrated Approach, 4th Ed. 10-36-1

PROBLEM 10-36
Statement: Determine the size of key necessary to give a safety factor of at least 2 against both shear and
bearing failure for the crank/shaft connection of Problem 10-35. Assume a shaft diameter of 58
mm and a key made from SAE 1040 CR steel.

Given: Shaft diameter d  58 mm Shaft properties: S ut1  814  MPa


Maximum torque Tmax  2.2 kN  m S y1  703  MPa
Minimum torque Tmin  1.1 kN  m Key properties: S ut2  586  MPa
Design safety factor Nd  2 S y2  490  MPa
Assumptions: The finish is machined, reliability is 50%, and the shaft is at room temperature.

Solution: See Figure P10-8 and Mathcad file P1036.

1. As recommended in Table 10-2, for a shaft diameter of d  58 mm


Key width w  16 mm

2. Determine the alternating and mean key shear force components.

Mean and alternating 1 Tmax  Tmin


force components Fa   Fa  18.97  kN
2 0.5 d

1 Tmax  Tmin
Fm   Fm  56.90  kN
2 0.5 d
3. Write the equations for the mean and alternating components of the shear stress and the effective von Mises
stress.
Key shear area as
function of length Ashear = w L

Mean and alternating Fa Fm


shear stresses τa = τm =
w L w L

Mean and alternating Fa Fm


von Mises stresses σ'a = 3  τa = 3  σ'm = 3  τm = 3 
w L w L

4. Using the modified Goodman failure criterion, the design equation is

S e S ut S e S ut
Nd = =
S ut σ'a  S e σ'm  3 
   S ut Fa  S e Fm
 w L 

3  Nd   S ut Fa  S e Fm
Solving for L L=
w S ut S e

5. Determine the key material endurance limit.


Uncorrected endurance
strength S'e  0.5 S ut2 S'e  293  MPa

© 2011 Pearson Education, Inc., Upper Saddle River, NJ. All rights reserved. This publication is protected by Copyright and written permission should be
MACHINE DESIGN - An Integrated Approach, 4th Ed. 10-36-2

Correction factors:
Load Cload  1
w L
Size A95 = w L d eq =
0.0766

 0.097
Csize = 1.189  d eq

 0.5 0.097
Cs  1.189   
w
 Cs  0.918
 0.0766 mm 
 0.0485
Csize = 0.805  L
 0.265
 S ut2 
Surface (machined) Csurf  4.51   Csurf  0.833
 MPa 
Temperature Ctemp  1

Reliability Creliab  1.000 (R = 50%)

 0.0485  0.0485
S e = Cload  Csize Csurf  Ctemp Creliab S'e = 1  Cs L  Csurf  1  Crelaib S'e = Ce L

where Ce  Cs Csurf  Creliab S'e Ce  224  MPa

6. Substitute S e into the design equation.

3  Nd   S ut Fa  Ce L  Fm


 0.0485
L=
 0.0485
w S ut Ce L

Solving by iteration, let

  0.0485 
3  Nd  S ut2 Fa  Ce 
L
  Fm
RHS( L) 
  in  
 0.0485
w S ut2 Ce 
L

 in 
Guess L  25 mm RHS( L)  39.339 mm
L  RHS( L) RHS( L)  39.747 mm

Tentatively, let L  40 mm

7. Check the realised factor of safety against fatigue failure.

 0.0485
S e  Ce 
L
Endurance limit  S e  219.1  MPa
 in 
© 2011 Pearson Education, Inc., Upper Saddle River, NJ. All rights reserved. This publication is protected by Copyright and written permission should be
MACHINE DESIGN - An Integrated Approach, 4th Ed. 10-36-3

S e S ut2
Realised factor Nf  Nf  2.0
 3 
  S ut2 Fa  Se Fm
of safety

 w L 
8. Check worst-case static bearing stress.
1 2
Bearing area Abear   w L Abear  320  mm
2

Maximum force Fmax  Fa  Fm Fmax  75.9 kN

Maximum bearing stress Fmax


on key σmax  σmax  237.07 MPa
Abear

Factor of safety against S y2


static bearing yield Ns  Ns  2.1
σmax

9. Design Summary

Key width w  16 mm (16 x 10)


Key length L  40 mm

© 2011 Pearson Education, Inc., Upper Saddle River, NJ. All rights reserved. This publication is protected by Copyright and written permission should be
MACHINE DESIGN - An Integrated Approach, 4th Ed. 10-37-1
PROBLEM 10-37
Statement: As an alternative to the keyed connection in Problem 10-35 determine the amount of diametral
interference needed to provide a suitable interference fit for the crank of Figure P10-8 using a
shaft diameter of 58 mm, such that the stresses in the hub and shaft will be safe and the
maximum torque can be transmitted through the interference fit. The crank material is the same
as the shaft and its length along the shaft is 64 mm. The effective outside diameter of the crank
is 150 mm.
Given: Gear hub diameter d hub  150  mm Shaft diameter: d shaft  58 mm
Gear hub length L  64 mm Young's modulus E  209  GPa
Peak shaft torque Tp  2.2 kN  m Poisson's ratio ν  0.28
Material properties are: S ut  814  MPa, and S y  703  MPa.

Assumptions: The coefficient of friction between the hub and shaft is μ  0.15.
Solution: See Figure P10-8 and Mathcad file P1037.
1. As a design choice, let the design factor of safety on torque capacity and hub failure be Nd  2
2. Calculate hub and shaft radii.
Hub radius ro  0.5 d hub ro  75 mm
Nominal interface radius r  0.5 d shaft r  29 mm
Shaft inside diameter ri  0  mm

3. The minimum interference is determine by the desired torque capacity. Use equation 10.14c to solve for the
minimum diametral interference.

π L r μ  δmin E  ro  r 
2 2
Torque capacity T= 
Nd 2
2  ro
2
2  ro  Nd  Tp
Solving for min δmin  δmin  0.057  mm
π L r μ  E  ro  r 
2 2

Let the minimum diametral interference be δmin  0.06 mm

4. Find the von Mises stress in the hub as a function of the unknown maximum diametral interference .

δ E  ro  r 
2 2
Interference pressure p ( δ) 
2
4  r ro
Stress in shaft
Tangential σti ( δ)  p ( δ)
Radial σri( δ)  p ( δ)
Stress in hub
2 2
ro  r
Tangential σto( δ)  p ( δ) 
2 2
ro  r

Radial σro( δ)  p ( δ)
These are principal stresses. The tangential stress is σ1 and the radial is σ3. Use equation (5.7c) to find the von
Mises stress.

© 2011 Pearson Education, Inc., Upper Saddle River, NJ. All rights reserved. This publication is protected by Copyright and written permission should be
MACHINE DESIGN - An Integrated Approach, 4th Ed. 10-37-2

0.5
δ E
  3  ro  r
4 4
von Mises σ'o( δ)  
2
4  r ro

5. There is no bending stress in the shaft at the crank, therefore Kt  1


6. The safety factors against failure (yielding in the shaft and hub) during press fit can now be used to find the
maximum diametral interference:
Guess δ  δmin

S y
Shaft Given Nd =
Kt σti ( δ)

δsmax  Find ( δ) δsmax  0.23 mm

Sy
Hub Given Nd =
Kt σ'o( δ)

δhmax  Find( δ) δhmax  0.11 mm

Using the smaller of the two, δmax  δhmax δmax  0.11 mm

7. The torque capacity of the joint (with an assumed coefficient of friction of μ  0.15 ) is about 2 times the
peak transmitted torque and the safety factors against failure of the shaft and hub exceed 2 therefore, the
minimum and maximum diametral interference below is acceptable.
Minimum diametral interference δmin  0.06 mm
Minimum diametral interference δmax  0.12 mm
8. If we divide the tolerance on the shaft and hub equally and use the basic hole system, the shaft and hub bore
specifications are:

Tolerance on shaft or hub t  0.5  δmax  δmin t  0.03 mm

Hub bore diameter: Dmin  d shaft Dmin  58.00  mm

Dmax  Dmin  t Dmax  58.03  mm

Shaft diameter: d max  d shaft  δmax d max  58.12  mm

d min  d max  t d min  58.09  mm

Maximum interference δmax  d max  Dmin δmax  0.12 mm

Minimum interference δmin  d min  Dmax δmin  0.06 mm

9. Check the safety factors against torque capacity and static failure of the hub.

π L r μ  δmin E  ro  r 
2 2
Torque Ntorque   Ntorque  2.1
Tp 2
2  ro
Sy
Hub strength Nhub  Nhub  1.9
Kt σ'o δmax
© 2011 Pearson Education, Inc., Upper Saddle River, NJ. All rights reserved. This publication is protected by Copyright and written permission should be
MACHINE DESIGN - An Integrated Approach, 4th Ed. 10-38-1

PROBLEM 10-38
Statement: An electric motor-driven shaft that has a fluctuating load torque turns at an average speed of
1600 rpm. The load torque varies sinusoidally once per revolution of the shaft with a peak
torque of 29500 in-lb. The shaft diameter is 2.0 in. Design a suitable flywheel for this system
that will provide a coefficient of fluctuation of 0.05 and an overspeed safety factor of at least 4.
The flywheel is to be a hollow circular disc of constant thickness made from SAE 1020 CR steel.

3
Given: Yield strength of SAE 1020 CR steel S y  57 ksi ν  0.28 γ  0.28 lbf  in
Average shaft speed ωavg  1600 rpm Coeff. of fluct. Cf  0.05
Shaft diameter d i  2.000  in Peak torque Tpeak  29500  in lbf

Solution: See Mathcad file P1038.

1. Plot the load torque over one revolution of the shaft.

Tl( θ )  Tpeak  sin( θ ) θ  0 0.1  2  π

4
4 10
4
2 10
Load Torque

T l( θ)
0
in lbf
4
 2 10
4
 4 10
0 60 120 180 240 300 360
θ
deg
Shaft Position

The average load torque is Tavg  0  in lbf

The minimum shaft speed occurs at θmin  180  deg

The maximum shaft speed occurs at θmax  360  deg

2. Calculate the required change in kinetic energy by integrating the left-hand side of equation 10.18d.

θ
 max
Energy variation Ek   Tl( θ ) dθ Ek  59000  in lbf
θ
min
3. Calculate the required system moment of inertia using equation 10.22.

Ek 2
System moment Is  Is  42.032 lbf  in sec
of inertia 2
Cf  ωavg

Let Im  Is
4. As a design choice, let the disc thickness be t  0.4 d i , t  0.800  in (this value of t was repeatedly changed
to arrive at a value of 4.0 for Nos). Calculate the outside radius required for this thickness using equation
10.17d.
© 2011 Pearson Education, Inc., Upper Saddle River, NJ. All rights reserved. This publication is protected by Copyright and written permission should be
MACHINE DESIGN - An Integrated Approach, 4th Ed. 10-38-2

0.25
 2  Im g  d i  4
Outside radius ro     ro  14.655 in
 π γ  t  2  

5. Let the maximum tangential stress (at r = ri) be equal to the yield stress and solve equation 10.23a for the yield
speed.

0.5
 S y g 
Yield speed ωyield   
  3  ν   di  2 2
1  3 ν   i   
    2  ro  
2 d
 γ      
  8   2   3  ν   2  

ωyield  6376 rpm

ωyield
Factor of safety against overspeeding Nos  Nos  4.0
ωavg

6. DESIGN SUMMARY

Inside radius ri  0.5 d i ri  1  in

Outside radius ro  14.655 in

Tickness t  0.800  in

© 2011 Pearson Education, Inc., Upper Saddle River, NJ. All rights reserved. This publication is protected by Copyright and written permission should be
MACHINE DESIGN - An Integrated Approach, 4th Ed. 10-39-1

PROBLEM 10-39
Statement: Repeat Problem 10-38 for a peak load torque of 40500 in-lb and a torque vs shaft position
function of Tl = Tpeak (sin  + sin 2).

3
Given: Yield strength of SAE 1020 CR steel S y  57 ksi ν  0.28 γ  0.28 lbf  in
Average shaft speed ωavg  1600 rpm Coeff. of fluct. Cf  0.05
Shaft diameter d i  2.000  in Peak torque Tpeak  40500  in lbf

Solution: See Mathcad file P1039.


1. Plot the load torque over one revolution of the shaft.

Tl( θ )  Tpeak  ( sin( θ )  sin( 2  θ ) ) θ  0 0.1  2  π

5
1 10
4
5 10
Load Torque

T l( θ)
0
in lbf
4
 5 10
5
 1 10
0 60 120 180 240 300 360
θ
deg
Shaft Position

The average load torque is Tavg  0  in lbf

The minimum shaft speed occurs at θmin  120  deg (and again at 240 deg)

The maximum shaft speed occurs at θmax  360  deg

2. Calculate the required change in kinetic energy by integrating the left-hand side of equation 10.18d.

θ
 max
Energy variation Ek   Tl( θ ) dθ Ek  91125  in lbf
θ
min

3. Calculate the required system moment of inertia using equation 10.22.

Ek 2
System moment Is  Is  64.919 lbf  in sec
of inertia 2
Cf  ωavg

Let Im  Is

4. As a design choice, let the disc thickness be t  0.6 d i , t  1.200  in (this value of t was repeatedly changed to
arrive at a value of 4.0 for Nos). Calculate the outside radius required for this thickness using equation 10.17d.
0.25
 2  I  g  d  4
ro    
m i
Outside radius ro  14.762 in
 π γ  t  2  
© 2011 Pearson Education, Inc., Upper Saddle River, NJ. All rights reserved. This publication is protected by Copyright and written permission should be
MACHINE DESIGN - An Integrated Approach, 4th Ed. 10-39-2

5. Let the maximum tangential stress (at r = ri) be equal to the yield stress and solve equation 10.23a for the yield
speed.

0.5
 S y g 
Yield speed ωyield   
 3   2
2  1  3 ν   di   
2
ν   d i 
 γ  
 2    2  ro      
  8     3  ν   2  
ωyield  6330 rpm

ωyield
Factor of safety against overspeeding Nos  Nos  4.0
ωavg

6. DESIGN SUMMARY

Inside radius ri  0.5 d i ri  1.000  in

Outside radius ro  14.762 in

Tickness t  1.200  in

© 2011 Pearson Education, Inc., Upper Saddle River, NJ. All rights reserved. This publication is protected by Copyright and written permission should be
MACHINE DESIGN - An Integrated Approach, 4th Ed. 10-40a-1
PROBLEM 10-40a
Statement: As an alternative to the keyed connection in Problem 10-6 determine the amount of diametral
interference needed to provide a suitable interference fit for the gear of Figure P10-3 using the
data of Problem 10-6 and row a in Table P10-1. The gear hub diameter is 3.50 in, its length along
the shaft is 2.00 in, and it has the same properties as the shaft material.
Given: Gear hub diameter d hub  3.50 in Shaft diameter: d shaft  1.75 in
6
Gear hub length L  2.00 in Young's modulus E  30 10  psi
Peak shaft torque Tp  2000 lbf  in Poisson's ratio ν  0.28
Material properties are: S ut  108  ksi, and S y  62 ksi.

Assumptions: The coefficient of friction between the hub and shaft is μ  0.15.
Solution: See Figure P10-3 and Mathcad file P1040a.
1. As a design choice, let the design factor of safety on torque capacity and hub failure be Nd  2
2. Calculate hub and shaft radii.
Hub radius ro  0.5 d hub ro  1.75 in
Nominal interface radius r  0.5 d shaft r  0.875  in
Shaft inside diameter ri  0  in
3. The minimum interference is determine by the desired torque capacity. Use equation 10.14c to solve for the
minimum diametral interference.

π L r μ  δmin E  ro  r 
2 2
Torque capacity T= 
Nd 2
2  ro
2
2  ro  Nd  Tp
Solving for min δmin  δmin  0.00043  in
π L r μ  E  ro  r 
2 2

Let the minimum diametral interference be δmin  0.0005 in

Note that it is not practical to specify a dimension in inches that has more than 4 decimal places.
4. Find the von Mises stress in the hub as a function of the unknown maximum diametral interference .

δ E  ro  r 
2 2
Interference pressure p ( δ) 
2
4  r ro
Stress in shaft
Tangential σti ( δ)  p ( δ)

Radial σri( δ)  p ( δ)

Stress in hub
2 2
ro  r
Tangential σto( δ)  p ( δ) 
2 2
ro  r

Radial σro( δ)  p ( δ)
These are principal stresses. The tangential stress is σ1 and the radial is σ3. Use equation (5.7c) to find the von
Mises stress.
© 2011 Pearson Education, Inc., Upper Saddle River, NJ. All rights reserved. This publication is protected by Copyright and written permission should be
MACHINE DESIGN - An Integrated Approach, 4th Ed. 10-40a-2

0.5
δ E
  3  ro  r
4 4
von Mises σ'o( δ)  
2
4  r ro

5. There is no bending stress in the shaft at the gear, therefore Kt  1


6. The safety factors against failure (yielding in the shaft and hub) during press fit can now be used to find the
maximum diametral interference:
Guess δ  δmin

S y
Shaft Given Nd =
Kt σti ( δ)

δsmax  Find ( δ) δsmax  0.0048 in

Sy
Hub Given Nd =
Kt σ'o( δ)

δhmax  Find( δ) δhmax  0.0021 in

Using the smaller of the two, δmax  δhmax δmax  0.0021 in

7. The torque capacity of the joint (with an assumed coefficient of friction of μ  0.15 ) is about 2 times the
peak transmitted torque and the safety factors against failure of the shaft and hub exceed 2 therefore, the
minimum and maximum diametral interference below is acceptable.
Minimum diametral interference δmin  0.0005 in

Minimum diametral interference δmax  0.0021 in


8. If we divide the tolerance on the shaft and hub equally and use the basic hole system, the shaft and hub bore
specifications are:

Tolerance on shaft or hub t  0.5  δmax  δmin t  0.0008 in

Hub bore diameter: Dmin  d shaft Dmin  1.7500 in

Dmax  Dmin  t Dmax  1.7508 in

Shaft diameter: d max  d shaft  δmax d max  1.7521 in

d min  d max  t d min  1.7513 in

Maximum interference δmax  d max  Dmin δmax  0.0021 in

Minimum interference δmin  d min  Dmax δmin  0.0005 in

9. Check the safety factors against torque capacity and static failure of the hub.

π L r μ  δmin E  ro  r 
2 2
Torque Ntorque   Ntorque  2.3
Tp 2
2  ro
Sy
Hub strength Nhub  Nhub  2.0
Kt σ'o δmax
© 2011 Pearson Education, Inc., Upper Saddle River, NJ. All rights reserved. This publication is protected by Copyright and written permission should be
MACHINE DESIGN - An Integrated Approach, 4th Ed. 10-41-1
PROBLEM 10-41
Statement: Figure P10-6 shows a shaft, running at 400 rpm, with two attached gears. The right-hand gear (2)
furnishes the load torque and the left-hand gear (1) furnishes an equal but opposite direction
input torque. The torque varies between 2.2 kN-m and 6.2 kN-m. Determine the minimum,
maximum, and average power transmitted by the shaft.

rad
Given: Shaft speed ω  400  rpm ω  41.888
sec
Minimum torque Tmin  2.2 kN  m
Maximum torque Tmax  6.2 kN  m

Solution: See Figure P10-6 and Mathcad file P1041.

1. Use equations (10.1) to calculate the power values.

Tmax  Tmin
Average torque Tavg  Tavg  4  kN  m
2

Maximum power Pmax  Tmax ω Pmax  259.7  kW

Minimum power Pmin  Tmin ω Pmin  92 kW

Average power Pavg  Tavg ω Pavg  175.9  kW

2. Note, if doing these calculations by hand, that suitable unit conversion factors must be used.

© 2011 Pearson Education, Inc., Upper Saddle River, NJ. All rights reserved. This publication is protected by Copyright and written permission should be
MACHINE DESIGN - An Integrated Approach, 4th Ed. 10-42-1
PROBLEM 10-42
Statement: Figure P10-6 shows a shaft, running at 750 rpm, with two attached gears. The right-hand gear (2)
furnishes the load torque and the left-hand gear (1) furnishes an equal but opposite direction
input torque. The forces on a gear act at the pitch diameter and have a radial (P in the figure)
and a tangential component (not shown). If the pitch diameter of gear 1 is 250 mm and the
tangential force varies between 15 kN and 60 kN, determine the minimum, maximum, and average
power transmitted by the shaft.

rad
Given: Shaft speed ω  750  rpm ω  78.54 
sec
Pitch diameter d  250  mm
Minimum tangential force Fmin  15 kN
Maximum tangential force Fmax  60 kN

Solution: See Figure P10-6 and Mathcad file P1042.


1. Determine the minimum and maximum torque.
Minimum torque Tmin  0.5 d  Fmin Tmin  1.875  kN  m
Maximum torque Tmax  0.5 d  Fmax Tmax  7.5 kN  m

1. Use equations (10.1) to calculate the power values.

Tmax  Tmin
Average torque Tavg  Tavg  5  kN  m
2

Maximum power Pmax  Tmax ω Pmax  589.0  kW

Minimum power Pmin  Tmin ω Pmin  147  kW

Average power Pavg  Tavg ω Pavg  368.2  kW

2. Note, if doing these calculations by hand, that suitable unit conversion factors must be used.

© 2011 Pearson Education, Inc., Upper Saddle River, NJ. All rights reserved. This publication is protected by Copyright and written permission should be
MACHINE DESIGN - An Integrated Approach, 4th Ed. 10-43-1
PROBLEM 10-43
Statement: Determine the amount of diametral interference needed to provide a suitable interference fit
(instead of the keyed attachment shown) for the 5-in diameter by 1.5-in thick gear (1) of Figure
P10-6 using a shaft diameter of 1.5 in, such that the stresses in the hub and shaft will be safe and
the input torque of 1500 lbf-in can be transmitted through the interference fit. Both parts are SAE
4130 steel normalized @ 1650F.

Given: Gear hub diameter d hub  5.00 in Shaft diameter: d shaft  1.50 in
6
Gear hub length L  1.50 in Young's modulus E  30 10  psi
Peak shaft torque Tp  1500 lbf  in Poisson's ratio ν  0.28
Material properties are: S ut  97 ksi S y  63 ksi

Assumptions: The coefficient of friction between the hub and shaft is μ  0.15.
Solution: See Figure P10-6 and Mathcad file P1043.

1. As a design choice, let the design factor of safety on torque capacity and hub failure be Nd  2
2. Calculate hub and shaft radii.
Hub radius ro  0.5 d hub ro  2.5 in
Nominal interface radius r  0.5 d shaft r  0.75 in
Shaft inside diameter ri  0  in

3. The minimum interference is determine by the desired torque capacity. Use equation 10.14c to solve for the
minimum diametral interference.

π L r μ  δmin E  ro  r 
2 2
Torque capacity T= 
Nd 2
2  ro
2
2  ro  Nd  Tp
Solving for min δmin  δmin  0.00041  in
π L r μ  E  ro  r 
2 2

Let the minimum diametral interference be δmin  0.0004 in

Note that it is not practical to specify a dimension in inches that has more than 4 decimal places.

4. Find the von Mises stress in the hub as a function of the unknown maximum diametral interference .

δ E  ro  r 
2 2
Interference pressure p ( δ) 
2
4  r ro
Stress in shaft
Tangential σti ( δ)  p ( δ)

Radial σri( δ)  p ( δ)
Stress in hub
2 2
ro  r
Tangential σto( δ)  p ( δ) 
2 2
ro  r
Radial σro( δ)  p ( δ)

© 2011 Pearson Education, Inc., Upper Saddle River, NJ. All rights reserved. This publication is protected by Copyright and written permission should be
MACHINE DESIGN - An Integrated Approach, 4th Ed. 10-43-2

These are principal stresses. The tangential stress is σ1 and the radial is σ3. Use equation (5.7c) to find the von
Mises stress.
0.5
δ E
  3  ro  r
4 4
von Mises σ'o( δ)  
2
4  r ro

5. There is very little bending stress in the shaft at the gear, therefore Kt  1
6. The safety factors against failure (yielding in the shaft and hub) during press fit can now be used to find the
maximum diametral interference:
Guess δ  δmin

S y
Shaft Given Nd =
Kt σti ( δ)

δsmax  Find ( δ) δsmax  0.0035 in

Sy
Hub Given Nd =
Kt σ'o( δ)

δhmax  Find( δ) δhmax  0.0018 in

Using the smaller of the two, δmax  δhmax δmax  0.0018 in

7. The torque capacity of the joint (with an assumed coefficient of friction of μ  0.15 ) is about 2 times the
peak transmitted torque and the safety factors against failure of the shaft and hub exceed 2 therefore, the
minimum and maximum diametral interference below is acceptable.
Minimum diametral interference δmin  0.0004 in
Maximum diametral interference δmax  0.0018 in
8. If we divide the tolerance on the shaft and hub equally and use the basic hole system, the shaft and hub bore
specifications are:
Tolerance on shaft or hub t  0.5  δmax  δmin t  0.0007 in

Hub bore diameter: Dmin  d shaft Dmin  1.5000 in

Dmax  Dmin  t Dmax  1.5007 in

Shaft diameter: d max  d shaft  δmax d max  1.5018 in

d min  d max  t d min  1.5011 in

Maximum interference δmax  d max  Dmin δmax  0.0018 in

Minimum interference δmin  d min  Dmax δmin  0.0004 in

9. Check the safety factors against torque capacity and static failure of the hub.

π L r μ  δmin E  ro  r 
2 2
Torque Ntorque   Ntorque  1.9
Tp 2
2  ro
Sy
Hub strength Nhub  Nhub  2.0
Kt σ'o δmax
© 2011 Pearson Education, Inc., Upper Saddle River, NJ. All rights reserved. This publication is protected by Copyright and written permission should be
MACHINE DESIGN - An Integrated Approach, 4th Ed. 10-44-1
PROBLEM 10-44
Statement: Determine the amount of diametral interference needed to provide a suitable interference fit
(instead of the keyed attachment shown) for the 125-mm diameter by 75-mm thick gear (1) of
Figure P10-6 using a shaft diameter of 80 mm, such that the stresses in the hub and shaft will be
safe and the input torque of 170 N-m can be transmitted through the interference fit. Both parts
are SAE 4140 steel normalized @ 1650F.

Given: Gear hub diameter d hub  125  mm Shaft diameter: d shaft  80 mm
Gear hub length L  75 mm Young's modulus E  209  GPa
Peak shaft torque Tp  170  N  m Poisson's ratio ν  0.28
Material properties are: S ut  1020 MPa S y  655  MPa

Assumptions: The coefficient of friction between the hub and shaft is μ  0.15.
Solution: See Figure P10-6 and Mathcad file P1044.

1. As a design choice, let the design factor of safety on torque capacity and hub failure be Nd  2
2. Calculate hub and shaft radii.
Hub radius ro  0.5 d hub ro  62.5 mm
Nominal interface radius r  0.5 d shaft r  40 mm
Shaft inside diameter ri  0  mm

3. The minimum interference is determine by the desired torque capacity. Use equation 10.14c to solve for the
minimum diametral interference.

π L r μ  δmin E  ro  r 
2 2
Torque capacity T= 
Nd 2
2  ro
2
2  ro  Nd  Tp
Solving for min δmin  δmin  0.0039 mm
π L r μ  E  ro  r 
2 2

Let the minimum diametral interference be δmin  0.004  mm

Note that it is not practical to specify a dimension in milimeters that has more than 3 decimal places.

4. Find the von Mises stress in the hub as a function of the unknown maximum diametral interference .

δ E  ro  r 
2 2
Interference pressure p ( δ) 
2
4  r ro
Stress in shaft
Tangential σti ( δ)  p ( δ)

Radial σri( δ)  p ( δ)
Stress in hub
2 2
ro  r
Tangential σto( δ)  p ( δ) 
2 2
ro  r
Radial σro( δ)  p ( δ)

© 2011 Pearson Education, Inc., Upper Saddle River, NJ. All rights reserved. This publication is protected by Copyright and written permission should be
MACHINE DESIGN - An Integrated Approach, 4th Ed. 10-44-2

These are principal stresses. The tangential stress is σ1 and the radial is σ3. Use equation (5.7c) to find the von
Mises stress.
0.5
δ E
  3  ro  r
4 4
von Mises σ'o( δ)  
2
4  r ro

5. There is very little bending stress in the shaft at the gear, therefore Kt  1
6. The safety factors against failure (yielding in the shaft and hub) during press fit can now be used to find the
maximum diametral interference:
Guess δ  δmin

S y
Shaft Given Nd =
Kt σti ( δ)

δsmax  Find ( δ) δsmax  0.425  mm

Sy
Hub Given Nd =
Kt σ'o( δ)

δhmax  Find( δ) δhmax  0.141  mm

Using the smaller of the two, δmax  δhmax δmax  0.140  mm

7. The torque capacity of the joint (with an assumed coefficient of friction of μ  0.15 ) is about 2 times the
peak transmitted torque and the safety factors against failure of the shaft and hub exceed 2 therefore, the
minimum and maximum diametral interference below is acceptable.
Minimum diametral interference δmin  0.004  mm
Maximum diametral interference δmax  0.140  mm
8. If we divide the tolerance on the shaft and hub equally and use the basic hole system, the shaft and hub bore
specifications are:
Tolerance on shaft or hub t  0.5  δmax  δmin t  0.0680 mm

Hub bore diameter: Dmin  d shaft Dmin  80.000 mm

Dmax  Dmin  t Dmax  80.068 mm

Shaft diameter: d max  d shaft  δmax d max  80.140 mm

d min  d max  t d min  80.072 mm

Maximum interference δmax  d max  Dmin δmax  0.140  mm

Minimum interference δmin  d min  Dmax δmin  0.004  mm

9. Check the safety factors against torque capacity and static failure of the hub.

π L r μ  δmin E  ro  r 
2 2
Torque Ntorque   Ntorque  2.1
Tp 2
2  ro
Sy
Hub strength Nhub  Nhub  2.0
Kt σ'o δmax
© 2011 Pearson Education, Inc., Upper Saddle River, NJ. All rights reserved. This publication is protected by Copyright and written permission should be
MACHINE DESIGN - An Integrated Approach, 4th Ed. 10-45-1

PROBLEM 10-45
Statement: Table P10-2 shows the energy pulses delivered to (positive) and from (negative) a rotating system
along with the shaft angles at which the torque function crosses the average torque line in a
torque-time function. Using this data, determine the shaft angles at which minimum and maximum
shaft speed occur and the total change in energy from the shaft position at which maximum speed
occurs to the position at which minimum speed occurs.

Given: Range ΔE
θ0  0  deg to θ1  75 deg ∆E1  1040 N  m
θ1  75 deg to θ2  195  deg ∆E2  2260 N  m
θ2  195  deg to θ3  330  deg ∆E3  2950 N  m
θ3  330  deg to θ4  360  deg ∆E4  1740 N  m

Solution: See Table P10-2 and Mathcad file P1045.

1. The minimum shaft speed occurs at the shaft angle at the end of the largest positive energy pulse. In this case,
it is at θ2  195  deg.

2. The maximum shaft speed occurs at the shaft angle at the end of the largest negative energy pulse. In this case,
it is at θ3  330  deg.

3. The total energy change is the algebraic difference between the accumulated energy between the maximum and
minimum shaft speeds.

E1  ∆E1 E1  1040 N  m

E2  E1  ∆E2 E2  1220 N  m

E3  E2  ∆E3 E3  1730 N  m

E4  E3  ∆E4 E4  10 N  m

∆Energy  E3  E2 ∆Energy  2950 N  m

© 2011 Pearson Education, Inc., Upper Saddle River, NJ. All rights reserved. This publication is protected by Copyright and written permission should be
MACHINE DESIGN - An Integrated Approach, 4th Ed. 10-46-1

PROBLEM 10-46
Statement: An electric motor-driven shaft that has a fluctuating load torque turns at an average speed of
1950 rpm. The energy pulses to (positive) and from (negative) the driven system are given in
Table P10-2. The shaft diameter is 50 mm. Design a suitable flywheel for this system that will
provide a coefficient of fluctuation of 0.05 and an overspeed safety factor of at least 5. The
flywheel is to be a hollow circular disc of constant thickness made from SAE 1040 CR steel.

3
Given: Yield strength of SAE 1040 CR steel S y  490  MPa ρ  7800 kg m
Average shaft speed ωavg  1950 rpm Coeff. of fluct. Cf  0.05
Shaft diameter d i  50 mm ν  0.28

Solution: See Table P10-2 and Mathcad file P1046.

1. Calculate the change in kinetic energy from the data in Table P10-2.
The minimum shaft speed occurs at θmin  195  deg
The maximum shaft speed occurs at θmax  330  deg

The change in kinetic energy between the maximum and minimum speeds is Ek  2950 N  m

2. Calculate the required system moment of inertia using equation 10.22.

Ek 2
System moment Is  Is  1.415  kg m
of inertia 2
Cf  ωavg

Let Im  Is

3. As a design choice, let the disc thickness be t  0.4 d i , t  20.0 mm (this value of t was repeatedly changed to
arrive at a value of 5.0 for Nos). Calculate the outside radius required for this thickness using equation 10.17d.

0.25
 2 Im  d i  4
Outside radius ro     ro  276  mm
 π ρ  t  2  

4. Let the maximum tangential stress (at r = ri) be equal to the yield stress and solve equation 10.23a for the yield
speed.

0.5
 Sy 
Yield speed ωyield   
 3   2
2  1  3 ν   i  
2
ν   d i  d
ρ    2   2  ro   3  ν    2   
  8        

ωyield  9580 rpm

ωyield
Factor of safety against overspeeding Nos  Nos  5
ωavg
5. DESIGN SUMMARY

Inside radius ri  0.5 d i ri  25 mm


Outside radius ro  276  mm Tickness t  20 mm
© 2011 Pearson Education, Inc., Upper Saddle River, NJ. All rights reserved. This publication is protected by Copyright and written permission should be
MACHINE DESIGN - An Integrated Approach, 4th Ed. 12-1-1
PROBLEM 12-1
Statement: A 20-deg pressure angle, 27-tooth spur gear has a diametral pitch of 5. Find the pitch diameter,
addendum, dedendum, outside diameter, and circular pitch.
1
Given: Diametral pitch p d  5  in Number of teeth N  27
Pressure angle ϕ  20 deg
Solution: See Mathcad file P1201.

N
Pitch diameter d  d  5.400  in
pd

1.0
Addendum a  a  0.200  in
pd

1.25
Dedendum b  b  0.250  in
pd

Outside diameter Do  d  2  a Do  5.800  in

π
Circular pitch p c  p c  0.628  in
pd

© 2011 Pearson Education, Inc., Upper Saddle River, NJ. All rights reserved. This publication is protected by Copyright and written permission should be
MACHINE DESIGN - An Integrated Approach, 4th Ed. 12-2-1
PROBLEM 12-2
Statement: A 25-deg pressure angle, 43-tooth spur gear has a diametral pitch of 8. Find the pitch diameter,
addendum, dedendum, outside diameter, and circular pitch.
1
Given: Diametral pitch p d  8  in Number of teeth N  43
Pressure angle ϕ  25 deg

Solution: See Mathcad file P1202.

N
Pitch diameter d  d  5.375  in
pd

1.0
Addendum a  a  0.125  in
pd

1.25
Dedendum b  b  0.156  in
pd

Outside diameter Do  d  2  a Do  5.625  in

π
Circular pitch p c  p c  0.393  in
pd

© 2011 Pearson Education, Inc., Upper Saddle River, NJ. All rights reserved. This publication is protected by Copyright and written permission should be
MACHINE DESIGN - An Integrated Approach, 4th Ed. 12-3-1
PROBLEM 12-3
Statement: A 57-tooth spur gear is in mesh with a 23-tooth pinion. For the diametral pitch and pressure angle
given below, find the contact ratio.

Given: Tooth numbers:


1
pinion Np  23 Diametral pitch p d  6  in
gear Ng  57 Pressure angle ϕ  25 deg

Solution: See Mathcad file P1203.

π
Circular pitch p c  p c  0.524  in
pd

Base pitch p b  p c  cos( ϕ) p b  0.475  in

Pinion:
Np
Pitch dia d p  d p  3.833  in
pd

Pitch rad rp  0.5 d p rp  1.917  in

Gear:
Ng
Pitch dia d g  d g  9.500  in
pd

Pitch rad rg  0.5 d g rg  4.750  in

1.0
Addendum a  a  0.167  in
pd

Np  Ng
Center distance C  C  6.667  in
2 pd

Length of action

Z  rp  a 2  rp cos( ϕ)  2   rg  a 2  rg cos( ϕ)  2  C sin( ϕ)


Z  0.708  in

Z
Contact ratio mp  mp  1.491
pb

© 2011 Pearson Education, Inc., Upper Saddle River, NJ. All rights reserved. This publication is protected by Copyright and written permission should be
MACHINE DESIGN - An Integrated Approach, 4th Ed. 12-4-1
PROBLEM 12-4
Statement: A 78-tooth spur gear is in mesh with a 27-tooth pinion. For the diametral pitch and pressure angle
given below, find the contact ratio.

Given: Tooth numbers:


1
pinion Np  27 Diametral pitch p d  6  in
gear Ng  78 Pressure angle ϕ  20 deg

Solution: See Mathcad file P1203.

π
Circular pitch p c  p c  0.524  in
pd

Base pitch p b  p c  cos( ϕ) p b  0.492  in

Pinion:
Np
Pitch dia d p  d p  4.500  in
pd

Pitch rad rp  0.5 d p rp  2.250  in

Gear:
Ng
Pitch dia d g  d g  13.000 in
pd

Pitch rad rg  0.5 d g rg  6.500  in

1.0
Addendum a  a  0.167  in
pd

Np  Ng
Center distance C  C  8.750  in
2 pd

Length of action

Z  rp  a 2  rp cos( ϕ)  2   rg  a 2  rg cos( ϕ)  2  C sin( ϕ)


Z  0.849  in

Z
Contact ratio mp  mp  1.726
pb

© 2011 Pearson Education, Inc., Upper Saddle River, NJ. All rights reserved. This publication is protected by Copyright and written permission should be
MACHINE DESIGN - An 12-5-1
PROBLEM 12-5
Statement: What will the pressure angle be if the center distance of the spur gearset in Problem 12-3 is
increased by 5%?
Given: Tooth numbers: new Base Circle
pinion Np  23 Old Pitch Circle
New Pitch Circle
gear Ng  57
1 rbp
Diametral pitch p d  6  in fc rp
Pressure angle ϕ  25 deg
C + C
Solution: See Figure 12-5 and Mathcad fc r g rbg
file P1205.
1. Define the factor by which the center
distance changes.

C  ∆C
Let = fc
C
Dividing numerator and denominator of the left
side by C,
FIGURE 12-5
∆C
fc = 1  Diagram Showing Center Change for Problem 12-5
C

2. In this case, fc  1  0.05 fc  1.05


3. Determine the pitch diameter and pitch radius of the pinion.

Np
Pitch diameter d p  d p  3.833  in
pd

Pitch radius rp  0.5 d p rp  1.917  in

4. When the centers are moved apart the base circle diameters don't change but new pitch circle diameters are
defined by the intersection of the new line of action, which is always tangent to the two base circles, with the
line of centers. The new pitch radii are proportional to fc. Thus, from Figure 12-5, the new pressure angle is

rbp
cos ϕnew =
fc  rp

Substituting rp cos( ϕ) for rbp and solving for


 rp cos( ϕ) 
ϕnew  acos  ϕnew  30.33  deg
 fc  rp 
new,

© 2011 Pearson Education, Inc., Upper Saddle River, NJ. All rights reserved. This publication is protected by Copyright and written permission should be
MACHINE DESIGN - An 12-6-1
PROBLEM 12-6
Statement: What will the pressure angle be if the center distance of the spur gearset in Problem 12-4 is
increased by 7%?
Given: Tooth numbers: new Base Circle
pinion Np  27 Old Pitch Circle
New Pitch Circle
gear Ng  78
1 rbp
Diametral pitch p d  6  in fc rp
Pressure angle ϕ  20 deg
C + C
Solution: See Figure 12-6 and Mathcad fc r g rbg
file P1206.
1. Define the factor by which the center
distance changes.

C  ∆C
Let = fc
C
Dividing numerator and denominator of the left
side by C,
FIGURE 12-6
∆C
fc = 1  Diagram Showing Center Change for Problem 12-6
C

2. In this case, fc  1  0.07 fc  1.07


3. Determine the pitch diameter and pitch radius of the pinion.

Np
Pitch diameter d p  d p  4.500  in
pd

Pitch radius rp  0.5 d p rp  2.250  in

4. When the centers are moved apart the base circle diameters don't change but new pitch circle diameters are
defined by the intersection of the new line of action, which is always tangent to the two base circles, with the
line of centers. The new pitch radii are proportional to fc. Thus, from Figure 12-5, the new pressure angle is

rbp
cos ϕnew =
fc  rp

Substituting rp cos( ϕ) for rbp and solving for


 rp cos( ϕ) 
ϕnew  acos  ϕnew  28.57  deg
 fc  rp 
new,

© 2011 Pearson Education, Inc., Upper Saddle River, NJ. All rights reserved. This publication is protected by Copyright and written permission should be
MACHINE DESIGN - An Integrated Approach, 4th Ed. 12-7-1

PROBLEM 12-7
Statement: If the spur gearsets in Problems 12-3 and 12-4 are compounded as shown in Figure 12-14, what wil
the overall train ratio be?
Given: Tooth numbers N2  57 N3  23
N4  78 N5  27

Solution: See Mathcad file P1207.

1. Using equation 12.9b and assuming that gears 2 and 4 are the driver gears, the train (velocity) ratio is

N2 N4
mv  mv  7.159
N3 N5

2. On the other hand, if gears 3 and 5 are the driver gears then the train ratio is

N3 N5
mv  mv  0.1397
N2 N4

© 2011 Pearson Education, Inc., Upper Saddle River, NJ. All rights reserved. This publication is protected by Copyright and written permission should be
MACHINE DESIGN - An Integrated Approach, 4th Ed. 12-8-1

PROBLEM 12-8
Statement: A paper mill processes rolls of paper having a density of 984 kg/m3. The paper roll is 1.50-m
outside diameter (OD) x 0.22-m inside diameter (ID) x 3.23-m long and is on a simple supported,
hollow, steel shaft with S ut = 400 MPa. Design a 2.5:1 reduction spur gearset to drive this roll
shaft to obtain a minimum dynamic safety factor of 2 for 10-year life if the roll turns at 50 rpm
with 1.2 hp absorbed.
Units: yr  2080 hr
Given: Bending factor of safety Nfb  2 Life (years) Life  10 yr
Surface factor of safety Nfs  2 Gear speed n g  50 rpm
Power to be transmitted H  1.2 hp Gear ratio mg  2.5
Assumptions: 1. If both pinion and gear are the same material, it will only be necessary to determine the pinion
size as it will be governing for the set.
2. If the gears are not surface hardened, it will only be necessary to design to the surface
requirement as it will be governing for both bending and surface stresses.

Solution: See Mathcad file P1208.


Design Choices:
Pressure angle ϕ  25 deg AGMA Quality level Qv  7
Number of pinion teeth Np  26 Reliability R  0.99
Material: AGMA Grade 2 steel for both pinion and gear, through hardened to
HB  350 S ac  ( 27000  364  HB)  psi S ac  154400 psi
1. Determine the number of gear teeth, the pinion speed, and the cycle life.

Number of gear teeth Ng  Np mg Ng  65

Rotational speed of pinion (rpm) n p  n g mg n p  125  rpm

np 8
Cycle life: N  Life N  1.56  10
2 π
2. Determine the surface geometry factor, I.

 sin( ϕ)  cos( ϕ)   Ng
I    I  0.1368
 2  Np  Ng
3. Write the equations for pitch diameter, pitch-line velocity, and transmitted load in terms of the unknown
diametral pitch, pd. Note that, in Mathcad, unit conversion factors are not included.
Np Ng
Pitch diameter of pinion (in) d p p d   d g p d  
pd pd

d p p d   n p
Pitch-line velocity (fpm) Vt p d  
2

Wt p d  
H
Transmitted load (lbf)
Vt  p d 

4. Set the application factor, Ca Ca  1

B  0.25  12  Qv
0.6667
5. Write the equation for the dynamic load factor, C B  0.731
v

A  50  56 ( 1  B) A  65.062

© 2011 Pearson Education, Inc., Upper Saddle River, NJ. All rights reserved. This publication is protected by Copyright and written permission should be
MACHINE DESIGN - An Integrated Approach, 4th Ed. 12-8-2

B
Cv p d    A 
 
 A  Vt  p d  
min

 ft 

6. Tentatively choose the mounting factor, Cm (Assume 0 < F < 2 in Cm  1.6


0.5
7. Choose an elastic coefficient from Table 12-18 (steel on steel). Cp  2300 psi

C a W t  p d   C m
8. The surface stress equation for the pinion is σcp p d F   Cp
Cv p d   F  d p p d   I

9. Determine the endurance strength of the pinion.


 0.056
Life factor CL  2.466  N CL  0.857
Reliability CR  0.7  0.15 log( 1  R) CR  1
Temperature factor CT  1
Material surface strength (psi)
S ac  154400 psi
Grade 2, 350 HB
S ac CL
Endurance strength S fcp  S fcp  132381 psi
CT  CR
10. Write the design equations using the range of face-width to diametral-pitch ratio given in the text, and the
bending stress equation, solved for the unknown face width.
2
Limits and safety factor FU  p d  
 Sfc 
FL  p d  
16 8
Nfc =  
pd pd  σc 

C a W t  p d   C m
2
 Cp 
Face width F  p d      Nfs
Cv p d   d p p d   I  S fcp 

6 6.5 16
11. Plot F(P) vs p over the range p d   
d in in in
3
12. From the graph, choose a standard value of p d
from Table 12-2.  
F pd
1 in
p d  10 in 2
 
FL pd
13. The calculated value of F is
in
F  p d   1.393  in
 
FU pd 1
14. Round this up to the decimal equivalent of a in
common fractional value.

F  1.375  in
0
6 8 10 12 14 16
15. Then, the parameters that depend on p d and F are:
pd  in

d p p d   2.600  in d g p d   6.500  in FIGURE 12-8


Graph of Face Width and Limits for the Pinion
(surface) in Problem 12-8
© 2011 Pearson Education, Inc., Upper Saddle River, NJ. All rights reserved. This publication is protected by Copyright and written permission should be
MACHINE DESIGN - An Integrated Approach, 4th Ed. 12-8-3

Vt p d   85 Wt p d   465  lbf σcp p d F   94207  psi


ft
min

16. There is a wide range of choice for the number of teeth on the pinion. The total weight of the gears goes down
very slightly with increasing pinion tooth number. The choice of Np  26 was somewhat arbitrary but was
arrived at after trying values from 18 to 26.

© 2011 Pearson Education, Inc., Upper Saddle River, NJ. All rights reserved. This publication is protected by Copyright and written permission should be
MACHINE DESIGN - An Integrated Approach, 4th Ed. 12-9-1
PROBLEM 12-9
Statement: Design a two-stage compound spur gear train for an overall ratio of approximately 47:1. Specify
tooth numbers for each gear in the train.
Given: Approximate ratio mG  47 Number of stages n  2

Assumptions: The gears will be cut with a hob and the pressure angle will be ϕ  25 deg

Solution: See Mathcad file P1209.


1
n
1. For equal stages, the stage ratio is mGs  mG mGs  6.856

2. The minimum number of teeth that we can have without interference on a 25-deg gear cut with a hob is 14. Try
pinions with 14, 15, 16, etc. teeth to see if the mating gear will have close to an integral number of teeth.

14 mGs  95.979

15 mGs  102.835

16 mGs  109.69


n
mG   96  mG  47.02
3. The first trial is very close to an integer so try 96 teeth:  14 
 
4. We can get slightly closer by using two 14-tooth pinions with 95 and 97-tooth gears
95 97
mG  mG  47.015
14 14

5. However, it may be harder to find the 97-tooth gear and, it may be less expensive to have two gears with the
same number of teeth than to have different numbers of teeth.

© 2011 Pearson Education, Inc., Upper Saddle River, NJ. All rights reserved. This publication is protected by Copyright and written permission should be
MACHINE DESIGN - An Integrated Approach, 4th Ed. 12-10-1
PROBLEM 12-10
Statement: Design a three-stage compound spur gear train for an overall ratio of approximately 656:1. Specify
tooth numbers for each gear in the train.
Given: Gear ratio mG  656 Number of stages s  3

Solution: See Mathcad file P1210.


1
s
1. The average stage ratio is ravg  mG ravg  8.689

2. If the minimum number of teeth is (20 deg pa) Nmin  18


then the number of teeth on the driven gear for each stage is
NG1  floor  ravg Nmin NG1  156

or NG2  ceil ravg Nmin NG2  157

3. But, the prime factors of 656 are 2 4 and 41. This suggests that one of the gears have a number of teeth that is a
multiple of 41, say 164

164
 18.875 Let N2  18 N3  164
ravg

N2
Then, the remaing 2-stage ratio is mG2  mG mG2  72
N3

N5 N7
Thus, = 72 if N4  18 and N6  18
N4 N6

5 6
then N5 N7 = 72 18 18 = 2  3

4 2 1 4
Try N5  2  3 N7  2  3 then N5 N7  23328

Summarizing, N2  18 N3  164

N4  18 N5  144

N6  18 N7  162

4. Since the driven gears all have less than 180 teeth, no stage ratio is greater than 10.
5. Checking the overall gear ratio

N 3 N 5 N 7
mG  mG  656.00
N 2 N 4 N 6

© 2011 Pearson Education, Inc., Upper Saddle River, NJ. All rights reserved. This publication is protected by Copyright and written permission should be
MACHINE DESIGN - An Integrated Approach, 4th Ed. 12-11-1

PROBLEM 12-11
Statement: An epicycle spur gear train as shown in Figure 12-16 has a sun gear of 33 teeth and a planet gear
of 21 teeth. Find the required number of teeth in the ring gear and determine the ratio between
the arm and the sun gear if the ring gear is held stationary.

Given: Sun tooth number Ns  33 Arm speed ωa  1  rpm


Planet tooth number Np  21 Ring speed ωr  0  rpm

Solution: See Mathcad file P1211.


1. From Figure 12-16 we see that the diameter of the ring gear is equal to the sum of the diameter of the sun gear
and two times the diameter of the planet gear. Since sun, planets, and ring gear must have the same diametral
pitch,

Number of teeth on ring: Nr  Ns  2  Np Nr  75

2. Choose the sun gear as the first gear and the ring gear as the last. Then, write equation (12.11c).

ωra ωr  ωa Ns
= =
ωsa ωs  ωa Nr

Nr ωa  Ns ωa  Nr ωr


Solving for the sun speed, ωs  ωs  3.273  rpm
Ns

ωa
3. The ratio between the arm and sun gear is  0.306
ωs

© 2011 Pearson Education, Inc., Upper Saddle River, NJ. All rights reserved. This publication is protected by Copyright and written permission should be
MACHINE DESIGN - An Integrated Approach, 4th Ed. 12-12-1

PROBLEM 12-12
Statement: An epicycle spur gear train as shown in Figure 12-16 has a sun gear of 23 teeth and a planet gear
of 31 teeth. Find the required number of teeth in the ring gear and determine the ratio between
the arm and the ring gear if the sun gear is held stationary.

Given: Sun tooth number Ns  23 Arm speed ωa  1  rpm


Planet tooth number Np  31 Sun speed ωs  0  rpm

Solution: See Mathcad file P1212.


1. From Figure 12-16 we see that the diameter of the ring gear is equal to the sum of the diameter of the sun gear
and two times the diameter of the planet gear. Since sun, planets, and ring gear must have the same diametral
pitch,

Number of teeth on ring: Nr  Ns  2  Np Nr  85

2. Choose the sun gear as the first gear and the ring gear as the last. Then, write equation (12.11c).

ωra ωr  ωa Ns
= =
ωsa ωs  ωa Nr

Nr ωa  Ns ωa  Ns ωs


Solving for the ring speed, ωr  ωr  1.271  rpm
Nr

ωa
3. The ratio between the arm and ring gear is  0.787
ωr

© 2011 Pearson Education, Inc., Upper Saddle River, NJ. All rights reserved. This publication is protected by Copyright and written permission should be
MACHINE DESIGN - An Integrated Approach, 4th Ed. 12-13-1

PROBLEM 12-13
Statement: An epicycle spur gear train as shown in Figure 12-16 has a sun gear of 23 teeth and a planet gear
of 31 teeth. Find the required number of teeth in the ring gear and determine the ratio between
the sun and the ring gear if the arm is held stationary.

Given: Sun tooth number Ns  23 Arm speed ωa  0  rpm


Planet tooth number Np  31 Sun speed ωs  1  rpm
Solution: See Mathcad file P1213.
1. From Figure 12-16 we see that the diameter of the ring gear is equal to the sum of the diameter of the sun gear
and two times the diameter of the planet gear. Since sun, planets, and ring gear must have the same diametral
pitch,

Number of teeth on ring: Nr  Ns  2  Np Nr  85

2. Choose the sun gear as the first gear and the ring gear as the last. Then, write equation (12.11c).

ωra ωr  ωa Ns
= =
ωsa ωs  ωa Nr

Nr ωa  Ns ωa  Ns ωs


Solving for the ring speed, ωr  ωr  0.271  rpm
Nr

ωs
3. The ratio between the sun and ring gear is  3.696
ωr

© 2011 Pearson Education, Inc., Upper Saddle River, NJ. All rights reserved. This publication is protected by Copyright and written permission should be
MACHINE DESIGN - An Integrated Approach, 4th Ed. 12-14-1
PROBLEM 12-14
Statement: If the gearset in Problem 12-3 transmits 125 HP at 1000 pinion rpm, find the torque on each shaft.

Given: Tooth numbers:


pinion Np  23 Pinion speed ωp  1000 rpm
gear Ng  57 Transmitted power P  125  hp

Assumptions: There is no loss of power in the gear mesh (100% efficiency).


Solution: See Mathcad file P1214.

P
1. For the pinion shaft Tp  Tp  7878 in lbf
ωp
Tp  656.5  ft  lbf

2. The gear shaft will rotate at a lower speed, which is determined by the gear ratio. (The speed will be
decreased in proportion to the ratio and the torque will be increased by the reciprocal of the ratio).

Np
3. For the gear shaft ωg   ωp ωg  403.509  rpm
Ng
P
Tg  Tg  19524  in lbf
ωg
Tg  1627 ft  lbf

4. We could have calculated the torque on the gear shaft directly without finding the gear shaft speed,
Ng
Tg   Tp Tg  19524  in lbf
Np
Tg  1627 ft  lbf

© 2011 Pearson Education, Inc., Upper Saddle River, NJ. All rights reserved. This publication is protected by Copyright and written permission should be
MACHINE DESIGN - An Integrated Approach, 4th Ed. 12-15-1

PROBLEM 12-15
Statement: If the gearset in Problem 12-4 transmits 33 kW at 1600 pinion rpm, find the torque on each shaft.

Given: Tooth numbers:


pinion Np  27 Pinion speed ωp  1600 rpm
gear Ng  78 Transmitted power P  33 kW

Assumptions: There is no loss of power in the gear mesh (100% efficiency).


Solution: See Mathcad file P1215.

P
1. For the pinion shaft Tp  Tp  197  N  m
ωp

2. The gear shaft will rotate at a lower speed, which is determined by the gear ratio. (The speed will be decreased i
proportion to the ratio and the torque will be increased by the reciprocal of the ratio).
Np
3. For the gear shaft ωg   ωp ωg  553.846  rpm
Ng
P
Tg  Tg  569  N  m
ωg

4. We could have calculated the torque on the gear shaft directly without finding the gear shaft speed,
Ng
Tg   Tp Tg  569  N  m
Np

© 2011 Pearson Education, Inc., Upper Saddle River, NJ. All rights reserved. This publication is protected by Copyright and written permission should be
MACHINE DESIGN - An Integrated Approach, 4th Ed. 12-16-1

PROBLEM 12-16
Statement: Size the spur gears in problem 12-14 for a bending factor of safety of at least 2 assuming a steady
torque, 25 deg pressure angle, full depth teeth, quality index of 9, an AISI 4140 steel pinion, and a
class 40 cast iron gear.

Given: Factor of safety Nfb  2 Number of gear teeth Ng  57


Power to be transmitted (hp) H  125  hp Reliability R  0.99
Rotational speed of pinion n  1000 rpm AGMA Quality level Qv  9
7
Number of pinion teeth Np  23 Life (cycles) N  10

Solution: See Mathcad file P1216.


Pinion
1. Determine the bending geometry factor, J (Table 12-13) Jp  0.42
2. Write the equations for pitch diameter, pitch-line velocity, and transmitted load in terms of the unknown
diametral pitch, pd. Note that, in Mathcad, unit conversion factors are not included.
Np
Pitch diameter of pinion (in) d  p d  
pd
dpd  n
Pitch-line velocity (fpm) Vt p d  
2

Wt p d  
H
Transmitted load (lbf)
Vt  p d 

3. Set the application factor, Ka Ka  1


B  0.25  12  Qv
0.6667
4. Write the equation for the dynamic load factor, K B  0.52
v

A  50  56 ( 1  B) A  76.878
B
Kv p d    
A

 A  Vt  p d  
min

 ft 

5. Tentatively choose the mounting factor, Km (Assume 2 < F < 6 in) Km  1.7

Ka Wt p d   p d  Km
6. The bending stress equation for the pinion is σbp p d F  
Kv p d   F  Jp

7. Determine the endurance strength of the pinion.


 0.0323
Life factor KL  1.6831 N KL  1
Reliability KR  0.7  0.15 log( 1  R) KR  1
Temperature factor KT  1
Material bending strength (psi)
S atp  40000  psi
AISI 4140 Nitrided steel

S atp KL
Endurance strength S fbp  S fbp  40001  psi
KT  KR

© 2011 Pearson Education, Inc., Upper Saddle River, NJ. All rights reserved. This publication is protected by Copyright and written permission should be
MACHINE DESIGN - An Integrated Approach, 4th Ed. 12-16-2

8. Write the design equations using the range of face-width to diametral-pitch ratio given in the text, and the
bending stress equation, solved for the unknown face width.

FU  p d  
16
Upper limit
pd

FL  p d  
8
Lower limit
pd

S fb
Safety factor Nfb =
σb

Ka Wt p d   p d  Km Nfb


Face width F  p d  
Kv p d   Jp S fbp

1 1.5 6
9. Plot F(P) vs. p over the range p d   
d in in in
4
10. From the graph, choose a standard value of p d

 
from Table 12-2.
F pd
1 3
p d  4  in in

11. The calculated value of F is  


FL pd
2
F  p d   2.743  in
in

 
FU pd
12. Round this up to the decimal equivalent of a
in 1
common fractional value.

F  2.750  in
13. Then, the parameters that depend on p d and F are: 0
2 2.5 3 3.5 4 4.5 5 5.5 6
d  p d   5.750  in Kv p d   0.809 pd  in

Vt p d   1505 Wt p d   2740 lbf


ft FIGURE 12-16A
min Graph of Face Width and Limits for the Pinion in
Problem 12-16
σbp p d F   19952  psi

The assumption made in step 5 is correct so no further iteration is required.

Gear
14. Determine the bending geometry factor, J (Table 12-13) Jg  0.49
Ka Wt p d   p d  Km
15. The bending stress equation for the gear is σbg p d F  
Kv p d   F  Jg

16. Determine the endurance strength of the pinion.


Material bending strength (psi)
S atg  13000  psi
Class 40 cast iron

S atg KL
Endurance strength S fbg  S fbg  13000  psi
KT  KR
© 2011 Pearson Education, Inc., Upper Saddle River, NJ. All rights reserved. This publication is protected by Copyright and written permission should be
MACHINE DESIGN - An Integrated Approach, 4th Ed. 12-16-3

17. Write the design equations using the range of face-width to diametral-pitch ratio given in the text, and the
bending stress equation, solved for the unknown face width.

FU  p d  
16
Upper limit
pd

FL  p d  
8
Lower limit
pd
S fb
Safety factor Nfb =
σb

Ka Wt p d   p d  Km Nfb


Face width F  p d  
Kv p d   Jg S fbg

1 1.5 6
18. Plot F(P) vs. p over the range p d   
d in in in

19. From the graph, choose a standard value of p d


from Table 12-2. 6
1
p d  3  in
 
F pd
4.5
20. The calculated value of F is in
F  p d   4.178  in  
FL pd
3
21. Round this up to the decimal equivalent of a in
common fractional value.  
FU pd
F  4.250  in in 1.5
22. Then, the parameters that depend on P and F are:
d  p d   7.667  in Kv p d   0.788 0
2 2.5 3 3.5 4 4.5 5 5.5 6
pd  in
Vt p d   2007 Wt p d   2055 lbf
ft
min FIGURE 12-16B
Graph of Face Width and Limits for the Gear in
σbg p d F   6391 psi Problem 12-16

23. The gear dimensions are larger (smaller diametral pitch means bigger teeth) than for the pinion. This means
that we will accept the gear requirements for the pinion, thus, for the set

1
Diametral pitch p d  3  in

Face width F  4.250  in


24. Determine the realized factor of safety for the gear using the above values for F and p d.

S fbg
Gear factor of safety Nfbg  Nfbg  2.0
σbg p d F 

25. Check the factor of safety on the pinion:


S fbp
Pinion factor of safety Nfbp  Nfbp  5.4
σbp p d F 
© 2011 Pearson Education, Inc., Upper Saddle River, NJ. All rights reserved. This publication is protected by Copyright and written permission should be
MACHINE DESIGN - An Integrated Approach, 4th Ed. 12-17-1

PROBLEM 12-17
Statement: Size the spur gears in problem 12-15 for a bending factor of safety of at least 2.5 assuming a
steady torque, 20 deg pressure angle, full depth teeth, quality index of 11, an AISI 4340 steel
pinion, and an A-7-d nodular iron gear.

Given: Factor of safety Nfb  2.5 Number of gear teeth Ng  78


Power to be transmitted (hp) H  33 kW Reliability R  0.99
Rotational speed of pinion n  1600 rpm AGMA Quality level Qv  11
7
Number of pinion teeth Np  27 Life (cycles) N  10

Solution: See Mathcad file P1217.


Pinion
1. Determine the bending geometry factor, J (Table 12-13) Jp  0.37
2. Write the equations for pitch diameter, pitch-line velocity, and transmitted load in terms of the unknown
diametral pitch, p d. Note that, in Mathcad, unit conversion factors are not included.
Np
Pitch diameter of pinion (in) d  p d  
pd
dpd  n
Pitch-line velocity (fpm) Vt p d  
2

Wt p d  
H
Transmitted load (lbf)
Vt  p d 

3. Set the application factor, Ka Ka  1

B  0.25  12  Qv
0.6667
4. Write the equation for the dynamic load factor, K B  0.25
v

A  50  56 ( 1  B) A  92
B
Kv p d    
A

 A  Vt  p d  
min

 ft 

5. Tentatively choose the mounting factor, Km (Assume 0 < F < 2 in) Km  1.6

Ka Wt p d   p d  Km
6. The bending stress equation for the pinion is σbp p d F  
Kv p d   F  Jp

7. Determine the endurance strength of the pinion.


 0.0323
Life factor KL  1.6831 N KL  1
Reliability KR  0.7  0.15 log( 1  R) KR  1
Temperature factor KT  1
Material bending strength (psi)
S atp  42000  psi
AISI 4340 Nitrided steel

S atp KL
Endurance strength S fbp  S fbp  42001  psi
KT  KR

8. Write the design equations using the range of face-width to diametral-pitch ratio given in the text, and the
bending stress equation, solved for the unknown face width.
© 2011 Pearson Education, Inc., Upper Saddle River, NJ. All rights reserved. This publication is protected by Copyright and written permission should be
MACHINE DESIGN - An Integrated Approach, 4th Ed. 12-17-2

FU  p d  
16
Upper limit
pd

FL  p d  
8
Lower limit
pd

S fb
Safety factor Nfb =
σb

Ka Wt p d   p d  Km Nfb


Face width F  p d  
Kv p d   Jp S fbp

1 1.5 10
9. Plot F(P) vs. p over the range p d   
d in in in
4
10. From the graph, choose a standard value of p d

 
from Table 12-2.
F pd
1 3
p d  6  in in

11. The calculated value of F is  


FL pd
2
F  p d   1.318  in
in

 
FU pd
12. Round this up to the decimal equivalent of a
in 1
common fractional value.

F  1.375  in
13. Then, the parameters that depend on p d and F are: 0
4 5 6 7 8 9 10
d  p d   4.500  in Kv p d   0.908 pd  in

Vt p d   1885 Wt p d   775  lbf


ft FIGURE 12-17A
min Graph of Face Width and Limits for the Pinion in
Problem 12-17
σbp p d F   16103  psi

The assumption made in step 5 is correct so no further iteration is required.

Gear
14. Determine the bending geometry factor, J (Table 12-13) Jg  0.42
Ka Wt p d   p d  Km
15. The bending stress equation for the gear is σbg p d F  
Kv p d   F  Jg

16. Determine the endurance strength of the pinion.


Material bending strength (psi)
S atg  34000  psi
A-7-d nodular iron

S atg KL
Endurance strength S fbg  S fbg  34001  psi
KT  KR

17. Write the design equations using the range of face-width to diametral-pitch ratio given in the text, and the
bending stress equation, solved for the unknown face width.
© 2011 Pearson Education, Inc., Upper Saddle River, NJ. All rights reserved. This publication is protected by Copyright and written permission should be
MACHINE DESIGN - An Integrated Approach, 4th Ed. 12-17-3

FU  p d  
16
Upper limit
pd

FL  p d  
8
Lower limit
pd
S fb
Safety factor Nfb =
σb

Ka Wt p d   p d  Km Nfb


Face width F  p d  
Kv p d   Jg S fbg

1 1.5 10
18. Plot F(P) vs. p over the range p d   
d in in in

19. From the graph, choose a standard value of p d


from Table 12-2. 6
1
p d  6  in
 
F pd
4.5
20. The calculated value of F is in
F  p d   1.434  in  
FL pd
3
21. Round this up to the decimal equivalent of a in
common fractional value.  
FU pd
F  1.500  in in 1.5
22. Then, the parameters that depend on P and F are:
d  p d   4.500  in Kv p d   0.908 0
4 5 6 7 8 9 10
pd  in
Vt p d   1885 Wt p d   775  lbf
ft
min FIGURE 12-17B
Graph of Face Width and Limits for the Gear in
σbg p d F   13004  psi Problem 12-17

23. The gear dimensions are larger than those for the pinion. This means that we will accept the gear requirements
for the pinion, thus, for the set

1
Diametral pitch p d  6  in

Face width F  1.500  in


24. Determine the realized factor of safety for the gear using the above values for F and p d.

S fbg
Gear factor of safety Nfbg  Nfbg  2.6
σbg p d F 

25. Check the factor of safety on the pinion:


S fbp
Pinion factor of safety Nfbp  Nfbp  2.8
σbp p d F 

© 2011 Pearson Education, Inc., Upper Saddle River, NJ. All rights reserved. This publication is protected by Copyright and written permission should be
MACHINE DESIGN - An Integrated Approach, 4th Ed. 12-18-1

PROBLEM 12-18
Statement: Size the spur gears in problem 12-14 for a surface factor of safety of at least 2 assuming a steady
torque, 25 deg pressure angle, full depth teeth, quality index of 9, an AISI 4140 steel pinion, and
a class 40 cast iron gear.

Given: Factor of safety Nfs  2 Number of gear teeth Ng  57


Power to be transmitted (hp) H  125  hp Reliability R  0.99
Rotational speed of pinion n  1000 rpm AGMA Quality level Qv  9
7
Number of pinion teeth Np  23 Life (cycles) N  10
Pressure angle ϕ  25 deg
Solution: See Mathcad file P1218.
Pinion
1. Determine the surface geometry factor, I.

 sin( ϕ)  cos( ϕ)   Ng
I    I  0.1365
 2  Np  Ng
2. Write the equations for pitch diameter, pitch-line velocity, and transmitted load in terms of the unknown
diametral pitch, p d. Note that, in Mathcad, unit conversion factors are not included.

Np
Pitch diameter of pinion (in) d p p d  
pd

d p p d   n
Pitch-line velocity (fpm) Vt p d  
2

Wt p d  
H
Transmitted load (lbf)
Vt  p d 

3. Set the application factor, Ca Ca  1

B  0.25  12  Qv
0.6667
4. Write the equation for the dynamic load factor, C B  0.52
v

A  50  56 ( 1  B) A  76.878
B
Cv p d    
A

 A  Vt  p d  
min

 ft 

5. Tentatively choose the mounting factor, Cm (Assume 2 < F < 6 in Cm  1.7


0.5
6. Choose an elastic coefficient from Table 12-18. (Steel on cast-iron) Cp  2100 psi

C a W t  p d   C m
6. The surface stress equation for the pinion is σcp p d F   Cp
Cv p d   F  d p p d   I

7. Determine the endurance strength of the pinion.


 0.056
Life factor CL  2.466  N CL  1

© 2011 Pearson Education, Inc., Upper Saddle River, NJ. All rights reserved. This publication is protected by Copyright and written permission should be
MACHINE DESIGN - An Integrated Approach, 4th Ed. 12-18-2

Reliability CR  0.7  0.15 log( 1  R) CR  1


Temperature factor CT  1
Material surface strength (psi)
S acp  165000 psi
AISI 4140 Nitrided steel
S acp CL
Endurance strength S fcp  S fcp  164997 psi
CT  CR

8. Write the design equations using the range of face-width to diametral-pitch ratio given in the text, and the
surface stress equation, solved for the unknown face width.

FU  p d  
16
Upper limit
pd

FL  p d  
8
Lower limit
pd
2
 Sfc 
Safety factor Nfc = σ 
 c

C a W t  p d   C m
2
 Cp 
Face width F  p d      Nfs
Cv p d   d p p d   I  S fcp 

1 1.5 6
9. Plot F(P) vs. p over the range p d   
d in in in

10. From the graph, choose a standard value of p d 4


from Table 12-2.
1 F pd 
p d  4  in 3
in
11. The calculated value of F is  
FL pd
F  p d   2.379  in in 2

12. Round this up to the decimal equivalent of a  


FU pd
common fractional value. in 1

F  2.375  in
13. Then, the parameters that depend on p d and F are: 0
2 2.5 3 3.5 4 4.5 5 5.5 6
d p p d   5.750  in Cv p d   0.809
pd  in

Vt p d   1505 Wt p d   2740 lbf


ft
FIGURE 12-18A
min Graph of Face Width and Limits for the Pinion in
σcp p d F   116767 psi Problem 12-18

The assumption made in step 5 is correct so no further iteration is required.

Gear

Ng
14. Write the equation for the gear pitch diameter d g p d  
pd
© 2011 Pearson Education, Inc., Upper Saddle River, NJ. All rights reserved. This publication is protected by Copyright and written permission should be
MACHINE DESIGN - An Integrated Approach, 4th Ed. 12-18-3

C a W t  p d   C m
15. The surface stress equation for the gear is σcg p d F   Cp
Cv p d   F  d g p d   I

16. Determine the endurance strength of the pinion.


Material bending strength (psi) S acg  80000  psi
Class 40 cast iron
Hp
Hardness factor Hp  230 Hg  200  1.2
Hg

From equation 12.26, A  0


Ng
Gear ratio mg  mg  2.478
Np

Hardness ratio factor CH  1  A   mg  1  CH  1

S acg CL  CH
Endurance strength S fcg  S fcg  79999  psi
CT  CR
17. Write the design equations using the range of face-width to diametral-pitch ratio given in the text, and the
surface stress equation, solved for the unknown face width.

FU  p d  
16
Upper limit
pd

FL  p d  
8
Lower limit
pd
2
 Sfc 
Safety factor Nfc =  
 σc 
C a W t  p d   C m
2
 Cp 
Face width F  p d      Nfs
Cv p d   d g p d   I  S fcg 

1 1.5 6
18. Plot F(P) vs. p over the range p d   
d in in in
6
19. From the graph, choose a standard value of p d

 
from Table 12-2.
F pd
1 4.5
p d  4  in in

20. The calculated value of F is FL pd 


3
F  p d   4.083  in
in

FU pd 
21. Round this up to the decimal equivalent of a
in 1.5
common fractional value.
F  4.125  in
22. Then, the parameters that depend on p d and F are: 0
2 2.5 3 3.5 4 4.5 5 5.5 6
d g p d   14.250 in Cv p d   0.809 pd  in
FIGURE 12-18B
Vt p d   1505 Wt p d   2740 lbf
ft
Graph of Face Width and Limits for the Gear in
min Problem 12-18
© 2011 Pearson Education, Inc., Upper Saddle River, NJ. All rights reserved. This publication is protected by Copyright and written permission should be
MACHINE DESIGN - An Integrated Approach, 4th Ed. 12-18-4

σcg p d F   56282  psi

23. The gear face width is larger than for the pinion. This means that we will accept the gear requirements for the
pinion thus, for the set

1
Diametral pitch p d  4  in

Face width F  4.125  in


24. Determine the realized factor of safety for the gear using the above values for F and p d.

2
 Sfcg 
Gear factor of safety Nfsg  σ  Nfsg  2.0
 cg pd F  
25. Check the factor of safety on the pinion:
2
 Sfcp 
Pinion factor of safety Nfsp    Nfsp  3.5
 σcp pd F  

© 2011 Pearson Education, Inc., Upper Saddle River, NJ. All rights reserved. This publication is protected by Copyright and written permission should be
MACHINE DESIGN - An Integrated Approach, 4th Ed. 12-19-1

PROBLEM 12-19
Statement: Size the spur gears in problem 12-15 for a surface factor of safety of at least 1.2 assuming a
steady torque, 20 deg pressure angle, full depth teeth, quality index of 11, an AISI 4340 steel
pinion, and an A-7-d nodular iron gear.

Given: Factor of safety Nfs  1.2 Number of gear teeth Ng  78


Power to be transmitted (hp) H  33 kW Reliability R  0.99
Rotational speed of pinion n  1600 rpm AGMA Quality level Qv  11
7
Number of pinion teeth Np  27 Life (cycles) N  10
Pressure angle ϕ  20 deg
Solution: See Mathcad file P1219.
Pinion
1. Determine the surface geometry factor, I.

 sin( ϕ)  cos( ϕ)   Ng
I    I  0.1194
 2  Np  Ng
2. Write the equations for pitch diameter, pitch-line velocity, and transmitted load in terms of the unknown
diametral pitch, p d. Note that, in Mathcad, unit conversion factors are not included.

Np
Pitch diameter of pinion (in) d p p d  
pd

d p p d   n
Pitch-line velocity (fpm) Vt p d  
2

Wt p d  
H
Transmitted load (lbf)
Vt  p d 

3. Set the application factor, Ca Ca  1

B  0.25  12  Qv
0.6667
4. Write the equation for the dynamic load factor, C B  0.25
v

A  50  56 ( 1  B) A  92
B
Cv p d    A 
 
 A  Vt  p d  
min

 ft 

5. Tentatively choose the mounting factor, Cm (Assume 0 < F < 2 in Cm  1.6


0.5
6. Choose an elastic coefficient from Table 12-18. (Steel on nodular-iron) Cp  2160 psi

C a W t  p d   C m
6. The surface stress equation for the pinion is σcp p d F   Cp
Cv p d   F  d p p d   I

7. Determine the endurance strength of the pinion.


 0.056
Life factor CL  2.466  N CL  1

Reliability CR  0.7  0.15 log( 1  R) CR  1


© 2011 Pearson Education, Inc., Upper Saddle River, NJ. All rights reserved. This publication is protected by Copyright and written permission should be
MACHINE DESIGN - An Integrated Approach, 4th Ed. 12-19-2

Temperature factor CT  1
Material surface strength (psi)
S acp  162000 psi
AISI 4340 Nitrided steel
S acp CL
Endurance strength S fcp  S fcp  161997 psi
CT  CR

8. Write the design equations using the range of face-width to diametral-pitch ratio given in the text, and the
surface stress equation, solved for the unknown face width.

FU  p d  
16
Upper limit
pd

FL  p d  
8
Lower limit
pd
2
 Sfc 
Safety factor Nfc = σ 
 c

C a W t  p d   C m
2
 Cp 
Face width F  p d      Nfs
Cv p d   d p p d   I  S fcp 

6 6.5 16
9. Plot F(P) vs. p over the range p d   
d in in in

10. From the graph, choose a standard value of p d 2


from Table 12-2.
1 F pd 
p d  10 in 1.5
in
11. The calculated value of F is  
FL pd
F  p d   1.478  in in 1

12. Round this up to the decimal equivalent of a  


FU pd
common fractional value. in 0.5

F  1.500  in
13. Then, the parameters that depend on p d and F are: 0
6 8 10 12 14 16
d p p d   2.700  in Cv p d   0.925
pd  in

Vt p d   1131 Wt p d   1291 lbf


ft
FIGURE 12-19A
min Graph of Face Width and Limits for the Pinion in
σcp p d F   146808 psi Problem 12-19

The assumption made in step 5 is correct so no further iteration is required.

Gear
Ng
14. Write the equation for the gear pitch diameter d g p d  
pd

C a W t  p d   C m
15. The surface stress equation for the gear is σcg p d F   Cp
Cv p d   F  d g p d   I
© 2011 Pearson Education, Inc., Upper Saddle River, NJ. All rights reserved. This publication is protected by Copyright and written permission should be
MACHINE DESIGN - An Integrated Approach, 4th Ed. 12-19-3

16. Determine the endurance strength of the pinion.


Material bending strength (psi) S acg  97000  psi
A-7-d nodular iron
Hp
Hardness factor Hp  235 Hg  230  1.0
Hg

From equation 12.26, A  0


Ng
Gear ratio mg  mg  2.889
Np

Hardness ratio factor CH  1  A   mg  1  CH  1

S acg CL  CH
Endurance strength S fcg  S fcg  96998  psi
CT  CR
17. Write the design equations using the range of face-width to diametral-pitch ratio given in the text, and the
surface stress equation, solved for the unknown face width.

FU  p d  
16
Upper limit
pd

FL  p d  
8
Lower limit
pd
2
 Sfc 
Safety factor Nfc =  
 σc 
C a W t  p d   C m
2
 Cp 
Face width F  p d      Nfs
Cv p d   d g p d   I  S fcg 

6 6.5 16
18. Plot F(P) vs. p over the range p d   
d in in in
3
19. From the graph, choose a standard value of p d

 
from Table 12-2.
F pd
1
p d  10 in in
2
20. The calculated value of F is FL pd 
F  p d   1.427  in
in

 
FU pd 1
21. Round this up to the decimal equivalent of a
common fractional value. in
F  1.500  in
22. Then, the parameters that depend on p d and F are: 0
6 8 10 12 14 16
d g p d   7.800  in Cv p d   0.925 pd  in
FIGURE 12-19B
Vt p d   1131 Wt p d   1291 lbf
ft
Graph of Face Width and Limits for the Gear in
min Problem 12-19

σcg p d F   86374  psi


© 2011 Pearson Education, Inc., Upper Saddle River, NJ. All rights reserved. This publication is protected by Copyright and written permission should be
MACHINE DESIGN - An Integrated Approach, 4th Ed. 12-19-4

23. The gear face width is the same as that for the pinion. This means that we will accept the gear requirements for
the set

1
Diametral pitch p d  10 in

Face width F  1.500  in


24. Determine the realized factor of safety for the gear using the above values for F and p d.

2
 Sfcg 
Gear factor of safety Nfsg  σ  Nfsg  1.3
 cg pd F  
25. Check the factor of safety on the pinion:
2
 Sfcp 
Pinion factor of safety Nfsp    Nfsp  1.2
 σcp pd F  

© 2011 Pearson Education, Inc., Upper Saddle River, NJ. All rights reserved. This publication is protected by Copyright and written permission should be
MACHINE DESIGN - An Integrated Approach, 4th Ed. 12-20-1

PROBLEM 12-20
Statement: If the gearset in Problem 12-11 transmits 83 kW at 1200 arm rpm, find the torque on each shaft.

Given: Sun tooth number Ns  33 Transmitted power P  83 kW


Planet tooth number Np  21 Arm speed ωa  1200 rpm
Ring gear speed ωr  0  rpm

Solution: See Mathcad file P1220.

1. Determine the number of teeth on the ring: Nr  Ns  2  Np Nr  75

2. Choose the sun gear as the first gear and the ring gear as the last. Then, write equation (12.11c).

ωra ωr  ωa Ns
= =
ωsa ωs  ωa Nr

Nr ωa  Ns ωa  Nr ωr


Solving for the planet speed, ωs  ωs  3927 rpm
Ns

3. Calculate the torque on the sun and arm shafts.


P
Arm shaft Ta  Ta  660  N  m
ωa

P
Sun shaft Ts  Ts  202  N  m
ωs

© 2011 Pearson Education, Inc., Upper Saddle River, NJ. All rights reserved. This publication is protected by Copyright and written permission should be
MACHINE DESIGN - An Integrated Approach, 4th Ed. 12-21-1
PROBLEM 12-21
Statement: If the gearset in Problem 12-12 transmits 39 HP at 2600 arm rpm, find the torque on each shaft.

Given: Tooth numbers: sun Ns  23 planet Np  31


Arm speed ωa  2600 rpm Sun speed ωs  0  rpm
Transmitted power P  39 hp

Assumptions: There is no loss of power in the gear mesh (100% efficiency).


Solution: See Mathcad file P1221.

P
1. For the arm shaft Ta  Ta  945  in lbf
ωa

2. The ring gear will rotate at a higher speed, which is determined by the gear ratio.

3. For the ring gear (see Problem 12-12)

Number of teeth on ring: Nr  Ns  2  Np Nr  85

Nr ωa  Ns ωa  Ns ωs


ωr  ωr  3303.529 rpm
Nr

P
Tr  Tr  744  in lbf
ωr

© 2011 Pearson Education, Inc., Upper Saddle River, NJ. All rights reserved. This publication is protected by Copyright and written permission should be
MACHINE DESIGN - An Integrated Approach, 4th Ed. 12-22-1
PROBLEM 12-22
Statement: If the gearset in Problem 12-13 transmits 23 kW at 4800 sun rpm, find the torque on each shaft.

Given: Tooth numbers:


sun Ns  23 Arm speed ωa  0  rpm
planet Np  31 Sun speed ωs  4800 rpm
Transmitted power P  23 kW

Assumptions: There is no loss of power in the gear mesh (100% efficiency).


Solution: See Mathcad file P1222.
P
1. For the sun shaft Ts  Ts  45.8 N  m
ωs

2. The ring gear will rotate at a lower speed, which is determined by the gear ratio.

3. For the ring gear (see Problem 12-13)

Number of teeth on ring: Nr  Ns  2  Np Nr  85

Nr ωa  Ns ωa  Ns ωs


ωr  ωr  1298.8 rpm
Nr

P
Tr  Tr  169  N  m
ωr

© 2011 Pearson Education, Inc., Upper Saddle River, NJ. All rights reserved. This publication is protected by Copyright and written permission should be
MACHINE DESIGN - An Integrated Approach, 4th Ed. 12-23-1

PROBLEM 12-23
Statement: Size the spur gears in problem 12-20 for a bending factor of safety of at least 2.8 assuming a
steady torque, 25 deg pressure angle, full depth teeth, quality index of 9, an AISI 4140 steel
pinion, and a class 40 cast iron gear.

Given: Factor of safety Nfb  2.8 Number of gear teeth Ng  33


Power to be transmitted H  83 kW Reliability R  0.99
Rotational speed of pinion n  1200 rpm AGMA Quality level Qv  9
7
Number of pinion teeth Np  21 Life (cycles) N  10

Solution: See Mathcad file P1223.


Pinion
1. Determine the bending geometry factor, J (Table 12-13) Jp  0.40
2. Write the equations for pitch diameter, pitch-line velocity, and transmitted load in terms of the unknown
diametral pitch, p d. Note that, in Mathcad, unit conversion factors are not included.

Np Ng
Pitch diameter of pinion and gear (in) d p p d   d g p d  
pd pd
d g p d   n
Pitch-line velocity (fpm) Vt p d  
2

Wt p d  
H
Transmitted load (lbf)
Vt  p d 
3. Set the application factor, Ka Ka  1

B  0.25  12  Qv
0.6667
4. Write the equation for the dynamic load factor, K B  0.52
v

A  50  56 ( 1  B) A  76.878
B
Kv p d    A 
 
 A  Vt  p d  
min

 ft 

5. Tentatively choose the mounting factor, Km (Assume 2 < F < 6 in) Km  1.7

Ka Wt p d   p d  Km
6. The bending stress equation for the pinion is σbp p d F  
Kv p d   F  Jp

7. Determine the endurance strength of the pinion.


 0.0323
Life factor KL  1.6831 N KL  1
Reliability KR  0.7  0.15 log( 1  R) KR  1
Temperature factor KT  1
Material bending strength (psi)
S atp  40000  psi
AISI 4140 Nitrided steel

S atp KL
Endurance strength S fbp  S fbp  40001  psi
KT  KR

© 2011 Pearson Education, Inc., Upper Saddle River, NJ. All rights reserved. This publication is protected by Copyright and written permission should be
MACHINE DESIGN - An Integrated Approach, 4th Ed. 12-23-2

8. Write the design equations using the range of face-width to diametral-pitch ratio given in the text, and the
bending stress equation, solved for the unknown face width.

FU  p d  
16
Upper limit
pd

FL  p d  
8
Lower limit
pd

S fb
Safety factor Nfb =
σb

Ka Wt p d   p d  Km Nfb


Face width F  p d  
Kv p d   Jp S fbp

1 1.5 6
9. Plot F(P) vs. p over the range p d   
d in in in
6
10. From the graph, choose a standard value of p d

 
from Table 12-2. 5
F pd
1
p d  4  in in
4
11. The calculated value of F is  
FL pd
3
F  p d   2.196  in
in

 
FU pd 2
12. Round this up to the decimal equivalent of a
common fractional value. in
1
F  2.375  in
13. Then, the parameters that depend on p d and F are: 0
2 3 4 5 6
d p p d   5.250  in Kv p d   0.768 pd  in

Vt p d   2592 Wt p d   1417 lbf


ft FIGURE 12-23A
min Graph of Face Width and Limits for the Pinion in
Problem 12-23
σbp p d F   13212  psi

The assumption made in step 5 is correct so no further iteration is required.

Gear
14. Determine the bending geometry factor, J (Table 12-13) Jg  0.45
Ka Wt p d   p d  Km
15. The bending stress equation for the gear is σbg p d F  
Kv p d   F  Jg

16. Determine the endurance strength of the pinion.


Material bending strength (psi)
S atg  13000  psi
Class 40 cast iron

S atg KL
Endurance strength S fbg  S fbg  13000  psi
KT  KR
© 2011 Pearson Education, Inc., Upper Saddle River, NJ. All rights reserved. This publication is protected by Copyright and written permission should be
MACHINE DESIGN - An Integrated Approach, 4th Ed. 12-23-3

17. Write the design equations using the range of face-width to diametral-pitch ratio given in the text, and the
bending stress equation, solved for the unknown face width.

FU  p d  
16
Upper limit
pd

FL  p d  
8
Lower limit
pd
S fb
Safety factor Nfb =
σb

Ka Wt p d   p d  Km Nfb


Face width F  p d  
Kv p d   Jg S fbg

1 1.25 6
18. Plot F(P) vs. p over the range p d   
d in in in

19. From the graph, choose a standard value of p d


from Table 12-2. 6
1
p d  3  in
 
F pd
5
20. The calculated value of F is in
F  p d   3.486  in  
FL pd
4
21. Round this up to the decimal equivalent of a in
common fractional value.  
FU pd
F  3.500  in in 3
22. Then, the parameters that depend on P and F are:
d g p d   11.000 in Kv p d   0.744 2
2 3 4
pd  in
Vt p d   3456 Wt p d   1063 lbf
ft
min FIGURE 12-23B
Graph of Face Width and Limits for the Gear in
σbg p d F   4624 psi Problem 12-23

23. The gear dimensions are larger (smaller diametral pitch means bigger teeth) than for the pinion. This means that
we will accept the gear requirements for the pinion, thus, for the set

1
Diametral pitch p d  3  in

Face width F  3.500  in


24. Determine the realized factor of safety for the gear using the above values for F and p d.

S fbg
Gear factor of safety Nfbg  Nfbg  2.8
σbg p d F 
25. Check the factor of safety on the pinion:
S fbp
Pinion factor of safety Nfbp  Nfbp  7.7
σbp p d F 
© 2011 Pearson Education, Inc., Upper Saddle River, NJ. All rights reserved. This publication is protected by Copyright and written permission should be
MACHINE DESIGN - An Integrated Approach, 4th Ed. 12-24-1

PROBLEM 12-24
Statement: Size the spur gears in Problem 12-21 for a bending factor of safety of at least 2.4 assuming a
steady torque, 20 deg pressure angle, full depth teeth, quality index of 11, an AISI 4340 steel
pinion, and an A-7-d nodular iron gear.

Given: Factor of safety Nfb  2.4


Power to be transmitted (hp) H  39 hp Reliability R  0.99
Number of pinion (sun) teeth Np  23 AGMA Quality level Qv  11
7
Number of gear (planet) teeth Ng  31 Life (cycles) N  10

Solution: See Mathcad file P1224.


Take the sun as the pinion and the planet as the gear, which is also an idler. With a stationary sun, the planet
gear will have one point (where it meshes with the sun) that has an instantaneous zero velocity. All other
points on the planet will have a linear velocity that is proportional to its distance from this instant center. The
point that meshes with the ring gear has maximum velocity. The pitchline velocity at the ring gear is a
function of the tooth numbers and the diametral pitch of the set. But the absolute planet velocity depends
only on the tooth numbers and the arm velocity. For the data of Problem 12-21, the absolute planet (gear)
rotational velocity is

For ωa  2600 rpm ωr  3303.53  rpm

Np  2  Ng
ωg   ωr ωg  9058.1 rpm
Ng

The sun (pinion) is stationary and, therefore, has a zero pitchline velocity. However, the planet (gear) mesh
point rotates about the pitch circle of the sun at the rotational velocity of the arm. Therefore, take the pinion
pitchline velocity to be
Np
Vtp = rp ωa =  ωa
2 pd

The maximum pitchline velocity of the planet (gear) is at the point where it meshes with the ring gear, which is
also the pitchline velocity of the ring gear. Thus, for the gear, the pitchline velocity is

Np  2  Ng
Vtg = rr ωr =  ωr
2 pd

The torques on the arm and ring were determined in Problem 12-21 to be Ta  945.38 in lbf and
Tr  744.05 in lbf . Load analysis of the gearset reveals that the transmitted load on the sun, planet, and ring
gear teeth is

2  p d  Tr
Wt =
Np  2  Ng

Pinion
1. Determine the bending geometry factor, J (Table 12-13) Jp  0.34
2. Write the equations for pitch diameter, pitch-line velocity, and transmitted load in terms of the unknown
diametral pitch, pd. Note that, in Mathcad, unit conversion factors are not included.
Np
Pitch diameter of pinion (in) d  p d  
pd

© 2011 Pearson Education, Inc., Upper Saddle River, NJ. All rights reserved. This publication is protected by Copyright and written permission should be
MACHINE DESIGN - An Integrated Approach, 4th Ed. 12-24-2

Np
Pitch-line velocity (fpm) Vtp p d    ωa
2 pd
2  p d  Tr
Transmitted load (lbf) Wt p d  
Np  2  Ng
3. Set the application factor, Ka Ka  1

B  0.25  12  Qv
0.6667
4. Write the equation for the dynamic load factor, K B  0.25
v

A  50  56 ( 1  B) A  92
B
Kvp p d    A 
 
 A  Vtp pd  
min

 ft 

5. Tentatively choose the mounting factor, Km (Assume 0 < F < 2 in) Km  1.6

Ka Wt p d   p d  Km
6. The bending stress equation for the pinion is σbp p d F  
Kvp p d   F  Jp

7. Determine the endurance strength of the pinion.


 0.0323
Life factor KL  1.6831 N KL  1
Reliability KR  0.7  0.15 log( 1  R) KR  1
Temperature factor KT  1
Material bending strength (psi)
S atp  42000  psi
AISI 4340 Nitrided steel

S atp KL
Endurance strength S fbp  S fbp  42001  psi
KT  KR
8. Write the design equations using the range of face-width to diametral-pitch ratio given in the text, and the
bending stress equation, solved for the unknown face width.

FU  p d  
16
Upper limit
pd

FL  p d  
8
Lower limit
pd

S fb
Safety factor Nfb =
σb

Ka Wt p d   p d  Km Nfb


Face width F  p d  
Kvp p d   Jp S fbp

10 11.5 20
9. Plot F(P) vs. p over the range p d   
d in in in

10. From the graph, choose a standard value of p d from Table 12-2.

© 2011 Pearson Education, Inc., Upper Saddle River, NJ. All rights reserved. This publication is protected by Copyright and written permission should be
MACHINE DESIGN - An Integrated Approach, 4th Ed. 12-24-3

1
p d  14 in 2

11. The calculated value of F is


 
F pd
F  p d   0.997  in 1.5
in
12. Round this up to the decimal equivalent of a  
FL pd
common fractional value. 1
in
F  1.000  in
 
FU pd
13. Then, the parameters that depend on p d and F are:
in 0.5
d  p d   1.643  in Kvp p d   0.925

Vtp p d   1118 Wt p d   245  lbf


ft
0
min 10 12 14 16 18 20
pd  in
σbp p d F   17449  psi
FIGURE 12-24A
The assumption made in step 5 is correct so no Graph of Face Width and Limits for the Pinion in
further iteration is required. Problem 12-24

Gear
14. Determine the bending geometry factor, J (Table 12-9) Jg  0.37
15. Write the equations for pitch diameter and pitchline velocity for the gear in terms of the unknown diametral pitc
p d. Note that, in Mathcad, unit conversion factors are not included.
Ng
Pitch diameter of pinion (in) dg p d  
pd

Np  Ng
Pitchline velocity (fpm) Vtg p d    ωr
2 pd

B
Kvg p d    A 
16. Write the equation for the dynamic load factor, K  
 A  Vtg pd  
min
v

 ft 

17. Account for the fact that the gear is an idler by setting the idler factor KI  1.42

Ka Wt p d   p d  Km KI


17. The bending stress equation for the gear is σbg p d F  
Kvg p d   F  Jg

18. Determine the endurance strength of the pinion.


Material bending strength (psi)
A-7-d nodular iron S atg  34000  psi

S atg KL
Endurance strength S fbg  S fbg  34001  psi
KT  KR

19. Write the design equations using the range of face-width to diametral-pitch ratio given in the text, and the
bending stress equation, solved for the unknown face width.

© 2011 Pearson Education, Inc., Upper Saddle River, NJ. All rights reserved. This publication is protected by Copyright and written permission should be
MACHINE DESIGN - An Integrated Approach, 4th Ed. 12-24-4

FU  p d  
16
Upper limit
pd

FL  p d  
8
Lower limit
pd

S fb
Safety factor Nfb =
σb

Ka Wt p d   p d  Km KI  Nfb


Face width F  p d  
Kvg p d   Jg S fbg

6 6.5 16
17. Plot F(P) vs. p d over the range p d   
in in in
2
18. From the graph, choose a standard value of
p d from Table 12-2. F pd 
1.5
1 in
p d  12 in
 
FL pd
19. The calculated value of F is 1
in
F  p d   1.244  in
FU pd 
20. Round this up to the decimal equivalent of a in 0.5
common fractional value.

F  1.250  in
0
6 8 10 12 14 16
21. Then, the parameters that depend on P and F are:
pd  in
FIGURE 12-24B
dg p d   2.583  in Kvg p d   0.879 Graph of Face Width and Limits for the Gear in
Problem 12-24

Vtg p d   3892 Wt p d   210  lbf


ft
min

σbg p d F   14095  psi

22. The gear dimensions are larger (smaller diametral pitch means bigger teeth) than for the pinion. This means that
we will accept the gear requirements for the pinion, thus, for the set
1
Diametral pitch p d  12 in

Face width F  1.250  in


23. Determine the realized factor of safety for the gear using the above values for F and p d.

S fbg
Gear factor of safety Nfbg  Nfbg  2.4
σbg p d F 

24. Check the factor of safety on the pinion:

S fbp
Pinion factor of safety Nfbp  Nfbp  4.1
σbp p d F 
© 2011 Pearson Education, Inc., Upper Saddle River, NJ. All rights reserved. This publication is protected by Copyright and written permission should be
MACHINE DESIGN - An Integrated Approach, 4th Ed. 12-25-1

PROBLEM 12-25
Statement: Size the spur gears in problem 12-20 for a surface factor of safety of at least 1.7 assuming a
steady torque, 25 deg pressure angle, full depth teeth, quality index of 9, an AISI 4140 steel
pinion, and a class 40 cast iron gear.

Given: Factor of safety Nfs  1.7 Number of gear teeth Ng  33


Power to be transmitted H  83 kW Reliability R  0.99
Rotational speed of pinion n g  1200 rpm AGMA Quality level Qv  9
7
Number of pinion teeth Np  21 Life (cycles) N  10
Pressure angle ϕ  25 deg
Solution: See Mathcad file P1225.
Pinion
1. Determine the surface geometry factor, I.

 sin( ϕ)  cos( ϕ)   Ng
I    I  0.1170
 2  Np  Ng
2. Write the equations for pitch diameter, pitchline velocity, and transmitted load in terms of the unknown
diametral pitch, p d. Note that, in Mathcad, unit conversion factors are not included.

Np Ng
Pitch diameters of pinion and gear (in) d p p d   d g p d  
pd pd

d g p d   n g
pitchline velocity (fpm) Vt p d  
2

Wt p d  
H
Transmitted load (lbf)
Vt  p d 

3. Set the application factor, Ca Ca  1

B  0.25  12  Qv
0.6667
4. Write the equation for the dynamic load factor, C B  0.52
v

A  50  56 ( 1  B) A  76.878
B
Cv p d    A 
 
 A  Vt  p d  
min

 ft 

5. Tentatively choose the mounting factor, Cm (Assume 2 < F < 6 in Cm  1.7


0.5
6. Choose an elastic coefficient from Table 12-18. (Steel on cast iron). Cp  2100 psi

C a W t  p d   C m
7. The surface stress equation for the pinion is σcp p d F   Cp
Cv p d   F  d p p d   I

8. Determine the endurance strength of the pinion.


 0.056
Life factor CL  2.466  N CL  1
Reliability CR  0.7  0.15 log( 1  R) CR  1
© 2011 Pearson Education, Inc., Upper Saddle River, NJ. All rights reserved. This publication is protected by Copyright and written permission should be
MACHINE DESIGN - An Integrated Approach, 4th Ed. 12-25-2

Temperature factor CT  1
Material surface strength (psi)
S acp  165000 psi
AISI 4140 Nitrided steel
S acp CL
Endurance strength S fcp  S fcp  164997 psi
CT  CR

9. Write the design equations using the range of face-width to diametral-pitch ratio given in the text, and the
bending stress equation, solved for the unknown face width.

FU  p d  
16
Upper limit
pd

FL  p d  
8
Lower limit
pd
2
 Sfc 
Safety factor Nfc =  
 σc 

C a W t  p d   C m
2
 Cp 
Face width F  p d      Nfs
Cv p d   d p p d   I  S fcp 

1 1.5 6
10. Plot F(P) vs. p d over the p d   
range in in in
4

11. From the graph, choose a standard value of


p d from Table 12-2.  
F pd
3
in
1
p d  5  in
 
FL pd
12. The calculated value of F is 2
in
F  p d   2.149  in  
FU pd
in 1
13. Round this up to the decimal equivalent of a
common fractional value.

F  2.250  in 0
2 3 4 5 6
14. Then, the parameters that depend on p d and F are: pd  in
FIGURE 12-25A
d p p d   4.200  in Cv p d   0.785
Graph of Face Width and Limits for the Pinion in
Problem 12-25

Vt p d   2073 Wt p d   1771 lbf


ft
min
σcp p d F   123671 psi
The assumption made in step 5 is correct so no further iteration is required.
Gear
Ng
15. Write the equation for the gear pitch diameter d g p d  
pd
C a W t  p d   C m
16. The surface stress equation for the gear is σcg p d F   Cp
Cv p d   F  d g p d   I
© 2011 Pearson Education, Inc., Upper Saddle River, NJ. All rights reserved. This publication is protected by Copyright and written permission should be
MACHINE DESIGN - An Integrated Approach, 4th Ed. 12-25-3

17. Determine the endurance strength of the pinion.


Material bending strength (psi)
Class 40 cast iron S acg  80000  psi

Hp
Hardness factor Hp  230 Hg  200  1.2
Hg
From equation 12.26, A  0
Ng
Gear ratio mg  mg  1.571
Np

Hardness ratio factor CH  1  A   mg  1  CH  1

S acg CL  CH
Endurance strength S fcg  S fcg  79999  psi
CT  CR

18. Write the design equations using the range of face-width to diametral-pitch ratio given in the text, and the
bending stress equation, solved for the unknown face width.

FU  p d  
16
Upper limit
pd

FL  p d  
8
Lower limit
pd
2
 Sfc 
Safety factor Nfc =  
 σc 
C a W t  p d   C m
2
 Cp 
Face width F  p d      Nfs
Cv p d   d g p d   I  S fcg 

1 1.5 6
19. Plot F(P) vs. p d over the p d   
range in in in
6
20. From the graph, choose a standard value of
p d from Table 12-2.  
F pd
4.5
1 in
p d  4  in
 
FL pd
21. The calculated value of F is 3
in
F  p d   3.807  in
FU pd 
22. Round this up to the decimal equivalent of a in 1.5
common fractional value.

F  4.000  in
0
2 2.5 3 3.5 4 4.5 5 5.5 6
23. Then, the parameters that depend on P and F are:
pd  in
FIGURE 12-25B
d g p d   8.250  in Cv p d   0.768 Graph of Face Width and Limits for the Gear in
Problem 12-25

© 2011 Pearson Education, Inc., Upper Saddle River, NJ. All rights reserved. This publication is protected by Copyright and written permission should be
MACHINE DESIGN - An Integrated Approach, 4th Ed. 12-25-4

Vt p d   2592 Wt p d   1417 lbf σcg p d F   59858  psi


ft
min

24. The gear dimensions are larger (smaller diametral pitch means bigger teeth) than for the pinion. This means
that we will accept the gear requirements for the pinion, thus for the set

1
Diametral pitch p d  4  in

Face width F  4.000  in


25. Determine the realized factor of safety for the gear using the above values for F and p d.
2
 Sfcg 
Gear factor of safety Nfsg  σ  Nfsg  1.8
 cg pd F  
26. Check the factor of safety on the pinion:
2
 Sfcp 
Pinion factor of safety Nfsp    Nfsp  4.8
 σcp pd F  

© 2011 Pearson Education, Inc., Upper Saddle River, NJ. All rights reserved. This publication is protected by Copyright and written permission should be
MACHINE DESIGN - An Integrated Approach, 4th Ed. 12-26-1

PROBLEM 12-26
Statement: Size the spur gears in Problem 12-21 for a surface factor of safety of at least 1.3 assuming a
steady torque, 20 deg pressure angle, full depth teeth, quality index of 11, an AISI 4340 steel
pinion, and an A-7-d nodular iron gear.

Given: Factor of safety Nfs  1.3 Pressure angle ϕ  20 deg


Power to be transmitted (hp) H  39 hp Reliability R  0.99
Number of pinion (sun) teeth Np  23 AGMA Quality level Qv  11
7
Number of gear (planet) teeth Ng  31 Life (cycles) N  10

Solution: See Mathcad file P1226.


Take the sun as the pinion and the planet as the gear, which is also an idler. With a stationary sun, the planet
gear will have one point (where it meshes with the sun) that has an instantaneous zero velocity. All other
points on the planet will have a linear velocity that is proportional to its distance from this instant center. The
point that meshes with the ring gear has maximum velocity. The pitchline velocity at the ring gear is a
function of the tooth numbers and the diametral pitch of the set. But the absolute planet velocity depends
only on the tooth numbers and the arm velocity. For the data of Problem 12-21, the absolute planet (gear)
rotational velocity is

For ωa  2600 rpm ωr  3303.53  rpm

Np  2  Ng
ωg   ωr ωg  9058.1 rpm
Ng

The sun (pinion) is stationary and, therefore, has a zero pitchline velocity. However, the planet (gear) mesh
point rotates about the pitch circle of the sun at the rotational velocity of the arm. Therefore, take the pinion
pitchline velocity to be
Np
Vtp = rp ωa =  ωa
2 pd
The maximum pitchline velocity of the planet (gear) is at the point where it meshes with the ring gear, which is
also the pitchline velocity of the ring gear. Thus, for the gear, the pitchline velocity is

Np  2  Ng
Vtg = rr ωr =  ωr
2 pd
The torques on the arm and ring were determined in Problem 12-21 to be Ta  945.38 in lbf and
Tr  744.05 in lbf . Load analysis of the gearset reveals that the transmitted load on the sun, planet, and ring
gear teeth is
2  p d  Tr
Wt =
Np  2  Ng

Pinion
1. Determine the surface geometry factor, I.

 sin( ϕ)  cos( ϕ)   Ng
I    I  0.0923
 2  Np  Ng
2. Write the equations for pitch diameter, pitchline velocity, and transmitted load in terms of the unknown diametr
pitch, p d. Note that, in Mathcad, unit conversion factors are not included.

Np
Pitch diameter of pinion (in) d p p d  
pd
© 2011 Pearson Education, Inc., Upper Saddle River, NJ. All rights reserved. This publication is protected by Copyright and written permission should be
MACHINE DESIGN - An Integrated Approach, 4th Ed. 12-26-2

Np
pitchline velocity (fpm) Vtp p d    ωa
2 pd

2  p d  Tr
Transmitted load (lbf) Wt p d  
Np  2  Ng

3. Set the application factor, Ca Ca  1

B  0.25  12  Qv
0.6667
4. Write the equation for the dynamic load factor, C B  0.25
v

A  50  56 ( 1  B) A  92
B
Cvp p d    A 
 
 A  Vtp pd  
min

 ft 

5. Tentatively choose the mounting factor, Cm (Assume 0 < F < 2 in) Cm  1.6
0.5
6. Choose an elastic coefficient from Table 12-18. Cp  2100 psi

Ca Wt p d   Cm
7. The surface stress equation for the pinion is σcp p d F   Cp
Cvp p d   F  d p p d   I

8. Determine the endurance strength of the pinion.


 0.056
Life factor CL  2.466  N CL  1
Reliability CR  0.7  0.15 log( 1  R) CR  1
Temperature factor CT  1
Material surface strength (psi)
S acp  162000 psi
AISI 4340 Nitrided steel
S acp CL
Endurance strength S fcp  S fcp  161997 psi
CT  CR

9. Write the design equations using the range of face-width to diametral-pitch ratio given in the text, and the
bending stress equation, solved for the unknown face width.

FU  p d  
16
Upper limit
pd

FL  p d  
8
Lower limit
pd
2
 Sfc 
Safety factor Nfc =  
 σc 
C a W t  p d   C m
2
 Cp 
Face width F  p d      Nfs
Cvp p d   d p p d   I  S fcp 

© 2011 Pearson Education, Inc., Upper Saddle River, NJ. All rights reserved. This publication is protected by Copyright and written permission should be
MACHINE DESIGN - An Integrated Approach, 4th Ed. 12-26-3

6 6.5 16
10. Plot F(P) vs. p over the range p d   
d in in in

11. From the graph, choose a standard value of 2


p d from Table 12-2.

p d  14 in
1  
F pd
1.5
in
12. The calculated value of F is
 
FL pd
F  p d   0.611  in 1
in
13. Round this up to the decimal equivalent of a  
FU pd
common fractional value.
in 0.5
F  0.625  in

14. Then, the parameters that depend on p d and F are: 0


6 8 10 12 14 16
d p p d   1.643  in Cvp p d   0.925 pd  in
FIGURE 12-26A
Graph of Face Width and Limits for the Pinion in
Vtp p d   1118 Wt p d   245  lbf
ft
Problem 12-26
min
σcp p d F   140460 psi

The assumption made in step 5 is correct so no further iteration is required.


Gear
15. Write the equations for pitch diameter and pitchline velocity for the gear in terms of the unknown diametral pitc
p d. Note that, in Mathcad, unit conversion factors are not included.

Ng
Pitch diameter of pinion (in) d g p d  
pd
Np  Ng
pitchline velocity (fpm) Vtg p d    ωr
2 pd

B
Cvg p d    
A
16. Write the equation for the dynamic load factor, C 
 A  Vtg pd  
min
v

 ft 

Ca Wt p d   Cm
17. The surface stress equation for the gear is σcg p d F   Cp
Cvg p d   F  d g p d   I
18. Determine the endurance strength of the pinion.
Material bending strength (psi)
A-7-d nodular iron S acg  97000  psi

Hp
Hardness factor Hp  235 Hg  230  1.0
Hg
From equation 12.26, A  0

© 2011 Pearson Education, Inc., Upper Saddle River, NJ. All rights reserved. This publication is protected by Copyright and written permission should be
MACHINE DESIGN - An Integrated Approach, 4th Ed. 12-26-4

Ng
Gear ratio mg  mg  1.348
Np

Hardness ratio factor CH  1  A   mg  1  CH  1

S acg CL  CH
Endurance strength S fcg  S fcg  96998  psi
CT  CR

19. Write the design equations using the range of face-width to diametral-pitch ratio given in the text, and the
bending stress equation, solved for the unknown face width.

FU  p d  
16
Upper limit
pd

FL  p d  
8
Lower limit
pd
2
 Sfc 
Safety factor Nfc =  
 σc 
C a W t  p d   C m
2
 Cp 
Face width F  p d      Nfs
Cvg p d   d g p d   I  S fcg 

6 6.5 16
20. Plot F(P) vs. p d over the p d   
range in in in

21. From the graph, choose a standard value of 3


p d from Table 12-2.

1  
F pd 2.5
p d  12 in
in
2
22. The calculated value of F is  
FL pd
F  p d   0.978  in in 1.5

23. Round this up to the decimal equivalent of a  


FU pd 1
common fractional value. in
0.5
F  1.000  in

24. Then, the parameters that depend on P and F are: 0


6 8 10 12 14 16
d g p d   2.583  in Cvg p d   0.879 pd  in

Vtg p d   3892 Wt p d   210  lbf


ft FIGURE 12-26B
min Graph of Face Width and Limits for the Gear in
Problem 12-26
σcg p d F   84140  psi
25. The gear dimensions are larger than that for the pinion. Thus, for the set
1
Diametral pitch p d  12 in
Face width F  1.000  in

26. Determine the realized factor of safety for the gear using the above values for F and p d.

© 2011 Pearson Education, Inc., Upper Saddle River, NJ. All rights reserved. This publication is protected by Copyright and written permission should be
MACHINE DESIGN - An Integrated Approach, 4th Ed. 12-26-5

2
 Sfcg 
Gear factor of safety Nfsg   σ p F  Nfsg  1.3
 cg d  
27. Check the factor of safety on the pinion:
2
 Sfcp 
Pinion factor of safety Nfsp    Nfsp  2.9
 σcp pd F  

© 2011 Pearson Education, Inc., Upper Saddle River, NJ. All rights reserved. This publication is protected by Copyright and written permission should be
MACHINE DESIGN - An Integrated Approach, 4th Ed. 12-27-1

PROBLEM 12-27
Statement: If the gearset in Problem 12-10 transmits 190 kW at 1800 input pinion rpm, find the torque on each
of the four shafts.
Given: Transmitted power P  190  kW Input speed ωp1  1800 rpm

Assumptions: Tooth numbers:


Stage 1 Np1  18 Ng1  164
Stage 2 Np2  18 Ng2  144
Stage 3 Np3  18 Ng3  162
Solution: See Mathcad file P1227.

1. Confirm that the gear ratio is correct.

Ng1 Ng2 Ng3


mG  mG  656.000
Np1 Np2 Np3

2. Calculate the gear speeds.

Np1
Stage 1 ωp1  1800 rpm ωg1   ωp1 ωg1  197.561  rpm
Ng1

Np2
Stage 2 ωp2  ωg1 ωg2   ωp2 ωg2  24.695 rpm
Ng2

Np3
Stage 3 ωp3  ωg2 ωg3   ωp3 ωg3  2.744  rpm
Ng3

3. Calculate the shaft torque.


P
Shaft 1 T1  T1  1008 N  m Input
ωp1

P
Shaft 2 T2  T2  9184 N  m
ωg1

P
Shaft 3 T3  T3  73471  N  m
ωg2

P
Shaft 4 T4  T4  661236 N  m Output
ωg3

© 2011 Pearson Education, Inc., Upper Saddle River, NJ. All rights reserved. This publication is protected by Copyright and written permission should be
MACHINE DESIGN - An Integrated Approach, 4th Ed. 12-28-1

PROBLEM 12-28
Statement: Size the first-stage spur gears in problem 12-27 for a bending factor of safety of at least 3.2 and
a surface factor of safety of at least 1.7 assuming a steady torque, 25 deg pressure angle, full
depth teeth, Qv = 8, and AISI 4140 steel for all gears.

Given: Pressure angle ϕ  25 deg


Bending factor of safety Nfb  3.2 Reliability R  0.99
Surface factor of safety Nfs  1.7 AGMA Quality level Qv  8
7
Power to be transmitted (hp) H  190  kW Life (cycles) N  10
Number of pinion teeth Np  18 Number of gear teeth Ng  164
Rotational speed of pinion (rpm) n  1800 rpm

Assumptions: Since both pinion and gear from each stage are the same material, it will only be necessary to
determine the pinion size as it will be governing for the set.
Solution: See Mathcad file P1228.

Stage 1 Pinion - Bending


1. Determine the bending geometry factor, J (Table 12-13) Jp  0.38

2. Write the equations for pitch diameter, pitchline velocity, and transmitted load in terms of the unknown
diametral pitch, pd. Note that, in Mathcad, unit conversion factors are not included.

Np Ng
Pitch diameter of pinion and gear (in) d p p d   d g p d  
pd pd

d p p d   n
pitchline velocity (fpm) Vt p d  
2

Wt p d  
H
Transmitted load (lbf)
Vt  p d 

3. Set the application factor, Ka Ka  1

4. Write the equation for the dynamic load factor, Kv

B  0.25  12  Qv
0.6667
A  50  56 ( 1  B) A  70.721
B
B  0.63
Kv p d    A 
 
 A  Vt  p d  
min

 ft 

5. Tentatively choose the mounting factor, Km (Assume 2 < F < 6 in) Km  1.7

6. The bending stress equation for the pinion is

Ka Wt p d   p d  Km
σbp p d F  
Kv p d   F  Jp

7. Determine the endurance strength of the pinion.


 0.0323
Life factor KL  1.6831 N KL  1

© 2011 Pearson Education, Inc., Upper Saddle River, NJ. All rights reserved. This publication is protected by Copyright and written permission should be
MACHINE DESIGN - An Integrated Approach, 4th Ed. 12-28-2

Reliability KR  0.7  0.15 log( 1  R) KR  1


Temperature factor KT  1
Material bending strength (psi)
S atp  40000  psi
AISI 4140 Nitrided steel
S atp KL
Endurance strength S fbp  S fbp  40001  psi
KT  KR

8. Write the design equations using the range of face-width to diametral-pitch ratio given in the text, and the
bending stress equation, solved for the unknown face width.

FU  p d  
16
Upper limit
pd

FL  p d  
8
Lower limit
pd
S fb
Safety factor Nfb =
σb

Ka Wt p d   p d  Km Nfb


Face width F  p d  
Kv p d   Jp S fbp

1 1.1 5
9. Plot F(P) vs. p over the range p d   
d in in in

10. From the graph, choose a standard value of 8


p d from Table 12-2.

p d  3  in
1  
F pd
6
in
11. The calculated value of F is
 
FL pd
F  p d   4.546  in 4
in
12. Round this up to the decimal equivalent of  
FU pd
a common fractional value.
in 2
F  4.500  in

13. Then, the parameters that depend on p d and F are: 0


2 2.5 3 3.5 4
pd  in
d p p d   6.000  in d g p d   54.667 in
FIGURE 12-28A
Vt p d   2827 Wt p d   2974 lbf
ft Graph of Face Width and Limits for the Stage 1
min Pinion (bending) in Problem 12-28

σbp p d F   12627  psi Kv p d   0.702

The assumption made in step 5 is correct so no further iteration is required.

Stage 1 Pinion - Surface Fatigue


1. Determine the surface geometry factor, I.

© 2011 Pearson Education, Inc., Upper Saddle River, NJ. All rights reserved. This publication is protected by Copyright and written permission should be
MACHINE DESIGN - An Integrated Approach, 4th Ed. 12-28-3

 sin( ϕ)  cos( ϕ)   Ng
I    I  0.1726
 2  Np  Ng
2. Write the equations for pitch diameter, pitchline velocity, and transmitted load in terms of the unknown
diametral pitch, pd. Note that, in Mathcad, unit conversion factors are not included.

Np
Pitch diameter of pinion (in) d p p d  
pd

d p p d   n
pitchline velocity (fpm) Vt p d  
2

Wt p d  
H
Transmitted load (lbf)
Vt  p d 

3. Set the application factor, Ca Ca  1

4. Write the equation for the dynamic load factor, Cv

B  0.25  12  Qv
0.6667
A  50  56 ( 1  B) A  70.721
B
Cv p d    
B  0.63 A

 A  Vt  p d  
min

 ft 

5. Tentatively choose the mounting factor, Cm (Assume 2 < F < 6 in) Cm  1.7
0.5
6. Choose an elastic coefficient from Table 12-18. (Steel on steel). Cp  2300 psi

7. The surface stress equation for the pinion is

C a W t  p d   C m
σcp p d F   Cp
Cv p d   F  d p p d   I

8. Determine the endurance strength of the pinion.


 0.056
Life factor CL  2.466  N CL  1
Reliability CR  0.7  0.15 log( 1  R) CR  1
Temperature factor CT  1
Material surface strength (psi)
S acp  165000 psi
AISI 4140 Nitrided steel
S acp CL
Endurance strength S fcp  S fcp  164997 psi
CT  CR

9. Write the design equations using the range of face-width to diametral-pitch ratio given in the text, and the
bending stress equation, solved for the unknown face width.

FU  p d  
16
Upper limit
pd

© 2011 Pearson Education, Inc., Upper Saddle River, NJ. All rights reserved. This publication is protected by Copyright and written permission should be
MACHINE DESIGN - An Integrated Approach, 4th Ed. 12-28-4

FL  p d  
8
Lower limit
pd
2
 Sfc 
Safety factor Nfc = σ 
 c
C a W t  p d   C m
2
 Cp 
Face width F  p d      Nfs
Cv p d   d p p d   I  S fcp 

1 1.1 6
10. Plot F(P) vs. p over the range p d   
d in in in
6
11. From the graph, choose a standard value of
p d from Table 12-2.
 
F pd
1
p d  4  in in
4
12. The calculated value of F is  
FL pd

F  p d   3.933  in
in

 
FU pd 2
13. Round this up to the decimal equivalent of
a common fractional value. in

F  4.000  in
0
14. The bending requirement is governing in 2 3 4 5 6
this case. The diametral pitch and face pd  in
width for the stage 2 gearset are:
FIGURE 12-28B
1
p d  3  in F  4.500  in Graph of Face Width and Limits for the Stage 1
Pinion (surface) in Problem 12-28

© 2011 Pearson Education, Inc., Upper Saddle River, NJ. All rights reserved. This publication is protected by Copyright and written permission should be
MACHINE DESIGN - An Integrated Approach, 4th Ed. 12-29-1

PROBLEM 12-29
Statement: Size the second-stage spur gears in problem 12-27 for a bending factor of safety of at least 3.2
and a surface factor of safety of at least 1.7 assuming a steady torque, 25 deg pressure angle,
full depth teeth, quality index of 8, and AISI 4140 steel for all gears.

Given: Pressure angle ϕ  25 deg


Bending factor of safety Nfb  3.2 Reliability R  0.99
Surface factor of safety Nfs  1.7 AGMA Quality level Qv  8
7
Power to be transmitted H  190  kW Life (cycles) N  10
Number of pinion teeth Np  18 Number of gear teeth Ng  144
Rotational speed of pinion (rpm) n  197.561  rpm

Assumptions: Since both pinion and gear from each stage are the same material, it will only be necessary to
determine the pinion size as it will be governing for the set.
Solution: See Mathcad file P1229.
Stage 2 Pinion - Bending
1. Determine the bending geometry factor, J (Table 12-13) Jp  0.38

2. Write the equations for pitch diameter, pitchline velocity, and transmitted load in terms of the unknown
diametral pitch, p d. Note that, in Mathcad, unit conversion factors are not included.

Np Ng
Pitch diameter of pinion and gear (in) d p p d   d g p d  
pd pd

d p p d   n
pitchline velocity (fpm) Vt p d  
2

Wt p d  
H
Transmitted load (lbf)
Vt  p d 

3. Set the application factor, Ka Ka  1

4. Write the equation for the dynamic load factor, Kv

B  0.25  12  Qv
0.6667
A  50  56 ( 1  B) A  70.721
B
B  0.63
Kv p d    A 
 
 A  Vt  p d  
min

 ft 

5. Tentatively choose the mounting factor, Km (Assume 6 < F < 9 in) Km  1.8

6. The bending stress equation for the pinion is

Ka Wt p d   p d  Km
σbp p d F  
Kv p d   F  Jp

7. Determine the endurance strength of the pinion.

 0.0323
Life factor KL  1.6831 N KL  1
© 2011 Pearson Education, Inc., Upper Saddle River, NJ. All rights reserved. This publication is protected by Copyright and written permission should be
MACHINE DESIGN - An Integrated Approach, 4th Ed. 12-29-2

Reliability KR  0.7  0.15 log( 1  R) KR  1


Temperature factor KT  1
Material bending strength (psi)
S atp  40000  psi
AISI 4140 Nitrided steel
S atp KL
Endurance strength S fbp  S fbp  40001  psi
KT  KR

8. Write the design equations using the range of face-width to diametral-pitch ratio given in the text, and the
bending stress equation, solved for the unknown face width.

FU  p d  
16
Upper limit
pd

FL  p d  
8
Lower limit
pd
S fb
Safety factor Nfb =
σb

Ka Wt p d   p d  Km Nfb


Face width F  p d  
Kv p d   Jp S fbp

1 1.1 3
9. Plot F(P) vs. p over the range p d   
d in in in

10. From the graph, choose a standard value of


p d from Table 12-2.
20
1
p d  1.5 in
 
F pd
11. The calculated value of F is 15
in
F  p d   9.313  in
 
FL pd
12. Round this up to the decimal equivalent of 10
in
a common fractional value.
 
FU pd
F  9.375  in
in 5
13. Then, the parameters that depend on p d and F are:

0
d p p d   12.000 in d g p d   96.000 in 1 1.5 2 2.5
pd  in

Vt p d   621  Wt p d   13547  lbf


ft
FIGURE 12-29A
min Graph of Face Width and Limits for the Stage 2
σbp p d F   12417  psi Kv p d   0.827 Pinion (bending) in Problem 12-29

The assumption made in step 5 is correct so no further iteration is required.

Stage 2 Pinion - Surface Fatigue


1. Determine the surface geometry factor, I.

© 2011 Pearson Education, Inc., Upper Saddle River, NJ. All rights reserved. This publication is protected by Copyright and written permission should be
MACHINE DESIGN - An Integrated Approach, 4th Ed. 12-29-3

 sin( ϕ)  cos( ϕ)   Ng
I    I  0.1702
 2  Np  Ng
2. Write the equations for pitch diameter, pitchline velocity, and transmitted load in terms of the unknown
diametral pitch, p d. Note that, in Mathcad, unit conversion factors are not included.

Np
Pitch diameter of pinion (in) d p p d  
pd

d p p d   n
pitchline velocity (fpm) Vt p d  
2

Wt p d  
H
Transmitted load (lbf)
Vt  p d 

3. Set the application factor, Ca Ca  1

4. Write the equation for the dynamic load factor, Cv

B  0.25  12  Qv
0.6667
A  50  56 ( 1  B) A  70.721
B
Cv p d    
B  0.63 A

 A  Vt  p d  
min

 ft 

5. Tentatively choose the mounting factor, Cm (Assume 6 < F < 9 in) Cm  1.8

0.5
6. Choose an elastic coefficient from Table 12-18. (Steel on steel). Cp  2300 psi

7. The surface stress equation for the pinion is

C a W t  p d   C m
σcp p d F   Cp
Cv p d   F  d p p d   I

8. Determine the endurance strength of the pinion.


 0.056
Life factor CL  2.466  N CL  1
Reliability CR  0.7  0.15 log( 1  R) CR  1
Temperature factor CT  1
Material surface strength (psi)
S acp  165000 psi
AISI 4140 Nitrided steel
S acp CL
Endurance strength S fcp  S fcp  164997 psi
CT  CR

9. Write the design equations using the range of face-width to diametral-pitch ratio given on page 740 of the text,
and the bending stress equation, solved for the unknown face width.

FU  p d  
16
Upper limit
pd
© 2011 Pearson Education, Inc., Upper Saddle River, NJ. All rights reserved. This publication is protected by Copyright and written permission should be
MACHINE DESIGN - An Integrated Approach, 4th Ed. 12-29-4

FL  p d  
8
Lower limit
pd
2
 Sfc 
Safety factor Nfc =  
 σc 
C a W t  p d   C m
2
 Cp 
Face width F  p d      Nfs
Cv p d   d p p d   I  S fcp 

1 1.1 3
10. Plot F(P) vs. p over the range p d   
d in in in
20
11. From the graph, choose a standard value of
p d from Table 12-2.
 
F pd
1 15
p d  2  in in

12. The calculated value of F is  


FL pd
10
F  p d   8.290  in
in

 
FU pd
13. Round this up to the decimal equivalent of
in 5
a common fractional value.

F  8.250  in
0
14. The bending requirement is governing in 1 1.5 2 2.5 3
this case. The diametral pitch and face pd  in
width for the stage 2 gearset are:
FIGURE 12-29B
1 Graph of Face Width and Limits for the Stage 2
p d  1.5 in F  9.375  in Pinion (surface) in Problem 12-29

© 2011 Pearson Education, Inc., Upper Saddle River, NJ. All rights reserved. This publication is protected by Copyright and written permission should be
MACHINE DESIGN - An Integrated Approach, 4th Ed. 12-30-1

PROBLEM 12-30
Statement: Size the third-stage spur gears in problem 12-27 for a bending factor of safety of at least 3.2 and a
surface factor of safety of at least 1.7 assuming a steady torque, 25 deg pressure angle, full depth
teeth, quality index of 8, and AISI 4140 steel for all gears.

Given: Pressure angle ϕ  25 deg


Bending factor of safety Nfb  3.2 Reliability R  0.99
Surface factor of safety Nfs  1.7 AGMA Quality level Qv  8
7
Power to be transmitted H  190  kW Life (cycles) N  10
Number of pinion teeth Np  18 Number of gear teeth Ng  162
Rotational speed of pinion n  24.695 rpm
Assumptions: Since both pinion and gear from each stage are the same material, it will only be necessary to
determine the pinion size as it will be governing for the set.
Solution: See Mathcad file P1230.

Stage 3 Pinion - Bending

1. Determine the bending geometry factor, J (Table 12-13) Jp  0.38

2. Write the equations for pitch diameter, pitchline velocity, and transmitted load in terms of the unknown
diametral pitch, p d. Note that, in Mathcad, unit conversion factors are not included.

Np Ng
Pitch diameter of pinion and gear (in) d p p d   d g p d  
pd pd

d p p d   n
pitchline velocity (fpm) Vt p d  
2

Wt p d  
H
Transmitted load (lbf)
Vt  p d 

3. Set the application factor, Ka Ka  1

4. Write the equation for the dynamic load factor, Kv

B  0.25  12  Qv
0.6667
A  50  56 ( 1  B) A  70.721
B
B  0.63
Kv p d    A 
 
 A  Vt  p d  
min

 ft 

5. Tentatively choose the mounting factor, Km (Assume 9 < F < 20 in) Km  2.0

6. The bending stress equation for the pinion is

Ka Wt p d   p d  Km
σbp p d F  
Kv p d   F  Jp

7. Determine the endurance strength of the pinion.


 0.0323
Life factor KL  1.6831 N KL  1

© 2011 Pearson Education, Inc., Upper Saddle River, NJ. All rights reserved. This publication is protected by Copyright and written permission should be
MACHINE DESIGN - An Integrated Approach, 4th Ed. 12-30-2

Reliability KR  0.7  0.15 log( 1  R) KR  1


Temperature factor KT  1
Material bending strength (psi)
S atp  40000  psi
AISI 4140 Nitrided steel
S atp KL
Endurance strength S fbp  S fbp  40001  psi
KT  KR

8. Write the design equations using the range of face-width to diametral-pitch ratio given in the text, and the
bending stress equation, solved for the unknown face width.

FU  p d  
16
Upper limit
pd

FL  p d  
8
Lower limit
pd
S fb
Safety factor Nfb =
σb

Ka Wt p d   p d  Km Nfb


Face width F  p d  
Kv p d   Jp S fbp

0.5 0.6 2
9. Plot F(P) vs. p over the range p d   
d in in in

10. From the graph, choose a standard value of 40


p d from Table 12-2.

p d  0.75 in
1  
F pd
30
in
11. The calculated value of F is
 
FL pd
F  p d   18.954 in 20
in
12. Round this up to the decimal equivalent of  
FU pd
a common fractional value.
in 10
F  19.000 in

13. Then, the parameters that depend on p d and F are: 0


0.5 0.75 1 1.25 1.5 1.75 2
pd  in
d p p d   24.000 in d g p d   216.000  in
FIGURE 12-30A
Vt p d   155  Wt p d   54189  lbf
ft Graph of Face Width and Limits for the Stage 3
min Pinion (bending) in Problem 12-30

σbp p d F   12470  psi Kv p d   0.903

The assumption made in step 5 is correct so no further iteration is required.

Stage 3 Pinion - Surface Fatigue


1. Determine the surface geometry factor, I.

© 2011 Pearson Education, Inc., Upper Saddle River, NJ. All rights reserved. This publication is protected by Copyright and written permission should be
MACHINE DESIGN - An Integrated Approach, 4th Ed. 12-30-3

 sin( ϕ)  cos( ϕ)   Ng
I    I  0.1724
 2  Np  Ng
2. Write the equations for pitch diameter, pitchline velocity, and transmitted load in terms of the unknown
diametral pitch, p d. Note that, in Mathcad, unit conversion factors are not included.

Np
Pitch diameter of pinion (in) d p p d  
pd

d p p d   n
pitchline velocity (fpm) Vt p d  
2

Wt p d  
H
Transmitted load (lbf)
Vt  p d 

3. Set the application factor, Ca Ca  1

4. Write the equation for the dynamic load factor, Cv

B  0.25  12  Qv
0.6667
A  50  56 ( 1  B) A  70.721
B
B  0.63
Cv p d    A 
 
 A  Vt  p d  
min

 ft 

5. Tentatively choose the mounting factor, Cm (Assume 9 < F < 20 in) Cm  2.0
0.5
6. Choose an elastic coefficient from Table 12-18. (Steel on steel). Cp  2300 psi

7. The surface stress equation for the pinion is

C a W t  p d   C m
σcp p d F   Cp
Cv p d   F  d p p d   I

8. Determine the endurance strength of the pinion.


 0.056
Life factor CL  2.466  N CL  1
Reliability CR  0.7  0.15 log( 1  R) CR  1
Temperature factor CT  1
Material surface strength (psi)
S acp  165000 psi
AISI 4140 Nitrided steel
S acp CL
Endurance strength S fcp  S fcp  164997 psi
CT  CR

9. Write the design equations using the range of face-width to diametral-pitch ratio given in the text, and the
bending stress equation, solved for the unknown face width.

FU  p d  
16
Upper limit
pd
© 2011 Pearson Education, Inc., Upper Saddle River, NJ. All rights reserved. This publication is protected by Copyright and written permission should be
MACHINE DESIGN - An Integrated Approach, 4th Ed. 12-30-4

FL  p d  
8
Lower limit
pd
2
 Sfc 
Safety factor Nfc = σ 
 c
C a W t  p d   C m
2
 Cp 
Face width F  p d      Nfs
Cv p d   d p p d   I  S fcp 

0.5 0.6 2
10. Plot F(P) vs. p over the range p d   
d in in in
40
11. From the graph, choose a standard value of
p d from Table 12-2.
 
F pd
1 30
p d  1  in in

12. The calculated value of F is  


FL pd
20
F  p d   16.825 in
in

FU pd 
13. Round this up to the decimal equivalent of
in 10
a common fractional value.

F  16.875 in
0
14. The bending requirement is governing in 0.5 0.75 1 1.25 1.5 1.75 2
this case. The diametral pitch and face pd  in
width for the stage 2 gearset are: FIGURE 12-30B
Graph of Face Width and Limits for the Stage 3
1
p d  0.75 in F  19.000 in Pinion (surface) in Problem 12-30

© 2011 Pearson Education, Inc., Upper Saddle River, NJ. All rights reserved. This publication is protected by Copyright and written permission should be
MACHINE DESIGN - An Integrated Approach, 4th Ed. 12-31-1

PROBLEM 12-31
Statement: Design a two-stage compound spur gear train for an overall ratio of approximately 78:1. Specify
tooth numbers for each gear in the train.
Given: Approximate ratio mG  78 Number of stages n  2
Assumptions: The gears will be cut with a hob and the pressure angle will be ϕ  25 deg
Solution: See Mathcad file P1231.
1. For equal stages, the stage ratio is
1
n
mGs  mG mGs  8.832

2. The minimum number of teeth that we can have without interference on a 25-deg gear cut with a hob is 12. Try
pinions with 12, 13, 14, etc. teeth to see if the mating gear will have close to an integral number of teeth.

12 mGs  105.981 15 mGs  132.476

13 mGs  114.813 16 mGs  141.308

14 mGs  123.645 17 mGs  150.14

3. The first trial is very close to an integer so try 106 teeth:


n
 106 
mG   12  mG  78.028
 

4. We can get the exact ratio by using 12-tooth and 16-tooth pinions with 104- and 144-tooth gears

Let Stage 1 Np1  12 Ng1  104

Stage 2 Np2  16 Ng2  144

Ng1 Ng2
Then, the gear ratio is mG  mG  78.000
Np1 Np2

© 2011 Pearson Education, Inc., Upper Saddle River, NJ. All rights reserved. This publication is protected by Copyright and written permission should be
MACHINE DESIGN - An Integrated Approach, 4th Ed. 12-32-1

PROBLEM 12-32
Statement: Figure P12-1 shows the same paper machine that was analyzed in Problem 6-46 and in other
problems in previous chapters. Using the data of Problem 4-46, design a suitable spur-gear set
for this application for a 10-yr life against surface failure. State all assumptions.

Units: yr  2080 hr

Given: Gear ratio mg  2 Life (years) Life  10 yr


Torque on gear Tg  74240  in lbf

Assumptions: 1. If both pinion and gear are the same material, it will only be necessary to determine the pinion
size as it will be governing for the set.
2. If the gears are not surface hardened, it will only be necessary to design to the surface
requirement as it will be governing for both bending and surface stresses.
3. Pinion speed is n p  3  rpm (5 seconds for a 90-deg rotation).

Design Choices:
Pressure angle ϕ  25 deg AGMA Quality level Qv  8
Number of pinion teeth Np  26 Reliability R  0.99
Material: AGMA Grade 2 steel for both pinion and gear, through hardened to
HB  300 S ac  ( 27000  364  HB)  psi S ac  136200 psi
Surface factor of safety Nfs  2 Number of shifts shifts  3

Solution: See Mathcad file P1232.


1. Determine the number of gear teeth and the cycle life.
Number of gear teeth Ng  Np mg Ng  52

np 7
Cycle life: N  Life shifts N  1.123  10
2 π
2. Determine the surface geometry factor, I.

 sin( ϕ)  cos( ϕ)   Ng
I    I  0.1277
 2  Np  Ng
3. Write the equations for pitch diameter, pitchline velocity, and transmitted load in terms of the unknown
diametral pitch, p d. Note that, in Mathcad, unit conversion factors are not included.
Np Ng
Pitch diameter of pinion (in) d p p d   d g p d  
pd pd
d p p d   n p
pitchline velocity (fpm) Vt p d  
2
2  Tg
Transmitted load (lbf) Wt p d  
d g p d 

4. Set the application factor, Ca Ca  1


B  0.25  12  Qv
0.6667
5. Write the equation for the dynamic load factor, C B  0.63
v

A  50  56 ( 1  B) A  70.721

© 2011 Pearson Education, Inc., Upper Saddle River, NJ. All rights reserved. This publication is protected by Copyright and written permission should be
MACHINE DESIGN - An Integrated Approach, 4th Ed. 12-32-2
B
Cv p d    
A

 A  Vt  p d  
min

 ft 

6. Tentatively choose the mounting factor, Cm (Assume 2 < F < 6 in) Cm  1.7
0.5
7. Choose an elastic coefficient from Table 12-18 (steel on steel). Cp  2300 psi

C a W t  p d   C m
8. The surface stress equation for the pinion is σcp p d F   Cp
Cv p d   F  d p p d   I
9. Determine the endurance strength of the pinion.
 0.056
Life factor CL  2.466  N CL  0.993
Reliability CR  0.7  0.15 log( 1  R) CR  1
Temperature factor CT  1
Material surface strength (psi)
S ac  136200 psi
Grade 2, 300 HB
S ac CL
Endurance strength S fcp  S fcp  135315 psi
CT  CR

10. Write the design equations using the range of face-width to diametral-pitch ratio given in the text, and the
bending stress equation, solved for the unknown face width.
2
Limits and safety factor FU  p d  
 Sfc 
FL  p d  
16 8
Nfc = σ 
pd pd  c
C a W t  p d   C m
2
 Cp 
Face width F  p d      Nfs
Cv p d   d p p d   I  S fcp 

1 1.5 10
9. Plot F(P) vs. p d over the p d   
range in in in
8

10. From the graph, choose a standard value of


p d from Table 12-2.  
F pd
6
in
1
p d  2.5 in
 
FL pd
11. The calculated value of F is 4
in
F  p d   5.414  in  
FU pd
in 2
12. Round this up to the decimal equivalent of a
common fractional value.

F  5.500  in 0
1 1.5 2 2.5 3 3.5 4
13. Then, the parameters that depend on p d and F are:
pd  in

d p p d   10.400 in d g p d   20.800 in FIGURE 12-32


Graph of Face Width and Limits for the Pinion
(surface) in Problem 12-32

© 2011 Pearson Education, Inc., Upper Saddle River, NJ. All rights reserved. This publication is protected by Copyright and written permission should be
MACHINE DESIGN - An Integrated Approach, 4th Ed. 12-32-3

Vt  p d   8  Wt p d   7138 lbf σcp p d F   94935  psi


ft
min
14. There is a wide range of choice for the number of teeth on the pinion. The total weight of the gears goes down
very slightly with increasing pinion tooth number. The choice of Np  26 was somewhat arbitrary but was
arrived at after trying values from 18 to 26.

© 2011 Pearson Education, Inc., Upper Saddle River, NJ. All rights reserved. This publication is protected by Copyright and written permission should be
MACHINE DESIGN - An Integrated Approach, 4th Ed. 12-33-1

PROBLEM 12-33
Statement: Design a non reverted compound transmission based on the arrangement shown in Figure
12-14a for an overall train ratio of approximately 90:1. It should be capable of transmitting 50
hp at 1000 rpm input shaft speed. State all assumptions.

Given: Power to be transmitted H  50 hp Approximate gear ratio mv  90


Input speed (rpm) n in  1000 rpm Number of stages n  2

Assumptions: 1. If both pinion and gear from each stage are the same material, it will only be necessary to
determine the pinion size as it will be governing for the set.
2. If the gears are not surface hardened, it will only be necessary to design to the surface
requirement as it will be governing for both bending and surface stresses.

Design Choices:
Pressure angle ϕ  25 deg AGMA Quality level Qv  8
Surface factor of safety Nfs  1.7 Reliability R  0.99
7
Life (cycles) N  10
Material: AGMA Grade 2 steel for both pinion and gear, both stages, through hardened to
HB  300 S ac  ( 27000  364  HB)  psi S ac  136200 psi
Solution: See Mathcad file P1233.
1. For equal stage ratios, the stage ratio is
1
n
mvs  mv mvs  9.487

2. The minimum number of teeth that we can have without interference on a 25-deg gear cut with a hob is 14. Try
pinions with 14, 15, 16, etc. teeth to see if the mating gear will have close to an integral number of teeth.

14 mvs  132.816 20 mvs  189.737 26 mvs  246.658 32 mvs  303.579

15 mvs  142.302 21 mvs  199.223 27 mvs  256.144 33 mvs  313.065

16 mvs  151.789 22 mvs  208.71 28 mvs  265.631 34 mvs  322.552

17 mvs  161.276 23 mvs  218.197 29 mvs  275.118 35 mvs  332.039

18 mvs  170.763 24 mvs  227.684 30 mvs  284.605 36 mvs  341.526

19 mvs  180.25 25 mvs  237.171 31 mvs  294.092 37 mvs  351.013

3. The last trial is very close to an integer so try 351 teeth:


n
 351 
mv   37  mv  89.993
 

4. For each stage let Np  37 Ng  351


Stage 1 Pinion
Given: Rotational speed of pinion (rpm) n 1  n in n 1  1000 rpm

1. Determine the surface geometry factor, I.

 sin( ϕ)  cos( ϕ)   Ng
I    I  0.1732
 2  Np  Ng
© 2011 Pearson Education, Inc., Upper Saddle River, NJ. All rights reserved. This publication is protected by Copyright and written permission should be
MACHINE DESIGN - An Integrated Approach, 4th Ed. 12-33-2

2. Write the equations for pitch diameter, pitch-line velocity, and transmitted load in terms of the unknown
diametral pitch, p d. Note that, in Mathcad, unit conversion factors are not included.
Np Ng
Pitch diameter of pinion (in) d p p d   d g p d  
pd pd

d p p d   n 1
Pitch-line velocity (fpm) Vt p d  
2

Wt p d  
H
Transmitted load (lbf)
Vt  p d 

3. Set the application factor, Ca Ca  1

4. Write the equation for the dynamic load factor, Cv

B  0.25  12  Qv
0.6667
A  50  56 ( 1  B) A  70.721
B
B  0.63 Cv p d    A 
 
 A  Vt  p d  
min

 ft 

5. Tentatively choose the mounting factor, Cm (Assume 0 < F < 2 in) Cm  1.6
0.5
6. Choose an elastic coefficient from Table 12-18. Cp  2300 psi

7. The surface stress equation for the pinion is

C a W t  p d   C m
σcp p d F   Cp
Cv p d   F  d p p d   I

8. Determine the endurance strength of the pinion.


 0.056
Life factor CL  2.466  N CL  1
Reliability CR  0.7  0.15 log( 1  R) CR  1
Temperature factor CT  1
Material surface strength (psi)
S ac  136200 psi
AISI 4140 Nitrided steel
S ac CL
Endurance strength S fcp  S fcp  136198 psi
CT  CR

9. Write the design equations using the range of face-width to diametral-pitch ratio given in the text, and the
bending stress equation, solved for the unknown face width.

FU  p d  
16
Upper limit
pd

FL  p d  
8
Lower limit
pd

© 2011 Pearson Education, Inc., Upper Saddle River, NJ. All rights reserved. This publication is protected by Copyright and written permission should be
MACHINE DESIGN - An Integrated Approach, 4th Ed. 12-33-3

2
 Sfc 
Safety factor Nfc =  
 σc 
C a W t  p d   C m
2
 Cp 
Face width F  p d      Nfs
Cv p d   d p p d   I  S fcp 

6 6.5 16
10. Plot F(P) vs. p over the range p d   
d in in in

11. From the graph, choose a standard


value of p d from Table 12-2.
3
1
p d  8  in
 
F pd
12. The calculated value of F is
in
F  p d   1.697  in
2
 
FL pd
13. Round this up to the decimal in
equivalent of a common fractional value.
 
FU pd 1
F  1.750  in
in
14. Then, the parameters that depend on p d and F are:

d p p d   4.625  in d g p d   43.875 in 0
6 8 10 12 14 16
pd  in

Vt p d   1211 Wt p d   1363 lbf


ft
min FIGURE 12-33A
Graph of Face Width and Limits for the Stage 1
σcp p d F   102877 psi Pinion (surface) in Problem 12-33

Stage 2 Pinion
Np
Given: Rotational speed of pinion (rpm) n 2   n in n 2  105.413  rpm
Ng

1. Write the equations for pitch diameter, pitch-line velocity, and transmitted load in terms of the unknown
diametral pitch, p d. Note that, in Mathcad, unit conversion factors are not included.

Np
Pitch diameter of pinion (in) d p p d  
pd

d p p d   n 2
Pitch-line velocity (fpm) Vt p d  
2

Wt p d  
H
Transmitted load (lbf)
Vt  p d 

2. Write the equation for the dynamic load factor, Cv


B
Cv p d    
A

 A  Vt  p d  
min

 ft 
© 2011 Pearson Education, Inc., Upper Saddle River, NJ. All rights reserved. This publication is protected by Copyright and written permission should be
MACHINE DESIGN - An Integrated Approach, 4th Ed. 12-33-4

3. Tentatively choose the mounting factor, Cm (Assume 2 < F < 6 in) Cm  1.7

4. The surface stress equation for the pinion is

C a W t  p d   C m
σcp p d F   Cp
Cv p d   F  d p p d   I

5. Write the design equations using the range of face-width to diametral-pitch ratio given in the text, and the
bending stress equation, solved for the unknown face width.

FU  p d  
16
Upper limit
pd

FL  p d  
8
Lower limit
pd
2
 Sfc 
Safety factor Nfc = σ 
 c
C a W t  p d   C m
2
 Cp 
Face width F  p d      Nfs
Cv p d   d p p d   I  S fcp 

2 2.5 12
6. Plot F(P) vs. p over the range p d   
d in in in
8
7. From the graph, choose a standard value of
p d from Table 12-2.  
F pd
1 in
p d  4  in 5.333
 
FL pd
8. The calculated value of F is
in
F  p d   3.779  in
 
FU pd 2.667
9. Round this up to the decimal equivalent of in
a common fractional value.

F  3.750  in
0
10. Then, the parameters that depend on p d and F are: 2 3 4 5 6 7 8
pd  in
d p p d   9.250  in d g p d   87.750 in
FIGURE 12-33B
Graph of Face Width and Limits for the Stage 2
Vt p d   255  Wt p d   6464 lbf
ft
Pinion (surface) in Problem 12-33
min

σcp p d F   104866 psi

© 2011 Pearson Education, Inc., Upper Saddle River, NJ. All rights reserved. This publication is protected by Copyright and written permission should be
MACHINE DESIGN - An Integrated Approach, 4th Ed. 12-34-1

PROBLEM 12-34
Statement: Design a reverted compound transmission based on the arrangement shown in Figure 12-14b
for an overall train ratio of approximately 80:1. It should be capable of transmitting 30 hp at
1500 rpm input shaft speed. State all assumptions.

Given: Power to be transmitted H  30 hp Approximate gear ratio mv  80


Input speed (rpm) n in  1500 rpm Number of stages n  2

Assumptions: 1. If both pinion and gear from each stage are the same material, it will only be necessary to
determine the pinion size as it will be governing for the set.
2. If the gears are not surface hardened, it will only be necessary to design to the surface
requirement as it will be governing for both bending and surface stresses.
3. If both stages have the same tooth numbers, then both must have the same diametral pitch in
order that the center distances be the same. In this case, the dimensions of the second stage will
govern the design.
Design Choices:
Pressure angle ϕ  25 deg AGMA Quality level Qv  8
Surface factor of safety Nfs  1.7 Reliability R  0.99
7
Life (cycles) N  10
Material: AGMA Grade 2 steel for both pinion and gear, both stages, through hardened to
HB  300 S ac  ( 27000  364  HB)  psi S ac  136200 psi

Solution: See Mathcad file P1234.


1. For equal stage ratios, the stage ratio is
1
n
mvs  mv mvs  8.944

2. The minimum number of teeth that we can have without interference on a 25-deg gear cut with a hob is 14.
Try pinions with 14, 15, 16, etc. teeth to see if the mating gear will have close to an integral number of teeth.

14 mvs  125.22 20 mvs  178.885 26 mvs  232.551 32 mvs  286.217

15 mvs  134.164 21 mvs  187.83 27 mvs  241.495 33 mvs  295.161

16 mvs  143.108 22 mvs  196.774 28 mvs  250.44 34 mvs  304.105

17 mvs  152.053 23 mvs  205.718 29 mvs  259.384 35 mvs  313.05

18 mvs  160.997 24 mvs  214.663 30 mvs  268.328 36 mvs  321.994

19 mvs  169.941 25 mvs  223.607 31 mvs  277.272 37 mvs  330.938

3. The next to last trial is very close to an integer so try 322 teeth:
n
 322 
mv   36  mv  80.003
 
4. For each stage let Np  36 Ng  322
Stage 2 Pinion
Np
Given: Rotational speed of pinion (rpm) n 2   n in n 2  167.702  rpm
Ng

© 2011 Pearson Education, Inc., Upper Saddle River, NJ. All rights reserved. This publication is protected by Copyright and written permission should be
MACHINE DESIGN - An Integrated Approach, 4th Ed. 12-34-2

1. Determine the surface geometry factor, I.

I   sin( ϕ)  cos( ϕ)   Ng I  0.1723


  N N
 2  p g

2. Write the equations for pitch diameter, pitch-line velocity, and transmitted load in terms of the unknown
diametral pitch, p d. Note that, in Mathcad, unit conversion factors are not included.

Np Ng
Pitch diameter of pinion (in) d p p d   d g p d  
pd pd

d p p d   n 2
Pitch-line velocity (fpm) Vt p d  
2

Wt p d  
H
Transmitted load (lbf)
Vt  p d 

3. Set the application factor, Ca Ca  1

4. Write the equation for the dynamic load factor, Cv

B  0.25  12  Qv
0.6667
A  50  56 ( 1  B) A  70.721
B
B  0.63 Cv p d    A 
 
 A  Vt  p d  
min

 ft 

5. Tentatively choose the mounting factor, Cm (Assume 2 < F < 6 in) Cm  1.7
0.5
6. Choose an elastic coefficient from Table 12-18. Cp  2300 psi

7. The surface stress equation for the pinion is

C a W t  p d   C m
σcp p d F   Cp
Cv p d   F  d p p d   I

8. Determine the endurance strength of the pinion.


 0.056
Life factor CL  2.466  N CL  1
Reliability CR  0.7  0.15 log( 1  R) CR  1
Temperature factor CT  1
Material surface strength (psi)
S ac  136200 psi
AISI 4140 Nitrided steel
S ac CL
Endurance strength S fcp  S fcp  136198 psi
CT  CR

9. Write the design equations using the range of face-width to diametral-pitch ratio given in the text, and the
bending stress equation, solved for the unknown face width.

FU  p d  
16
Upper limit
pd
© 2011 Pearson Education, Inc., Upper Saddle River, NJ. All rights reserved. This publication is protected by Copyright and written permission should be
MACHINE DESIGN - An Integrated Approach, 4th Ed. 12-34-3

FL  p d  
8
Lower limit
pd
2
 Sfc 
Safety factor Nfc =  
 σc 
C a W t  p d   C m
2
 Cp 
Face width F  p d      Nfs
Cv p d   d p p d   I  S fcp 

2 2.1 12
10. Plot F(P) vs. p over the range p d   
d in in in

11. From the graph, choose a standard value of 4


p d from Table 12-2.

1  
F pd
p d  5  in 3
in
12. The calculated value of F is  
FL pd
F  p d   2.397  in in 2

13. Round this up to the decimal equivalent of FU pd 


a common fractional value. in 1

F  2.375  in

14. Then, the parameters that depend on p d and F are: 0


2 3 4 5 6 7 8 9 10 11 12
pd  in
d p p d   7.200  in d g p d   64.400 in
FIGURE 12-34A
Graph of Face Width and Limits for the Stage 2
Vt p d   316  Wt p d   3132 lbf
ft
Pinion (surface) in Problem 12-34
min

σcp p d F   104942 psi

15. This diametral pitch and face width should be used for both stages in the gearset.

© 2011 Pearson Education, Inc., Upper Saddle River, NJ. All rights reserved. This publication is protected by Copyright and written permission should be
MACHINE DESIGN - An Integrated Approach, 4th Ed. 12-35-1

PROBLEM 12-35 _____

Statement: If the 23-tooth pinion in the gearset in Problem 12-3 is on the input shaft, find the velocity ratio,
torque ratio, and gear ratio for the gearset.
Given: Input tooth number Nin  23 Output tooth number Nout  57

Solution: See Mathcad file P1235.

1. Calculate the velocity ratio using equation 12.1a. Note that, for a given diametral pitch, the pitch radius of a
gear is directly proportional to the number of teeth on the gear.

Nin
mV   mV  0.404
Nout

2. Calculate the torque ratio using equation 12.1b.

1
mA  mA  2.478
mV

3. Calculate the gear ratio using equation 12.1c.

mG  mA mG  2.478

© 2011 Pearson Education, Inc., Upper Saddle River, NJ. All rights reserved. This publication is protected by Copyright and written permission should be
MACHINE DESIGN - An Integrated Approach, 4th Ed. 12-36-1

PROBLEM 12-36 _____

Statement: If the 78-tooth gear in the gearset in Problem 12-4 is on the input shaft, find the velocity ratio,
torque ratio, and gear ratio for the gearset.
Given: Input tooth number Nin  78 Output tooth number Nout  27

Solution: See Mathcad file P1236.

1. Calculate the velocity ratio using equation 12.1a. Note that, for a given diametral pitch, the pitch radius of a
gear is directly proportional to the number of teeth on the gear.

Nin
mV   mV  2.889
Nout

2. Calculate the torque ratio using equation 12.1b.

1
mA  mA  0.346
mV

3. Calculate the gear ratio using equation 12.1c.

mG  mV mG  2.889

© 2011 Pearson Education, Inc., Upper Saddle River, NJ. All rights reserved. This publication is protected by Copyright and written permission should be
MACHINE DESIGN - An Integrated Approach, 4th Ed. 12-37-1

PROBLEM 12-37 _____

Statement: Figure P12-2 shows an involute of a circle that starts at point A(0, rb) and continues to point P(x,y)
The angle  is known as the roll angle and  is the involute pressure angle. Derive expressions for
the x,y coordinates of P in terms of the base circle radius rb and the involute pressure angle only.
Plot y vs. x over the range 0 <  < 40 deg for rb = 2 in.
Given: Base circle radius rb  2.00 in

Solution: See Figure P12-2 and Mathcad file P1237.

1. From the definition of the involute, the distance from P to Q is the arc-length along the base circle subtended
by the angle . Thus,

PQ  rb θ

2. The distance OP = rp is then

2 2 2 2
rp  rb  rb  θ rp  rb 1  θ

3. Looking at triangle OPQ, we see that

rb θ
tan ( ϕ)  θ  tan( ϕ)
rb

4. Writing the x and y coordinates of P in terms of the above relationships, we have

2
x( ϕ)  rb 1  tan ( ϕ)  sin( tan ( ϕ)  ϕ)

2
y ( ϕ)  rb 1  tan( ϕ)  cos( tan( ϕ)  ϕ)

5. Plot y vs. x over the range ϕ  0  deg 1  deg  40 deg

2.6

2.4

y ( ϕ)
in

2.2

2
0 0.1 0.2 0.3 0.4
x( ϕ)
in
© 2011 Pearson Education, Inc., Upper Saddle River, NJ. All rights reserved. This publication is protected by Copyright and written permission should be
MACHINE DESIGN - An Integrated Approach, 4th Ed. 12-38-1

PROBLEM 12-38 _____

Statement: Derive equation 12.2 using Figure 12-5.

Solution: See Figure 12-5 and Mathcad file P1238.

1. Label points on Figure 12-5 as follows:

Center of pinion, Op
Center of gear, Og
Beginning of contact, E
Pitch point, P
Leaving contact, F
Extend line of action to tangency with pinion base circle, A (not shown on figure)
Extend line of action to tangency with gear base circle, B (not shown on figure).

2. The length of action, Z, is equal to the distance EF = EP + PF.

3. The distance PF = AF - AP and EP = BE - BP. Thus, Z = AF + BE - (AP + BP).

4. The distance OpF = rp + a p, and OgE = rg + a g, where a p is the pinion addendum and a g is the gear addendum .

5. Since the line of action is tangent to both base circles, angle AOpP = angle BOgP = , the gear pressure angle.
Also, the distance OpA = rbp = rpcos() and OgB = rbg = rgcos().

6. From the relationships above,

AF  rp  ap2   rp cos( ϕ) 2 BE   rg  ag2   rg cos( ϕ) 2

AP  rp sin( ϕ) BP  rg sin( ϕ)

7. Substituting the equations in step 6 into the equation for Z in step 3,

Z  rp  ap2   rp cos( ϕ) 2  rg  ag 2  rg cos( ϕ)  2  rp  rg  sin( ϕ)
But, rp + rg = C, the center distance. So,

Z  rp  ap2   rp cos( ϕ) 2  rg  ag 2  rg cos( ϕ)  2  C sin( ϕ)


which is equations 12-2.

© 2011 Pearson Education, Inc., Upper Saddle River, NJ. All rights reserved. This publication is protected by Copyright and written permission should be
MACHINE DESIGN - An Integrated Approach, 4th Ed. 12-39-1

PROBLEM 12-39 _____

Statement: A 39-tooth spur gear is in mesh with an 18-tooth pinion. The p d = 8 and  = 25 deg. Find the
contact ratio.

Given: Tooth numbers: Pinion Np  18 Gear Ng  39


1
Diametral pitch p d  8  in Pressure angle ϕ  25 deg

Solution: See Mathcad file P1239.

π
Circular pitch p c  p c  0.393  in
pd

Base pitch p b  p c  cos( ϕ) p b  0.356  in

Pinion:
Np
Pitch dia d p  d p  2.250  in
pd

Pitch rad rp  0.5 d p rp  1.125  in

Gear:
Ng
Pitch dia d g  d g  4.875  in
pd

Pitch rad rg  0.5 d g rg  2.438  in

1.0
Addendum a  a  0.125  in
pd

Np  Ng
Center distance C  C  3.563  in
2 pd

Length of action

Z  rp  a 2  rp cos( ϕ)  2   rg  a 2  rg cos( ϕ)  2  C sin( ϕ)

Z  0.516  in

Z
Contact ratio mp  mp  1.450
pb

© 2011 Pearson Education, Inc., Upper Saddle River, NJ. All rights reserved. This publication is protected by Copyright and written permission should be
MACHINE DESIGN - An Integrated Approach, 4th Ed. 12-40-1

PROBLEM 12-40 _____

Statement: A 79-tooth spur gear is in mesh with a 20-tooth pinion. The p d = 8 and  = 20 deg. Find the
contact ratio.

Given: Tooth numbers: Pinion Np  20 Gear Ng  79


1
Diametral pitch p d  8  in Pressure angle ϕ  20 deg

Solution: See Mathcad file P1240.

π
Circular pitch p c  p c  0.393  in
pd

Base pitch p b  p c  cos( ϕ) p b  0.369  in

Pinion:
Np
Pitch dia d p  d p  2.500  in
pd

Pitch rad rp  0.5 d p rp  1.250  in

Gear:
Ng
Pitch dia d g  d g  9.875  in
pd

Pitch rad rg  0.5 d g rg  4.938  in

1.0
Addendum a  a  0.125  in
pd

Np  Ng
Center distance C  C  6.188  in
2 pd

Length of action

Z  rp  a 2  rp cos( ϕ)  2   rg  a 2  rg cos( ϕ)  2  C sin( ϕ)

Z  0.624  in

Z
Contact ratio mp  mp  1.690
pb

© 2011 Pearson Education, Inc., Upper Saddle River, NJ. All rights reserved. This publication is protected by Copyright and written permission should be
MACHINE DESIGN - An Integrated Approach, 4th Ed. 12-41-1

PROBLEM 12-41 _____


Statement: What will the pressure angle be if the center distance of the spur gearset in Problem 12-39 is
increased by 6%?

Given: Tooth numbers: Pinion Np  18 Gear Ng  39


1
Diametral pitch p d  8  in Pressure angle ϕ  25 deg
Solution: See Figure 12-41 and Mathcad file P1241.

1. Define the factor by which the center


distance changes.
new Base Circle
C  ∆C Old Pitch Circle
Let = fc
C New Pitch Circle
Dividing numerator and denominator of the left
side by C, rbp
fc rp
∆C
fc = 1 
C
C + C
2. In this case, fc  1  0.06 fc  1.060 fc r g rbg

3. Determine the pitch diameter and pitch radius of


the pinion.

Np
Pitch diameter d p 
pd

d p  2.250  in
FIGURE 12-41
Pitch radius rp  0.5 d p Diagram Showing Center Change for Problem 12-41

rp  1.125  in

4. When the centers are moved apart the base circle diameters don't change but new pitch circle diameters are
defined by the intersection of the new line of action, which is always tangent to the two base circles, with the
line of centers. The new pitch radii are proportional to f c. Thus, from Figure 12-41, the new pressure angle is

rbp
cos ϕnew =
fc  rp

Substituting rp cos( ϕ) for rbp and solving for

new,
 rp cos( ϕ) 
ϕnew  acos  ϕnew  31.24  deg
 fc  rp 

© 2011 Pearson Education, Inc., Upper Saddle River, NJ. All rights reserved. This publication is protected by Copyright and written permission should be
MACHINE DESIGN - An Integrated Approach, 4th Ed. 12-42-1

PROBLEM 12-42 _____


Statement: What will the pressure angle be if the center distance of the spur gearset in Problem 12-40 is
increased by 5%?

Given: Tooth numbers: Pinion Np  20 Gear Ng  79


1
Diametral pitch p d  8  in Pressure angle ϕ  20 deg

Solution: See Figure 12-42 and Mathcad file P1242.

1. Define the factor by which the center


distance changes. new Base Circle
Old Pitch Circle
C  ∆C New Pitch Circle
Let = fc
C
Dividing numerator and denominator of the left rbp
fc rp
side by C,
∆C
fc = 1  C + C
C
fc r g rbg
2. In this case, fc  1  0.05 fc  1.050

3. Determine the pitch diameter and pitch radius of


the pinion.

Np
Pitch diameter d p 
pd

d p  2.500  in FIGURE 12-42


Diagram Showing Center Change for Problem 12-42
Pitch radius rp  0.5 d p

rp  1.250  in

4. When the centers are moved apart the base circle diameters don't change but new pitch circle diameters are
defined by the intersection of the new line of action, which is always tangent to the two base circles, with the
line of centers. The new pitch radii are proportional to f c. Thus, from Figure 12-41, the new pressure angle is

rbp
cos ϕnew =
fc  rp

Substituting rp cos( ϕ) for rbp and solving for

new,
 rp cos( ϕ) 
ϕnew  acos  ϕnew  26.50  deg
 fc  rp 

© 2011 Pearson Education, Inc., Upper Saddle River, NJ. All rights reserved. This publication is protected by Copyright and written permission should be
MACHINE DESIGN - An Integrated Approach, 4th Ed. 12-43-1

PROBLEM 12-43
Statement: If the spur gearsets in Problems 12-39 and 12-40 are compounded as shown in Figure 12-14,
what will the overall train ratio be?
Given: Tooth numbers N2  18 N3  39
N4  20 N5  79

Solution: See Mathcad file P1243.

1. Using equation 12.9b and assuming that gears 2 and 4 are the driver gears, the train (velocity) ratio is

N2 N4
mv  mv  0.117
N3 N5

2. On the other hand, if gears 3 and 5 are the driver gears then the train ratio is

N3 N5
mv  mv  8.5583
N2 N4

© 2011 Pearson Education, Inc., Upper Saddle River, NJ. All rights reserved. This publication is protected by Copyright and written permission should be
MACHINE DESIGN - An Integrated Approach, 4th Ed. 12-44-1

PROBLEM 12-44 _____


Statement: A 20-deg pressure angle, 23-tooth spur gear has a diametral pitch of 6. Find the pitch diameter,
addendum, dedendum, outside diameter, and circular pitch.
Given: Number of teeth N  23 Pressure angle ϕ  20 deg
1
Diametral pitch p d  6  in

Solution: See Mathcad file P1244.

N
Pitch diameter d  d  3.833  in
pd

1.0
Addendum a  a  0.167  in
pd

1.25
Dedendum b  b  0.208  in
pd

Outside diameter Do  d  2  a Do  4.167  in

π
Circular pitch p c  p c  0.524  in
pd

© 2011 Pearson Education, Inc., Upper Saddle River, NJ. All rights reserved. This publication is protected by Copyright and written permission should be
MACHINE DESIGN - An Integrated Approach, 4th Ed. 12-45-1

PROBLEM 12-45 _____


Statement: A 25-deg pressure angle, 32-tooth spur gear has a diametral pitch of 4. Find the pitch diameter,
addendum, dedendum, outside diameter, and circular pitch.
Given: Number of teeth N  32 Pressure angle ϕ  20 deg
1
Diametral pitch p d  4  in

Solution: See Mathcad file P1245.

N
Pitch diameter d  d  8.000  in
pd

1.0
Addendum a  a  0.250  in
pd

1.25
Dedendum b  b  0.313  in
pd

Outside diameter Do  d  2  a Do  8.500  in

π
Circular pitch p c  p c  0.785  in
pd

© 2011 Pearson Education, Inc., Upper Saddle River, NJ. All rights reserved. This publication is protected by Copyright and written permission should be
MACHINE DESIGN - An Integrated Approach, 4th Ed. 12-46-1

PROBLEM 12-46 _____

Statement: Design a two-stage compound spur gear train for an overall ratio of approximately 53:1. Specify
tooth numbers for each gear in the train.

Given: Approximate ratio mG  53 Number of stages n  2

Assumptions: The gears will be cut with a hob and the pressure angle will be ϕ  25 deg

Solution: See Mathcad file P1246.

1. For equal stage ratios, the stage ratio is approximately


1
n
mGs  mG mGs  7.280

2. The minimum number of teeth that we can have without interference on a 25-deg gear cut with a hob is 14.
Try pinions with 14, 15, 16, etc. teeth to see if the mating gear will have close to an integral number of teeth.

14 mGs  101.922

15 mGs  109.202

16 mGs  116.482

3. The first trial is very close to an integer so try 102 teeth:


n
 102 
mG   14  mG  53.082
 

4. We can get slightly closer by using two 14-tooth pinions with 101 and 103-tooth gears
101  103
mG  mG  53.077
14 14

5. However, it may be harder to find the 101 and 103-tooth gears and, it may be less expensive to have two gears
with the same number of teeth than to have different numbers of teeth.

© 2011 Pearson Education, Inc., Upper Saddle River, NJ. All rights reserved. This publication is protected by Copyright and written permission should be
MACHINE DESIGN - An Integrated Approach, 4th Ed. 12-47-1

PROBLEM 12-47 _____

Statement: Design a three-stage compound spur gear train for an overall ratio of approximately 592:1.
Specify tooth numbers for each gear in the train.

Given: Approximate ratio mG  592 Number of stages s  3

Solution: See Mathcad file P1247.


1
s
1. The average stage ratio is ravg  mG ravg  8.397

2. If the minimum number of teeth is (for a 20 deg pressure angle) Nmin  18 then the number of teeth on the
driven gear for each stage is

NG1  floor  ravg Nmin NG1  151

or NG2  ceil ravg Nmin NG2  152

3. But, the prime factors of 592 are 2 4 and 37. This suggests that one of the gears have a number of teeth that is
a multiple of 37, say 148.

148
 17.626 Let N2  18 N3  148
ravg

N5 N7
Thus, = 72 if N4  18 and N6  18
N4 N6

5 6
then N5 N7 = 72 18 18 = 2  3

4 2 1 4
Try N5  2  3 N7  2  3 then N5 N7  23328

Summarizing, N2  18 N3  148

N4  18 N5  144

N6  18 N7  162

4. Since the driven gears all have less than 180 teeth, no stage ratio is greater than 10.

5. Checking the overall gear ratio

N 3 N 5 N 7
mG  mG  592.00
N 2 N 4 N 6

© 2011 Pearson Education, Inc., Upper Saddle River, NJ. All rights reserved. This publication is protected by Copyright and written permission should be
MACHINE DESIGN - An Integrated Approach, 4th Ed. 12-48-1

PROBLEM 12-48 _____

Statement: Design a planetary gear train similar to that shown in Figure 12-16 for an overall velocity ratio of
exactly 0.2 if the sun gear is the input, the arm is the output, and the ring gear is stationary.
Specify tooth numbers for each gear in the train.

Given: Exact ratio mv  0.2

Assumptions: The gears will be cut with a hob and the pressure angle will be ϕ  20 deg

Solution: See Figure 12-16 and Mathcad file P1248.

1. Let the first gear be the sun gear (gear 2) and the last be the ring gear (gear 4). Then, equation 12.11c can be
written as

ω4  ωarm N2
 
ω2  ωarm N4

2. Taking 4 = 0 and solving for out/in, we have

ωout ωarm N2
= = = mv
ωin ω2 N2  N4

3. Let the sun gear have tooth number N2  20 , then the number of teeth required for the ring gear is

1  mv
N4   N2 N4  80
mv

4. In order to mesh properly, the number of teeth on the ring gear must be equal to the sum of the number of
teeth on the sun gear plus two times the number of teeth on the planet gear(s). Thus,

N4  N2
N3  N3  30
2

5. Check the overall velocity ratio and summarize the tooth numbers.

N2
Overall velocity ratio mv  mv  0.200
N2  N4

Sun gear N2  20

Planet gear(s) N3  30

Ring gear N4  80

© 2011 Pearson Education, Inc., Upper Saddle River, NJ. All rights reserved. This publication is protected by Copyright and written permission should be
MACHINE DESIGN - An Integrated Approach, 4th Ed. 12-49-1

PROBLEM 12-49 _____

Statement: Design a planetary gear train similar to that shown in Figure 12-16 for an overall velocity ratio
of exactly 4/3 if the sun gear is stationary, the arm is the input, and the ring gear is the output.
Specify tooth numbers for each gear in the train.
4
Given: Exact ratio mv 
3

Assumptions: The gears will be cut with a hob and the pressure angle will be ϕ  20 deg

Solution: See Figure 12-16 and Mathcad file P1249.

1. Let the first gear be the sun gear (gear 2) and the last be the ring gear (gear 4). Then, equation 12.11c can be
written as

ω4  ωarm N2
 
ω2  ωarm N4

2. Taking 2 = 0 and solving for out/in, we have

ωout ω4 N4  N2
= = = mv
ωin ωarm N4

3. Let the sun gear have tooth number N2  20 , then the number of teeth required for the ring gear is

N2
N4  N4  60
mv  1

4. In order to mesh properly, the number of teeth on the ring gear must be equal to the sum of the number of
teeth on the sun gear plus two times the number of teeth on the planet gear(s). Thus,

N4  N2
N3  N3  20
2

5. Check the overall velocity ratio and summarize the tooth numbers.

N4  N2
Overall velocity ratio mv  mv  1.333
N4

Sun gear N2  20

Planet gear(s) N3  20

Ring gear N4  60

© 2011 Pearson Education, Inc., Upper Saddle River, NJ. All rights reserved. This publication is protected by Copyright and written permission should be
MACHINE DESIGN - An Integrated Approach, 4th Ed. 12-50-1

PROBLEM 12-50 _____


Statement: A 21-tooth pinion rotating at 1800 rpm meshes with a 33-tooth gear in a spur gear reducer. Both
pinion and gear are manufactured to a quality level of 9. A reliability of 0.9 has been specified,
and the transmitted tangential load is 2800 lb. Conditions are such that Km = 1.7. It is proposed
that standard 25-degree, full-depth teeth be used, with both pinion and gear hobbed from an
AISI 4140 nitrided steel. The diametral pitch is 6, and the face width 2.000 in. Estimate the
number of cycles of bending stress (using the AGMA equations) that the gearset can
withstand.

Given: Tooth numbers Np  21 Ng  33


Pinion speed n  1800 rpm Mounting factor Km  1.7
1
Diametral pitch p d  6  in Quality index Qv  9
Face width F  2.000  in Pressure angle ϕ  25 deg
Tangential load Wt  2800 lbf Reliability R  0.90
Assumptions: The life of the pinion will be less than that of the gear.
Solution: See Mathcad file P1250.
1. Calculate the pitch diameter of the pinion using equation 12.4a and the pitch-line velocity of the gearset.
Np d n ft
d  d  3.500  in V  V  1649
pd 2 min
2. Calculate the dynamic factor Kv using equations 12.16 and 12.17.

B  0.25  12  Qv
0.6667
B  0.520 A  50  56 ( 1  B) A  76.878
B
Kv   A  Kv  0.802
 V  min 

 A   
  ft  
3. Determine the bending geometry factor, J (Table 12-13) J  0.40
4. Assume an application factor, Ka Ka  1

5. Calculate the bending stress using equation 12.15.


Ka Wt p d  Km
σb  σb  44.51  ksi
Kv F  J

6. Determine the material bending fatigue strength from Table 12-20 using the average value. S'fb  39.5 ksi
7. Determine the reliability factor for R  0.90 from Table 12-19. KR  0.85
8. Assume a temperature factor, KT KT  1

9. Set the bending stress equal to the corrected bending-fatigue strength in equation 12.24 and solve for the life
factor, KL .
KT  KR σb
KL  KL  0.958
S'fb
10. Assume that the fatigue life is over 10 6 cycles and use the lower curve in Figure 12-24 to solve for the cycle life
1
 0.0323 0.0323
KL = 1.6831 N
N   1.6831  N  3.80  10
7
 K 
 L 
© 2011 Pearson Education, Inc., Upper Saddle River, NJ. All rights reserved. This publication is protected by Copyright and written permission should be
MACHINE DESIGN - An Integrated Approach, 4th Ed. 12-51-1

PROBLEM 12-51 _____


Statement: A 21-tooth pinion rotating at 1800 rpm meshes with a 33-tooth gear in a spur gear reducer.
Both pinion and gear are manufactured to a quality level of 9. A reliability of 0.9 has been
specified, and the transmitted tangential load is 2800 lb. Conditions are such that Km = 1.7. It
is proposed that standard 25-degree, full-depth teeth be used, with both pinion and gear
hobbed from an AISI 4140 nitrided steel. The diametral pitch is 6, and the face width 2.000 in.
Estimate the number of cycles of contact (surface) stress (using the AGMA equations) that the
gearset can withstand.

Given: Tooth numbers Np  21 Ng  33


Pinion speed n  1800 rpm Mounting factor Cm  1.7
1
Diametral pitch p d  6  in Quality index Qv  9
Face width F  2.000  in Pressure angle ϕ  25 deg
Tangential load Wt  2800 lbf Reliability R  0.90
Assumptions: The life of the pinion will be less than that of the gear.
Solution: See Mathcad file P1251.
1. Calculate the pitch diameter of the pinion using equation 12.4a and the pitch-line velocity of the gearset.
Np d n ft
d  d  3.500  in V  V  1649
pd 2 min
2. Calculate the dynamic factor Kv using equations 12.16 and 12.17.
B  0.25  12  Qv
0.6667
B  0.520 A  50  56 ( 1  B) A  76.878
B
Cv   A  Cv  0.802
 

 A   
V min

  ft  

3. Calculate the geometry factor, I

 sin( ϕ)  cos( ϕ)   Ng
I    I  0.117
 2  Np  Ng
0.5
4. Determine the elastic coefficient from Table 12-18. Cp  2300 psi
5. Assume an application factor, Ca Ca  1

6. Calculate the contact stress using equation 12.21.


1
2
 Ca Wt 1 Cm 
σc  Cp     σc  195.76 ksi
 Cv F  d I 
7. Determine the material surface-fatigue strength from Table 12-21 using the average value. S'fc  167.5  ksi
8. Determine the reliability factor for R  0.90 from Table 12-19. CR  0.85

9. Assume a temperature factor, CT CT  1

10. Set the contact stress equal to the corrected surface-fatigue strength in equation 12.25 and solve for the life
factor, CL .

© 2011 Pearson Education, Inc., Upper Saddle River, NJ. All rights reserved. This publication is protected by Copyright and written permission should be
MACHINE DESIGN - An Integrated Approach, 4th Ed. 12-51-2

CT  CR σc
CL  CL  0.993
S'fc

11. Use the lower curve in Figure 12-26 to solve for the cycle life.

1
0.056
CL = 2.466  N
 0.056
N   2.466  N  1.13  10
7
 C 
 L 

© 2011 Pearson Education, Inc., Upper Saddle River, NJ. All rights reserved. This publication is protected by Copyright and written permission should be
MACHINE DESIGN - An Integrated Approach, 4th Ed. 12-52-1

PROBLEM 12-52
Statement: If the gearset in Problem 12-46 transmits 7.5 kW at 1750 input pinion rpm, find the torque on
each of the three shafts.
Given: Transmitted power P  7.5 kW Input speed ωp1  1750 rpm

Assumptions: Tooth numbers:


Stage 1 Np1  14 Ng1  101
Stage 2 Np2  14 Ng2  103

Solution: See Mathcad file P1252.

1. Confirm that the gear ratio is correct.

Ng1 Ng2
mG  mG  53.077
Np1 Np2

2. Calculate the gear speeds.

Np1
Stage 1 ωp1  1750 rpm ωg1   ωp1 ωg1  242.574  rpm
Ng1

Np2
Stage 2 ωp2  ωg1 ωg2   ωp2 ωg2  32.971 rpm
Ng2

3. Calculate the shaft torque


P
Shaft 1 T1  T1  40.9 N  m Input
ωp1

P
Shaft 2 T2  T2  295  N  m
ωg1

P
Shaft 3 T3  T3  2172 N  m Output
ωg2

© 2011 Pearson Education, Inc., Upper Saddle River, NJ. All rights reserved. This publication is protected by Copyright and written permission should be
MACHINE DESIGN - An Integrated Approach, 4th Ed. 12-53-1

PROBLEM 12-53
Statement: Size the first-stage spur gears in problem 12-52 for a bending factor of safety of at least 2.8 and
a surface factor of safety of at least 1.8 assuming a steady torque, 25-deg pressure angle, full
depth teeth, Qv = 9, and AISI 4340 steel for all gears.

Given: Pressure angle ϕ  25 deg


Bending factor of safety Nfb  2.8 Reliability R  0.99
Surface factor of safety Nfs  1.8 AGMA Quality level Qv  9
7
Power to be transmitted (hp) H  10 hp Life (cycles) N  10
Number of pinion teeth Np  14 Number of gear teeth Ng  101
Rotational speed of pinion (rpm) n  1750 rpm

Assumptions: Since both pinion and gear from each stage are the same material, it will only be necessary to
determine the pinion size as it will be governing for the set.
Solution: See Mathcad file P1253.
Stage 1 Pinion - Bending
1. Determine the bending geometry factor, J (Table 12-13) Jp  0.35

2. Write the equations for pitch diameter, pitchline velocity, and transmitted load in terms of the unknown
diametral pitch, pd. Note that, in Mathcad, unit conversion factors are not included.

Np Ng
Pitch diameter of pinion and gear (in) d p p d   d g p d  
pd pd

d p p d   n
pitchline velocity (fpm) Vt p d  
2

Wt p d  
H
Transmitted load (lbf)
Vt  p d 

3. Set the application factor, Ka Ka  1

4. Write the equation for the dynamic load factor, Kv

B  0.25  12  Qv
0.6667
A  50  56 ( 1  B) A  76.878
B
B  0.52
Kv p d    A 
 
 A  Vt  p d  
min

 ft 

5. Tentatively choose the mounting factor, Km (Assume 0 < F < 2 in) Km  1.6

6. The bending stress equation for the pinion is


Ka Wt p d   p d  Km
σbp p d F  
Kv p d   F  Jp

7. Determine the endurance strength of the pinion.


 0.0323
Life factor KL  1.6831 N KL  1
Reliability KR  0.7  0.15 log( 1  R) KR  1
© 2011 Pearson Education, Inc., Upper Saddle River, NJ. All rights reserved. This publication is protected by Copyright and written permission should be
MACHINE DESIGN - An Integrated Approach, 4th Ed. 12-53-2

Temperature factor KT  1
Material bending strength (psi)
S atp  41500  psi
AISI 4340 Nitrided steel
S atp KL
Endurance strength S fbp  S fbp  41501  psi
KT  KR

8. Write the design equations using the range of face-width to diametral-pitch ratio given in the text, and the
bending stress equation, solved for the unknown face width.

FU  p d  
16
Upper limit
pd

FL  p d  
8
Lower limit
pd
S fb
Safety factor Nfb =
σb

Ka Wt p d   p d  Km Nfb


Face width F  p d  
Kv p d   Jp S fbp

5 5.1 10
9. Plot F(P) vs. p over the range p d   
d in in in

10. From the graph, choose a standard value of 4


p d from Table 12-2.

p d  8  in
1  
F pd
3
in
11. The calculated value of F is
 
FL pd
F  p d   1.195  in 2
in
12. Round this up to the decimal equivalent of  
FU pd
a common fractional value.
in 1
F  1.250  in

13. Then, the parameters that depend on p d and F are: 0


5 6 7 8 9 10
pd  in
d p p d   1.750  in d g p d   12.625 in
FIGURE 12-53A
Vt p d   802  Wt p d   412  lbf
ft
min Graph of Face Width and Limits for the Stage 1
Pinion (bending) in Problem 12-53
σbp p d F   14175  psi Kv p d   0.850

The assumption made in step 5 is correct so no further iteration is required.

Stage 1 Pinion - Surface Fatigue


1. Determine the surface geometry factor, I.

I   sin( ϕ)  cos( ϕ)   Ng I  0.1682


  N N
 2  p g
© 2011 Pearson Education, Inc., Upper Saddle River, NJ. All rights reserved. This publication is protected by Copyright and written permission should be
MACHINE DESIGN - An Integrated Approach, 4th Ed. 12-53-3

2. Write the equations for pitch diameter, pitchline velocity, and transmitted load in terms of the unknown
diametral pitch, pd. Note that, in Mathcad, unit conversion factors are not included.

Np
Pitch diameter of pinion (in) d p p d  
pd

d p p d   n
pitchline velocity (fpm) Vt p d  
2

Wt p d  
H
Transmitted load (lbf)
Vt  p d 

3. Set the application factor, Ca Ca  1

4. Write the equation for the dynamic load factor, Cv

B  0.25  12  Qv
0.6667
A  50  56 ( 1  B) A  76.878
B
Cv p d    
B  0.52 A

 A  Vt  p d  
min

 ft 

5. Tentatively choose the mounting factor, Cm (Assume 0 < F < 2 in) Cm  1.6
0.5
6. Choose an elastic coefficient from Table 12-18. (Steel on steel). Cp  2300 psi

7. The surface stress equation for the pinion is

C a W t  p d   C m
σcp p d F   Cp
Cv p d   F  d p p d   I

8. Determine the endurance strength of the pinion.


 0.056
Life factor CL  2.466  N CL  1
Reliability CR  0.7  0.15 log( 1  R) CR  1
Temperature factor CT  1
Material surface strength (psi)
S acp  162500 psi
AISI 4340 Nitrided steel
S acp CL
Endurance strength S fcp  S fcp  162497 psi
CT  CR

9. Write the design equations using the range of face-width to diametral-pitch ratio given on page 740 of the text,
and the bending stress equation, solved for the unknown face width.

FU  p d  
16
Upper limit
pd

FL  p d  
8
Lower limit
pd

© 2011 Pearson Education, Inc., Upper Saddle River, NJ. All rights reserved. This publication is protected by Copyright and written permission should be
MACHINE DESIGN - An Integrated Approach, 4th Ed. 12-53-4

2
 Sfc 
Safety factor Nfc = σ 
 c
C a W t  p d   C m
2
 Cp 
Face width F  p d      Nfs
Cv p d   d p p d   I  S fcp 

5 5.1 10
10. Plot F(P) vs. p over the range p d   
d in in in

11. From the graph, choose a standard value of 4


p d from Table 12-2.

1  
F pd
p d  8  in 3
in
12. The calculated value of F is  
FL pd
F  p d   0.950  in in 2

13. Round this up to the decimal equivalent of  


FU pd
a common fractional value. in 1

F  1.000  in

14. The bending requirement is governing in 0


5 6 7 8 9 10
this case. The diametral pitch and face
width for the stage 1 gearset are: pd  in
FIGURE 12-53B
1
p d  8  in F  1.250  in
Graph of Face Width and Limits for the Stage 1
Pinion (surface) in Problem 12-53

© 2011 Pearson Education, Inc., Upper Saddle River, NJ. All rights reserved. This publication is protected by Copyright and written permission should be
MACHINE DESIGN - An Integrated Approach, 4th Ed. 12-54-1

PROBLEM 12-54
Statement: Size the second-stage spur gears in problem 12-52 for a bending factor of safety of at least 2.8
and a surface factor of safety of at least 1.8 assuming a steady torque, 25-deg pressure angle,
full depth teeth, Qv = 9, and AISI 4340 steel for all gears.

Given: Pressure angle ϕ  25 deg


Bending factor of safety Nfb  2.8 Reliability R  0.99
Surface factor of safety Nfs  1.8 AGMA Quality level Qv  9
7
Power to be transmitted (hp) H  10 hp Life (cycles) N  10
Number of pinion teeth Np  14 Number of gear teeth Ng  103
Rotational speed of pinion (rpm) n  242.574  rpm

Assumptions: Since both pinion and gear from each stage are the same material, it will only be necessary to
determine the pinion size as it will be governing for the set.
Solution: See Mathcad file P1254.

Stage 1 Pinion - Bending


1. Determine the bending geometry factor, J (Table 12-13) Jp  0.35

2. Write the equations for pitch diameter, pitchline velocity, and transmitted load in terms of the unknown
diametral pitch, pd. Note that, in Mathcad, unit conversion factors are not included.

Np Ng
Pitch diameter of pinion and gear (in) d p p d   d g p d  
pd pd

d p p d   n
pitchline velocity (fpm) Vt p d  
2

Wt p d  
H
Transmitted load (lbf)
Vt  p d 

3. Set the application factor, Ka Ka  1

4. Write the equation for the dynamic load factor, Kv

B  0.25  12  Qv
0.6667
A  50  56 ( 1  B) A  76.878
B
B  0.52
Kv p d    A 
 
 A  Vt  p d  
min

 ft 

5. Tentatively choose the mounting factor, Km (Assume 1.99 < F < 6 in) Km  1.7

6. The bending stress equation for the pinion is

Ka Wt p d   p d  Km
σbp p d F  
Kv p d   F  Jp

7. Determine the endurance strength of the pinion.


 0.0323
Life factor KL  1.6831 N KL  1

© 2011 Pearson Education, Inc., Upper Saddle River, NJ. All rights reserved. This publication is protected by Copyright and written permission should be
MACHINE DESIGN - An Integrated Approach, 4th Ed. 12-54-2

Reliability KR  0.7  0.15 log( 1  R) KR  1


Temperature factor KT  1
Material bending strength (psi)
S atp  41500  psi
AISI 4340 Nitrided steel
S atp KL
Endurance strength S fbp  S fbp  41501  psi
KT  KR

8. Write the design equations using the range of face-width to diametral-pitch ratio given in the text, and the
bending stress equation, solved for the unknown face width.

FU  p d  
16
Upper limit
pd

FL  p d  
8
Lower limit
pd
S fb
Safety factor Nfb =
σb

Ka Wt p d   p d  Km Nfb


Face width F  p d  
Kv p d   Jp S fbp

2 2.1 6
9. Plot F(P) vs. p over the range p d   
d in in in

10. From the graph, choose a standard value of 6


p d from Table 12-2.

p d  4  in
1  
F pd
in
11. The calculated value of F is 4
 
FL pd
F  p d   2.134  in
in
12. Round this up to the decimal equivalent of  
FU pd 2
a common fractional value.
in
F  2.250  in

13. Then, the parameters that depend on p d and F are: 0


1 2 3 4 5 6
pd  in
d p p d   3.500  in d g p d   25.750 in
FIGURE 12-54A
Vt p d   222  Wt p d   1485 lbf
ft
min Graph of Face Width and Limits for the Stage 2
Pinion (bending) in Problem 12-54
σbp p d F   14058  psi Kv p d   0.912

The assumption made in step 5 is correct so no further iteration is required.

Stage 1 Pinion - Surface Fatigue


1. Determine the surface geometry factor, I.

© 2011 Pearson Education, Inc., Upper Saddle River, NJ. All rights reserved. This publication is protected by Copyright and written permission should be
MACHINE DESIGN - An Integrated Approach, 4th Ed. 12-54-3

 sin( ϕ)  cos( ϕ)   Ng
I    I  0.1686
 2  Np  Ng
2. Write the equations for pitch diameter, pitchline velocity, and transmitted load in terms of the unknown
diametral pitch, pd. Note that, in Mathcad, unit conversion factors are not included.

Np
Pitch diameter of pinion (in) d p p d  
pd

d p p d   n
pitchline velocity (fpm) Vt p d  
2

Wt p d  
H
Transmitted load (lbf)
Vt  p d 

3. Set the application factor, Ca Ca  1

4. Write the equation for the dynamic load factor, Cv

B  0.25  12  Qv
0.6667
A  50  56 ( 1  B) A  76.878
B
Cv p d    
B  0.52 A

 A  Vt  p d  
min

 ft 

5. Tentatively choose the mounting factor, Cm (Assume 1.99 < F < 6 in) Cm  1.7
0.5
6. Choose an elastic coefficient from Table 12-18. (Steel on steel). Cp  2300 psi

7. The surface stress equation for the pinion is

C a W t  p d   C m
σcp p d F   Cp
Cv p d   F  d p p d   I

8. Determine the endurance strength of the pinion.


 0.056
Life factor CL  2.466  N CL  1
Reliability CR  0.7  0.15 log( 1  R) CR  1
Temperature factor CT  1
Material surface strength (psi)
S acp  162500 psi
AISI 4340 Nitrided steel
S acp CL
Endurance strength S fcp  S fcp  162497 psi
CT  CR

9. Write the design equations using the range of face-width to diametral-pitch ratio given in of the text, and the
bending stress equation, solved for the unknown face width.

FU  p d  
16
Upper limit
pd

© 2011 Pearson Education, Inc., Upper Saddle River, NJ. All rights reserved. This publication is protected by Copyright and written permission should be
MACHINE DESIGN - An Integrated Approach, 4th Ed. 12-54-4

FL  p d  
8
Lower limit
pd
2
 Sfc 
Safety factor Nfc = σ 
 c
C a W t  p d   C m
2
 Cp 
Face width F  p d      Nfs
Cv p d   d p p d   I  S fcp 

2 2.1 6
10. Plot F(P) vs. p over the range p d   
d in in in

11. From the graph, choose a standard value of 4


p d from Table 12-2.

1  
F pd
p d  4  in 3
in
12. The calculated value of F is  
FL pd
F  p d   1.691  in in 2

13. Round this up to the decimal equivalent of FU pd 


a common fractional value. in 1

F  1.750  in

14. The bending requirement is governing in 0


1 2 3 4 5 6
this case. The diametral pitch and face
width for the stage 2 gearset are: pd  in
FIGURE 12-54B
1
p d  4  in F  2.250  in
Graph of Face Width and Limits for the Stage 2
Pinion (surface) in Problem 12-54

© 2011 Pearson Education, Inc., Upper Saddle River, NJ. All rights reserved. This publication is protected by Copyright and written permission should be
MACHINE DESIGN - An Integrated Approach, 4th Ed. 12-55-1

PROBLEM 12-55
Statement: If the gearset in Problem 12-47 transmits 18.5 kW at 1184 input pinion rpm, find the torque on
each of the four shafts.
Given: Transmitted power P  18.5 kW Input speed ωp1  1184 rpm

Assumptions: Tooth numbers:


Stage 1 Np1  18 Ng1  148
Stage 2 Np2  18 Ng2  144
Stage 3 Np3  18 Ng3  162

Solution: See Mathcad file P1255.

1. Confirm that the gear ratio is correct.

Ng1 Ng2 Ng3


mG  mG  592.000
Np1 Np2 Np3

2. Calculate the gear speeds.

Np1
Stage 1 ωp1  1184 rpm ωg1   ωp1 ωg1  144  rpm
Ng1

Np2
Stage 2 ωp2  ωg1 ωg2   ωp2 ωg2  18 rpm
Ng2
Np3
Stage 3 ωp3  ωg2 ωg3   ωp3 ωg3  2  rpm
Ng3

3. Calculate the shaft torque.


P
Shaft 1 T1  T1  149.2  N  m Input
ωp1

P
Shaft 2 T2  T2  1227 N  m
ωg1

P
Shaft 3 T3  T3  9815 N  m
ωg2

P
Shaft 4 T4  T4  88331  N  m Output
ωg3

© 2011 Pearson Education, Inc., Upper Saddle River, NJ. All rights reserved. This publication is protected by Copyright and written permission should be
MACHINE DESIGN - An Integrated Approach, 4th Ed. 12-56-1

PROBLEM 12-56
Statement: Size the first-stage spur gears in problem 12-55 for a bending factor of safety of at least 2.4 and
a surface factor of safety of at least 2.0 assuming a steady torque, 25-deg pressure angle, full
depth teeth, Qv = 10, and AISI 4140 steel for all gears.

Given: Pressure angle ϕ  25 deg


Bending factor of safety Nfb  2.4 Reliability R  0.99
Surface factor of safety Nfs  2.0 AGMA Quality level Qv  10
7
Power to be transmitted (hp) H  25 hp Life (cycles) N  10
Number of pinion teeth Np  18 Number of gear teeth Ng  148
Rotational speed of pinion (rpm) n  1184 rpm

Assumptions: Since both pinion and gear from each stage are the same material, it will only be necessary to
determine the pinion size as it will be governing for the set.
Solution: See Mathcad file P1256.

Stage 1 Pinion - Bending


1. Determine the bending geometry factor, J (Table 12-13) Jp  0.39

2. Write the equations for pitch diameter, pitchline velocity, and transmitted load in terms of the unknown
diametral pitch, pd. Note that, in Mathcad, unit conversion factors are not included.

Np Ng
Pitch diameter of pinion and gear (in) d p p d   d g p d  
pd pd

d p p d   n
pitchline velocity (fpm) Vt p d  
2

Wt p d  
H
Transmitted load (lbf)
Vt  p d 

3. Set the application factor, Ka Ka  1

4. Write the equation for the dynamic load factor, Kv

B  0.25  12  Qv
0.6667
A  50  56 ( 1  B) A  83.776
B
B  0.397
Kv p d    A 
 
 A  Vt  p d  
min

 ft 

5. Tentatively choose the mounting factor, Km (Assume 1.99 < F < 6 in) Km  1.7

6. The bending stress equation for the pinion is

Ka Wt p d   p d  Km
σbp p d F  
Kv p d   F  Jp

7. Determine the endurance strength of the pinion.


 0.0323
Life factor KL  1.6831 N KL  1

© 2011 Pearson Education, Inc., Upper Saddle River, NJ. All rights reserved. This publication is protected by Copyright and written permission should be
MACHINE DESIGN - An Integrated Approach, 4th Ed. 12-56-2

Reliability KR  0.7  0.15 log( 1  R) KR  1


Temperature factor KT  1
Material bending strength (psi)
S atp  40000  psi
AISI 4140 Nitrided steel
S atp KL
Endurance strength S fbp  S fbp  40001  psi
KT  KR

8. Write the design equations using the range of face-width to diametral-pitch ratio given in the text, and the
bending stress equation, solved for the unknown face width.

FU  p d  
16
Upper limit
pd

FL  p d  
8
Lower limit
pd
S fb
Safety factor Nfb =
σb

Ka Wt p d   p d  Km Nfb


Face width F  p d  
Kv p d   Jp S fbp

5 5.1 10
9. Plot F(P) vs. p over the range p d   
d in in in

10. From the graph, choose a standard value of 4


p d from Table 12-2.

p d  6  in
1  
F pd
3
in
11. The calculated value of F is
 
FL pd
F  p d   1.575  in 2
in
12. Round this up to the decimal equivalent of  
FU pd
a common fractional value.
in 1
F  1.750  in

13. Then, the parameters that depend on p d and F are: 0


5 6 7 8 9 10
pd  in
d p p d   3.000  in d g p d   24.667 in
FIGURE 12-56A
Vt p d   930  Wt p d   887  lbf
ft
min Graph of Face Width and Limits for the Stage 1
Pinion (bending) in Problem 12-56
σbp p d F   14997  psi Kv p d   0.884

The assumption made in step 5 is correct so no further iteration is required.

Stage 1 Pinion - Surface Fatigue


1. Determine the surface geometry factor, I.

© 2011 Pearson Education, Inc., Upper Saddle River, NJ. All rights reserved. This publication is protected by Copyright and written permission should be
MACHINE DESIGN - An Integrated Approach, 4th Ed. 12-56-3

 sin( ϕ)  cos( ϕ)   Ng
I    I  0.1707
 2  Np  Ng
2. Write the equations for pitch diameter, pitchline velocity, and transmitted load in terms of the unknown
diametral pitch, pd. Note that, in Mathcad, unit conversion factors are not included.

Np
Pitch diameter of pinion (in) d p p d  
pd

d p p d   n
pitchline velocity (fpm) Vt p d  
2

Wt p d  
H
Transmitted load (lbf)
Vt  p d 

3. Set the application factor, Ca Ca  1

4. Write the equation for the dynamic load factor, Cv

B  0.25  12  Qv
0.6667
A  50  56 ( 1  B) A  83.776
B
Cv p d    
B  0.397 A

 A  Vt  p d  
min

 ft 

5. Tentatively choose the mounting factor, Cm (Assume 1.99 < F < 6 in) Cm  1.7
0.5
6. Choose an elastic coefficient from Table 12-18. (Steel on steel). Cp  2300 psi

7. The surface stress equation for the pinion is

C a W t  p d   C m
σcp p d F   Cp
Cv p d   F  d p p d   I

8. Determine the endurance strength of the pinion.


 0.056
Life factor CL  2.466  N CL  1
Reliability CR  0.7  0.15 log( 1  R) CR  1
Temperature factor CT  1
Material surface strength (psi)
S acp  165000 psi
AISI 4140 Nitrided steel
S acp CL
Endurance strength S fcp  S fcp  164997 psi
CT  CR

9. Write the design equations using the range of face-width to diametral-pitch ratio given in the text, and the
bending stress equation, solved for the unknown face width.

FU  p d  
16
Upper limit
pd

© 2011 Pearson Education, Inc., Upper Saddle River, NJ. All rights reserved. This publication is protected by Copyright and written permission should be
MACHINE DESIGN - An Integrated Approach, 4th Ed. 12-56-4

FL  p d  
8
Lower limit
pd
2
 Sfc 
Safety factor Nfc = σ 
 c
C a W t  p d   C m
2
 Cp 
Face width F  p d      Nfs
Cv p d   d p p d   I  S fcp 

5 5.1 10
10. Plot F(P) vs. p over the range p d   
d in in in

11. From the graph, choose a standard value of 4


p d from Table 12-2.

1  
F pd
p d  6  in 3
in
12. The calculated value of F is  
FL pd
F  p d   1.294  in in 2

13. Round this up to the decimal equivalent of FU pd 


a common fractional value. in 1

F  1.375  in

14. The bending requirement is governing in 0


5 6 7 8 9 10
this case. The diametral pitch and face
width for the stage 1 gearset are: pd  in
FIGURE 12-56B
1
p d  6  in F  1.750  in
Graph of Face Width and Limits for the Stage 1
Pinion (surface) in Problem 12-56

© 2011 Pearson Education, Inc., Upper Saddle River, NJ. All rights reserved. This publication is protected by Copyright and written permission should be
MACHINE DESIGN - An Integrated Approach, 4th Ed. 12-57-1

PROBLEM 12-57
Statement: Size the second-stage spur gears in problem 12-55 for a bending factor of safety of at least 2.4
and a surface factor of safety of at least 2.0 assuming a steady torque, 25-deg pressure angle,
full depth teeth, Qv = 10, and AISI 4140 steel for all gears.

Given: Pressure angle ϕ  25 deg


Bending factor of safety Nfb  2.4 Reliability R  0.99
Surface factor of safety Nfs  2.0 AGMA Quality level Qv  10
7
Power to be transmitted (hp) H  25 hp Life (cycles) N  10
Number of pinion teeth Np  18 Number of gear teeth Ng  144
Rotational speed of pinion (rpm) n  144  rpm

Assumptions: Since both pinion and gear from each stage are the same material, it will only be necessary to
determine the pinion size as it will be governing for the set.
Solution: See Mathcad file P1257.
Stage 1 Pinion - Bending
1. Determine the bending geometry factor, J (Table 12-13) Jp  0.39

2. Write the equations for pitch diameter, pitchline velocity, and transmitted load in terms of the unknown
diametral pitch, pd. Note that, in Mathcad, unit conversion factors are not included.

Np Ng
Pitch diameter of pinion and gear (in) d p p d   d g p d  
pd pd

d p p d   n
pitchline velocity (fpm) Vt p d  
2

Wt p d  
H
Transmitted load (lbf)
Vt  p d 

3. Set the application factor, Ka Ka  1

4. Write the equation for the dynamic load factor, Kv

B  0.25  12  Qv
0.6667
A  50  56 ( 1  B) A  83.776
B
B  0.397
Kv p d    A 
 
 A  Vt  p d  
min

 ft 

5. Tentatively choose the mounting factor, Km (Assume 1.99 < F < 6 in) Km  1.7

6. The bending stress equation for the pinion is

Ka Wt p d   p d  Km
σbp p d F  
Kv p d   F  Jp

7. Determine the endurance strength of the pinion.


 0.0323
Life factor KL  1.6831 N KL  1

© 2011 Pearson Education, Inc., Upper Saddle River, NJ. All rights reserved. This publication is protected by Copyright and written permission should be
MACHINE DESIGN - An Integrated Approach, 4th Ed. 12-57-2
Reliability KR  0.7  0.15 log( 1  R) KR  1

Temperature factor KT  1
Material bending strength (psi)
S atp  40000  psi
AISI 4140 Nitrided steel
S atp KL
Endurance strength S fbp  S fbp  40001  psi
KT  KR

8. Write the design equations using the range of face-width to diametral-pitch ratio given in the text, and the
bending stress equation, solved for the unknown face width.

FU  p d  
16
Upper limit
pd

FL  p d  
8
Lower limit
pd
S fb
Safety factor Nfb =
σb

Ka Wt p d   p d  Km Nfb


Face width F  p d  
Kv p d   Jp S fbp

1 1.1 6
9. Plot F(P) vs. p over the range p d   
d in in in

10. From the graph, choose a standard value of 6


p d from Table 12-2.

p d  3  in
1  
F pd
in
11. The calculated value of F is 4
 
FL pd
F  p d   3.055  in
in
12. Round this up to the decimal equivalent of  
FU pd 2
a common fractional value.
in
F  3.250  in

13. Then, the parameters that depend on p d and F are: 0


1 2 3 4 5 6
pd  in
d p p d   6.000  in d g p d   48.000 in
FIGURE 12-57A
Vt p d   226  Wt p d   3647 lbf
ft
min Graph of Face Width and Limits for the Stage
2 Pinion (bending) in Problem 12-57
σbp p d F   15669  psi Kv p d   0.937

The assumption made in step 5 is correct so no further iteration is required.


Stage 1 Pinion - Surface Fatigue
1. Determine the surface geometry factor, I.

© 2011 Pearson Education, Inc., Upper Saddle River, NJ. All rights reserved. This publication is protected by Copyright and written permission should be
MACHINE DESIGN - An Integrated Approach, 4th Ed. 12-57-3

 sin( ϕ)  cos( ϕ)   Ng
I    I  0.1702
 2  Np  Ng

2. Write the equations for pitch diameter, pitchline velocity, and transmitted load in terms of the unknown
diametral pitch, pd. Note that, in Mathcad, unit conversion factors are not included.

Np
Pitch diameter of pinion (in) d p p d  
pd

d p p d   n
pitchline velocity (fpm) Vt p d  
2

Wt p d  
H
Transmitted load (lbf)
Vt  p d 

3. Set the application factor, Ca Ca  1

4. Write the equation for the dynamic load factor, Cv

B  0.25  12  Qv
0.6667
A  50  56 ( 1  B) A  83.776
B
B  0.397
Cv p d    A 
 
 A  Vt  p d  
min

 ft 

5. Tentatively choose the mounting factor, Cm (Assume 1.99 < F < 6 in) Cm  1.7
0.5
6. Choose an elastic coefficient from Table 12-18. (Steel on steel). Cp  2300 psi

7. The surface stress equation for the pinion is

C a W t  p d   C m
σcp p d F   Cp
Cv p d   F  d p p d   I

8. Determine the endurance strength of the pinion.


 0.056
Life factor CL  2.466  N CL  1
Reliability CR  0.7  0.15 log( 1  R) CR  1
Temperature factor CT  1
Material surface strength (psi)
S acp  165000 psi
AISI 4140 Nitrided steel
S acp CL
Endurance strength S fcp  S fcp  164997 psi
CT  CR

9. Write the design equations using the range of face-width to diametral-pitch ratio given on page 740 of the text,
and the bending stress equation, solved for the unknown face width.

FU  p d  
16
Upper limit
pd

© 2011 Pearson Education, Inc., Upper Saddle River, NJ. All rights reserved. This publication is protected by Copyright and written permission should be
MACHINE DESIGN - An Integrated Approach, 4th Ed. 12-57-4

FL  p d  
8
Lower limit
pd

2
 Sfc 
Safety factor Nfc =  
 σc 
C a W t  p d   C m
2
 Cp 
Face width F  p d      Nfs
Cv p d   d p p d   I  S fcp 

1 1.1 6
10. Plot F(P) vs. p over the range p d   
d in in in

11. From the graph, choose a standard value of 6


p d from Table 12-2.

1  
F pd
p d  3  in
in
4
12. The calculated value of F is  
FL pd
F  p d   2.519  in in

13. Round this up to the decimal equivalent of  


FU pd 2
a common fractional value. in
F  2.750  in

14. The bending requirement is governing in 0


1 2 3 4 5 6
this case. The diametral pitch and face
width for the stage 2 gearset are: pd  in
FIGURE 12-57B
1
p d  3  in F  3.250  in
Graph of Face Width and Limits for the Stage 2
Pinion (surface) in Problem 12-57

© 2011 Pearson Education, Inc., Upper Saddle River, NJ. All rights reserved. This publication is protected by Copyright and written permission should be
MACHINE DESIGN - An Integrated Approach, 4th Ed. 12-58-1

PROBLEM 12-58
Statement: Size the third-stage spur gears in problem 12-55 for a bending factor of safety of at least 2.4 and
a surface factor of safety of at least 2.0 assuming a steady torque, 25-deg pressure angle, full
depth teeth, Qv = 10, and AISI 4140 steel for all gears.

Given: Pressure angle ϕ  25 deg


Bending factor of safety Nfb  2.4 Reliability R  0.99
Surface factor of safety Nfs  2.0 AGMA Quality level Qv  10
7
Power to be transmitted (hp) H  25 hp Life (cycles) N  10
Number of pinion teeth Np  18 Number of gear teeth Ng  162
Rotational speed of pinion (rpm) n  18 rpm

Assumptions: Since both pinion and gear from each stage are the same material, it will only be necessary to
determine the pinion size as it will be governing for the set.
Solution: See Mathcad file P1258.

Stage 1 Pinion - Bending


1. Determine the bending geometry factor, J (Table 12-13) Jp  0.39

2. Write the equations for pitch diameter, pitchline velocity, and transmitted load in terms of the unknown
diametral pitch, pd. Note that, in Mathcad, unit conversion factors are not included.

Np Ng
Pitch diameter of pinion and gear (in) d p p d   d g p d  
pd pd

d p p d   n
pitchline velocity (fpm) Vt p d  
2

Wt p d  
H
Transmitted load (lbf)
Vt  p d 

3. Set the application factor, Ka Ka  1

4. Write the equation for the dynamic load factor, Kv

B  0.25  12  Qv
0.6667
A  50  56 ( 1  B) A  83.776
B
B  0.397
Kv p d    A 
 
 A  Vt  p d  
min

 ft 

5. Tentatively choose the mounting factor, Km (Assume 5.99 < F < 6 in) Km  1.8

6. The bending stress equation for the pinion is

Ka Wt p d   p d  Km
σbp p d F  
Kv p d   F  Jp

7. Determine the endurance strength of the pinion.


 0.0323
Life factor KL  1.6831 N KL  1

© 2011 Pearson Education, Inc., Upper Saddle River, NJ. All rights reserved. This publication is protected by Copyright and written permission should be
MACHINE DESIGN - An Integrated Approach, 4th Ed. 12-58-2

Reliability KR  0.7  0.15 log( 1  R) KR  1


Temperature factor KT  1
Material bending strength (psi)
S atp  40000  psi
AISI 4140 Nitrided steel
S atp KL
Endurance strength S fbp  S fbp  40001  psi
KT  KR

8. Write the design equations using the range of face-width to diametral-pitch ratio given in the text, and the
bending stress equation, solved for the unknown face width.

FU  p d  
16
Upper limit
pd

FL  p d  
8
Lower limit
pd
S fb
Safety factor Nfb =
σb

Ka Wt p d   p d  Km Nfb


Face width F  p d  
Kv p d   Jp S fbp

1 1.1 6
9. Plot F(P) vs. p over the range p d   
d in in in

10. From the graph, choose a standard value of 10


p d from Table 12-2.

p d  1.5 in
1  
F pd 8
in
11. The calculated value of F is
 
FL pd 6
F  p d   6.27 in
in

 
4
12. Round this up to the decimal equivalent of FU pd
a common fractional value.
in
2
F  6.500  in

13. Then, the parameters that depend on p d and F are: 0


1 1.25 1.5 1.75 2 2.25 2.5 2.75 3
pd  in
d p p d   12.000 in d g p d   108.000  in
FIGURE 12-58A
Vt p d   57 Wt p d   14589  lbf
ft
min Graph of Face Width and Limits for the Stage 3
Pinion (bending) in Problem 12-58
σbp p d F   16078  psi Kv p d   0.966

The assumption made in step 5 is correct so no further iteration is required.

Stage 1 Pinion - Surface Fatigue


1. Determine the surface geometry factor, I.

© 2011 Pearson Education, Inc., Upper Saddle River, NJ. All rights reserved. This publication is protected by Copyright and written permission should be
MACHINE DESIGN - An Integrated Approach, 4th Ed. 12-58-3

 sin( ϕ)  cos( ϕ)   Ng
I    I  0.1724
 2  Np  Ng
2. Write the equations for pitch diameter, pitchline velocity, and transmitted load in terms of the unknown
diametral pitch, pd. Note that, in Mathcad, unit conversion factors are not included.

Np
Pitch diameter of pinion (in) d p p d  
pd

d p p d   n
pitchline velocity (fpm) Vt p d  
2

Wt p d  
H
Transmitted load (lbf)
Vt  p d 

3. Set the application factor, Ca Ca  1

4. Write the equation for the dynamic load factor, Cv

B  0.25  12  Qv
0.6667
A  50  56 ( 1  B) A  83.776
B
Cv p d    
B  0.397 A

 A  Vt  p d  
min

 ft 

5. Tentatively choose the mounting factor, Cm (Assume 5.99 < F < 9 in) Cm  1.8
0.5
6. Choose an elastic coefficient from Table 12-18. (Steel on steel). Cp  2300 psi

7. The surface stress equation for the pinion is

C a W t  p d   C m
σcp p d F   Cp
Cv p d   F  d p p d   I

8. Determine the endurance strength of the pinion.


 0.056
Life factor CL  2.466  N CL  1
Reliability CR  0.7  0.15 log( 1  R) CR  1
Temperature factor CT  1
Material surface strength (psi)
S acp  165000 psi
AISI 4140 Nitrided steel
S acp CL
Endurance strength S fcp  S fcp  164997 psi
CT  CR

9. Write the design equations using the range of face-width to diametral-pitch ratio given in the text, and the
bending stress equation, solved for the unknown face width.

FU  p d  
16
Upper limit
pd

© 2011 Pearson Education, Inc., Upper Saddle River, NJ. All rights reserved. This publication is protected by Copyright and written permission should be
MACHINE DESIGN - An Integrated Approach, 4th Ed. 12-58-4

FL  p d  
8
Lower limit
pd
2
 Sfc 
Safety factor Nfc =  
 σc 
C a W t  p d   C m
2
 Cp 
Face width F  p d      Nfs
Cv p d   d p p d   I  S fcp 

1 1.1 6
10. Plot F(P) vs. p over the range p d   
d in in in

11. From the graph, choose a standard value of 10


p d from Table 12-2.

1  
F pd 8
p d  1.5 in
in
12. The calculated value of F is  
FL pd 6

F  p d   5.105  in in

 
4
13. Round this up to the decimal equivalent of FU pd
a common fractional value. in
2
F  5.250  in

14. The bending requirement is governing in 0


1 1.25 1.5 1.75 2 2.25 2.5 2.75 3
this case. The diametral pitch and face
width for the stage 3 gearset are: pd  in
FIGURE 12-58B
1
p d  1.5 in F  6.500  in
Graph of Face Width and Limits for the Stage 3
Pinion (surface) in Problem 12-58

© 2011 Pearson Education, Inc., Upper Saddle River, NJ. All rights reserved. This publication is protected by Copyright and written permission should be
MACHINE DESIGN - An Integrated Approach, 4th Ed. 12-59-1

PROBLEM 12-59
Statement: The pinion of an external gearset has pitch radius rp = 40 mm and the gear has pitch radius rg =
160 mm. If the pinion is the input member of the set, determine the velocity ratio, the torque
ratio, and the gear ratio of the set.

Given: Pitch radii rp  40 mm rg  160  mm

Solution: See Mathcad file P1259.

1. Equate the pinion pitch radius to rin and the gear pitch radius to rout (in this case).
rin  rp rin  40 mm rout  rg rout  160 mm

2. Using equation 12.1a, calculate the velocity ratio using the minus sign because this is an external set.

 rin 
Velocity ratio mV    mV  0.25
 rout 
3. Using equation 12.1b, calculate the torque ratio using the minus sign because this is an external set.

1
Torque ratio mA  mA  4
mV

4. Using equation 12.1c, calculate the gear ratio using the torque ratio since it is greater than 1.

Gear ratio mG  mA mG  4

This is usually stated as being 4:1.

© 2011 Pearson Education, Inc., Upper Saddle River, NJ. All rights reserved. This publication is protected by Copyright and written permission should be
MACHINE DESIGN - An Integrated Approach, 4th Ed. 12-60-1

PROBLEM 12-60
Statement: A pinion having 20 teeth and a diametral pitch of 8 (in -1) is in mesh with a rack. If the pinion
rotates one revolution, how far will the rack move?
1
Given: Number of teeth Np  20 Diametral pitch p d  8  in

Solution: See Mathcad file P1260.

1. Calculate the pitch diameter using equation 12-4a.

Np
Pitch diameter d  d  2.500  in
pd

2. The rack and pinion can be modeled as a cylinder, with diameter equal to the pitch diameter of the gear, in
contact with a plane along the pitch line of the rack. When the gear is rotated one revolution, the rack will
move a distance equal to the circumference of the cylinder.

Distance moved s  π d s  7.854 in

© 2011 Pearson Education, Inc., Upper Saddle River, NJ. All rights reserved. This publication is protected by Copyright and written permission should be
MACHINE DESIGN - An Integrated Approach, 4th Ed. 12-61-1

PROBLEM 12-61
Statement: A gearset with full-depth teeth is designed to have a pinion with 24 teeth, a gear with 54 teeth,
and a diametral pitch of 6. Compare the contact ratio for this set for pressure angles of 14.5, 20,
and 25 degrees.

Given: Number of teeth Np  24 Ng  54


1
Diametral pitch p d  6  in

Solution: See Mathcad file P1261.

1. Calculate the pitch radus and addendum for each gear, and the nominal center distamce.

1 Np
Pitch radius: rp   rp  2.000 in
2 pd

1 Ng
rg   rg  4.500 in
2 pd
1
Addendum: a p  a p  0.167 in
pd
a g  a p a g  0.167 in
Center distance C  rp  rg C  6.500 in

2. Write the equation for the length of action as a function of pressure angle using equation 12.2.

Z ( ϕ)  rp  ap 2  rp cos( ϕ)  2   rg  ag2   rg cos( ϕ) 2  C sin( ϕ)


3. Write the equation for the contact ratio as a function of pressure angle using equation 12.7b.

p d  Z ( ϕ)
mp( ϕ) 
π cos( ϕ)

4. Evaluate the contact ratio for the three pressure angles.

14.5 degrees mp( 14.5 deg)  2.007

20 degrees mp( 20 deg)  1.685

25 degrees mp( 25 deg)  1.492

© 2011 Pearson Education, Inc., Upper Saddle River, NJ. All rights reserved. This publication is protected by Copyright and written permission should be
MACHINE DESIGN - An Integrated Approach, 4th Ed. 12-62-1

PROBLEM 12-62 _____

Statement: Design a planetary gear train similar to that shown in Figure 12-16 for an overall velocity ratio
of exactly 5 if the ring gear is stationary, the arm is the input, and the sun gear is the output.
Specify tooth numbers for each gear in the train.

Given: Exact ratio mV  5


Assumptions: The gears will be cut with a hob and the pressure angle will be ϕ  20 deg

Solution: See Figure 12-16 and Mathcad file P1262.

1. Let the first gear be the sun gear (gear 2) and the last be the ring gear (gear 4). Then, equation 12.11c can be
written as

ω4  ωarm N2
=
ω2  ωarm N4

2. Taking 4 = 0 and solving for out/in, we have

ωout ω2 N4
= =  1 = mv
ωin ωa N2

3. Let the sun gear have tooth number N2  20 , then the number of teeth required for the ring gear is

N4  N2  mV  1  N4  80

4. In order to mesh properly, the number of teeth on the ring gear must be equal to the sum of the number of
teeth on the sun gear plus two times the number of teeth on the planet gear(s). Thus,

N4  N2
N3  N3  30
2

5. Check the overall velocity ratio and summarize the tooth numbers.

N4
Overall velocity ratio mV  1 mV  5.000
N2

Sun gear N2  20

Planet gear(s) N3  30

Ring gear N4  80

© 2011 Pearson Education, Inc., Upper Saddle River, NJ. All rights reserved. This publication is protected by Copyright and written permission should be
MACHINE DESIGN - An Integrated Approach, 4th Ed. 12-63-1

PROBLEM 12-63 _____


Statement: A 22-tooth pinion rotating at 1650 rpm meshes with a 66-tooth gear in a spur gear reducer. Both
pinion and gear are manufactured to a quality level of 10. A reliability of 0.9 has been specified,
and the transmitted tangential load is 5000 lb. Conditions are such that Km = 1.7. It is proposed
that standard 25-degree, full-depth teeth be used, with both pinion and gear hobbed from an
AISI 4340 nitrided steel. The diametral pitch is 5, and the face width 2.500 in. Estimate the
number of cycles of bending stress (using the AGMA equations) that the gearset can
withstand.

Given: Tooth numbers Np  22 Ng  66


Pinion speed n  1650 rpm Mounting factor Km  1.7
1
Diametral pitch p d  5  in Quality index Qv  10
Face width F  2.500  in Pressure angle ϕ  25 deg
Tangential load Wt  5000 lbf Reliability R  0.90
Assumptions: The life of the pinion will be less than that of the gear.
Solution: See Mathcad file P1263.
1. Calculate the pitch diameter of the pinion using equation 12.4a and the pitch-line velocity of the gearset.
Np d n ft
d  d  4.400  in V  V  1901
pd 2 min
2. Calculate the dynamic factor Kv using equations 12.16 and 12.17.

B  0.25  12  Qv
0.6667
B  0.397 A  50  56 ( 1  B) A  83.776
B
Kv   A  Kv  0.847
 V  min 

 A   
  ft  
3. Determine the bending geometry factor, J (Table 12-13) J  0.42
4. Assume an application factor, Ka Ka  1

5. Calculate the bending stress using equation 12.15.


Ka Wt p d  Km
σb  σb  47.80  ksi
Kv F  J

6. Determine the material bending fatigue strength from Table 12-20 using the average value. S'fb  41.5 ksi
7. Determine the reliability factor for R  0.90 from Table 12-19. KR  0.85
8. Assume a temperature factor, KT KT  1

9. Set the bending stress equal to the corrected bending-fatigue strength in equation 12.24 and solve for the life
factor, KL .
KT  KR σb
KL  KL  0.979
S'fb
10. Assume that the fatigue life is over 10 6 cycles and use the lower curve in Figure 12-24 to solve for the cycle life
1
 0.0323 0.0323
KL = 1.6831 N
N   1.6831  N  1.93  10
7
 K 
 L 
© 2011 Pearson Education, Inc., Upper Saddle River, NJ. All rights reserved. This publication is protected by Copyright and written permission should be
MACHINE DESIGN - An Integrated Approach, 4th Ed. 12-64-1

PROBLEM 12-64 _____


Statement: A 22-tooth pinion rotating at 1650 rpm meshes with a 66-tooth gear in a spur gear reducer.
Both pinion and gear are manufactured to a quality level of 10. A reliability of 0.9 has been
specified, and the transmitted tangential load is 5000 lb. Conditions are such that Km = 1.7. It
is proposed that standard 25-degree, full-depth teeth be used, with both pinion and gear
hobbed from an AISI 4340 nitrided steel. The diametral pitch is 5, and the face width 2.500 in.
Estimate the number of cycles of contact (surface) stress (using the AGMA equations) that the
gearset can withstand.

Given: Tooth numbers Np  22 Ng  66


Pinion speed n  1650 rpm Mounting factor Cm  1.7
1
Diametral pitch p d  5  in Quality index Qv  10
Face width F  2.500  in Pressure angle ϕ  25 deg
Tangential load Wt  5000 lbf Reliability R  0.90
Assumptions: The life of the pinion will be less than that of the gear.
Solution: See Mathcad file P1264.
1. Calculate the pitch diameter of the pinion using equation 12.4a and the pitch-line velocity of the gearset.
Np d n ft
d  d  4.400  in V  V  1901
pd 2 min
2. Calculate the dynamic factor Kv using equations 12.16 and 12.17.
B  0.25  12  Qv
0.6667
B  0.397 A  50  56 ( 1  B) A  83.776
B
Cv   A  Cv  0.847
 

 A   
V min

  ft  

3. Calculate the geometry factor, I

 sin( ϕ)  cos( ϕ)   Ng
I    I  0.144
 2  Np  Ng
0.5
4. Determine the elastic coefficient from Table 12-18. Cp  2300 psi
5. Assume an application factor, Ca Ca  1

6. Calculate the contact stress using equation 12.21.


1
2
 Ca Wt 1 Cm 
σc  Cp     σc  183.32 ksi
 Cv F  d I 
7. Determine the material surface-fatigue strength from Table 12-21 using the average value. S'fc  162.5  ksi
8. Determine the reliability factor for R  0.90 from Table 12-19. CR  0.85

9. Assume a temperature factor, CT CT  1

10. Set the contact stress equal to the corrected surface-fatigue strength in equation 12.25 and solve for the life
factor, CL .

© 2011 Pearson Education, Inc., Upper Saddle River, NJ. All rights reserved. This publication is protected by Copyright and written permission should be
MACHINE DESIGN - An Integrated Approach, 4th Ed. 12-64-2

CT  CR σc
CL  CL  0.959
S'fc

11. Use the lower curve in Figure 12-26 to solve for the cycle life.

1
0.056
CL = 2.466  N
 0.056
N   2.466  N  2.11  10
7
 C 
 L 

© 2011 Pearson Education, Inc., Upper Saddle River, NJ. All rights reserved. This publication is protected by Copyright and written permission should be
MACHINE DESIGN - An Integrated Approach, 4th Ed. 12-65-1

PROBLEM 12-65
Statement: The pinion of an internal gearset has pitch radius rp = 30 mm and the gear has pitch radius rg =
150 mm. If the pinion is the input member of the set, determine the velocity ratio, the torque
ratio, and the gear ratio of the set.

Given: Pitch radii rp  30 mm rg  150  mm

Solution: See Mathcad file P1265.

1. Equate the pinion pitch radius to rin and the gear pitch radius to rout (in this case).
rin  rp rin  30 mm rout  rg rout  150 mm

2. Using equation 12.1a, calculate the velocity ratio using the plus sign because this is an internal set.

 rin 
Velocity ratio mV  r  mV  0.2
 out 
3. Using equation 12.1b, calculate the torque ratio using the minus sign because this is an external set.

1
Torque ratio mA  mA  5
mV

4. Using equation 12.1c, calculate the gear ratio using the torque ratio since it is greater than 1.

Gear ratio mG  mA mG  5

This is usually stated as being 5:1.

© 2011 Pearson Education, Inc., Upper Saddle River, NJ. All rights reserved. This publication is protected by Copyright and written permission should be
MACHINE DESIGN - An Integrated Approach, 4th Ed. 12-66-1

PROBLEM 12-66
Statement: A pinion having 18 teeth and a diametral pitch of 10 (in -1) is in mesh with a rack. If the rack
moves 1 in, how many degrees will the pinion rotate?
1
Given: Number of teeth Np  18 Diametral pitch p d  10 in

Solution: See Mathcad file P1266.

1. Calculate the pitch diameter using equation 12-4a.

Np
Pitch diameter d  d  1.800  in
pd

2. The rack and pinion can be modeled as a cylinder, with diameter equal to the pitch diameter of the gear, in
contact with a plane along the pitch line of the rack. When the rack is moved 1 in, the pinion will rotate
through an angle θ = d/2s, where d is the diameter of the cylinder and s is the distance that the rack moves.

Distance moved s  1  in

d
Pinion rotation θ  θ  51.566 deg
2 s

© 2011 Pearson Education, Inc., Upper Saddle River, NJ. All rights reserved. This publication is protected by Copyright and written permission should be
MACHINE DESIGN - An Integrated Approach, 4th Ed. 12-67-1

PROBLEM 12-67
Statement: A gearset with 25 deg, full-depth teeth is designed to have a pinion with 24 teeth, a gear with 54
teeth, and a diametral pitch of 6. Compare the contact ratio for this set for a range of center
distances from 0.90C to 1.10C.

Given: Number of teeth Np  24 Ng  54


1
Diametral pitch p d  6  in Pressure angle ϕ  25 deg
Solution: See Mathcad file P1267.

1. Calculate the pitch radus, base radius, and addendum for each gear, and the nominal center distamce.

1 Np
Pitch radius: rp   rp  2.000 in
2 pd

1 Ng
rg   rg  4.500 in
2 pd

Base circle radius: rbp  rp cos( ϕ) rbp  1.813 in

1
Addendum: a p  a p  0.167 in
pd
a g  a p a g  0.167 in
Center distance C  rp  rg C  6.500 in

2. Write the equation for the pressure angle as a function of variation in center distance. Let f be a factor which,
when multiplied by C gives the actual center distance C'. Then, the actual pitch radius of the pinion will be rp'
frp and the actual pressure angle will be ϕ = acos(rbp/frp).

 rbp  Note that ϕ'( 1 )  25 deg


ϕ'( f )  acos 
 f  rp 
3. Write the equation for the length of action as a function of actual pressure angle and center distance using
equation 12.2.

Z ( f )  rp  ap2   rp cos( ϕ'( f ) )  2   rg  ag2   rg cos( ϕ'( f ) )  2  f  C sin( ϕ'( f ) )
4. Write the equation for the contact ratio as a function of pressure angle using equation 12.7b.

pd  Z ( f )
mp( f ) 
π cos( ϕ'( f ) )

5. Evaluate the contact ratio for the range of center distance. Let f  0.90 0.905  1.10

See the graph on the next page.

© 2011 Pearson Education, Inc., Upper Saddle River, NJ. All rights reserved. This publication is protected by Copyright and written permission should be
MACHINE DESIGN - An Integrated Approach, 4th Ed. 12-67-2

CONTACT RATIO vs CHANGE IN CENTER DISANCE


4

3
Contact Ratio

m p( f ) 2

0
0.9 0.95 1 1.05 1.1
f
Change in Center Distance Factor

© 2011 Pearson Education, Inc., Upper Saddle River, NJ. All rights reserved. This publication is protected by Copyright and written permission should be
MACHINE DESIGN - An Integrated Approach, 4th Ed. 12-68-1

PROBLEM 12-68 _____

Statement: Design a planetary gear train similar to that shown in Figure 12-16 for an overall velocity ratio
of exactly 1.25 if the sun gear is stationary, the arm is the input, and the ring gear is the output.
Specify tooth numbers for each gear in the train.

Given: Exact ratio mV  1.25


Assumptions: The gears will be cut with a hob and the pressure angle will be ϕ  20 deg

Solution: See Figure 12-16 and Mathcad file P1268.

1. Let the first gear be the sun gear (gear 2) and the last be the ring gear (gear 4). Then, equation 12.11c can be
written as

ω4  ωarm N2
=
ω2  ωarm N4

2. Taking 2 = 0 and solving for out/in, we have

ωout ω4 N2
= =  1 = mv
ωin ωa N4

3. Let the sun gear have tooth number N2  20 , then the number of teeth required for the ring gear is

N2
N4  N4  80
mV  1
4. In order to mesh properly, the number of teeth on the ring gear must be equal to the sum of the number of
teeth on the sun gear plus two times the number of teeth on the planet gear(s). Thus,

N4  N2
N3  N3  30
2

5. Check the overall velocity ratio and summarize the tooth numbers.

N2
Overall velocity ratio mV  1 mV  1.250
N4

Sun gear N2  20

Planet gear(s) N3  30

Ring gear N4  80

© 2011 Pearson Education, Inc., Upper Saddle River, NJ. All rights reserved. This publication is protected by Copyright and written permission should be
MACHINE DESIGN - An Integrated Approach, 4th Ed. 13-1-1
PROBLEM 13-1
Statement: A 20-deg pressure angle, 30-deg helix angle, 27-tooth helical gear has a diametral pitch p d = 5.
Find the pitch diameter, addendum, dedendum, outside diameter, normal, transverse, and axial
pitch.
Given: Pressure angle ϕ  20 deg Helix angle ψ  30 deg
1
Number of teeth N  27 Diametral pitch p d  5  in

Solution: See Mathcad file P1301.

1. Use equations 13.1 to find the pitch diameter, transverse pitch, normal pitch, and axial pitch.

N
Pitch diameter d  d  5.400  in
pd

π
Transverse pitch p t  p t  0.628  in
pd

Normal pitch p n  p t  cos( ψ) p n  0.544  in

pn
Axial pitch p x  p x  1.088  in
sin( ψ)

2. Use the equations in Table 12-1 to calculate the addendum, dedendum, and outside diameter.
1
Addendum a  a  0.200  in
pd

1.25
Dedendum b  b  0.250  in
pd

Outside diameter D  d  2  a D  5.800  in

© 2011 Pearson Education, Inc., Upper Saddle River, NJ. All rights reserved. This publication is protected by Copyright and written permission should be
MACHINE DESIGN - An Integrated Approach, 4th Ed. 13-2-1
PROBLEM 13-2
Statement: A 25-deg pressure angle, 20-deg helix angle, 43-tooth helical gear has a diametral pitch p d = 8.
Find the pitch diameter, addendum, dedendum, outside diameter, normal, transverse, and axial
pitch.
Given: Pressure angle ϕ  25 deg Helix angle ψ  20 deg
1
Number of teeth N  43 Diametral pitch p d  8  in

Solution: See Mathcad file P1302.

1. Use equations 13.1 to find the pitch diameter, transverse pitch, normal pitch, and axial pitch.

N
Pitch diameter d  d  5.375  in
pd

π
Transverse pitch p t  p t  0.393  in
pd

Normal pitch p n  p t  cos( ψ) p n  0.369  in

pn
Axial pitch p x  p x  1.079  in
sin( ψ)

2. Use the equations in Table 12-1 to calculate the addendum, dedendum, and outside diameter.
1
Addendum a  a  0.125  in
pd

1.25
Dedendum b  b  0.156  in
pd

Outside diameter D  d  2  a D  5.625  in

© 2011 Pearson Education, Inc., Upper Saddle River, NJ. All rights reserved. This publication is protected by Copyright and written permission should be
MACHINE DESIGN - An Integrated Approach, 4th Ed. 13-3-1
PROBLEM 13-3
Statement: A 57-tooth, 10-deg helix angle, helical gear is in mesh with a 23-tooth pinion. The p d = 6 and  =
25 deg. Find the transverse and axial contact ratios.
Given: Pressure angle ϕ  25 deg Helix angle ψ  10 deg
1
Number of pinion teeth Np  23 Diametral pitch p d  6  in
Number of gear teeth Ng  57

Assumptions: The face width factor is fwf  10.


Solution: See Mathcad file P1303.
1. Use equation 12.7 to find the transverse contact ratio.
π
Circular pitch p c  p c  0.524  in
pd

Base pitch p b  p c  cos( ϕ) p b  0.475  in

Pinion:
Np
Pitch dia d p  d p  3.833  in
pd

Pitch rad rp  0.5 d p rp  1.917  in

Gear:
Ng
Pitch dia d g  d g  9.500  in
pd

Pitch rad rg  0.5 d g rg  4.750  in

1.0
Addendum a  a  0.167  in
pd

Np  Ng
Center distance C  C  6.667  in
2 pd

Length of action

Z  rp  a 2  rp cos( ϕ)  2   rg  a 2  rg cos( ϕ)  2  C sin( ϕ)


Z  0.708  in
Z
Transverse contact ratio mp  mp  1.491
pb

2. Use equation 13.5 to find the axial contact ratio.


fwf
Face width F  F  1.667  in
pd

F  p d  tan ( ψ)
Axial contact ratio mF  mF  0.561
π

© 2011 Pearson Education, Inc., Upper Saddle River, NJ. All rights reserved. This publication is protected by Copyright and written permission should be
MACHINE DESIGN - An Integrated Approach, 4th Ed. 13-4-1
PROBLEM 13-4
Statement: A 78-tooth, 30-deg helix angle, helical gear is in mesh with a 27-tooth pinion. The p d = 6 and  =
20 deg. Find the transverse and axial contact ratios.
Given: Pressure angle ϕ  20 deg Helix angle ψ  30 deg
1
Number of pinion teeth Np  27 Diametral pitch p d  6  in
Number of gear teeth Ng  78

Assumptions: The face width factor is fwf  12.


Solution: See Mathcad file P1304.
1. Use equation 12.7 to find the transverse contact ratio.
π
Circular pitch p c  p c  0.524  in
pd

Base pitch p b  p c  cos( ϕ) p b  0.492  in

Pinion:
Np
Pitch dia d p  d p  4.500  in
pd

Pitch rad rp  0.5 d p rp  2.250  in

Gear:
Ng
Pitch dia d g  d g  13.000 in
pd

Pitch rad rg  0.5 d g rg  6.500  in

1.0
Addendum a  a  0.167  in
pd

Np  Ng
Center distance C  C  8.750  in
2 pd

Length of action

Z  rp  a 2  rp cos( ϕ)  2   rg  a 2  rg cos( ϕ)  2  C sin( ϕ)


Z  0.849  in
Z
Transverse contact ratio mp  mp  1.726
pb

2. Use equation 13.5 to find the axial contact ratio.


fwf
Face width F  F  2.000  in
pd

F  p d  tan ( ψ)
Axial contact ratio mF  mF  2.205
π

© 2011 Pearson Education, Inc., Upper Saddle River, NJ. All rights reserved. This publication is protected by Copyright and written permission should be
MACHINE DESIGN - An Integrated Approach, 4th Ed. 13-5-1
PROBLEM 13-5
Statement: A 90-deg straight bevel gearset is needed to give a 9:1 reduction. Determine the pitch cone
angles, pitch diameters, and gear forces if the 25-deg pressure angle pinion has 14 teeth of p d =
6, and the transmitted power is 746 W at 1000 pinion rpm.
Given: Power transmitted H  746  W Gear ratio mG  9
1
Pinion speed ωp  1000 rpm Diametral pitch p d  6  in
Teeth on pinion Np  14 Pressure angle ϕ  25 deg

Solution: See Mathcad file P1305.

1. Use equation 13.7b to calculate number of teeth on the gear. Ng  mG Np Ng  126
2. Use equation 12.4a to calculate the pitch diameters.

Np
Pinion d p  d p  2.333  in
pd

Ng
Gear d g  d g  21.000 in
pd

3. Use equation 13.7b to calculate the pitch cone angles.

αp  atan 
1
Pinion  αp  6.340  deg
 mG 

Gear αg  atan mG αg  83.660 deg

Note that αp  αg  90 deg

4. Determine the torque on the pinion shaft and the transmitted force.
H
Pinion Tp  Tp  63.051 in lbf
ωp

2  Tp
Transmitted force Wt  Wt  54.043 lbf
dp

5. Use equations 13.8a to calculate the pinion and gear forces.

Pinion Wap  Wt tan ( ϕ)  sin αp Wap  2.78 lbf

Wrp  Wt tan( ϕ)  cos αp Wrp  25.05  lbf

Gear Wag  Wt tan ( ϕ)  sin αg Wag  25.05  lbf

Wrg  Wt tan( ϕ)  cos αg Wrg  2.78 lbf


1
Total force W  Wt ( cos( ϕ) ) W  59.63  lbf

© 2011 Pearson Education, Inc., Upper Saddle River, NJ. All rights reserved. This publication is protected by Copyright and written permission should be
MACHINE DESIGN - An Integrated Approach, 4th Ed. 13-6-1
PROBLEM 13-6
Statement: A 90-deg straight bevel gearset is needed to give a 4.5:1 reduction. Determine the pitch cone
angles, pitch diameters, and gear forces if the 20-deg pressure angle pinion has 18 teeth of p d =
5, and the transmitted power is 7460 W at 800 pinion rpm.
Given: Power transmitted H  7460 W Gear ratio mG  4.5
1
Pinion speed ωp  800  rpm Diametral pitch p d  5  in
Teeth on pinion Np  18 Pressure angle ϕ  20 deg

Solution: See Mathcad file P1306.

1. Use equation 13.7b to calculate number of teeth on the gear. Ng  mG Np Ng  81


2. Use equation 12.4a to calculate the pitch diameters.

Np
Pinion d p  d p  3.600  in
pd

Ng
Gear d g  d g  16.200 in
pd

3. Use equation 13.7b to calculate the pitch cone angles.

αp  atan 
1
Pinion  αp  12.529 deg
 mG 

Gear αg  atan mG αg  77.471 deg

Note that αp  αg  90 deg

4. Determine the torque on the pinion shaft and the transmitted force.
H
Pinion Tp  Tp  788.134  in lbf
ωp

2  Tp
Transmitted force Wt  Wt  437.852  lbf
dp

5. Use equations 13.8a to calculate the pinion and gear forces.

Pinion Wap  Wt tan ( ϕ)  sin αp Wap  34.57  lbf

Wrp  Wt tan( ϕ)  cos αp Wrp  155.57 lbf

Gear Wag  Wt tan ( ϕ)  sin αg Wag  155.57 lbf

Wrg  Wt tan( ϕ)  cos αg Wrg  34.57  lbf


1
Total force W  Wt ( cos( ϕ) ) W  465.953  lbf

© 2011 Pearson Education, Inc., Upper Saddle River, NJ. All rights reserved. This publication is protected by Copyright and written permission should be
MACHINE DESIGN - An Integrated Approach, 4th Ed. 13-7-1
PROBLEM 13-7
Statement: A 90-deg straight bevel gearset is needed to give a 5:1 reduction. Determine the pitch cone
angles, pitch diameters, and gear forces if the 20-deg pressure angle pinion has 16 teeth of p d =
7, and the transmitted power is 3 hp at 600 pinion rpm.

Given: Power transmitted H  3  hp Gear ratio mG  5


1
Pinion speed ωp  600  rpm Diametral pitch p d  7  in
Teeth on pinion Np  16 Pressure angle ϕ  20 deg

Assumptions: The spiral angle is ψ  35 deg


Solution: See Mathcad file P1307.
1. Use equation 13.7b to calculate number of teeth on the gear. Ng  mG Np Ng  80
2. Use equation 12.4a to calculate the pitch diameters.

Np
Pinion d p  d p  2.286  in
pd

Ng
Gear d g  d g  11.429 in
pd

3. Use equation 13.7b to calculate the pitch cone angles.

αp  atan 
1
Pinion  αp  11.310 deg
 mG 

Gear αg  atan mG αg  78.690 deg

Note that αp  αg  90 deg

4. Determine the torque on the pinion shaft and the transmitted force.
H
Pinion Tp  Tp  315.127  in lbf
ωp

2  Tp
Transmitted force Wt  Wt  275.736  lbf
dp

5. Use equations 13.2 to calculate the pressure angle in the normal plane.

ϕn  atan( tan( ϕ)  cos( ψ) ) ϕn  16.602 deg

6. Use equations 13.8b to calculate the pinion and gear forces.


Pinion
Wt
Wap    tan ϕn  sin αp  sin( ψ)  cos αp  Wap  169.64 lbf
cos( ψ)
Wt
Wrp    tan  ϕn  cos αp  sin( ψ)  sin αp  Wrp  136.28 lbf
cos( ψ)
© 2011 Pearson Education, Inc., Upper Saddle River, NJ. All rights reserved. This publication is protected by Copyright and written permission should be
MACHINE DESIGN - An Integrated Approach, 4th Ed. 13-7-2

Gear
Wt
Wag    tan ϕn  sin αg  sin( ψ)  cos αg  Wag  60.55  lbf
cos( ψ)
Wt
Wrg    tan  ϕn  cos αg  sin( ψ)  sin αg  Wrg  209.01 lbf
cos( ψ)

1
Total force W  Wt ( cos( ϕ) ) W  293.432  lbf

© 2011 Pearson Education, Inc., Upper Saddle River, NJ. All rights reserved. This publication is protected by Copyright and written permission should be
MACHINE DESIGN - An Integrated Approach, 4th Ed. 13-8-1

PROBLEM 13-8
Statement: A paper mill processes rolls of paper having a density of 984 kg/m3. The paper roll is 1.50-m
outside diameter (OD) x 0.22-m inside diameter (ID) x 3.23-m long and is on a simple supported,
hollow, steel shaft with S ut = 400 MPa. Design a 2.5:1 reduction helical gearset to drive this roll
shaft to obtain a minimum dynamic safety factor of 2 for 10-year life if the roll turns at 50 rpm
with 1.2 hp absorbed.
Given: Surface factor of safety Nfc  2 Life (years) Life  10 yr
Power to be transmitted H  1.2 hp Gear speed n g  50 rpm
Gear ratio mg  2.5

Assumptions: 1. If both pinion and gear are the same material, it will only be necessary to determine the pinion
size as it will be governing for the set.
2. If the gears are not surface hardened, it will only be necessary to design to the surface
requirement as it will be governing for both bending and surface stresses.

Design Choices:
Pressure angle ϕ  25 deg AGMA Quality level Qv  11
Number of pinion teeth Np  14 Relibility R  0.99
Helix angle ψ  20 deg
Material: AGMA Grade 1 steel for both pinion and gear, through hardened to
HB  230 S ac  ( 26000  327  HB)  psi S ac  101210 psi

Solution: See Mathcad file P1308.


1. Determine the number of gear teeth, the pinion speed, and the cycle life.
Number of gear teeth Ng  Np mg Ng  35

Rotational speed of pinion (rpm) n p  n g mg n p  125  rpm

np 8
Cycle life: N  Life N  6.574  10
2 π
2. Determine the surface geometry factor, I.

a  p d  
1
Addendum
pd

Np Ng
Pitch radii rp p d   rg p d  
2 pd 2 pd

Center distance C p d   rp p d   rg p d 

Radii of curvature

ρp  p d   0.5  rp p d   C pd   rg pd     rp p d   cos( ϕ) 


2 2

ρg  p d   C p d   sin( ϕ)  ρp  p d 

Normal plane pressure angle ϕn  atan( tan( ϕ)  cos( ψ) ) ϕn  23.662 deg

 cos ϕn 
Base helix angle ψb  acos cos( ψ)   ψb  18.256 deg
 cos( ϕ) 
© 2011 Pearson Education, Inc., Upper Saddle River, NJ. All rights reserved. This publication is protected by Copyright and written permission should be
MACHINE DESIGN - An Integrated Approach, 4th Ed. 13-8-2

Length of action

Z  p d  
  r  p   a  p   2   r  p   cos( ϕ)  2  
 p d d p d 
 r p  a p 2  r p  cos( ϕ) 2  C p  sin( ϕ) 
  g d   d    g d    d 

Z pd 
Transverse contact ratio mp p d  
π
 cos( ϕ)
pd

F  p d  tan( ψ)
Axial contact ratio mF  p d F  
π

Fractional parts of mp and mF n r p d   mp p d   floor  mp p d  

n a p d F   mF  p d F   floor  mF  p d F  

p t  p d   p n  p d   p t  p d   cos( ψ)
π
Axial pitch
pd

pn  pd 
p x  p d  
sin( ψ)

Minimum length of lines of contact

mp pd  F  napd F  nrpd   px pd  


Lmin p d F   return if n a p d F    1  n r p d  
cos ψb

mp pd   F   1  na p d F     1  nr pd    px  p d 


return if n a p d F    1  n r p d  
cos ψb

mN  p d F  
F
Load sharing ratio
Lmin p d F 

I  p d F  
cos( ϕ)
Geometry factor
 1  1   2 r  p   m  p F 
 ρ p  ρ p   p d N d
 p d g d 

3. Write the equations for pitch diameter, pitchline velocity, and transmitted load in terms of the unknown diametr
pitch, p d. Note that, in Mathcad, unit conversion factors are not included.
Np Ng
Pitch diameter of pinion (in) d p p d   d g p d  
pd pd

d p p d   n p
pitchline velocity (fpm) Vt p d  
2

Wt p d  
H
Transmitted load (lbf)
Vt  p d 
© 2011 Pearson Education, Inc., Upper Saddle River, NJ. All rights reserved. This publication is protected by Copyright and written permission should be
MACHINE DESIGN - An Integrated Approach, 4th Ed. 13-8-3

4. Set the application factor, Ca Ca  1

B  0.25  12  Qv
0.6667
5. Write the equation for the dynamic load factor, C B  0.250
v

A  50  56 ( 1  B) A  92.000
B
Cv p d    A 
 
 A  Vt  p d  
min

 ft 

6. Tentatively choose the mounting factor, Cm (Assume 2 < F < 6 in Cm  1.7


0.5
7. Choose an elastic coefficient from Table 12-18 (steel on steel). Cp  2300 psi

C a W t  p d   C m
8. The surface stress equation for the pinion is σcp p d F   Cp
Cv p d   F  d p p d   I  p d F 
9. Determine the endurance strength of the pinion.
 0.056
Life factor CL  2.466  N CL  0.791
Reliability CR  0.7  0.15 log( 1  R) CR  1.000
Temperature factor CT  1
Material surface strength (psi)
S ac  101210 psi
Grade 1, 230 HB
S ac CL
Endurance strength S fcp  S fcp  80060  psi
CT  CR

10. Write the design equations using the range of face-width to diametral-pitch ratio given in the text, and the
bending stress equation, solved for the unknown face width.

FU  p d  
16
Upper limit
pd

FL  p d  
8
Lower limit
pd

2
 Sfc 
Safety factor Nfc =  
 σc 

C a W t  p d   C m
2
 Cp 
Face width F' p d F      Nfc
Cv p d   d p p d   I  p d F   S fcp 

Guess value for F F  2.35 in Found by iteration with value in step 11.

© 2011 Pearson Education, Inc., Upper Saddle River, NJ. All rights reserved. This publication is protected by Copyright and written permission should be
MACHINE DESIGN - An Integrated Approach, 4th Ed. 13-8-4

2 2.5 12
9. Plot F(P) vs. p d over the p d   
range in in in
4

10. From the graph, choose a standard value of


p d from Table 12-2. 
F' pd F 
3
in
1
p d  5  in
 
FL pd
11. The calculated value of F is 2
in
F' p d F   2.355  in FU pd  
in 1
12. Round this up to the decimal equivalent of a
common fractional value.

F  2.375  in 0
2 3 4 5 6 7 8 9 10 11 12
13. Then, the parameters that depend on p d and F are:
pd  in

d p p d   2.800  in d g p d   7.000  in FIGURE 13-8


Graph of Face Width and Limits for the Pinion
(surface) in Problem 13-8
Vt p d   92 Wt p d   432  lbf
ft
min

σcp p d F   56448  psi mp p d   1.420 mF  p d F   1.376 n r p d   0.420

n a p d F   0.376 mN  p d F   0.728 Lmin p d F   3.264  in I  p d F   0.188

14. The realized factor of safety against surface failure in the pinion is

2
 Sfcp 
Nf  p d F   σ  Nf  p d F   2.0
 cp pd F  

© 2011 Pearson Education, Inc., Upper Saddle River, NJ. All rights reserved. This publication is protected by Copyright and written permission should be
MACHINE DESIGN - An Integrated Approach, 4th Ed. 13-9-1

PROBLEM 13-9
Statement: A 2-start wormset has the dimensions given below. Find the lead, lead angle, worm gear
diameter, and center distance. Will it self-lock? The input speed is 2200 rpm.

Given: Worm pitch diameter d  50 mm Gear ratio mG  22


Axial pitch p x  10 mm Input (worm) speed n w  2200 rpm
Threads on worm Nw  2

Solution: See Mathcad file P1309.


1. Use equation 13.13 to calculate the lead. L  p x  Nw L  20 mm

λ  atan 
L
2. Use equation 13.12 to calculate the lead angle.  λ  7.256  deg
 π d 
3. Calculate the number of teeth on the gear Ng  mG Nw Ng  44
4. Use equation 13.13 to calculate gear diameter. p c  p x

p c  Ng
d g  d g  140.06 mm
π
d  dg
5. Use equation 13.17 to calculate the center distance. C  C  95.03  mm
2
6. If the lead angle per worm tooth is less than 6 deg, the set is self-locking. The lead angle per tooth is

λ
λt  λt  3.628  deg Self-locking
Nw

© 2011 Pearson Education, Inc., Upper Saddle River, NJ. All rights reserved. This publication is protected by Copyright and written permission should be
MACHINE DESIGN - An Integrated Approach, 4th Ed. 13-10-1
PROBLEM 13-10
Statement: A 3-start wormset has the dimensions given below. Find the lead, lead angle, worm gear
diameter, and center distance. Will it self-lock? The input speed is 1400 rpm.

Given: Worm pitch diameter d  1.750  in Gear ratio mG  17


Axial pitch p x  0.200  in Input (worm) speed n w  1400 rpm
Threads on worm Nw  3

Solution: See Mathcad file P1310.


1. Use equation 13.13 to calculate the lead. L  p x  Nw L  0.600  in

λ  atan 
L
2. Use equation 13.12 to calculate the lead angle.  λ  6.228  deg
 π d 
3. Calculate the number of teeth on the gear Ng  mG Nw Ng  51
4. Use equation 13.13 to calculate gear diameter. p c  p x

p c  Ng
d g  d g  3.247  in
π
d  dg
5. Use equation 13.17 to calculate the center distance. C  C  2.498  in
2
6. If the lead angle per worm tooth is less than 6 deg, the set is self-locking. The lead angle per tooth is

λ
λt  λt  2.076  deg Self-locking
Nw

© 2011 Pearson Education, Inc., Upper Saddle River, NJ. All rights reserved. This publication is protected by Copyright and written permission should be
MACHINE DESIGN - An Integrated Approach, 4th Ed. 13-11-1

PROBLEM 13-11
Statement: A 1-start wormset has the dimensions given below. Find the lead, lead angle, worm gear
diameter, and center distance. Will it self-lock? The input speed is 4500 rpm.
Given: Worm pitch diameter d  40 mm Gear ratio mG  82
Axial pitch p x  5  mm Input (worm) speed n w  4500 rpm
Threads on worm Nw  1

Solution: See Mathcad file P1311.


1. Use equation 13.13 to calculate the lead. L  p x  Nw L  5  mm

λ  atan 
L
2. Use equation 13.12 to calculate the lead angle.  λ  2.279  deg
 π d 
3. Calculate the number of teeth on the gear Ng  mG Nw Ng  82
4. Use equation 13.13 to calculate gear diameter. p c  p x

p c  Ng
d g  d g  130.51 mm
π
d  dg
5. Use equation 13.17 to calculate the center distance. C  C  85.25  mm
2
6. If the lead angle per worm tooth is less than 6 deg, the set is self-locking. The lead angle per tooth is

λ
λt  λt  2.279  deg Self-locking
Nw

© 2011 Pearson Education, Inc., Upper Saddle River, NJ. All rights reserved. This publication is protected by Copyright and written permission should be
MACHINE DESIGN - An Integrated Approach, 4th Ed. 13-12-1
PROBLEM 13-12
Statement: Determine the power transmitted and the torques and forces in the mesh for the wormset in Problem
13-9 if it turns at 1000 worm rpm.
1
Units: fpm  ft  min

Given: Worm pitch diameter d w  50 mm Gear ratio mG  22


Axial pitch p x  10 mm Input (worm) speed n w  1000 rpm
Threads on worm Nw  2 Pressure angle ϕ  20 deg

Solution: See Mathcad file P1312.


1. Use equation 13.13 to calculate the lead. L  p x  Nw L  20 mm

λ  atan 
L
2. Use equation 13.12 to calculate the lead angle.  λ  7.256  deg
 π d w 
3. Calculate the number of teeth on the gear Ng  mG Nw Ng  44

4. Use equation 13.13 to calculate gear diameter. p c  p x


p c  Ng
5. Use equation 13.17 to calculate the center distance. d g  d g  140.06 mm
π
dw  dg
C  C  95.03  mm
2
6. Find the maximum recommended face width from equation 13.19.
Fmax  0.67 d w Fmax  33.5 mm
7. Find the materials factor Cs from equation 13.24. Since C < 203 mm, Cs  1000.

8. Find the ratio correction factor Cm from equations 13.25. Based on mG  22 , the second of the
expressions in that equation set will be used.
2
Cm  0.0107 mG  56 mG  5145 Cm  0.821

9. Find the tangential velocity Vt from equation 13.27.


nw dw
Vt  Vt  519.5  fpm
2  cos( λ)
10. Use this velocity to find the velocity factor Cv from equations 13.26. For this value of Vt, the first of these
equations is appropriate.

Vt
 0.0011
fpm
Cv  0.659  e Cv  0.372

11. Find the tangential load Wt from equation 13.23.


0.8
 dg  N
Wtg  Cs Cm Cv    Fmax Wtg  7028 N
 mm  75.948 mm

12. Find the coefficient of friction from the third expression in equation 13.29.
0.450
 Vt 
 0.110  
μ  0.103  e  fpm   0.012 μ  0.028
© 2011 Pearson Education, Inc., Upper Saddle River, NJ. All rights reserved. This publication is protected by Copyright and written permission should be
MACHINE DESIGN - An Integrated Approach, 4th Ed. 13-12-2

13. Find the friction force Wf from equation 13.28.

μ  Wtg
Wf  Wf  214.5  N
cos( λ)  cos( ϕ)

14. Find the rated output power from equation 13.21.

nw dg
Φ o    Wtg Φ o  2.34 kW
mG 2

15. Find the power lost in the mesh from equation 13.22.
Φ l  Vt Wf Φ l  0.57 kW
16. Find the rated input power from equation 13.20.
Φ  Φ o  Φ l Φ  2.91 kW

17. The efficiency of the gearset is


Φo
e  e  80.5 %
Φ
18. Find the rated output torque from equation 13.31.

dg
Tg  Wtg Tg  492  N  m
2

© 2011 Pearson Education, Inc., Upper Saddle River, NJ. All rights reserved. This publication is protected by Copyright and written permission should be
MACHINE DESIGN - An Integrated Approach, 4th Ed. 13-13-1
PROBLEM 13-13
Statement: Determine the power transmitted and the torques and forces in the mesh for the wormset in
Problem 13-10 if it turns at 500 worm rpm.
1
Units: fpm  ft  min
Given: Worm pitch diameter d w  1.75 in Gear ratio mG  17
Axial pitch p x  0.200  in Input (worm) speed n w  500  rpm
Threads on worm Nw  3 Pressure angle ϕ  20 deg

Solution: See Mathcad file P1313.


1. Use equation 13.13 to calculate the lead. L  p x  Nw L  0.600  in

λ  atan 
L
2. Use equation 13.12 to calculate the lead angle.  λ  6.228  deg
 π d w 
3. Calculate the number of teeth on the gear Ng  mG Nw Ng  51

4. Use equation 13.13 to calculate gear diameter. p c  p x


p c  Ng
5. Use equation 13.17 to calculate the center distance. d g  d g  3.247  in
π
dw  dg
C  C  2.498  in
2
6. Find the maximum recommended face width from equation 13.19.
Fmax  0.67 d w Fmax  1.173  in
7. Find the materials factor Cs from equation 13.24. Since C < 8 in, Cs  1000.

8. Find the ratio correction factor Cm from equations 13.25. Based on mG  17 , the first of the expressions in
that equation set will be used.
2
Cm  0.0200 mG  40 mG  76  0.46 Cm  0.815

9. Find the tangential velocity Vt from equation 13.27.

nw dw
Vt  Vt  230.4  fpm
2  cos( λ)

10. Use this velocity to find the velocity factor Cv from equations 13.26. For this value of Vt, the first of these
equations is appropriate.

Vt
 0.0011
fpm
Cv  0.659  e Cv  0.511

11. Find the tangential load Wt from equation 13.23.


0.8
 dg  lbf
Wtg  Cs Cm Cv    Fmax Wtg  1254 lbf
 in  in

12. Find the coefficient of friction from the third expression in equation 13.29.
0.450
 
Vt
 0.110  
μ  0.103  e  fpm   0.012 μ  0.041
© 2011 Pearson Education, Inc., Upper Saddle River, NJ. All rights reserved. This publication is protected by Copyright and written permission should be
MACHINE DESIGN - An Integrated Approach, 4th Ed. 13-13-2

13. Find the friction force Wf from equation 13.28.

μ  Wtg
Wf  Wf  54.8 lbf
cos( λ)  cos( ϕ)

14. Find the rated output power from equation 13.21.

nw dg
Φ o    Wtg Φ o  0.95 hp
mG 2

15. Find the power lost in the mesh from equation 13.22.
Φ l  Vt Wf Φ l  0.38 hp
16. Find the rated input power from equation 13.20.
Φ  Φ o  Φ l Φ  1.33 hp

17. The efficiency of the gearset is


Φo
e  e  71.3 %
Φ
18. Find the rated output torque from equation 13.31.

dg
Tg  Wtg Tg  2035 in lbf
2

© 2011 Pearson Education, Inc., Upper Saddle River, NJ. All rights reserved. This publication is protected by Copyright and written permission should be
MACHINE DESIGN - An Integrated Approach, 4th Ed. 13-14-1
PROBLEM 13-14
Statement: If the gearset in Problem 13-3 transmits 125 HP at 1000 pinion rpm, find the torque on each shaft.

Given: Tooth numbers:


Number of pinion teeth Np  23 Pinion speed ωp  1000 rpm
Number of gear teeth Ng  57 Transmitted power P  125  hp

Assumptions: There is no loss of power in the gear mesh (100% efficiency).


Solution: See Mathcad file P1314.
Solution: See Mathcad file P1214. P
1. For the pinion shaft Tp  Tp  7878 in lbf Tp  656.5  ft  lbf
ωp

2. The gear shaft will rotate at a lower speed, which is determined by the gear ratio. (The speed will be
decreased in proportion to the ratio and the torque will be increased by the reciprocal of the ratio).
Np
3. For the gear shaft ωg   ωp ωg  403.509  rpm
Ng
P
Tg  Tg  19524  in lbf Tg  1627 ft  lbf
ωg

4. We could have calculated the torque on the gear shaft directly without finding the gear shaft speed,

Ng
Tg   Tp Tg  19524  in lbf Tg  1627 ft  lbf
Np

© 2011 Pearson Education, Inc., Upper Saddle River, NJ. All rights reserved. This publication is protected by Copyright and written permission should be
MACHINE DESIGN - An Integrated Approach, 4th Ed. 13-15-1
PROBLEM 13-15
Statement: If the gearset in Problem 13-4 transmits 33 kW at 1600 pinion rpm, find the torque on each shaft.

Given: Tooth numbers:


Number of pinion teeth Np  27 Pinion speed ωp  1600 rpm
Number of gear teeth Ng  78 Transmitted power P  33 kW

Assumptions: There is no loss of power in the gear mesh (100% efficiency).


Solution: See Mathcad file P1315.
P
1. For the pinion shaft Tp  Tp  197  N  m
ωp

2. The gear shaft will rotate at a lower speed, which is determined by the gear ratio. (The speed will be
decreased in proportion to the ratio and the torque will be increased by the reciprocal of the ratio).
Np
3. For the gear shaft ωg   ωp ωg  553.846  rpm
Ng
P
Tg  Tg  569  N  m
ωg

4. We could have calculated the torque on the gear shaft directly without finding the gear shaft speed,

Ng
Tg   Tp Tg  569  N  m
Np

© 2011 Pearson Education, Inc., Upper Saddle River, NJ. All rights reserved. This publication is protected by Copyright and written permission should be
MACHINE DESIGN - An Integrated Approach, 4th Ed. 13-16-1

PROBLEM 13-16
Statement: Size the helical gears in problem 13-14 for a bending factor of safety of at least 2 assuming a
steady torque, 25 deg pressure angle, full depth teeth, a face width factor of 10, quality index of
9, an AISI 4140 steel pinion, and a class 40 cast iron gear.

Given: Factor of safety Nfb  2 Number of gear teeth Ng  57


Power to be transmitted H  125  hp Reliability R  0.99
Rotational speed of pinion n  1000 rpm AGMA Quality level Qv  9
7
Number of pinion teeth Np  23 Life (cycles) N  10
Pressure angle ϕ  25 deg Helix angle ψ  10 deg
Solution: See Mathcad file P1316.
Pinion
1. Determine the bending geometry factor, J (Table 13-4) Jp  0.57
2. Write the equations for pitch diameter, pitchline velocity, and transmitted load in terms of the unknown
diametral pitch, p d. Note that, in Mathcad, unit conversion factors are not included.
Np
Pitch diameter of pinion (in) d  p d  
pd

dpd  n
pitchline velocity (fpm) Vt p d  
2

Wt p d  
H
Transmitted load (lbf)
Vt  p d 

3. Set the application factor, Ka Ka  1

B  0.25  12  Qv
0.6667
4. Write the equation for the dynamic load factor, K B  0.52
v

A  50  56 ( 1  B) A  76.878
B
Kv p d    
A

 A  Vt  p d  
min

 ft 

5. Tentatively choose the mounting factor, Km (Assume 2 < F < 6 in) Km  1.7

Ka Wt p d   p d  Km
6. The bending stress equation for the pinion is σbp p d F  
Kv p d   F  Jp

7. Determine the endurance strength of the pinion.


 0.0323
Life factor KL  1.6831 N KL  1
Reliability KR  0.7  0.15 log( 1  R) KR  1
Temperature factor KT  1
Material bending strength (psi)
S atp  40000  psi
AISI 4140 Nitrided steel

© 2011 Pearson Education, Inc., Upper Saddle River, NJ. All rights reserved. This publication is protected by Copyright and written permission should be
MACHINE DESIGN - An Integrated Approach, 4th Ed. 13-16-2

S atp KL
Endurance strength S fbp  S fbp  40001  psi
KT  KR

8. Write the design equations using the range of face-width to diametral-pitch ratio given in the text, and the
bending stress equation, solved for the unknown face width.

FU  p d  
16
Upper limit
pd

FL  p d  
8
Lower limit
pd
S fb
Safety factor Nfb =
σb

Ka Wt p d   p d  Km Nfb


Face width F  p d  
Kv p d   Jp S fbp

1 1.5 6
9. Plot F(P) vs. p d over the p d   
range in in in
4
10. From the graph, choose a standard value of
p d from Table 11-2.
 
F pd
1 3
p d  5  in in

11. The calculated value of F is  


FL pd
2
F  p d   3.100  in
in

 
FU pd
12. Round this up to the decimal equivalent of a
in 1
common fractional value.

F  3.125  in
0
13. Then, the parameters that depend on p d and F are: 2 2.5 3 3.5 4 4.5 5 5.5 6
pd  in
d  p d   4.600  in Kv p d   0.824
FIGURE 13-16A
Graph of Face Width and Limits for the Pinion in
Vt p d   1204 Wt p d   3425 lbf
ft
Problem 13-16
min

σbp p d F   19839  psi

The assumption made in step 5 is correct so no further iteration is required.


Gear

14. Determine the bending geometry factor, J (Table 13-4) Jg  0.63


Ka Wt p d   p d  Km
15. The bending stress equation for the gear is σbg p d F  
Kv p d   F  Jg

16. Determine the endurance strength of the pinion.

© 2011 Pearson Education, Inc., Upper Saddle River, NJ. All rights reserved. This publication is protected by Copyright and written permission should be
MACHINE DESIGN - An Integrated Approach, 4th Ed. 13-16-3

Material bending strength (psi)


Class 40 cast iron S atg  13000  psi

S atg KL
Endurance strength S fbg  S fbg  13000  psi
KT  KR

17. Write the design equations using the range of face-width to diametral-pitch ratio given in the text, and the
bending stress equation, solved for the unknown face width.

FU  p d  
16
Upper limit
pd

FL  p d  
8
Lower limit
pd
S fb
Safety factor Nfb =
σb

Ka Wt p d   p d  Km Nfb


Face width F  p d  
Kv p d   Jg S fbg

1 1.5 6
18. Plot F(P) vs. p d over the range p d   
in in in

19. From the graph, choose a standard value of 6


p d from Table 11-2.

p d  3  in
1  
F pd
4.5
in
20. The calculated value of F is
 
FL pd
F  p d   3.250  in 3
in
21. Round this up to the decimal equivalent of a  
FU pd
common fractional value.
in 1.5
F  3.250  in

22. Then, the parameters that depend on P and F are: 0


2 2.5 3 3.5 4 4.5 5 5.5 6
d  p d   7.667  in Kv p d   0.788 pd  in
FIGURE 13-16B
Graph of Face Width and Limits for the Gear in
Vt p d   2007 Wt p d   2055 lbf
ft
Problem 13-16
min

σbg p d F   6500 psi

23. The gear dimensions are larger (smaller diametral pitch means bigger teeth) than for the pinion. This means that
we will accept the gear requirements for the pinion, thus, for the set

1
Diametral pitch p d  3  in

Face width F  3.250  in


24. Determine the realized factor of safety for the gear using the above values for F and p d.

© 2011 Pearson Education, Inc., Upper Saddle River, NJ. All rights reserved. This publication is protected by Copyright and written permission should be
MACHINE DESIGN - An Integrated Approach, 4th Ed. 13-16-4

S fbg
Gear factor of safety Nfbg  Nfbg  2.0
σbg p d F 

25. Check the factor of safety on the pinion:

S fbp
Pinion factor of safety Nfbp  Nfbp  5.6
σbp p d F 

© 2011 Pearson Education, Inc., Upper Saddle River, NJ. All rights reserved. This publication is protected by Copyright and written permission should be
MACHINE DESIGN - An Integrated Approach, 4th Ed. 13-17-1

PROBLEM 13-17
Statement: Size the helical gears in problem 13-15 for a bending factor of safety of at least 2.5 assuming a
steady torque, 20 deg pressure angle, full depth teeth, a face width factor of 12, quality index of
11, an AISI 4340 steel pinion, and an A-7-d nodular 40 iron gear.

Given: Factor of safety Nfb  2.5 Number of gear teeth Ng  78


Power to be transmitted H  33 kW Reliability R  0.99
Rotational speed of pinion n  1600 rpm AGMA Quality level Qv  11
7
Number of pinion teeth Np  27 Life (cycles) N  10
Pressure angle ϕ  20 deg Helix angle ψ  30 deg
Solution: See Mathcad file P1317.
Pinion
1. Determine the bending geometry factor, J (Table 13-4) Jp  0.48
2. Write the equations for pitch diameter, pitchline velocity, and transmitted load in terms of the unknown
diametral pitch, p d. Note that, in Mathcad, unit conversion factors are not included.
Np
Pitch diameter of pinion (in) d  p d  
pd

dpd  n
pitchline velocity (fpm) Vt p d  
2

Wt p d  
H
Transmitted load (lbf)
Vt  p d 

3. Set the application factor, Ka Ka  1

B  0.25  12  Qv
0.6667
4. Write the equation for the dynamic load factor, K B  0.25
v

A  50  56 ( 1  B) A  92
B
Kv p d    
A

 A  Vt  p d  
min

 ft 

5. Tentatively choose the mounting factor, Km (Assume 2 < F < 6 in) Km  1.7

Ka Wt p d   p d  Km
6. The bending stress equation for the pinion is σbp p d F  
Kv p d   F  Jp

7. Determine the endurance strength of the pinion.


 0.0323
Life factor KL  1.6831 N KL  1
Reliability KR  0.7  0.15 log( 1  R) KR  1
Temperature factor KT  1
Material bending strength (psi)
S atp  42000  psi
AISI 4340 Nitrided steel

© 2011 Pearson Education, Inc., Upper Saddle River, NJ. All rights reserved. This publication is protected by Copyright and written permission should be
MACHINE DESIGN - An Integrated Approach, 4th Ed. 13-17-2

S atp KL
Endurance strength S fbp  S fbp  42001  psi
KT  KR

8. Write the design equations using the range of face-width to diametral-pitch ratio given in the text, and the
bending stress equation, solved for the unknown face width.

FU  p d  
16
Upper limit
pd

FL  p d  
8
Lower limit
pd
S fb
Safety factor Nfb =
σb

Ka Wt p d   p d  Km Nfb


Face width F  p d  
Kv p d   Jp S fbp

1 1.5 10
9. Plot F(P) vs. p d over the p d   
range in in in
4
10. From the graph, choose a standard value of
p d from Table 12-2.
 
F pd
1 3
p d  8  in in

11. The calculated value of F is  


FL pd
2
F  p d   1.898  in
in

 
FU pd
12. Round this up to the decimal equivalent of a
in 1
common fractional value.

F  2.000  in
0
13. Then, the parameters that depend on p d and F are: 4 5 6 7 8 9 10
pd  in
d  p d   3.375  in Kv p d   0.918
FIGURE 13-17A
Graph of Face Width and Limits for the Pinion in
Vt p d   1414 Wt p d   1033 lbf
ft
Problem 13-17
min

σbp p d F   15943  psi

The assumption made in step 5 is correct so no further iteration is required.


Gear
14. Determine the bending geometry factor, J (Table 13-4) Jg  0.51

Ka Wt p d   p d  Km
15. The bending stress equation for the gear is σbg p d F  
Kv p d   F  Jg

16. Determine the endurance strength of the pinion.

© 2011 Pearson Education, Inc., Upper Saddle River, NJ. All rights reserved. This publication is protected by Copyright and written permission should be
MACHINE DESIGN - An Integrated Approach, 4th Ed. 13-17-3

Material bending strength (psi)


A-7-d nodular iron S atg  34000  psi

S atg KL
Endurance strength S fbg  S fbg  34001  psi
KT  KR

17. Write the design equations using the range of face-width to diametral-pitch ratio given in the text, and the
bending stress equation, solved for the unknown face width.

FU  p d  
16
Upper limit
pd

FL  p d  
8
Lower limit
pd
S fb
Safety factor Nfb =
σb

Ka Wt p d   p d  Km Nfb


Face width F  p d  
Kv p d   Jg S fbg

1 1.5 10
18. Plot F(P) vs. p d over the range p d   
in in in

19. From the graph, choose a standard value of 6


p d from Table 12-2.

1  
F pd
p d  8  in 4.5
in
20. The calculated value of F is  
FL pd
F  p d   2.207  in in 3

21. Round this up to the decimal equivalent of a  


FU pd
common fractional value. in 1.5

F  2.250  in

22. Then, the parameters that depend on P and F are: 0


4 5 6 7 8 9 10
pd  in
d  p d   3.375  in Kv p d   0.918
FIGURE 13-17B
Graph of Face Width and Limits for the Gear in
Vt p d   1414 Wt p d   1033 lbf
ft
Problem 13-17
min

σbg p d F   13338  psi

23. The gear dimensions are larger (smaller diametral pitch means bigger teeth) than for the pinion. This means that
we will accept the gear requirements for the pinion, thus, for the set

1
Diametral pitch p d  8  in

Face width F  2.250  in


24. Determine the realized factor of safety for the gear using the above values for F and p d.
© 2011 Pearson Education, Inc., Upper Saddle River, NJ. All rights reserved. This publication is protected by Copyright and written permission should be
MACHINE DESIGN - An Integrated Approach, 4th Ed. 13-17-4

S fbg
Gear factor of safety Nfbg  Nfbg  2.5
σbg p d F 

25. Check the factor of safety on the pinion:

S fbp
Pinion factor of safety Nfbp  Nfbp  3.0
σbp p d F 

© 2011 Pearson Education, Inc., Upper Saddle River, NJ. All rights reserved. This publication is protected by Copyright and written permission should be
MACHINE DESIGN - An Integrated Approach, 4th Ed. 13-18-1

PROBLEM 13-18
Statement: Size the helical gears in problem 13-14 for a surface factor of safety of at least 1.6 assuming a
steady torque, 25 deg pressure angle, full depth teeth, a face width factor of 10, quality index of
9, an AISI 4140 steel pinion, and a class 40 cast iron gear.

Given: Factor of safety Nfc  1.6 Number of gear teeth Ng  57


Power to be transmitted H  125  hp Reliability R  0.99
Rotational speed of pinion n  1000 rpm AGMA Quality level Qv  9
7
Number of pinion teeth Np  23 Life (cycles) N  10
Pressure angle ϕ  25 deg Helix angle ψ  10 deg
Solution: See Mathcad file P1318.
Pinion
1. Determine the surface geometry factor, I.

a  p d  
1
Addendum
pd

Np Ng
Pitch radii rp p d   rg p d  
2 pd 2 pd

Center distance C p d   rp p d   rg p d 

Radii of curvature

ρp  p d   0.5  rp p d   C pd   rg pd     rp p d   cos( ϕ) 


2 2

ρg  p d   C p d   sin( ϕ)  ρp  p d 

Normal plane pressure angle ϕn  atan( tan( ϕ)  cos( ψ) ) ϕn  24.666 deg

 cos ϕn 
Base helix angle ψb  acos cos( ψ)   ψb  9.079  deg
 cos( ϕ) 
Length of action

Z  p d  
 r p  a p 2  r p  cos( ϕ) 2  
  p d   d    p d   
 r p  a p 2  r p  cos( ϕ) 2  C p  sin( ϕ) 
  g d   d    g d    d 

Z pd 
Transverse contact ratio mp p d  
π
 cos( ϕ)
pd

F  p d  tan( ψ)
Axial contact ratio mF  p d F  
π
Fractional parts of mp and mF n r p d   mp p d   floor  mp p d  
© 2011 Pearson Education, Inc., Upper Saddle River, NJ. All rights reserved. This publication is protected by Copyright and written permission should be
MACHINE DESIGN - An Integrated Approach, 4th Ed. 13-18-2

n a p d F   mF  p d F   floor  mF  p d F  

p t  p d   p n  p d   p t  p d   cos( ψ)
π
Axial pitch
pd

pn  pd 
p x  p d  
sin( ψ)

Minimum length of lines of contact

mp pd  F  napd F  nrpd   px pd  


Lmin p d F   return if n a p d F    1  n r p d  
cos ψb

mp pd   F   1  na p d F     1  nr pd    px  p d 


return if n a p d F    1  n r p d  
cos ψb

mN  p d F  
F
Load sharing ratio
Lmin p d F 

I  p d F  
cos( ϕ)
Geometry factor
 1  1   2 r  p   m  p F 
 ρ p  ρ p   p d N d
 p d g d 

2. Write the equations for pitch diameter, pitchline velocity, and transmitted load in terms of the unknown
diametral pitch, p d. Note that, in Mathcad, unit conversion factors are not included.

Np Ng
Pitch diameter of pinion (in) d p p d   d g p d  
pd pd

d p p d   n
pitchline velocity (fpm) Vt p d  
2

Wt p d  
H
Transmitted load (lbf)
Vt  p d 

3. Set the application factor, Ca Ca  1

B  0.25  12  Qv
0.6667
4. Write the equation for the dynamic load factor, C B  0.52
v

A  50  56 ( 1  B) A  76.878
B
Cv p d    A 
 
 A  Vt  p d  
min

 ft 

5. Tentatively choose the mounting factor, Cm (Assume 2 < F < 6 in Cm  1.7


0.5
6. Choose an elastic coefficient from Table 12-18 (steel on CI). Cp  2100 psi

C a W t  p d   C m
7. The surface stress equation for the pinion is σcp p d F   Cp
Cv p d   F  d p p d   I  p d F 
© 2011 Pearson Education, Inc., Upper Saddle River, NJ. All rights reserved. This publication is protected by Copyright and written permission should be
MACHINE DESIGN - An Integrated Approach, 4th Ed. 13-18-3

8. Determine the endurance strength of the pinion.


 0.056
Life factor CL  2.466  N CL  1
Reliability CR  0.7  0.15 log( 1  R) CR  1
Temperature factor CT  1
Material surface strength (psi)
S ac  165000 psi
AISI 4140 steel
S ac CL
Endurance strength S fcp  S fcp  164997 psi
CT  CR

9. Write the design equations using the range of face-width to diametral-pitch ratio given in the text, and the
bending stress equation, solved for the unknown face width.
2
 Sfc 
FU  p d   FL  p d  
16 8
Limits, safety factor Nfc =  
pd pd  σc 

C a W t  p d   C m
2
 Cp 
Face width F' p d F      Nfc
Cv p d   d p p d   I  p d F   S fcp 

Guess value for F F  2.347  in Found by iteration with value in step 12.

2 2.5 12
10. Plot F(P) vs. p d over the range p d   
in in in

11. From the graph, choose a standard value of 4


p d from Table 12-2.

1 
F' pd F 
p d  5  in 3
in
12. The calculated value of F is  
FL pd
F' p d F   2.347  in in 2

13. Round this up to the decimal equivalent of a FU pd  


common fractional value. in 1

F  2.375  in
14. Then, the parameters that depend on p d and F are: 0
2 3 4 5 6 7 8 9 10 11 12
d p p d   4.600  in d g p d   11.400 in pd  in
FIGURE 13-18A
Vt p d   1204 Wt p d   3425 lbf
ft Graph of Face Width and Limits for the Pinion
min (surface) in Problem 13-18

σcp p d F   129197 psi mp p d   1.491 mF  p d F   0.667 n r p d   0.491

n a p d F   0.667 mN  p d F   0.798 Lmin p d F   2.975  in I  p d F   0.171

© 2011 Pearson Education, Inc., Upper Saddle River, NJ. All rights reserved. This publication is protected by Copyright and written permission should be
MACHINE DESIGN - An Integrated Approach, 4th Ed. 13-18-4

Gear

C a W t  p d   C m
15. The surface stress equation for the gear is σcg p d F   Cp
Cv p d   F  d g p d   I  p d F 

16. Determine the endurance strength of the gear.


 0.056
Life factor CL  2.466  N CL  1
Reliability CR  0.7  0.15 log( 1  R) CR  1
Temperature factor CT  1
Material surface strength (psi)
S ac  80000  psi
Class 40 CI
S ac CL
Endurance strength S fcg  S fcg  79999  psi
CT  CR

17. Write the design equations using the range of face-width to diametral-pitch ratio given in the text, and the
bending stress equation, solved for the unknown face width.
2
 Sfc 
FU  p d   FL  p d  
16 8
Limits, safety factor Nfc = σ 
pd pd  c

C a W t  p d   C m
2
 Cp 
Face width F' p d F      Nfc
Cv p d   d g p d   I  p d F   S fcg 

Guess value for F F  2.745  in Found by iteration with value in step 20.

2 2.5 12
18. Plot F(P) vs. p d over the range p d   
in in in
8
19. From the graph, choose a standard value of
p d from Table 12-2. 
F' pd F 
6
1 in
p d  4  in
 
FL pd
20. The calculated value of F is 4
in
F' p d F   2.746  in
FU pd  
21. Round this up to the decimal equivalent of a in 2
common fractional value.

F  2.750  in
0
22. Then, the parameters that depend on p d and F are: 2 2.5 3 3.5 4 4.5 5 5.5 6 6.5 7
pd  in
d p p d   5.750  in d g p d   14.250 in
FIGURE 13-18B
Graph of Face Width and Limits for the Gear
Vt p d   1505 Wt p d   2740 lbf
ft
(surface) in Problem 13-18
min

σcp p d F   99427  psi mp p d   1.491 mF  p d F   0.617 n r p d   0.491

© 2011 Pearson Education, Inc., Upper Saddle River, NJ. All rights reserved. This publication is protected by Copyright and written permission should be
MACHINE DESIGN - An Integrated Approach, 4th Ed. 13-18-5

n a p d F   0.617 mN  p d F   0.84 Lmin p d F   3.276  in I  p d F   0.163

23. The gear teeth are larger than for the pinion. This means that we will accept the gear requirements for the
pinion thus, for the set

1
Diametral pitch p d  4  in

Face width F  2.750  in

24. Determine the realized factor of safety for the gear using the above values for F and p d.

2
 Sfcg 
Gear factor of safety Nfsg    Nfsg  1.6
 σcg pd F  
25. Check the factor of safety on the pinion:
2
 Sfcp 
Pinion factor of safety Nfsp   σ p F  Nfsp  2.8
 cp d  

© 2011 Pearson Education, Inc., Upper Saddle River, NJ. All rights reserved. This publication is protected by Copyright and written permission should be
MACHINE DESIGN - An Integrated Approach, 4th Ed. 13-19-1

PROBLEM 13-19
Statement: Size the helical gears in problem 13-15 for a surface factor of safety of at least 1.2 assuming a
steady torque, 20 deg pressure angle, full depth teeth, a face width factor of 12, quality index of
11, an AISI 4340 steel pinion, and an A-7-d nodular iron gear.

Given: Factor of safety Nfc  1.2 Number of gear teeth Ng  78


Power to be transmitted H  33 kW Reliability R  0.99
Rotational speed of pinion n  1600 rpm AGMA Quality level Qv  11
7
Number of pinion teeth Np  27 Life (cycles) N  10
Pressure angle ϕ  20 deg Helix angle ψ  30 deg
Solution: See Mathcad file P1319.
Pinion
1. Determine the surface geometry factor, I.

a  p d  
1
Addendum
pd

Np Ng
Pitch radii rp p d   rg p d  
2 pd 2 pd

Center distance C p d   rp p d   rg p d 

Radii of curvature

ρp  p d   0.5  rp p d   C pd   rg pd     rp p d   cos( ϕ) 


2 2

ρg  p d   C p d   sin( ϕ)  ρp  p d 

Normal plane pressure angle ϕn  atan( tan( ϕ)  cos( ψ) ) ϕn  17.495 deg

 cos ϕn 
Base helix angle ψb  acos cos( ψ)   ψb  28.481 deg
 cos( ϕ) 
Length of action

Z  p d  
 r p  a p 2  r p  cos( ϕ) 2  
  p d   d    p d   
 r p  a p 2  r p  cos( ϕ) 2  C p  sin( ϕ) 
  g d   d    g d    d 

Z pd 
Transverse contact ratio mp p d  
π
 cos( ϕ)
pd

F  p d  tan( ψ)
Axial contact ratio mF  p d F  
π
Fractional parts of mp and mF n r p d   mp p d   floor  mp p d  
© 2011 Pearson Education, Inc., Upper Saddle River, NJ. All rights reserved. This publication is protected by Copyright and written permission should be
MACHINE DESIGN - An Integrated Approach, 4th Ed. 13-19-2

n a p d F   mF  p d F   floor  mF  p d F  

p t  p d   p n  p d   p t  p d   cos( ψ)
π
Axial pitch
pd

pn  pd 
p x  p d  
sin( ψ)

Minimum length of lines of contact

mp pd  F  napd F  nrpd   px pd  


Lmin p d F   return if n a p d F    1  n r p d  
cos ψb

mp pd   F   1  na p d F     1  nr pd    px  p d 


return if n a p d F    1  n r p d  
cos ψb

mN  p d F  
F
Load sharing ratio
Lmin p d F 

I  p d F  
cos( ϕ)
Geometry factor
 
 ρ  p   ρ  p    2 rp pd   mN  p d F 
1 1
 p d g d 

2. Write the equations for pitch diameter, pitchline velocity, and transmitted load in terms of the unknown
diametral pitch, p d. Note that, in Mathcad, unit conversion factors are not included.

Np Ng
Pitch diameter of pinion (in) d p p d   d g p d  
pd pd

d p p d   n
pitchline velocity (fpm) Vt p d  
2

Wt p d  
H
Transmitted load (lbf)
Vt  p d 

3. Set the application factor, Ca Ca  1

B  0.25  12  Qv
0.6667
4. Write the equation for the dynamic load factor, C B  0.25
v

A  50  56 ( 1  B) A  92
B
Cv p d    A 
 
 A  Vt  p d  
min

 ft 

5. Tentatively choose the mounting factor, Cm (Assume 0 < F < 2 in Cm  1.6


0.5
6. Choose an elastic coefficient from Table 11-18 (steel on NI). Cp  2160 psi

C a W t  p d   C m
7. The surface stress equation for the pinion is σcp p d F   Cp
Cv p d   F  d p p d   I  p d F 
© 2011 Pearson Education, Inc., Upper Saddle River, NJ. All rights reserved. This publication is protected by Copyright and written permission should be
MACHINE DESIGN - An Integrated Approach, 4th Ed. 13-19-3

8. Determine the endurance strength of the pinion.


 0.056
Life factor CL  2.466  N CL  1
Reliability CR  0.7  0.15 log( 1  R) CR  1
Temperature factor CT  1
Material surface strength (psi)
S ac  160000 psi
AISI 4340 steel
S ac CL
Endurance strength S fcp  S fcp  159997 psi
CT  CR

9. Write the design equations using the range of face-width to diametral-pitch ratio given in the text, and the
bending stress equation, solved for the unknown face width.
2
 Sfc 
FU  p d   FL  p d  
16 8
Limits, safety factor Nfc =  
pd pd  σc 

C a W t  p d   C m
2
 Cp 
Face width F' p d F      Nfc
Cv p d   d p p d   I  p d F   S fcp 

Guess value for F F  1.140  in Found by iteration with value in step 12.

6 6.1 16
10. Plot F(P) vs. p d over the range p d   
in in in

11. From the graph, choose a standard value of 2


p d from Table 12-2.

1 
F' pd F 
p d  12 in 1.5
in
12. The calculated value of F is  
FL pd
F' p d F   1.140  in in 1

13. Round this up to the decimal equivalent of a FU pd  


common fractional value. in 0.5

F  1.250  in
14. Then, the parameters that depend on p d and F are: 0
6 7 8 9 10 11 12 13 14 15 16
d p p d   2.250  in d g p d   6.500  in pd  in
FIGURE 13-19A
Vt p d   942  Wt p d   1550 lbf
ft Graph of Face Width and Limits for the Pinion
min (surface) in Problem 13-19

σcp p d F   138282 psi mp p d   1.726 mF  p d F   2.757 n r p d   0.726

n a p d F   0.757 mN  p d F   0.516 Lmin p d F   2.42 in I  p d F   0.231

© 2011 Pearson Education, Inc., Upper Saddle River, NJ. All rights reserved. This publication is protected by Copyright and written permission should be
MACHINE DESIGN - An Integrated Approach, 4th Ed. 13-19-4

Gear

C a W t  p d   C m
15. The surface stress equation for the gear is σcg p d F   Cp
Cv p d   F  d g p d   I  p d F 

16. Determine the endurance strength of the gear.


 0.056
Life factor CL  2.466  N CL  1
Reliability CR  0.7  0.15 log( 1  R) CR  1
Temperature factor CT  1
Material surface strength (psi)
S ac  100000 psi
A-7-d nodular iron
S ac CL
Endurance strength S fcg  S fcg  99998  psi
CT  CR

17. Write the design equations using the range of face-width to diametral-pitch ratio given in the text, and the
bending stress equation, solved for the unknown face width.
2
 Sfc 
FU  p d   FL  p d  
16 8
Limits, safety factor Nfc = σ 
pd pd  c

C a W t  p d   C m
2
 Cp 
Face width F' p d F      Nfc
Cv p d   d g p d   I  p d F   S fcg 

Guess value for F F  1.028  in Found by iteration with value in step 20.

6 6.1 16
18. Plot F(P) vs. p d over the range p d   
in in in
2
19. From the graph, choose a standard value of
p d from Table 12-2. 
F' pd F 
1.5
1 in
p d  12 in
 
FL pd
20. The calculated value of F is 1
in
F' p d F   1.030  in
FU pd  
21. Round this up to the decimal equivalent of a in 0.5
common fractional value.

F  1.125  in
0
22. Then, the parameters that depend on p d and F are: 6 7 8 9 10 11 12 13 14 15 16
pd  in
d p p d   2.250  in d g p d   6.500  in
FIGURE 13-19B
Graph of Face Width and Limits for the Gear
Vt p d   942  Wt p d   1550 lbf
ft
(surface) in Problem 13-19
min

σcp p d F   147201 psi mp p d   1.726 mF  p d F   2.481 n r p d   0.726

© 2011 Pearson Education, Inc., Upper Saddle River, NJ. All rights reserved. This publication is protected by Copyright and written permission should be
MACHINE DESIGN - An Integrated Approach, 4th Ed. 13-19-5

n a p d F   0.481 mN  p d F   0.527 Lmin p d F   2.136  in I  p d F   0.227

23. The gear teeth are the same size as the pinion. This means that we will accept the gear requirements for the
pinion thus, for the set

1
Diametral pitch p d  12 in

Face width F  1.125  in

24. Determine the realized factor of safety for the gear using the above values for F and p d.

2
 Sfcg 
Gear factor of safety Nfsg    Nfsg  1.3
 σcg pd F  
25. Check the factor of safety on the pinion:
2
 Sfcp 
Pinion factor of safety Nfsp   σ p F  Nfsp  1.2
 cp d  

© 2011 Pearson Education, Inc., Upper Saddle River, NJ. All rights reserved. This publication is protected by Copyright and written permission should be
MACHINE DESIGN - An Integrated Approach, 4th Ed. 13-20-1

PROBLEM 13-20
Statement: Size the bevel gears in problem 13-5 for a bending factor of safety of at least 2 assuming a 5-year,
1-shift life, steady torque, quality index of 9, an AISI 4140 steel pinion and gear.
Given: Factor of safety Nfb  2 Number of gear teeth Ng  126
Power to be transmitted (hp) H  746  W Reliability R  0.99
Rotational speed of pinion (rpm) n p  1000 rpm AGMA Quality level Qv  9
Number of pinion teeth Np  14 Life (years) Life  5  yr

Solution: See Mathcad file P1320.


Pinion
1. Determine the bending geometry factor, J (Figure 13-5) Jp  0.285
2. Write the equations for pitch diameter, pitchline velocity, and transmitted torque in terms of the unknown
diametral pitch, p d. Note that, in Mathcad, unit conversion factors are not included.

Np Ng
Pitch diameter of pinion (in) d p p d   d g p d  
pd pd
d p p d   n p
pitchline velocity (fpm) Vt p d  
2

H
Transmitted torque (in-lbf) Tp  Tp  63.051 in lbf
np

3. Set the application, size, and type factors. Ka  1 Ks  1 Kx  1

B  0.25  12  Qv
0.6667
4. Write the equation for the dynamic load factor, Kv B  0.52

A  50  56 ( 1  B) A  76.878
B
Kv p d    A 
 
 A  Vt  p d  
min

 ft 

5. Tentatively choose the mounting factor, Km (Assume 0 < F < 2 in) Km  1.6

2  Tp pd Ka Km Ks
6. The bending stress equation for the pinion is σbp p d F    
d p p d  F  Jp Kv p d   Kx

7. Determine the endurance strength of the pinion.


np 9
Cycle life N  Life N  2.63  10
2 π
 0.0323
Life factor KL  1.6831 N KL  0.835
Reliability KR  0.7  0.15 log( 1  R) KR  1
Temperature factor KT  1
Material bending strength (psi)
S atp  40000  psi
AISI 4140 Nitrided steel
S atp KL
Endurance strength S fbp  S fbp  33412  psi
KT  KR
© 2011 Pearson Education, Inc., Upper Saddle River, NJ. All rights reserved. This publication is protected by Copyright and written permission should be
MACHINE DESIGN - An Integrated Approach, 4th Ed. 13-20-2

8. Write the design equations using the face-width to pitch-cone length ratio given in the text, and the bending
stress equation, solved for the unknown face width.

Np
Upper limit, F = L/3 FLover3 p d  
  Np  
6  p d  sin atan 
  Ng  
S fb
Safety factor Nfb =
σb

2  Tp p d  Ka Km Ks Nfb


Face width F  p d   
d p p d   Jp Kv p d   Kx S fbp

16 16.5 26
9. Plot F(P) vs. p d over the p d   
range in in in
2

10. From the graph, choose a standard value of


p d from Table 12-2.
  1.5
F pd
1
p d  18 in in

  1
11. The calculated value of F is FLover3 pd

F  p d   1.071  in in
0.5
12. Round this to the decimal equivalent of a
common fractional value.
0
F  1.125  in 16 18 20 22 24 26
pd  in
13. Then, the parameters that depend on p d and F are:
FIGURE 13-20A
d p p d   0.778  in Kv p d   0.915
Graph of Face Width and Limits for the Pinion in
Problem 13-20

Vt p d   204 
ft
min

σbp p d F   15912  psi

The assumption made in step 5 is correct so no further iteration is required.


GEAR
14. Determine the bending geometry factor, J (Figure 13-5) Jg  0.220

2  Tp pd Ka Km Ks
15. The bending stress equation for the gear is σbg p d F    
d g p d  F  Jg Kv p d   Kx

16. Write the design equations using the face-width to pitch-cone length ratio given in the text, and the bending
stress equation, solved for the unknown face width.

Np
F = L/3 FLover3 p d  
  Np  
6  p d  sin atan 
  Ng  
© 2011 Pearson Education, Inc., Upper Saddle River, NJ. All rights reserved. This publication is protected by Copyright and written permission should be
MACHINE DESIGN - An Integrated Approach, 4th Ed. 13-20-3

S fb
Safety factor Nfb =
σb

2  Tp p d  Ka Km Ks Nfb


Face width F  p d   
d g p d   Jg Kv p d   Kx S fbp

30 30.5 40
17. Plot F(P) vs. p d over the range p d   
in in in
2

18. From the graph, choose a standard value of


p d from Table 12-2.
  1.5
F pd
1
p d  36 in in

  1
19. The calculated value of F is FLover3 pd

F  p d   0.602  in in
0.5
20. Round this to the decimal equivalent of a
common fractional value.
0
F  0.625  in 30 32 34 36 38 40
pd  in
21. Then, the parameters that depend on p d and F are:
FIGURE 13-20B
d g p d   3.500  in Kv p d   0.938
Graph of Face Width and Limits for the Gear in
Problem 13-20

Vt p d   102  σbg p d F   16093  psi


ft
min
22. The gear dimensions are smaller (larger diametral pitch means smaller teeth) than for the pinion. This means
that we will accept the pinion requirements for the gear, thus, for the set
1
Diametral pitch p d  18 in
Face width F  1.125  in
 Np 
Pitch cone angle αp  atan  αp  6.34 deg
 Ng 
d p p d  L
Pitch-cone length L  L  3.522  in  1.174  in
2  sin αp 3

23. Determine the realized factor of safety for the pinion using the above values for F and p d.
S fbp
Pinion factor of safety Nfbp  Nfbp  2.1
σbp p d F 

25. Check the factor of safety on the gear:


S fbp
Gear factor of safety Nfbg  Nfbg  14.6
σbg p d F 

© 2011 Pearson Education, Inc., Upper Saddle River, NJ. All rights reserved. This publication is protected by Copyright and written permission should be
MACHINE DESIGN - An Integrated Approach, 4th Ed. 13-21-1

PROBLEM 13-21
Statement: Size the bevel gears in problem 13-6 for a bending factor of safety of at least 2.5 assuming a
15-year, 3-shift life, steady torque, quality index of 11, an AISI 4340 steel pinion and gear.
Units: yr  2080 hr
Given: Factor of safety Nfb  2.5 Number of gear teeth Ng  81
Power to be transmitted (hp) H  7460 W Reliability R  0.99
Rotational speed of pinion (rpm) n p  800  rpm AGMA Quality level Qv  11
Number of pinion teeth Np  18 Life (years) Life  5  yr

Solution: See Mathcad file P1321.


Pinion
1. Determine the bending geometry factor, J (Figure 13-5) Jp  0.267
2. Write the equations for pitch diameter, pitchline velocity, and transmitted torque in terms of the unknown
diametral pitch, p d. Note that, in Mathcad, unit conversion factors are not included.

Np Ng
Pitch diameter of pinion (in) d p p d   d g p d  
pd pd

d p p d   n p
pitchline velocity (fpm) Vt p d  
2

H
Transmitted torque (in-lbf) Tp  Tp  788.134  in lbf
np

3. Set the application, size, and type factors. Ka  1 Ks  1 Kx  1

B  0.25  12  Qv
0.6667
4. Write the equation for the dynamic load factor, Kv B  0.25

A  50  56 ( 1  B) A  92
B
Kv p d    A 
 
 A  Vt  p d  
min

 ft 

5. Tentatively choose the mounting factor, Km (Assume 0 < F < 2 in) Km  1.6

2  Tp pd Ka Km Ks
6. The bending stress equation for the pinion is σbp p d F    
d p p d  F  Jp Kv p d   Kx

7. Determine the endurance strength of the pinion.


np 9
Cycle life shifts  3 N  Life shifts N  1.498  10
2 π
 0.0323
Life factor KL  1.6831 N KL  0.851
Reliability KR  0.7  0.15 log( 1  R) KR  1
Temperature factor KT  1
Material bending strength (psi)
S atp  42000  psi
AISI 4340 Nitrided steel
© 2011 Pearson Education, Inc., Upper Saddle River, NJ. All rights reserved. This publication is protected by Copyright and written permission should be
MACHINE DESIGN - An Integrated Approach, 4th Ed. 13-21-2

S atp KL
Endurance strength S fbp  S fbp  35727  psi
KT  KR

8. Write the design equations using the face-width to pitch-cone length ratio given in the text, and the bending
stress equation, solved for the unknown face width.

Np
Upper limit, F = L/3 FLover3 p d  
  Np  
6  p d  sin atan 
  Ng  
S fb
Safety factor Nfb =
σb

2  Tp p d  Ka Km Ks Nfb


Face width F  p d   
d p p d   Jp Kv p d   Kx S fbp

2 2.5 12
9. Plot F(P) vs. p d over the p d   
range in in in

10. From the graph, choose a standard value of 4


p d from Table 12-2.

1
 
p d  6  in 3
F pd

11. The calculated value of F is in

F  p d   1.404  in FLover3 pd  2
in
12. Round this to the decimal equivalent of a 1
common fractional value.

F  1.500  in
0
2 4 6 8 10 12
13. Then, the parameters that depend on p d and F are:
pd  in

d p p d   3.000  in Kv p d   0.942 FIGURE 13-21A


Graph of Face Width and Limits for the Pinion in
Vt p d   628 
ft
Problem 13-21
min

σbp p d F   13376  psi

The assumption made in step 5 is correct so no further iteration is required.


GEAR
14. Determine the bending geometry factor, J (Figure 13-5) Jg  0.221

2  Tp pd Ka Km Ks
15. The bending stress equation for the gear is σbg p d F    
d g p d  F  Jg Kv p d   Kx

16. Write the design equations using the face-width to pitch-cone length ratio given in the text, and the bending
stress equation, solved for the unknown face width.

© 2011 Pearson Education, Inc., Upper Saddle River, NJ. All rights reserved. This publication is protected by Copyright and written permission should be
MACHINE DESIGN - An Integrated Approach, 4th Ed. 13-21-3

Np
F = L/3 FLover3 p d  
  Np  
6  p d  sin atan 
  Ng  
S fb
Safety factor Nfb =
σb

2  Tp p d  Ka Km Ks Nfb


Face width F  p d   
d g p d   Jg Kv p d   Kx S fbp

6 6.5 16
17. Plot F(P) vs. p d over the range p d   
in in in

18. From the graph, choose a standard value of 2


p d from Table 12-2.

1
 
p d  10 in 1.5
F pd

19. The calculated value of F is in

  1
F  p d   1.034  in FLover3 pd
in
20. Round this to the decimal equivalent of a 0.5
common fractional value.

F  1.125  in
0
6 8 10 12 14 16
21. Then, the parameters that depend on p d and F are:
pd  in

d g p d   8.100  in Kv p d   0.953 FIGURE 13-21B


Graph of Face Width and Limits for the Gear in

Vt p d   377  σbg p d F   13137  psi


ft Problem 13-21
min
22. The gear dimensions are smaller (larger diametral pitch means smaller teeth) than for the pinion. This means th
we will accept the pinion requirements for the gear, thus, for the set
1
Diametral pitch p d  6  in
Face width F  1.500  in
 Np 
Pitch cone angle αp  atan  αp  12.529 deg
 Ng 
d p p d  L
Pitch-cone length L  L  6.915  in  2.305  in
2  sin αp 3

23. Determine the realized factor of safety for the pinion using the above values for F and p d.
S fbp
Pinion factor of safety Nfbp  Nfbp  2.7
σbp p d F 

25. Check the factor of safety on the gear:


S fbp
Gear factor of safety Nfbg  Nfbg  9.9
σbg p d F 
© 2011 Pearson Education, Inc., Upper Saddle River, NJ. All rights reserved. This publication is protected by Copyright and written permission should be
MACHINE DESIGN - An Integrated Approach, 4th Ed. 13-22-1

PROBLEM 13-22
Statement: Size the spiral gears in problem 13-7 for a bending factor of safety of at least 2.2 assuming a
10-year, 3-shift life, steady torque, quality index of 8, an AISI 4340 steel pinion and gear.
Units: yr  2080 hr
Given: Factor of safety Nfb  2.2 Number of gear teeth Ng  80
Power to be transmitted H  3  hp Reliability R  0.99
Rotational speed of pinion n p  600  rpm AGMA Quality level Qv  8
Number of pinion teeth Np  16 Life (years) Life  10 yr

Solution: See Mathcad file P1322.


Pinion
1. Determine the bending geometry factor, J (Figure 13-8) Jp  0.305

2. Write the equations for pitch diameter, pitchline velocity, and transmitted torque in terms of the unknown
diametral pitch, p d. Note that, in Mathcad, unit conversion factors are not included.
Np Ng
Pitch diameter of pinion (in) d p p d   d g p d  
pd pd

d p p d   n p
pitchline velocity (fpm) Vt p d  
2

H
Transmitted torque (in-lbf) Tp  Tp  315.127  in lbf
np

3. Set the application, size, and type factors. Ka  1 Ks  1 Kx  1.15

B  0.25  12  Qv
0.6667
4. Write the equation for the dynamic load factor, K B  0.63
v

A  50  56 ( 1  B) A  70.721
B
Kv p d    A 
 
 A  Vt  p d  
min

 ft 

5. Tentatively choose the mounting factor, Km (Assume 0 < F < 2 in) Km  1.6

2  Tp pd Ka Km Ks
6. The bending stress equation for the pinion is σbp p d F    
d p p d  F  Jp Kv p d   Kx
7. Determine the endurance strength of the pinion.
np 9
Cycle life shifts  3 N  Life shifts N  2.246  10
2 π
 0.0323
Life factor KL  1.6831 N KL  0.84
Reliability KR  0.7  0.15 log( 1  R) KR  1
Temperature factor KT  1
Material bending strength (psi)
S atp  42000  psi
AISI 4340 Nitrided steel

© 2011 Pearson Education, Inc., Upper Saddle River, NJ. All rights reserved. This publication is protected by Copyright and written permission should be
MACHINE DESIGN - An Integrated Approach, 4th Ed. 13-22-2

S atp KL
Endurance strength S fbp  S fbp  35262  psi
KT  KR

8. Write the design equations using the face-width to pitch-cone length ratio given in the text, and the bending
stress equation, solved for the unknown face width.

Np
Upper limit, F = L/3 FLover3 p d  
  Np  
6  p d  sin atan 
  Ng  
S fb
Safety factor Nfb =
σb

2  Tp p d  Ka Km Ks Nfb


Face width F  p d   
d p p d   Jp Kv p d   Kx S fbp

2 2.5 12
9. Plot F(P) vs. p d over the p d   
range in in in

10. From the graph, choose a standard value of 4


p d from Table 12-2.

1
p d  10 in
  3
F pd
11. The calculated value of F is in
F  p d   1.273  in FLover3 pd  2
12. Round this to the decimal equivalent of a in
common fractional value. 1

F  1.375  in
0
13. Then, the parameters that depend on p d and F are: 2 4 6 8 10 12
pd  in
d p p d   1.600  in Kv p d   0.880
FIGURE 13-21A
Graph of Face Width and Limits for the Pinion in
Vt p d   251 
ft
Problem 13-21
min

σbp p d F   14844  psi

The assumption made in step 5 is correct so no further iteration is required.

Gear
14. Determine the bending geometry factor, J (Figure 13-8) Jg  0.302

2  Tp pd Ka Km Ks
15. The bending stress equation for the gear is σbg p d F    
d g p d  F  Jg Kv p d   Kx

16. Write the design equations using the face-width to pitch-cone length ratio given in the text, and the bending
stress equation, solved for the unknown face width.

© 2011 Pearson Education, Inc., Upper Saddle River, NJ. All rights reserved. This publication is protected by Copyright and written permission should be
MACHINE DESIGN - An Integrated Approach, 4th Ed. 13-22-3

Np
F = L/3 FLover3 p d  
  Np  
6  p d  sin atan 
  Ng  
S fb
Safety factor Nfb =
σb

2  Tp p d  Ka Km Ks Nfb


Face width F  p d   
d g p d   Jg Kv p d   Kx S fbp

10 11.5 20
17. Plot F(P) vs. p d over the range p d   
in in in

18. From the graph, choose a standard value of 2


p d from Table 12-2.

1
 
p d  16 in 1.5
F pd

19. The calculated value of F is in

  1
F  p d   0.642  in FLover3 pd
in
20. Round this to the decimal equivalent of a 0.5
common fractional value.

F  0.750  in
0
10 12 14 16 18 20
21. Then, the parameters that depend on p d and F are:
pd  in

d g p d   5.000  in Kv p d   0.902


FIGURE 13-21B
Graph of Face Width and Limits for the Gear in
Vt p d   157  σbg p d F   13730  psi
ft
Problem 13-21
min

22. The gear dimensions are smaller (larger diametral pitch means smaller teeth) than for the pinion. This means tha
we will accept the pinion requirements for the gear, thus, for the set
1
Diametral pitch p d  10 in
Face width F  1.375  in
 Np 
Pitch cone angle αp  atan  αp  11.31  deg
 Ng 
d p p d  L
Pitch-cone length L  L  4.079  in  1.360  in
2  sin αp 3

23. Determine the realized factor of safety for the pinion using the above values for F and p d.
S fbp
Pinion factor of safety Nfbp  Nfbp  2.4
σbp p d F 

25. Check the factor of safety on the gear:


S fbp
Gear factor of safety Nfbg  Nfbg  11.8
σbg p d F 
© 2011 Pearson Education, Inc., Upper Saddle River, NJ. All rights reserved. This publication is protected by Copyright and written permission should be
MACHINE DESIGN - An Integrated Approach, 4th Ed. 13-23-1

PROBLEM 13-23
Statement: Size the bevel gears in problem 13-5 for a minimum safety factor of 1.4 for any mode of failure of
pinion or gear assuming a 5-year, 1-shift life, steady torque, quality index of 9, an AISI 4140 steel
pinion and gear.

Units: yr  2080 hr

Given: Factor of safety Nfc  1.4 Number of gear teeth Ng  126


Power to be transmitted (hp) H  746  W Reliability R  0.99
Rotational speed of pinion (rpm) n p  1000 rpm AGMA Quality level Qv  9
Number of pinion teeth Np  14 Life (years) Life  5  yr

Assumptions: 1. If both pinion and gear are the same material, it will only be necessary to determine the pinion
size as it will be governing for the set.
2. If the gears are not surface hardened, it will only be necessary to design to the surface
requirement as it will be governing for both bending and surface stresses.

Solution: See Mathcad file P1323.


1. Determine the surface geometry factor, I, from Figure 13-6. I  0.092
2. Write the equations for pitch diameter, pitchline velocity, and transmitted load in terms of the unknown
diametral pitch, p d. Note that, in Mathcad, unit conversion factors are not included.

Np Ng
Pitch diameter of pinion (in) d p p d   d g p d  
pd pd

Np
pitchline velocity (fpm) Vtp p d    np
2 pd

H
Transmitted load (lbf) Tp  Tp  63.051 in lbf
np

3. Set the application factor, Ca Ca  1 Cs  1 Cf  1

B  0.25  12  Qv
0.6667
4. Write the equation for the dynamic load factor, C B  0.52
v

A  50  56 ( 1  B) A  76.878
B
Cvp p d    A 
 
 A  Vtp pd  
min

 ft 

5. Tentatively choose the mounting factor, Cmd Cmd  3.0 Cxc  1

6. Choose an elastic coefficient from Table 11-18 0.5


(steel on steel) and a stress adjustment constant. Cp  2100 psi Cb  0.634

7. Determine the endurance strength of the pinion.


np 8
Cycle life shifts  1 N  Life shifts N  6.24  10
2 π
 0.056
Life factor CL  2.466  N CL  0.793
Reliability CR  0.7  0.15 log( 1  R) CR  1

© 2011 Pearson Education, Inc., Upper Saddle River, NJ. All rights reserved. This publication is protected by Copyright and written permission should be
MACHINE DESIGN - An Integrated Approach, 4th Ed. 13-23-2

Hardness CH  1
Temperature factor CT  1
Material surface strength (psi)
S acp  167500 psi
AISI 4140 Nitrided steel
S acp CL
Endurance strength S fcp  S fcp  132885 psi
CT  CR
8. Write the equation for the design pinion torque.

I  Cvp p d   Sacp d p pd  0.774  CH 


2
TD p d F  
F
   
2 Cs Cmd  Cf  Ca Cxc  Cp Cb CT  CR 

9. Define the exponent z. z p d F   return 0.667 if Tp  TD p d F 


1 otherwise

10. The surface stress equation for the pinion is


z pd F 
 2 TD pd F    Tp  Ca Cmd
σcp p d F   Cp Cb      Cs Cf  Cxc
 F  d p pd  2 I   TD p d F   Cvp p d 
 
11. Write the design equations using the face-width to pitch-cone length ratio given in the text, and the bending
stress equation, solved for the unknown face width.
2
Np  Sfc 
Upper limit, F = L/3 FLover3 p d   Nfc =  
  Np    σc 
6  p d  sin atan 
Face width
  Ng  


z pd F 
 2  TD p d F    Tp 
2
 Cp Cb  Ca Cmd
F' p d F          Cs Cf  Cxc Nfc
 S fcp   d p pd  2 I   TD pd F   Cvp p d 
 
12 12.5 22
12. Plot F(P) vs. p d over the range p d    Guess value for F F  0.955  in
in in in
2
13. From the graph, choose a standard value of p d from
Table 12-2.

 
1 1.5
p d  16 in F' pd F
in
14. The calculated value of F is
  1
FLover3 pd
F' p d F   0.956  in
in
15. Round this to the decimal equivalent of a common 0.5
fractional value.

F  1.000  in 0
12 14 16 18 20 22
16. Then, the parameters that depend on p d and F are:
pd  in

d p p d   0.875  in d g p d   7.875  in FIGURE 13-23


Graph of Face Width and Limits for for Problem 13-23
© 2011 Pearson Education, Inc., Upper Saddle River, NJ. All rights reserved. This publication is protected by Copyright and written permission should be
MACHINE DESIGN - An Integrated Approach, 4th Ed. 13-23-3

Vtp p d   229  Cvp p d   0.911 σcp p d F   1  10  psi TD p d F   101  in lbf
ft 5
min

17. When both pinion and gear are of the same material, the stress in the gear will always be lower than in the pinion
Therefore, the face width and diametral pitch found for the pinion should also be used for the gear.

1
Diametral pitch p d  16 in

Face width F  1.000  in

 Np 
Pitch cone angle αp  atan  αp  6.34 deg
 Ng 
d p p d  L
Pitch-cone length L  L  3.962  in  1.321  in
2  sin αp 3

18. Determine the realized factor of safety for the pinion using the above values for F and p d.

2
 Sfcp 
Pinion factor of safety Nfcp    Nfcp  1.4
 σcp pd F  

© 2011 Pearson Education, Inc., Upper Saddle River, NJ. All rights reserved. This publication is protected by Copyright and written permission should be
MACHINE DESIGN - An Integrated Approach, 4th Ed. 13-24-1

PROBLEM 13-24
Statement: Size the bevel gears in problem 13-6 for a surface factor of safety of at least 1.3 assuming a
15-year, 3-shift life, steady torque, quality index of 11 an AISI 4340 steel pinion and gear.
Units: yr  2080 hr
Given: Factor of safety Nfc  1.3 Number of gear teeth Ng  81
Power to be transmitted (hp) H  7460 W Reliability R  0.99
Rotational speed of pinion (rpm) n p  800  rpm AGMA Quality level Qv  11
Number of pinion teeth Np  18 Life (years) Life  15 yr

Solution: See Mathcad file P1324.

1. Determine the surface geometry factor, I, from Figure 13-6. I  0.091

2. Write the equations for pitch diameter, pitchline velocity, and transmitted load in terms of the unknown
diametral pitch, p d. Note that, in Mathcad, unit conversion factors are not included.
Np Ng
Pitch diameter of pinion (in) d p p d   d g p d  
pd pd

Np
pitchline velocity (fpm) Vtp p d    np
2 pd

H
Transmitted load (lbf) Tp  Tp  788.134  in lbf
np

3. Set the application factor, Ca Ca  1 Cs  1 Cf  1

B  0.25  12  Qv
0.6667
4. Write the equation for the dynamic load factor, C B  0.25
v

A  50  56 ( 1  B) A  92
B
Cvp p d    A 
 
 A  Vtp pd  
min

 ft 

5. Tentatively choose the mounting factor, Cmd Cmd  3.0 Cxc  1

6. Choose an elastic coefficient from Table 12-18 0.5


(steel on steel) and a stress adjustment constant. Cp  2100 psi Cb  0.634

7. Determine the endurance strength of the pinion.


np 9
Cycle life shifts  3 N  Life shifts N  4.493  10
2 π
 0.056
Life factor CL  2.466  N CL  0.71
Reliability CR  0.7  0.15 log( 1  R) CR  1

Hardness CH  1
Temperature factor CT  1
Material surface strength (psi)
S acp  162000 psi
AISI 4340 Nitrided steel
© 2011 Pearson Education, Inc., Upper Saddle River, NJ. All rights reserved. This publication is protected by Copyright and written permission should be
MACHINE DESIGN - An Integrated Approach, 4th Ed. 13-24-2

S acp CL
Endurance strength S fcp  S fcp  115071 psi
CT  CR
8. Write the equation for the design pinion torque.

I  Cvp p d   Sacp d p pd  0.774  CH 


2
TD p d F  
F
   
2 Cs Cmd  Cf  Ca Cxc  Cp Cb CT  CR 

9. Define the exponent z. z p d F   return 0.667 if Tp  TD p d F 


1 otherwise

10. The surface stress equation for the pinion is


z pd F 
 2 TD pd F    Tp  Ca Cmd
σcp p d F   Cp Cb      Cs Cf  Cxc
 F  d p pd  2 I   TD p d F   Cvp p d 
 
11. Write the design equations using the face-width to pitch-cone length ratio given in the text, and the bending
stress equation, solved for the unknown face width.
2
Np  Sfc 
Upper limit, F = L/3 FLover3 p d   Nfc = σ 
  Np    c
6  p d  sin atan 
Face width
  Ng  


z pd F 
 2  TD p d F    Tp 
2
 Cp Cb  Ca Cmd
F' p d F          Cs Cf  Cxc Nfc
 S fcp   d p pd  2 I   TD pd F   Cvp p d 
 
2 2.5 12
12. Plot F(P) vs. p d over the range p d    Guess value for F F  1.325  in
in in in
4
13. From the graph, choose a standard value of p d from
Table 12-2.

 
1 3
p d  6  in F' pd F
in
14. The calculated value of F is
FLover3 pd  2
F' p d F   1.325  in
in
15. Round this to the decimal equivalent of a common 1
fractional value.

F  1.375  in 0
2 4 6 8 10 12
16. Then, the parameters that depend on p d and F are:
pd  in

d p p d   3.000  in d g p d   13.500 in FIGURE 13-24


Graph of Face Width and Limits for for Problem 13-24

Vtp p d   628  Cvp p d   0.942 σcp p d F   99697  psi TD p d F   1567 in lbf
ft
min

© 2011 Pearson Education, Inc., Upper Saddle River, NJ. All rights reserved. This publication is protected by Copyright and written permission should be
MACHINE DESIGN - An Integrated Approach, 4th Ed. 13-24-3

17. When both pinion and gear are of the same material, the stress in the gear will always be lower than in the pinion
Therefore, the face width and diametral pitch found for the pinion should also be used for the gear.

1
Diametral pitch p d  6  in

Face width F  1.375  in

 Np 
Pitch cone angle αp  atan  αp  12.529 deg
 Ng 
d p p d  L
Pitch-cone length L  L  6.915  in  2.305  in
2  sin αp 3

18. Determine the realized factor of safety for the pinion using the above values for F and p d.

2
 Sfcp 
Pinion factor of safety Nfcp    Nfcp  1.3
 σcp pd F  

© 2011 Pearson Education, Inc., Upper Saddle River, NJ. All rights reserved. This publication is protected by Copyright and written permission should be
MACHINE DESIGN - An Integrated Approach, 4th Ed. 13-25-1

PROBLEM 13-25
Statement: Size the spiral gears in problem 13-7 for a surface factor of safety of at least 1.4 assuming a
10-year, 3-shift life, steady torque, quality index of 8, an AISI 4340 steel pinion and gear.
Units: yr  2080 hr
Given: Factor of safety Nfc  1.4 Number of gear teeth Ng  80
Power to be transmitted H  3  hp Reliability R  0.99
Rotational speed of pinion n p  600  rpm AGMA Quality level Qv  8
Number of pinion teeth Np  16 Life (years) Life  10 yr

Solution: See Mathcad file P1325.


Pinion
1. Determine the surface geometry factor, I, from Figure 13-7. I  0.158
2. Write the equations for pitch diameter, pitchline velocity, and transmitted load in terms of the unknown
diametral pitch, p d. Note that, in Mathcad, unit conversion factors are not included.
Np Ng
Pitch diameter of pinion (in) d p p d   d g p d  
pd pd

Np
pitchline velocity (fpm) Vtp p d    np
2 pd

H
Transmitted load (lbf) Tp  Tp  315.127  in lbf
np

3. Set the application factor, Ca Ca  1 Cs  1 Cf  1

B  0.25  12  Qv
0.6667
4. Write the equation for the dynamic load factor, C B  0.63
v

A  50  56 ( 1  B) A  70.721
B
Cvp p d    
A

 A  Vtp pd  
min

 ft 

5. Tentatively choose the mounting factor, Cmd Cmd  3.0 Cxc  1

6. Choose an elastic coefficient from Table 11-18


0.5
(steel on steel) and a stress adjustment constant. Cp  2100 psi Cb  0.634

7. Determine the endurance strength of the pinion.


np 9
Cycle life shifts  3 N  Life shifts N  2.246  10
2 π
 0.056
Life factor CL  2.466  N CL  0.738
Reliability CR  0.7  0.15 log( 1  R) CR  1
Hardness CH  1
Temperature factor CT  1
Material surface strength (psi)
S acp  162000 psi
AISI 4340 Nitrided steel
© 2011 Pearson Education, Inc., Upper Saddle River, NJ. All rights reserved. This publication is protected by Copyright and written permission should be
MACHINE DESIGN - An Integrated Approach, 4th Ed. 13-25-2

S acp CL
Endurance strength S fcp  S fcp  119626 psi
CT  CR

8. Write the equation for the design pinion torque.

I  Cvp p d   Sacp d p pd  0.774  CH 


2
TD p d F  
F
   
2 Cs Cmd  Cf  Ca Cxc  Cp Cb CT  CR 

9. Define the exponent z. z p d F   return 0.667 if Tp  TD p d F 


1 otherwise
10. The surface stress equation for the pinion is


z pd F 
 2 TD pd F    Tp  Ca Cmd
σcp p d F   Cp Cb      Cs Cf  Cxc
 F  d p pd  2 I   TD p d F   Cvp p d 
 
11. Write the design equations using the face-width to pitch-cone length ratio given in the text, and the bending
stress equation, solved for the unknown face width.
2
Np  Sfc 
Upper limit, F = L/3 FLover3 p d   Nfc =  
  Np    σc 
6  p d  sin atan 
Face width
  Ng  


z pd F 
 2  TD p d F    Tp 
2
 Cp Cb  Ca Cmd
F' p d F          Cs Cf  Cxc Nfc
 S fcp   d p pd  2 I   TD pd F   Cvp p d 
 
Guess value for F F  1.141  in Found by iteration with value in step 14.
2 2.5 12
12. Plot F(P) vs. p d over the range p d   
in in in
4

13. From the graph, choose a standard value of


p d from Table 12-2.
  3
F' pd F
1
p d  10 in in

14. The calculated value of F is FLover3 pd  2


F' p d F   1.141  in in
1
15. Round this to the decimal equivalent of a
common fractional value.
0
F  1.250  in 2 3 4 5 6 7 8 9 10 11 12
pd  in
16. Then, the parameters that depend on p d and F are:
FIGURE 13-25
d p p d   1.600  in d g p d   8.000  in Graph of Face Width and Limits for the Pinion in
Problem 13-25

© 2011 Pearson Education, Inc., Upper Saddle River, NJ. All rights reserved. This publication is protected by Copyright and written permission should be
MACHINE DESIGN - An Integrated Approach, 4th Ed. 13-25-3

Vtp p d   251  Cvp p d   0.880 σcp p d F   98091  psi TD p d F   658  in lbf
ft
min

17. When both pinion and gear are of the same material, the stress in the gear will always be lower than in the pinion
Therefore, the face width and diametral pitch found for the pinion should also be used for the gear.

1
Diametral pitch p d  10 in

Face width F  1.250  in

 Np 
Pitch cone angle αp  atan  αp  11.31  deg
 Ng 
d p p d  L
Pitch-cone length L  L  4.079  in  1.360  in
2  sin αp 3

18. Determine the realized factor of safety for the pinion using the above values for F and p d.

2
 Sfcp 
Pinion factor of safety Nfcp    Nfcp  1.5
 σcp pd F  

© 2011 Pearson Education, Inc., Upper Saddle River, NJ. All rights reserved. This publication is protected by Copyright and written permission should be
MACHINE DESIGN - An Integrated Approach, 4th Ed. 13-26-1
PROBLEM 13-26
Statement: Find the rated power and rated output torque of the wormset in Problem 13-9 with an input speed o
2200 rpm.
1
Units: fpm  ft  min

Given: Worm pitch diameter d w  50 mm Gear ratio mG  22


Axial pitch p x  10 mm Input (worm) speed n w  2200 rpm
Threads on worm Nw  2 Pressure angle ϕ  20 deg

Solution: See Mathcad file P1326.


1. Use equation 13.13 to calculate the lead. L  p x  Nw L  20 mm

λ  atan 
L
2. Use equation 13.12 to calculate the lead angle.  λ  7.256  deg
 π d w 
3. Calculate the number of teeth on the gear Ng  mG Nw Ng  44

4. Use equation 13.13 to calculate gear diameter. p c  p x


p c  Ng
d g  d g  140.056  mm
π
dw  dg
5. Use equation 13.17 to calculate the center distance. C 
2

6. Find the maximum recommended face width from equation 13.19.


Fmax  0.67 d w Fmax  33.5 mm

7. Find the materials factor Cs from equation 13.24. Since C < 203 mm, Cs  1000.

8. Find the ratio correction factor Cm from equations 13.25. Based on mG  22 , the second of the
expressions in that equation set will be used.
2
Cm  0.0107 mG  56 mG  5145 Cm  0.821
9. Find the tangential velocity Vt from equation 13.27.

nw dw
Vt  Vt  1143 fpm
2  cos( λ)
10. Use this velocity to find the velocity factor Cv from equations 13.26. For this value of Vt, the second of
these equations is appropriate.
 0.571
 Vt 
Cv  13.31    Cv  0.239
 fpm 
11. Find the tangential load Wt from equation 13.23.
0.8
 dg  N
Wtg  Cs Cm Cv    Fmax Wtg  4510 N
 mm  75.948 mm

12. Find the coefficient of friction from the third expression in equation 13.29.
0.450
 
Vt
 0.110  
μ  0.103  e  fpm   0.012 μ  0.020
© 2011 Pearson Education, Inc., Upper Saddle River, NJ. All rights reserved. This publication is protected by Copyright and written permission should be
MACHINE DESIGN - An Integrated Approach, 4th Ed. 13-26-2

13. Find the friction force Wf from equation 13.28.

μ  Wtg
Wf  Wf  94.5 N
cos( λ)  cos( ϕ)

14. Find the rated output power from equation 13.21.


nw dg
Φ o    Wtg Φ o  3.31 kW
mG 2

15. Find the power lost in the mesh from equation 13.22.
Φ l  Vt Wf Φ l  0.55 kW
16. Find the rated input power from equation 13.20.
Φ  Φ o  Φ l Φ  3.86 kW

17. The efficiency of the gearset is


Φo
e  e  85.8 %
Φ
18. Find the rated output torque from equation 13.31.

dg
Tg  Wtg Tg  316  N  m
2

© 2011 Pearson Education, Inc., Upper Saddle River, NJ. All rights reserved. This publication is protected by Copyright and written permission should be
MACHINE DESIGN - An Integrated Approach, 4th Ed. 13-27-1
PROBLEM 13-27
Statement: Find the rated power and rated output torque of the wormset in Problem 13-10 with an input
speed of 1400 rpm.
1
Units: fpm  ft  min
Given: Worm pitch diameter d w  1.75 in Gear ratio mG  17
Axial pitch p x  0.200  in Input (worm) speed n w  1400 rpm
Threads on worm Nw  3 Pressure angle ϕ  20 deg
Solution: See Mathcad file P1327.
1. Use equation 13.13 to calculate the lead. L  p x  Nw L  0.600  in

λ  atan 
L
2. Use equation 13.12 to calculate the lead angle.  λ  6.228  deg
 π d w 
3. Calculate the number of teeth on the gear Ng  mG Nw Ng  51

4. Use equation 13.13 to calculate gear diameter. p c  p x


p c  Ng
d g  d g  3.247  in
π
dw  dg
5. Use equation 13.17 to calculate the center distance. C  C  2.498  in
2

6. Find the maximum recommended face width from equation 13.19.


Fmax  0.67 d w Fmax  1.173  in

7. Find the materials factor Cs from equation 13.24. Since C < 203 mm, Cs  1000.

8. Find the ratio correction factor Cm from equations 13.25. Based on mG  17 , the first of the expressions
in that equation set will be used.
2
Cm  0.0200 mG  40 mG  76  0.46 Cm  0.815
9. Find the tangential velocity Vt from equation 13.27.

nw dw
Vt  Vt  645  fpm
2  cos( λ)

10. Use this velocity to find the velocity factor Cv from equations 13.26. For this value of Vt, the first of these
equations is appropriate.
Vt
 0.0011
fpm
Cv  0.659  e Cv  0.324

11. Find the tangential load Wt from equation 13.23.


0.8
 dg  N
Wtg  Cs Cm Cv    Fmax Wtg  794  lbf
 mm  75.948 mm

12. Find the coefficient of friction from the third expression in equation 13.29.
0.450
 Vt 
 0.110  
μ  0.103  e  fpm   0.012 μ  0.026
© 2011 Pearson Education, Inc., Upper Saddle River, NJ. All rights reserved. This publication is protected by Copyright and written permission should be
MACHINE DESIGN - An Integrated Approach, 4th Ed. 13-27-2

13. Find the friction force Wf from equation 13.28.

μ  Wtg
Wf  Wf  21.8 lbf
cos( λ)  cos( ϕ)

14. Find the rated output power from equation 13.21.


nw dg
Φ o    Wtg Φ o  1.69 hp
mG 2

15. Find the power lost in the mesh from equation 13.22.
Φ l  Vt Wf Φ l  0.43 hp
16. Find the rated input power from equation 13.20.
Φ  Φ o  Φ l Φ  2.11 hp

17. The efficiency of the gearset is


Φo
e  e  79.8 %
Φ
18. Find the rated output torque from equation 13.31.

dg
Tg  Wtg Tg  1290 in lbf
2

© 2011 Pearson Education, Inc., Upper Saddle River, NJ. All rights reserved. This publication is protected by Copyright and written permission should be
MACHINE DESIGN - An Integrated Approach, 4th Ed. 13-28-1
PROBLEM 13-28
Statement: Find the rated power and rated output torque of the wormset in Problem 13-11 with an input speed
of 4500 rpm.
1
Units: fpm  ft  min

Given: Worm pitch diameter d w  40 mm Gear ratio mG  82


Axial pitch p x  5  mm Input (worm) speed n w  4500 rpm
Threads on worm Nw  1 Pressure angle ϕ  20 deg
Solution: See Mathcad file P1328.
1. Use equation 13.13 to calculate the lead. L  p x  Nw L  5  mm

λ  atan 
L
2. Use equation 13.12 to calculate the lead angle.  λ  2.279  deg
 π d w 
3. Calculate the number of teeth on the gear Ng  mG Nw Ng  82

4. Use equation 13.13 to calculate gear diameter. p c  p x


p c  Ng
d g  d g  130.51 mm
π
dw  dg
5. Use equation 13.17 to calculate the center distance. C  C  85.25  mm
2

6. Find the maximum recommended face width from equation 13.19.


Fmax  0.67 d w Fmax  26.8 mm

7. Find the materials factor Cs from equation 13.24. Since C < 203 mm, Cs  1000.

8. Find the ratio correction factor Cm from equations 13.25. Based on mG  82 , the third of the expressions
in that equation set will be used.

Cm  1.1483  0.00658  mG Cm  0.609


9. Find the tangential velocity Vt from equation 13.27.

nw dw
Vt  Vt  1857 fpm
2  cos( λ)

10. Use this velocity to find the velocity factor Cv from equations 13.26. For this value of Vt, the second of these
equations is appropriate.
 0.571
 Vt 
Cv  13.31    Cv  0.181
 fpm 
11. Find the tangential load Wt from equation 13.23.
0.8
 dg  N
Wtg  Cs Cm Cv    Fmax Wtg  1915 N
 mm  75.948 mm

12. Find the coefficient of friction from the third expression in equation 13.29.
0.450
 
Vt
 0.110  
μ  0.103  e  fpm   0.012 μ  0.016
© 2011 Pearson Education, Inc., Upper Saddle River, NJ. All rights reserved. This publication is protected by Copyright and written permission should be
MACHINE DESIGN - An Integrated Approach, 4th Ed. 13-28-2

13. Find the friction force Wf from equation 13.28.

μ  Wtg
Wf  Wf  32.6 N
cos( λ)  cos( ϕ)

14. Find the rated output power from equation 13.21.


nw dg
Φ o    Wtg Φ o  718  W
mG 2

15. Find the power lost in the mesh from equation 13.22.
Φ l  Vt Wf Φ l  307  W
16. Find the rated input power from equation 13.20.
Φ  Φ o  Φ l Φ  1026 W

17. The efficiency of the gearset is


Φo
e  e  70.0 %
Φ
18. Find the rated output torque from equation 13.31.

dg
Tg  Wtg Tg  125  N  m
2

© 2011 Pearson Education, Inc., Upper Saddle River, NJ. All rights reserved. This publication is protected by Copyright and written permission should be
MACHINE DESIGN - An Integrated Approach, 4th Ed. 13-29-1

PROBLEM 13-29 _____

Statement: A 23-tooth helical gear is cut with a 20-degree-pressure-angle hob at a helix angle of 25 deg.
The hob has a standard diametral pitch of 5. The resulting teeth have standard spur-gear
dimensions in the normal plane. Find the pitch diameter, addendum, dedendum, outside
diameter, normal, transverse, and axial pitch, and the transverse pressure angle.

Given: Normal plane data:


1
Pressure angle ϕn  20 deg Diametral pitch p nd  5  in
Other data:
Number of teeth N  23 Helix angle ψ  25 deg

Solution: See Mathcad file P1329.

1. Use equation 13.1d to calculate the transverse diametral pitch.


1
p d  p nd  cos( ψ) p d  4.532  in

2. Use equations 13.1 to find the pitch diameter, transverse pitch, normal pitch, and axial pitch.

N
Pitch diameter d  d  5.076  in
pd

π
Transverse pitch p t  p t  0.693  in
pd

Normal pitch p n  p t  cos( ψ) p n  0.628  in

pn
Axial pitch p x  p x  1.487  in
sin( ψ)

3. Use the equations in Table 12-1 with the normal diametral pitch to calculate the addendum, dedendum, and
outside diameter.

1
Addendum a  a  0.200  in
p nd

1.25
Dedendum b  b  0.250  in
p nd

Outside diameter D  d  2  a D  5.476  in

4. Use equation 13.2 to find the pressure angle in the transverse plane.

 tan ϕn 
ϕt  atan  ϕt  21.880 deg
 cos( ψ) 

© 2011 Pearson Education, Inc., Upper Saddle River, NJ. All rights reserved. This publication is protected by Copyright and written permission should be
MACHINE DESIGN - An Integrated Approach, 4th Ed. 13-30-1

PROBLEM 13-30 _____

Statement: A 38-tooth helical gear is cut with a 25-degree-pressure-angle hob at a helix angle of 30 deg.
The hob has a standard diametral pitch of 4. The resulting teeth have standard spur-gear
dimensions in the normal plane. Find the pitch diameter, addendum, dedendum, outside
diameter, normal, transverse, and axial pitch, and the transverse pressure angle.

Given: Normal plane data:


1
Pressure angle ϕn  25 deg Diametral pitch p nd  4  in
Other data:
Number of teeth N  38 Helix angle ψ  30 deg

Solution: See Mathcad file P1330.

1. Use equation 13.1d to calculate the transverse diametral pitch.


1
p d  p nd  cos( ψ) p d  3.464  in

2. Use equations 13.1 to find the pitch diameter, transverse pitch, normal pitch, and axial pitch.

N
Pitch diameter d  d  10.970 in
pd

π
Transverse pitch p t  p t  0.907  in
pd

Normal pitch p n  p t  cos( ψ) p n  0.785  in

pn
Axial pitch p x  p x  1.571  in
sin( ψ)

3. Use the equations in Table 12-1 with the normal diametral pitch to calculate the addendum, dedendum, and
outside diameter.

1
Addendum a  a  0.250  in
p nd

1.25
Dedendum b  b  0.313  in
p nd

Outside diameter D  d  2  a D  11.470 in

4. Use equation 13.2 to find the pressure angle in the transverse plane.

 tan ϕn 
ϕt  atan  ϕt  28.300 deg
 cos( ψ) 

© 2011 Pearson Education, Inc., Upper Saddle River, NJ. All rights reserved. This publication is protected by Copyright and written permission should be
MACHINE DESIGN - An Integrated Approach, 4th Ed. 13-31-1

PROBLEM 13-31 _____


Statement: A 39-tooth, 20-deg helix angle, helical gear is in mesh with an 18-tooth pinion. The p d = 8 and
 = 25 deg. Find the transverse and axial contact ratios.

Given: Tooth numbers: Ng  39 Np  18


Pressure angle ϕ  25 deg Helix angle ψ  20 deg
1
Diametral pitch p d  8  in

Assumptions: The face width factor is fwf  12


Solution: See Mathcad file P1331.
1. Use equation 12.7 to find the transverse contact ratio.
π
Circular pitch p c  p c  0.393  in
pd

Base pitch p b  p c  cos( ϕ) p b  0.356  in

Pinion:
Np
Pitch dia d p  d p  2.250  in
pd

Pitch rad rp  0.5 d p rp  1.125  in

Gear:
Ng
Pitch dia d g  d g  4.875  in
pd

Pitch rad rg  0.5 d g rg  2.438  in

1.0
Addendum a  a  0.125  in
pd

Np  Ng
Center distance C  C  3.563  in
2 pd

Length of action

Z  rp  a 2  rp cos( ϕ)  2   rg  a 2  rg cos( ϕ)  2  C sin( ϕ)


Z  0.516  in
Z
Transverse contact ratio mp  mp  1.450
pb

2. Use equation 13.5 to find the axial contact ratio.


fwf
Face width F  F  1.500  in
pd

F  p d  tan ( ψ)
Axial contact ratio mF  mF  1.390
π
© 2011 Pearson Education, Inc., Upper Saddle River, NJ. All rights reserved. This publication is protected by Copyright and written permission should be
MACHINE DESIGN - An Integrated Approach, 4th Ed. 13-32-1

PROBLEM 13-32 _____


Statement: A 79-tooth, 30-deg helix angle, helical gear is in mesh with an 20-tooth pinion. The p d = 6 and
 = 20 deg. Find the transverse and axial contact ratios.

Given: Tooth numbers: Ng  79 Np  20


Pressure angle ϕ  20 deg Helix angle ψ  30 deg
1
Diametral pitch p d  6  in

Assumptions: The face width factor is fwf  12


Solution: See Mathcad file P1332.
1. Use equation 12.7 to find the transverse contact ratio.
π
Circular pitch p c  p c  0.524  in
pd

Base pitch p b  p c  cos( ϕ) p b  0.492  in

Pinion:
Np
Pitch dia d p  d p  3.333  in
pd

Pitch rad rp  0.5 d p rp  1.667  in

Gear:
Ng
Pitch dia d g  d g  13.167 in
pd

Pitch rad rg  0.5 d g rg  6.583  in

1.0
Addendum a  a  0.167  in
pd

Np  Ng
Center distance C  C  8.250  in
2 pd

Length of action

Z  rp  a 2  rp cos( ϕ)  2   rg  a 2  rg cos( ϕ)  2  C sin( ϕ)


Z  0.832  in
Z
Transverse contact ratio mp  mp  1.690
pb

2. Use equation 13.5 to find the axial contact ratio.


fwf
Face width F  F  2.000  in
pd

F  p d  tan ( ψ)
Axial contact ratio mF  mF  2.205
π
© 2011 Pearson Education, Inc., Upper Saddle River, NJ. All rights reserved. This publication is protected by Copyright and written permission should be
MACHINE DESIGN - An Integrated Approach, 4th Ed. 13-33-1

PROBLEM 13-33 _____

Statement: If the gearset in Problem 13-31 transmits 135 HP at 1200 pinion rpm, find the torque on each shaft

Given: Tooth numbers: Ng  39 Np  18


Pinion speed ωp  1200 rpm Transmitted power P  135  hp

Assumptions: There is no loss of power in the gear mesh (100% efficiency).

Solution: See Mathcad file P1333.

1. For the pinion shaft


P
Tp  Tp  7090 in lbf Tp  590.9  ft  lbf
ωp

2. The gear shaft will rotate at a lower speed, which is determined by the gear ratio. (The speed will be
decreased in proportion to the ratio and the torque will be increased by the reciprocal of the ratio).

3. For the gear shaft

Np
ωg   ωp ωg  553.846  rpm
Ng

P
Tg  Tg  15362  in lbf Tg  1280 ft  lbf
ωg

4. We could have calculated the torque on the gear shaft directly without finding the gear shaft speed,

Ng
Tg   Tp Tg  15362  in lbf Tg  1280 ft  lbf
Np

© 2011 Pearson Education, Inc., Upper Saddle River, NJ. All rights reserved. This publication is protected by Copyright and written permission should be
MACHINE DESIGN - An Integrated Approach, 4th Ed. 13-34-1
PROBLEM 13-34
Statement: If the gearset in Problem 13-32 transmits 30 kW at 1200 pinion rpm, find the torque on each shaft.

Given: Tooth numbers:


Number of pinion teeth Np  20 Pinion speed ωp  1200 rpm
Number of gear teeth Ng  79Transmitted power P  30 kW

Assumptions: There is no loss of power in the gear mesh (100% efficiency).


Solution: See Mathcad file P1334.
P
1. For the pinion shaft Tp  Tp  239  N  m
ωp

2. The gear shaft will rotate at a lower speed, which is determined by the gear ratio. (The speed will be
decreased in proportion to the ratio and the torque will be increased by the reciprocal of the ratio).
Np
3. For the gear shaft ωg   ωp ωg  303.797  rpm
Ng
P
Tg  Tg  943  N  m
ωg

4. We could have calculated the torque on the gear shaft directly without finding the gear shaft speed,

Ng
Tg   Tp Tg  943  N  m
Np

© 2011 Pearson Education, Inc., Upper Saddle River, NJ. All rights reserved. This publication is protected by Copyright and written permission should be
MACHINE DESIGN - An Integrated Approach, 4th Ed. 13-35-1

PROBLEM 13-35
Statement: Size the helical gears in problem 13-33 for a bending factor of safety of at least 2 assuming a
steady torque, 25 deg pressure angle, full depth teeth, quality index of 9, an AISI 4140 steel
pinion, and a class 40 cast iron gear.
Given: Factor of safety Nfb  2 Number of gear teeth Ng  39
Power to be transmitted H  135  hp Reliability R  0.99
Rotational speed of pinion n  1200 rpm AGMA Quality level Qv  9
7
Number of pinion teeth Np  18 Life (cycles) N  10
Pressure angle ϕ  25 deg Helix angle ψ  10 deg
Solution: See Mathcad file P1335.
Pinion
1. Determine the bending geometry factor, J (Table 13-4) Jp  0.57
2. Write the equations for pitch diameter, pitchline velocity, and transmitted load in terms of the unknown
diametral pitch, p d. Note that, in Mathcad, unit conversion factors are not included.
Np
Pitch diameter of pinion (in) d  p d  
pd

dpd  n
pitchline velocity (fpm) Vt p d  
2

Wt p d  
H
Transmitted load (lbf)
Vt  p d 

3. Set the application factor, Ka Ka  1

B  0.25  12  Qv
0.6667
4. Write the equation for the dynamic load factor, K B  0.52
v

A  50  56 ( 1  B) A  76.878
B
Kv p d    A 
 
 A  Vt  p d  
min

 ft 

5. Tentatively choose the mounting factor, Km (Assume 2 < F < 6 in) Km  1.7

Ka Wt p d   p d  Km
6. The bending stress equation for the pinion is σbp p d F  
Kv p d   F  Jp

7. Determine the endurance strength of the pinion.


 0.0323
Life factor KL  1.6831 N KL  1
Reliability KR  0.7  0.15 log( 1  R) KR  1
Temperature factor KT  1
Material bending strength (psi)
S atp  40000  psi
AISI 4140 Nitrided steel

© 2011 Pearson Education, Inc., Upper Saddle River, NJ. All rights reserved. This publication is protected by Copyright and written permission should be
MACHINE DESIGN - An Integrated Approach, 4th Ed. 13-35-2

S atp KL
Endurance strength S fbp  S fbp  40001  psi
KT  KR

8. Write the design equations using the range of face-width to diametral-pitch ratio given in the text, and the
bending stress equation, solved for the unknown face width.

FU  p d  
16
Upper limit
pd

FL  p d  
8
Lower limit
pd
S fb
Safety factor Nfb =
σb

Ka Wt p d   p d  Km Nfb


Face width F  p d  
Kv p d   Jp S fbp

1 1.5 6
9. Plot F(P) vs. p d over the p d   
range in in in
4
10. From the graph, choose a standard value of
p d from Table 12-2.
 
F pd
1 3
p d  3  in in

11. The calculated value of F is  


FL pd
2
F  p d   1.335  in
in

 
FU pd
12. Round this up to the decimal equivalent of a
in 1
common fractional value.

F  1.375  in
0
13. Then, the parameters that depend on p d and F are: 2 2.5 3 3.5 4 4.5 5 5.5 6
pd  in
d  p d   6.000  in Kv p d   0.792
FIGURE 13-35A
Graph of Face Width and Limits for the Pinion in
Vt p d   1885 Wt p d   2363 lbf
ft
Problem 13-35
min

σbp p d F   19411  psi

The assumption made in step 5 is correct so no further iteration is required.


Gear
14. Determine the bending geometry factor, J (Table 13-4) Jg  0.64
Ka Wt p d   p d  Km
15. The bending stress equation for the gear is σbg p d F  
Kv p d   F  Jg

16. Determine the endurance strength of the pinion.

Material bending strength (psi)


Class 40 cast iron S atg  13000  psi
© 2011 Pearson Education, Inc., Upper Saddle River, NJ. All rights reserved. This publication is protected by Copyright and written permission should be
MACHINE DESIGN - An Integrated Approach, 4th Ed. 13-35-3

S atg KL
Endurance strength S fbg  S fbg  13000  psi
KT  KR

17. Write the design equations using the range of face-width to diametral-pitch ratio given in the text, and the
bending stress equation, solved for the unknown face width.

FU  p d  
16
Upper limit
pd

FL  p d  
8
Lower limit
pd
S fb
Safety factor Nfb =
σb

Ka Wt p d   p d  Km Nfb


Face width F  p d  
Kv p d   Jg S fbg

1 1.5 6
18. Plot F(P) vs. p d over the range p d   
in in in

19. From the graph, choose a standard value of 6


p d from Table 12-2.

p d  3  in
1  
F pd
4.5
in
20. The calculated value of F is
 
FL pd
F  p d   3.657  in 3
in
21. Round this up to the decimal equivalent of a  
FU pd
common fractional value.
in 1.5
F  3.750  in

22. Then, the parameters that depend on P and F are: 0


2 2.5 3 3.5 4 4.5 5 5.5 6
d  p d   6.000  in Kv p d   0.792 pd  in
FIGURE 13-35B
Graph of Face Width and Limits for the Gear in
Vt p d   1885 Wt p d   2363 lbf
ft
Problem 13-35
min

σbg p d F   6339 psi

23. The gear dimensions are larger (smaller diametral pitch means bigger teeth) than for the pinion. This means that
we will accept the gear requirements for the pinion, thus, for the set

1
Diametral pitch p d  3  in

Face width F  3.750  in


24. Determine the realized factor of safety for the gear using the above values for F and p d.

S fbg
Gear factor of safety Nfbg  Nfbg  2.1
σbg p d F 
© 2011 Pearson Education, Inc., Upper Saddle River, NJ. All rights reserved. This publication is protected by Copyright and written permission should be
MACHINE DESIGN - An Integrated Approach, 4th Ed. 13-35-4

25. Check the factor of safety on the pinion:

S fbp
Pinion factor of safety Nfbp  Nfbp  5.6
σbp p d F 

© 2011 Pearson Education, Inc., Upper Saddle River, NJ. All rights reserved. This publication is protected by Copyright and written permission should be
MACHINE DESIGN - An Integrated Approach, 4th Ed. 13-36-1

PROBLEM 13-36
Statement: Size the helical gears in problem 13-34 for a bending factor of safety of at least 2.5 assuming a
steady torque, 20 deg pressure angle, full depth teeth, quality index of 11, an AISI 4340 steel pinio
and an A-7-d nodular 40 iron gear.

Given: Factor of safety Nfb  2.5 Number of gear teeth Ng  79


Power to be transmitted H  30 kW Reliability R  0.99
Rotational speed of pinion n  1200 rpm AGMA Quality level Qv  11
7
Number of pinion teeth Np  20 Life (cycles) N  10
Pressure angle ϕ  20 deg Helix angle ψ  30 deg
Solution: See Mathcad file P1336.
Pinion
1. Determine the bending geometry factor, J (Table 13-4) Jp  0.46
2. Write the equations for pitch diameter, pitchline velocity, and transmitted load in terms of the unknown
diametral pitch, p d. Note that, in Mathcad, unit conversion factors are not included.
Np
Pitch diameter of pinion (in) d  p d  
pd

dpd  n
pitchline velocity (fpm) Vt p d  
2

Wt p d  
H
Transmitted load (lbf)
Vt  p d 

3. Set the application factor, Ka Ka  1

B  0.25  12  Qv
0.6667
4. Write the equation for the dynamic load factor, K B  0.25
v

A  50  56 ( 1  B) A  92
B
Kv p d    
A

 A  Vt  p d  
min

 ft 

5. Tentatively choose the mounting factor, Km (Assume 2 < F < 6 in) Km  1.7

Ka Wt p d   p d  Km
6. The bending stress equation for the pinion is σbp p d F  
Kv p d   F  Jp

7. Determine the endurance strength of the pinion.


 0.0323
Life factor KL  1.6831 N KL  1
Reliability KR  0.7  0.15 log( 1  R) KR  1
Temperature factor KT  1
Material bending strength (psi)
S atp  42000  psi
AISI 4340 Nitrided steel

© 2011 Pearson Education, Inc., Upper Saddle River, NJ. All rights reserved. This publication is protected by Copyright and written permission should be
MACHINE DESIGN - An Integrated Approach, 4th Ed. 13-36-2

S atp KL
Endurance strength S fbp  S fbp  42001  psi
KT  KR

8. Write the design equations using the range of face-width to diametral-pitch ratio given in the text, and the
bending stress equation, solved for the unknown face width.

FU  p d  
16
Upper limit
pd

FL  p d  
8
Lower limit
pd
S fb
Safety factor Nfb =
σb

Ka Wt p d   p d  Km Nfb


Face width F  p d  
Kv p d   Jp S fbp

1 1.5 10
9. Plot F(P) vs. p d over the p d   
range in in in
4
10. From the graph, choose a standard value of
p d from Table 12-2.
 
F pd
1 3
p d  6  in in

11. The calculated value of F is  


FL pd
2
F  p d   1.804  in
in

 
FU pd
12. Round this up to the decimal equivalent of a
in 1
common fractional value.

F  1.875  in
0
13. Then, the parameters that depend on p d and F are: 4 5 6 7 8 9 10
pd  in
d  p d   3.333  in Kv p d   0.927
FIGURE 13-36A
Graph of Face Width and Limits for the Pinion in
Vt p d   1047 Wt p d   1268 lbf
ft
Problem 13-36
min

σbp p d F   16166  psi

The assumption made in step 5 is correct so no further iteration is required.


Gear
14. Determine the bending geometry factor, J (Table 13-4) Jg  0.50

Ka Wt p d   p d  Km
15. The bending stress equation for the gear is σbg p d F  
Kv p d   F  Jg

16. Determine the endurance strength of the pinion.


Material bending strength (psi)
A-7-d nodular iron S atg  34000  psi
© 2011 Pearson Education, Inc., Upper Saddle River, NJ. All rights reserved. This publication is protected by Copyright and written permission should be
MACHINE DESIGN - An Integrated Approach, 4th Ed. 13-36-3

S atg KL
Endurance strength S fbg  S fbg  34001  psi
KT  KR

17. Write the design equations using the range of face-width to diametral-pitch ratio given in the text, and the
bending stress equation, solved for the unknown face width.

FU  p d  
16
Upper limit
pd

FL  p d  
8
Lower limit
pd
S fb
Safety factor Nfb =
σb

Ka Wt p d   p d  Km Nfb


Face width F  p d  
Kv p d   Jg S fbg

1 1.5 10
18. Plot F(P) vs. p d over the range p d   
in in in

19. From the graph, choose a standard value of 6


p d from Table 12-2.

1  
F pd
p d  6  in 4.5
in
20. The calculated value of F is  
FL pd
F  p d   2.050  in in 3

21. Round this up to the decimal equivalent of a  


FU pd
common fractional value. in 1.5

F  2.125  in

22. Then, the parameters that depend on P and F are: 0


4 5 6 7 8 9 10
pd  in
d  p d   3.333  in Kv p d   0.927
FIGURE 13-36B
Graph of Face Width and Limits for the Gear in
Vt p d   1047 Wt p d   1268 lbf
ft
Problem 13-36
min

σbg p d F   13123  psi

23. The gear dimensions are larger (smaller diametral pitch means bigger teeth) than for the pinion. This means that
we will accept the gear requirements for the pinion, thus, for the set

1
Diametral pitch p d  6  in

Face width F  2.125  in


24. Determine the realized factor of safety for the gear using the above values for F and p d.

S fbg
Gear factor of safety Nfbg  Nfbg  2.6
σbg p d F 

© 2011 Pearson Education, Inc., Upper Saddle River, NJ. All rights reserved. This publication is protected by Copyright and written permission should be
MACHINE DESIGN - An Integrated Approach, 4th Ed. 13-36-4

25. Check the factor of safety on the pinion:

S fbp
Pinion factor of safety Nfbp  Nfbp  2.9
σbp p d F 

© 2011 Pearson Education, Inc., Upper Saddle River, NJ. All rights reserved. This publication is protected by Copyright and written permission should be
MACHINE DESIGN - An Integrated Approach, 4th Ed. 13-37-1

PROBLEM 13-37
Statement: Size the helical gears in problem 13-33 for a surface factor of safety of at least 1.6 assuming a
steady torque, 25 deg pressure angle, full depth teeth, quality index of 9, an AISI 4140 steel
pinion, and a class 40 cast iron gear.

Given: Factor of safety Nfc  1.6 Number of gear teeth Ng  39


Power to be transmitted H  135  hp Reliability R  0.99
Rotational speed of pinion n  1200 rpm AGMA Quality level Qv  9
7
Number of pinion teeth Np  18 Life (cycles) N  10
Pressure angle ϕ  25 deg Helix angle ψ  20 deg
Solution: See Mathcad file P1337.
Pinion
1. Determine the surface geometry factor, I.

a  p d  
1
Addendum
pd

Np Ng
Pitch radii rp p d   rg p d  
2 pd 2 pd

Center distance C p d   rp p d   rg p d 

Radii of curvature

ρp  p d   0.5  rp p d   C pd   rg pd     rp p d   cos( ϕ) 


2 2

ρg  p d   C p d   sin( ϕ)  ρp  p d 

Normal plane pressure angle ϕn  atan( tan( ϕ)  cos( ψ) ) ϕn  23.662 deg

 cos ϕn 
Base helix angle ψb  acos cos( ψ)   ψb  18.256 deg
 cos( ϕ) 
Length of action

Z  p d  
 r p  a p 2  r p  cos( ϕ) 2  
  p d   d    p d   
 r p  a p 2  r p  cos( ϕ) 2  C p  sin( ϕ) 
  g d   d    g d    d 

Z pd 
Transverse contact ratio mp p d  
π
 cos( ϕ)
pd

F  p d  tan( ψ)
Axial contact ratio mF  p d F  
π
Fractional parts of mp and mF n r p d   mp p d   floor  mp p d  
© 2011 Pearson Education, Inc., Upper Saddle River, NJ. All rights reserved. This publication is protected by Copyright and written permission should be
MACHINE DESIGN - An Integrated Approach, 4th Ed. 13-37-2

n a p d F   mF  p d F   floor  mF  p d F  

p t  p d   p n  p d   p t  p d   cos( ψ)
π
Axial pitch
pd

pn  pd 
p x  p d  
sin( ψ)

Minimum length of lines of contact

mp pd  F  napd F  nrpd   px pd  


Lmin p d F   return if n a p d F    1  n r p d  
cos ψb

mp pd   F   1  na p d F     1  nr pd    px  p d 


return if n a p d F    1  n r p d  
cos ψb

mN  p d F  
F
Load sharing ratio
Lmin p d F 

I  p d F  
cos( ϕ)
Geometry factor
 
 ρ  p   ρ  p    2 rp pd   mN  p d F 
1 1
 p d g d 

2. Write the equations for pitch diameter, pitchline velocity, and transmitted load in terms of the unknown
diametral pitch, p d. Note that, in Mathcad, unit conversion factors are not included.

Np Ng
Pitch diameter of pinion (in) d p p d   d g p d  
pd pd

d p p d   n
pitchline velocity (fpm) Vt p d  
2

Wt p d  
H
Transmitted load (lbf)
Vt  p d 

3. Set the application factor, Ca Ca  1

B  0.25  12  Qv
0.6667
4. Write the equation for the dynamic load factor, C B  0.52
v

A  50  56 ( 1  B) A  76.878
B
Cv p d    A 
 
 A  Vt  p d  
min

 ft 

5. Tentatively choose the mounting factor, Cm (Assume 2 < F < 6 in Cm  1.7


0.5
6. Choose an elastic coefficient from Table 12-18 (steel on CI). Cp  2100 psi

C a W t  p d   C m
7. The surface stress equation for the pinion is σcp p d F   Cp
Cv p d   F  d p p d   I  p d F 

© 2011 Pearson Education, Inc., Upper Saddle River, NJ. All rights reserved. This publication is protected by Copyright and written permission should be
MACHINE DESIGN - An Integrated Approach, 4th Ed. 13-37-3

8. Determine the endurance strength of the pinion.


 0.056
Life factor CL  2.466  N CL  1
Reliability CR  0.7  0.15 log( 1  R) CR  1
Temperature factor CT  1
Material surface strength (psi)
S ac  165000 psi
AISI 4140 steel
S ac CL
Endurance strength S fcp  S fcp  164997 psi
CT  CR

9. Write the design equations using the range of face-width to diametral-pitch ratio given in the text, and the
bending stress equation, solved for the unknown face width.
2
 Sfc 
FU  p d   FL  p d  
16 8
Limits, safety factor Nfc =  
pd pd  σc 

C a W t  p d   C m
2
 Cp 
Face width F' p d F      Nfc
Cv p d   d p p d   I  p d F   S fcp 

Guess value for F F  3.059  in Found by iteration with value in step 12.

2 2.5 12
10. Plot F(P) vs. p d over the range p d   
in in in

11. From the graph, choose a standard value of 4


p d from Table 12-2.

1 
F' pd F 
p d  5  in 3
in
12. The calculated value of F is  
FL pd
F' p d F   3.059  in in 2

13. Round this up to the decimal equivalent of a FU pd  


common fractional value. in 1

F  3.125  in
14. Then, the parameters that depend on p d and F are: 0
2 3 4 5 6 7 8 9 10 11 12
d p p d   3.600  in d g p d   7.800  in pd  in
FIGURE 13-37A
Vt p d   1131 Wt p d   3939 lbf
ft Graph of Face Width and Limits for the Pinion
min (surface) in Problem 13-37

σcp p d F   128455 psi mp p d   1.45 mF  p d F   1.81 n r p d   0.45

n a p d F   0.81 mN  p d F   0.682 Lmin p d F   4.582  in I  p d F   0.192

© 2011 Pearson Education, Inc., Upper Saddle River, NJ. All rights reserved. This publication is protected by Copyright and written permission should be
MACHINE DESIGN - An Integrated Approach, 4th Ed. 13-37-4

Gear

C a W t  p d   C m
15. The surface stress equation for the gear is σcg p d F   Cp
Cv p d   F  d g p d   I  p d F 

16. Determine the endurance strength of the gear.


 0.056
Life factor CL  2.466  N CL  1
Reliability CR  0.7  0.15 log( 1  R) CR  1
Temperature factor CT  1
Material surface strength (psi)
S ac  80000  psi
Class 40 CI
S ac CL
Endurance strength S fcg  S fcg  79999  psi
CT  CR

17. Write the design equations using the range of face-width to diametral-pitch ratio given in the text, and the
bending stress equation, solved for the unknown face width.
2
 Sfc 
FU  p d   FL  p d  
16 8
Limits, safety factor Nfc = σ 
pd pd  c

C a W t  p d   C m
2
 Cp 
Face width F' p d F      Nfc
Cv p d   d g p d   I  p d F   S fcg 

Guess value for F F  3.887  in Found by iteration with value in step 20.

2 2.5 12
18. Plot F(P) vs. p d over the range p d   
in in in
6
19. From the graph, choose a standard value of
p d from Table 12-2. 
F' pd F 
4.5
1 in
p d  4  in
 
FL pd
20. The calculated value of F is 3
in
F' p d F   3.887  in
FU pd  
21. Round this up to the decimal equivalent of a in 1.5
common fractional value.

F  3.875  in
0
22. Then, the parameters that depend on p d and F are: 2 2.5 3 3.5 4 4.5 5 5.5 6
pd  in
d p p d   4.500  in d g p d   9.750  in
FIGURE 13-37B
Graph of Face Width and Limits for the Gear
Vt p d   1414 Wt p d   3151 lbf
ft
(surface) in Problem 13-37
min

σcp p d F   93300  psi mp p d   1.45 mF  p d F   1.796 n r p d   0.45

© 2011 Pearson Education, Inc., Upper Saddle River, NJ. All rights reserved. This publication is protected by Copyright and written permission should be
MACHINE DESIGN - An Integrated Approach, 4th Ed. 13-37-5

n a p d F   0.796 mN  p d F   0.684 Lmin p d F   5.662  in I  p d F   0.191

23. The gear teeth are larger than for the pinion. This means that we will accept the gear requirements for the pinion
thus, for the set

1
Diametral pitch p d  4  in

Face width F  3.875  in

24. Determine the realized factor of safety for the gear using the above values for F and p d.

2
 Sfcg 
Gear factor of safety Nfsg    Nfsg  1.6
 σcg pd F  
25. Check the factor of safety on the pinion:
2
 Sfcp 
Pinion factor of safety Nfsp   σ p F  Nfsp  3.1
 cp d  

© 2011 Pearson Education, Inc., Upper Saddle River, NJ. All rights reserved. This publication is protected by Copyright and written permission should be
MACHINE DESIGN - An Integrated Approach, 4th Ed. 13-38-1

PROBLEM 13-38
Statement: Size the helical gears in problem 13-34 for a surface factor of safety of at least 1.2 assuming a stead
torque, 20 deg pressure angle, full depth teeth, quality index of 11, an AISI 4340 steel pinion, and
A-7-d nodular iron gear.

Given: Factor of safety Nfc  1.2 Number of gear teeth Ng  79


Power to be transmitted H  30 kW Reliability R  0.99
Rotational speed of pinion n  1200 rpm AGMA Quality level Qv  11
7
Number of pinion teeth Np  20 Life (cycles) N  10
Pressure angle ϕ  20 deg Helix angle ψ  30 deg
Solution: See Mathcad file P1338.
Pinion
1. Determine the surface geometry factor, I.

a  p d  
1
Addendum
pd

Np Ng
Pitch radii rp p d   rg p d  
2 pd 2 pd

Center distance C p d   rp p d   rg p d 

Radii of curvature

ρp  p d   0.5  rp p d   C pd   rg pd     rp p d   cos( ϕ) 


2 2

ρg  p d   C p d   sin( ϕ)  ρp  p d 

Normal plane pressure angle ϕn  atan( tan( ϕ)  cos( ψ) ) ϕn  17.495 deg

 cos ϕn 
Base helix angle ψb  acos cos( ψ)   ψb  28.481 deg
 cos( ϕ) 
Length of action

Z  p d  
 r p  a p 2  r p  cos( ϕ) 2  
  p d   d    p d   
 r p  a p 2  r p  cos( ϕ) 2  C p  sin( ϕ) 
  g d   d    g d    d 

Z pd 
Transverse contact ratio mp p d  
π
 cos( ϕ)
pd

F  p d  tan( ψ)
Axial contact ratio mF  p d F  
π
Fractional parts of mp and mF n r p d   mp p d   floor  mp p d  
© 2011 Pearson Education, Inc., Upper Saddle River, NJ. All rights reserved. This publication is protected by Copyright and written permission should be
MACHINE DESIGN - An Integrated Approach, 4th Ed. 13-38-2

n a p d F   mF  p d F   floor  mF  p d F  

p t  p d   p n  p d   p t  p d   cos( ψ)
π
Axial pitch
pd

pn  pd 
p x  p d  
sin( ψ)

Minimum length of lines of contact

mp pd  F  napd F  nrpd   px pd  


Lmin p d F   return if n a p d F    1  n r p d  
cos ψb

mp pd   F   1  na p d F     1  nr pd    px  p d 


return if n a p d F    1  n r p d  
cos ψb

mN  p d F  
F
Load sharing ratio
Lmin p d F 

I  p d F  
cos( ϕ)
Geometry factor
 1  1   2 r  p   m  p F 
 ρ p  ρ p   p d N d
 p d g d 

2. Write the equations for pitch diameter, pitchline velocity, and transmitted load in terms of the unknown
diametral pitch, p d. Note that, in Mathcad, unit conversion factors are not included.

Np Ng
Pitch diameter of pinion (in) d p p d   d g p d  
pd pd

d p p d   n
pitchline velocity (fpm) Vt p d  
2

Wt p d  
H
Transmitted load (lbf)
Vt  p d 

3. Set the application factor, Ca Ca  1

B  0.25  12  Qv
0.6667
4. Write the equation for the dynamic load factor, C B  0.25
v

A  50  56 ( 1  B) A  92
B
Cv p d    A 
 
 A  Vt  p d  
min

 ft 

5. Tentatively choose the mounting factor, Cm (Assume 0 < F < 2 in Cm  1.6


0.5
6. Choose an elastic coefficient from Table 12-18 (steel on NI). Cp  2160 psi

C a W t  p d   C m
7. The surface stress equation for the pinion is σcp p d F   Cp
Cv p d   F  d p p d   I  p d F 
© 2011 Pearson Education, Inc., Upper Saddle River, NJ. All rights reserved. This publication is protected by Copyright and written permission should be
MACHINE DESIGN - An Integrated Approach, 4th Ed. 13-38-3

8. Determine the endurance strength of the pinion.


 0.056
Life factor CL  2.466  N CL  1
Reliability CR  0.7  0.15 log( 1  R) CR  1
Temperature factor CT  1
Material surface strength (psi)
S ac  160000 psi
AISI 4340 steel
S ac CL
Endurance strength S fcp  S fcp  159997 psi
CT  CR

9. Write the design equations using the range of face-width to diametral-pitch ratio given in the text, and the
bending stress equation, solved for the unknown face width.
2
 Sfc 
FU  p d   FL  p d  
16 8
Limits, safety factor Nfc =  
pd pd  σc 

C a W t  p d   C m
2
 Cp 
Face width F' p d F      Nfc
Cv p d   d p p d   I  p d F   S fcp 

Guess value for F F  1.599  in Found by iteration with value in step 12.

6 6.5 16
10. Plot F(P) vs. p d over the range p d   
in in in

11. From the graph, choose a standard value of 2


p d from Table 12-2.

1 
F' pd F 
p d  10 in 1.5
in
12. The calculated value of F is  
FL pd
F' p d F   1.598  in in 1

13. Round this up to the decimal equivalent of a FU pd  


common fractional value. in 0.5

F  1.625  in
14. Then, the parameters that depend on p d and F are: 0
6 7 8 9 10 11 12 13 14 15 16
d p p d   2.000  in d g p d   7.900  in pd  in
FIGURE 13-38A
Vt p d   628  Wt p d   2113 lbf
ft Graph of Face Width and Limits for the Pinion
min (surface) in Problem 13-38

σcp p d F   144632 psi mp p d   1.69 mF  p d F   2.986 n r p d   0.69

n a p d F   0.986 mN  p d F   0.52 Lmin p d F   3.123  in I  p d F   0.246

© 2011 Pearson Education, Inc., Upper Saddle River, NJ. All rights reserved. This publication is protected by Copyright and written permission should be
MACHINE DESIGN - An Integrated Approach, 4th Ed. 13-38-4

Gear

C a W t  p d   C m
15. The surface stress equation for the gear is σcg p d F   Cp
Cv p d   F  d g p d   I  p d F 

16. Determine the endurance strength of the gear.


 0.056
Life factor CL  2.466  N CL  1
Reliability CR  0.7  0.15 log( 1  R) CR  1
Temperature factor CT  1
Material surface strength (psi)
S ac  100000 psi
A-7-d nodular iron
S ac CL
Endurance strength S fcg  S fcg  99998  psi
CT  CR

17. Write the design equations using the range of face-width to diametral-pitch ratio given in the text, and the
bending stress equation, solved for the unknown face width.
2
 Sfc 
FU  p d   FL  p d  
16 8
Limits, safety factor Nfc = σ 
pd pd  c

C a W t  p d   C m
2
 Cp 
Face width F' p d F      Nfc
Cv p d   d g p d   I  p d F   S fcg 

Guess value for F F  1.040  in Found by iteration with value in step 20.

6 6.5 16
18. Plot F(P) vs. p d over the range p d   
in in in
2
19. From the graph, choose a standard value of
p d from Table 12-2. 
F' pd F 
1.5
1 in
p d  10 in
 
FL pd
20. The calculated value of F is 1
in
F' p d F   1.041  in
FU pd  
21. Round this up to the decimal equivalent of a in 0.5
common fractional value.

F  1.125  in
0
22. Then, the parameters that depend on p d and F are: 6 7 8 9 10 11 12 13 14 15 16
pd  in
d p p d   2.000  in d g p d   7.900  in
FIGURE 13-38B
Graph of Face Width and Limits for the Gear
Vt p d   628  Wt p d   2113 lbf
ft
(surface) in Problem 13-38
min

σcg p d F   88013  psi mp p d   1.69 mF  p d F   2.067 n r p d   0.69

© 2011 Pearson Education, Inc., Upper Saddle River, NJ. All rights reserved. This publication is protected by Copyright and written permission should be
MACHINE DESIGN - An Integrated Approach, 4th Ed. 13-38-5

n a p d F   0.067 mN  p d F   0.527 Lmin p d F   2.135  in I  p d F   0.243

23. The gear teeth are smaller than for the pinion. This means that we will accept the pinnion requirements for the
gear thus, for the set

1
Diametral pitch p d  10 in

Face width F  1.625  in

24. Determine the realized factor of safety for the gear using the above values for F and p d.

2
 Sfcg 
Gear factor of safety Nfsg    Nfsg  1.9
 σcg pd F  
25. Check the factor of safety on the pinion:
2
 Sfcp 
Pinion factor of safety Nfsp   σ p F  Nfsp  1.2
 cp d  

© 2011 Pearson Education, Inc., Upper Saddle River, NJ. All rights reserved. This publication is protected by Copyright and written permission should be
MACHINE DESIGN - An Integrated Approach, 4th Ed. 13-39-1

PROBLEM 13-39 _____

Statement: A 90-deg straight bevel gearset is needed to give a 3:1 reduction. Determine the pitch cone
angles, pitch diameters, and gear forces if the 25-deg pressure angle pinion has 15 teeth of p d
= 4, and the transmitted power is 8 hp at 550 pinion rpm.
Given: Power transmitted H  8  hp Gear ratio mG  3
1
Pinion speed ωp  550  rpm Diametral pitch p d  4  in
Teeth on pinion Np  15 Pressure angle ϕ  25 deg
Solution: See Mathcad file P1339.
1. Use equation 13.7b to calculate number of teeth on the gear. Ng  mG Np Ng  45

2. Use equation 12.4a to calculate the pitch diameters.

Np
Pinion d p  d p  3.750  in
pd

Ng
Gear d g  d g  11.250 in
pd

3. Use equation 13.7b to calculate the pitch cone angles.

αp  atan 
1
Pinion  αp  18.435 deg
 mG 
Gear αg  atan mG αg  71.565 deg

Note that αp  αg  90.000 deg

4. Determine the torque on the pinion shaft and the transmitted force.
H
Pinion Tp  Tp  916.7  in lbf
ωp

2  Tp
Transmitted force Wt  Wt  488.9  lbf
dp

5. Use equations 13.8a to calculate the pinion and gear forces.

Pinion Wap  Wt tan ( ϕ)  sin αp Wap  72.10  lbf

Wrp  Wt tan( ϕ)  cos αp Wrp  216.29 lbf

Gear Wag  Wt tan ( ϕ)  sin αg Wag  216.29 lbf

Wrg  Wt tan( ϕ)  cos αg Wrg  72.10  lbf


1
Total force W  Wt ( cos( ϕ) ) W  539.468  lbf

© 2011 Pearson Education, Inc., Upper Saddle River, NJ. All rights reserved. This publication is protected by Copyright and written permission should be
MACHINE DESIGN - An Integrated Approach, 4th Ed. 13-40-1

PROBLEM 13-40 _____

Statement: A 90-deg straight bevel gearset is needed to give a 6:1 reduction. Determine the pitch cone
angles, pitch diameters, and gear forces if the 20-deg pressure angle pinion has 20 teeth of p d
= 8, and the transmitted power is 3 kW at 900 pinion rpm.

Given: Power transmitted H  3  kW Gear ratio mG  6


1
Pinion speed ωp  900  rpm Diametral pitch p d  8  in
Teeth on pinion Np  20 Pressure angle ϕ  20 deg
Solution: See Mathcad file P1340.
1. Use equation 13.7b to calculate number of teeth on the gear. Ng  mG Np Ng  120

2. Use equation 12.4a to calculate the pitch diameters.

Np
Pinion d p  d p  2.500  in
pd

Ng
Gear d g  d g  15.000 in
pd

3. Use equation 13.7b to calculate the pitch cone angles.

αp  atan 
1
Pinion  αp  9.462  deg
 mG 
Gear αg  atan mG αg  80.538 deg

Note that αp  αg  90.000 deg

4. Determine the torque on the pinion shaft and the transmitted force.
H
Pinion Tp  Tp  281.7  in lbf
ωp

2  Tp
Transmitted force Wt  Wt  225.4  lbf
dp

5. Use equations 13.8a to calculate the pinion and gear forces.

Pinion Wap  Wt tan ( ϕ)  sin αp Wap  13.49  lbf

Wrp  Wt tan( ϕ)  cos αp Wrp  80.92  lbf

Gear Wag  Wt tan ( ϕ)  sin αg Wag  80.92  lbf

Wrg  Wt tan( ϕ)  cos αg Wrg  13.49  lbf


1
Total force W  Wt ( cos( ϕ) ) W  239.847  lbf

© 2011 Pearson Education, Inc., Upper Saddle River, NJ. All rights reserved. This publication is protected by Copyright and written permission should be
MACHINE DESIGN - An Integrated Approach, 4th Ed. 13-41-1

PROBLEM 13-41 _____

Statement: A 90-deg spiral bevel gearset is needed to give an 8:1 reduction. Determine the pitch cone
angles, pitch diameters, and gear forces if the 20-deg pressure angle pinion has 21 teeth of p d =
10, and the transmitted power is 2.5 kW at 1100 pinion rpm.

Given: Power transmitted H  2.5 kW Gear ratio mG  8


1
Pinion speed ωp  1100 rpm Diametral pitch p d  10 in
Teeth on pinion Np  21 Pressure angle ϕ  20 deg

Assumptions: The spiral angle is ψ  35 deg.

Solution: See Mathcad file P1341.


1. Use equation 13.7b to calculate number of teeth on the gear. Ng  mG Np Ng  168

2. Use equation 12.4a to calculate the pitch diameters.

Np
Pinion d p  d p  2.100  in
pd

Ng
Gear d g  d g  16.800 in
pd

3. Use equation 13.7b to calculate the pitch cone angles.

αp  atan 
1
Pinion  αp  7.125  deg
 mG 
Gear αg  atan mG αg  82.875 deg

Note that αp  αg  90.000 deg

4. Determine the torque on the pinion shaft and the transmitted force.
H
Pinion Tp  Tp  192.1  in lbf
ωp

2  Tp
Transmitted force Wt  Wt  182.9  lbf
dp

5. Use equations 13.2 to calculate the pressure angle in the normal plane.

ϕn  atan( tan( ϕ)  cos( ψ) ) ϕn  16.602 deg

6. Use equations 13.8b to calculate the pinion and gear forces.


Pinion
Wt
Wap    tan ϕn  sin αp  sin( ψ)  cos αp  Wap  118.85 lbf
cos( ψ)

© 2011 Pearson Education, Inc., Upper Saddle River, NJ. All rights reserved. This publication is protected by Copyright and written permission should be
MACHINE DESIGN - An Integrated Approach, 4th Ed. 13-41-2

Wt
Wrp    tan  ϕn  cos αp  sin( ψ)  sin αp  Wrp  81.96  lbf
cos( ψ)

Gear
Wt
Wag    tan ϕn  sin αg  sin( ψ)  cos αg  Wag  50.18  lbf
cos( ψ)
Wt
Wrg    tan  ϕn  cos αg  sin( ψ)  sin αg  Wrg  135.37 lbf
cos( ψ)

1
Total force W  Wt ( cos( ϕ) ) W  194.681  lbf

© 2011 Pearson Education, Inc., Upper Saddle River, NJ. All rights reserved. This publication is protected by Copyright and written permission should be
MACHINE DESIGN - An Integrated Approach, 4th Ed. 13-42-1

PROBLEM 13-42 _____

Statement: A 1-start wormset has d = 2.00 in, p x = 0.25 in, mG = 40. Find the lead, lead angle, worm gear
diameter, and center distance. Will it self-lock? The input speed is 1100 rpm.

Given: Threads on worm Nw  1


Worm pitch diameter d  2.00 in Gear ratio mG  40
Axial pitch p x  0.25 in Input (worm) speed n w  1100 rpm
Solution: See Mathcad file P1342.
1. Use equation 13.13 to calculate the lead.

L  p x  Nw L  0.250  in

2. Use equation 13.12 to calculate the lead angle.

λ  atan 
L
 λ  2.279  deg
 π d 
3. Calculate the number of teeth on the gear

Ng  mG Nw Ng  40

4. Use equation 13.13 to calculate gear diameter.

p c  Ng
p c  p x d g  d g  3.183  in
π

5. Use equation 13.17 to calculate the center distance.

d  dg
C  C  2.592  in
2

6. If the lead angle per worm tooth is less than 6 deg, the set is self-locking. The lead angle per tooth is

λ
λt  λt  2.279  deg Self-locking
Nw

© 2011 Pearson Education, Inc., Upper Saddle River, NJ. All rights reserved. This publication is protected by Copyright and written permission should be
MACHINE DESIGN - An Integrated Approach, 4th Ed. 13-43-1

PROBLEM 13-43 _____

Statement: A 2-start wormset has d = 2.50 in, p x = 0.30 in, mG = 50. Find the lead, lead angle, worm gear
diameter, and center distance. Will it self-lock? The input speed is 1800 rpm.

Given: Threads on worm Nw  2


Worm pitch diameter d  2.50 in Gear ratio mG  50
Axial pitch p x  0.30 in Input (worm) speed n w  1800 rpm
Solution: See Mathcad file P1343.
1. Use equation 13.13 to calculate the lead.

L  p x  Nw L  0.600  in

2. Use equation 13.12 to calculate the lead angle.

λ  atan 
L
 λ  4.369  deg
 π d 
3. Calculate the number of teeth on the gear

Ng  mG Nw Ng  100

4. Use equation 13.13 to calculate gear diameter.

p c  Ng
p c  p x d g  d g  9.549  in
π

5. Use equation 13.17 to calculate the center distance.

d  dg
C  C  6.025  in
2

6. If the lead angle per worm tooth is less than 6 deg, the set is self-locking. The lead angle per tooth is

λ
λt  λt  2.184  deg Self-locking
Nw

© 2011 Pearson Education, Inc., Upper Saddle River, NJ. All rights reserved. This publication is protected by Copyright and written permission should be
MACHINE DESIGN - An Integrated Approach, 4th Ed. 13-44-1

PROBLEM 13-44 _____

Statement: A 3-start wormset has d = 60 mm, p x = 12 mm, mG = 60. Find the lead, lead angle, worm gear
diameter, and center distance. Will it self-lock? The input speed is 2500 rpm.

Given: Threads on worm Nw  3


Worm pitch diameter d  60 mm Gear ratio mG  60
Axial pitch p x  12 mm Input (worm) speed n w  1800 rpm
Solution: See Mathcad file P1344.
1. Use equation 13.13 to calculate the lead.

L  p x  Nw L  36.000 mm

2. Use equation 13.12 to calculate the lead angle.

λ  atan 
L
 λ  10.812 deg
 π d 
3. Calculate the number of teeth on the gear

Ng  mG Nw Ng  180

4. Use equation 13.13 to calculate gear diameter.

p c  Ng
p c  p x d g  d g  687.55 mm
π

5. Use equation 13.17 to calculate the center distance.

d  dg
C  C  373.77 mm
2

6. If the lead angle per worm tooth is less than 6 deg, the set is self-locking. The lead angle per tooth is

λ
λt  λt  3.604  deg Self-locking
Nw

© 2011 Pearson Education, Inc., Upper Saddle River, NJ. All rights reserved. This publication is protected by Copyright and written permission should be
MACHINE DESIGN - An Integrated Approach, 4th Ed. 13-45-1

PROBLEM 13-45 _____


Statement: Determine the power transmitted and the torques and forces in the mesh for the wormset in Problem
13-42 if it runs at 800 worm rpm.
1
Units: fpm  ft  min
Given: Threads on worm Nw  1 Pressure angle ϕ  20 deg
Worm pitch diameter d w  2.00 in Gear ratio mG  40
Axial pitch p x  0.25 in Input (worm) speed n w  800  rpm
Solution: See Mathcad file P1345.
1. Use equation 13.13 to calculate the lead.

L  p x  Nw L  0.250  in

2. Use equation 13.12 to calculate the lead angle.

λ  atan 
L
 λ  2.279  deg
 π d w 
3. Calculate the number of teeth on the gear

Ng  mG Nw Ng  40

4. Use equation 13.13 to calculate gear diameter.


p c  Ng
p c  p x d g  d g  3.183  in
π
5. Use equation 13.17 to calculate the center distance.

dw  dg
C  C  2.592  in
2
6. Find the maximum recommended face width from equation 13.19.
Fmax  0.67 d w Fmax  1.340  in

7. Find the materials factor Cs from equation 13.24. Since C < 8 in, Cs  1000.

8. Find the ratio correction factor Cm from equations 13.25. Based on mG  40 , the second of the
expressions in that equation set will be used.

2
Cm  0.0107 mG  56 mG  5145 Cm  0.814

9. Find the tangential velocity Vt from equation 13.27.

nw dw
Vt  Vt  419.2  fpm
2  cos( λ)

10. Use this velocity to find the velocity factor Cv from equations 13.26. For this value of Vt, the first of these
equations is appropriate.

Vt
 0.0011
fpm
Cv  0.659  e Cv  0.416
© 2011 Pearson Education, Inc., Upper Saddle River, NJ. All rights reserved. This publication is protected by Copyright and written permission should be
MACHINE DESIGN - An Integrated Approach, 4th Ed. 13-45-2

11. Find the tangential load Wt from equation 13.23.

0.8
 dg  lbf
Wtg  Cs Cm Cv    Fmax Wtg  1144 lbf
 in  in

12. Find the coefficient of friction from the third expression in equation 13.29.
0.450
 Vt 
 0.110  
μ  0.103  e  fpm   0.012 μ  0.031

13. Find the friction force Wf from equation 13.28.

μ  Wtg
Wf  Wf  38.4 lbf
cos( λ)  cos( ϕ)

14. Find the rated output power from equation 13.21.

nw dg
Φ o    Wtg Φ o  0.58 hp
mG 2

15. Find the power lost in the mesh from equation 13.22.
Φ l  Vt Wf Φ l  0.49 hp

16. Find the rated input power from equation 13.20.

Φ  Φ o  Φ l Φ  1.07 hp

17. The efficiency of the gearset is

Φo
e  e  54.2 %
Φ

18. Find the rated output torque from equation 13.31.

dg
Tg  Wtg Tg  1821 in lbf
2

© 2011 Pearson Education, Inc., Upper Saddle River, NJ. All rights reserved. This publication is protected by Copyright and written permission should be
MACHINE DESIGN - An Integrated Approach, 4th Ed. 13-46-1

PROBLEM 13-46 _____


Statement: Determine the power transmitted and the torques and forces in the mesh for the wormset in Problem
13-43 if it runs at 1200 worm rpm.
1
Units: fpm  ft  min
Given: Threads on worm Nw  2 Pressure angle ϕ  20 deg
Worm pitch diameter d w  2.50 in Gear ratio mG  50
Axial pitch p x  0.30 in Input (worm) speed n w  1200 rpm
Solution: See Mathcad file P1346.
1. Use equation 13.13 to calculate the lead.
L  p x  Nw L  0.600  in
2. Use equation 13.12 to calculate the lead angle.

λ  atan 
L
 λ  4.369  deg
 π d w 
3. Calculate the number of teeth on the gear
Ng  mG Nw Ng  100
4. Use equation 13.13 to calculate gear diameter.

p c  Ng
p c  p x d g  d g  9.549  in
π
5. Use equation 13.17 to calculate the center distance.
dw  dg
C  C  6.025  in
2
6. Find the maximum recommended face width from equation 13.19.
Fmax  0.67 d w Fmax  1.675  in
7. Find the materials factor Cs from equation 13.24. Since C < 8 in, Cs  1000.

8. Find the ratio correction factor Cm from equations 13.25. Based on mG  50 , the second of the
expressions in that equation set will be used.

2
Cm  0.0107 mG  56 mG  5145 Cm  0.790

9. Find the tangential velocity Vt from equation 13.27.

nw dw
Vt  Vt  787.7  fpm
2  cos( λ)

10. Use this velocity to find the velocity factor Cv from equations 13.26. For this value of Vt, the second of these
equations is appropriate.

 0.571
 Vt 
Cv  13.31    Cv  0.295
 fpm 

11. Find the tangential load Wt from equation 13.23.

© 2011 Pearson Education, Inc., Upper Saddle River, NJ. All rights reserved. This publication is protected by Copyright and written permission should be
MACHINE DESIGN - An Integrated Approach, 4th Ed. 13-46-2

0.8
 dg  lbf
Wtg  Cs Cm Cv    Fmax Wtg  2375 lbf
 in  in

12. Find the coefficient of friction from the third expression in equation 13.29.

0.450
 
Vt
 0.110  
μ  0.103  e  fpm   0.012 μ  0.023

13. Find the friction force Wf from equation 13.28.

μ  Wtg
Wf  Wf  59.0 lbf
cos( λ)  cos( ϕ)

14. Find the rated output power from equation 13.21.

nw dg
Φ o    Wtg Φ o  4.32 hp
mG 2

15. Find the power lost in the mesh from equation 13.22.
Φ l  Vt Wf Φ l  1.41 hp

16. Find the rated input power from equation 13.20.

Φ  Φ o  Φ l Φ  5.73 hp

17. The efficiency of the gearset is

Φo
e  e  75.4 %
Φ

18. Find the rated output torque from equation 13.31.

dg
Tg  Wtg Tg  11342  in lbf
2

© 2011 Pearson Education, Inc., Upper Saddle River, NJ. All rights reserved. This publication is protected by Copyright and written permission should be
MACHINE DESIGN - An Integrated Approach, 4th Ed. 13-47-1

PROBLEM 13-47 _____

Statement: A 2-start wormset has L = 2.00 in, C = 9.00 in, mG = 20, and the angle between the shafts is 90
deg. Find the pitch diameters of the worm and worm gear, the lead angle, and the axial pitch.

Given: Threads on worm Nw  2 Center distance C  9.00 in


Worm lead L  2.00 in Gear ratio mG  20

Solution: See Mathcad file P1347.

1. Determine the pitch diameter of the worm gear using equation 13.13.

L π d g Nw π d g
= L = π d g =
Nw Ng Ng mG

Solving for d g,

mG L
d g  d g  12.732 in
π

2. Determine the pitch diameter of the worm using equation 13.17 solved for d w.

d w  2  C  d g d w  5.268  in

3. Calculate the lead angle using equation 13.12.

λ  atan 
L
 λ  6.891  deg
 π d w 

4. Calculate the axial pitch using equation 13.13.

L
p x  p x  1.000  in
Nw

© 2011 Pearson Education, Inc., Upper Saddle River, NJ. All rights reserved. This publication is protected by Copyright and written permission should be
MACHINE DESIGN - An Integrated Approach, 4th Ed. 13-48-1

PROBLEM 13-48 _____

Statement: A 5-start wormset has  = 20 deg, C = 2.75 in, Ng = 33, and the angle between the shafts is 90
deg. Find the pitch diameters of the worm and worm gear, the lead, and the axial pitch.

Given: Threads on worm Nw  5 Center distance C  2.75 in


Worm lead angle λ  20 deg Teeth on worm gear Ng  33

Solution: See Mathcad file P1348.

1. Determine the pitch diameter of the worm gear using equations 13.12, 13.13, and 13.17.

L
tan ( λ) = L = π d w tan ( λ)
π d w

L π d g Nw
= L = π d g
Nw Ng Ng

Eliminating L and solving for d w,

Nw
dw =  dg
Ng tan ( λ)

Substitute into equation 13.17 and solve for d g,

Nw 2 C
dg = 2 C   dg d g  d g  3.883  in
Ng tan ( λ) Nw
1
Ng tan ( λ)

2. Determine the pitch diameter of the worm using equation 13.17 solved for d w.

d w  2  C  d g d w  1.617  in

3. Calculate the lead using equation 13.12.

L  π d w tan( λ) L  1.848  in

4. Calculate the axial pitch using equation 13.13.

L
p x  p x  0.370  in
Nw

© 2011 Pearson Education, Inc., Upper Saddle River, NJ. All rights reserved. This publication is protected by Copyright and written permission should be
MACHINE DESIGN - An Integrated Approach, 4th Ed. 13-49-1

PROBLEM 13-49
Statement: Size the bevel gears in problem 13-40 for a bending factor of safety of at least 2.5 assuming a
5-year, 2-shift life, steady torque, quality index of 8, and an AISI 4140 steel pinion and gear.
Units: yr  2080 hr
Given: Factor of safety Nfb  2.5 Number of gear teeth Ng  120
Power to be transmitted (hp) H  3  kW Reliability R  0.99
Rotational speed of pinion (rpm) n p  900  rpm AGMA Quality level Qv  8
Number of pinion teeth Np  20 Life (years) Life  5  yr

Solution: See Mathcad file P1349.


Pinion
1. Determine the bending geometry factor, J (Figure 13-5) Jp  0.300
2. Write the equations for pitch diameter, pitchline velocity, and transmitted torque in terms of the unknown
diametral pitch, p d. Note that, in Mathcad, unit conversion factors are not included.

Np Ng
Pitch diameter of pinion (in) d p p d   d g p d  
pd pd

d p p d   n p
pitchline velocity (fpm) Vt p d  
2

H
Transmitted torque (in-lbf) Tp  Tp  281.728  in lbf
np

3. Set the application, size, and type factors. Ka  1 Ks  1 Kx  1

B  0.25  12  Qv
0.6667
4. Write the equation for the dynamic load factor, Kv B  0.63

A  50  56 ( 1  B) A  70.721
B
Kv p d    A 
 
 A  Vt  p d  
min

 ft 

5. Tentatively choose the mounting factor, Km (Assume 0 < F < 2 in) Km  1.6

2  Tp pd Ka Km Ks
6. The bending stress equation for the pinion is σbp p d F    
d p p d  F  Jp Kv p d   Kx

7. Determine the endurance strength of the pinion.


np 9
Cycle life shifts  2 N  Life shifts N  1.123  10
2 π
 0.0323
Life factor KL  1.6831 N KL  0.859
Reliability KR  0.7  0.15 log( 1  R) KR  1
Temperature factor KT  1
Material bending strength (psi)
S atp  40000  psi
AISI 4140 Nitrided steel
© 2011 Pearson Education, Inc., Upper Saddle River, NJ. All rights reserved. This publication is protected by Copyright and written permission should be
MACHINE DESIGN - An Integrated Approach, 4th Ed. 13-49-2

S atp KL
Endurance strength S fbp  S fbp  34343  psi
KT  KR

8. Write the design equations using the face-width to pitch-cone length ratio given in the text, and the bending
stress equation, solved for the unknown face width.

Np
Upper limit, F = L/3 FLover3 p d  
  Np  
6  p d  sin atan 
  Ng  
S fb
Safety factor Nfb =
σb

2  Tp p d  Ka Km Ks Nfb


Face width F  p d   
d p p d   Jp Kv p d   Kx S fbp

8 8.5 18
9. Plot F(P) vs. p d over the p d   
range in in in

10. From the graph, choose a standard value of


p d from Table 12-2. 4

1
p d  10 in
  3
F pd
11. The calculated value of F is
in
F  p d   1.295  in
FLover3 pd  2
12. Round this to the decimal equivalent of a in
common fractional value. 1
F  1.375  in

13. Then, the parameters that depend on p d and F are: 0


8 10 12 14 16 18
pd  in
d p p d   2.000  in Kv p d   0.845

Vt p d   471 
ft FIGURE 13-49A
min Graph of Face Width and Limits for the Pinion in
Problem 13-49
σbp p d F   12935  psi

The assumption made in step 5 is correct so no further iteration is required.

GEAR
14. Determine the bending geometry factor, J (Figure 13-5) Jg  0.245

2  Tp pd Ka Km Ks
15. The bending stress equation for the gear is σbg p d F    
d g p d  F  Jg Kv p d   Kx

16. Write the design equations using the face-width to pitch-cone length ratio given in the text, and the bending
stress equation, solved for the unknown face width.
© 2011 Pearson Education, Inc., Upper Saddle River, NJ. All rights reserved. This publication is protected by Copyright and written permission should be
MACHINE DESIGN - An Integrated Approach, 4th Ed. 13-49-3

Np
F = L/3 FLover3 p d  
  Np  
6  p d  sin atan 
  Ng  
S fb
Safety factor Nfb =
σb

2  Tp p d  Ka Km Ks Nfb


Face width F  p d   
d g p d   Jg Kv p d   Kx S fbp

14 14.5 24
17. Plot F(P) vs. p d over the range p d   
in in in

18. From the graph, choose a standard value of 2


p d from Table 12-2.

1
p d  20 in
  1.5
F pd
19. The calculated value of F is in
F  p d   1.01 in FLover3 pd   1

20. Round this to the decimal equivalent of a in


common fractional value. 0.5

F  1.000  in
0
21. Then, the parameters that depend on p d and F are: 14 16 18 20 22 24
pd  in
d g p d   6.000  in Kv p d   0.884
FIGURE 13-49B
Vt p d   236  σbg p d F   13881  psi
ft Graph of Face Width and Limits for the Gear in
min Problem 13-49

22. The gear dimensions are smaller (larger diametral pitch means smaller teeth) than for the pinion. This means th
we will accept the pinion requirements for the gear, thus, for the set
1
Diametral pitch p d  10 in
Face width F  1.375  in
 Np 
Pitch cone angle αp  atan  αp  9.462  deg
 Ng 
d p p d  L
Pitch-cone length L  L  6.083  in  2.028  in
2  sin αp 3

23. Determine the realized factor of safety for the pinion using the above values for F and p d.
S fbp
Pinion factor of safety Nfbp  Nfbp  2.7
σbp p d F 

24. Check the factor of safety on the gear:


S fbp
Gear factor of safety Nfbg  Nfbg  13.0
σbg p d F 

© 2011 Pearson Education, Inc., Upper Saddle River, NJ. All rights reserved. This publication is protected by Copyright and written permission should be
MACHINE DESIGN - An Integrated Approach, 4th Ed. 13-50-1

PROBLEM 13-50
Statement: Size the bevel gears in problem 13-40 for a minimum safety factor of 1.8 for any mode of failure
of pinion or gear assuming a 5-year, 2-shift life, steady torque, quality index of 8, and an AISI
4140 steel pinion and gear.

Units: yr  2080 hr
Given: Factor of safety Nfc  1.8 Number of gear teeth Ng  120
Power to be transmitted (hp) H  3  kW Reliability R  0.99
Rotational speed of pinion (rpm) n p  900  rpm AGMA Quality level Qv  8
Number of pinion teeth Np  20 Life (years) Life  5  yr

Assumptions: 1. If both pinion and gear are the same material, it will only be necessary to determine the
pinion size as it will be governing for the set.
2. If the gears are not surface hardened, it will only be necessary to design to the surface
requirement as it will be governing for both bending and surface stresses.

Solution: See Mathcad file P1350.


1. Determine the surface geometry factor, I, from Figure 13-6. I  0.107
2. Write the equations for pitch diameter, pitchline velocity, and transmitted load in terms of the unknown
diametral pitch, p d. Note that, in Mathcad, unit conversion factors are not included.
Np Ng
Pitch diameter of pinion (in) d p p d   d g p d  
pd pd

Np
pitchline velocity (fpm) Vtp p d    np
2 pd

H
Transmitted load (lbf) Tp  Tp  281.728  in lbf
np

3. Set the application factor, Ca Ca  1 Cs  1 Cf  1

B  0.25  12  Qv
0.6667
4. Write the equation for the dynamic load factor, Cv B  0.63

A  50  56 ( 1  B) A  70.721
B
Cvp p d    A 
 
 A  Vtp pd  
min

 ft 

5. Tentatively choose the mounting factor, Cmd Cmd  3.0 Cxc  1


0.5
6. Choose an elastic coefficient from Table 12-18 (steel Cp  2100 psi Cb  0.634
on steel) and a stress adjustment constant.
7. Determine the endurance strength of the pinion.
np 9
Cycle life shifts  2 N  Life shifts N  1.123  10
2 π
 0.056
Life factor CL  2.466  N CL  0.768
Reliability CR  0.7  0.15 log( 1  R) CR  1

© 2011 Pearson Education, Inc., Upper Saddle River, NJ. All rights reserved. This publication is protected by Copyright and written permission should be
MACHINE DESIGN - An Integrated Approach, 4th Ed. 13-50-2
Hardness CH  1

Temperature factor CT  1
Material surface strength (psi)
S acp  167500 psi
AISI 4140 Nitrided steel
S acp CL
Endurance strength S fcp  S fcp  128583 psi
CT  CR
8. Write the equation for the design pinion torque.

I  Cvp p d   Sacp d p pd  0.774  CH 


2
TD p d F   
F
  
2 Cs Cmd  Cf  Ca Cxc  Cp Cb CT  CR 

9. Define the exponent z. z p d F   return 0.667 if Tp  TD p d F 


1 otherwise

10. The surface stress equation for the pinion is


z pd F 
 2 TD pd F    Tp  Ca Cmd
σcp p d F   Cp Cb      Cs Cf  Cxc
 F  d p pd  2 I   TD p d F   Cvp p d 
 
11. Write the design equations using the face-width to pitch-cone length ratio given in the text, and the bending
stress equation, solved for the unknown face width.
2
Np  Sfc 
Upper limit, F = L/3 FLover3 p d   Nfc =  
  Np    σc 
6  p d  sin atan 
Face width
  Ng  


z pd F 
 2  TD p d F    Tp 
2
 Cp Cb  Ca Cmd
F' p d F          Cs Cf  Cxc Nfc
 S fcp   d p pd  2 I   TD pd F   Cvp p d 
 
8 8.5 18
12. Plot F(P) vs. p d over the range p d    Guess value for F F  1.21 in
in in in
4
13. From the graph, choose a standard value of p d from
Table 12-2.

 
1 3
p d  10 in F' pd F
in
14. The calculated value of F is
FLover3 pd  2
F' p d F   1.217  in
in
15. Round this to the decimal equivalent of a common 1
fractional value.

F  1.250  in 0
8 10 12 14 16 18
16. Then, the parameters that depend on p d and F are:
pd  in

© 2011 Pearson Education, Inc., Upper Saddle River, NJ. All rights reserved. This publication is protected by Copyright and written permission should be
MACHINE DESIGN - An Integrated Approach, 4th Ed. 13-50-3
d p p d   2.000  in d g p d   12.000 in
FIGURE 13-50
Graph of Face Width and Limits for for Problem 13-50

Vtp p d   471  Cvp p d   0.845 σcp p d F   95063  psi TD p d F   714  in lbf
ft
min

17. When both pinion and gear are of the same material, the stress in the gear will always be lower than in the
pinion. Therefore, the face width and diametral pitch found for the pinion should also be used for the gear.

1
Diametral pitch p d  10 in

Face width F  1.250  in

 Np 
Pitch cone angle αp  atan  αp  9.462  deg
 Ng 
d p p d  L
Pitch-cone length L  L  6.083  in  2.028  in
2  sin αp 3

18. Determine the realized factor of safety for the pinion using the above values for F and p d.

2
 Sfcp 
Pinion factor of safety Nfcp   σ p F  Nfcp  1.8
 cp d  

© 2011 Pearson Education, Inc., Upper Saddle River, NJ. All rights reserved. This publication is protected by Copyright and written permission should be
MACHINE DESIGN - An Integrated Approach, 4th Ed. 13-51-1

PROBLEM 13-51
Statement: Size the spiral gears in problem 13-41 for a bending factor of safety of at least 2.0 assuming a
7-year, 3-shift life, steady torque, quality index of 8, an AISI 4340 steel pinion and gear.

Units: yr  2080 hr
Given: Factor of safety Nfb  2.0 Number of gear teeth Ng  168
Power to be transmitted (kW) H  3  kW Reliability R  0.99
Rotational speed of pinion (rpm) n p  1100 rpm AGMA Quality level Qv  8
Number of pinion teeth Np  21 Life (years) Life  7  yr

Solution: See Mathcad file P1351.

Pinion
1. Determine the bending geometry factor, J (Figure 13-8) Jp  0.360 (assumed)

2. Write the equations for pitch diameter, pitchline velocity, and transmitted torque in terms of the
unknown diametral pitch, p d. Note that, in Mathcad, unit conversion factors are not included.
Np Ng
Pitch diameter of pinion (in) d p p d   d g p d  
pd pd

d p p d   n p
pitchline velocity (fpm) Vt p d  
2

H
Transmitted torque (in-lbf) Tp  Tp  230.505  in lbf
np

3. Set the application, size, and type factors. Ka  1 Ks  1 Kx  1.15

B  0.25  12  Qv
0.6667
4. Write the equation for the dynamic load factor, Kv B  0.63

A  50  56 ( 1  B) A  70.721
B
Kv p d    A 
 
 A  Vt  p d  
min

 ft 

5. Tentatively choose the mounting factor, Km (Assume 0 < F < 2 in) Km  1.6

2  Tp pd Ka Km Ks
6. The bending stress equation for the pinion is σbp p d F    
d p p d  F  Jp Kv p d   Kx

7. Determine the endurance strength of the pinion.


np 9
Cycle life shifts  3 N  Life shifts N  2.883  10
2 π
 0.0323
Life factor KL  1.6831 N KL  0.833
Reliability KR  0.7  0.15 log( 1  R) KR  1
Temperature factor KT  1
Material bending strength (psi)
S atp  42000  psi
AISI 4340 Nitrided steel
© 2011 Pearson Education, Inc., Upper Saddle River, NJ. All rights reserved. This publication is protected by Copyright and written permission should be
MACHINE DESIGN - An Integrated Approach, 4th Ed. 13-51-2

S atp KL
Endurance strength S fbp  S fbp  34979  psi
KT  KR

8. Write the design equations using the face-width to pitch-cone length ratio given in the text, and the bending
stress equation, solved for the unknown face width.

Np
Upper limit, F = L/3 FLover3 p d  
  Np  
6  p d  sin atan 
  Ng  
S fb
Safety factor Nfb =
σb

2  Tp p d  Ka Km Ks Nfb


Face width F  p d   
d p p d   Jp Kv p d   Kx S fbp

2 2.5 20
9. Plot F(P) vs. p d over the p d   
range in in in
4

10. From the graph, choose a standard value of


p d from Table 12-2.
  3
F pd
1
p d  16 in in

11. The calculated value of F is FLover3 pd  2


F  p d   1.447  in in
1
12. Round this to the decimal equivalent of a
common fractional value.
0
F  1.500  in 2 4 6 8 10 12 14 16 18 20
pd  in
13. Then, the parameters that depend on p d and F are:
FIGURE 13-51A
d p p d   1.313  in Kv p d   0.858
Graph of Face Width and Limits for the Pinion in
Problem 13-51

Vt p d   378 
ft
min

σbp p d F   16874  psi

The assumption made in step 5 is correct so no further iteration is required.

GEAR
14. Determine the bending geometry factor, J (Figure 13-8) Jg  0.360

2  Tp pd Ka Km Ks
15. The bending stress equation for the gear is σbg p d F    
d g p d  F  Jg Kv p d   Kx

16. Write the design equations using the face-width to pitch-cone length ratio given in the text, and the bending
stress equation, solved for the unknown face width.

© 2011 Pearson Education, Inc., Upper Saddle River, NJ. All rights reserved. This publication is protected by Copyright and written permission should be
MACHINE DESIGN - An Integrated Approach, 4th Ed. 13-51-3

Np
F = L/3 FLover3 p d  
  Np  
6  p d  sin atan 
  Ng  
S fb
Safety factor Nfb =
σb

2  Tp p d  Ka Km Ks Nfb


Face width F  p d   
d g p d   Jg Kv p d   Kx S fbp

10 11.5 50
17. Plot F(P) vs. p d over the range p d   
in in in
2

18. From the graph, choose a standard value of


p d from Table 12-2.
  1.5
F pd
1
p d  32 in in

  1
19. The calculated value of F is FLover3 pd

F  p d   0.694  in in
0.5
20. Round this to the decimal equivalent of a
common fractional value.
0
F  0.750  in 10 15 20 25 30 35 40 45 50
pd  in
21. Then, the parameters that depend on p d and F are:
FIGURE 13-51B
d g p d   5.250  in Kv p d   0.894
Graph of Face Width and Limits for the Gear in
Problem 13-51

Vt p d   189  σbg p d F   16194  psi


ft
min

22. The gear dimensions are smaller (larger diametral pitch means smaller teeth) than for the pinion. This means th
we will accept the pinion requirements for the gear, thus, for the set
1
Diametral pitch p d  16 in
Face width F  1.500  in
 Np 
Pitch cone angle αp  atan  αp  7.125  deg
 Ng 
d p p d  L
Pitch-cone length L  L  5.291  in  1.764  in
2  sin αp 3

23. Determine the realized factor of safety for the pinion using the above values for F and p d.
S fbp
Pinion factor of safety Nfbp  Nfbp  2.1
σbp p d F 

25. Check the factor of safety on the gear:


S fbp
Gear factor of safety Nfbg  Nfbg  16.6
σbg p d F 
© 2011 Pearson Education, Inc., Upper Saddle River, NJ. All rights reserved. This publication is protected by Copyright and written permission should be
MACHINE DESIGN - An Integrated Approach, 4th Ed. 14-1-1

PROBLEM 14-1
Statement: A linear spring is to give 200 N at its maximum deflection of 150 mm and 40 N at its minimum
deflection of 50 mm. What is the spring rate?

Given: Working force Fwork  200  N Initial force Finit  40 N


Working deflection ywork  150  mm Initial deflection yinit  50 mm

Solution: See Mathcad file P1401.

1. Spring rate is the slope of the load vs. deflection function. Thus, for a linear spring

Fwork  Finit N
Spring rate k  k  1.60
ywork  yinit mm

© 2011 Pearson Education, Inc., Upper Saddle River, NJ. All rights reserved. This publication is protected by Copyright and written permission should be
MACHINE DESIGN - An Integrated Approach, 4th Ed. 14-2-1

PROBLEM 14-2
Statement: Find the ultimate tensile strength, the ultimate shear strength, and the torsional yield strength
of a 1.8-mm-dia, A229 oil-tempered steel wire.

Given: Wire diameter d  1.8 mm

Solution: See Mathcad file P1402.


1. From Table 14-4, for A229
A  1831.2 MPa b  0.1833
2. Using equation 14.3,
b
S ut  A   d
Tensile strength  S ut  1644 MPa
 
mm

3. From Table 14-6, with a factor of 0.50,


Maximum torsional
yield strength S ys  0.50 S ut S ys  822  MPa

4. Using equation 14.4,

Ultimate shear
strength S us  0.67 S ut S us  1102 MPa

© 2011 Pearson Education, Inc., Upper Saddle River, NJ. All rights reserved. This publication is protected by Copyright and written permission should be
MACHINE DESIGN - An Integrated Approach, 4th Ed. 14-3-1

PROBLEM 14-3
Statement: Find the torsional yield and ultimate shear strength of an 0.105-in-dia, unset A230 wire to be
used in a helical compression spring.

Given: Wire diameter d  0.105  in d  2.667  mm

Solution: See Mathcad file P1403.


1. ASTM A230 is not listed in Table 14-4, but it is shown in Figure 14-3. At the wire diameter given, it has a
tensile strength that is approximately the same as that for A229. From Table 14-4, for A229

A  146.78 ksi b  0.1833


2. Using equation 14.3,
b
S ut  A   
d
Tensile strength  S ut  222  ksi
 in 
3. From Table 14-6, with a factor of 0.50,
Maximum torsional
yield strength S ys  0.50 S ut S ys  111  ksi

4. Using equation 14.4,

Ultimate shear
strength S us  0.67 S ut S us  149  ksi

© 2011 Pearson Education, Inc., Upper Saddle River, NJ. All rights reserved. This publication is protected by Copyright and written permission should be
MACHINE DESIGN - An Integrated Approach, 4th Ed. 14-4-1

PROBLEM 14-4
Statement: What is the torsional fatigue strength of the wire in Problem 14-3 at N = 5E6 cycles?

Given: Wire diameter d  0.105  in d  2.667  mm


6
Cycle life N  5  10

Solution: See Mathcad file P1404.


1. ASTM A230 is not listed in Table 14-4, but it is shown in Figure 14-3. At the wire diameter given, it has a
tensile strength that is approximately the same as that for A229. From Table 14-4, for A229

A  146.78 ksi b  0.1833


2. Using equation 14.3,
b
S ut  A   
d
Tensile strength  S ut  222  ksi
 in 
3. The required life falls between the values given in Table 14-7 for 1E6 and 1E7 cycles. We will use an
exponential fit to the data for interpolation. Using equation 6.10

 6
b
At 1E6 cycles 0.40 S ut = a  10

= a   10 
b
7
At 1E7 cycles 0.38 S ut

4. Solving both equations for a, equating the results and solving for b

0.40 S ut 0.38 S ut
=
106 107
b b

b
 107 
b  log( 10) = log
  = 0.38 0.38 

 106  0.40  0.40 
 

b  log 
0.38  1
 b  0.022
 0.40  log( 10)
0.40 S ut
a  a  120.726  ksi
106
b

5. Interpolating,

Fatigue strength b
at 5E6 cycles S fs  a  N S fs  85.6 ksi

© 2011 Pearson Education, Inc., Upper Saddle River, NJ. All rights reserved. This publication is protected by Copyright and written permission should be
MACHINE DESIGN - An Integrated Approach, 4th Ed. 14-5-1
PROBLEM 14-5
Statement: Draw the Modified Goodman diagram for the wire of Problem 14-3.
Given: Wire diameter d  0.105  in d  2.667  mm
6
Cycle life N  5  10

Solution: See Mathcad file P1405.


1. ASTM A230 is not listed in Table 14-4, but it is shown in Figure 14-3. At the wire diameter given, it has a
tensile strength that is approximately the same as that for A229. From Table 14-4, for A229

A  146.78 ksi b  0.1833


2. Using equation 14.3,
b
S ut  A   
d
Tensile strength  S ut  221.9  ksi
 in 
3. From Table 14-6, with a factor of 0.50,
Maximum torsional
yield strength S ys  0.50 S ut S ys  110.9  ksi

4. Using equation 14.4,


Ultimate shear
strength S us  0.67 S ut S us  148.6  ksi

5. Using Table 14-7 and a cycle life of 1E6, for unpeened wire
Torsional fatigue
strength (R=0) S fw  .40 S ut S fw  88.7 ksi

6. From equation (d) in Example 14-2,

S fw S us
S fs  0.5 S fs  63.3 ksi
S us  0.5 S fw
S fs
7. Now, the equation for the Goodman line is τa τm    τm  S fs
S us

8. Plotting this over the range τm  0  ksi 10 ksi  150  ksi

100
Alternating shear stress

80

60

40

20

0
0 25 50 75 100 125 150

Mean shear stress

© 2011 Pearson Education, Inc., Upper Saddle River, NJ. All rights reserved. This publication is protected by Copyright and written permission should be
MACHINE DESIGN - An Integrated Approach, 4th Ed. 14-6-1

PROBLEM 14-6
Statement: What are the spring rate and spring index of a squared and ground compression spring with the
data given below?
Given: Wire diameter d  1  mm Total coils Nt  12
Mean coil dia D  10 mm

Assumptions: The spring wire is steel so that G  80.8 GPa.


Solution: See Mathcad file P1406.
1. From Figure 14-9, the number of active coils is Na  Nt  2 Na  10

2. Using equation 14.7,


4
d G N
Spring rate k  k  1.01
3 mm
8  D  Na

3. Using equation 14.5,


D
Spring Index C  C  10
d

© 2011 Pearson Education, Inc., Upper Saddle River, NJ. All rights reserved. This publication is protected by Copyright and written permission should be
MACHINE DESIGN - An Integrated Approach, 4th Ed. 14-7-1

PROBLEM 14-7
Statement: Find the natural frequency of the spring in Problem 14-6.
Given: Wire diameter d  1  mm Total coils Nt  12
Mean coil dia D  10 mm
3 3
Assumptions: The spring wire is steel so that G  80.8 GPa and γ  0.28 lbf  in , γ  76005  N  m
Solution: See Mathcad file P1407.
1. From Figure 14-9, the number of active coils is Na  Nt  2 Na  10

2. Using equation 14.7,


4
d G N
Spring rate k  k  1.01
3 mm
8  D  Na

3. Using equation 14.5,


D
Spring Index C  C  10
d

4. Calculate the natural frequency using equation 14.11c.

2 d G g
Natural frequency fn    fn  363.4  Hz
π N a 2 32 γ
D

© 2011 Pearson Education, Inc., Upper Saddle River, NJ. All rights reserved. This publication is protected by Copyright and written permission should be
MACHINE DESIGN - An Integrated Approach, 4th Ed. 14-8-1
PROBLEM 14-8
Statement: A paper mill processes rolls of paper having a density of 984 kg/m3. The paper roll is 1.50 m
outside dia (OD) by 0.22 m inside dia (ID) by 3.23 m long and is on a simply supported, hollow,
steel shaft with 22-cm OD x 20-cm ID and as long as the paper roll. Find the spring rate of the
shaft and the fundamental natural frequency of the shaft-roll assembly.

kg
Given: Paper density ρ  984  Roll dimensions:
3
m
Shaft outside dia od  220  mm Outside diameter OD  1.50 m
Shaft inside dia id  200  mm Inside diameter ID  0.22 m
Young's modulus E  207  GPa Length L  3.23 m

Assumptions: The shaft (beam) supporting the paper roll is simply-supported at the ends and is the same
length as the paper roll. The paper acts as a distributed load over the length of the shaft.
Solution: See Mathcad file P1408.
1. The mass of the paper roll is equal to its volume times the paper density.

mroll 
π
4
 2
 OD  ID  L ρ
2  mroll  5495.74  kg

2. The area moment of inertia of the shaft is I 


π
 4
 od  id
4 7
I  3.645  10  mm
4
64
3. The spring rate (stiffness) of the shaft can be found from the deflection equation in Figure B-2(b) in Appendix B
When the distributed load covers the entire span, the maximum deflection is
4 3
5  w L 5 W  L W 384  E I
ymax = = k= =
384  E I 384  E I y 3
5 L

384  E I 4 N
Solving for k, k  k  1.720  10 
3 mm
5 L
4. Use equations 3.4 to find the natural frequency of the system.

k
ωn  rad
mroll ωn  55.9
sec

ωn
fn 
2 π fn  8.90 Hz

© 2011 Pearson Education, Inc., Upper Saddle River, NJ. All rights reserved. This publication is protected by Copyright and written permission should be
MACHINE DESIGN - An Integrated Approach, 4th Ed. 14-9-1

PROBLEM 14-9
Statement: Determine the minimum allowable bending radius for an 50 HRC strip steel spring of 1-mm
thickness.

Given: Thickness t  1  mm

Solution: See Mathcad file P1409.

1. From Table 14-5, the bend factor for spring steel with a 50 HRC is bf  5

2. Use the bend factor definition given in the text to calculate the minimum bend radius.

bf  t
rmin  rmin  2.5 mm
2

© 2011 Pearson Education, Inc., Upper Saddle River, NJ. All rights reserved. This publication is protected by Copyright and written permission should be
MACHINE DESIGN - An Integrated Approach, 4th Ed. 14-10-1
PROBLEM 14-10
Statement: An over-hung diving board is shown in Figure P14-1a. A 100-kg person is standing at the free end
Assume cross-section dimensions of 305 mm x 32 mm and a material E = 10.3 GPa. What is the
spring rate and fundamental natural frequency of the diver-board combination?
2000 = L

Given: Weight of person W  100  kgf R1 P


Board dimensions:
Distance to support a  0.7 m
Length of board L  2  m
R2
Cross-section w  305  mm
t  32 mm 700 = a
Young's modulus E  10.3 GPa

Assumptions: The weight of the board is negligible


FIGURE 14-10
compared to the applied load and so can
be ignored. Free Body Diagram for Problem 14-10

Solution: See Figure 14-10 and Mathcad file P1410.


3
w t 5 4
1. The area moment of inertia of the board is I  I  8.329  10  mm
12
2. The spring rate (stiffness) of the board can be found from the deflection equation in Figure B-3(a) in Appendix
B. When the load is at the end of the beam, the maximum deflection is

F L 2 F 3  E I
ymax =  ( L  a) k= =
3  E I y 2
L ( L  a )

3  E I N
Solving for k, k  k  7.614 
2 mm
L ( L  a )

3. Use equations 3.4 to find the natural frequency of the system.

k g
ωn  rad
W ωn  8.73
sec

ωn
fn 
2 π fn  1.39 Hz

© 2011 Pearson Education, Inc., Upper Saddle River, NJ. All rights reserved. This publication is protected by Copyright and written permission should be
MACHINE DESIGN - An Integrated Approach, 4th Ed. 14-11-1
PROBLEM 14-11
Statement: Design a helical compression spring for a static load of 45 lb at a deflection of 1.25 in with a
safety factor of 2.5. Use C = 7.5. Specify all parameters necessary to manufacture the
spring.

6
Given: Working force Fwork  45 lbf Shear modulus G  11.5 10  psi
Working deflection ywork  1.25 in Safety factor Ns  2.5
Spring index C  7.5
Design choices:
Clash allowance α  0.15 Set removed Km  0.65
ASTM A228 wire A  184.649  ksi
b  0.1625

Solution: See Mathcad file P1411.

Fwork lbf
1. Determine the desired spring rate. k  k  36
ywork in

2. Use the design equation from Example 14-3A (Mathcad Supplement) to determine the wire diameter.
1
2 b
 8 Ns ( C  0.5)  Fwork  ( 1  α)
d   in
 π Km A  in
2 
 
Wire diameter d  0.125  in Let d  0.125  in

3. Calculate the mean coil diameter and number of active coils.


Mean coil dia D  C d D  0.938  in
4
Number of active d G
coils Na  Na  11.831
3
8 D  k Na  11.75

Note that we round it to the nearest 1/4 coil as the manufacturing tolerance cannot achieve better than that
accuracy. We must now calculate the actual (corrected) spring rate:
4
Corrected spring d G lbf
rate k  k  36.249
3 in
8  D  Na
4. Assume squared and ground ends making the total number of coils, from Figure 14-9:

Total coils Nt  Na  2 Nt  13.75

5. The shut height can now be determined.


Shut height Ls  d  Nt Ls  1.719  in
6. The free length (see Figure 14-8) can now be found from
Deflection
to shut height yshut  ywork  α ywork yshut  1.438  in

Free length Lf  Ls  yshut Lf  3.156  in

7. To check for buckling, two ratios need to be calculated, Lf/D and y max/Lf.
© 2011 Pearson Education, Inc., Upper Saddle River, NJ. All rights reserved. This publication is protected by Copyright and written permission should be
MACHINE DESIGN - An Integrated Approach, 4th Ed. 14-11-2
Lf
Slenderness ratio sr  sr  3.367
D
ywork
Deflection ratio y'  y'  0.396
Lf

Take these two values to Figure 14-14 and find that their coordinates are safely within the zones that are stable
against buckling for either end-condition case.
8. The inside and outside coil diameters are
Inside coil dia Di  D  d Di  0.813  in
Outside coil dia Do  D  d Do  1.063  in
9. The smallest hole and largest pin that should be used with this spring are
Smallest hole holemin  Do  0.05 D holemin  1.11 in

Largest pin pin max  Di  0.05 D pin max  0.77 in


10. The total weight of the spring is
3
Weight density ρ  0.28 lbf  in

2 2
π  d  D Nt ρ
Weight Wt  Wt  0.14 lbf
4

11. We now have a complete design specification for this A228 wire spring:

Wire diameter d  0.125  in


Outside diameter Do  1.063  in
Total coils Nt  13.75 ends squared and ground
Free length Lf  3.156  in

© 2011 Pearson Education, Inc., Upper Saddle River, NJ. All rights reserved. This publication is protected by Copyright and written permission should be
MACHINE DESIGN - An Integrated Approach, 4th Ed. 14-12-1
PROBLEM 14-12
Statement: Repeat Problem 14-10 using the cantilevered diving board design in Figure P14-1b.

2000

1300 = L
Given: Weight at free end P  100  kgf
Board dimensions: P
Length of board L  1.3 m
Cross-section w  305  mm
t  32 mm M1
R1
Young's modulus E  10.3 GPa
700
Assumptions: The weight of the board is negligible
compared to the applied load and so can FIGURE 14-12
be ignored. Free Body Diagram for Problem 14-12

Solution: See Figure 14-12 and Mathcad file P1412.


3
w t 5 4
1. The area moment of inertia of the board is I  I  8.329  10  mm
12

2. The spring rate (stiffness) of the board can be found from the deflection equation in Figure B-1(a) in Appendix
B. When the load is at the end of the beam, the maximum deflection is
3
F L F 3  E I
ymax = k= =
3  E I y 3
L

3  E I N
Solving for k, k  k  11.71 
3 mm
L

3. Use equations 3.4 to find the natural frequency of the system.

k g rad
ωn  ωn  10.82 
P sec

ωn
fn  fn  1.72 Hz
2 π

© 2011 Pearson Education, Inc., Upper Saddle River, NJ. All rights reserved. This publication is protected by Copyright and written permission should be
MACHINE DESIGN - An Integrated Approach, 4th Ed. 14-13-1
PROBLEM 14-13
Statement: For the data given below, find Na, D, Lf, Lshut, yinitial, and the minimum hole diameter for the
spring. Infinite life is desired with a safety factor of 1.4. Choose an acceptable spring index.

Setting will be done.


Given: Minimum force Fmin  50 lbf Chrome-vanadium wire properties:
Maximum force Fmax  250  lbf Diameter d  0.312  in
Working deflection ∆y  0.75 in Unpeened
Clash allowance α  0.15 Set after winding
6
Shear modulus G  11.5 10  psi Squared ends

Solution: See Mathcad file P1413.


1. Find the the mean and alternating forces from equation 14.15a:

Fmax  Fmin
Alternating force Fa  Fa  100.0  lbf
2
Fmax  Fmin
Mean force Fm  Fm  150.0  lbf
2

2. Assume a spring index (found by trial and error to get the correct safety factor) and calculate the mean coil
diameter D from equation 14.5.
Spring index C  4.4
Mean coil diameter D  C d D  1.373  in

3. Find the direct shear factor Ks and use it to calculate the shear stress i at the initial deflection (lowest defined
force), and the mean stress m.

0.5
Direct shear factor Ks  1  Ks  1.114
C
8  Fmin D
Stress at Fmin τi  Ks τi  6.4 ksi
3
π d
8  Fm D
Stress at Fm τm  Ks τm  19.2 ksi
3
π d
4. Find the Wahl factor Kw and use it to calculate the alternating shear stress a in the coil.

4 C  1 0.615
Wahl factor Kw   Kw  1.36
4 C  4 C
8  F a D
Alternating stress τa  Kw τa  15.7 ksi
3
π d

5. Find the ultimate tensile strength of this wire material from equation 14.3 and Table 14-4 and use it to find the
ultimate shear strength from equation 14.4 and the torsional yield strength from Table 14-6, assuming that the
set has been removed and using the low end of the recommended range.

From Table 14-4, for A228 music wire A  173.128  ksi b  0.1453
b
S ut  A   
Ultimate tensile d
strength  S ut  205.1  ksi
 
in
© 2011 Pearson Education, Inc., Upper Saddle River, NJ. All rights reserved. This publication is protected by Copyright and written permission should be
MACHINE DESIGN - An Integrated Approach, 4th Ed. 14-13-2

Shear yield
strength S ys  0.65 S ut S ys  133.3  ksi

Ultimate shear
strength S us  0.67 S ut S us  137.4  ksi

6. Find the wire endurance limit for unpeened springs in repeated loading from equation 14.12 and convert it to fu
reversed endurance strength with equation 14.16c.
Wire endurance limit S ew  45 ksi

S ew S us
Fully reversed S es  0.5 S es  26.91  ksi
endurance limit S us  0.5 S ew

7. The safety factor is calculated from equation 14.16b.

Fatigue factor S es  S us  τi


Nfs  Nfs  1.41
of safety S es  τm  τi  S us τa

8. The spring rate is defined from the two specified forces at their relative deflection.

Fmax  Fmin lbf


Spring rate k  k  266.7 
∆y in

9. To get the defined spring rate, the number of active coils must satisfy equation 14.7, solving for Na yields:
4
Number of active d G
coils Na  Na  19.744 Na  19.75
3
8 D  k

Note that we round it to the nearest 1/4 coil as the manufacturing tolerance cannot achieve better than that
accuracy. Having rounded the number of active coils, we must now calculate the spring rate using equation 14.
4
Corrected d G lbf
spring rate k  k  266.59
3 in
8  D  Na

10. For squared ends the total number of coils, from Figure 14-9:
Total coils Nt  Na  2 Nt  21.75

11. The shut height can now be determined.


Shut height Lshut  d  Nt Lshut  6.786  in

12. The initial deflection to reach the smaller of the two loads is
Fmin
Initial deflection yinit  yinit  0.188  in
k

13. For the given clash allowance of 15% of the working deflection:
Clash allowance ∆yclash  α ∆y ∆yclash  0.112  in

14. The free length (see Figure 14-8) can now be found from
Lf  Lshut  ∆yclash  ∆y  yinit Lf  7.836  in

15. The deflection to the shut height is


© 2011 Pearson Education, Inc., Upper Saddle River, NJ. All rights reserved. This publication is protected by Copyright and written permission should be
MACHINE DESIGN - An Integrated Approach, 4th Ed. 14-13-3
yshut  Lf  Lshut yshut  1.05 in

16. The force at this shut height is


Fshut  k yshut Fshut  279.9  lbf

17. The shut-height stress and safety factor are


8  Fshut D
Stress at Fshut τshut  Ks τshut  35.9 ksi
3
π d

Safety factor at S ys
working deflection Ns  Ns  3.71
τshut
which is acceptable.

18. To check for buckling, two ratios need to be calculated, Lf/D and ymax /Lf.

Lf
Slenderness ratio sr  sr  5.708
D
yinit  ∆y
Deflection ratio y'  y'  0.12
Lf

Take these two values to Figure 14-14 and find that their coordinates are safely within the zones that are stable
against buckling for either end-condition case.

19. The inside and outside coil diameters and minimum hole dia are
Inside coil dia Di  D  d Di  1.061  in
Outside coil dia Do  D  d Do  1.685  in
Minimum hole dia d hole  Do  0.05 D d hole  1.753  in

20. Summarizing the results:


Number of active coils Nt  21.75
Mean coil diameter D  1.373  in
Free length Lf  7.836  in
Shut length Lshut  6.786  in
Deflection to Fmin yinit  0.188  in
Minimum hole dia d hole  1.753  in

© 2011 Pearson Education, Inc., Upper Saddle River, NJ. All rights reserved. This publication is protected by Copyright and written permission should be
MACHINE DESIGN - An Integrated Approach, 4th Ed. 14-14-1
PROBLEM 14-14
Statement: Figure P14-2 shows a child's toy called a pogo stick. The child stands on the pads, applying half
her weight on each side. She jumps off the ground, holding the pads up against her feet, and
bounces along with the spring cushioning the impact and storing energy to help each rebound.
Assume a 60-lb child and a spring constant of 100 lb/in. Design the helical compression spring to
survive jumping 2 in off the ground with a dynamic safety factor of 2 for a finite life of 5E4 cycles
Determine the fundamental natural frequency of the system.
1
Given: Weight of child W  60 lbf Spring constant k  100  lbf  in
4 6
Spring life Ncycles  5  10 Shear modulus G  11.5 10  psi

Design Choices:
Clash allowance α  0.15 Music wire properties:
Spring index C  7 Strength A  184.65 ksi b  0.1625
Squared and ground ends Set after winding, unpeened
Design safety factor Nfsd  1.5

Solution: See Mathcad file P1414.


1. From Problem 3-14, Maximum force Fmax  224.2  lbf
2. Let the force of holding the pads against the feet be Minimum force Fmin  20 lbf
Fmax  Fmin
3. Then, the working deflection is ∆y  ∆y  2.042  in
k
4. Find the the mean and alternating forces from equation 14.15a:
Fmax  Fmin
Alternating force Fa  Fa  102.1  lbf
2
Fmax  Fmin
Mean force Fm  Fm  122.1  lbf
2
5. Calculate the factors necessary to find the wire diameter.
0.5
Direct shear Ks  1  Ks  1.071
factor C

4 C  1 0.615
Wahl factor Kw   Kw  1.213
4 C  4 C
Yield strength
factor Kys  0.60 (Table 14-6, set removed)

Fatigue strength
factor Kfw  0.37 (Table 14-7, unpeened)

Ultimate shear
strength factor KU  0.67 (equation 14.4)

6. Solving for the wire diameter using an equation similar to equation g in Example 14-4A (Mathcad Supplement)
where S fw is used in equation 14.16b instead of S es and S fw = Kfw S ut.

1
2 b
 8 C Nfsd  Nfsd  1 
d    K s  Fm   Ks Fmin   in
 π KU  A  in2  Nfsd 
   U Kfw 
2  K  
     K w  Fa 
   Kfw  
© 2011 Pearson Education, Inc., Upper Saddle River, NJ. All rights reserved. This publication is protected by Copyright and written permission should be
MACHINE DESIGN - An Integrated Approach, 4th Ed. 14-14-2

d  0.281  in Let the wire diameter be d  0.281  in

7. Calculate the ultimate tensile strength, ultimate shear strength, and the fatigue strength at 5E4 cycles.
b
S ut  A   
d
Ultimate tensile strength  S ut  227  ksi
 
in

Ultimate shear strength S us  KU  S ut S us  152  ksi

Wire fatigue strength S fw  Kfw S ut S fw  84.0 ksi

Shear yield strength S ys  Kys S ut S ys  136  ksi

S fw S us
Fatigue strength S fs  0.5 S fs  58.0 ksi
S us  0.5 S fw

8. Calculate the mean coil diameter D from equation 14.5.


Mean coil diameter D  C d D  1.967  in

9. Calculate the shear stress i at the initial deflection (lowest defined force), and the mean stress m.

8  Fmin D
Stress at Fmin τi  Ks τi  4.8 ksi
3
π d
8  Fm D
Stress at Fm τm  Ks τm  29.5 ksi
3
π d

10. Calculate the alternating shear stress a in the coil.

8  F a D
Alternating stress τa  Kw τa  28.0 ksi
3
π d

11. The safety factor is calculated from equation 14.16b.

Fatigue factor S fs  S us  τi


Nfs  Nfs  1.50
of safety S fs  τm  τi  S us τa

12. To get the defined spring rate, the number of active coils must satisfy equation 14.7, solving for Na yields:
4
Number of active d G
coils Na  Na  11.777 Na  11.75
3
8 D  k

Note that we round it to the nearest 1/4 coil as the manufacturing tolerance cannot achieve better than that
accuracy. Having rounded the number of active coils, we must now calculate the spring rate using equation 14.

4
Corrected d G lbf
spring rate k  k  100.23
3 in
8  D  Na
13. For squared and ground ends the total number of coils, from Figure 14-9:
Total coils Nt  Na  2 Nt  13.75
© 2011 Pearson Education, Inc., Upper Saddle River, NJ. All rights reserved. This publication is protected by Copyright and written permission should be
MACHINE DESIGN - An Integrated Approach, 4th Ed. 14-14-3

14. The shut height can now be determined.


Shut height Lshut  d  Nt Lshut  3.864  in

15. The initial deflection to reach the smaller of the two loads is
Fmin
Initial deflection yinit  yinit  0.200  in
k

16. For the given clash allowance factor:


Clash allowance ∆yclash  α ∆y ∆yclash  0.306  in

17. The free length (see Figure 14-8) can now be found from
Lf  Lshut  ∆yclash  ∆y  yinit Lf  6.412  in

18. The deflection to the shut height is


yshut  Lf  Lshut yshut  2.548  in

19. The force at this shut height is


Fshut  k yshut Fshut  255.4  lbf

20. The shut-height stress and safety factor are


8  Fshut D
Stress at Fshut τshut  Ks τshut  61.8 ksi
3
π d

Safety factor at S ys
shut height Ns  Ns  2.20
τshut
which is acceptable.

21. To check for buckling, two ratios need to be calculated, Lf/D and ymax /Lf.

Lf
Slenderness ratio sr  sr  3.26
D
yinit  ∆y
Deflection ratio y'  y'  0.35
Lf

Take these two values to Figure 14-14 and find that their coordinates are safely within the zones that are stable
against buckling for either end-condition case.

22. The inside and outside coil diameters and minimum hole dia are
Inside coil dia Di  D  d Di  1.686  in
Outside coil dia Do  D  d Do  2.248  in
Maximum pin dia d pin  Di  0.05 D d pin  1.588  in

23. Summarizing the results:


Number of active coils Nt  13.75 Deflection to Fmin yinit  0.2 in
Mean coil diameter D  1.967  in Maximum pin dia d pin  1.588  in
Free length Lf  6.412  in Shut length Lshut  3.864  in

© 2011 Pearson Education, Inc., Upper Saddle River, NJ. All rights reserved. This publication is protected by Copyright and written permission should be
MACHINE DESIGN - An Integrated Approach, 4th Ed. 14-15-1
PROBLEM 14-15
Statement: Draw the Modified Goodman diagram for the spring data below and find its safety factor.
Given: Fatigue strength (R=0) S fw  40 ksi Alternating shear stress τa  12 ksi
Ultimate shear strength S us  200  ksi Mean shear stress τm  95 ksi
Initial shear stress τi  75 ksi

Solution: See Mathcad file P1415.


1. From equation (d) in Example 14-2,

S fw S us
S fs  0.5 S fs  22.2 ksi
S us  0.5 S fw

S fs
2. Now, the equation for the Goodman line is τ'a τ'm    τ'm  S fs
S us

τa
Equation for load line τaL τ'm   τ'm
τm

3. Plotting this over the range τ'm  0  ksi 10 ksi  200  ksi

25
τm
Alternating shear stress

20 ksi

15
τa
10 ksi

0
0 25 50 75 100 125 150 175 200

Mean shear stress

4. Use equation 14.16b to calculate the safety factor.

S fs  S us  τi
Nfs  Nfs  0.98
S fs  τm  τi  S us τa

© 2011 Pearson Education, Inc., Upper Saddle River, NJ. All rights reserved. This publication is protected by Copyright and written permission should be
MACHINE DESIGN - An Integrated Approach, 4th Ed. 14-16-1
PROBLEM 14-16
Statement: Problem 6-16 describes a track for bowling balls that are 4.5-in dia and 2.5-lb weight. Design a
spring-loaded launcher that will allow quadriplegic bowlers to launch the balls down the
bowling alley from the point where the track of Problem 6-16 drops them with only a switch
closure that releases the launcher. The launcher's plunger will be cocked by an assistant and
the energy stored in the helical compression spring, which you will design, will drive the
plunger into the ball and roll it down the bowling alley.
6
Given: Ball weight Wball  2.5 lbf Shear modulus G  11.5 10  psi
Design Choices:
Clash allowance α  0.10 Music wire properties:
Spring index C  12 Strength A  184.65 ksi b  0.1625
Design safety factor Nfsd  2 Set after winding, unpeened
1
Ball initial velocity vi  150  in sec Squared and ground ends
Deflection to Fmax ∆y  5.000  in

Solution: See Mathcad file P1416.


1. Determine the required spring rate by equating the spring potential energy with the ball's initial velocity when
the spring force reaches zero.
1 2 1 2
PE = KE  k ∆y =  mball  vi
2 2
2
Wball  3 lbf  sec
mball  mball  6.475  10 
g in
2
 vi  lbf
k  mball    k  5.828 
 ∆y  in

2. Determine the maximum spring force and set the minimum force to zero.

Fmin  0  lbf Fmax  k ∆y Fmax  29.138 lbf


3. Find the the mean and alternating forces from equation 14.15a:

Fmax  Fmin
Alternating force Fa  Fa  14.6 lbf
2
Fmax  Fmin
Mean force Fm  Fm  14.6 lbf
2

4. Calculate the factors necessary to find the wire diameter.


0.5
Direct shear Ks  1  Ks  1.042
factor C

4 C  1 0.615
Wahl factor Kw   Kw  1.119
4 C  4 C
Yield strength
factor Kys  0.60 (Table 14-6, set removed)

Ultimate shear
strength factor KU  0.67 (equation 14.4)

Torsional
endurance limit S ew  45 ksi (unpeened)
© 2011 Pearson Education, Inc., Upper Saddle River, NJ. All rights reserved. This publication is protected by Copyright and written permission should be
MACHINE DESIGN - An Integrated Approach, 4th Ed. 14-16-2

5. Solve for the wire diameter using equation g in Example 14-4A (Mathcad Supplement).
Guess d  0.2 in
Given
1
2 b
 8 C Nfsd  Nfsd  1 
d=  Ks Fm   Ks Fmin    in
2 Nfsd
 0.67 π A  in  
   d
b
 
   0.67  in 
A 
 
     1  Kw Fa 
   0.50 S ew  

d  Find ( d ) d  0.209  in Let the wire diameter be d  0.207  in

6. Calculate the ultimate tensile strength, ultimate shear strength, and the shear endurance limit.
b
S ut  A   
d
Ultimate tensile strength  S ut  239  ksi
 in 
Ultimate shear strength S us  KU  S ut S us  160  ksi

Shear yield strength S ys  Kys S ut S ys  143  ksi

S ew S us
Fatigue strength S es  0.5 S es  26.2 ksi
S us  0.5 S ew

7. Calculate the mean coil diameter D from equation 14.5.


Mean coil diameter D  C d D  2.484  in

8. Calculate the shear stress i at the initial deflection (lowest defined force), and the mean stress m.

8  Fmin D
Stress at Fmin τi  Ks τi  0.0 ksi
3
π d
8  Fm D
Stress at Fm τm  Ks τm  10.8 ksi
3
π d

9. Calculate the alternating shear stress a in the coil.

8  F a D
Alternating stress τa  Kw τa  11.6 ksi
3
π d

10. The safety factor is calculated from equation 14.16b.

Fatigue factor S es  S us  τi


Nfs  Nfs  2.0
of safety S es  τm  τi  S us τa
11. To get the defined spring rate, the number of active coils must satisfy equation 14.7, solving for Na yields:
4
Number of active d G
coils Na  Na  29.549 Na  29.5
3
8 D  k
© 2011 Pearson Education, Inc., Upper Saddle River, NJ. All rights reserved. This publication is protected by Copyright and written permission should be
MACHINE DESIGN - An Integrated Approach, 4th Ed. 14-16-3

Note that we round it to the nearest 1/4 coil as the manufacturing tolerance cannot achieve better than that
accuracy. Having rounded the number of active coils, we must now calculate the spring rate using equation 14.
4
Corrected d G lbf
spring rate k  k  5.84
3 in
8  D  Na

12. For squared and ground ends the total number of coils, from Figure 14-9:
Total coils Nt  Na  2 Nt  31.50

13. The shut height can now be determined.


Shut height Lshut  d  Nt Lshut  6.52 in

14. The initial deflection to reach the smaller of the two loads is
Fmin
Initial deflection yinit  yinit  0.000  in
k

15. For the given clash allowance factor:


Clash allowance ∆yclash  α ∆y ∆yclash  0.500  in

16. The free length (see Figure 14-8) can now be found from
Lf  Lshut  ∆yclash  ∆y  yinit Lf  12.021 in
17. The deflection to the shut height is
yshut  Lf  Lshut yshut  5.500  in
18. The force at this shut height is
Fshut  k yshut Fshut  32.1 lbf
19. The shut-height stress and safety factor are
8  Fshut D
Stress at Fshut τshut  Ks τshut  23.8 ksi
3
π d

Safety factor at S ys
shut height Ns  Ns  6.00
τshut
which is acceptable.
20. To check for buckling, two ratios need to be calculated, Lf/D and ymax /Lf.

Lf
Slenderness ratio sr  sr  4.839
D
yinit  ∆y
Deflection ratio y'  y'  0.416
Lf

Take these two values to Figure 14-14 and find that their coordinates are safely within the zone that is stable
against buckling for parallel ends.
21. The inside and outside coil diameters and minimum hole dia are
Inside coil dia Di  D  d Di  2.277  in
Outside coil dia Do  D  d Do  2.691  in
Maximum pin dia d pin  Di  0.05 D d pin  2.153  in

© 2011 Pearson Education, Inc., Upper Saddle River, NJ. All rights reserved. This publication is protected by Copyright and written permission should be
MACHINE DESIGN - An Integrated Approach, 4th Ed. 14-16-4

22. Summarizing the results:


Number of active coils Nt  31.50
Mean coil diameter D  2.484  in
Free length Lf  12.021 in
Shut length Lshut  6.520  in
Deflection to Fmin yinit  0  in
Maximum pin dia d pin  2.153  in

© 2011 Pearson Education, Inc., Upper Saddle River, NJ. All rights reserved. This publication is protected by Copyright and written permission should be
MACHINE DESIGN - An Integrated Approach, 4th Ed. 14-17-1
PROBLEM 14-17
Statement: Design a helical extension spring to handle a dynamic load that varies from 175 N to 225 N
over 8.5 mm working deflection. Use music wire and standard hooks. The forcing frequency
is 1500 rpm. Infinite life is desired. Minimize the package size. Choose appropriate safety
factors against fatigue, yielding, and surging.

Given: Minimum force Fmin  175  N Life L  ∞


Maximum force Fmax  225  N Shear modulus G  80.8 GPa
Working deflection ∆y  8.5 mm Forcing freq ωf  1500 rpm
Design Choices:
Fatigue safety factor Nfb  1.5 Spring index C  8
Wire endurance limit S ew  310  MPa Wire strength A  2153.5 MPa
b  0.1625

Solution: See Mathcad file P1417.


1. Find the mean and alternating loads.
Fmax  Fmin
Fa  Fa  25.0 N
2
Fmax  Fmin
Fm  Fm  200.0  N
2

2. Calculate the hook bending factor.


2
4 C  C  1
Kb  Kb  1.103
4 C ( C  1)

3. Solve for d by iteration using an equation derived from the safety factor equation for bending in the hooks.
1
2 b
4   4  K b C  1 
d=   Nfb Fm   Nfb  1   Fmin 


 π A  
  0.577   0.67 A  db  0.5 S ew  
  
  Nfb Fa  0.5 0.67 S ew  
   
1
2 b
 4  4 Kb C  1 
RHS( d )    Nfb Fm   Nfb  1   Fmin    mm
 π A  mm2    b 

    
0.577 0.67 A   d 
 0.5 S ew 

   

 Nfb Fa 
  mm  
  0.5 0.67 S ew  
    

d  1.000  mm RHS( d )  3.561  mm


d  RHS( d ) RHS( d )  3.329  mm
d  RHS( d ) RHS( d )  3.340  mm
d  RHS( d ) RHS( d )  3.339  mm d  3.5 mm
4. Calculate the mean coil diameter.
Mean coil diameter D  C d D  28 mm
© 2011 Pearson Education, Inc., Upper Saddle River, NJ. All rights reserved. This publication is protected by Copyright and written permission should be
MACHINE DESIGN - An Integrated Approach, 4th Ed. 14-17-2

5. Use the assumed value of C to find an appropiate value of initial coil stress i from equations 14.21:

 3 2 
τi1  4.231  C  181.5  C  3387 C  28640  psi τi1  75.8 MPa

  2.987  C  139.7  C  3427 C  38404   psi


3 2
τi2 τi2  126.9  MPa

τi1  τi2
τi  τi  101.3  MPa
2
6. Find the direct shear factor Ks :
0.5
Direct shear factor Ks  1  Ks  1.063
C
7. Use the value of i from (c) in equation 14.8 to find the corresponding initial coil-tension force Fi:
3
π d  τi
Fi  Fi  57.3 N
8  K s D

Check that this force is less than the required minimum applied force Fmin, which in this case, it is. Any applied
force smaller than Fi will not deflect the spring.
8. Use the direct shear factor Ks and previously assumed values to find the mean stress m:

8  Fm D
Stress at Fm τm  Ks τm  353.4  MPa
3
π d

9. Find the Wahl factor Kw and use it to calculate the alternating shear stress a in the coil.
4 C  1 0.615
Wahl factor Kw   Kw  1.184
4 C  4 C
8  F a D
Alternating stress τa  Kw τa  49.2 MPa
3
π d
10. Find the ultimate tensile strength of this wire material from equation 14.3 and Table 14-4 and use it to find the
ultimate shear strength from equation 14.4 and the torsional yield strength for the coil body from Table 14-10,
assuming no set removal.
b
S ut  A    d
Ultimate tensile strength  S ut  1756.8 MPa
 
mm

Shear yield strength S ys  0.45 S ut S ys  790.6  MPa

Ultimate shear strength S us  0.67 S ut S us  1177.1 MPa

11. Find the wire endurance limit for unpeened springs from equation 14.13 and convert it to fully reversed
endurance strength with equation 14.16c.
S ew S us
Fully reversed S es  0.5 S es  178.51 MPa
endurance limit S us  0.5 S ew

12. The fatigue safety factor for the coils in torsion is calculated from equation 14.16b.
Minimum stress τmin  τm  τa τmin  304.2  MPa

S es  S us  τmin
Fatigue safety factor Nfs  Nfs  2.34
S es  τm  τmin  S us τa

© 2011 Pearson Education, Inc., Upper Saddle River, NJ. All rights reserved. This publication is protected by Copyright and written permission should be
MACHINE DESIGN - An Integrated Approach, 4th Ed. 14-17-3

Note that the minimum stress due to force Fmin is used in this calculation, not the coil-winding stress from step 5

13. The stresses in the end hooks also need to be determined. The bending stresses in the hook are found from
equation 14.23:

2  R1 2 D
C1 = = =C C1  C C1  8.00
d 2 d
2
4 C1  C1  1
Kb  Kb  1.103
4  C1  C1  1 

16 D Fa 4  Fa
σa  Kb  σa  94.29  MPa
3 2
π d π d

16 D Fm 4  Fm
σm  Kb  σm  754.3  MPa
3 2
π d π d

16 D Fmin 4  Fmin


σmin  Kb  σmin  660.01 MPa
3 2
π d π d

14. Convert the torsional endurance strength to a tensile endurance strength with the von Mises relationship and
use it and the ultimate tensile strength in equation 14.16 to find a fatigue safety factor for the hook in bending:

S es
S e  S e  309.37 MPa
0.577

S e  S ut  σmin
Nfb  Nfb  1.74
S e  σm  σmin  S ut σa

15. The torsional stresses in the hook are found from equation 14.24 using an assumed value of C2  5.
C2 d
R2  R2  8.75 mm
2
4  C2  1
Kw2  Kw2  1.188
4  C2  4

8  Fa D
τBa  Kw2 τBa  49.4 MPa
3
π d
8  Fm D
τBm  Kw2 τBm  395.0  MPa
3
π d
8  Fmin D
τBmin  Kw2 τBmin  345.6  MPa
3
π d
16. The fatigue safety factor for the hook in torsion is calculated from equation 14.16b.

S es  S us  τBmin
Nfs  Nfs  2.22
S es  τBm  τBmin  S us τBa
© 2011 Pearson Education, Inc., Upper Saddle River, NJ. All rights reserved. This publication is protected by Copyright and written permission should be
MACHINE DESIGN - An Integrated Approach, 4th Ed. 14-17-4

17. The spring rate is defined from the two specified forces at their relative deflection.

Fmax  Fmin N
Spring rate k  k  5882.4
∆y m

18. To get the defined spring rate, the number of active coils must satisfy equation 14.7, solving for Na yields:
4
d G
Number of active coils Na  Na  11.737 Na  11.75
3
8 D  k
Note that we round it to the nearest 1/4 coil as the manufacturing tolerance cannot achieve better than that
accuracy. Having rounded the number of active coils, we must now calculate the spring rate using equation 14.
Corrected spring rate 4
d G N
k  k  5876.0
3 m
8  D  Na
19. The total number of coils in the body and the body length are
Total coils Nt  Na  1 Nt  12.75

Body length Lb  Nt d Lb  44.6 mm

20. The free length can now be determined. The length of a standard hook is equal to the coil inside diameter:
Hook length Lhook  D  d Lhook  24.5 mm

Free length Lf  Lb  2  Lhook Lf  93.6 mm

21. The initial coil tension force must be found again in order to obtain the deflection to reach the larger of the two
loads.
 3 2 
τi1  4.231  C  181.5  C  3387 C  28640  psi τi1  75.8 MPa

  2.987  C  139.7  C  3427 C  38404   psi


3 2
τi2 τi2  126.9  MPa
τi1  τi2
τi  τi  101.3  MPa
2
3
π d  τi
Fi  Fi  57.3 N
8  K s D

Fmax  Fi
ymax  ymax  28.53  mm
k

22. The inside and outside coil diameters are


Inside coil dia Di  D  d Di  24.50  mm
Outside coil dia Do  D  d Do  31.50  mm

23. The weight of the spring's active coils is found from equation 14.11b and is
3
Weight density ρ  0.28 lbf  in
2 2
π  d  D Na ρ
Weight Wa  Wa  0.756  N
4
24. The natural frequency of this spring is found from equation 14.11a and is:

© 2011 Pearson Education, Inc., Upper Saddle River, NJ. All rights reserved. This publication is protected by Copyright and written permission should be
MACHINE DESIGN - An Integrated Approach, 4th Ed. 14-17-5

Natural frequency 1 k g
fn   fn  138.1  Hz
2 Wa

25. The ratio between the natural frequency and the forcing frequency is

ωf
Forcing frequency ff  ff  25 Hz
2 π

fn
Frequency ratio  5.5 which could be higher.
ff

26. We now have a complete design specification for this A228-wire spring:
Wire diameter d  3.50 mm Total coils Nt  12.75
Outside diameter Do  31.50  mm Free length Lf  93.63  mm
Standard hooks

© 2011 Pearson Education, Inc., Upper Saddle River, NJ. All rights reserved. This publication is protected by Copyright and written permission should be
MACHINE DESIGN - An Integrated Approach, 4th Ed. 14-18-1
PROBLEM 14-18
Statement: Design a helical extension spring with standard hooks to handle a dynamic load that varies from
300 lb to 500 lb over 2 in working deflection. Use chrome-vanadium wire. The forcing
frequency is 1000 rpm. Infinite life is desired. Minimize the package size. Choose appropriate
safety factors against fatigue, yielding, and surging.

Given: Minimum force Fmin  300  lbf Life L  ∞


6
Maximum force Fmax  500  lbf Shear modulus G  11.7 10  psi
Working deflection ∆y  2.00 in Forcing freq. ωf  1000 rpm

Design Choices:
Fatigue safety factor Nfb  1.28 Spring index C  5.5
Wire endurance limit S ew  45 ksi Wire strength A  173.13 ksi
b  0.1453
Solution: See Mathcad file P1418.
1. Find the mean and alternating loads.
Fmax  Fmin
Fa  Fa  100.0  lbf
2
Fmax  Fmin
Fm  Fm  400.0  lbf
2
2. Calculate the hook bending factor.
2
4 C  C  1
Kb  Kb  1.157
4 C ( C  1)

3. Solve for d by iteration using an equation derived from the safety factor equation for bending in the hooks.
1
2 b
4   4  K b C  1 
d=   Nfb Fm   Nfb  1   Fmin 


 π A  
  0.577   0.67 A  db  0.5 S ew  
  
  Nfb Fa  0.5 0.67 S ew  
   
1
2 b
 4  4 Kb C  1 
RHS( d )    Nfb Fm   Nfb  1   Fmin    in
 π A  in2  
   b  
0.577 0.67 A    0.5 S ew 

 d
  
     in   
 Nfb Fa   
 0.5 0.67 S ew
    

d  0.1 in RHS( d )  0.438  in


d  RHS( d ) RHS( d )  0.404  in
d  RHS( d ) RHS( d )  0.406  in
d  RHS( d ) RHS( d )  0.406  in d  0.406  in
4. Calculate the mean coil diameter.
Mean coil diameter D  C d D  2.233  in
© 2011 Pearson Education, Inc., Upper Saddle River, NJ. All rights reserved. This publication is protected by Copyright and written permission should be
MACHINE DESIGN - An Integrated Approach, 4th Ed. 14-18-2

5. Use the assumed value of C to find an appropiate value of initial coil stress i from equations 14.21:

 3 2 
τi1  4.231  C  181.5  C  3387 C  28640  psi τi1  14.8 ksi

  2.987  C  139.7  C  3427 C  38404   psi


3 2
τi2 τi2  23.3 ksi

τi1  τi2
τi  τi  19.0 ksi
2
6. Find the direct shear factor Ks :
0.5
Direct shear factor Ks  1  Ks  1.091
C
7. Use the value of i from (c) in equation 14.8 to find the corresponding initial coil-tension force Fi:
3
π d  τi
Fi  Fi  205.4  lbf
8  K s D

Check that this force is less than the required minimum applied force Fmin, which in this case, it is. Any
applied force smaller than Fi will not deflect the spring.
8. Use the direct shear factor Ks and previously assumed values to find the mean stress m:

8  Fm D
Stress at Fm τm  Ks τm  37.1 ksi
3
π d

9. Find the Wahl factor Kw and use it to calculate the alternating shear stress a in the coil.
4 C  1 0.615
Wahl factor Kw   Kw  1.278
4 C  4 C

8  F a D
Alternating stress τa  Kw τa  10.9 ksi
3
π d
10. Find the ultimate tensile strength of this wire material from equation 14.3 and Table 14-4 and use it to find the
ultimate shear strength from equation 14.4 and the torsional yield strength for the coil body from Table 14-10,
assuming no set removal.
b
S ut  A   
d
Ultimate tensile strength  S ut  197.4  ksi
 in 
Shear yield strength S ys  0.50 S ut S ys  98.7 ksi

Ultimate shear strength S us  0.67 S ut S us  132.2  ksi

11. Find the wire endurance limit for unpeened springs from equation 14.13 and convert it to fully reversed
endurance strength with equation 14.16c.
S ew S us
Fully reversed S es  0.5 S es  27.11  ksi
endurance limit S us  0.5 S ew

12. The fatigue safety factor for the coils in torsion is calculated from equation 14.16b.
Minimum stress τmin  τm  τa τmin  26.214 ksi

S es  S us  τmin
Fatigue safety factor Nfs  Nfs  1.66
S es  τm  τmin  S us τa
© 2011 Pearson Education, Inc., Upper Saddle River, NJ. All rights reserved. This publication is protected by Copyright and written permission should be
MACHINE DESIGN - An Integrated Approach, 4th Ed. 14-18-3

Note that the minimum stress due to force Fmin is used in this calculation, not the coil-winding stress from
step 5.

13. The stresses in the end hooks also need to be determined. The bending stresses in the hook are found from
equation 14.23:

2  R1 2 D
C1 = = =C C1  C C1  5.50
d 2 d
2
4 C1  C1  1
Kb  Kb  1.157
4  C1  C1  1 

16 D Fa 4  Fa
σa  Kb  σa  20.43  ksi
3 2
π d π d

16 D Fm 4  Fm
σm  Kb  σm  81.7 ksi
3 2
π d π d

16 D Fmin 4  Fmin


σmin  Kb  σmin  61.28  ksi
3 2
π d π d

14. Convert the torsional endurance strength to a tensile endurance strength with the von Mises relationship and
use it and the ultimate tensile strength in equation 14.16 to find a fatigue safety factor for the hook in bending:

S es
S e  S e  46.99  ksi
0.577
S e  S ut  σmin
Nfb  Nfb  1.28
S e  σm  σmin  S ut σa

15. The torsional stresses in the hook are found from equation 14.24 using an assumed value of C2  5.
C2 d
R2  R2  1.015  in
2
4  C2  1
Kw2  Kw2  1.188
4  C2  4
8  Fa D
τBa  Kw2 τBa  10.1 ksi
3
π d
8  Fm D
τBm  Kw2 τBm  40.4 ksi
3
π d
8  Fmin D
τBmin  Kw2 τBmin  30.3 ksi
3
π d
16. The fatigue safety factor for the hook in torsion is calculated from equation 14.16b.
S es  S us  τBmin
Nfs  Nfs  1.72
S es  τBm  τBmin  S us τBa

© 2011 Pearson Education, Inc., Upper Saddle River, NJ. All rights reserved. This publication is protected by Copyright and written permission should be
MACHINE DESIGN - An Integrated Approach, 4th Ed. 14-18-4
17. The spring rate is defined from the two specified forces at their relative deflection.

Fmax  Fmin lbf


Spring rate k  k  100.0 
∆y in

18. To get the defined spring rate, the number of active coils must satisfy equation 14.7, solving for Na yields:
4
d G
Number of active coils Na  Na  35.689 Na  35.75
3
8 D  k
Note that we round it to the nearest 1/4 coil as the manufacturing tolerance cannot achieve better than that
accuracy. Having rounded the number of active coils, we must now calculate the spring rate using equation 14.
4
d G lbf
Corrected spring rate k  k  99.8
3 in
8  D  Na

19. The total number of coils in the body and the body length are
Total coils Nt  Na  1 Nt  36.75

Body length Lb  Nt d Lb  14.92  in

20. The free length can now be determined. The length of a standard hook is equal to the coil inside diameter:
Hook length Lhook  D  d Lhook  1.827  in

Free length Lf  Lb  2  Lhook Lf  18.57  in

21. The initial coil tension force must be found again in order to obtain the deflection to reach the larger of the
two loads.
 3 2 
τi1  4.231  C  181.5  C  3387 C  28640  psi τi1  14.8 ksi

  2.987  C  139.7  C  3427 C  38404   psi


3 2
τi2 τi2  23.3 ksi
τi1  τi2
τi  τi  19.0 ksi
2
3
π d  τi
Fi  Fi  205.4  lbf
8  K s D
Fmax  Fi
ymax  ymax  2.95 in
k
22. The inside and outside coil diameters are
Inside coil dia Di  D  d Di  1.83 in
Outside coil dia Do  D  d Do  2.64 in

23. The weight of the spring's active coils is found from equation 14.11b and is
3
Weight density ρ  0.28 lbf  in
2 2
π  d  D Na ρ
Weight Wa  Wa  9.091  lbf
4
24. The natural frequency of this spring is found from equation 14.11a and is:
Natural frequency 1 k g
fn   fn  32.6 Hz
2 Wa
© 2011 Pearson Education, Inc., Upper Saddle River, NJ. All rights reserved. This publication is protected by Copyright and written permission should be
MACHINE DESIGN - An Integrated Approach, 4th Ed. 14-18-5

25. The ratio between the natural frequency and the forcing frequency is

ωf
Forcing frequency ff  ff  16.667 Hz
2 π
fn
Frequency ratio  2.0 which could be higher.
ff

26. We now have a complete design specification for this A228-wire spring:
Wire diameter d  0.41 in Total coils Nt  36.75
Outside diameter Do  2.64 in Free length Lf  18.57  in
Standard hooks

© 2011 Pearson Education, Inc., Upper Saddle River, NJ. All rights reserved. This publication is protected by Copyright and written permission should be
MACHINE DESIGN - An Integrated Approach, 4th Ed. 14-19-1
PROBLEM 14-19
Statement: Design a helical compression spring to handle a dynamic load that varies from 780 N to 1000 N
over a 22-mm working deflection. Use squared and ground, unpeened music wire and a 10%
clash allowance. The forcing frequency is 500 rpm. Infinite life is desired. Minimize the
package size. Choose appropriate safety factors against fatigue, yielding, and surging.

Given: Minimum force Fmin  780  N Forcing freq. ωf  500  rpm


Maximum force Fmax  1000 N Shear modulus G  79.6 GPa
Working deflection ∆y  22 mm Clash factor α  0.10
Music wire properties:
Strength A  2153.5 MPa b  0.1625
Set after winding, unpeened
Design Choices:
Design safety factor Nfsd  1.1 Spring index C  9

Solution: See Mathcad file P1419.


1. Find the the mean and alternating forces from equation 14.15a:

Fmax  Fmin
Alternating force Fa  Fa  110.0  N
2
Fmax  Fmin
Mean force Fm  Fm  890.0  N
2

2. Calculate the factors necessary to find the wire diameter.


0.5
Direct shear Ks  1  Ks  1.056
factor C

4 C  1 0.615
Wahl factor Kw   Kw  1.162
4 C  4 C
Yield strength
factor Kys  0.60 (Table 14-6, set removed)

Ultimate shear
strength factor KU  0.67 (equation 14.4)

Torsional
endurance limit S ew  310  MPa (unpeened)

3. Solve for the wire diameter using equation g in Example 14-4A (Mathcad Supplement).
Guess d  5  mm
Given
1
2 b
 8  C Nfsd  Nfsd  1 
d=  Ks Fm   Ks Fmin    mm
2 Nfsd
 0.67  π  A  mm  
    d 
b  
 
  0.67  mm 
A    
     1  Kw Fa 
   0.50 S ew  

d  Find ( d ) d  6.172  mm Let the wire diameter be d  6.5 mm


4. Calculate the ultimate tensile strength, ultimate shear strength, and the shear endurance limit.
© 2011 Pearson Education, Inc., Upper Saddle River, NJ. All rights reserved. This publication is protected by Copyright and written permission should be
MACHINE DESIGN - An Integrated Approach, 4th Ed. 14-19-2
b
S ut  A    d
Ultimate tensile strength  S ut  1589 MPa
 
mm

Ultimate shear strength S us  KU  S ut S us  1064 MPa

Shear yield strength S ys  Kys S ut S ys  953  MPa

S ew S us
Fatigue strength S es  0.5 S es  181.4  MPa
S us  0.5 S ew

5. Calculate the mean coil diameter D from equation 14.5.


Mean coil diameter D  C d D  58.5 mm

6. Calculate the shear stress i at the initial deflection (lowest defined force), and the mean stress m.

8  Fmin D
Stress at Fmin τi  Ks τi  446.6  MPa
3
π d
8  Fm D
Stress at Fm τm  Ks τm  509.6  MPa
3
π d

7. Calculate the alternating shear stress a in the coil.

8  F a D
Alternating stress τa  Kw τa  69.3 MPa
3
π d

8. The safety factor is calculated from equation 14.16b.

Fatigue factor S es  S us  τi


Nfs  Nfs  1.3
of safety S es  τm  τi  S us τa

9. The spring rate is defined in this problem because of the two specified forces at a particular relative deflection.

Fmax  Fmin N
Spring rate k  k  10.00 
∆y mm

10. To get the defined spring rate, the number of active coils must satisfy equation 14.7, solving for Na yields:
4
Number of active d G
coils Na  Na  8.872 Na  8.75
3
8 D  k

Note that we round it to the nearest 1/4 coil as the manufacturing tolerance cannot achieve better than that
accuracy. Having rounded the number of active coils, we must now calculate the spring rate using equation 14.
4
Corrected d G N
spring rate k  k  10.14 
3 mm
8  D  Na

11. For squared and ground ends the total number of coils, from Figure 14-9:
Total coils Nt  Na  2 Nt  10.75

© 2011 Pearson Education, Inc., Upper Saddle River, NJ. All rights reserved. This publication is protected by Copyright and written permission should be
MACHINE DESIGN - An Integrated Approach, 4th Ed. 14-19-3
12. The shut height can now be determined.
Shut height Lshut  d  Nt Lshut  69.9 mm

13. The initial deflection to reach the smaller of the two loads is
Fmin
Initial deflection yinit  yinit  76.9 mm
k

14. For the given clash allowance factor:


Clash allowance ∆yclash  α ∆y ∆yclash  2.2 mm

15. The free length (see Figure 14-8) can now be found from
Lf  Lshut  ∆yclash  ∆y  yinit Lf  171.0  mm

16. The deflection to the shut height is


yshut  Lf  Lshut yshut  101.1  mm

17. The force at this shut height is


Fshut  k yshut Fshut  1025 N

18. The shut-height stress and safety factor are


8  Fshut D
Stress at Fshut τshut  Ks τshut  587.1  MPa
3
π d

Safety factor at S ys
shut height Ns  Ns  1.62
τshut
which is acceptable.

20. To check for buckling, two ratios need to be calculated, Lf/D and ymax /Lf.

Lf
Slenderness ratio sr  sr  2.923
D
yinit  ∆y
Deflection ratio y'  y'  0.579
Lf

Take these two values to Figure 14-14 and find that their coordinates are safely within the zones that are stable
against buckling for both cases.

21. The inside and outside coil diameters are


Inside coil dia Di  D  d Di  52 mm
Outside coil dia Do  D  d Do  65 mm
22. The smallest hole and largest pin that should be used with this spring are
Smallest hole holemin  Do  0.05 D holemin  67.9 mm

Largest pin pin max  Di  0.05 D pin max  49.1 mm

23. The weight of the springs active coils is found from equation 14.11b
3
Weight density ρ  0.285  lbf  in

© 2011 Pearson Education, Inc., Upper Saddle River, NJ. All rights reserved. This publication is protected by Copyright and written permission should be
MACHINE DESIGN - An Integrated Approach, 4th Ed. 14-19-4

2 2
π  d  D Na ρ
Weight Wa  Wa  4.13 N
4

24. The natural frequency of this spring is found from equation 14.11a and is:

1 k g
Natural frequency fn   fn  77.6 Hz
2 Wa

25. The ratio between the natural frequency and the forcing frequency is

ωf
Forcing frequency ff  ff  8.333  Hz
2 π
fn
Frequency ratio  9.3
ff

which is sufficiently high.

26. We now have a complete design specification for this A228 wire spring:

Wire diameter d  6.5 mm


Outside diameter Do  65.0 mm
Total coils Nt  10.75 ends squared and ground
Free length Lf  171  mm

25. The initial and working forces are


Initial force Finit  k yinit Finit  780  N

Working force Fwork  k  yinit  ∆y  Fwork  1003 N

26. Safety factors:

S ys
Yielding Nys  Nys  1.6
τshut

S es  S us  τi
Fatigue Nfs  Nfs  1.32
S es  τm  τi  S us τa

fn
Surging Nsurge  Nsurge  9.3
ff

© 2011 Pearson Education, Inc., Upper Saddle River, NJ. All rights reserved. This publication is protected by Copyright and written permission should be
MACHINE DESIGN - An Integrated Approach, 4th Ed. 14-20-1
PROBLEM 14-20
Statement: Design a helical compression spring to handle a dynamic load that varies from 135 N to 220 N
over a 32-mm working deflection. Use squared, peened chrome-vanadium wire and a 15% clash
allowance. The forcing frequency is 250 rpm. Infinite life is desired. Minimize the package size.
Choose appropriate safety factors against fatigue, yielding, and surging.

Given: Minimum force Fmin  135  N Forcing freq. ωf  250  rpm


Maximum force Fmax  220  N Shear modulus G  79.6 GPa
Working deflection ∆y  32 mm Clash factor α  0.15
Chrome-vanadium wire properties:
Strength A  1909.9 MPa b  0.1453
Set after winding, unpeened
Design Choices:
Design safety factor Nfsd  1.1 Spring index C  10.5

Solution: See Mathcad file P1420.


1. Find the the mean and alternating forces from equation 14.15a:

Fmax  Fmin
Alternating force Fa  Fa  42.5 N
2
Fmax  Fmin
Mean force Fm  Fm  177.5  N
2

2. Calculate the factors necessary to find the wire diameter.


0.5
Direct shear Ks  1  Ks  1.048
factor C

4 C  1 0.615
Wahl factor Kw   Kw  1.138
4 C  4 C
Yield strength
factor Kys  0.65 (Table 14-6, set removed)

Ultimate shear
strength factor KU  0.67 (equation 14.4)

Torsional
endurance limit S ew  465  MPa (peened)

3. Solve for the wire diameter using equation g in Example 14-4A (Mathcad Supplement).
Guess d  5  mm
Given
1
2 b
 8  C Nfsd  Nfsd  1 
d=   Ks Fm   Ks Fmin    mm
2 Nfsd
 0.67 π A  mm  
    d 
b  
   0.67  mm 
A 
 
     1  Kw Fa 
   0.50 S ew  

d  Find ( d ) d  3.081  mm Let the wire diameter be d  3.5 mm

4. Calculate the ultimate tensile strength, ultimate shear strength, and the shear endurance limit.
© 2011 Pearson Education, Inc., Upper Saddle River, NJ. All rights reserved. This publication is protected by Copyright and written permission should be
MACHINE DESIGN - An Integrated Approach, 4th Ed. 14-20-2
b
S ut  A   
d
Ultimate tensile strength  S ut  1592 MPa
 mm 
Ultimate shear strength S us  KU  S ut S us  1067 MPa

Shear yield strength S ys  Kys S ut S ys  1035 MPa

S ew S us
Fatigue strength S es  0.5 S es  297.3  MPa
S us  0.5 S ew

5. Calculate the mean coil diameter D from equation 14.5.


Mean coil diameter D  C d D  36.8 mm

6. Calculate the shear stress i at the initial deflection (lowest defined force), and the mean stress m.

8  Fmin D
Stress at Fmin τi  Ks τi  308.7  MPa
3
π d
8  Fm D
Stress at Fm τm  Ks τm  405.9  MPa
3
π d

7. Calculate the alternating shear stress a in the coil.

8  F a D
Alternating stress τa  Kw τa  105.5  MPa
3
π d

8. The safety factor is calculated from equation 14.16b.

Fatigue factor S es  S us  τi


Nfs  Nfs  1.6
of safety S es  τm  τi  S us τa

9. The spring rate is defined in this problem because of the two specified forces at a particular relative deflection.

Fmax  Fmin N
Spring rate k  k  2.66
∆y mm

10. To get the defined spring rate, the number of active coils must satisfy equation 14.7, solving for Na yields:
4
Number of active d G
coils Na  Na  11.325 Na  9.75
3
8 D  k
Note that we round it to the nearest 1/4 coil as the manufacturing tolerance cannot achieve better than that
accuracy. Having rounded the number of active coils, we must now calculate the spring rate using equation 14.
4
Corrected d G N
spring rate k  k  3.09
3 mm
8  D  Na

11. For squared and ground ends the total number of coils, from Figure 14-9:
Total coils Nt  Na  2 Nt  11.75
12. The shut height can now be determined.
Shut height Lshut  d  Nt Lshut  1.619  in
© 2011 Pearson Education, Inc., Upper Saddle River, NJ. All rights reserved. This publication is protected by Copyright and written permission should be
MACHINE DESIGN - An Integrated Approach, 4th Ed. 14-20-3
13. The initial deflection to reach the smaller of the two loads is
Fmin
Initial deflection yinit  yinit  43.8 mm
k

14. For the given clash allowance factor:


Clash allowance ∆yclash  α ∆y ∆yclash  4.8 mm

15. The free length (see Figure 14-8) can now be found from
Lf  Lshut  ∆yclash  ∆y  yinit Lf  121.7  mm

16. The deflection to the shut height is


yshut  Lf  Lshut yshut  80.6 mm

17. The force at this shut height is


Fshut  k yshut Fshut  248.5  N

18. The shut-height stress and safety factor are


8  Fshut D
Stress at Fshut τshut  Ks τshut  568.3  MPa
3
π d

Safety factor at S ys
shut height Ns  Ns  1.82
τshut
which is acceptable.

20. To check for buckling, two ratios need to be calculated, Lf/D and ymax /Lf.

Lf
Slenderness ratio sr  sr  3.311
D
yinit  ∆y
Deflection ratio y'  y'  0.623
Lf

Take these two values to Figure 14-14 and find that their coordinates are safely within the zones that are stable
against buckling for both cases.

21. The inside and outside coil diameters are


Inside coil dia Di  D  d Di  33.25  mm
Outside coil dia Do  D  d Do  40.25  mm
22. The smallest hole and largest pin that should be used with this spring are
Smallest hole holemin  Do  0.05 D holemin  42.1 mm

Largest pin pin max  Di  0.05 D pin max  31.4 mm


23. The weight of the springs active coils is found from equation 14.11b
3
Weight density ρ  0.285  lbf  in

2 2
π  d  D Na ρ
Weight Wa  Wa  0.188  lbf
4

24. The natural frequency of this spring is found from equation 14.11a and is:
© 2011 Pearson Education, Inc., Upper Saddle River, NJ. All rights reserved. This publication is protected by Copyright and written permission should be
MACHINE DESIGN - An Integrated Approach, 4th Ed. 14-20-4

1 k g
Natural frequency fn   fn  95.02  Hz
2 Wa

25. The ratio between the natural frequency and the forcing frequency is

ωf
Forcing frequency ff  ff  4.167  Hz
2 π
fn
Frequency ratio  22.8
ff

which is sufficiently high.

26. We now have a complete design specification for this A228 wire spring:

Wire diameter d  3.5 mm


Outside diameter Do  40.3 mm
Total coils Nt  11.75 ends squared
Free length Lf  122  mm

25. The initial and working forces are


Initial force Finit  k yinit Finit  135  N

Working force Fwork  k  yinit  ∆y  Fwork  234  N

26. Safety factors:

S ys
Yielding Nys  Nys  1.8
τshut

S es  S us  τi
Fatigue Nfs  Nfs  1.6
S es  τm  τi  S us τa

fn
Surging Nsurge  Nsurge  23
ff

© 2011 Pearson Education, Inc., Upper Saddle River, NJ. All rights reserved. This publication is protected by Copyright and written permission should be
MACHINE DESIGN - An Integrated Approach, 4th Ed. 14-21-1
PROBLEM 14-21
Statement: Design a helical compression spring for a static load of 400 N at a deflection of 45 mm with a
safety factor of 2.5. Use C = 8. Specify all parameters necessary to manufacture the spring.
Given: Working force Fwork  400  N Shear modulus G  80.8 GPa
Working deflection ywork  45 mm Safety factor Ns  2.5
Spring index C  8
Design choices:
Clash allowance α  0.15 Set removed Km  0.65
ASTM A228 wire A  2153.5 MPa
b  0.1625

Solution: See Mathcad file P1421.

Fwork N
1. Determine the desired spring rate. k  k  8.889 
ywork mm

2. Use the design equation from Example 14-3A (Mathcad Supplement) to determine the wire diameter.
1
2 b
 8 Ns ( C  0.5)  Fwork  ( 1  α)
d   mm
 π Km A  mm
2 
 
Wire diameter d  4.789  mm Let d  5  mm

3. Calculate the mean coil diameter and number of active coils.


Mean coil dia D  C d D  40 mm
4
Number of active d G
coils Na  Na  11.096
3
8 D  k Na  11

Note that we round it to the nearest 1/4 coil as the manufacturing tolerance cannot achieve better than that
accuracy. We must now calculate the actual (corrected) spring rate:
4
Corrected spring d G N
rate k  k  8.967 
3 mm
8  D  Na
4. Assume squared and ground ends making the total number of coils, from Figure 14-9:

Total coils Nt  Na  2 Nt  13.00

5. The shut height can now be determined.


Shut height Ls  d  Nt Ls  65 mm
6. The free length (see Figure 14-8) can now be found from
Deflection
to shut height yshut  ywork  α ywork yshut  51.75  mm

Free length Lf  Ls  yshut Lf  116.75 mm

7. To check for buckling, two ratios need to be calculated, Lf/D and y max/Lf.
Lf
Slenderness ratio sr  sr  2.919
D
© 2011 Pearson Education, Inc., Upper Saddle River, NJ. All rights reserved. This publication is protected by Copyright and written permission should be
MACHINE DESIGN - An Integrated Approach, 4th Ed. 14-21-2

ywork
Deflection ratio y'  y'  0.385
Lf

Take these two values to Figure 14-14 and find that their coordinates are safely within the zones that are
stable against buckling for either end-condition case.
8. The inside and outside coil diameters are
Inside coil dia Di  D  d Di  35 mm
Outside coil dia Do  D  d Do  45 mm
9. The smallest hole and largest pin that should be used with this spring are
Smallest hole holemin  Do  0.05 D holemin  47.00  mm

Largest pin pin max  Di  0.05 D pin max  33.00  mm


10. The total weight of the spring is
3
Weight density ρ  0.28 lbf  in

2 2
π  d  D Nt ρ
Weight Wt  Wt  2.44 N
4

11. We now have a complete design specification for this A228 wire spring:

Wire diameter d  5  mm
Outside diameter Do  45 mm
Total coils Nt  13.00 ends squared and ground
Free length Lf  116.75 mm

© 2011 Pearson Education, Inc., Upper Saddle River, NJ. All rights reserved. This publication is protected by Copyright and written permission should be
MACHINE DESIGN - An Integrated Approach, 4th Ed. 14-22-1
PROBLEM 14-22
Statement: Design a straight-ended helical torsion spring for a static load 200 N-m at a deflection of 45 deg
with a safety factor of 1.8. Specify all parameters necessary to manufacture the spring. State all
assumptions.
Given: Applied moment M  200  N  m Bending modulus E  206.8  GPa
Deflection at load θ  45 deg θ  0.125  rev
Design Choices:
Use unpeened oil tempered wire with 40-mm-long, straight ends. The coil is loaded to close it.
Design safety factor Nyd  1.8 Spring index C  11
Yield strength factor Ks  0.85 (from Table 14-13)
Length of ends L1  40 mm L2  40 mm
Material strength factors A  1831.2 MPa b  0.1833 (ASTM A229 wire)

Solution: See Mathcad file P1422.


1. Calculate the Wahl bending factors for inside surface.
2
4 C  C  1
Kbi  Kbi  1.073
4  C ( C  1 )

2. Solve for d using the static yield criterion.


1
3 b
 32 Kbi Nyd  M 
d    mm d  16.144 mm
 π Ks A  mm3 
 
3. Use a wire diameter from the available sizes in Table 14-2. Calculate the mean coil diameter D from equation
14.5.
Wire diameter d  16 mm
Spring index C  11
Mean coil diameter D  C d D  176  mm

4. Calculate the maximum compressive stress in the coil at the inner surface.
32 M
σimax  Kbi σimax  533.5  MPa
3
π d
5. Find the ultimate tensile strength of this oil tempered material from equation 14.3 and Table 14-4 and use it to
find the bending yield strength from Table 14-13, assuming stress relieving.
b
S ut  A   
Ultimate tensile d
strength  S ut  1102 MPa
 
mm

Bending yield
strength S y  Ks S ut S y  936  MPa

Sy
6. The realized static safety factor against yielding is Nyb  Nyb  1.8
σimax

7. The spring rate is defined from the two specified moments at their relative deflection.
M N m
Spring rate k  k  1600
θ rev
8. To get the defined spring rate, the number of active coils must satisfy equation 14.28, solving for Na yields:
© 2011 Pearson Education, Inc., Upper Saddle River, NJ. All rights reserved. This publication is protected by Copyright and written permission should be
MACHINE DESIGN - An Integrated Approach, 4th Ed. 14-22-2

4
Number of active d E
coils Na  Na  4.46
10.8 D k rev

Note that to force k to be in units of N-m per rev we must multiply k by rev.

9. The ends contribute to the active coils from equation 14.26a as

L1  L2
Ne  Ne  0.05
3  π D

and, from equation 14.26b, the number of body coils in the spring are Nb  Na  Ne Nb  4.4

10. Check the angular deflection at the specified load from equation 14.27c.

M  D Na
θ  10.8  rev θ  45.0 deg
4
d E

© 2011 Pearson Education, Inc., Upper Saddle River, NJ. All rights reserved. This publication is protected by Copyright and written permission should be
MACHINE DESIGN - An Integrated Approach, 4th Ed. 14-23-1
PROBLEM 14-23
Statement: Design a straight-ended helical torsion spring for a static load 300 in-lb at a deflection of 75 deg
with a safety factor of 2. Specify all parameters necessary to manufacture the spring. State all
assumptions.
6
Given: Applied moment M  300  in lbf Bending modulus E  30 10  psi
Deflection at load θ  75 deg θ  0.208  rev

Design Choices:
Use unpeened music wire with 2-in-long, straight ends. The coil is loaded to close it.
Design safety factor Nyd  2 Spring index C  7.5
Yield strength factor Ks  1 (from Table 14-13)
Length of ends L1  2  in L2  2  in
Material strength factors A  184.65 ksi b  0.1625 (ASTM A228 wire)

Solution: See Mathcad file P1423.


1. Calculate the Wahl bending factors for inside surface.
2
4 C  C  1
Kbi  Kbi  1.110
4  C ( C  1 )

2. Solve for d using the static yield criterion.


1
3 b
 32 Kbi Nyd  M 
d    in d  0.312  in
 π Ks A  in3 
 
3. Use a wire diameter from the available sizes in Table 14-2. Calculate the mean coil diameter D from
equation 14.5.
Wire diameter d  0.312  in
Spring index C  7.5
Mean coil diameter D  C d D  2.34 in

4. Calculate the maximum compressive stress in the coil at the inner surface.
32 M
σimax  Kbi σimax  111.7  ksi
3
π d
5. Find the ultimate tensile strength of this oil tempered material from equation 14.3 and Table 14-4 and use it to
find the bending yield strength from Table 14-13, assuming stress relieving.
b
S ut  A   
Ultimate tensile d
strength  S ut  223  ksi
 
in

Bending yield
strength S y  Ks S ut S y  223  ksi

Sy
6. The realized static safety factor against yielding is Nyb  Nyb  2.0
σimax

7. The spring rate is defined from the two specified moments at their relative deflection.
M in lbf
Spring rate k  k  1440
θ rev

© 2011 Pearson Education, Inc., Upper Saddle River, NJ. All rights reserved. This publication is protected by Copyright and written permission should be
MACHINE DESIGN - An Integrated Approach, 4th Ed. 14-23-2

8. To get the defined spring rate, the number of active coils must satisfy equation 14.28, solving for Na yields:
4
Number of active d E
coils Na  Na  7.81
10.8 D k rev

Note that to force k to be in units of N-m per rev we must multiply k by rev.

9. The ends contribute to the active coils from equation 14.26a as

L1  L2
Ne  Ne  0.18
3  π D

and, from equation 14.26b, the number of body coils in the spring are Nb  Na  Ne Nb  7.6

10. Check the angular deflection at the specified load from equation 14.27c.

M  D Na
θ  10.8  rev θ  75.0 deg
4
d E

© 2011 Pearson Education, Inc., Upper Saddle River, NJ. All rights reserved. This publication is protected by Copyright and written permission should be
MACHINE DESIGN - An Integrated Approach, 4th Ed. 14-24-1
PROBLEM 14-24
Statement: Design a straight-ended helical torsion spring for a dynamic load 50-105 N-m at a deflection of
80 deg with a safety factor of 2. Specify all parameters necessary to manufacture the spring.
State all assumptions.
Given: Minimum moment Mmin  50 N  m Bending modulus E  206.8  GPa
Maximum moment Mmax  105  N  m Deflection at load ∆θ  80 deg

Design Choices:
Use unpeened oil tempered wire with 40-mm-long, straight ends. The coil is loaded to close it.
Design safety factor Nd  2.5
Yield strength factor Ks  1 (from Table 14-13)
Length of ends L1  40 mm L2  40 mm
Material strength factors A  1831.2 MPa b  0.1833 (ASTM A229 wire)
Wire endurance limit S ewb  537  MPa Spring index C  8.3
Solution: See Mathcad file P1424.
1. Find the mean and alternating loads.
Mmax  Mmin
Ma  Ma  27.5 N  m
2
Mmax  Mmin
Mm  Mm  77.5 N  m
2
2. Calculate the Wahl bending factors for inside and outside surfaces.
2
4 C  C  1
Kbi  Kbi  1.099
4  C ( C  1 )
2
4 C  C  1
Kbo  Kbo  0.916
4  C ( C  1 )
3. Solve for d using the static yield criterion. Use this value of d for starting the iteration in the next step.
1
3 b
 32 Kbi Nd  Mmax 
d    mm d  13.739 mm
 π Ks A  mm3 
 
4. Solve for d by iteration using the fatigue criterion.
1
3 b
 
b  
A  
 d  

   N  M   mm   M  
 32 Kbo  fb a 0.5 S ewb min 
d=

 π A 
1
3 b
 
b  
A  
d 
  
 32 Kbo Nd  Ma  mm   Mmin 
  0.5 S ewb 
RHS( d )  
   mm
3 
 π A  mm 
© 2011 Pearson Education, Inc., Upper Saddle River, NJ. All rights reserved. This publication is protected by Copyright and written permission should be
MACHINE DESIGN - An Integrated Approach, 4th Ed. 14-24-2

d  RHS( d ) d  14.120 mm
d  RHS( d ) d  14.099 mm
d  RHS( d ) d  14.100 mm

Since this results in a slightly larger wire diameter, the fatigue criterion is governing and the spring must be
designed for fatigue.
3. Use a wire diameter from the available sizes in Table 14-2. Calculate the mean coil diameter D from equation 14
Wire diameter d  15 mm
Spring index C  8.3
Mean coil diameter D  C d D  124.5  mm

4. Calculate the maximum compressive stress in the coil at the inner surface.

32 Mmax
σimax  Kbi σimax  348.1  MPa
3
π d
5. Calculate the maximum, minimum, alternating, and mean tensile stresses in the coil at the outer surface.

32 Mmin
σomin  Kbo σomin  138.2  MPa
3
π d
32 Mmax
σomax  Kbo σomax  290.3  MPa
3
π d
σomax  σomin
σm  σm  214.3  MPa
2

σomax  σomin
σa  σa  76.0 MPa
2

6. Find the ultimate tensile strength of this oil tempered material from equation 14.3 and Table 14-4 and use it
to find the bending yield strength from Table 14-13, assuming stress relieving.
b
S ut  A   d
Ultimate tensile strength  S ut  2017 MPa
 
in

Bending yield strength S y  Ks S ut S y  2017 MPa


7. Convert the wire bending endurance limit for unpeened springs from equation 14.33 to fully reversed
endurance strength with equation 14.34b.
S ewb S ut
Fully reversed S e  0.5 S e  309.74 MPa
endurance limit S ut  0.5 S ewb
8. The fatigue safety factor for the coils in bending is calculated from equation 14.34a.

Fatigue factor S e  S ut  σomin


Nfb  Nfb  3.3
of safety S e  σm  σomin  S ut σa

9. The static safety factor against yielding is


Static factor Sy
of safety Nyb  Nyb  5.8
σimax
© 2011 Pearson Education, Inc., Upper Saddle River, NJ. All rights reserved. This publication is protected by Copyright and written permission should be
MACHINE DESIGN - An Integrated Approach, 4th Ed. 14-24-3

These are both acceptable safety factors.

10. The spring rate is defined from the two specified moments at their relative deflection.

Mmax  Mmin N m
Spring rate k  k  247.5 
∆θ rev

11. To get the defined spring rate, the number of active coils must satisfy equation 14.28, solving for Na yields:

4
Number of active d E
coils Na  Na  31.46
10.8 D k rev

Note that to force k to be in units of N-m per rev we must multiply k by rev.

12. The ends contribute to the active coils from equation 14.26a as

L1  L2
Ne  Ne  0.07
3  π D

and, from equation 14.26b, the number of body coils in the spring are Nb  Na  Ne Nb  31

13. The angular deflections at the specified loads from equation 14.27c are

Mmin D Na
θmin  10.8  rev θmin  73 deg
4
d E

Mmax D Na
θmax  10.8  rev θmax  153  deg
4
d E

© 2011 Pearson Education, Inc., Upper Saddle River, NJ. All rights reserved. This publication is protected by Copyright and written permission should be
MACHINE DESIGN - An Integrated Approach, 4th Ed. 14-25-1
PROBLEM 14-25
Statement: Design a straight-ended helical torsion spring for a dynamic load 150-350 in-lb over a
deflection of 50 deg with a safety factor of 1.4. Specify all parameters necessary to
manufacture the spring. State all assumptions.
6
Given: Minimum moment Mmin  150  in lbf Bending modulus E  30 10 psi
Maximum moment Mmax  350  in lbf Deflection at load ∆θ  50 deg

Design Choices:
Use unpeened music wire with 2-in-long, straight ends. The coil is loaded to close it.
Design safety factor Nd  1.4
Yield strength factor Ks  0.80 (from Table 14-13)
Length of ends L1  2  in L2  2  in
Material strength factors A  184.65 ksi b  0.1625 (ASTM A228 wire)
Wire endurance limit S ewb  77.99  ksi Spring index C  4.5

Solution: See Mathcad file P1425.


1. Find the mean and alternating loads.
Mmax  Mmin
Ma  Ma  100.0  in lbf
2
Mmax  Mmin
Mm  Mm  250.0  in lbf
2
2. Calculate the Wahl bending factors for inside and outside surfaces.
2
4 C  C  1
Kbi  Kbi  1.198
4  C ( C  1 )
2
4 C  C  1
Kbo  Kbo  0.854
4  C ( C  1 )
3. Solve for d using the static yield criterion.
1
3 b
 32 Kbi Nd  Mmax 
d    in d  0.323  in
 π Ks A  in3 
 
4. Solve for d by iteration using the fatigue criterion.
1
3 b
 
b  
A  
 d  

   N  M   mm   M  
 32 Kbo  d a 0.5 S ewb min 
d=

 π A 
1
3 b
  b  
A   
d
 
 32 Kbo Nd  Ma  in   Mmin 
  0.5 S ewb 
RHS( d )  
   in
3 
 π A  in 
© 2011 Pearson Education, Inc., Upper Saddle River, NJ. All rights reserved. This publication is protected by Copyright and written permission should be
MACHINE DESIGN - An Integrated Approach, 4th Ed. 14-25-2

d  RHS( d ) d  0.334  in
d  RHS( d ) d  0.334  in

Since this results in a slightly larger wire diameter, the fatigue criterion is governing and the spring must be
designed for fatigue.
3. Use a wire diameter from the available sizes in Table 14-2. Calculate the mean coil diameter D from equation 14
Wire diameter d  0.343  in
Spring index C  4.5
Mean coil diameter D  C d D  1.544  in

4. Calculate the maximum compressive stress in the coil at the inner surface.

32 Mmax
σimax  Kbi σimax  105.9  ksi
3
π d
5. Calculate the maximum, minimum, alternating, and mean tensile stresses in the coil at the outer surface.

32 Mmin
σomin  Kbo σomin  32.3 ksi
3
π d
32 Mmax
σomax  Kbo σomax  75.4 ksi
3
π d
σomax  σomin
σm  σm  53.9 ksi
2

σomax  σomin
σa  σa  21.5 ksi
2

6. Find the ultimate tensile strength of this oil tempered material from equation 14.3 and Table 14-4 and use it to
find the bending yield strength from Table 14-13, assuming stress relieving.
b
S ut  A   
d
Ultimate tensile strength  S ut  220  ksi
 
in

Bending yield strength S y  Ks S ut S y  176  ksi

7. Convert the wire bending endurance limit for unpeened springs from equation 14.33 to fully reversed
endurance strength with equation 14.34b.

S ewb S ut
Fully reversed S e  0.5 S e  47.41  ksi
endurance limit S ut  0.5 S ewb

8. The realized fatigue safety factor for the coils in bending is calculated from equation 14.34a.

Fatigue factor S e  S ut  σomin


Nfb  Nfb  1.5
of safety S e  σm  σomin  S ut σa

9. The static safety factor against yielding is

© 2011 Pearson Education, Inc., Upper Saddle River, NJ. All rights reserved. This publication is protected by Copyright and written permission should be
MACHINE DESIGN - An Integrated Approach, 4th Ed. 14-25-3

Static factor Sy
of safety Nyb  Nyb  1.7
σimax

These are both acceptable safety factors.

10. The spring rate is defined from the two specified moments at their relative deflection.

Mmax  Mmin in lbf


Spring rate k  k  1440
∆θ rev

11. To get the defined spring rate, the number of active coils must satisfy equation 14.28, solving for Na yields:

4
Number of active d E
coils Na  Na  17.30
10.8 D k rev

Note that to force k to be in units of in-lb per rev we must multiply k by rev.

12. The ends contribute to the active coils from equation 14.26a as
L1  L2
Ne  Ne  0.27
3  π D

and, from equation 14.26b, the number of body coils in the spring are Nb  Na  Ne Nb  17

13. The angular deflections at the specified loads from equation 14.27c are

Mmin D Na
θmin  10.8  rev θmin  38 deg
4
d E

Mmax D Na
θmax  10.8  rev θmax  88 deg
4
d E

© 2011 Pearson Education, Inc., Upper Saddle River, NJ. All rights reserved. This publication is protected by Copyright and written permission should be
MACHINE DESIGN - An Integrated Approach, 4th Ed. 14-26-1
PROBLEM 14-26
Statement: Design a Belleville spring to give a constant 400 N ±10% force over a 1-mm deflection.
Given: Nominal force Fflat  400  N Working deflection ywork  1  mm

Design Choices:
6
The diameter ratio is Rd  2. Use unset carbon spring steel 50HRC. Properties, E  30 10  psi ,
ν  0.28 . The outside diameter of the spring is Do  39.55  mm
Solution: See Mathcad file P1426.
1. Since a constant force spring is needed, the h/t ratio (see Figure 14-30) is
h/t ratio hovert  1.414

2. The required force variation of not more than ±10% can be met by choosing an appropriate deflection range to
operate in from Figure 14-31. If the deflection is kept between about 53% and 146% of the flat deflection, this
tolerance will be achieved. The nominal force will then occur at the flat position and the spring must operate
on both sides, so must be mounted in similar fashion to that shown in Figure 14-33.

3. Use the above assumptions and the specified nominal force in equation 14.37a to find an appropriate spring
thickness t:
1
4
 F Do 
2
1  flat

t    t  0.760  mm Let t  0.760  mm
10  132.4  MPa hovert 

4. The height h can now be found: h  hovert t h  1.075  mm


5. Based on the choices in step 2, find the minimum and maximum deflections:
ymin  0.53 h ymin  0.57 mm
ymax  1.46 h ymax  1.57 mm

The difference between these distances is equal to the required deflection range, so the force tolerance can be
met over that range.
6. Figure 14-34 shows that the worst stress state will occur at the largest deflection ymax, so solve equations 14.36
for stresses at that deflection:
  Rd  1  2
 
6
K1  K1  0.689
π ln Rd   R2 
 d 
6  Rd  1 
K2    1 K2  1.22
π ln Rd   ln Rd  
6  Rd  1 
K3    K3  1.378
π ln Rd   2 

 Rd  ln Rd    Rd  1  Rd 
K4   ln Rd 
 K4  1.115
    Rd  1  2
Rd
K5  K5  1
2   Rd  1 

© 2011 Pearson Education, Inc., Upper Saddle River, NJ. All rights reserved. This publication is protected by Copyright and written permission should be
MACHINE DESIGN - An Integrated Approach, 4th Ed. 14-26-2

4  E ymax   ymax  
σc    K2  h    K3 t σc  1831 MPa
2 
K1 Do  1  ν 
2   2  

 4  E ymax   ymax  
σti     K2  h    K3 t σti  906.1  MPa

 K1 Do2 1  ν2
   

2  

 4  E ymax   ymax  
σto     K4  h    K5 t σto  1416 MPa

 K1 Do2 1  ν2
   

2  

7. Table 14-5 gives S ut  246  ksi for this material. Table 14-15 indicates that 120% of this value can be used
for an unset spring. The safety factor for static loading is then

1.20 S ut
Ns  Ns  1.1
σc

which is acceptable.

8. A summary of the spring design is

Outside diameter Do  39.55  mm

Do
Inside diameter Di  Di  19.77  mm
Rd

Thickness t  0.76 mm

Height h  1.07 mm

© 2011 Pearson Education, Inc., Upper Saddle River, NJ. All rights reserved. This publication is protected by Copyright and written permission should be
MACHINE DESIGN - An Integrated Approach, 4th Ed. 14-27-1
PROBLEM 14-27
Statement: Design a Belleville spring for bimodal operation between ±50 N.
Given: Nominal force Fflat  50 N

Design Choices:
The diameter ratio is Rd  2. Use unset carbon spring steel 50HRC. Properties:
6
E  30 10  psi, ν  0.28 . The outside diameter of the spring is Do  24.50  mm

Solution: See Mathcad file P1427.


1. Since a bimodal force spring is needed, the h/t ratio (see Figure 14-30) is
h/t ratio hovert  2.828
2. The required force variation of not more than ±100% can be met by choosing an appropriate deflection range
to operate in from Figure 14-30. If the deflection is kept between about 0% and 200% of the flat deflection,
this will be achieved. A zero force will then occur at the flat position and the spring must operate on both
sides, so must be mounted in similar fashion to that shown in Figure 14-32.

3. Use the above assumptions and the specified nominal force in equation 14.37a to find an appropriate spring
thickness t:
1
4
 Do 
2
t 
 Fflat
 t  0.299  mm Let t  0.30 mm
 7 hovert 
 19.2  10  psi 
4. The height h can now be found: h  hovert t h  0.848  mm
5. Based on the choices in step 2, find the minimum and maximum deflections:
ymin  0  h ymin  0  mm
ymax  2.00 h ymax  1.70 mm

The difference between these distances is equal to the required deflection range, so
Working deflection ywork  ymax ywork  1.70 mm

6. Figure 14-34 shows that the worst stress state will occur at the largest deflection ymax, so solve equations
14.36 for stresses at that deflection:
  Rd  1  2
 
6
K1  K1  0.689
π ln Rd   R2 
 d 
6  Rd  1 
K2    1 K2  1.22
π ln Rd   ln Rd  
6  Rd  1 
K3    K3  1.378
π ln Rd   2 

 Rd  ln Rd    Rd  1  Rd 
K4   ln Rd 
 K4  1.115
    Rd  1  2
 
Rd
K5  K5  1
2   Rd  1 
© 2011 Pearson Education, Inc., Upper Saddle River, NJ. All rights reserved. This publication is protected by Copyright and written permission should be
MACHINE DESIGN - An Integrated Approach, 4th Ed. 14-27-2

4  E ymax   ymax  
σc    K2  h    K3 t σc  1523 MPa
2 
K1 Do  1  ν 
2   2  

 4  E ymax   ymax  
σti     K2  h    K3 t σti  1523 MPa

 K1 Do2 1  ν2
   

2  

 4  E ymax   ymax  
σto     K4  h    K5 t σto  1105 MPa

 K1 Do2 1  ν2
   

2  

7. Table 14-5 gives S ut  246  ksi for this material. Table 14-15 indicates that 120% of this value can be used
for
an unset spring. The safety factor for static loading is then
1.20 S ut
Ns  Ns  1.3
σc

which is acceptable.
8. A summary of the spring design is

Outside diameter Do  24.5 mm

Do
Inside diameter Di  Di  12.25  mm
Rd

Thickness t  0.3 mm

Height h  0.85 mm

© 2011 Pearson Education, Inc., Upper Saddle River, NJ. All rights reserved. This publication is protected by Copyright and written permission should be
MACHINE DESIGN - An Integrated Approach, 4th Ed. 14-28-1

PROBLEM 14-28
Statement: Given the data below for a helical compression spring loaded in fatigue, design the spring for
infinite life. State all assumptions and empirical data used.
Given: Minimum force Fmin  225  N Forcing freq. ωf  625  rpm
Maximum force Fmax  450  N Shear modulus G  79.3 GPa
Working deflection ∆y  20 mm Clash factor α  0.15
Music wire properties: Spring index C  8.5
Strength A  2153.5 MPa b  0.1625 Set after winding, unpeened
Wire diameter d  8  mm
Solution: See Mathcad file P1428.
1. Find the the mean and alternating forces from equation 14.15a:

Fmax  Fmin
Alternating force Fa  Fa  112.5  N
2
Fmax  Fmin
Mean force Fm  Fm  337.5  N
2

2. Calculate the factors necessary to find the fatigue factor of safety.


0.5
Direct shear Ks  1  Ks  1.059
factor C

4 C  1 0.615
Wahl factor Kw   Kw  1.172
4 C  4 C
Yield strength
factor Kys  0.60 (Table 14-6, set removed)

Ultimate shear
strength factor KU  0.67 (equation 14.4)

Torsional
endurance limit S ew  310  MPa (unpeened)

3. Calculate the ultimate tensile strength, ultimate shear strength, and the shear endurance limit.
b
S ut  A    d
Ultimate tensile strength  S ut  1536 MPa
 
mm

Ultimate shear strength S us  KU  S ut S us  1029 MPa

Shear yield strength S ys  Kys S ut S ys  922  MPa

S ew S us
Fatigue strength S es  0.5 S es  182.5  MPa
S us  0.5 S ew

4. Calculate the mean coil diameter D from equation 14.5.


Mean coil diameter D  C d D  68.0 mm
5. Calculate the shear stress i at the initial deflection (lowest defined force), and the mean stress m.

8  Fmin D
Stress at Fmin τi  Ks τi  80.6 MPa
3
π d
© 2011 Pearson Education, Inc., Upper Saddle River, NJ. All rights reserved. This publication is protected by Copyright and written permission should be
MACHINE DESIGN - An Integrated Approach, 4th Ed. 14-28-2

8  Fm D
Stress at Fm τm  Ks τm  120.9  MPa
3
π d

6. Calculate the alternating shear stress a in the coil.

8  F a D
Alternating stress τa  Kw τa  44.6 MPa
3
π d

7. The safety factor is calculated from equation 14.16b.

Fatigue factor S es  S us  τi


Nfs  Nfs  3.3
of safety S es  τm  τi  S us τa

8. The spring rate is defined in this problem because of the two specified forces at a particular relative deflection.

Fmax  Fmin N
Spring rate k  k  11.25 
∆y mm

9. To get the defined spring rate, the number of active coils must satisfy equation 14.7, solving for Na yields:
4
Number of active d G
coils Na  Na  11.478 Na  11.5
3
8 D  k

Note that we round it to the nearest 1/4 coil as the manufacturing tolerance cannot achieve better than that
accuracy. Having rounded the number of active coils, we must now calculate the spring rate using equation 14.
4
Corrected d G N
spring rate k  k  11.23 
3 mm
8  D  Na
10. For squared ends the total number of coils, from Figure 14-9:
Total coils Nt  Na  2 Nt  13.50

11. The shut height can now be determined.


Shut height Lshut  d  Nt Lshut  108  mm

12. The initial deflection to reach the smaller of the two loads is
Fmin
Initial deflection yinit  yinit  20.0 mm
k
13. For the given clash allowance factor:

Clash allowance ∆yclash  α ∆y ∆yclash  3.0 mm

14. The free length (see Figure 14-8) can now be found from
Lf  Lshut  ∆yclash  ∆y  yinit Lf  151  mm

15. The deflection to the shut height is


yshut  Lf  Lshut yshut  43.0 mm

16. The force at this shut height is


Fshut  k yshut Fshut  483.3  N
© 2011 Pearson Education, Inc., Upper Saddle River, NJ. All rights reserved. This publication is protected by Copyright and written permission should be
MACHINE DESIGN - An Integrated Approach, 4th Ed. 14-28-3

17. The shut-height stress and safety factor are


8  Fshut D
Stress at Fshut τshut  Ks τshut  173.1  MPa
3
π d

Safety factor at S ys
shut height Ns  Ns  5.3
τshut
which is acceptable.

18. To check for buckling, two ratios need to be calculated, Lf/D and ymax /Lf.

Lf
Slenderness ratio sr  sr  2.221
D
yinit  ∆y
Deflection ratio y'  y'  0.265
Lf

Take these two values to Figure 14-14 and find that their coordinates are safely within the zones that are stable
against buckling for both cases.

19. The inside and outside coil diameters are


Inside coil dia Di  D  d Di  60 mm
Outside coil dia Do  D  d Do  76 mm
20. The smallest hole and largest pin that should be used with this spring are
Smallest hole holemin  Do  0.05 D holemin  79.4 mm

Largest pin pin max  Di  0.05 D pin max  56.6 mm

21. The weight of the springs active coils is found from equation 14.11b
3
Weight density ρ  0.285  lbf  in

2 2
π  d  D Na ρ
Weight Wa  Wa  9.6 N
4

22. The natural frequency of this spring is found from equation 14.11a and is:

1 k g
Natural frequency fn   fn  53.68  Hz
2 Wa

23. The ratio between the natural frequency and the forcing frequency is

ωf
Forcing frequency ff  ff  10.42  Hz
2 π
fn
Frequency ratio  5.2
ff

which is a little low.

© 2011 Pearson Education, Inc., Upper Saddle River, NJ. All rights reserved. This publication is protected by Copyright and written permission should be
MACHINE DESIGN - An Integrated Approach, 4th Ed. 14-28-4

24. We now have a complete design specification for this A228 wire spring:

Wire diameter d  8  mm
Outside diameter Do  76.0 mm
Total coils Nt  13.50 ends squared
Free length Lf  151.038  mm

25. The initial and working forces are


Initial force Finit  k yinit Finit  225  N

Working force Fwork  k  yinit  ∆y  Fwork  450  N

26. Safety factors:

S ys
Yielding Nys  Nys  5.3
τshut

S es  S us  τi
Fatigue Nfs  Nfs  3.3
S es  τm  τi  S us τa

fn
Surging Nsurge  Nsurge  5.2
ff

© 2011 Pearson Education, Inc., Upper Saddle River, NJ. All rights reserved. This publication is protected by Copyright and written permission should be
MACHINE DESIGN - An Integrated Approach, 4th Ed. 14-29-1
PROBLEM 14-29
Statement: A helical extension spring, loaded in fatigue, has been designed for infinite life with the data
given below. Find the safety factors for failure in the standard hooks. State all assumptions
and sources of empirical data.

Given: Minimum force Fmin  665  N Life L  ∞


Maximum force Fmax  935  N Shear modulus G  79.3 GPa
Working deflection ∆y  2.00 in Spring index C  9
Wire endurance limit S ew  310  MPa Wire strength A  2059.2 MPa
Wire diameter d  8  mm (Chrome-silicon) b  0.0934
Number active coils Na  13.75

Solution: See Mathcad file P1429.


1. Find the mean and alternating loads.
Fmax  Fmin
Fa  Fa  135.0  N
2
Fmax  Fmin
Fm  Fm  800.0  N
2

2. Calculate the hook bending factor.


2
4 C  C  1
Kb  Kb  1.090
4 C ( C  1)

3. Calculate the mean coil diameter.


Mean coil diameter D  C d D  72 mm
4. Use the value of C to find an appropiate value of initial coil stress i from equations 14.21:

 3 2 
τi1  4.231  C  181.5  C  3387 C  28640  psi τi1  67.4 MPa

  2.987  C  139.7  C  3427 C  38404   psi


3 2
τi2 τi2  115.1  MPa

τi1  τi2
τi  τi  91.3 MPa
2

5. Find the direct shear factor Ks :


0.5
Direct shear factor Ks  1  Ks  1.056
C

6. Use the direct shear factor Ks and previously assumed values to find the mean stress m:

8  Fm D
Stress at Fm τm  Ks τm  302.4  MPa
3
π d

7. Find the Wahl factor Kw and use it to calculate the alternating shear stress a in the coil.

4 C  1 0.615
Wahl factor Kw   Kw  1.162
4 C  4 C
8  F a D
Alternating stress τa  Kw τa  56.2 MPa
3
π d
© 2011 Pearson Education, Inc., Upper Saddle River, NJ. All rights reserved. This publication is protected by Copyright and written permission should be
MACHINE DESIGN - An Integrated Approach, 4th Ed. 14-29-2

8. Find the ultimate tensile strength of this wire material from equation 14.3 and Table 14-4 and use it to find the
ultimate shear strength from equation 14.4 and the torsional yield strength for the coil body from Table 14-10,
assuming no set removal.
b
S ut  A   
d
Ultimate tensile strength  S ut  1695.7 MPa
 mm 
Shear yield strength S ys  0.50 S ut S ys  847.9  MPa

Ultimate shear strength S us  0.67 S ut S us  1136.1 MPa

9. Find the wire endurance limit for unpeened springs from equation 14.13 and convert it to fully reversed
endurance strength with equation 14.16c.
S ew S us
Fully reversed S es  0.5 S es  179.49 MPa
endurance limit S us  0.5 S ew

10. The stresses in the end hooks also need to be determined. The bending stresses in the hook are found from
equation 14.23:

2  R1 2 D
C1 = = =C C1  C C1  9.00
d 2 d
2
4 C1  C1  1
Kb  Kb  1.09
4  C1  C1  1 

16 D Fa 4  Fa
σa  Kb  σa  108.1  MPa
3 2
π d π d

16 D Fm 4  Fm
σm  Kb  σm  640.6  MPa
3 2
π d π d

16 D Fmin 4  Fmin


σmin  Kb  σmin  532.5  MPa
3 2
π d π d

11. Convert the torsional endurance strength to a tensile endurance strength with the von Mises relationship and
use it and the ultimate tensile strength in equation 14.16 to find a fatigue safety factor for the hook in bending:

S es
S e  S e  311.1  MPa
0.577

S e  S ut  σmin
Nfb  Nfb  1.67
S e  σm  σmin  S ut σa

12. The torsional stresses in the hook are found from equation 14.24 using an assumed value of C2  5.

C2 d
R2  R2  20 mm
2
4  C2  1
Kw2  Kw2  1.188
4  C2  4
© 2011 Pearson Education, Inc., Upper Saddle River, NJ. All rights reserved. This publication is protected by Copyright and written permission should be
MACHINE DESIGN - An Integrated Approach, 4th Ed. 14-29-3

8  Fa D
τBa  Kw2 τBa  57.4 MPa
3
π d
8  Fm D
τBm  Kw2 τBm  340.2  MPa
3
π d
8  Fmin D
τBmin  Kw2 τBmin  282.8  MPa
3
π d

13. The fatigue safety factor for the hook in torsion is calculated from equation 14.16b.

S es  S us  τBmin
Nfs  Nfs  2.0
S es  τBm  τBmin  S us τBa

© 2011 Pearson Education, Inc., Upper Saddle River, NJ. All rights reserved. This publication is protected by Copyright and written permission should be
MACHINE DESIGN - An Integrated Approach, 4th Ed. 14-30-1
PROBLEM 14-30
Statement: Given the following data for a helical torsion spring, loaded in fatigue, find the spring index,
unloaded coil diameter, minimum loaded coil diameter, and safety factor in fatigue. State all
assumptions and sources of empirical data used.
6
Given: Minimum deflection θmin  0.25 rev Bending modulus E  30 10  psi
1
Maximum deflection θmax  0.75 rev Spring rate k  60 in lbf  rev
Number active coils Na  20 Wire dia. d  0.192  in

Design Choices:
Use unpeened music wire with 2-in-long, straight ends. The coil is loaded to close it.
Yield strength factor Ks  0.80 (from Table 14-13)
Length of ends L1  2  in L2  2  in
Material strength factors A  184.65 ksi b  0.1625 (ASTM A228 wire)
Wire endurance limit S ewb  77.99  ksi

Solution: See Mathcad file P1430.


1. Find the minimum, maximum, mean, and alternating loads.
Mmin  k θmin Mmin  15 in lbf

Mmax  k θmax Mmax  45 in lbf

Mmax  Mmin
Ma  Ma  15.0 in lbf
2
Mmax  Mmin
Mm  Mm  30.0 in lbf
2
3
d E
2. Use equation 14.28 to calculate the spring index. k=
10.8 C Na

3
d E
C  C  16.38
10.8 k rev Na

3. Calculate the Wahl bending factors for inside and outside surfaces.
2
4 C  C  1
Kbi  Kbi  1.048
4  C ( C  1 )
2
4 C  C  1
Kbo  Kbo  0.956
4  C ( C  1 )

4. Calculate the mean coil diameter D from equation 14.5.


Mean coil diameter D  C d D  3.146  in
5. Calculate the maximum compressive stress in the coil at the inner surface.

32 Mmax
σimax  Kbi σimax  67.9 ksi
3
π d
6. Calculate the maximum, minimum, alternating, and mean tensile stresses in the coil at the outer surface.
32 Mmin
σomin  Kbo σomin  20.6 ksi
3
π d
© 2011 Pearson Education, Inc., Upper Saddle River, NJ. All rights reserved. This publication is protected by Copyright and written permission should be
MACHINE DESIGN - An Integrated Approach, 4th Ed. 14-30-2

32 Mmax
σomax  Kbo σomax  61.9 ksi
3
π d
σomax  σomin
σm  σm  41.3 ksi
2

σomax  σomin
σa  σa  20.6 ksi
2

7. Find the ultimate tensile strength of this oil tempered material from equation 14.3 and Table 14-4 and use it to
find the bending yield strength from Table 14-13, assuming stress relieving.
b
S ut  A   
d
Ultimate tensile strength  S ut  241  ksi
 
in

Bending yield strength S y  Ks S ut S y  193  ksi

8. Convert the wire bending endurance limit for unpeened springs from equation 14.33 to fully reversed
endurance strength with equation 14.34b.

S ewb S ut
Fully reversed S e  0.5 S e  46.51  ksi
endurance limit S ut  0.5 S ewb

9. The fatigue safety factor for the coils in bending is calculated from equation 14.34a.

Fatigue factor S e  S ut  σomin


Nfb  Nfb  1.7
of safety S e  σm  σomin  S ut σa

L1  L2
10. The ends contribute to the active coils from equation 14.26a as Ne  Ne  0.13
3  π D

and, from equation 14.26b, the number of body coils in the spring are Nb  Na  Ne Nb  20

11. The unloaded coil diameter and minimum loaded coil diameters are:

Unloaded coil diameter Di  D  d Di  2.954  in

D Nb
Loaded coil diameter Dimin  d Dimin  2.839  in
θmax
Nb 
rev

© 2011 Pearson Education, Inc., Upper Saddle River, NJ. All rights reserved. This publication is protected by Copyright and written permission should be
MACHINE DESIGN - An Integrated Approach, 4th Ed. 14-31-1
PROBLEM 14-31
Statement: A helical compression spring is required to provide a minimum force of 150 lb at installation and
have a working deflection of 1 in. The spring rate is 75 lb/in. The coil must fit in a 2.1-in-dia hole
with 0.1-in clearance. Use 0.250-in diameter, unpeened, music wire and squared/ground ends.
Using a 15% clash allowance, find:
(a) The stresses and safety factor for infinte life in fatigue.
(b) The shut height.
(c) The stress and safety factor at shut height.
(d) The total number of coils.
(e) The free length.
(f) The natural frequency in Hz.
(g) Draw a Goodman diagram and show the safety factor from (a) on it.

1
Given: Minimum force Fmin  150  lbf Spring rate k  75 lbf  in
6
Spring OD OD  2.000  in Shear modulus G  11.5 10  psi
Working deflection ∆y  1.000  in Clash factor α  0.15
Music wire properties: Wire diameter d  0.250  in
Strength A  184.65 ksi b  0.1625 Set after winding, unpeened
Solution: See Mathcad file P1431.
1. From the given information, find the maximum force and the spring index.
Maximum force Fmax  Fmin  k ∆y Fmax  225  lbf

OD  d
Spring index C  C7
d
2. Find the the mean and alternating forces from equation 14.15a:

Fmax  Fmin
Alternating force Fa  Fa  37.5 lbf
2
Fmax  Fmin
Mean force Fm  Fm  187.5  lbf
2

3. Calculate the factors necessary to find the fatigue factor of safety.


0.5
Direct shear Ks  1  Ks  1.071
factor C

4 C  1 0.615
Wahl factor Kw   Kw  1.213
4 C  4 C
Yield strength
factor Kys  0.60 (Table 14-6, set removed)

Ultimate shear
strength factor KU  0.67 (equation 14.4)

Torsional
endurance limit S ew  45 ksi (unpeened)

4. Calculate the ultimate tensile strength, ultimate shear strength, and the shear endurance limit.
b
S ut  A   
d
Ultimate tensile strength  S ut  231  ksi
 
in

Ultimate shear strength S us  KU  S ut S us  155  ksi

© 2011 Pearson Education, Inc., Upper Saddle River, NJ. All rights reserved. This publication is protected by Copyright and written permission should be
MACHINE DESIGN - An Integrated Approach, 4th Ed. 14-31-2

Shear yield strength S ys  Kys S ut S ys  139  ksi

S ew S us
Fatigue strength S es  0.5 S es  26.3 ksi
S us  0.5 S ew

5. Calculate the mean coil diameter D from equation 14.5.


Mean coil diameter D  C d D  1.750  in
6. Calculate the shear stress i at the initial deflection (lowest defined force), and the mean stress m.

8  Fmin D
Stress at Fmin τi  Ks τi  45.8 ksi
3
π d
8  Fm D
Stress at Fm τm  Ks τm  57.3 ksi
3
π d

7. Calculate the alternating shear stress a in the coil.

8  F a D
Alternating stress τa  Kw τa  13.0 ksi
3
π d

8. The safety factor is calculated from equation 14.16b.

Fatigue factor S es  S us  τi


Nfs  Nfs  1.2
of safety S es  τm  τi  S us τa

9. To get the defined spring rate, the number of active coils must satisfy equation 14.7, solving for Na yields:

4
Number of active d G
coils Na  Na  13.97 Na  14
3
8 D  k
Note that we round it to the nearest 1/4 coil as the manufacturing tolerance cannot achieve better than that
accuracy. Having rounded the number of active coils, we must now calculate the spring rate using equation 14.

4
Corrected d G lbf
spring rate k  k  74.84 
3 in
8  D  Na

10. For squared ends the total number of coils, from Figure 14-9:
Total coils Nt  Na  2 Nt  16.00

11. The shut height can now be determined.


Shut height Lshut  d  Nt Lshut  4  in
12. The initial deflection to reach the smaller of the two loads is
Fmin
Initial deflection yinit  yinit  2.004  in
k
13. For the given clash allowance factor:
Clash allowance ∆yclash  α ∆y ∆yclash  0.150  in
14. The free length (see Figure 14-8) can now be found from
© 2011 Pearson Education, Inc., Upper Saddle River, NJ. All rights reserved. This publication is protected by Copyright and written permission should be
MACHINE DESIGN - An Integrated Approach, 4th Ed. 14-31-3

Lf  Lshut  ∆yclash  ∆y  yinit Lf  7.154  in

15. The deflection to the shut height is


yshut  Lf  Lshut yshut  3.154  in

16. The force at this shut height is


Fshut  k yshut Fshut  236.1  lbf

17. The shut-height stress and safety factor are


8  Fshut D
Stress at Fshut τshut  Ks τshut  72.1 ksi
3
π d

Safety factor at S ys
shut height Ns  Ns  1.92
τshut
which is acceptable.
18. The inside and outside coil diameters are
Inside coil dia Di  D  d Di  1.5 in
Outside coil dia Do  D  d Do  2  in
19. The smallest hole and largest pin that should be used with this spring are
Smallest hole holemin  Do  0.05 D holemin  2.09 in

Largest pin pin max  Di  0.05 D pin max  1.41 in

20. The weight of the springs active coils is found from equation 14.11b
3
Weight density ρ  0.285  lbf  in

2 2
π  d  D Na ρ
Weight Wa  Wa  1.077  lbf
4

21. The natural frequency of this spring is found from equation 14.11a and is:

1 k g
Natural frequency fn   fn  81.91  Hz
2 Wa

22. We now have results for questions (a) through (f):

(a) Fatigue stresses τi  45.8 ksi τa  13 ksi τm  57.3 ksi


and safety factor Nfs  1.2
(b) Shut height Lshut  4.000  in
(c) Stress at shut height τshut  72.1 ksi
and safety factor Ns  1.9
(d) Total coils Nt  16.00 ends squared and ground
(e) Free length Lf  7.154  in
(f) Natural frequency fn  81.91  Hz

© 2011 Pearson Education, Inc., Upper Saddle River, NJ. All rights reserved. This publication is protected by Copyright and written permission should be
MACHINE DESIGN - An Integrated Approach, 4th Ed. 14-31-4

23. (g) Plot the Goodman diagram.

S es
The equation for the Goodman line is τ'a τ'm    τ'm  S es
S us

τa
Equation for load line τaL τ'm   τ'm
τm

Plotting this over the range τ'm  0  ksi 10 ksi  200  ksi

30
τm
Alternating shear stress

25 ksi

20

15 τa
ksi
10

0
0 25 50 75 100 125 150 175 200

Mean shear stress


The Goodman boundary is the solid line with negative slope. The stress state is at the intersection of the
vertical and horizontal dashed lines. The safety margin is the distance along the load line from the stress
state point to the intersection of the load line with the Goodman boundary.

© 2011 Pearson Education, Inc., Upper Saddle River, NJ. All rights reserved. This publication is protected by Copyright and written permission should be
MACHINE DESIGN - An Integrated Approach, 4th Ed. 14-32-1
PROBLEM 14-32
Statement: A helical compression spring is required to provide a time-varying force that ranges from a
mimimum of 100 lb to a maximum of 300 lb over a deflection of 1 in. It needs to work freely over
shaft of 1.25-in dia. Use a cold-drawn carbon steel wire having an S ut = 250 ksi. A spring index
of 6, a clash allowance of 15%, and squared and ground ends are desired.
1
Given: Minimum force Fmin  100  lbf Spring rate k  75 lbf  in
6
Maximum force Fmax  300  lbf Shear modulus G  11.5 10  psi
Working deflection ∆y  1.000  in Clash factor α  0.15
Wire strength S ut  250  ksi Spring index C  6
Unset after winding, unpeened Shaft OD ODshaft  1.250  in

Solution: See Mathcad file P1432.


1. From the given information, find a trial wire diameter.
Maximum shaft dia ODshaft = IDspring  0.05 D = ( D  d )  0.05 D = C d  d  0.05 C d

ODshaft
Trial wire diameter d  d  0.266  in
0.95 C  1

Standard wire size d  0.343  in (Found by iteration, see note in step 8)

2. Find the the mean and alternating forces from equation 14.15a:

Fmax  Fmin
Alternating force Fa  Fa  100.0  lbf
2
Fmax  Fmin
Mean force Fm  Fm  200.0  lbf
2

3. Calculate the factors necessary to find the fatigue factor of safety.


0.5
Direct shear Ks  1  Ks  1.083
factor C

4 C  1 0.615
Wahl factor Kw   Kw  1.252
4 C  4 C
Yield strength
factor Kys  0.45 (Table 14-6, set removed)

Ultimate shear
strength factor KU  0.67 (equation 14.4)

Torsional
endurance limit S ew  45 ksi (unpeened)

4. Calculate the ultimate tensile strength, ultimate shear strength, and the shear endurance limit.
Ultimate tensile strength S ut  250  ksi

Ultimate shear strength S us  KU  S ut S us  168  ksi

Shear yield strength S ys  Kys S ut S ys  113  ksi

S ew S us
Fatigue strength S es  0.5 S es  26.0 ksi
S us  0.5 S ew
© 2011 Pearson Education, Inc., Upper Saddle River, NJ. All rights reserved. This publication is protected by Copyright and written permission should be
MACHINE DESIGN - An Integrated Approach, 4th Ed. 14-32-2

5. Calculate the mean coil diameter D from equation 14.5.


Mean coil diameter D  C d D  2.058  in
6. Calculate the shear stress i at the initial deflection (lowest defined force), and the mean stress m.

8  Fmin D
Stress at Fmin τi  Ks τi  14.1 ksi
3
π d
8  Fm D
Stress at Fm τm  Ks τm  28.1 ksi
3
π d

7. Calculate the alternating shear stress a in the coil.

8  F a D
Alternating stress τa  Kw τa  16.3 ksi
3
π d

8. The safety factor is calculated from equation 14.16b.

Fatigue factor S es  S us  τi


Nfs  Nfs  1.3
of safety S es  τm  τi  S us τa

This was originally too low. The wire diameter in step 1 was repeatedly changed to get a satisfactory safety
factor.
9. The spring rate is defined in this problem because of the two specified forces at a particular relative deflection.

Fmax  Fmin lbf


Spring rate k  k  200.00
∆y in

10. To get the defined spring rate, the number of active coils must satisfy equation 14.7, solving for Na yields:
4
Number of active d G
coils Na  Na  11.413 Na  11.5
3
8 D  k
Note that we round it to the nearest 1/4 coil as the manufacturing tolerance cannot achieve better than that
accuracy. Having rounded the number of active coils, we must now calculate the spring rate using equation 14.
4
Corrected d G lbf
spring rate k  k  198.50
3 in
8  D  Na

11. For squared ends the total number of coils, from Figure 14-9:
Total coils Nt  Na  2 Nt  13.50

12. The shut height can now be determined.


Shut height Lshut  d  Nt Lshut  4.631  in

13. The initial deflection to reach the smaller of the two loads is
Fmin
Initial deflection yinit  yinit  0.504  in
k

14. For the given clash allowance factor:


Clash allowance ∆yclash  α ∆y ∆yclash  0.150  in
© 2011 Pearson Education, Inc., Upper Saddle River, NJ. All rights reserved. This publication is protected by Copyright and written permission should be
MACHINE DESIGN - An Integrated Approach, 4th Ed. 14-32-3

15. The free length (see Figure 14-8) can now be found from
Lf  Lshut  ∆yclash  ∆y  yinit Lf  6.284  in

16. The deflection to the shut height is


yshut  Lf  Lshut yshut  1.654  in

17. The force at this shut height is


Fshut  k yshut Fshut  328.3  lbf

18. The shut-height stress and safety factor are


8  Fshut D
Stress at Fshut τshut  Ks τshut  46.2 ksi
3
π d

Safety factor at S ys
shut height Ns  Ns  2.4
τshut
which is acceptable.
19. The inside and outside coil diameters are
Inside coil dia Di  D  d Di  1.715  in
Outside coil dia Do  D  d Do  2.401  in
20. The smallest hole and largest pin that should be used with this spring are
Smallest hole holemin  Do  0.05 D holemin  2.50 in

Largest pin pin max  Di  0.05 D pin max  1.61 in

21. The weight of the springs active coils is found from equation 14.11b
3
Weight density ρ  0.285  lbf  in

2 2
π  d  D Na ρ
Weight Wa  Wa  1.958  lbf
4

22. The natural frequency of this spring is found from equation 14.11a and is:

1 k g
Natural frequency fn   fn  98.92  Hz
2 Wa

23. We now have a complete design specification for this cold-drawn wire spring:

Wire diameter d  0.343  in


Outside diameter Do  2.401  in
Total coils Nt  13.50 ends squared and ground
Free length Lf  6.284  in

© 2011 Pearson Education, Inc., Upper Saddle River, NJ. All rights reserved. This publication is protected by Copyright and written permission should be
MACHINE DESIGN - An Integrated Approach, 4th Ed. 14-33-1

PROBLEM 14-33 _____

Statement: Find the ultimate tensile strength and the ultimate shear strength of ASTM A228 music wire in the
following preferred diameters: 0.5 mm, 1.0 mm, 2.0 mm, 4.0 mm, and 6.0 mm.
Given: Wire diameters: i  1 2  5
d  0.5 mm d  1.0 mm d  2.0 mm d  4.0 mm d  6.0 mm
1 2 3 4 5

Solution: See Mathcad file P1433.

1. Calculate the ultimate tensile strength using equation 14.3 and the coefficient and exponent from Table 14-4.
For ASTM A228 music wire, A  2153.5 MPa b  0.1625
b
 di  d
i
S ut
S ut  A    i
 
i  mm  mm GPa
0.5 2.41
1.0 2.15
2.0 1.92
4.0 1.72
6.0 1.61

2. Calculate the ultimate shear strength using equation 14.4.

d S us
i i
S us  0.67 S ut  
i i mm GPa
0.5 1.61
1.0 1.44
2.0 1.29
4.0 1.15
6.0 1.08

© 2011 Pearson Education, Inc., Upper Saddle River, NJ. All rights reserved. This publication is protected by Copyright and written permission should be
MACHINE DESIGN - An Integrated Approach, 4th Ed. 14-34-1

PROBLEM 14-34 _____


Statement: Find the ultimate tensile strength and the ultimate shear strength of ASTM A228 music wire in
the following preferred diameters: 0.020 in, 0.038 in, 0.081 in, 0.162 in, and 0.250 in.
Given: Wire diameters: i  1 2  5
d  0.020  in d  0.038  in d  0.081  in d  0.162  in d  0.250  in
1 2 3 4 5
Solution: See Mathcad file P1434.

1. Calculate the ultimate tensile strength using equation 14.3 and the coefficient and exponent from Table 14-4.
For ASTM A228 music wire, A  184.649  ksi b  0.1625
b
 di  d
i
S ut
S ut  A   
i
 
i  in  in ksi
0.020 348.7
0.038 314.1
0.081 277.8
0.162 248.2
0.250 231.3

2. Calculate the ultimate shear strength using equation 14.4.

d S us
i i
S us  0.67 S ut  
i i in ksi
0.020 234
0.038 210
0.081 186
0.162 166
0.250 155

© 2011 Pearson Education, Inc., Upper Saddle River, NJ. All rights reserved. This publication is protected by Copyright and written permission should be
MACHINE DESIGN - An Integrated Approach, 4th Ed. 14-35-1

PROBLEM 14-35 _____

Statement: Select the preferred diameter for an ASTM A227 cold-drawn wire that will have an ultimate
tensile strength as close to, but not less than, 180 kpsi.
Given: Minimum tensile strength: S ut  180  ksi

Solution: See Mathcad file P1435.

1. Calculate the maximum wire diameter using equation 14.3 and the coefficient and exponent from Table 14-4.
For ASTM A227 cold-drawn wire, A  141.04 ksi b  0.1822
1
b
 S ut 
d trial     in d trial  0.262  in
A

2. Choose the next lower diameter from Table 14-2 as it will have a higher ultimate strength than the next higher
diameter.

d  0.250  in

3. Verify that the strength of a wire of this diameter is greater than the minimum required value.

b
S ut  A   d
 S ut  182  ksi
 
in

© 2011 Pearson Education, Inc., Upper Saddle River, NJ. All rights reserved. This publication is protected by Copyright and written permission should be
MACHINE DESIGN - An Integrated Approach, 4th Ed. 14-36-1

PROBLEM 14-36 _____

Statement: Select the preferred diameter for an ASTM A229 oil-tempered wire that will have an ultimate
tensile strength as close to, but not less than, 1430 MPa.
Given: Minimum tensile strength: S ut  1430 MPa

Solution: See Mathcad file P1436.

1. Calculate the maximum wire diameter using equation 14.3 and the coefficient and exponent from Table 14-4.
For ASTM A229 oil-tempered wire, A  1831.2 MPa b  0.1833
1
b
 S ut 
d trial     mm d trial  3.85 mm
A

2. Choose the next lower diameter from Table 14-2 as it will have a higher ultimate strength than the next higher
diameter.

d  3.5 mm

3. Verify that the strength of a wire of this diameter is greater than the minimum required value.

b
S ut  A   d
 S ut  1455 MPa
 
mm

© 2011 Pearson Education, Inc., Upper Saddle River, NJ. All rights reserved. This publication is protected by Copyright and written permission should be
MACHINE DESIGN - An Integrated Approach, 4th Ed. 14-37-1
PROBLEM 14-37
Statement: Design a helical extension spring to handle a dynamic load that varies from 275 N to 325 N over
10 mm working deflection. Use chrome silicon wire and standard hooks. The forcing frequency
is 800 rpm. Infinite life is desired. Minimize the package size. Choose appropriate safety
factors against fatigue, yielding, and surging.
Given: Minimum force Fmin  275  N Life L  ∞
Maximum force Fmax  325  N Shear modulus G  80.8 GPa
Working deflection ∆y  10.0 mm Forcing freq. ωf  800  rpm
Design Choices:
Fatigue safety factor Nfb  1.5 Spring index C  9
Wire endurance limit S ew  310  MPa Wire strength A  2059.2 MPa
b  0.0934

Solution: See Mathcad file P1437.


1. Find the mean and alternating loads.
Fmax  Fmin
Fa  Fa  25.0 N
2
Fmax  Fmin
Fm  Fm  300.0  N
2

2. Calculate the hook bending factor.


2
4 C  C  1
Kb  Kb  1.090
4 C ( C  1)

3. Solve for d by iteration using an equation derived from the safety factor equation for bending in the hooks.
1
2 b
 4   4  K b C  1  
d=  Nfb Fm   Nfb  1   Fmin  
 π A  
  0.577   0.67 A  db  0.5 S ew  
  
  Nfb Fa     
  0.5 0.67 S ew  
1
2 b
 4  4 Kb C  1 
RHS( d )   Nfb Fm   Nfb  1   Fmin    mm
 π A  mm2    b 

  0.577  0.67 A  
d   
  0.5 S ew
     mm    
  Nfb Fa  0.5 0.67 S ew  
    

d  1.000  mm RHS( d )  4.011  mm


d  RHS( d ) RHS( d )  3.879  mm
d  RHS( d ) RHS( d )  3.882  mm
d  RHS( d ) RHS( d )  3.882  mm d  4.0 mm

4. Calculate the mean coil diameter.


Mean coil diameter D  C d D  36 mm
© 2011 Pearson Education, Inc., Upper Saddle River, NJ. All rights reserved. This publication is protected by Copyright and written permission should be
MACHINE DESIGN - An Integrated Approach, 4th Ed. 14-37-2

5. Use the assumed value of C to find an appropiate value of initial coil stress i from equations 14.21:

 3 2 
τi1  4.231  C  181.5  C  3387 C  28640  psi τi1  67.4 MPa

  2.987  C  139.7  C  3427 C  38404   psi


3 2
τi2 τi2  115.1  MPa

τi1  τi2
τi  τi  91.3 MPa
2

6. Find the direct shear factor Ks :


0.5
Direct shear factor Ks  1  Ks  1.056
C

7. Use the value of i from (c) in equation 14.8 to find the corresponding initial coil-tension force Fi:
3
π d  τi
Fi  Fi  60.4 N
8  K s D

Check that this force is less than the required minimum applied force Fmin, which in this case, it is. Any
applied force smaller than Fi will not deflect the spring.
8. Use the direct shear factor Ks and previously assumed values to find the mean stress m:

8  Fm D
Stress at Fm τm  Ks τm  453.6  MPa
3
π d

9. Find the Wahl factor Kw and use it to calculate the alternating shear stress a in the coil.
4 C  1 0.615
Wahl factor Kw   Kw  1.162
4 C  4 C

8  F a D
Alternating stress τa  Kw τa  41.6 MPa
3
π d
10. Find the ultimate tensile strength of this wire material from equation 14.3 and Table 14-4 and use it to find the
ultimate shear strength from equation 14.4 and the torsional yield strength for the coil body from Table 14-10,
assuming no set removal.
b
S ut  A   
d
Ultimate tensile strength  S ut  1809.1 MPa
 mm 
Shear yield strength S ys  0.45 S ut S ys  814.1  MPa

Ultimate shear strength S us  0.67 S ut S us  1212.1 MPa

11. Find the wire endurance limit for unpeened springs from equation 14.13 and convert it to fully reversed
endurance strength with equation 14.16c.
S ew S us
Fully reversed S es  0.5 S es  177.73 MPa
endurance limit S us  0.5 S ew

12. The fatigue safety factor for the coils in torsion is calculated from equation 14.16b.
Minimum stress τmin  τm  τa τmin  412.0  MPa

© 2011 Pearson Education, Inc., Upper Saddle River, NJ. All rights reserved. This publication is protected by Copyright and written permission should be
MACHINE DESIGN - An Integrated Approach, 4th Ed. 14-37-3

S es  S us  τmin
Fatigue safety factor Nfs  Nfs  2.5
S es  τm  τmin  S us τa

Note that the minimum stress due to force Fmin is used in this calculation, not the coil-winding stress from
step 5.
13. The stresses in the end hooks also need to be determined. The bending stresses in the hook are found from
equation 14.23:

2  R1 2 D
C1 = = =C C1  C C1  9.00
d 2 d
2
4 C1  C1  1
Kb  Kb  1.09
4  C1  C1  1 

16 D Fa 4  Fa
σa  Kb  σa  80.07  MPa
3 2
π d π d

16 D Fm 4  Fm
σm  Kb  σm  960.9  MPa
3 2
π d π d

16 D Fmin 4  Fmin


σmin  Kb  σmin  880.82 MPa
3 2
π d π d

14. Convert the torsional endurance strength to a tensile endurance strength with the von Mises relationship and
use it and the ultimate tensile strength in equation 14.16 to find a fatigue safety factor for the hook in bending:

S es
S e  S e  308.02 MPa
0.577
S e  S ut  σmin
Nfb  Nfb  1.7
S e  σm  σmin  S ut σa

15. The torsional stresses in the hook are found from equation 14.24 using an assumed value of C2  5.
C2 d
R2  R2  10 mm
2
4  C2  1
Kw2  Kw2  1.188
4  C2  4

8  Fa D
τBa  Kw2 τBa  42.5 MPa
3
π d
8  Fm D
τBm  Kw2 τBm  510.3  MPa
3
π d
8  Fmin D
τBmin  Kw2 τBmin  467.8  MPa
3
π d
© 2011 Pearson Education, Inc., Upper Saddle River, NJ. All rights reserved. This publication is protected by Copyright and written permission should be
MACHINE DESIGN - An Integrated Approach, 4th Ed. 14-37-4

16. The fatigue safety factor for the hook in torsion is calculated from equation 14.16b.

S es  S us  τBmin
Nfs  Nfs  2.2
S es  τBm  τBmin  S us τBa

17. The spring rate is defined from the two specified forces at their relative deflection.

Fmax  Fmin N
Spring rate k  k  5000.0
∆y m

18. To get the defined spring rate, the number of active coils must satisfy equation 14.7, solving for Na yields:
4
d G
Number of active coils Na  Na  11.084 Na  11.0
3
8 D  k
Note that we round it to the nearest 1/4 coil as the manufacturing tolerance cannot achieve better than that
accuracy. Having rounded the number of active coils, we must now calculate the spring rate using equation 14.
Corrected spring rate 4
d G N
k  k  5038.0
3 m
8  D  Na
19. The total number of coils in the body and the body length are
Total coils Nt  Na  1 Nt  12.00

Body length Lb  Nt d Lb  48.0 mm

20. The free length can now be determined. The length of a standard hook is equal to the coil inside diameter:
Hook length Lhook  D  d Lhook  32.0 mm

Free length Lf  Lb  2  Lhook Lf  112.0  mm

21. The initial coil tension force must be found again in order to obtain the deflection to reach the larger of the two
loads.
 3 2 
τi1  4.231  C  181.5  C  3387 C  28640  psi τi1  67.4 MPa

  2.987  C  139.7  C  3427 C  38404   psi


3 2
τi2 τi2  115.1  MPa
τi1  τi2
τi  τi  91.3 MPa
2
3
π d  τi
Fi  Fi  60.4 N
8  K s D

Fmax  Fi
ymax  ymax  52.53  mm
k
22. The inside and outside coil diameters are
Inside coil dia Di  D  d Di  32.00  mm
Outside coil dia Do  D  d Do  40.00  mm

23. The weight of the spring's active coils is found from equation 14.11b and is
3
Weight density ρ  0.28 lbf  in
© 2011 Pearson Education, Inc., Upper Saddle River, NJ. All rights reserved. This publication is protected by Copyright and written permission should be
MACHINE DESIGN - An Integrated Approach, 4th Ed. 14-37-5

2 2
π  d  D Na ρ
Weight Wa  Wa  1.188  N
4

24. The natural frequency of this spring is found from equation 14.11a and is:

Natural frequency 1 k g
fn   fn  102  Hz
2 Wa

25. The ratio between the natural frequency and the forcing frequency is

ωf
Forcing frequency ff  ff  13.333 Hz
2 π
fn
Frequency ratio  7.6 which could be higher.
ff

26. We now have a complete design specification for this A228-wire spring:
Wire diameter d  4.00 mm Total coils Nt  12.00
Outside diameter Do  40.00  mm Free length Lf  112.00 mm
Standard hooks

© 2011 Pearson Education, Inc., Upper Saddle River, NJ. All rights reserved. This publication is protected by Copyright and written permission should be
MACHINE DESIGN - An Integrated Approach, 4th Ed. 14-38-1
PROBLEM 14-38
Statement: Design a helical extension spring with standard hooks to handle a dynamic load that varies from
60 lb to 75 lb over 0.5 in working deflection. Use music wire. The forcing frequency is 1200 rpm.
Infinite life is desired. Minimize the package size. Choose appropriate safety factors against
fatigue, yielding, and surging.

Given: Minimum force Fmin  60 lbf Life L  ∞


6
Maximum force Fmax  75 lbf Shear modulus G  11.7 10  psi
Working deflection ∆y  0.50 in Forcing freq. ωf  1200 rpm

Design Choices:
Fatigue safety factor Nfb  1.5 Spring index C  9
Wire endurance limit S ew  45 ksi Wire strength A  184.649  ksi
b  0.1625
Solution: See Mathcad file P1438.
1. Find the mean and alternating loads.
Fmax  Fmin
Fa  Fa  7.5 lbf
2
Fmax  Fmin
Fm  Fm  67.5 lbf
2

2. Calculate the hook bending factor.


2
4 C  C  1
Kb  Kb  1.090
4 C ( C  1)

3. Solve for d by iteration using an equation derived from the safety factor equation for bending in the hooks.
1
2 b
4   4  K b C  1 
d=   Nfb Fm   Nfb  1   Fmin 


 π A  
  0.577   0.67 A  db  0.5 S ew  
  
  Nfb Fa  0.5 0.67 S ew  
   
1
2 b
 4  4 Kb C  1 
RHS( d )    Nfb Fm   Nfb  1   Fmin    in
 π A  in2  
   b  
0.577 0.67 A    0.5 S ew 

 d
  
     in   
 Nfb Fa   
 0.5 0.67 S ew
    

d  1.00 in RHS( d )  0.153  in


d  RHS( d ) RHS( d )  0.167  in
d  RHS( d ) RHS( d )  0.166  in
d  RHS( d ) RHS( d )  0.166  in d  0.177  in
4. Calculate the mean coil diameter.
Mean coil diameter D  C d D  1.593  in
© 2011 Pearson Education, Inc., Upper Saddle River, NJ. All rights reserved. This publication is protected by Copyright and written permission should be
MACHINE DESIGN - An Integrated Approach, 4th Ed. 14-38-2

5. Use the assumed value of C to find an appropiate value of initial coil stress i from equations 14.21:

 3 2 
τi1  4.231  C  181.5  C  3387 C  28640  psi τi1  9.77 ksi

  2.987  C  139.7  C  3427 C  38404   psi


3 2
τi2 τi2  16.70  ksi

τi1  τi2
τi  τi  13.2 ksi
2

6. Find the direct shear factor Ks :


0.5
Direct shear factor Ks  1  Ks  1.056
C

7. Use the value of i from (c) in equation 14.8 to find the corresponding initial coil-tension force Fi:
3
π d  τi
Fi  Fi  17.1 lbf
8  K s D

Check that this force is less than the required minimum applied force Fmin, which in this case, it is. Any
applied force smaller than Fi will not deflect the spring.
8. Use the direct shear factor Ks and previously assumed values to find the mean stress m:

8  Fm D
Stress at Fm τm  Ks τm  52.1 ksi
3
π d

9. Find the Wahl factor Kw and use it to calculate the alternating shear stress a in the coil.
4 C  1 0.615
Wahl factor Kw   Kw  1.162
4 C  4 C

8  F a D
Alternating stress τa  Kw τa  6.4 ksi
3
π d
10. Find the ultimate tensile strength of this wire material from equation 14.3 and Table 14-4 and use it to find the
ultimate shear strength from equation 14.4 and the torsional yield strength for the coil body from Table 14-10,
assuming no set removal.
b
S ut  A   d
Ultimate tensile strength  S ut  244.7  ksi
 
in

Shear yield strength S ys  0.50 S ut S ys  122.3  ksi

Ultimate shear strength S us  0.67 S ut S us  163.9  ksi

11. Find the wire endurance limit for unpeened springs from equation 14.13 and convert it to fully reversed
endurance strength with equation 14.16c.
S ew S us
Fully reversed S es  0.5 S es  26.08  ksi
endurance limit S us  0.5 S ew
12. The fatigue safety factor for the coils in torsion is calculated from equation 14.16b.
Minimum stress τmin  τm  τa τmin  45.75  ksi

S es  S us  τmin
Fatigue safety factor Nfs  Nfs  2.5
S es  τm  τmin  S us τa
© 2011 Pearson Education, Inc., Upper Saddle River, NJ. All rights reserved. This publication is protected by Copyright and written permission should be
MACHINE DESIGN - An Integrated Approach, 4th Ed. 14-38-3

Note that the minimum stress due to force Fmin is used in this calculation, not the coil-winding stress from
step 5.
13. The stresses in the end hooks also need to be determined. The bending stresses in the hook are found from
equation 14.23:

2  R1 2 D
C1 = = =C C1  C C1  9.00
d 2 d
2
4 C1  C1  1
Kb  Kb  1.09
4  C1  C1  1 

16 D Fa 4  Fa
σa  Kb  σa  12.27  ksi
3 2
π d π d

16 D Fm 4  Fm
σm  Kb  σm  110.4  ksi
3 2
π d π d

16 D Fmin 4  Fmin


σmin  Kb  σmin  98.15  ksi
3 2
π d π d

14. Convert the torsional endurance strength to a tensile endurance strength with the von Mises relationship and
use it and the ultimate tensile strength in equation 14.16 to find a fatigue safety factor for the hook in bending:

S es
S e  S e  45.20  ksi
0.577
S e  S ut  σmin
Nfb  Nfb  1.9
S e  σm  σmin  S ut σa

15. The torsional stresses in the hook are found from equation 14.24 using an assumed value of C2  5.
C2 d
R2  R2  0.442  in
2
4  C2  1
Kw2  Kw2  1.188
4  C2  4

8  Fa D
τBa  Kw2 τBa  6.5 ksi
3
π d
8  Fm D
τBm  Kw2 τBm  58.6 ksi
3
π d
8  Fmin D
τBmin  Kw2 τBmin  52.1 ksi
3
π d

16. The fatigue safety factor for the hook in torsion is calculated from equation 14.16b.

S es  S us  τBmin
Nfs  Nfs  2.4
S es  τBm  τBmin  S us τBa
© 2011 Pearson Education, Inc., Upper Saddle River, NJ. All rights reserved. This publication is protected by Copyright and written permission should be
MACHINE DESIGN - An Integrated Approach, 4th Ed. 14-38-4

17. The spring rate is defined from the two specified forces at their relative deflection.

Fmax  Fmin lbf


Spring rate k  k  30.0
∆y in

18. To get the defined spring rate, the number of active coils must satisfy equation 14.7, solving for Na yields:
4
d G
Number of active coils Na  Na  11.836 Na  11.75
3
8 D  k
Note that we round it to the nearest 1/4 coil as the manufacturing tolerance cannot achieve better than that
accuracy. Having rounded the number of active coils, we must now calculate the spring rate using equation 14.
4
d G lbf
Corrected spring rate k  k  30.2
3 in
8  D  Na

19. The total number of coils in the body and the body length are
Total coils Nt  Na  1 Nt  12.75

Body length Lb  Nt d Lb  2.3 in

20. The free length can now be determined. The length of a standard hook is equal to the coil inside diameter:
Hook length Lhook  D  d Lhook  1.42 in

Free length Lf  Lb  2  Lhook Lf  5.09 in

21. The initial coil tension force must be found again in order to obtain the deflection to reach the larger of the two
loads.
 3 2 
τi1  4.231  C  181.5  C  3387 C  28640  psi τi1  9.8 ksi

  2.987  C  139.7  C  3427 C  38404   psi


3 2
τi2 τi2  16.7 ksi
τi1  τi2
τi  τi  13.2 ksi
2
3
π d  τi
Fi  Fi  17.1 lbf
8  K s D
Fmax  Fi
ymax  ymax  1.91 in
k
22. The inside and outside coil diameters are
Inside coil dia Di  D  d Di  1.416  in
Outside coil dia Do  D  d Do  1.770  in
23. The weight of the spring's active coils is found from equation 14.11b and is
3
Weight density ρ  0.28 lbf  in
2 2
π  d  D Na ρ
Weight Wa  Wa  0.405  lbf
4
24. The natural frequency of this spring is found from equation 14.11a and is:

Natural frequency 1 k g
fn   fn  84.9 Hz
2 Wa
© 2011 Pearson Education, Inc., Upper Saddle River, NJ. All rights reserved. This publication is protected by Copyright and written permission should be
MACHINE DESIGN - An Integrated Approach, 4th Ed. 14-38-5

25. The ratio between the natural frequency and the forcing frequency is

ωf
Forcing frequency ff  ff  20 Hz
2 π
fn
Frequency ratio  4.2 which could be higher.
ff

26. We now have a complete design specification for this A228-wire spring:
Wire diameter d  0.177  in Total coils Nt  12.75
Outside diameter Do  1.770  in Free length Lf  5.089  in
Standard hooks

© 2011 Pearson Education, Inc., Upper Saddle River, NJ. All rights reserved. This publication is protected by Copyright and written permission should be
MACHINE DESIGN - An Integrated Approach, 4th Ed. 14-39-1
PROBLEM 14-39
Statement: Design a straight-ended helical torsion spring for a static load 50 N-m at a deflection of 60 deg
with a safety factor of 2. Specify all parameters necessary to manufacture the spring. State all
assumptions.
Given: Applied moment M  50 N  m Bending modulus E  206.8  GPa
Deflection at load θ  60 deg θ  0.167  rev
Design Choices:
Use unpeened oil tempered wire with 40-mm-long, straight ends. The coil is loaded to close it.
Design safety factor Nyd  2 Spring index C  10
Yield strength factor Ks  0.85 (from Table 14-13)
Length of ends L1  40 mm L2  40 mm
Material strength factors A  1831.2 MPa b  0.1833 (ASTM A229 wire)

Solution: See Mathcad file P1439.


1. Calculate the Wahl bending factors for inside surface.
2
4 C  C  1
Kbi  Kbi  1.081
4  C ( C  1 )

2. Solve for d using the static yield criterion.


1
3 b
 32 Kbi Nyd  M 
d    mm d  10.272 mm
 π Ks A  mm3 
 
3. Use a wire diameter from the available sizes in Table 14-2. Calculate the mean coil diameter D from equation
14.5.
Wire diameter d  11 mm
Spring index C  10
Mean coil diameter D  C d D  110  mm

4. Calculate the maximum compressive stress in the coil at the inner surface.
32 M
σimax  Kbi σimax  413.5  MPa
3
π d
5. Find the ultimate tensile strength of this oil tempered material from equation 14.3 and Table 14-4 and use it to
find the bending yield strength from Table 14-13, assuming stress relieving.
b
S ut  A   
Ultimate tensile d
strength  S ut  1180 MPa
 
mm

Bending yield
strength S y  Ks S ut S y  1003 MPa

Sy
6. The realized static safety factor against yielding is Nyb  Nyb  2.4
σimax

7. The spring rate is defined from the two specified moments at their relative deflection.
M N m
Spring rate k  k  300 
θ rev
8. To get the defined spring rate, the number of active coils must satisfy equation 14.28, solving for Na yields:
© 2011 Pearson Education, Inc., Upper Saddle River, NJ. All rights reserved. This publication is protected by Copyright and written permission should be
MACHINE DESIGN - An Integrated Approach, 4th Ed. 14-39-2

4
Number of active d E
coils Na  Na  8.50
10.8 D k rev

Note that to force k to be in units of N-m per rev we must multiply k by rev.
9. The ends contribute to the active coils from equation 14.26a as

L1  L2
Ne  Ne  0.08
3  π D

and, from equation 14.26b, the number of body coils in the spring are Nb  Na  Ne Nb  8.4

10. Check the angular deflection at the specified load from equation 14.27c.

M  D Na
θ  10.8  rev θ  60.0 deg
4
d E

© 2011 Pearson Education, Inc., Upper Saddle River, NJ. All rights reserved. This publication is protected by Copyright and written permission should be
MACHINE DESIGN - An Integrated Approach, 4th Ed. 14-40-1
PROBLEM 14-40
Statement: Design a straight-ended helical torsion spring for a static load 430 in-lb at a deflection of 55 deg
with a safety factor of 2. Specify all parameters necessary to manufacture the spring. State all
assumptions.
6
Given: Applied moment M  430  in lbf Bending modulus E  30 10  psi
Deflection at load θ  55 deg θ  0.153  rev

Design Choices:
Use unpeened music wire with 2-in-long, straight ends. The coil is loaded to close it.
Design safety factor Nyd  2 Spring index C  9
Yield strength factor Ks  1 (from Table 14-13)
Length of ends L1  2  in L2  2  in
Material strength factors A  184.65 ksi b  0.1625 (ASTM A228 wire)

Solution: See Mathcad file P1440.


1. Calculate the Wahl bending factors for inside surface.
2
4 C  C  1
Kbi  Kbi  1.090
4  C ( C  1 )

2. Solve for d using the static yield criterion.


1
3 b
 32 Kbi Nyd  M 
d    in d  0.352  in
 π Ks A  in3 
 
3. Use a wire diameter from the available sizes in Table 14-2. Calculate the mean coil diameter D from equation
14.5.
Wire diameter d  0.362  in
Spring index C9
Mean coil diameter D  C d D  3.258  in
4. Calculate the maximum compressive stress in the coil at the inner surface.
32 M
σimax  Kbi σimax  100.7  ksi
3
π d
5. Find the ultimate tensile strength of this oil tempered material from equation 14.3 and Table 14-4 and use it to
find the bending yield strength from Table 14-13, assuming stress relieving.
b
S ut  A   
Ultimate tensile d
strength  S ut  218  ksi
 in 
Bending yield
strength S y  Ks S ut S y  218  ksi

Sy
6. The realized static safety factor against yielding is Nyb  Nyb  2.2
σimax

7. The spring rate is defined from the two specified moments at their relative deflection.
M in lbf
Spring rate k  k  2815
θ rev
8. To get the defined spring rate, the number of active coils must satisfy equation 14.28, solving for Na yields:
© 2011 Pearson Education, Inc., Upper Saddle River, NJ. All rights reserved. This publication is protected by Copyright and written permission should be
MACHINE DESIGN - An Integrated Approach, 4th Ed. 14-40-2

4
Number of active d E
coils Na  Na  5.20
10.8 D k rev

Note that to force k to be in units of N-m per rev we must multiply k by rev.
9. The ends contribute to the active coils from equation 14.26a as

L1  L2
Ne  Ne  0.13
3  π D
and, from equation 14.26b, the number of body coils in the spring are Nb  Na  Ne Nb  5.1

10. Check the angular deflection at the specified load from equation 14.27c.

M  D Na
θ  10.8  rev θ  55.0 deg
4
d E

© 2011 Pearson Education, Inc., Upper Saddle River, NJ. All rights reserved. This publication is protected by Copyright and written permission should be
MACHINE DESIGN - An Integrated Approach, 4th Ed. 14-41-1

PROBLEM 14-41
Statement: Given the following data for a helical torsion spring, loaded in fatigue, find the spring index,
unloaded coil diameter, minimum loaded coil diameter, and safety factor in fatigue. State all
assumptions and sources of empirical data used. Deflection at assembly = 0.15 rev, working
deflection = 0.35 rev, k = 10 N-m/rev, Na = 25, 4.50 mm oil-tempered wire, unpeened.

Given: Minimum deflection θmin  0.15 rev Bending modulus E  206.8  GPa
1
Maximum deflection θmax  0.50 rev Spring rate k  10 N  m rev
Number active coils Na  25 Wire diameter d  4.50 mm

Design Choices:
Use unpeened oil-tempered wire with 40-mm-long, straight ends. The coil is loaded to close it.
Yield strength factor Ks  0.85 (from Table 14-13)
Length of ends L1  40 mm L2  40 mm
Material strength factors A  1831.2 MPa b  0.1833 (ASTM A229 wire)
Wire endurance limit S ewb  537  MPa

Solution: See Mathcad file P1441.


1. Find the minimum, maximum, mean, and alternating loads.
Mmin  k θmin Mmin  1.50 N  m

Mmax  k θmax Mmax  5.00 N  m

Mmax  Mmin
Ma  Ma  1.75 N  m
2
Mmax  Mmin
Mm  Mm  3.25 N  m
2
3
d E
2. Use equation 14.28 to calculate the spring index. k=
10.8 C Na
3
d E
C  C  6.98
10.8 k rev Na

3. Calculate the Wahl bending factors for inside and outside surfaces.
2
4 C  C  1
Kbi  Kbi  1.119
4  C ( C  1 )
2
4 C  C  1
Kbo  Kbo  0.902
4  C ( C  1 )

4. Calculate the mean coil diameter D from equation 14.5.


Mean coil diameter D  C d D  31.41  mm
5. Calculate the maximum compressive stress in the coil at the inner surface.

32 Mmax
σimax  Kbi σimax  625.7  MPa
3
π d
6. Calculate the maximum, minimum, alternating, and mean tensile stresses in the coil at the outer surface.

© 2011 Pearson Education, Inc., Upper Saddle River, NJ. All rights reserved. This publication is protected by Copyright and written permission should be
MACHINE DESIGN - An Integrated Approach, 4th Ed. 14-41-2

32 Mmin
σomin  Kbo σomin  151.2  MPa
3
π d
32 Mmax
σomax  Kbo σomax  503.9  MPa
3
π d
σomax  σomin
σm  σm  327.5  MPa
2

σomax  σomin
σa  σa  176.4  MPa
2

7. Find the ultimate tensile strength of this oil tempered material from equation 14.3 and Table 14-4 and use it to
find the bending yield strength from Table 14-13, assuming stress relieving.
b
S ut  A   d
Ultimate tensile strength  S ut  1390 MPa
 
mm

Bending yield strength S y  Ks S ut S y  1181 MPa

8. Convert the wire bending endurance limit for unpeened springs from equation 14.33 to fully reversed
endurance strength with equation 14.34b.

S ewb S ut
Fully reversed S e  0.5 S e  332.78 MPa
endurance limit S ut  0.5 S ewb

9. The fatigue safety factor for the coils in bending is calculated from equation 14.34a.

Fatigue factor S e  S ut  σomin


Nfb  Nfb  1.4
of safety S e  σm  σomin  S ut σa

L1  L2
10. The ends contribute to the active coils from equation 14.26a as Ne  Ne  0.27
3  π D
Nb  Na  Ne
and, from equation 14.26b, the number of body coils in the spring are Nb  25

11. The unloaded coil diameter and minimum loaded coil diameters are:

Unloaded coil diameter Di  D  d Di  26.9 mm

D Nb
Loaded coil diameter Dimin  d Dimin  26.3 mm
θmax
Nb 
rev

© 2011 Pearson Education, Inc., Upper Saddle River, NJ. All rights reserved. This publication is protected by Copyright and written permission should be
MACHINE DESIGN - An Integrated Approach, 4th Ed. 14-42-1
PROBLEM 14-42
Statement: Design a Belleville spring to give a static force of approximately 2000 lb at a maximum deflection
of 0.05 in with a nearly constant spring rate.
Given: Maximum force Fflat  2000 lbf Maximum deflection ymax  0.05 in

6
Assumptions: The diameter ratio is Rd  2. Use unset carbon spring steel 50HRC. Properties, E  30 10  psi ,
ν  0.28 .
Solution: See Mathcad file P1442.

1. Since a constant rate spring is needed, the h/t ratio (see Figure 14-30) is
h/t ratio hovert  0.40
2. Use the above choice and the specified maximum deflection to find the spring thickness t:
h
h  ymax t  t  0.125  in
hovert
3. Use equation 14.37 to find the force at maximum deflection. This must be done by iterating on the outside
diameter, Do.

Guess: Do  3.0 in

7 4
19.2 10  psi  hovert t
Fflat  Fflat  2083.33  lbf
2
Do

4. Figure 14-34 shows that the worst stress state will occur at the largest deflection ymax, so solve equations
14.36 for stresses at that deflection:
  Rd  1  2
 
6
K1  K1  0.689
π ln Rd   2 
 Rd 
6  Rd  1 
K2    1 K2  1.220
π ln Rd   ln Rd  
6  Rd  1
K3    K3  1.378
π ln Rd   2 

 Rd  ln Rd    Rd  1  Rd 
K4   ln Rd 
 K4  1.115
    Rd  1  2
Rd
K5  K5  1
2   Rd  1 

4  E ymax   ymax  
σc    K2  h    K3 t σc  213  ksi
2 
K1 Do  1  ν
2    2  

 4  E ymax    ymax  
σti    K2  h    K3 t σti  148.8  ksi
 K1 Do2 1  ν2
    

2  
© 2011 Pearson Education, Inc., Upper Saddle River, NJ. All rights reserved. This publication is protected by Copyright and written permission should be
MACHINE DESIGN - An Integrated Approach, 4th Ed. 14-42-2

 4  E ymax   ymax  
σto     K4  h    K5 t σto  161  ksi

 K1 Do2 1  ν2
   

2  

5. Table 14-5 gives S ut  246  ksi for this material. Table 14-15 indicates that 120% of this value can be used
for an unset spring. The safety factor for static loading is then

1.20 S ut
Ns  Ns  1.4
σc

which is acceptable.

6. A summary of the spring design is

Outside diameter Do  3.000  in

Do
Inside diameter Di  Di  1.500  in
Rd

Thickness t  0.125  in

Height h  0.050  in

© 2011 Pearson Education, Inc., Upper Saddle River, NJ. All rights reserved. This publication is protected by Copyright and written permission should be
MACHINE DESIGN - An Integrated Approach, 4th Ed. 14-43-1
PROBLEM 14-43
Statement: Design a Belleville spring to give a static force of 400 lb at 50% deflection to flat and 200 lb at fla
Given: Force at flat Fflat  200  lbf

6
Assumptions: The diameter ratio is Rd  2. Use unset carbon spring steel 50HRC. Properties, E  30 10  psi ,
ν  0.28 . The outside diameter of the spring is Do  4.00 in
Solution: See Mathcad file P1443.
1. Since a spring with 200% force to flat at 50% deflection to flat is needed, the h/t ratio (see Figure 14-30) is
h/t ratio hovert  2.828
2. Use the above assumptions and the specified nominal force in equation 14.37a to find an appropriate spring
thickness t:
1
4
 Do 
2
t 
 Fflat
 t  0.049  in
 7 hovert 
 19.2  10  psi 
3. The height h can now be found: h  hovert t h  0.139  in

4. Figure 14-34 shows that the worst stress state will occur at the largest deflection ymax, so solve equations
14.36 for stresses at that deflection:
ymax  h

  Rd  1  2
 
6
K1  K1  0.689
π ln Rd   R2 
 d 
6  Rd  1 
K2    1 K2  1.220
π ln Rd   ln Rd  
6  Rd  1 
K3    K3  1.378
π ln Rd   2 

 Rd  ln Rd    Rd  1  Rd 
K4   ln Rd 
 K4  1.115
    Rd  1  2
Rd
K5  K5  1
2   Rd  1 

4  E ymax   ymax  
σc    K2  h    K3 t σc  252  ksi
2 
K1 Do  1  ν
2   2  

 4  E ymax   ymax  
σti     K2  h    K3 t σti  28.2 ksi
 K1 Do2 1  ν2
    

2  

 4  E ymax    ymax  
σto    K4  h    K5 t σto  209  ksi
 K1 Do2 1  ν2
    

2  
© 2011 Pearson Education, Inc., Upper Saddle River, NJ. All rights reserved. This publication is protected by Copyright and written permission should be
MACHINE DESIGN - An Integrated Approach, 4th Ed. 14-43-2

5. Table 14-5 gives S ut  246  ksi for this material. Table 14-15 indicates that 120% of this value can be used
for an unset spring. The safety factor for static loading is then

1.20 S ut
Ns  Ns  1.2
σc

which is acceptable.

6. A summary of the spring design is

Outside diameter Do  4.000  in

Do
Inside diameter Di  Di  2.000  in
Rd

Thickness t  0.049  in

Height h  0.139  in

© 2011 Pearson Education, Inc., Upper Saddle River, NJ. All rights reserved. This publication is protected by Copyright and written permission should be
MACHINE DESIGN - An Integrated Approach, 4th Ed. 14-44-1
PROBLEM 14-44
Statement: Design a helical compression spring for a static load of 60 lb at a deflection of 1.50 in with a
safety factor of 2.0 to work in a 1.06-in hole. Specify all parameters necessary to
manufacture the spring.

6
Given: Working force Fwork  60 lbf Shear modulus G  11.5 10  psi
Working deflection ywork  1.50 in Safety factor Ns  2.0

Design choices:
Clash allowance α  0.15 Set removed Km  0.65
ASTM A228 wire A  184.649  ksi Spring index C  7.0
b  0.1625 (found by trial-and-error)
Solution: See Mathcad file P1444.

Fwork lbf
1. Determine the desired spring rate. k  k  40
ywork in

2. Use the design equation from Example 14-3A (Mathcad Supplement) to determine the wire diameter.
1
2 b
 8 Ns ( C  0.5)  Fwork  ( 1  α)
d   in
 π Km A  in
2 
 
Wire diameter d  0.125  in Let d  0.125  in

3. Calculate the mean and outside coil diameters and number of active coils.
Mean coil dia D  C d D  0.875  in
Outside coil dia Do  D  d Do  1.000  in
4
Number of active d G
coils Na  Na  13.097
3
8 D  k Na  13.0

Note that we round it to the nearest 1/4 coil as the manufacturing tolerance cannot achieve better than that
accuracy. We must now calculate the actual (corrected) spring rate:
4
Corrected spring d G lbf
rate k  k  40.298
3 in
8  D  Na
4. Assume squared and ground ends making the total number of coils, from Figure 14-9:

Total coils Nt  Na  2 Nt  15.00

5. The shut height can now be determined.


Shut height Ls  d  Nt Ls  1.875  in
6. The free length (see Figure 14-8) can now be found from
Deflection
to shut height yshut  ywork  α ywork yshut  1.725  in

Free length Lf  Ls  yshut Lf  3.600  in

© 2011 Pearson Education, Inc., Upper Saddle River, NJ. All rights reserved. This publication is protected by Copyright and written permission should be
MACHINE DESIGN - An Integrated Approach, 4th Ed. 14-44-2

7. To check for buckling, two ratios need to be calculated, Lf/D and y max/Lf.
Lf
Slenderness ratio sr  sr  4.114
D
ywork
Deflection ratio y'  y'  0.417
Lf

Take these two values to Figure 14-14 and find that their coordinates are safely within the zones that are
stable against buckling for either end-condition case.
8. The inside and outside coil diameters are
Inside coil dia Di  D  d Di  0.750  in
Outside coil dia Do  D  d Do  1.000  in
9. The smallest hole and largest pin that should be used with this spring are
Smallest hole holemin  Do  0.05 D holemin  1.04 in

Largest pin pin max  Di  0.05 D pin max  0.71 in


10. The total weight of the spring is
3
Weight density ρ  0.28 lbf  in

2 2
π  d  D Nt ρ
Weight Wt  Wt  0.14 lbf
4

11. We now have a complete design specification for this A228 wire spring:

Wire diameter d  0.125  in


Outside diameter Do  1.000  in
Total coils Nt  15.00 ends squared and ground
Free length Lf  3.600  in

© 2011 Pearson Education, Inc., Upper Saddle River, NJ. All rights reserved. This publication is protected by Copyright and written permission should be
MACHINE DESIGN - An Integrated Approach, 4th Ed. 14-45-1
PROBLEM 14-45
Statement: Design a helical compression spring for a static load of 200 N at a deflection of 40 mm with a
safety factor of 1.8 to work in a 25-mm hole. Specify all parameters necessary to manufacture
the spring.

Given: Working force Fwork  200  N Shear modulus G  80.8 GPa


Working deflection ywork  40 mm Safety factor Ns  1.8
Design choices:
Clash allowance α  0.15 Set removed Km  0.65
ASTM A228 wire A  2153.5 MPa Spring index C  7.5
b  0.1625 (found by trial-and-error)
Solution: See Mathcad file P1445.

Fwork N
1. Determine the desired spring rate. k  k  5.000 
ywork mm

2. Use the design equation from Example 14-3A (Mathcad Supplement) to determine the wire diameter.
1
2 b
 8 Ns ( C  0.5)  Fwork  ( 1  α)
d   mm
 π Km A  mm
2 
 
Wire diameter d  2.658  mm Let d  2.8 mm

3. Calculate the mean and outside coil diameters and number of active coils.
Mean coil dia D  C d D  21.000 mm
Outside coil dia Do  D  d Do  23.800 mm
4
Number of active d G
coils Na  Na  13.407
3
8 D  k Na  13.5

Note that we round it to the nearest 1/4 coil as the manufacturing tolerance cannot achieve better than that
accuracy. We must now calculate the actual (corrected) spring rate:
4
Corrected spring d G N
rate k  k  4.965 
3 mm
8  D  Na
4. Assume squared and ground ends making the total number of coils, from Figure 14-9:

Total coils Nt  Na  2 Nt  15.50

5. The shut height can now be determined.


Shut height Ls  d  Nt Ls  43.4 mm
6. The free length (see Figure 14-8) can now be found from
Deflection
to shut height yshut  ywork  α ywork yshut  46.0 mm

Free length Lf  Ls  yshut Lf  89.4 mm

7. To check for buckling, two ratios need to be calculated, Lf/D and y max/Lf.
© 2011 Pearson Education, Inc., Upper Saddle River, NJ. All rights reserved. This publication is protected by Copyright and written permission should be
MACHINE DESIGN - An Integrated Approach, 4th Ed. 14-45-2

Lf
Slenderness ratio sr  sr  4.257
D
ywork
Deflection ratio y'  y'  0.447
Lf

Take these two values to Figure 14-14 and find that their coordinates are safely within the zones that are
stable against buckling for either end-condition case.
8. The inside and outside coil diameters are
Inside coil dia Di  D  d Di  18.2 mm
Outside coil dia Do  D  d Do  23.8 mm
9. The smallest hole and largest pin that should be used with this spring are
Smallest hole holemin  Do  0.05 D holemin  24.85  mm

Largest pin pin max  Di  0.05 D pin max  17.15  mm


10. The total weight of the spring is
3
Weight density ρ  0.28 lbf  in

2 2
π  d  D Nt ρ
Weight Wt  Wt  0.48 N
4

11. We now have a complete design specification for this A228 wire spring:

Wire diameter d  2.8 mm


Outside diameter Do  23.8 mm
Total coils Nt  15.50 ends squared and ground
Free length Lf  89.4 mm

© 2011 Pearson Education, Inc., Upper Saddle River, NJ. All rights reserved. This publication is protected by Copyright and written permission should be
MACHINE DESIGN - An Integrated Approach, 4th Ed. 14-46-1

PROBLEM 14-46
Statement: Three springs are arranged in series similar to the configuration shown in Figure 3-1(a). They
have spring rates k1 = 50 N/mm, k2 = 150 N/mm, and k3 = 500 N/mm, respectively. Determine
the total spring rate, the deflection of each spring, and the overall deflection if a load of F =
600 N is applied.

N N N
Given: Spring rates: k1  50 k2  150  k3  500 
mm mm mm
Load: F  600  N

Solution: See Figure 14-1 and Mathcad file P1446.

1. Using equation 14.1, calculate the deflection of each spring.

F
y1  y1  12 mm
k1

F
y2  y2  4  mm
k2

F
y3  y3  1.2 mm
k3

2. The sum of the three is the overall deflection.

ytotal  y1  y2  y3 ytotal  17.2 mm

3. Alternatively, use equation 14.2b to calculate the total spring rate and use it to determine the total deflection.

k1 k2 k3 N
ktotal  ktotal  34.884
k2 k3  k1 k3  k1 k2 mm

F
ytotal  ytotal  17.2 mm
ktotal

© 2011 Pearson Education, Inc., Upper Saddle River, NJ. All rights reserved. This publication is protected by Copyright and written permission should be
MACHINE DESIGN - An Integrated Approach, 4th Ed. 14-47-1

PROBLEM 14-47
Statement: Three springs are arranged in parallel similar to the configuration shown in Figure 3-1(b).
They have spring rates k1 = 50 N/mm, k2 = 150 N/mm, and k3 = 500 N/mm, respectively.
Determine the total spring rate, the force carried by each spring, and the overall deflection if a
load of F = 600 N is applied.

N N N
Given: Spring rates: k1  50 k2  150  k3  500 
mm mm mm
Load: F  600  N

Solution: See Figure 14-1 and Mathcad file P1447.


1. Using equation 14.2a, calculate the total spring rate.

N
ktotal  k1  k2  k3 ktotal  700 
mm

2. Calculate the deflection, which is the same for each spring (assuming that the plate that carries the load does
not rotate).
F
y  y  0.857  mm
ktotal

3. Calculate the force carried by each of the three springs using equation 14.1

F1  y  k1 F1  42.857 N

F2  y  k2 F2  128.571  N

F3  y  k3 F3  428.571  N

4. The sum of the three is the total load carried by the plate.

Ftotal  F1  F2  F3 Ftotal  600  N

© 2011 Pearson Education, Inc., Upper Saddle River, NJ. All rights reserved. This publication is protected by Copyright and written permission should be
MACHINE DESIGN - An Integrated Approach, 4th Ed. 14-48-1

PROBLEM 14-48
Statement: A spring made from ASTM A228 wire with ends squared and ground, wire diameter d = 3 mm,
outside diameter Do = 27 mm, 14 total coils, and free length Lf = 80 mm has been chosen for an
application. Determine the static safety factor if the spring is subjected to a static load of 175
N.
Given: Working force Fwork  175  N Shear modulus G  80 GPa
Wire diameter d  3  mm
Outside diameter Do  27 mm Total coils Nt  14
Free length Lf  80 mm ASTM A228 wire A  2153.5 MPa
Ends squared and ground Ne  2 b  0.1625

Assumption: Set removed


Solution: See Mathcad file P1448.
1. Determine the mean diameter, spring index, shear factor, number of active coils, and spring rate.
Mean diameter D  Do  d D  24 mm
D
Spring index C  C8
d
0.5
Shear factor Ks  1  Na  Ks  1.063
C
Number of active coils Na  Nt  Ne Na  12
4
d G N
Spring rate k  k  4.883 
3 mm
8  D  Na

2. Calculate the ultimate tensile strength of the wire material from equation 14.3 and Table 14-4 and use it to find
the torsional yield strength from Table 14-6.
b
S ut  A   d
Ultimate tensile strength  S ut  1801 MPa
 mm 
Shear yield strength S ys  0.60 S ut S ys  1081 MPa
3. Calculate the shear stress in the coil at the working load and the factor of safety against yielding under the
working load.
8  Fwork  D
Working shear stress τwork  Ks τwork  420.9  MPa
3
π d
S ys
Factor of safety Nswork  Nswork  2.6
τwork
4. Calculate the shear stress in the coil at shut height and the factor of safety against yielding at shut height.
Shut height Ls  d  Nt Ls  42 mm
Deflection to shut height yshut  Lf  Ls yshut  38 mm
Force at shut height Fs  k yshut Fs  185.547  N

8  Fs  D
Shut height stress τs  Ks τs  446.2  MPa
3
π d
S ys
Factor of safety Nss  Nss  2.4
τs
© 2011 Pearson Education, Inc., Upper Saddle River, NJ. All rights reserved. This publication is protected by Copyright and written permission should be
MACHINE DESIGN - An Integrated Approach, 4th Ed. 14-49-1

PROBLEM 14-49
Statement: A spring with ends squared and ground, wire diameter d = 4 mm, outside diameter Do = 40
mm, 18 total coils, and free length Lf = 140 mm has been chosen for an application where the
initial deflection is 15 mm and the working deflection is 50 mm. Determine minimum working
length, shut height, clash allowance, spring index, and spring rate for this spring.

Given: Working deflection ywork  50 mm Initial deflection yinit  15 mm


Wire diameter d  4  mm Shear modulus G  80 GPa
Outside diameter Do  40 mm Total coils Nt  18
Free length Lf  140  mm
Ends squared and ground Ne  2

Solution: See Mathcad file P1449.


1. Determine the mean diameter, spring index, number of active coils, and spring rate.
Mean diameter D  Do  d D  36 mm
D
Spring index C  C9
d
Number of active coils Na  Nt  Ne Na  16
4
d G N
Spring rate k  k  3.429 
3 mm
8  D  Na

2. Calculate the minimum working length, shut height, and clash allowance.
Minimum work length Lmin  Lf  yinit  ywork Lmin  75 mm

Shut height Ls  d  Nt Ls  72 mm

Lmin  Ls
Clash allowance α  α  6 %
ywork

© 2011 Pearson Education, Inc., Upper Saddle River, NJ. All rights reserved. This publication is protected by Copyright and written permission should be
MACHINE DESIGN - An Integrated Approach, 4th Ed. 14-50-1

PROBLEM 14-50
Statement: An extension spring has wire diameter d = 3 mm and outside diameter Do = 27 mm. Determine
the preferred preload for this spring.

Given: Wire diameter d  3  mm Outside diameter Do  27 mm

Solution: See Mathcad file P1450.


1. Determine the mean diameter, spring index, and shear factor.
Mean diameter D  Do  d D  24 mm
D
Spring index C  C8
d
0.5
Shear factor Ks  1  Na  Ks  1.063
C
2. Use the value of C to find an appropiate value of initial coil stress i from equations 14.21:

 3 2 
τi1  4.231  C  181.5  C  3387 C  28640  psi τi1  75.8 MPa

  2.987  C  139.7  C  3427 C  38404   psi


3 2
τi2 τi2  126.9  MPa

τi1  τi2
τi  τi  101.3  MPa
2

3. Use the value of i from step 2 in equation 14.8b to find the corresponding initial coil-tension force Fi:

3
π d  τi
Fi  Fi  42.1 N
8  K s D

© 2011 Pearson Education, Inc., Upper Saddle River, NJ. All rights reserved. This publication is protected by Copyright and written permission should be
MACHINE DESIGN - An Integrated Approach, 4th Ed. 13-51-1

PROBLEM 14-51
Statement: A series-stack of Belleville springs is required for more deflection in a design. The stack will
be guided by an internal pin as shown in Figure 14-35(b). The minimum inside diameter of the
individual springs in the stack is Di = 25 mm. Determine the recommended surface conditions
for the pin and the maximum pin diamter. (Hint: go to Spirol.com on the internet and look up
disc springs, stacking).

Given: Inside diameter Di  25 mm

Solution: See Figure 14-35(b) and Mathcad file P1451.


1. The Spirol recommended surface conditions are "case hardened to a depth not less than 0.6 mm and a hardness
of 58 HRC. A surface finish of less than or equal to 4 microns is also recommended."

2. From the table of recommended clearances c  0.4 mm.

Maximum pin dia. d pin  Di  c d pin  24.6 mm

© 2011 Pearson Education, Inc., Upper Saddle River, NJ. All rights reserved. This publication is protected by Copyright and written permission should be
MACHINE DESIGN - An Integrated Approach, 4th Ed. 14-52-1

PROBLEM 14-52
Statement: Equation 14.9a defines a combined direct shear and stress concentration factor, Kw, to be used
with helical springs that are made with round wire. Equation 14.11 defines a similar factor, Krw,
that is to be used to calculate shear stress when the spring is made with rectangular wire.
Determine and plot the ratio Krw/Kw for square wire with values of spring index, C, ranging from
1.2 to 10.
Given: Constant values from Table 14-6 for b/h = 1/1:
S 0  1.6844 S 1  2.8219 S 2  2.4577 S 3  1.0591 S 4  0.1721

Solution: See Table 14-6 and Mathcad file P1452.

1. The Wahl factor for round wire is, from equation 14.9a:

4 C  1 0.615
Kw( C)  
4 C  4 C

2. The stress concentration factor for rectangular wire is, from equation 14.11:

s( C)  ln( C)

Krw ( C)  e
 S S  s( C) S  s( C) S  s( C) S  s( C) 
0 1 2
2
3
3
4
4

3. Plot the ratio of these two factors over the range: C  1.2 1.21  10

STRESS CONCENTRATION FACTOR RATIO FOR SQUARE vs. ROUND WIRE


1
Stress Concentration Factor Ratio, Square/Round

0.9

Krw ( C)
0.8
Kw( C)

0.7

0.6
0 2 4 6 8
C
Spring Index, C

© 2011 Pearson Education, Inc., Upper Saddle River, NJ. All rights reserved. This publication is protected by Copyright and written permission should be
MACHINE DESIGN - An Integrated Approach, 4th Ed. 14-53-1

PROBLEM 14-53
Statement: Repeat Problem 14-6 with 1-mm square wire instead of round wire.
Given: Wire dimension b  1  mm h  1  mm
Mean coil dia D  10 mm Total coils Nt  12

Assumptions: The spring wire is steel so that G  80.8 GPa.


Solution: See Table 14-7 and Mathcad file P1453.
1. From Figure 14-9, the number of active coils is Na  Nt  2 Na  10

2. Using equation 14.11d and Table 14-7,

Shape factor K1  0.180

3
b h  G N
Spring rate k  K1 k  1.454 
3 mm
D  Na

3. Using equation 14.5,


D
Spring Index C  C  10
b

© 2011 Pearson Education, Inc., Upper Saddle River, NJ. All rights reserved. This publication is protected by Copyright and written permission should be
MACHINE DESIGN - An Integrated Approach, 4th Ed. 14-54-1
PROBLEM 14-54
Statement: A helical compression spring is required to provide a minimum force of 650 N at installation and
have a working deflection of 25 mm. The spring rate is 13 N/mm. The coil must fit in a 53-mm-dia
hole with 3-mm clearance. Use 6-mm square, music wire with squared/ground ends. Using a 15%
clash allowance, find:
(a) The stress at the working deflection.
(b) The shut height.
(c) The stress at the shut height.
(d) The total number of coils.
(e) The free length.
(f) The natural frequency in Hz.
1
Given: Minimum force Fmin  650  N Spring rate k  13 N  mm
Spring OD OD  50 mm Shear modulus G  80.8 GPa
Working deflection ∆y  25 mm Clash factor α  0.15
Music wire dims: b  6  mm h  6  mm
Constant values from Table 14-6 for b/h = 1/1:
S 0  1.6844 S 1  2.8219 S 2  2.4577 S 3  1.0591 S 4  0.1721
Solution: See Mathcad file P1454.
1. From the given information, find the maximum force and the spring index.

Maximum force Fmax  Fmin  k ∆y Fmax  975  N

OD  b
Spring index C  C  7.333
b
2. Calculate the stress concentration factor from equation 14.11a.
s  ln( C)

Krw  e
 S S  s S  s  S  s  S  s 
0 1 2
2
3
3
4
4
Krw  1.166

3. Calculate the mean coil diameter D from equation 14.5.

Mean coil diameter D  C b D  44.0 mm


4. Calculate the shear stress w at the working deflection using the shape factor from Table 14-7 and equation
14.11b.

Shape factors K1  0.180 K2  2.41

K2 Fmax D
Stress at Fmax τw  Krw  τw  558.2  MPa
2
b h

5. To get the defined spring rate, the number of active coils must satisfy equation 14.11d, solving for Na yields:

3
b h  G
Number of active Na  K1 Na  17.021 Na  17
coils 3
D k
Note that we round it to the nearest 1/4 coil as the manufacturing tolerance cannot achieve better than that
accuracy. Having rounded the number of active coils, we must now calculate the spring rate using equation
14.11d:
3
Corrected b h  G N
spring rate k  K1 k  13.02 
3 mm
D  Na
© 2011 Pearson Education, Inc., Upper Saddle River, NJ. All rights reserved. This publication is protected by Copyright and written permission should be
MACHINE DESIGN - An Integrated Approach, 4th Ed. 14-54-2
6.For squared ends the total number of coils, from Figure 14-9:
Total coils Nt  Na  2 Nt  19.00

7. The shut height can now be determined.


Shut height Lshut  h  Nt Lshut  114  mm
8. The initial deflection to reach the smaller of the two loads is
Fmin
Initial deflection yinit  yinit  49.94  mm
k
9. For the given clash allowance factor:
Clash allowance ∆yclash  α ∆y ∆yclash  3.75 mm
10. The free length (see Figure 14-8) can now be found from
Lf  Lshut  ∆yclash  ∆y  yinit Lf  193  mm

11. The deflection to the shut height is


yshut  Lf  Lshut yshut  79 mm

12. The force at this shut height is


Fshut  k yshut Fshut  1024 N

13. The shut-height stress is


K2 Fshut D
Stress at Fshut τshut  Krw  τshut  586.3  MPa
2
b h

14. The inside and outside coil diameters are


Inside coil dia Di  D  b Di  38 mm
Outside coil dia Do  D  b Do  50 mm
15. The smallest hole and largest pin that should be used with this spring are
Smallest hole holemin  Do  0.05 D holemin  52.2 mm

Largest pin pin max  Di  0.05 D pin max  35.8 mm

16. The weight of the spring's active coils is found from equation 14.11b
3
Weight density ρ  0.285  lbf  in

Weight Wa  b  h  π D Na ρ Wa  6.545  N

17. The natural frequency of this spring is found from equation 14.11a and is:

1 k g
Natural frequency fn   fn  69.83  Hz
2 Wa

© 2011 Pearson Education, Inc., Upper Saddle River, NJ. All rights reserved. This publication is protected by Copyright and written permission should be
MACHINE DESIGN - An Integrated Approach, 4th Ed. 14-54-1
PROBLEM 14-54
Statement: A helical compression spring is required to provide a minimum force of 650 N at installation and
have a working deflection of 25 mm. The spring rate is 13 N/mm. The coil must fit in a 53-mm-dia
hole with 3-mm clearance. Use 6-mm square, music wire with squared/ground ends. Using a 15%
clash allowance, find:
(a) The stress at the working deflection.
(b) The shut height.
(c) The stress at the shut height.
(d) The total number of coils.
(e) The free length.
(f) The natural frequency in Hz.
1
Given: Minimum force Fmin  650  N Spring rate k  13 N  mm
Spring OD OD  50 mm Shear modulus G  80.8 GPa
Working deflection ∆y  25 mm Clash factor α  0.15
Music wire dims: b  6  mm h  6  mm
Constant values from Table 14-6 for b/h = 1/1:
S 0  1.6844 S 1  2.8219 S 2  2.4577 S 3  1.0591 S 4  0.1721
Solution: See Mathcad file P1454.
1. From the given information, find the maximum force and the spring index.

Maximum force Fmax  Fmin  k ∆y Fmax  975  N

OD  b
Spring index C  C  7.333
b
2. Calculate the stress concentration factor from equation 14.11a.
s  ln( C)

Krw  e
 S S  s S  s  S  s  S  s 
0 1 2
2
3
3
4
4
Krw  1.166

3. Calculate the mean coil diameter D from equation 14.5.

Mean coil diameter D  C b D  44.0 mm


4. Calculate the shear stress w at the working deflection using the shape factor from Table 14-7 and equation
14.11b.

Shape factors K1  0.180 K2  2.41

K2 Fmax D
Stress at Fmax τw  Krw  τw  558.2  MPa
2
b h

5. To get the defined spring rate, the number of active coils must satisfy equation 14.11d, solving for Na yields:

3
b h  G
Number of active Na  K1 Na  17.021 Na  17
coils 3
D k
Note that we round it to the nearest 1/4 coil as the manufacturing tolerance cannot achieve better than that
accuracy. Having rounded the number of active coils, we must now calculate the spring rate using equation
14.11d:
3
Corrected b h  G N
spring rate k  K1 k  13.02 
3 mm
D  Na
© 2011 Pearson Education, Inc., Upper Saddle River, NJ. All rights reserved. This publication is protected by Copyright and written permission should be
MACHINE DESIGN - An Integrated Approach, 4th Ed. 14-54-2
6.For squared ends the total number of coils, from Figure 14-9:
Total coils Nt  Na  2 Nt  19.00

7. The shut height can now be determined.


Shut height Lshut  h  Nt Lshut  114  mm
8. The initial deflection to reach the smaller of the two loads is
Fmin
Initial deflection yinit  yinit  49.94  mm
k
9. For the given clash allowance factor:
Clash allowance ∆yclash  α ∆y ∆yclash  3.75 mm
10. The free length (see Figure 14-8) can now be found from
Lf  Lshut  ∆yclash  ∆y  yinit Lf  193  mm

11. The deflection to the shut height is


yshut  Lf  Lshut yshut  79 mm

12. The force at this shut height is


Fshut  k yshut Fshut  1024 N

13. The shut-height stress is


K2 Fshut D
Stress at Fshut τshut  Krw  τshut  586.3  MPa
2
b h

14. The inside and outside coil diameters are


Inside coil dia Di  D  b Di  38 mm
Outside coil dia Do  D  b Do  50 mm
15. The smallest hole and largest pin that should be used with this spring are
Smallest hole holemin  Do  0.05 D holemin  52.2 mm

Largest pin pin max  Di  0.05 D pin max  35.8 mm

16. The weight of the spring's active coils is found from equation 14.11b
3
Weight density ρ  0.285  lbf  in

Weight Wa  b  h  π D Na ρ Wa  6.545  N

17. The natural frequency of this spring is found from equation 14.11a and is:

1 k g
Natural frequency fn   fn  69.83  Hz
2 Wa

© 2011 Pearson Education, Inc., Upper Saddle River, NJ. All rights reserved. This publication is protected by Copyright and written permission should be
MACHINE DESIGN - An Integrated Approach, 4th Ed. 15-1-1
PROBLEM 15-1
Statement: Compare the tensile load capacity of a 5/16-18 UNC thread and a 5/16-24 UNF thread made of the
same material. Which is stronger? Make the same comparison for M8 x 1.25 and M8 x 1 ISO
threads. Compare them all to the strength of a 5/16-14 Acme thread.

Assumptions: The material strength for all threads is S  100  ksi.


Solution: See Mathcad file P1501.

1. Get the tensile stress area for each thread specification from Tables 15-1 (for Unified National Standard sizes),
15-2 (for metric sizes), and 15-3 (for Acme threads).
Thread specification Tensile Stress Area
2
5/16-18 UNC a tUNC  0.0524 in
2
5/16-24 UNF a tUNF  0.0581 in
2
M8 x 1.25 a tMC  36.61  mm
2
M8 x 1 a tMF  39.17  mm
2
5/16-14 Acme a tAcme  0.053  in

2. Using the assumed strength, determine the allowable load for each thread specification.

Thread specification Equation Allowable Load


5/16-18 UNC FUNC  a tUNC S FUNC  5240 lbf
5/16-24 UNF FUNF  a tUNF  S FUNF  5810 lbf
M8 x 1.25 FMC  a tMC S FMC  25.24  kN
M8 x 1 FMF  a tMF  S FMF  27.01  kN
5/16-14 Acme FAcme  a tAcme S FAcme  5300 lbf
FAcme  23.58  kN

3. State the conclusions.

1. The fine thread has a higher capacity than the coarse thread for both the UNS and ISO
threads.
2. The Acme thread has a higher capacity than the UNS coarse thread, but has a lower
cpacity than the fine UNS thread.
3. The Acme thread has a lower capacity than either of the metric threads.

© 2011 Pearson Education, Inc., Upper Saddle River, NJ. All rights reserved. This publication is protected by Copyright and written permission should be
MACHINE DESIGN - An Integrated Approach, 4th Ed. 15-2-1

PROBLEM 15-2
Statement: A 3/4-6 Acme thraed screw is used to lift a 2-kN load. The mean collar diameter is 40 mm. Find
the torque to lift and to lower the load using a ball-bearing thrust washer. What are the
efficiencies? Is it sel-locking?

Given: Screw diameter d  0.750  in Applied load P  2  kN


Collar diameter d c  40 mm Radial thread angle α  14.5 deg
1
Threads per inch Nt  6  in

Assumptions: 1. The thread coefficient of friction is μ  0.15.


2. The collar coefficient of friction is μc  0.02.

Solution: See and Mathcad file P1502.

1. Get the thread pitch diameter from Table 15-3. d p  0.667  in


1
2. Determine the thread pitch and lead. p  p  0.167  in
Nt
L  p L  0.167  in

3. Use equations 15.5 to determine the lifting (up) and lowering (down) torques.

P d p μ  π dp  L cos( α)  dc
Tu    μc  P Tu  42.68  in lbf
2 π dp cos( α)  μ  L 2

P d p  μ  π d p  L cos( α)  dc
Td    μc  P Td  18.25  in lbf
2  π d p cos( α)  μ  L 2

4. Use equation 15.7c to determine the lifting (up) and lowering (down) efficiencies.
P L
eu  eu  27.9 %
2  π Tu

P L
ed  ed  65.4 %
2  π Td

5. Use equation 15.6a to determine if the screw is self-locking.

L
self_locking  return "yes" if μ   cos( α)
π d p
"no" otherwise

self_locking  "yes"

© 2011 Pearson Education, Inc., Upper Saddle River, NJ. All rights reserved. This publication is protected by Copyright and written permission should be
MACHINE DESIGN - An Integrated Approach, 4th Ed. 15-3-1

PROBLEM 15-3
Statement: A 1 3/8-4 Acme thraed screw is used to lift a 1-ton load. The mean collar diameter is 2 in. Find
the torque to lift and to lower the load using a ball-bearing thrust washer. What are the
efficiencies? Is it sel-locking?

Given: Screw diameter d  1.375  in Applied load P  2000 lbf


Collar diameter d c  2.00 in Radial thread angle α  14.5 deg
1
Threads per inch Nt  4  in

Assumptions: 1. The thread coefficient of friction is μ  0.15.


2. The collar coefficient of friction is μc  0.02.

Solution: See and Mathcad file P1503.

1. Get the thread pitch diameter from Table 15-3. d p  1.250  in


1
2. Determine the thread pitch and lead. p  p  0.250  in
Nt
L  p L  0.250  in

3. Use equations 15.5 to determine the lifting (up) and lowering (down) torques.

P d p μ  π dp  L cos( α)  dc
Tu    μc  P Tu  316.0  in lbf
2 π dp cos( α)  μ  L 2

P d p  μ  π d p  L cos( α)  dc
Td    μc  P Td  153.0  in lbf
2  π d p cos( α)  μ  L 2

4. Use equation 15.7c to determine the lifting (up) and lowering (down) efficiencies.
P L
eu  eu  25.2 %
2  π Tu

P L
ed  ed  52.0 %
2  π Td

5. Use equation 15.6a to determine if the screw is self-locking.

L
self_locking  return "yes" if μ   cos( α)
π d p
"no" otherwise

self_locking  "yes"

© 2011 Pearson Education, Inc., Upper Saddle River, NJ. All rights reserved. This publication is protected by Copyright and written permission should be
MACHINE DESIGN - An Integrated Approach, 4th Ed. 15-4-1

PROBLEM 15-4
Statement: The trailer hitch from Figure 1-1 (p. 12) has loads applied as shown in Figure P3-2. The tongue
weight of 100 kg acts downward and the pull force of 4905 N acts horizontally. Using the
dimensions of the ball bracket in Figure 1-5 (p. 15), draw a free-body diagram of the ball bracket
and find the tensile and shear loads applied to the two bolts that attach the bracket to the
channel in Figure 1-1. Size and specify the bolts and their preload for a safety factor of at least
1.7.
Given: Hitch dimensions: a  40 mm Tongue weight Mtongue  100  kg
b  31 mm Pull force Fpull  4.905  kN
c  70 mm Number of bolts Nbolts  2
d  20 mm Young's modulus E  206.8  GPa
t  19 mm Design safety factor Nd  1.7
Bolt modulus Ebolt  E
Member modulus Ememb  E

Assumptions: The shear load will be taken by friction between the hitch and the support.
Design Choices:
Use M12 x 1.75 , class 8.8 bolts.
Material properties for class 8.8: Bolt diameter d b  12 mm
Proof strength S p  600  MPa Clamp length l  30 mm
Yield strength S y  660  MPa Preload fraction fp  0.59

W tongue
70 = c

1 F pull 1

40 = a

2 A 2
B 19 = t B

31 = b Fc2x
C C

20 = d
D
D Fd2

F c2y

FIGURE S15-4
Dimensions and Free Body Diagram for Problem 15-4

Solution: See Figure S15-4 and Mathcad file P1504.


1. The weight on the tongue is

Wtongue  Mtongue g Wtongue  0.981  kN

2. The FBD of the hitch and bracket assembly is shown in Figure 3-4. The known external forces that act on the
ball are Fpull and Wtongue . The reactions on the bracket are at points C and D. The bolts at C provide tensile
(Fc2x) and shear (Fc2y) forces, and the bracket resists rotation about point D where the reaction force Fd2 is
applied by the channel to which the bracket is bolted.
© 2011 Pearson Education, Inc., Upper Saddle River, NJ. All rights reserved. This publication is protected by Copyright and written permission should be
MACHINE DESIGN - An Integrated Approach, 4th Ed. 15-4-2

3. Solving for the reactions by summing the horizontal and vertical forces and the moments about D:

 Fx :  Fpull  Fc2x  Fd2 = 0 (1)

 Fy : Fc2y  Wtongue = 0 (2)

 MD : Fc2x d  Fpull  ( a  t  b  d )  Wtongue c = 0 (3)

4. Solving equation (3) for Fc2x

Fpull  ( a  t  b  d )  Wtongue c
Fc2x  Fc2x  30.41  kN (4)
d

5. Substituting into (1) and solving for Fd2


Fd2  Fc2x  Fpull Fd2  25.505 kN (5)

6. Solving (2) for Fc2y


Fc2y  Wtongue Fc2y  0.981  kN (6)

7. The loads applied to the two bolts that attach the bracket to the channel are:

Axial force on two bolts Fc2x  30.4 kN Ptot  Fc2x

Shear force taken by friction Fc2y  0.98 kN

Ptot
8. Determine the load per bolt. P  P  15.20  kN
Nbolts
2
9. Get the tensile stress area from Table 15-2. At  84.27  mm

10. Calculate the preload. Fi  fp  S p At Fi  29.83  kN

11. Determine the relevant ratios for this joint from equations 15-18a and b.
db
Joint aspect ratio: j  j  0.400
l
Ememb
Plate to bolt modulus: r  r1
Ebolt

12. Calculate Cr = C using equation 15.19 and the coefficients p i from Table 15-8 for j  0.4 .

Coefficients from Table 15-8: p 0  0.7351

p 1  1.2612

p 2  1.1111

p 3  0.3779

3 2
Joint stiffness constant: C  p 3  r  p 2  r  p 1  r  p 0 C  0.207

© 2011 Pearson Education, Inc., Upper Saddle River, NJ. All rights reserved. This publication is protected by Copyright and written permission should be
MACHINE DESIGN - An Integrated Approach, 4th Ed. 15-4-3

13. The portions of the applied load P felt by the bolt and the material can now be found from equations 15.13.
Pb  C P Pb  3.15 kN

Pm  ( 1  C)  P Pm  12.1 kN
14. Find the resulting loads in bolt and material after the load P is applied.
Fb  Fi  Pb Fb  32.98  kN

Fm  Fi  Pm Fm  17.78  kN

15. The maximum tensile stress in the bolt is


Fb
σb  σb  391.4  MPa
At

16. This is a uniaxial stress situation, so the principal stress and von Mises stress are identical to the applied tensile
stress. The safety factor against yielding for class 8.8 with S y  660  MPa is then

Sy
Ny  Ny  1.7
σb

17. The load required to separate the joint and the safety factor against joint separation are found from equations
15.14c and 15.14d.

Fi
P0  P0  37.6 kN
1C
P0
Nsep  Nsep  2.5
P

© 2011 Pearson Education, Inc., Upper Saddle River, NJ. All rights reserved. This publication is protected by Copyright and written permission should be
MACHINE DESIGN - An Integrated Approach, 4th Ed. 15-5-1

PROBLEM 15-5
Statement: For the hitch of Problem 3-4, determine the horizontal force that will result on the ball from
accelerating a 2000-kg trailer to 60 m/sec in 20 sec. Size and specify the bolts and their preload
for a safety factor of at least 1.7.
Given: Hitch dimensions: a  40 mm Tongue weight Mtongue  100  kg
b  31 mm Pull force Fpull  6  kN
c  70 mm Number of bolts Nbolts  2
d  20 mm Young's modulus E  206.8  GPa
t  19 mm Design safety factor Nd  1.7
Bolt modulus Ebolt  E
Member modulus Ememb  E

Assumptions: The shear load will be taken by friction between the hitch and the support.
Design Choices:
Use M12 x 1.75 , class 8.8 bolts.
Material properties for class 8.8: Bolt diameter d b  12 mm
Proof strength S p  600  MPa Clamp length l  30 mm
Yield strength S y  660  MPa Preload fraction fp  0.58

W tongue
70 = c

1 F pull 1

40 = a

2 A 2
B 19 = t B

31 = b Fc2x
C C

20 = d
D
D Fd2

F c2y

FIGURE 15-5
Dimensions and Free Body Diagram for Problem 15-5

Solution: See Figure 15-5 and Mathcad file P1505.


1. The weight on the tongue is

Wtongue  Mtongue g Wtongue  0.981  kN

2. The FBD of the hitch and bracket assembly is shown in Figure 3-4. The known external forces that act on the
ball are Fpull and Wtongue . The reactions on the bracket are at points C and D. The bolts at C provide tensile
(Fc2x) and shear (Fc2y) forces, and the bracket resists rotation about point D where the reaction force Fd2 is
applied by the channel to which the bracket is bolted.
© 2011 Pearson Education, Inc., Upper Saddle River, NJ. All rights reserved. This publication is protected by Copyright and written permission should be
MACHINE DESIGN - An Integrated Approach, 4th Ed. 15-5-2

3. Solving for the reactions by summing the horizontal and vertical forces and the moments about D:

 Fx :  Fpull  Fc2x  Fd2 = 0 (1)

 Fy : Fc2y  Wtongue = 0 (2)

 MD : Fc2x d  Fpull  ( a  t  b  d )  Wtongue c = 0 (3)

4. Solving equation (3) for Fc2x

Fpull  ( a  t  b  d )  Wtongue c
Fc2x  Fc2x  36.43  kN (4)
d

5. Substituting into (1) and solving for Fd2


Fd2  Fc2x  Fpull Fd2  30.43  kN (5)

6. Solving (2) for Fc2y


Fc2y  Wtongue Fc2y  0.981  kN (6)

7. The loads applied to the two bolts that attach the bracket to the channel are:

Axial force on two bolts Fc2x  36.4 kN Ptot  Fc2x

Shear force taken by friction Fc2y  0.98 kN

Ptot
8. Determine the load per bolt. P  P  18.22  kN
Nbolts
2
9. Get the tensile stress area from Table 15-2. At  84.27  mm

10. Calculate the preload. Fi  fp  S p At Fi  29.33  kN

11. Determine the relevent ratios for this joint from equations 15-18a and b.
db
Joint aspect ratio: j  j  0.400
l
Ememb
Plate to bolt modulus: r  r1
Ebolt

12. Calculate Cr = C using equation 15.19 and the coefficients p i from Table 15-8 for j  0.4 .

Coeficients from Table 15-8: p 0  0.7351

p 1  1.2612

p 2  1.1111

p 3  0.3779

3 2
Joint stiffness constant: C  p 3  r  p 2  r  p 1  r  p 0 C  0.207

© 2011 Pearson Education, Inc., Upper Saddle River, NJ. All rights reserved. This publication is protected by Copyright and written permission should be
MACHINE DESIGN - An Integrated Approach, 4th Ed. 15-5-3

13. The portions of the applied load P felt by the bolt and the material can now be found from equations 15.13.
Pb  C P Pb  3.77 kN

Pm  ( 1  C)  P Pm  14.4 kN
14. Find the resulting loads in bolt and material after the load P is applied.
Fb  Fi  Pb Fb  33.10  kN

Fm  Fi  Pm Fm  14.88  kN

15. The maximum tensile stress in the bolt is


Fb
σb  σb  392.8  MPa
At

16. This is a uniaxial stress situation, so the principal stress and von Mises stress are identical to the applied tensile
stress. The safety factor against yielding for class 8.8 with S y  660  MPa is then

Sy
Ny  Ny  1.7
σb

17. The load required to separate the joint and the safety factor against joint separation are found from equations
15.14c and 15.14d.

Fi
P0  P0  37.0 kN
1C
P0
Nsep  Nsep  2.0
P

© 2011 Pearson Education, Inc., Upper Saddle River, NJ. All rights reserved. This publication is protected by Copyright and written permission should be
MACHINE DESIGN - An Integrated Approach, 4th Ed. 15-6-1

PROBLEM 15-6
Statement: For the trailer hitch of Problem 3-4, determine the horizontal force that will result on the ball from
an impact between the ball and the tongue of the 2000-kg trailer if the hitch deflects 2.8 mm
dynamically on impact. Size and specify the bolts and their preload for a safety factor of at least
1.7.

Given: Hitch dimensions: a  40 mm Tongue weight Mtongue  100  kg


b  31 mm Pull force Fpull  55.1 kN
c  70 mm Number of bolts Nbolts  2
d  20 mm Young's modulus E  206.8  GPa
t  19 mm Design safety factor Nd  1.7
Bolt modulus Ebolt  E
Member modulus Ememb  E

Assumptions: The shear load will be taken by friction between the hitch and the support.
Design Choices:
Use M24 x 3 , class 12.9 bolts.
Material properties for class 12.9: Bolt diameter d b  24 mm
Proof strength S p  970  MPa Clamp length l  30 mm
Yield strength S y  1100 MPa Preload fraction fp  0.55

W tongue
70 = c

1 F pull 1

40 = a

2 A 2
B 19 = t B

31 = b Fc2x
C C

20 = d
D
D Fd2

F c2y

FIGURE 15-6
Dimensions and Free Body Diagram for Problem 15-6

Solution: See Figure 15-6 and Mathcad file P1506.


1. The weight on the tongue is

Wtongue  Mtongue g Wtongue  0.981  kN

2. The FBD of the hitch and bracket assembly is shown in Figure 3-4. The known external forces that act on the
ball are Fpull and Wtongue . The reactions on the bracket are at points C and D. The bolts at C provide tensile
(Fc2x) and shear (Fc2y) forces, and the bracket resists rotation about point D where the reaction force Fd2 is
applied by the channel to which the bracket is bolted.
© 2011 Pearson Education, Inc., Upper Saddle River, NJ. All rights reserved. This publication is protected by Copyright and written permission should be
MACHINE DESIGN - An Integrated Approach, 4th Ed. 15-6-2

3. Solving for the reactions by summing the horizontal and vertical forces and the moments about D:

 Fx :  Fpull  Fc2x  Fd2 = 0 (1)

 Fy : Fc2y  Wtongue = 0 (2)

 MD : Fc2x d  Fpull  ( a  t  b  d )  Wtongue c = 0 (3)

4. Solving equation (3) for Fc2x

Fpull  ( a  t  b  d )  Wtongue c
Fc2x  Fc2x  306.48 kN (4)
d

5. Substituting into (1) and solving for Fd2


Fd2  Fc2x  Fpull Fd2  251.38 kN (5)

6. Solving (2) for Fc2y


Fc2y  Wtongue Fc2y  0.981  kN (6)

7. The loads applied to the two bolts that attach the bracket to the channel are:

Axial force on two bolts Fc2x  306.5  kN Ptot  Fc2x

Shear force taken by friction Fc2y  0.98 kN

Ptot
8. Determine the load per bolt. P  P  153.24 kN
Nbolts
2
9. Get the tensile stress area from Table 15-2. At  352.5  mm

10. Calculate the preload. Fi  fp  S p At Fi  188.06 kN

11. Determine the relevant ratios for this joint from equations 15-18a and b.
db
Joint aspect ratio: j  j  0.800
l
Ememb
Plate to bolt modulus: r  r1
Ebolt

12. Calculate Cr = C using equation 15.19 and the coefficients p i from Table 15-8 for j  0.8 .

Coefficients from Table 15-8: p 0  0.7800

p 1  1.2503

p 2  1.0672

p 3  0.3571

3 2
Joint stiffness constant: C  p 3  r  p 2  r  p 1  r  p 0 C  0.240

© 2011 Pearson Education, Inc., Upper Saddle River, NJ. All rights reserved. This publication is protected by Copyright and written permission should be
MACHINE DESIGN - An Integrated Approach, 4th Ed. 15-6-3

13. The portions of the applied load P felt by the bolt and the material can now be found from equations 15.13.
Pb  C P Pb  36.75  kN

Pm  ( 1  C)  P Pm  116.5  kN
14. Find the resulting loads in bolt and material after the load P is applied.
Fb  Fi  Pb Fb  224.81 kN

Fm  Fi  Pm Fm  71.56  kN

15. The maximum tensile stress in the bolt is


Fb
σb  σb  637.7  MPa
At

16. This is a uniaxial stress situation, so the principal stress and von Mises stress are identical to the
3
applied tensile stress. The safety factor against yielding for class 8.8 with S y  1.1  10  MPa is then

Sy
Ny  Ny  1.7
σb

17. The load required to separate the joint and the safety factor against joint separation are found from equations
15.14c and 15.14d.

Fi
P0  P0  247.4  kN
1C
P0
Nsep  Nsep  1.6
P

© 2011 Pearson Education, Inc., Upper Saddle River, NJ. All rights reserved. This publication is protected by Copyright and written permission should be
MACHINE DESIGN - An Integrated Approach, 4th Ed. 15-7-1
PROBLEM 15-7
Statement: A 1/2-in dia UNC, class 7 bolt with rolled threads is preloaded to 80% of its proof strength when
clamping a 3-in-thick sandwich of solid steel. Find the safety factors against static yielding and
joint separation when a static 1000-lb external load is applied. Use 99% reliability.

Given: Bolt diameter d  0.500  in Material properties for class 7:


Preload fraction fp  0.80 Proof strength S p  105  ksi
Clamp length l  3  in Yield strength S y  115  ksi
Number of bolts Nbolts  1 Ultimate strength S ut  133  ksi
6
Applied load Ptot  1000 lbf Young's modulus E  30 10  psi
Bolt modulus Ebolt  E Member modulus Ememb  E

Solution: See Mathcad file P1507.


Ptot
1. Determine the load per bolt. P  P  1000 lbf
Nbolts
2
2. Get the tensile stress area from Table 15-1. At  0.1419 in

3. Calculate the preload. Fi  fp  S p At Fi  11920  lbf

4. Determine the relevant ratios for this joint from equations 15-17a and b.
d
Joint aspect ratio: j  j  0.167
l
Ememb
Plate to bolt modulus: r  r1
Ebolt

5. Calculate Cr = C using equation 15.18 and the coefficients p i from Table 15-8 for j  0.167 . Use linear
interpolation between ja  0.1 and jb  0.2

pb  pa
Interpolation equation: p  p a p b     j  ja   p a
jb  ja

p 0  p ( 0.4389 0.6118)
Coefficients from Table 15-8: p 0  0.5542
p 1  p ( 0.9197 1.1715)
p 1  1.0876
p 2  p ( 0.8901 1.0875)
p 2  1.0217
p 3  p ( 0.3187 0.3806)
p 3  0.3600

3 2
Joint stiffness constant: C  p 3  r  p 2  r  p 1  r  p 0 C  0.128

6. The portions of the applied load P felt by the bolt and the material can now be found from equations 15.13.
Pb  C P Pb  128.3  lbf

Pm  ( 1  C)  P Pm  871.7  lbf
7 Find the resulting loads in bolt and material after the load P is applied.
Fb  Fi  Pb Fb  12048  lbf
© 2011 Pearson Education, Inc., Upper Saddle River, NJ. All rights reserved. This publication is protected by Copyright and written permission should be
MACHINE DESIGN - An Integrated Approach, 4th Ed. 15-7-2

Fm  Fi  Pm Fm  11048  lbf

8 The maximum tensile stress in the bolt is

Fb
σb  σb  84.9 ksi
At

9. This is a uniaxial stress situation, so the principal stress and von Mises stress are identical to the applied
tensile stress. The safety factor against yielding for Grade 7 with S y  115  ksi is then

Sy
Ny  Ny  1.4
σb

10. The load required to separate the joint and the safety factor against joint separation are found from equations
15.14c and 15.14d.

Fi
P0  P0  13674  lbf
1C
P0
Nsep  Nsep  13.7
P

© 2011 Pearson Education, Inc., Upper Saddle River, NJ. All rights reserved. This publication is protected by Copyright and written permission should be
MACHINE DESIGN - An Integrated Approach, 4th Ed. 15-8-1

PROBLEM 15-8
Statement: An M14 x 2, class 8.8 bolt with rolled threads is preloaded to 75% of its proof strength when
clamping a 30-mm-thick sandwich of solid aluminum. Find the safety factors against static
yielding and joint separation when a static 5-kN external load is applied.
Given: Bolt diameter d  14 mm Material properties for class 8.8:
Preload fraction fp  0.75 Proof strength S p  600  MPa
Clamp length l  30 mm Yield strength S y  660  MPa
Number of bolts Nbolts  1 Ultimate strength S ut  830  MPa
Applied load Ptot  5  kN Young's modulus Ememb  71.7 GPa
Young's modulus Ebolt  206.8  GPa

Solution: See Table 15-8 and Mathcad file P1508.

Ptot
1. Determine the load per bolt. P  P  5  kN
Nbolts
2
2. Get the tensile stress area from Table 15-2. At  115.44 mm

3. Calculate the preload. Fi  fp  S p At Fi  52 kN

4. Determine the relevant ratios for this joint from equations 15-18a and b.
d
Joint aspect ratio: j  j  0.467
l
Ememb
Plate to bolt modulus: r  r  0.347
Ebolt

5. Calculate Cr = C using equation 15.19 and the coefficients p i from Table 15-8 for j  0.467 . Use linear
interpolation between ja  0.4 and jb  0.5

pb  pa
Interpolation equation: p  p a p b     j  ja   p a
jb  ja

Coefficients from Table 15-8: p 0  p ( 0.7351 0.7580) p 0  0.7504

p 1  p ( 1.2612 1.2632) p 1  1.2625

p 2  p ( 1.1111 1.0979) p 2  1.1023

p 3  p ( 0.3779 0.3708) p 3  0.3732

3 2
Joint stiffness constant: C  p 3  r  p 2  r  p 1  r  p 0 C  0.430

6. The portions of the applied load P felt by the bolt and the material can now be found from equations 15.13.
Pb  C P Pb  2.1 kN
Pm  ( 1  C)  P Pm  2.9 kN
7 Find the resulting loads in bolt and material after the load P is applied.
Fb  Fi  Pb Fb  54.1 kN
Fm  Fi  Pm Fm  49.1 kN
© 2011 Pearson Education, Inc., Upper Saddle River, NJ. All rights reserved. This publication is protected by Copyright and written permission should be
MACHINE DESIGN - An Integrated Approach, 4th Ed. 15-8-2

8 The maximum tensile stress in the bolt is

Fb
σb  σb  468.6  MPa
At

9. This is a uniaxial stress situation, so the principal stress and von Mises stress are identical to the applied
tensile stress. The safety factor against yielding for Grade 8.8 with S y  660  MPa is then

Sy
Ny  Ny  1.4
σb

10. The load required to separate the joint and the safety factor against joint separation are found from equations
15.14c and 15.14d.

Fi
P0  P0  91.1 kN
1C
P0
Nsep  Nsep  18.2
P

© 2011 Pearson Education, Inc., Upper Saddle River, NJ. All rights reserved. This publication is protected by Copyright and written permission should be
MACHINE DESIGN - An Integrated Approach, 4th Ed. 15-9-1
PROBLEM 15-9
Statement: A 7/16-in dia UNC, Grade 7 bolt with rolled threads is preloaded to 70% of its proof strength
when clamping a 2.75-in-thick sandwich of solid steel. Find the safety factors against fatigue
failure, yielding, and joint separation when a 1000-lb (peak) fluctuating external load is applied.
Use 99% reliability.

Given: Bolt diameter d  0.4375 in Material properties for Grade 7:


Preload fraction fp  0.70 Proof strength S p  105  ksi
Clamp length l  2.75 in Yield strength S y  115  ksi
Number of bolts Nbolts  1 Ultimate strength S ut  133  ksi
6
Applied load Ptot  1000 lbf Young's modulus E  30 10  psi
Bolt modulus Ebolt  E Member modulus Ememb  E

Solution: See Mathcad file P1509.


Ptot
1. Determine the load per bolt. Pmax  Pmax  1000 lbf Pmin  0  lbf
Nbolts

2
2. Get the tensile stress area from Table 15-1. At  0.1063 in

3. Calculate the preload. Fi  fp  S p At Fi  7813 lbf


4. Determine the relevant ratios for this joint from equations 15-18a and b.
d
Joint aspect ratio: j  j  0.159
l
Ememb
Plate to bolt modulus: r  r1
Ebolt

5. Calculate Cr = C using equation 15.19 and the coefficients p i from Table 15-8 for j  0.159 . Use linear
interpolation between ja  0.1 and jb  0.2

pb  pa
Interpolation equation: p  p a p b     j  ja   p a
jb  ja

Coefficients from Table 15-8: p 0  p ( 0.4389 0.6118) p 0  0.5411

p 1  p ( 0.9197 1.1715) p 1  1.0685

p 2  p ( 0.8901 1.0875) p 2  1.0067

p 3  p ( 0.3187 0.3806) p 3  0.3553

3 2
Joint stiffness constant: C  p 3  r  p 2  r  p 1  r  p 0 C  0.124

6. The portions of the applied load P felt by the bolt and the material can now be found from equations 15.13.
Pb  C Pmax Pb  124.0  lbf
Pm  ( 1  C)  Pmax Pm  876.0  lbf
7. Find the resulting loads in bolt and material after the load P is applied.
Fb  Fi  Pb Fb  7937 lbf
© 2011 Pearson Education, Inc., Upper Saddle River, NJ. All rights reserved. This publication is protected by Copyright and written permission should be
MACHINE DESIGN - An Integrated Approach, 4th Ed. 15-9-2

Fm  Fi  Pm Fm  6937 lbf

8. Calculate the alternating and mean components of the fluctuating bolt load.

Fb  Fi
Falt  Falt  62.0 lbf
2
Fb  Fi
Fmean  Fmean  7875 lbf
2
9. The nominal mean and alternating stresses in the bolt are:

Fmean
Nominal mean stress σmnom  σmnom  74.083 ksi
At
Falt
Nominal alternating stress σanom  σanom  583  psi
At

10. The fatigue stress-concentration factor for this thread is found from equation 15.15c and the mean
stress-concentration factor factor Kfm is found from equation 6.17.

0.6812 d
Fatigue factor Kf  5.7  Kf  6.00
in

Nominal maximum stress σmaxnom  σmnom  σanom σmaxnom  74.667 ksi

Nominal minimum stress σminnom  σmnom  σanom σminnom  73.5 ksi

Kfm  return Kf if Kf  σmaxnom  S y


S y  Kf  σanom
return if Kf  σmaxnom  S y Kfm  1.505
σmnom
return 0 if Kf  σmaxnom  σminnom  2  S y

Falt
σalt  Kf  σalt  3.50 ksi
At
Fmean
σmean  Kfm σmean  111.50 ksi
At

11. The stress at the initial preload is

Fi
σinit  Kfm σinit  110.62 ksi
At

12. An endurance strength must be found for this material. Using the methods of Section 6.6 we find for
S ut  133  ksi
S'e  0.5 S ut S'e  66.5 ksi

13. From the tables and formulas in Section 6.6 we have:


Load Cload  0.70

Size Csize  1

© 2011 Pearson Education, Inc., Upper Saddle River, NJ. All rights reserved. This publication is protected by Copyright and written permission should be
MACHINE DESIGN - An Integrated Approach, 4th Ed. 15-9-3

Surface A  2.70 b  0.265


b
 S ut 
Csurf  A    Csurf  0.739
 ksi 
Temperature Ctemp  1

Reliability Creliab  0.814

and the endurance limit is


S e  Cload  Csize Csurf  Ctemp Creliab S'e S e  28.00  ksi

14. The corrected endurance strength and the ultimate tensile strength are used in equation 15.16 to find the safety
factor from the Goodman line.
S e  S ut  σinit
Nf 
S e  σmean  σinit  S ut σalt Nf  1.3

15. Calculate the maximum bolt stress and the safety factor against yielding for S y  115  ksi.
Fb
σb  σb  74.667 ksi
At
Sy
Ny  Ny  1.5
σb

16. The load required to separate the joint and the safety factor against joint separation are found from equations
15.14c and 15.14d.

Fi
Nsep  Nsep  8.9
Pmax ( 1  C)

© 2011 Pearson Education, Inc., Upper Saddle River, NJ. All rights reserved. This publication is protected by Copyright and written permission should be
MACHINE DESIGN - An Integrated Approach, 4th Ed. 15-10-1
PROBLEM 15-10
Statement: An M12 x 1.25, class 9.8 bolt with rolled threads is preloaded to 85% of its proof strength when
clamping a 5-cm-thick sandwich of aluminum. Find the safety factors against fatigue failure,
yielding, and joint separation when a 2.5-kN (peak) fluctuating external load is applied. Use 99%
reliability.

Given: Bolt diameter d  12 mm Material properties for class 9.8 bolt:
Preload fraction fp  0.85 Proof strength S p  650  MPa
Clamp length l  50 mm Yield strength S y  720  MPa
Number of bolts Nbolts  1 Ultimate strength S ut  900  MPa
Applied load Ptot  2.5 kN Young's modulus Ebolt  206.8  GPa
Member modulus Ememb  71.7 GPa

Solution: See Mathcad file P1510.


Ptot
1. Determine the load per bolt. Pmax  Pmax  3  kN Pmin  0  kN
Nbolts

2
2. Get the tensile stress area from Table 15-1. At  92.07  mm

3. Calculate the preload. Fi  fp  S p At Fi  50.87  kN

4. Determine the relevant ratios for this joint from equations 15-18a and b.
d
Joint aspect ratio: j  j  0.240
l
Ememb
Plate to bolt modulus: r  r  0.347
Ebolt

5. Calculate Cr = C using equation 15.19 and the coefficients p i from Table 15-8 for j  0.24 . Use linear
interpolation between ja  0.2 and jb  0.3

pb  pa
Interpolation equation: p  p a p b     j  ja   p a
jb  ja

Coefficients from Table 15-8: p 0  p ( 0.6118 0.6932) p 0  0.6444

p 1  p ( 1.1715 1.2426) p 1  1.1999

p 2  p ( 1.0875 1.1177) p 2  1.0996

p 3  p ( 0.3806 0.3845) p 3  0.3822

3 2
Joint stiffness constant: C  p 3  r  p 2  r  p 1  r  p 0 C  0.345

6. The portions of the applied load P felt by the bolt and the material can now be found from equations 15.13.
Pb  C Pmax Pb  0.9 kN

Pm  ( 1  C)  Pmax Pm  1.6 kN
7. Find the resulting loads in bolt and material after the load P is applied.
Fb  Fi  Pb Fb  51.73  kN
© 2011 Pearson Education, Inc., Upper Saddle River, NJ. All rights reserved. This publication is protected by Copyright and written permission should be
MACHINE DESIGN - An Integrated Approach, 4th Ed. 15-10-2

Fm  Fi  Pm Fm  49.23  kN

8. Calculate the alternating and mean components of the fluctuating bolt load.

Fb  Fi
Falt  Falt  0.43 kN
2
Fb  Fi
Fmean  Fmean  51.3 kN
2
9. The nominal mean and alternating stresses in the bolt are:

Fmean
Nominal mean stress σmnom  σmnom  557.2  MPa
At
Falt
Nominal alternating stress σanom  σanom  4.68 MPa
At

10. The fatigue stress-concentration factor for this thread is found from equation 15.15c and the mean
stress-concentration factor factor Kfm is found from equation 6.17.

0.02682  d
Fatigue factor Kf  5.7  Kf  6
mm

Nominal maximum stress σmaxnom  σmnom  σanom σmaxnom  561.9  MPa

Nominal minimum stress σminnom  σmnom  σanom σminnom  552.5  MPa

Kfm  return Kf if Kf  σmaxnom  S y


S y  Kf  σanom
return if Kf  σmaxnom  S y Kfm  1.242
σmnom
return 0 if Kf  σmaxnom  σminnom  2  S y

11. The local mean and alternating stresses in the bolt are then:
Local mean stress σm  Kfm σmnom σm  691.8  MPa

Local alternating stress σa  Kf  σanom σa  28.2 MPa

12. The stress at the initial preload is

Fi
σinit  Kfm σinit  686.02 MPa
At

13. An endurance strength must be found for this material. Using the methods of Section 6.6 we find for
S ut  130.534  ksi

S'e  0.5 S ut S'e  450.0  MPa

14. From the tables and formulas in Section 6.6 we have:


Load Cload  0.70

Size Csize  1

© 2011 Pearson Education, Inc., Upper Saddle River, NJ. All rights reserved. This publication is protected by Copyright and written permission should be
MACHINE DESIGN - An Integrated Approach, 4th Ed. 15-10-3

Surface A  4.51 b  0.265


b
 Sut 
Csurf  A    Csurf  0.744
 MPa 
Temperature Ctemp  1

Reliability Creliab  0.814

and the endurance limit is


S e  Cload  Csize Csurf  Ctemp Creliab S'e S e  190.65 MPa

15. The corrected endurance strength and the ultimate tensile strength are used in equation 15.16 to find the
safety factor from the Goodman line.

S e  S ut  σinit
Nf  Nf  1.5
S e  σm  σinit  S ut σa

16. Calculate the maximum bolt stress and the safety factor against yielding for S y  720  MPa.

Fb
σb  σb  561.9  MPa
At
Sy
Ny  Ny  1.3
σb

17. The load required to separate the joint and the safety factor against joint separation are found from equations
15.14c and 15.14d.

Fi
Nsep  Nsep  31.0
Pmax ( 1  C)

© 2011 Pearson Education, Inc., Upper Saddle River, NJ. All rights reserved. This publication is protected by Copyright and written permission should be
MACHINE DESIGN - An Integrated Approach, 4th Ed. 15-11-1

PROBLEM 15-11
Statement: Find the tightening torque required for the bolt in Problem 15-7.

Given: Bolt diameter d  0.500  in Material properties for Grade 7:


Preload fraction fp  0.80 Proof strength S p  105  ksi
Applied load Ptot  1000 lbf Number of bolts Nbolts  1

Solution: See Mathcad file P1511.

Ptot
1. Determine the load per bolt. P  P  1000 lbf
Nbolts
2
2. Get the tensile stress area from Table 15-1. At  0.1419 in

3. Calculate the preload. Fi  fp  S p At Fi  11920  lbf

4. Calculate the tightening torque required for each bolt using equation 15.23.

Ti  0.21 Fi d Ti  1252 in lbf

© 2011 Pearson Education, Inc., Upper Saddle River, NJ. All rights reserved. This publication is protected by Copyright and written permission should be
MACHINE DESIGN - An Integrated Approach, 4th Ed. 15-12-1

PROBLEM 15-12
Statement: Find the tightening torque required for the bolt in Problem 15-8.

Given: Bolt diameter d  14 mm Material properties for class 8.8:


Preload fraction fp  0.75 Proof strength S p  600  MPa
Applied load Ptot  5  kN Number of bolts Nbolts  1

Solution: See Mathcad file P1512.

Ptot
1. Determine the load per bolt. P  P  5  kN
Nbolts
2
2. Get the tensile stress area from Table 15-1. At  115.44 mm

3. Calculate the preload. Fi  fp  S p At Fi  51.9 kN

4. Calculate the tightening torque required for each bolt using equation 15.23.

Ti  0.21 Fi d Ti  153  N  m

© 2011 Pearson Education, Inc., Upper Saddle River, NJ. All rights reserved. This publication is protected by Copyright and written permission should be
MACHINE DESIGN - An Integrated Approach, 4th Ed. 15-13-1
PROBLEM 15-13
Statement: Find the tightening torque required for the bolt in Problem 15-9.

Given: Bolt diameter d  0.4375 in Material properties for Grade 7:


Preload fraction fp  0.70 Proof strength S p  105  ksi
Applied load Ptot  5000 lbf Number of bolts Nbolts  1

Solution: See Mathcad file P1513.

Ptot
1. Determine the load per bolt. P  P  5000 lbf
Nbolts
2
2. Get the tensile stress area from Table 15-1. At  0.1063 in

3. Calculate the preload. Fi  fp  S p At Fi  7813 lbf

4. Calculate the tightening torque required for each bolt using equation 15.23.

Ti  0.21 Fi d Ti  718  in lbf

© 2011 Pearson Education, Inc., Upper Saddle River, NJ. All rights reserved. This publication is protected by Copyright and written permission should be
MACHINE DESIGN - An Integrated Approach, 4th Ed. 15-14-1

PROBLEM 15-14
Statement: Find the tightening torque required for the bolt in Problem 15-10.
Given: Bolt diameter d  12 mm Material properties for class 9.8:
Preload fraction fp  0.85 Proof strength S p  650  MPa
Applied load Ptot  20 kN Number of bolts Nbolts  1

Solution: See Mathcad file P1514.

Ptot
1. Determine the load per bolt. P  P  20 kN
Nbolts
2
2. Get the tensile stress area from Table 15-1. At  92.07  mm

3. Calculate the preload. Fi  fp  S p At Fi  50.9 kN

4. Calculate the tightening torque required for each bolt using equation 15.23.

Ti  0.21 Fi d Ti  128  N  m

© 2011 Pearson Education, Inc., Upper Saddle River, NJ. All rights reserved. This publication is protected by Copyright and written permission should be
MACHINE DESIGN - An Integrated Approach, 4th Ed. 15-15-1
PROBLEM 15-15
Statement: An automobile manufacturer would like a feasibility study of the concept of building-in electric,
motor-powered, screw jacks at each end of the car to automatically jack the car wheels off the
ground for service. Assuming a 2-ton vehicle with a 60/40 front/rear weight distribution,
design a self locking screw jack capable of lifting either end of the car. The jack body will be
attached to the car frame and the screw will extend downward to engage the ground. Assume a
minimum installed clearance of 8 in under the retracted screw in the up position. It must lift the
car frame at least an additional 8 in. Use rolling element thrust bearings. Determine a minimu
screw size, safe against column buckling. Determine its required lifting torque and efficiency,
and the power required to lift it to full height in 45 sec. What is your recommendation as to the
feasibility of this idea?
Given: Weight of car Wcar  2000 lbf Front weight factor ffront  0.6
Noload extension xnoload  8  in Loaded extension xload  8  in
Time to rertract tup  45 sec

Design Choices:
1. The thread coefficient of friction is μ  0.15.
2. The collar coefficient of friction is μc  0.02.
6
3. Use AISI 1050 steel, Q&T @ 400F, with S y  117  ksi, and E  30 10  psi.
4. The column is fixed-free with end condition constant C  2.
5. Use the pitch diameter of the screw to compute the column radius of gyration.
6. Use a buckling design safety factor of Nbd  4.
7. Use a mean collar diameter of d c  2.00 in.
8. The length of engagement of the nut on the screw is Lnut  0.75 in.

Solution: See Mathcad file P1515.


1. Using the buckling critera (see Problem 4-53), find the minimum pitch diameter for the screw.

Column load P  ffront  Wcar P  1200 lbf

Effective column length Leff  C  xload  xnoload  Leff  32 in

2. Start by calculating the slenderness ratio that divides the unit load vs slenderness ratio graph into Johnson
and Euler regions.

2 E
S rD  π S rD  71.143
Sy

3. To start the iterative process, assume that the final design will be an Euler column with the critical load equal
to Nbd*P. From equation 4.38b,
2 2
π  E A  k 2 I
Pcr = and k =
2 A
L
2
π  E I
Substituting for k2 Pcr = = Nbd  P
2
L
2
Leff  Nbd  P
Solving for I I 
2
π E
4
The required moment of inertia, assuming an Euler column is I  0.0166 in
© 2011 Pearson Education, Inc., Upper Saddle River, NJ. All rights reserved. This publication is protected by Copyright and written permission should be
MACHINE DESIGN - An Integrated Approach, 4th Ed. 15-15-2

4. Using the equation for the moment of inertia of a solid, round rod, solve for the pich diameter of the screw.
1
4 4
π d  64 I 
I= d p   π  d p  0.763  in
64  
5. Using this diameter, calculate the slenderness ratio and compare to S rD. If it is greater than S rD the assumption
of an Euler column is correct, if not, recalculate using the Johnson equation.
π 2 2
Area Ar   dp Ar  0.457  in
4

I
Radius of gyration kr  kr  0.191  in
Ar

Leff
Slenderness ratio S r  S r  167.9
kr

Since this is greater than S rD, the assumption of an Euler column is correct.
6. Enter Table 15-3 with this minimum pitch diameter and chose a tentative screw size of 7/8-6.
1
Screw diameter d  0.875  in Threads per inch Nt  6  in
Radial thread angle α  14.5 deg Pich daimeter d p  0.792  in

1
7. Determine the thread pitch and lead. p  p  0.167  in
Nt
L  p L  0.167  in
8. Use equations 15.5 to determine the extending and retracting torques.

P d p  μ  π d p  L cos( α)  dc
Text    μc  P Text  130.6  in lbf
2  π d p cos( α)  μ  L 2

P d p μ  π dp  L cos( α)  dc
Tret    μc  P Tret  65.4 in lbf
2 π dp cos( α)  μ  L 2

9. Use equation 15.7c to determine the lifting (up) and lowering (down) efficiencies.
P L
eext  eext  24.4 %
2  π Text

P L
eret  eret  48.7 %
2  π Tret

10. Use equation 15.6a to determine if the screw is self-locking.

L
self_locking  return "yes" if μ   cos( α)
π d p
"no" otherwise

self_locking  "yes"

© 2011 Pearson Education, Inc., Upper Saddle River, NJ. All rights reserved. This publication is protected by Copyright and written permission should be
MACHINE DESIGN - An Integrated Approach, 4th Ed. 15-15-3
11. Check the safety factor against thread shear by stripping using equations 15.8. The area factor for thread
stripping is given in Table 15-5 as wi  0.77 for the minor diameter of an Acme thread.

Minor diameter d r  0.708  in


2
Total shear area As  π d r L Lnut Nt As  1.668  in
P
Shear stress τs  τs  719.3  psi
As

0.577  S y
Factor of safety Ns  Ns  94
τs

12. Determine the power required to lift the car.

xload  xnoload rad


Rotational speed ω  ω  2.133 
t up L sec

Power required H  Text ω H  0.042  hp

© 2011 Pearson Education, Inc., Upper Saddle River, NJ. All rights reserved. This publication is protected by Copyright and written permission should be
MACHINE DESIGN - An Integrated Approach, 4th Ed. 15-16-1
PROBLEM 15-16
Statement: Design a manual screw jack similar to that shown in Figure 15-4 for a 20-ton lift capacity and a
100-mm lift stroke. Assume that the operator can apply a 400-N force at the tip of its bar handle
to turn either the screw or the nut, depending on your design. Design the cylindrical bar handle
to fail in bending at the design load before the jackscrew fails so that one cannot lift an overload
and fail the screw. Use rolling element thrust bearings. Seek a safety factor of 3 for thread or
column failure. State all assumptions.
Given: Design jack load P  40000  lbf Design handle load Fhandle  400  N
Lift stroke stroke  100  mm Design safety factor Nd  3
6
Young's modulus E  30 10  psi

Design Choices:
1. The thread coefficient of friction is μ  0.15.
2. The collar coefficient of friction is μc  0.02.
3. For the screw, use AISI 1050 steel, Q&T @ 400F, with S yscrew  117  ksi.
4. For the handle, use AISI 1020 cold rolled steel, with S yhandle  57 ksi.
5. The column is fixed-free with end condition constant C  2.
6. Use the pitch diameter of the screw to compute the column radius of gyration.
7. Use a mean collar diameter of d c  2.00 in.
8. The length of engagement of the nut on the screw is Lnut  2.00 in.
9. The length of the handle from its base to midpoint of grip is Lhandle  10 in

Solution: See Mathcad file P1516.


1. Using the buckling critera (see Problem 4-53), find the minimum pitch diameter for the screw.
Effective column length Leff  C stroke Leff  7.874  in
2. Start by calculating the slenderness ratio that divides the unit load vs slenderness ratio graph into Johnson
and Euler regions.

2 E
S rD  π S rD  71.143
S yscrew

3. To start the iterative process, assume that the final design will be a Johnson column with the critical load
equal to Nbd*P. From equation 4.38b,
2 2 4 2
 Sy  π d π d
 
A L 2 I
Pcr = S y A     k = I= A=
E  2 π  k A 64 4
2
 Sy  A 2 L2
1
Substituting for k 2 Pcr = S y A      = Nd  P
E  2 π  I 1
2
 4 Nd  P 16 S yscrew  Leff  
2
d p   
 π Syscrew 
Solving for d p
 E  2 π  

The minimum required pitch diameter, assuming a Johnson column is d p  1.185  in

5. Using this diameter, calculate the slenderness ratio and compare to S rD. If it is less than S rD the assumption of
a Johnson column is correct, if not, recalculate using the Euler equation.

π 2 2
Area Ar   dp Ar  1.103  in
4
© 2011 Pearson Education, Inc., Upper Saddle River, NJ. All rights reserved. This publication is protected by Copyright and written permission should be
MACHINE DESIGN - An Integrated Approach, 4th Ed. 15-16-2

π 4 4
Moment of inertia I   dp I  0.097  in
64

I
Radius of gyration kr  kr  0.296  in
Ar

Leff
Slenderness ratio S r  S r  26.6
kr

Since this is less than S rD, the assumption of a Johnson column is correct.
6. Enter Table 15-3 with this minimum pitch diameter and chose a tentative screw size of 1 3/8-5.
1
Screw diameter d  1.375  in Threads per inch Nt  4  in
Radial thread angle α  14.5 deg Pich daimeter d p  1.250  in

1
7. Determine the thread pitch and lead. p  p  0.250  in
Nt
L  p L  0.250  in
8. Use equations 15.5 to determine the lifting (up) and lowering (down) torques.

P d p μ  π dp  L cos( α)  dc
Tu    μc  P Tu  6319 in lbf
2 π dp cos( α)  μ  L 2

P d p  μ  π d p  L cos( α)  dc
Td    μc  P Td  3060 in lbf
2  π d p cos( α)  μ  L 2

9. Use equation 15.7c to determine the lifting (up) and lowering (down) efficiencies.
P L
eu  eu  25.2 %
2  π Tu

P L
ed  ed  52.0 %
2  π Td

10. Use equation 15.6a to determine if the screw is self-locking.

L
self_locking  return "yes" if μ   cos( α)
π d p
"no" otherwise

self_locking  "yes"
11. Check the safety factor against thread shear by stripping using equations 15.8. The area factor for thread
stripping is given in Table 15-5 as wi  0.77 for the minor diameter of an Acme thread.

Minor diameter d r  0.708  in


2
Total shear area As  π d r L Lnut Nt As  4.448  in

P
Shear stress τs  τs  8.99 ksi
As
© 2011 Pearson Education, Inc., Upper Saddle River, NJ. All rights reserved. This publication is protected by Copyright and written permission should be
MACHINE DESIGN - An Integrated Approach, 4th Ed. 15-16-3

0.577  S yscrew
Factor of safety Ns  Ns  7.5
τs

12. Find a suitable length and diameter for the handle using a safety factor against yielding of one at the handle
design load.
Bending moment M  Tu M  6319 in lbf

M
Handle length Lhandle  Lhandle  70.3 in
Fhandle

M 3
Section modulus Z  Z  0.1109 in
S yhandle
1
3
 32 Z 
Handle diameter d   π  d  1.041  in
 

Use a handle with diameter d  1.00 in


However, the length is unrealistic (nearly 6 feet).

© 2011 Pearson Education, Inc., Upper Saddle River, NJ. All rights reserved. This publication is protected by Copyright and written permission should be
MACHINE DESIGN - An Integrated Approach, 4th Ed. 15-17a-1

PROBLEM 15-17a
Statement: Determine the effective spring constant of an aluminum, copper-asbestos, steel sandwich under
compressive load. It is uniformly loaded over its 10-cm2 area. The first and third members are
each 10 mm thick and the middle member is 1 mm thick, making a total thickness of 21 mm.
Which material dominates the calculation?
Given: Thicknesses t1  10 mm Modulus E1  71.7 GPa
t2  1  mm E2  93 GPa
t3  10 mm E3  206.8  GPa
2
Area A  10 cm
Solution: See Mathcad file P1517a.
1. Member stiffnesses

A  E1 9 N
Aluminum (1) k1  k1  7.170  10 
t1 m

A  E2 10 N
Copper-asbestos (2) k2  k2  9.300  10 
t2 m

A  E3 10 N
Steel (3) k3  k3  2.068  10 
t3 m

2. Sandwich stiffness:
1 9 N
ktot  ktot  5.04  10 
1 1 1 m
k1  k2  k3

3. The aluminum dominates the calculation.

© 2011 Pearson Education, Inc., Upper Saddle River, NJ. All rights reserved. This publication is protected by Copyright and written permission should be
MACHINE DESIGN - An Integrated Approach, 4th Ed. 15-18a-1

PROBLEM 15-18a
Statement: Determine the effective spring constant of an aluminum, copper-asbestos, steel sandwich under
compressive load. It is uniformly loaded over its 1.5-in 2 area. The first and third members are
each 0.4 in thick and the middle member is 0.04 in thick, making a total thickness of 0.84 in.
Which material dominates the calculation?
6
Given: Thicknesses t1  0.40 in Modulus E1  10.4 10  psi
6
t2  0.04 in E2  13.5 10  psi
6
t3  0.40 in E3  30 10  psi
2
Area A  1.5 in

Solution: See Mathcad file P1518a.


1. Member stiffnesses

A  E1 7 lbf
Aluminum (1) k1  k1  3.900  10 
t1 in

A  E2 8 lbf
Copper-asbestos (2) k2  k2  5.063  10 
t2 in

A  E3 8 lbf
Steel (3) k3  k3  1.125  10 
t3 in

2. Sandwich stiffness:
1 7 lbf
ktot  ktot  2.74  10 
1 1 1 in
k1  k2  k3

3. The aluminum dominates the calculation.

© 2011 Pearson Education, Inc., Upper Saddle River, NJ. All rights reserved. This publication is protected by Copyright and written permission should be
MACHINE DESIGN - An Integrated Approach, 4th Ed. 15-19a-1

PROBLEM 15-19a _____

Statement: A preloaded steel bolt similar to that shown in Figure 15-31(a) clamps two flanges of total
thickness l. Using the data given in the row(s) assigned in Table P15-1, find the joint stiffness
constant.
Given: Bolt specification: M8 x 1
Bolt diameter d  8  mm Clamped length l  30 mm
Member material matl  "steel" Bolt modulus Ebolt  206.8  GPa

Member modulus of elasticity function: Ememb  return 206.8  GPa if matl = "steel"
return 71.0 GPa if matl = "alum"
Solution: See Figure 15-31(a), Table P15-1, Table P15-8, and Mathcad file P1519a.
1. Determine the relevant ratios for this joint from equations 15-18a and b.
d
Joint aspect ratio: j  j  0.267
l
Ememb
Plate to bolt modulus: r  r  1.000
Ebolt

2. Calculate Cr = C using equation 15.19 and the coefficients p i from Table 15-8 for j  0.267 . Use linear
interpolation between ja  0.2 and jb  0.3
pb  pa
Interpolation equation: p  p a p b     j  ja   p a
jb  ja

Coefficients from Table 15-8: p 0  p ( 0.6118 0.6932) p 0  0.6661

p 1  p ( 1.1715 1.2426) p 1  1.2189

p 2  p ( 1.0875 1.1177) p 2  1.1076

p 3  p ( 0.3806 0.3845) p 3  0.3832

3 2
Joint stiffness constant: C  p 3  r  p 2  r  p 1  r  p 0 C  0.172

© 2011 Pearson Education, Inc., Upper Saddle River, NJ. All rights reserved. This publication is protected by Copyright and written permission should be
MACHINE DESIGN - An Integrated Approach, 4th Ed. 15-20-1

PROBLEM 15-20
Statement: A single-cylinder air compressor head sees forces that range from 0 to 18.5 kN each cycle. The
head is 80-mm-thick aluminum, the unconfined gasket is 1-mm-thick Teflon, and the block is
aluminum. The effective clamp length of the cap screw is 120 mm. The piston is 75 mm dia and
the cylinder is 140 mm outside dia. Specify a suitable number, class, preload, and bolt circle for
the cylinder head cap screws to give a minimum safety factor of 1.2 for any possible failure
mode.
Given: Total load Ptot  18.5 kN Gasket thickness tg  1  mm
Clamp length l  120  mm Member mod. Ememb  71.8 GPa
Piston diameter id  75 mm Bolt modulus Ebolt  206.8  GPa
Cylinder diameter od  140  mm Teflon mod. Eg  240  MPa
Design safety factor Nd  1.2

Design Choices: Use 10 M12 x 1.75 , class 8.8 cap screws.


Material properties for class 8.8: Bolt diameter d  12 mm
Proof strength S p  600  MPa Number of bolts Nbolts  8
Yield strength S y  660  MPa Preload fraction fp  0.90
Ultimate strength S ut  830  MPa Bolt circle diameter d bc  107.5  mm

Solution: See Mathcad file P1520.


Ptot
1. Determine the load per bolt. P  P  2313 N
Nbolts
Pmin  0  N Pmax  P
2
2. Get the tensile stress area from Table 15-2. At  84.27  mm

3. Calculate the preload. Fi  fp  S p At Fi  45.51  kN


4. Determine the relevant ratios for the joint without the gasket from equations 15-18a and b.
d
Joint aspect ratio: j  j  0.1
l
Ememb
Plate to bolt modulus: r  r  0.347
Ebolt

5. Calculate Cr = C using equation 15.19 and the coefficients p i from Table 15-8 for j  0.1 .

Coefficients from Table 15-8: p 0  0.4389


p 1  0.9197
p 2  0.8901
p 3  0.3187

3 2
Joint stiffness constant: C  p 3  r  p 2  r  p 1  r  p 0 C  0.214
6. We can estimate the stiffness of the bolt kb' from equation 15.17 using its tensile stress area from Table 15-1 and
then estimate the material stiffness km for the no gasket case by using the expression for C in equation 15.13c,
given kb' and C.

At  Ebolt 1
  1 
d kN
Estimated bolt stiffness kb'   kb'  132.0 
l  l mm
© 2011 Pearson Education, Inc., Upper Saddle River, NJ. All rights reserved. This publication is protected by Copyright and written permission should be
MACHINE DESIGN - An Integrated Approach, 4th Ed. 15-20-2

( 1  C) kN
Estimated member stiffness km  kb'  km  486 
C mm

7. Now we will address the unconfined gasket. The bolt stiffness is not affected by the gasket but the material
stiffness is. We now have two springs in series, the metal, whose stiffness is calculated above, and the
gasket. These combine according to equation 14.2b. The portion of the unconfined gasket subjected to the
clamp force can be assumed to be from the outside diameter of the flange shown in Figure 15-33 to the inside
diameter of the vessel. The bolt hole should be subtracted from the gasket area. The area of the clamped
gasket around one bolt is:

  od  id  Nbolts  d
π 2 2 2 2
Gasket area per bolt Ag   Ag  1259 mm
4  Nbolts

8. The stiffness of this piece of gasket material is found from equation 15.11c,
Ag  Eg kN
Gasket stiffness per bolt kg  kg  302.1 
tg mm

9. The combined stiffness of the gasketed joint (from equation 14.2b) is

Let the aluminum and gasket stiffnesses be km1  km and km2  kg

Combined gasket and cover 1 kN


stiffness km  km  186.3 
1 1 mm

km1 km2
10. The joint constant with the unconfined gasket is now

Joint constant with kb'


unconfined gasket C  C  0.415
km  kb'

and ( 1  C)  0.585

11. The portions of the applied load P felt by the bolt and the material can now be found from equations 15.13.
Pb  C P Pb  958.97 N

Pm  ( 1  C)  P Pm  1353.5 N

12. Find the resulting loads in bolt and material after the load P is applied.
Fb  Fi  Pb Fb  46.46  kN

Fm  Fi  Pm Fm  44.15  kN

13. Calculate the alternating and mean components of the fluctuating bolt load.
Fb  Fi
Falt  Falt  0.479  kN
2
Fb  Fi
Fmean  Fmean  45.99  kN
2
14. Because these loads are fluctuating, we need to calculate the mean and alternating components of the force felt
in the bolt. The mean and alternating forces are

Fb  Fi
Mean force Fmean  Fmean  46.0 kN
2
© 2011 Pearson Education, Inc., Upper Saddle River, NJ. All rights reserved. This publication is protected by Copyright and written permission should be
MACHINE DESIGN - An Integrated Approach, 4th Ed. 15-20-3

Fb  Fi
Alternating force Falt  Falt  479  N
2
15. The nominal mean and alternating stresses in the bolt are:

Fmean
Nominal mean stress σmnom  σmnom  545.7  MPa
At
Falt
Nominal alternating stress σanom  σanom  5.69 MPa
At

16. The fatigue stress-concentration factor for this thread is found from equation 15.15c and the mean
stress-concentration factor factor Kfm is found from equation 6.17.
0.02682  d
Fatigue factor Kf  5.7  Kf  6
mm

Nominal maximum stress σmaxnom  σmnom  σanom σmaxnom  551.4  MPa

Nominal minimum stress σminnom  σmnom  σanom σminnom  540.0  MPa

Kfm  return Kf if Kf  σmaxnom  S y


S y  Kf  σanom
return if Kf  σmaxnom  S y Kfm  1.147
σmnom
return 0 if Kf  σmaxnom  σminnom  2  S y

17. The local mean and alternating stresses in the bolt are then:
Local mean stress σm  Kfm σmnom σm  625.7  MPa

Local alternating stress σa  Kf  σanom σa  34.3 MPa

18. The stress at the initial preload is

Fi
σinit  Kfm σinit  619.21 MPa
At

19. An endurance strength must be found for this material. Using the methods of Section 6.6 we find for
S ut  830  MPa
S'e  0.5 S ut S'e  415.0  MPa

20. From the tables and formulas in Section 6.6 we have:


Load Cload  0.70

Size Csize  1

Surface A  4.51 b  0.265


b
 Sut 
Csurf  A    Csurf  0.76
 MPa 
Temperature Ctemp  1

© 2011 Pearson Education, Inc., Upper Saddle River, NJ. All rights reserved. This publication is protected by Copyright and written permission should be
MACHINE DESIGN - An Integrated Approach, 4th Ed. 15-20-4

Reliability Creliab  0.814

and the endurance limit is


S e  Cload  Csize Csurf  Ctemp Creliab S'e S e  179.64 MPa

21. The corrected endurance strength and the ultimate tensile strength are used in equation 15.16 to find the safety
factor from the Goodman line.

S e  S ut  σinit
Nf  Nf  1.3
S e  σm  σinit  S ut σa

22. Calculate the maximum bolt stress and the safety factor against yielding for S y  660  MPa.
Fb
σb  σb  551.38 MPa
At
Sy
Ny  Ny  1.2
σb

23. The load required to separate the joint and the safety factor against joint separation are found from equations
15.14c and 15.14d.

Fi
Nsep  Nsep  34
Pmax ( 1  C)

24. Check the screw spacing, comparing it to the rules given in the text.

π d bc
Bolt spacing space bolt  space bolt  42.2 mm
Nbolts
space bolt
Space/dia ratio ratio  ratio  3.5 which is OK
d

© 2011 Pearson Education, Inc., Upper Saddle River, NJ. All rights reserved. This publication is protected by Copyright and written permission should be
MACHINE DESIGN - An Integrated Approach, 4th Ed. 15-21-1

PROBLEM 15-21
Statement: A single-cylinder engine head sees forces that range from 0 to 4000 lb each cycle. The head is
2.5-in-thick cast iron, the unconfined gasket is 0.125-in-thick copper-asbestos, and the block is
cast iron. The effective clamp length of the cap screw is 3.125 in. The piston is 3 in dia and the
cylinder is 5.5 in outside dia. Specify a suitable number, class, preload, and bolt circle for the
cylinder head cap screws to give a minimum safety factor of 1.2 for any possible failure mode.

Given: Total load Ptot  4000 lbf Gasket thickness tg  0.125  in


6
Clamp length l  3.125  in Member mod. Ememb  15 10  psi
6
Piston diameter id  3.00 in Bolt modulus Ebolt  30 10  psi
6
Cylinder dia. od  5.50 in Gasket modulus Eg  13.5 10  psi
Design safety factor Nd  1.2

Design Choices:
Use 8 5/16-18 UNC , grade 5 cap screws.
Material properties for grade 5: Bolt diameter d  0.3125 in
Proof strength S p  85 ksi Number of bolts Nbolts  8
Yield strength S y  92 ksi Preload fraction fp  0.90
Ultimate strength S ut  120  ksi Bolt circle diameter d bc  4.250  in

Solution: See Mathcad file P1521.

Ptot
1. Determine the load per bolt. P  P  500  lbf
Nbolts
Pmin  0  lbf Pmax  P
2
2. Get the tensile stress area from Table 15-2. At  0.0524 in

4. Calculate the preload. Fi  fp  S p At Fi  4009 lbf

4. Determine the relevant ratios for the joint without the gasket from equations 15-18a and b.
d
Joint aspect ratio: j  j  0.1
l
Ememb
Plate to bolt modulus: r  r  0.5
Ebolt

5. Calculate Cr = C using equation 15.19 and the coefficients p i from Table 15-8 for j  0.1 .

Coefficients from Table 15-8: p 0  0.4389


p 1  0.9197
p 2  0.8901
p 3  0.3187

3 2
Joint stiffness constant: C  p 3  r  p 2  r  p 1  r  p 0 C  0.162
6. We can estimate the stiffness of the bolt kb' from equation 15.17 using its tensile stress area from Table 15-1 and
then estimate the material stiffness km for the no gasket case by using the expression for C in equation 15.13c,
given kb' and C.

© 2011 Pearson Education, Inc., Upper Saddle River, NJ. All rights reserved. This publication is protected by Copyright and written permission should be
MACHINE DESIGN - An Integrated Approach, 4th Ed. 15-21-2

At  Ebolt 1
  1 
d kip
Estimated bolt stiffness kb'   kb'  457.3 
l  l in

( 1  C) kip
Estimated member stiffness km  kb'  km  2370
C in

7. Now we will address the unconfined gasket. The bolt stiffness is not affected by the gasket but the material
stiffness is. We now have two springs in series, the metal, whose stiffness is calculated above, and the
gasket. These combine according to equation 14.2b. The portion of the unconfined gasket subjected to the
clamp force can be assumed to be from the outside diameter of the flange shown in Figure 15-33 to the inside
diameter of the vessel. The bolt hole should be subtracted from the gasket area. The area of the clamped
gasket around one bolt is:

  od  id  Nbolts  d
π 2 2 2 2
Gasket area per bolt Ag   Ag  2.01 in
4  Nbolts

8. The stiffness of this piece of gasket material is found from equation 15.11c,
Ag  Eg kip
Gasket stiffness per bolt kg  kg  217028
tg in

9. The combined stiffness of the gasketed joint (from equation 14.2b) is

Let the cast iron and gasket stiffnesses be km1  km and km2  kg

Combined gasket and head 1 kip


stiffness km  km  2345
1 1 in

km1 km2
10. The joint constant with the unconfined gasket is now

Joint constant with kb'


unconfined gasket C  C  0.163
km  kb'

11. The portions of the applied load P felt by the bolt and the material can now be found from equations 15.13.
Pb  C P Pb  81.61  lbf

Pm  ( 1  C)  P Pm  418.4  lbf
12. Find the resulting loads in bolt and material after the load P is applied.
Fb  Fi  Pb Fb  4090 lbf

Fm  Fi  Pm Fm  3590 lbf

13. Calculate the alternating and mean components of the fluctuating bolt load.

Fb  Fi
Falt  Falt  40.80  lbf
2
Fb  Fi
Fmean  Fmean  4.05 kip
2

14. The nominal mean and alternating stresses in the bolt are:

© 2011 Pearson Education, Inc., Upper Saddle River, NJ. All rights reserved. This publication is protected by Copyright and written permission should be
MACHINE DESIGN - An Integrated Approach, 4th Ed. 15-21-3

Fmean
Nominal mean stress σmnom  σmnom  77.3 ksi
At
Falt
Nominal alternating stress σanom  σanom  779  psi
At

15. The fatigue stress-concentration factor for this thread is found from equation 15.15c and the mean
stress-concentration factor factor Kfm is found from equation 6.17.
0.02682  d
Fatigue factor Kf  5.7  Kf  5.9
mm

Nominal maximum stress σmaxnom  σmnom  σanom σmaxnom  538.2  MPa

Nominal minimum stress σminnom  σmnom  σanom σminnom  527.4  MPa

Kfm  return Kf if Kf  σmaxnom  S y


S y  Kf  σanom
return if Kf  σmaxnom  S y Kfm  1.131
σmnom
return 0 if Kf  σmaxnom  σminnom  2  S y

16. The local mean and alternating stresses in the bolt are then:
Local mean stress σm  Kfm σmnom σm  602.6  MPa

Local alternating stress σa  Kf  σanom σa  31.7 MPa

17. The stress at the initial preload is


Fi
σinit  Kfm σinit  86.51  ksi
At
18. An endurance strength must be found for this material. Using the methods of Section 6.6 we find for
S ut  120  ksi
S'e  0.5 S ut S'e  60.0 ksi

19. From the tables and formulas in Section 6.6 we have:


Load Cload  0.70

Size Csize  1

Surface A  2.7 b  0.265


b
 S ut 
Csurf  A    Csurf  0.759
 ksi 
Temperature Ctemp  1

Reliability Creliab  0.814

and the endurance limit is

S e  Cload  Csize Csurf  Ctemp Creliab S'e S e  25.957 ksi


© 2011 Pearson Education, Inc., Upper Saddle River, NJ. All rights reserved. This publication is protected by Copyright and written permission should be
MACHINE DESIGN - An Integrated Approach, 4th Ed. 15-21-4

20. The corrected endurance strength and the ultimate tensile strength are used in equation 15.16 to find the
safety factor from the Goodman line.

S e  S ut  σinit
Nf  Nf  1.5
S e  σm  σinit  S ut σa

21. Calculate the maximum bolt stress and the safety factor against yielding for S y  92 ksi.

Fb
σb  σb  78.06  ksi
At
Sy
Ny  Ny  1.2
σb

22. The load required to separate the joint and the safety factor against joint separation are found from
equations 15.14c and 15.14d.

Fi
Nsep  Nsep  10
Pmax ( 1  C)

23. Check the screw spacing, comparing it to the rules given in the text.

π d bc
Bolt spacing space bolt  space bolt  1.67 in
Nbolts
space bolt
Space/dia ratio ratio  ratio  5.3 which is OK
d

© 2011 Pearson Education, Inc., Upper Saddle River, NJ. All rights reserved. This publication is protected by Copyright and written permission should be
MACHINE DESIGN - An Integrated Approach, 4th Ed. 15-22-1

PROBLEM 15-22
Statement: The forged steel connecting rod for the engine of problem 15-21 is split around the 38-mm-dia
crankpin and fastened with two bolts and nuts that hold its two halves together. The total load
on the two bolts varies from 0 to 8.5 kN each cycle. Design these bolts for infinite life. Specify
their size, class, and preload.

Given: Total force Ptot  8.5 kN Member mod. Ememb  206.8  GPa
Number of bolts Nbolts  2 Bolt modulus Ebolt  206.8  GPa

Design Choices:
Use M12 x 1.25 , class 9.8 bolts.
Material properties for class 9.8: Bolt diameter d  12 mm
Proof strength S p  650  MPa Preload fraction fp  0.70
Yield strength S y  720  MPa Design safety factor Nd  1.5
Ultimate strength S ut  900  MPa Clamp length l  2.5 d

Solution: See Mathcad file P1522.


1. Determine the load per bolt.
Ptot
P  P  4.25 kN
Nbolts

Pmin  0  N Pmax  P
2
2. Get the tensile stress area from Table 15-2. At  92.07  mm

3. Calculate the preload. Fi  fp  S p At Fi  41.9 kN

4. Determine the relevant ratios for this joint from equations 15-18a and b.
d
Joint aspect ratio: j  j  0.400
l
Ememb
Plate to bolt modulus: r  r1
Ebolt

5. Calculate Cr = C using equation 15.19 and the coefficients p i from Table 15-8 for j  0.4 .

Coefficients from Table 15-8: p 0  0.7351

p 1  1.2612

p 2  1.1111

p 3  0.3779

3 2
Joint stiffness constant: C  p 3  r  p 2  r  p 1  r  p 0 C  0.207

6. The portions of the applied load P felt by the bolt and the material can now be found from equations 15.13.
Pb  C P Pb  0.88 kN
Pm  ( 1  C)  P Pm  3.4 kN
7. Find the resulting loads in bolt and material after the load P is applied.
Fb  Fi  Pb Fb  42.77  kN
© 2011 Pearson Education, Inc., Upper Saddle River, NJ. All rights reserved. This publication is protected by Copyright and written permission should be
MACHINE DESIGN - An Integrated Approach, 4th Ed. 15-22-2

Fm  Fi  Pm Fm  38.52  kN
8. Calculate the alternating and mean components of the fluctuating bolt load.

Fb  Fi
Falt  Falt  0.440  kN
2
Fb  Fi
Fmean  Fmean  42.33  kN
2
9. The nominal mean and alternating stresses in the bolt are:

Fmean
Nominal mean stress σmnom  σmnom  459.8  MPa
At

Falt
Nominal alternating stress σanom  σanom  4.78 MPa
At

10. The fatigue stress-concentration factor for this thread is found from equation 15.15c and the mean
stress-concentration factor factor Kfm is found from equation 6.17.
0.02682  d
Fatigue factor Kf  5.7  Kf  6.0
mm

Nominal maximum stress σmaxnom  σmnom  σanom σmaxnom  67.379 ksi

Nominal minimum stress σminnom  σmnom  σanom σminnom  65.992 ksi

Kfm  return Kf if Kf  σmaxnom  S y


S y  Kf  σanom
return if Kf  σmaxnom  S y Kfm  1.503
σmnom
return 0 if Kf  σmaxnom  σminnom  2  S y

11. The local mean and alternating stresses in the bolt are then:
Local mean stress σm  Kfm σmnom σm  691.2  MPa

Local alternating stress σa  Kf  σanom σa  28.78  MPa


12. The stress at the initial preload is

Fi
σinit  Kfm σinit  684.0  MPa
At

13. An endurance strength must be found for this material. Using the methods of Section 6.6 we find for
S ut  900  MPa
S'e  0.5 S ut S'e  450.0  MPa

14. From the tables and formulas in Section 6.6 we have:


Load Cload  0.70
Size Csize  1

Surface A  4.51 b  0.265


© 2011 Pearson Education, Inc., Upper Saddle River, NJ. All rights reserved. This publication is protected by Copyright and written permission should be
MACHINE DESIGN - An Integrated Approach, 4th Ed. 15-22-3

b
 Sut 
Csurf  A    Csurf  0.744
 MPa 
Temperature Ctemp  1

Reliability Creliab  0.814

and the endurance limit is

S e  Cload  Csize Csurf  Ctemp Creliab S'e S e  190.65 MPa

15. The corrected endurance strength and the ultimate tensile strength are used in equation 15.16 to find the safety
factor from the Goodman line.

S e  S ut  σinit
Nf  Nf  1.5
S e  σm  σinit  S ut σa

16. Calculate the maximum bolt stress and the safety factor against yielding for S y  720  MPa.

Fb
σb  σb  464.6  MPa
At
Sy
Ny  Ny  1.5
σb

17. The load required to separate the joint and the safety factor against joint separation are found from equations
15.14c and 15.14d.

Fi
Nsep  Nsep  12.4
Pmax ( 1  C)

© 2011 Pearson Education, Inc., Upper Saddle River, NJ. All rights reserved. This publication is protected by Copyright and written permission should be
MACHINE DESIGN - An Integrated Approach, 4th Ed. 15-23a-1

PROBLEM 15-23a
Statement: (See also Problem 4-33.) The bracket in Figure P15-2 is fastened to the wall by 4 cap screws
equispaced on a 10-cm-dia bolt circle and arranged as shown. The wall is the same material as
the bracket. The bracket is subjected to a static force F, where F and the beam's other data are
given in row a in Table P15-1. Find the forces acting on each of the 4 cap screws due to this
loading and choose a suitable cap screw diameter, length, and preload that will give a minimum
safety factor of 2 for any possible mode of failure.

Given: Tube length L  100  mm Applied force F  50 N


Arm length a  400  mm Modulus of elasticity E  207  GPa
Wall plate thickness t  10 mm Number of bolts Nbolts  2
Bolt circle diameter d bc  100  mm (top or bottom)
Design safety factor Nd  2
Bolt modulus Ebolt  E Member modulus Ememb  E
Clamp length l  2  t
Assumptions: 1. The cap screws and wall provide the couple shown in the FBD of Figure 15-23. This is an
idealization, but is conservative.
2. The torsion reaction and the vertical reaction at the wall are provided by frictional forces
between the wall and the wall plate on the bracket and the screws are subjected to the horizontal
forces only.

Design Choices:
Use M5 X 0.8 , class 4.6 cap screws.
Material properties for class 4.6: Cap screw diameter d  5  mm
Proof strength S p  225  MPa
Yield strength S y  240  MPa Preload fraction fp  0.54

y
dbc F

Ptot

d T T x

Ptot
L

R
FIGURE 15-23
Free Body Diagram of Tube and Wall Plate for Problem 15-23

Solution: See Figure 15-23 and Mathcad file P1523a.


1. Calculate the magnitude of the couple in Figure 15-23 by summing moments. Ptot  d'  F  L = 0

Distance d' d'  d bc sin( 45 deg) d'  70.711 mm


L
Total screw force Ptot  F  Ptot  70.71  N
d'
Ptot
2. Determine the load per screw. P  P  35.36  N
Nbolts
© 2011 Pearson Education, Inc., Upper Saddle River, NJ. All rights reserved. This publication is protected by Copyright and written permission should be
MACHINE DESIGN - An Integrated Approach, 4th Ed. 15-23a-2

2
3. Get the tensile stress area from Table 15-2. At  14.18  mm

4. Calculate the preload. Fi  fp  S p At Fi  1.72 kN


5. Determine the relevant ratios for this joint from equations 15-18a and b.
d
Joint aspect ratio: j  j  0.25
l
Ememb
Plate to bolt modulus: r  r1
Ebolt

6. Calculate Cr = C using equation 15.19 and the coefficients p i from Table 15-8 for j  0.25 . Use linear
interpolation between ja  0.2 and jb  0.3

pb  pa
Interpolation equation: p  p a p b     j  ja   p a
jb  ja

Coefficients from Table 15-8: p 0  p ( 0.6118 0.6932) p 0  0.6525

p 1  p ( 1.1715 1.2426) p 1  1.2070

p 2  p ( 1.0875 1.1177) p 2  1.1026

p 3  p ( 0.3806 0.3845) p 3  0.3825


3 2
Joint stiffness constant: C  p 3  r  p 2  r  p 1  r  p 0 C  0.165

7. The portions of the applied load P felt by the screw and the material can now be found from equations 15.13.
Pb  C P Pb  5.85 N
Pm  ( 1  C)  P Pm  29.5 N
8. Find the resulting loads in screw and material after the load P is applied.
Fb  Fi  Pb Fb  1.73 kN
Fm  Fi  Pm Fm  1.69 kN
9. The maximum tensile stress in the screw is
Fb
σb  σb  121.9  MPa
At
10. This is a uniaxial stress situation, so the principal stress and von Mises stress are identical to the applied
tensile stress. The safety factor against yielding for class 4.6 with S y  240  MPa is then

Sy
Ny  Ny  2.0
σb
11. The load required to separate the joint and the safety factor against joint separation are found from
equations 15.14c and 15.14d.

Fi
P0  P0  2.1 kN
1C
P0
Nsep  Nsep  58
P
© 2011 Pearson Education, Inc., Upper Saddle River, NJ. All rights reserved. This publication is protected by Copyright and written permission should be
MACHINE DESIGN - An Integrated Approach, 4th Ed. 15-24a-1

PROBLEM 15-24a
Statement: (See also Problem 6-33.) The bracket in Figure P15-2 is fastened to the wall by 4 cap screws
equispaced on a 10-cm-dia bolt circle and arranged as shown. The wall is the same material as
the bracket. The bracket is subjected to a sinusoidal force time function with Fmax = F, and Fmin
= -F, where F and the beam's other data are given in row a in Table P15-1. Find the forces
acting on each of the 4 cap screws due to this loading and choose a suitable cap screw
diameter, length, and preload that will give a minimum safety factor of 1.5 for any possible mode
of failure for N = 5E8 cycles.

Given: Tube length L  100  mm Applied force F  50 N


Arm length a  400  mm Modulus Ememb  207  GPa
Wall plate thickness t  10 mm Number of bolts Nbolts  2
Bolt circle diameter d bc  100  mm (top or bottom)
Design safety factor Nd  1.5
Assumptions: 1. The cap screws and wall provide the couple shown in the FBD of Figure 15-23. This is an
idealization, but is conservative.
2. The torsion reaction and the vertical reaction at the wall are provided by frictional forces
between the wall and the wall plate on the bracket and the screws are subjected to the horizontal
forces only.
Design Choices:
Use M4 x 0.7 , class 4.8 cap screws.
Material properties for class 4.8: Cap screw diameter d  4  mm
Proof strength S p  310  MPa Clamp length l  16 mm
Yield strength S y  340  MPa Preload fraction fp  0.75
Ultimate strength S ut  420  MPa Modulus Ebolt  207  GPa

y
dbc F

Ptot

d T T x

Ptot
L

R
FIGURE 15-24
Free Body Diagram of Tube and Wall Plate for Problem 15-24

Solution: See Figure 15-24 and Mathcad file P1524a.


1. Calculate the magnitude of the couple in Figure 15-24 by summing moments. Ptot  d'  F  L = 0

Distance d' d'  d bc sin( 45 deg) d'  70.711 mm


L
Total screw force Ptot  F  Ptot  70.71  N
d'
Ptot
2. Determine the load per screw. P  P  35.36  N
Nbolts
© 2011 Pearson Education, Inc., Upper Saddle River, NJ. All rights reserved. This publication is protected by Copyright and written permission should be
MACHINE DESIGN - An Integrated Approach, 4th Ed. 15-24a-2

Pmin  P Pmax  P
2
3. Get the tensile stress area from Table 15-2. At  8.78 mm

4. Calculate the preload. Fi  fp  S p At Fi  2.04 kN


5. Determine the relevant ratios for this joint from equations 15-18a and b.
d
Joint aspect ratio: j  j  0.25
l
Ememb
Plate to bolt modulus: r  r1
Ebolt

6. Calculate Cr = C using equation 15.19 and the coefficients p i from Table 15-8 for j  0.25 . Use linear
interpolation between ja  0.2 and jb  0.3

pb  pa
Interpolation equation: p  p a p b     j  ja   p a
jb  ja

Coefficients from Table 15-8: p 0  p ( 0.6118 0.6932) p 0  0.6525

p 1  p ( 1.1715 1.2426) p 1  1.2070

p 2  p ( 1.0875 1.1177) p 2  1.1026

p 3  p ( 0.3806 0.3845) p 3  0.3825

3 2
Joint stiffness constant: C  p 3  r  p 2  r  p 1  r  p 0 C  0.165

7. The portions of the applied load P felt by the screw and the material can now be found from equations 15.13.
Pb  C P Pb  5.85 N

Pm  ( 1  C)  P Pm  29.5 N
8. Find the resulting loads in screw and material after the load P is applied.
Fb  Fi  Pb Fb  2.05 kN

Fm  Fi  Pm Fm  2.01 kN
9. Calculate the alternating and mean components of the fluctuating screw load.

Fb  Fi
Falt  Falt  2.926  N
2
Fb  Fi
Fmean  Fmean  2.04 kN
2
10. The nominal mean and alternating stresses in the bolt are:

Fmean
Nominal mean stress σmnom  σmnom  232.8  MPa
At

© 2011 Pearson Education, Inc., Upper Saddle River, NJ. All rights reserved. This publication is protected by Copyright and written permission should be
MACHINE DESIGN - An Integrated Approach, 4th Ed. 15-24a-3

Falt
Nominal alternating stress σanom  σanom  0.333  MPa
At

11. The fatigue stress-concentration factor for this thread is found from equation 15.15c and the mean
stress-concentration factor factor Kfm is found from equation 6.17.
0.6812 d
Fatigue factor Kf  5.7  Kf  5.8
in

Nominal maximum stress σmaxnom  σmnom  σanom σmaxnom  233.2  MPa

Nominal minimum stress σminnom  σmnom  σanom σminnom  232.5  MPa

Kfm  return Kf if Kf  σmaxnom  S y


S y  Kf  σanom
return if Kf  σmaxnom  S y Kfm  1.452
σmnom
return 0 if Kf  σmaxnom  σminnom  2  S y

12. The local mean and alternating stresses in the bolt are then:
Local mean stress σm  Kfm σmnom σm  338.1  MPa

Local alternating stress σa  Kf  σanom σa  1.935  MPa

13. The stress at the initial preload is

Fi
σinit  Kfm σinit  337.58 MPa
At

14. An endurance strength must be found for this material. Using the methods of Section 6.6 we find for
S ut  420  MPa
S'e  0.5 S ut S'e  210.0  MPa
15. From the tables and formulas in Section 6.6 we have:
Load Cload  0.70

Size Csize  1
Surface A  4.51 b  0.265
b
 Sut 
Csurf  A    Csurf  0.91
 MPa 
Temperature Ctemp  1

Reliability Creliab  0.814

and the endurance limit is


S e  Cload  Csize Csurf  Ctemp Creliab S'e S e  108.88 MPa

16. The corrected endurance strength and the ultimate tensile strength are used in equation 15.16 to find the
safety factor from the Goodman line.

© 2011 Pearson Education, Inc., Upper Saddle River, NJ. All rights reserved. This publication is protected by Copyright and written permission should be
MACHINE DESIGN - An Integrated Approach, 4th Ed. 15-24a-4

S e  S ut  σinit
Nf  Nf  10
S e  σm  σinit  S ut σa

17. Calculate the maximum bolt stress and the safety factor against yielding for S y  340  MPa.

Fb
σb  σb  233.166  MPa
At
Sy
Ny  Ny  1.5
σb

18. The load required to separate the joint and the safety factor against joint separation are found from
equations 15.14c and 15.14d.

Fi
Nsep  Nsep  69
Pmax ( 1  C)

© 2011 Pearson Education, Inc., Upper Saddle River, NJ. All rights reserved. This publication is protected by Copyright and written permission should be
MACHINE DESIGN - An Integrated Approach, 4th Ed. 15-25a-1

PROBLEM 15-25a
Statement: (See also Problem 6-34.) The bracket in Figure P15-2 is fastened to the wall by 4 cap screws
equispaced on a 10-cm-dia bolt circle and arranged as shown. The wall is the same material as
the bracket. The bracket is subjected to a sinusoidal force time function with Fmax = F, and
Fmin = 0, where F and the beam's other data are given in row a in Table P15-1. Find the forces
acting on each of the 4 cap screws due to this loading and choose a suitable cap screw
diameter, length, and preload that will give a minimum safety factor of 1.5 for any possible
mode of failure for N = 5E8 cycles.
Given: Tube length L  100  mm Applied force F  50 N
Arm length a  400  mm Modulus Ememb  207  GPa
Wall plate thickness t  10 mm Number of bolts Nbolts  2
Bolt circle diameter d bc  100  mm (top or bottom)
Design safety factor Nd  1.5

Assumptions: 1. The cap screws and wall provide the couple shown in the FBD of Figure 15-23. This is an
idealization, but is conservative.
2. The torsion reaction and the vertical reaction at the wall are provided by frictional forces
between the wall and the wall plate on the bracket and the screws are subjected to the horizontal
forces only.
Design Choices:
Use M4 x 0.7 , class 4.8 cap screws.
Material properties for class 4.8: Cap screw diameter d  4  mm
Proof strength S p  310  MPa Clamp length l  16 mm
Yield strength S y  340  MPa Preload fraction fp  0.75
Ultimate strength S ut  420  MPa Modulus Ebolt  207  GPa

y
dbc F

Ptot

d T T x

Ptot
L

R
FIGURE 15-25
Free Body Diagram of Tube and Wall Plate for Problem 15-25

Solution: See Figure 15-25 and Mathcad file P1525a.


1. Calculate the magnitude of the couple in Figure 15-25 by summing moments. Ptot  d  F  L = 0

Distance d' d'  d bc sin( 45 deg) d'  70.711 mm


L
Total screw force Ptot  F  Ptot  70.71  N
d'
Ptot
2. Determine the load per screw. P  P  35.36  N
Nbolts
© 2011 Pearson Education, Inc., Upper Saddle River, NJ. All rights reserved. This publication is protected by Copyright and written permission should be
MACHINE DESIGN - An Integrated Approach, 4th Ed. 15-25a-2

Pmin  0  N Pmax  P

2
3. Get the tensile stress area from Table 15-2. At  8.78 mm

4. Calculate the preload. Fi  fp  S p At Fi  2.04 kN

5. Determine the relevant ratios for this joint from equations 15-18a and b.
d
Joint aspect ratio: j  j  0.25
l
Ememb
Plate to bolt modulus: r  r1
Ebolt

6. Calculate Cr = C using equation 15.19 and the coefficients p i from Table 15-8 for j  0.25 . Use linear
interpolation between ja  0.2 and jb  0.3

pb  pa
Interpolation equation: p  p a p b     j  ja   p a
jb  ja

Coefficients from Table 15-8: p 0  p ( 0.6118 0.6932) p 0  0.6525

p 1  p ( 1.1715 1.2426) p 1  1.2070

p 2  p ( 1.0875 1.1177) p 2  1.1026

p 3  p ( 0.3806 0.3845) p 3  0.3825

3 2
Joint stiffness constant: C  p 3  r  p 2  r  p 1  r  p 0 C  0.165

7. The portions of the applied load P felt by the screw and the material can now be found from equations 15.13.
Pb  C P Pb  5.85 N

Pm  ( 1  C)  P Pm  29.5 N
8. Find the resulting loads in screw and material after the load P is applied.
Fb  Fi  Pb Fb  2.05 kN

Fm  Fi  Pm Fm  2.01 kN
9. Calculate the alternating and mean components of the fluctuating screw load.

Fb  Fi
Falt  Falt  2.926  N
2
Fb  Fi
Fmean  Fmean  2.04 kN
2
10. The nominal mean and alternating stresses in the bolt are:

Fmean
Nominal mean stress σmnom  σmnom  232.8  MPa
At
© 2011 Pearson Education, Inc., Upper Saddle River, NJ. All rights reserved. This publication is protected by Copyright and written permission should be
MACHINE DESIGN - An Integrated Approach, 4th Ed. 15-25a-3

Falt
Nominal alternating stress σanom  σanom  0.333  MPa
At

11. The fatigue stress-concentration factor for this thread is found from equation 15.15c and the mean
stress-concentration factor factor Kfm is found from equation 6.17.
0.6812 d
Fatigue factor Kf  5.7  Kf  5.8
in

Nominal maximum stress σmaxnom  σmnom  σanom σmaxnom  233.2  MPa

Nominal minimum stress σminnom  σmnom  σanom σminnom  232.5  MPa

Kfm  return Kf if Kf  σmaxnom  S y


S y  Kf  σanom
return if Kf  σmaxnom  S y Kfm  1.452
σmnom
return 0 if Kf  σmaxnom  σminnom  2  S y

12. The local mean and alternating stresses in the bolt are then:
Local mean stress σm  Kfm σmnom σm  338.1  MPa

Local alternating stress σa  Kf  σanom σa  1.935  MPa

13. The stress at the initial preload is

Fi
σinit  Kfm σinit  337.58 MPa
At

14. An endurance strength must be found for this material. Using the methods of Section 6.6 we find for
S ut  420  MPa
S'e  0.5 S ut S'e  210.0  MPa
15. From the tables and formulas in Section 6.6 we have:
Load Cload  0.70

Size Csize  1
Surface A  4.51 b  0.265
b
 Sut 
Csurf  A    Csurf  0.91
 MPa 
Temperature Ctemp  1

Reliability Creliab  0.814

and the endurance limit is


S e  Cload  Csize Csurf  Ctemp Creliab S'e S e  108.88 MPa

16. The corrected endurance strength and the ultimate tensile strength are used in equation 15.16 to find the
safety factor from the Goodman line.
© 2011 Pearson Education, Inc., Upper Saddle River, NJ. All rights reserved. This publication is protected by Copyright and written permission should be
MACHINE DESIGN - An Integrated Approach, 4th Ed. 15-25a-4

S e  S ut  σinit
Nf  Nf  10
S e  σm  σinit  S ut σa

17. Calculate the maximum bolt stress and the safety factor against yielding for S y  340  MPa.

Fb
σb  σb  233.2  MPa
At
Sy
Ny  Ny  1.5
σb

18. The load required to separate the joint and the safety factor against joint separation are found from
equations 15.14c and 15.14d.

Fi
Nsep  Nsep  69
Pmax ( 1  C)

© 2011 Pearson Education, Inc., Upper Saddle River, NJ. All rights reserved. This publication is protected by Copyright and written permission should be
MACHINE DESIGN - An Integrated Approach, 4th Ed. 15-26-1
PROBLEM 15-26
Statement: (See also Problem 6-42.) A cylindrical tank with hemispherical ends is required to hold 425 kPa
of pressurized air at room temperature. The pressure cycles from zero to maximum. The tank
diameter is 0.5 m and its length is 1 m. The hemispherical ends are attached by some number of
bolts through mating flanges on each part of the tank. A 0.5-mm-thick, compressed asbestos,
unconfined gasket is used between the 10-mm-thick steel flanges. Determine a suitable number,
class, preload for, and size of bolts to fasten the ends of the tank. Specify the bolt circle and
outside diameter of the flange needed to prevent leakage. A minimum safety factor of 2 is
desired against leakage and a safety factor of 1.5 against bolt failure for infinite life.
Given: Maximum pressure p max  425  kPa Gasket thickness tg  0.5 mm
Flange thickness tf  10 mm Member mod. Ememb  206.8  GPa
Tank diameter id  500  mm Bolt modulus Ebolt  206.8  GPa
Leakage safety factor Nsep  2 Asbestos mod. Eg  480  MPa
Bolt safety factor Nb  1.5

Design Choices:
Use 40 M16 x 1.5 , class 12.9 bolts.
Material properties for class 10.9: Bolt diameter d  16 mm
Proof strength S p  970  MPa Number of bolts Nbolts  40
Yield strength S y  1100 MPa Preload fraction fp  0.75
Ultimate strength S ut  1220 MPa

Solution: See Mathcad file P1526.


2
π id
1. Determine the load per bolt. Ptot   p max Ptot  83.4 kN
4
Ptot
P  P  2086 N
Nbolts
Pmin  0  N Pmax  P

2
2. Get the tensile stress area from Table 15-2. At  167.25 mm

3. Calculate the preload. Fi  fp  S p At Fi  121.67 kN

4. Calculate the clamp length.


Clamp length l  2  t f  tg l  20.5 mm

5. Determine the relevant ratios for the joint without the gasket from equations 15-18a and b.
d
Joint aspect ratio: j  j  0.78
l
Ememb
Plate to bolt modulus: r  r1
Ebolt

6. Calculate Cr = C using equation 15.19 and the coefficients p i from Table 15-8 for j  0.78 . Use linear
interpolation between ja  0.7 and jb  0.8
pb  pa
Interpolation equation: p  p a p b     j  ja   p a
jb  ja

Coefficients from Table 15-8: p 0  p ( 0.7773 0.7800) p 0  0.7795


© 2011 Pearson Education, Inc., Upper Saddle River, NJ. All rights reserved. This publication is protected by Copyright and written permission should be
MACHINE DESIGN - An Integrated Approach, 4th Ed. 15-26-2

p 1  p ( 1.2543 1.2503) p 1  1.2511

p 2  p ( 1.0735 1.0672) p 2  1.0684

p 3  p ( 0.3595 0.3571) p 3  0.3576

3 2
Joint stiffness constant: C  p 3  r  p 2  r  p 1  r  p 0 C  0.239

7. We can estimate the stiffness of the bolt kb' from equation 15.17 using its tensile stress area from Table 15-1 and
then estimate the material stiffness km for the no gasket case by using the expression for C in equation 15.13c,
given kb' and C.

At  Ebolt 1
  1 
d kN
Estimated bolt stiffness kb'   kb'  947.6 
l  l  mm

( 1  C) kN
Estimated member stiffness km  kb'  km  3013
C mm
8. Now we will address the unconfined gasket case. The bolt stiffness is not affected by the gasket but the
material stiffness is. We now have two springs in series, the metal, whose stiffness is calculated above, and
the gasket. These combine according to equation 14.2b. The portion of the unconfined gasket subjected to
the clamp force can be assumed to be from the outside diameter of the flange shown in Figure 15-33 to the
inside diameter of the vessel. The bolt hole should be subtracted from the gasket area. The area of the
clamped gasket around one bolt is:

Flange OD od  id  6  d od  596  mm

  od  id  Nbolts  d
π 2 2 2 2
Gasket area per bolt Ag   Ag  1865 mm
4  Nbolts

9. The stiffness of this piece of gasket material is found from equation 15.11c, where, from Table 15-10.
Ag  Eg kN
Gasket stiffness per bolt kg  kg  1790.3
tg mm

10. The combined stiffness of the gasketed joint (from equation 14.2b) is

Let the steel and gasket stiffnesses be km1  km and km2  kg

Combined gasket and cover 1 kN


stiffness km  km  1123.0
1 1 mm

km1 km2

11. The joint constant with the unconfined gasket is now


Joint constant with kb'
unconfined gasket C  C  0.458
km  kb'

and ( 1  C)  0.542

12. The portions of the applied load P felt by the bolt and the material can now be found from equations 15.13.
Pb  C P Pb  0.95 kN
Pm  ( 1  C)  P Pm  1131.5 N

© 2011 Pearson Education, Inc., Upper Saddle River, NJ. All rights reserved. This publication is protected by Copyright and written permission should be
MACHINE DESIGN - An Integrated Approach, 4th Ed. 15-26-3

13. Find the resulting loads in bolt and material after the load P is applied.
Fb  Fi  Pb Fb  122.63 kN
Fm  Fi  Pm Fm  120.54 kN
14. Because these loads are fluctuating, we need to calculate the mean and alternating components of the force felt
in the bolt. The mean and alternating forces are

Fb  Fi
Mean force Fmean  Fmean  122.2  kN
2
Fb  Fi
Alternating force Falt  Falt  477  N
2
15. The nominal mean and alternating stresses in the bolt are:

Fmean
Nominal mean stress σmnom  σmnom  730.4  MPa
At
Falt
Nominal alternating stress σanom  σanom  2.85 MPa
At

16. The fatigue stress-concentration factor for this thread is found from equation 15.15c and the mean
stress-concentration factor factor Kfm is found from equation 6.17.
0.02682  d
Fatigue factor Kf  5.7  Kf  6.1
mm

Nominal maximum stress σmaxnom  σmnom  σanom σmaxnom  733.2  MPa

Nominal minimum stress σminnom  σmnom  σanom σminnom  727.5  MPa

Kfm  return Kf if Kf  σmaxnom  S y


S y  Kf  σanom
return if Kf  σmaxnom  S y Kfm  1.482
σmnom
return 0 if Kf  σmaxnom  σminnom  2  S y

17. The local mean and alternating stresses in the bolt are then:
Local mean stress σm  Kfm σmnom σm  1082.5 MPa

Local alternating stress σa  Kf  σanom σa  17.5 MPa

18. The stress at the initial preload is

Fi 3
σinit  Kfm σinit  1.08  10  MPa
At
19. An endurance strength must be found for this material. Using the methods of Section 6.6 we find for
S ut  1220 MPa
S'e  0.5 S ut S'e  610.0  MPa
20. From the tables and formulas in Section 6.6 we have:
Load Cload  0.70
Size Csize  1

© 2011 Pearson Education, Inc., Upper Saddle River, NJ. All rights reserved. This publication is protected by Copyright and written permission should be
MACHINE DESIGN - An Integrated Approach, 4th Ed. 15-26-4

Surface A  4.51 b  0.265


b
 Sut 
Csurf  A    Csurf  0.686
 MPa 
Temperature Ctemp  1
Reliability Creliab  0.814
and the endurance limit is
S e  Cload  Csize Csurf  Ctemp Creliab S'e S e  238.42 MPa

21. The corrected endurance strength and the ultimate tensile strength are used in equation 15.16 to find the
safety factor from the Goodman line.

S e  S ut  σinit
Nf  Nf  1.5
S e  σm  σinit  S ut σa

22. Calculate the maximum bolt stress and the safety factor against yielding for S y  1100 MPa.
Fb
σb  σb  733.2  MPa
At
Sy
Ny  Ny  1.5
σb

23. The load required to separate the joint and the safety factor against joint separation are found from
equations 15.14c and 15.14d.
Fi
Nsep  Nsep  108
Pmax ( 1  C)

24. Check the bolt spacing, comparing it to the rule given in the text.

Bolt circle dia d bc  0.5 ( od  id) d bc  548  mm

π d bc
Bolt spacing space bolt  space bolt  43 mm
Nbolts
space bolt
Space/dia ratio ratio  ratio  2.7 which is OK
d

© 2011 Pearson Education, Inc., Upper Saddle River, NJ. All rights reserved. This publication is protected by Copyright and written permission should be
MACHINE DESIGN - An Integrated Approach, 4th Ed. 15-27-1
PROBLEM 15-27
Statement: (See also Problem 6-42.) A cylindrical tank with hemispherical ends is required to hold 250 kPa
of pressurized air at room temperature. The pressure cycles from zero to maximum. The tank
diameter is 0.5 m and its length is 1 m. The hemispherical ends are attached by some number of
bolts through mating flanges on each part of the tank. A confined o-ring gasket is used
between the 10-mm-thick steel flanges. Determine a suitable number, class, preload for, and size
of bolts to fasten the ends of the tank. Specify the bolt circle and outside diameter of the flange
needed to prevent leakage. A minimum safety factor of 2 is desired against leakage and a
safety factor of 1.5 against bolt failure for infinite life.
Given: Maximum pressure p max  250  kPa Bolt safety factor Nb  1.5
Flange thickness tf  10 mm Member mod. Ememb  206.8  GPa
Tank diameter id  500  mm Bolt modulus Ebolt  206.8  GPa
Leakage safety factor Nsep  2
Design Choices:
Use 24 M12 x 1.25 , class 10.9 cap screws.
Material properties for class 10.9: Bolt diameter d  12 mm
Proof strength S p  830  MPa Number of bolts Nbolts  24
Yield strength S y  940  MPa Preload fraction fp  0.75
Ultimate strength S ut  1040 MPa

Solution: See Mathcad file P1527.


2
π id
1. Determine the load per screw. Ptot   p max Ptot  49.1 kN
4
Ptot
P  P  2045.31  N
Nbolts
Pmin  0  N Pmax  P
2
2. Get the tensile stress area from Table 15-2. At  92.07  mm

3. Calculate the preload. Fi  fp  S p At Fi  57.31  kN


4. Calculate the clamp length.
Clamp length l  2  t f l  20 mm

5. Determine the relevant ratios for the joint without the gasket from equations 15-18a and b.
d
Joint aspect ratio: j  j  0.6
l
Ememb
Plate to bolt modulus: r  r1
Ebolt

6. Calculate Cr = C using equation 15.19 and the coefficients p i from Table 15-8 for j  0.6 .

Coefficients from Table 15-8: p 0  0.7709

p 1  1.2600

p 2  1.0851

p 3  0.3647

© 2011 Pearson Education, Inc., Upper Saddle River, NJ. All rights reserved. This publication is protected by Copyright and written permission should be
MACHINE DESIGN - An Integrated Approach, 4th Ed. 15-27-2

3 2
Joint stiffness constant: C  p 3  r  p 2  r  p 1  r  p 0 C  0.231

7. The portions of the applied load P felt by the bolt and the material can now be found from equations 15.13.
Pb  C P Pb  0.47 kN

Pm  ( 1  C)  P Pm  1.6 kN
8. Find the resulting loads in bolt and material after the load P is applied.
Fb  Fi  Pb Fb  57.79  kN

Fm  Fi  Pm Fm  55.74  kN
9. Calculate the alternating and mean components of the fluctuating bolt load.

Fb  Fi
Falt  Falt  0.237  kN
2
Fb  Fi
Fmean  Fmean  57.55  kN
2

10. The nominal mean and alternating stresses in the bolt are:

Fmean
Nominal mean stress σmnom  σmnom  625.1  MPa
At
Falt
Nominal alternating stress σanom  σanom  2.57 MPa
At

11. The fatigue stress-concentration factor for this thread is found from equation 15.15c and the mean
stress-concentration factor factor Kfm is found from equation 6.17.
0.02682  d
Fatigue factor Kf  5.7  Kf  6
mm

Nominal maximum stress σmaxnom  σmnom  σanom σmaxnom  627.6  MPa

Nominal minimum stress σminnom  σmnom  σanom σminnom  622.5  MPa

Kfm  return Kf if Kf  σmaxnom  S y


S y  Kf  σanom
return if Kf  σmaxnom  S y Kfm  1.479
σmnom
return 0 if Kf  σmaxnom  σminnom  2  S y

12. The local mean and alternating stresses in the bolt are then:
Local mean stress σm  Kfm σmnom σm  924.5  MPa

Local alternating stress σa  Kf  σanom σa  15.5 MPa


13. The stress at the initial preload is

Fi
σinit  Kfm σinit  920.73 MPa
At
14. An endurance strength must be found for this material. Using the methods of Section 6.6 we find for
S ut  1040 MPa
© 2011 Pearson Education, Inc., Upper Saddle River, NJ. All rights reserved. This publication is protected by Copyright and written permission should be
MACHINE DESIGN - An Integrated Approach, 4th Ed. 15-27-3

S'e  0.5 S ut S'e  520.0  MPa

15. From the tables and formulas in Section 6.6 we have:


Load Cload  0.70
Size Csize  1
Surface A  4.51 b  0.265

b
 Sut 
Csurf  A    Csurf  0.716
 MPa 
Temperature Ctemp  1
Reliability Creliab  0.814
and the endurance limit is
S e  Cload  Csize Csurf  Ctemp Creliab S'e S e  212.03 MPa
16. The corrected endurance strength and the ultimate tensile strength are used in equation 15.16 to find the safety
factor from the Goodman line.

S e  S ut  σinit
Nf  Nf  1.5
S e  σm  σinit  S ut σa

17. Calculate the maximum bolt stress and the safety factor against yielding for S y  940  MPa.

Fb
σb  σb  627.6  MPa
At
Sy
Ny  Ny  1.5
σb
18. The load required to separate the joint and the safety factor against joint separation are found from
equations 15.14c and 15.14d.

Fi
Nsep  Nsep  36
Pmax ( 1  C)

19. Check the bolt spacing, comparing it to the rules given in the text.

Flange OD od  id  6  d od  572  mm

Bolt circle dia d bc  0.5 ( od  id) d bc  536  mm

π d bc
Bolt spacing space bolt  space bolt  70.2 mm
Nbolts
space bolt
Space/dia ratio ratio  ratio  5.8 which is OK
d

© 2011 Pearson Education, Inc., Upper Saddle River, NJ. All rights reserved. This publication is protected by Copyright and written permission should be
MACHINE DESIGN - An Integrated Approach, 4th Ed. 15-28-1

PROBLEM 15-28
Statement: Calculate the proof loads (load that causes a tensile stress equal to the proof strength) for 1/2-13
UNC bolts in each SAE grade listed in Table 15-6.
Given: Bolt specification: 1/2-13 UNC
2
Bolt diameter d  0.500  in Tensile stress area At  0.1419 in

Solution: See Mathcad file P1528.

1. There are eight grade numbers in Table 15-6 that are applicable to the bolt size specified. The grade numbers an
proof strength values are:

Grade  S p 
i  1 2  8 i i

"1" 33 ksi


"2" 55 ksi
"4" 65 ksi
"5" 85 ksi
"5.2" 85 ksi
"7" 105  ksi
"8" 120  ksi
"8.2" 120  ksi

2. Use equation 15.2 to calculate the proof load.

Fp
Fp  At  S p i
i i 
lbf
 "1"
 "2"  4683
  7805
 "4"  9224
 "5"  12062
Grade   
i
 "5.2"  12062
 "7"  14900
 "8"  17028
 
 "8.2"  17028

© 2011 Pearson Education, Inc., Upper Saddle River, NJ. All rights reserved. This publication is protected by Copyright and written permission should be
MACHINE DESIGN - An Integrated Approach, 4th Ed. 15-29-1

PROBLEM 15-29
Statement: Calculate the proof loads (load that causes a tensile stress equal to the proof strength) for M20 x
2.50 bolts in each class listed in Table 15-7.
Given: Bolt specification: M20 x 2.50
2
Bolt diameter d  20 mm Tensile stress area At  244.79 mm

Solution: See Mathcad file P1529.

1. There are seven grade numbers in Table 15-7. The grade numbers and proof strength values are:

Class  S p 
i  1 2  7 i i

"4.6" 225  MPa


"4.8" 310  MPa
"5.8" 380  MPa
"8.8" 600  MPa
"9.8" 650  MPa
"10.9" 830  MPa
"12.9" 970  MPa

2. Use equation 15.2 to calculate the proof load.

Fp
Fp  At  S p i
i i 
kN
 "4.6" 
  55
 "4.8"  76
 "5.8"  93
Class   "8.8"  147
i  
 "9.8"  159
 "10.9"  203
 "12.9"  237
 

© 2011 Pearson Education, Inc., Upper Saddle River, NJ. All rights reserved. This publication is protected by Copyright and written permission should be
MACHINE DESIGN - An Integrated Approach, 4th Ed. 15-30-1

PROBLEM 15-30 _____


Statement: Determine the joint stiffness constant for the bolt and members in Problem 15-7.
Given: Bolt specification: 1/2-13 UNC
2
Bolt diameter d  0.500  in Tensile area At  0.1419 in Grip l  3  in
6
Member material matl  "steel" Bolt modulus Ebolt  30 10  psi

6
Member modulus of elasticity function: Ememb  return 30 10  psi if matl = "steel"
6
Member stiffness parameters: return 10.4 10  psi if matl = "alum"
Solution: See Problem 15-9, Table 15-8, and Mathcad file P1530.
1. Determine the relevant ratios for this joint from equations 15-18a and b.
d
Joint aspect ratio: j  j  0.167
l
Ememb
Plate to bolt modulus: r  r  1.000
Ebolt

2. Calculate Cr = C using equation 15.19 and the coefficients p i from Table 15-8 for j  0.167 . Use linear
interpolation between ja  0.1 and jb  0.2
pb  pa
Interpolation equation: p  p a p b     j  ja   p a
jb  ja

Coefficients from Table 15-8: p 0  p ( 0.4389 0.6118) p 0  0.5542

p 1  p ( 0.9197 1.1715) p 1  1.0876

p 2  p ( 0.8901 1.0875) p 2  1.0217

p 3  p ( 0.3187 0.3806) p 3  0.3600

3 2
Joint stiffness constant: C  p 3  r  p 2  r  p 1  r  p 0 C  0.128

© 2011 Pearson Education, Inc., Upper Saddle River, NJ. All rights reserved. This publication is protected by Copyright and written permission should be
MACHINE DESIGN - An Integrated Approach, 4th Ed. 15-31-1

PROBLEM 15-31 _____

Statement: Determine the joint stiffness constant for the bolt and members in Problem 15-8.
Given: Bolt specification: M14 x 2
Bolt diameter d  14 mm Clamped length l  30 mm
Member material matl  "alum" Bolt modulus Ebolt  206.8  GPa

Member modulus of elasticity function: Ememb  return 206.8  GPa if matl = "steel"
return 71.0 GPa if matl = "alum"
Solution: See problem 15-8, Table P15-8, and Mathcad file P1531.

1. Determine the relevant ratios for this joint from equations 15-18a and b.
d
Joint aspect ratio: j  j  0.467
l
Ememb
Plate to bolt modulus: r  r  0.343
Ebolt

2. Calculate Cr = C using equation 15.19 and the coefficients p i from Table 15-8 for j  0.467 . Use linear
interpolation between ja  0.4 and jb  0.5
pb  pa
Interpolation equation: p  p a p b     j  ja   p a
jb  ja

Coefficients from Table 15-8: p 0  p ( 0.7351 0.7580) p 0  0.7504

p 1  p ( 1.2612 1.2632) p 1  1.2625

p 2  p ( 1.1111 1.0979) p 2  1.1023

p 3  p ( 0.3779 0.3708) p 3  0.3732

3 2
Joint stiffness constant: C  p 3  r  p 2  r  p 1  r  p 0 C  0.432

© 2011 Pearson Education, Inc., Upper Saddle River, NJ. All rights reserved. This publication is protected by Copyright and written permission should be
MACHINE DESIGN - An Integrated Approach, 4th Ed. 15-32-1

PROBLEM 15-32 _____


Statement: Determine the joint stiffness constant for the bolt and members in Problem 15-9.
Given: Bolt specification: 7/16-14 UNC
Bolt diameter d  0.4375 in Clamped length l  2.75 in
6
Member material matl  "steel" Bolt modulus Ebolt  30 10  psi

6
Member modulus of elasticity function: Ememb  return 30 10  psi if matl = "steel"
6
return 10.4 10  psi if matl = "alum"
Solution: See Problem 15-9, Table 15-8, and Mathcad file P1532.
1. Determine the relevant ratios for this joint from equations 15-18a and b.
d
Joint aspect ratio: j  j  0.159
l
Ememb
Plate to bolt modulus: r  r  1.000
Ebolt

2. Calculate Cr = C using equation 15.19 and the coefficients p i from Table 15-8 for j  0.159 . Use linear
interpolation between ja  0.1 and jb  0.2
pb  pa
Interpolation equation: p  p a p b     j  ja   p a
jb  ja

Coefficients from Table 15-8: p 0  p ( 0.4389 0.6118) p 0  0.5411

p 1  p ( 0.9197 1.1715) p 1  1.0685

p 2  p ( 0.8901 1.0875) p 2  1.0067

p 3  p ( 0.3187 0.3806) p 3  0.3553

3 2
Joint stiffness constant: C  p 3  r  p 2  r  p 1  r  p 0 C  0.124

© 2011 Pearson Education, Inc., Upper Saddle River, NJ. All rights reserved. This publication is protected by Copyright and written permission should be
MACHINE DESIGN - An Integrated Approach, 4th Ed. 15-33-1

PROBLEM 15-33 _____

Statement: Determine the joint stiffness constant for the bolt and members in Problem 15-10.
Given: Bolt specification: M12 x 1.25
Bolt diameter d  12 mm Clamped length l  50 mm
Member material matl  "alum" Bolt modulus Ebolt  206.8  GPa

Member modulus of elasticity function: Ememb  return 206.8  GPa if matl = "steel"
return 71.0 GPa if matl = "alum"
Solution: See problem 15-10, Table 15-8, and Mathcad file P1533.
1. Determine the relevant ratios for this joint from equations 15-18a and b.
d
Joint aspect ratio: j  j  0.240
l
Ememb
Plate to bolt modulus: r  r  0.343
Ebolt

2. Calculate Cr = C using equation 15.19 and the coefficients p i from Table 15-8 for j  0.240 . Use linear
interpolation between ja  0.2 and jb  0.3

pb  pa
Interpolation equation: p  p a p b     j  ja   p a
jb  ja

Coefficients from Table 15-8: p 0  p ( 0.6118 0.6932) p 0  0.6444

p 1  p ( 1.1715 1.2426) p 1  1.1999

p 2  p ( 1.0875 1.1177) p 2  1.0996

p 3  p ( 0.3806 0.3845) p 3  0.3822

3 2
Joint stiffness constant: C  p 3  r  p 2  r  p 1  r  p 0 C  0.347

© 2011 Pearson Education, Inc., Upper Saddle River, NJ. All rights reserved. This publication is protected by Copyright and written permission should be
MACHINE DESIGN - An Integrated Approach, 4th Ed. 15-34a-1

PROBLEM 15-34a _____

Statement: Figure P15-3 shows a bolted joint eccentrically loaded in shear. The shear loads are taken by the
dowel pins, the number and size of which are given in Table P15-3. Although the figure shows 5
dowel pins, that is not the case for every row in the table. For a = 4 in, b = 4 in, l = 10 in, P = 2500
lb, and the data in the row(s) assigned from Table P15-3, find the magnitude and direction of the
total shear force on each dowel.
Given: Dimensions: a  4  in b  4  in l  10 in Eccentric load P  2500 lbf
Dowel diameters and position coordinates with respect to position C: i  1 2  5
A: d  0.250  in x  0.5 a y  0.5 b x  2.000  in y  2.000  in
1 1 1 1 1
B: d  0.250  in x  0.5 a y  0.5 b x  2.000  in y  2.000  in
2 2 2 2 2
C: d  0.250  in x  0  in y  0  in x  0.000  in y  0.000  in
3 3 3 3 3
D: d  0.250  in x  0.5 a y  0.5 b x  2.000  in y  2.000  in
4 4 4 4 4
E: d  0.250  in x  0.5 a y  0.5 b x  2.000  in y  2.000  in
5 5 5 5 5
Number of dowels in group: n  5

Solution: See Mathcad file P1534a.

1. Calculate the cross-sectional (shear) area of each dowel.

 i
π 2
A   d
i 4
2. Use equations 15.24 to find the centroid of the dowel group with respect to position C.

 A ixi  A iy i
i i
xbar  xbar  0.000  in ybar  ybar  0.000  in
 A
i  A
i
i i

3. Determine the radial distance from the group centroid to each dowel.

 
i   y i  ybar 
2 2
r  if d  0  in  x  xbar 0  in
i  i 
4. Calculate the direct shear at each dowel due to the force P acting through the centroid using equation 15.25a.

F1  if  d  0  in  0  lbf 
P
F1  500.0  lbf
i  i n  1

5. Calculate the magnitude of the moment acting on the group.


M  P ( l  xbar) M  25000  in lbf

6. Calculate the magnitude of the indirect shear at each dowel due to the moment acting on the group using
equation 15.25b.

M r
i
j  1 2  5 F2 

 rj 
i 2

7. Calculate the angle that the vector F2i makes with the positive x axis.

© 2011 Pearson Education, Inc., Upper Saddle River, NJ. All rights reserved. This publication is protected by Copyright and written permission should be
MACHINE DESIGN - An Integrated Approach, 4th Ed. 15-34a-2

s  if ( M  0  in lbf 1 1 )

   i 
α  if  x  xbar  0  in  y  ybar  0  in atan2 x  xbar y  ybar 0  deg  s 90 deg
i i i i 

i 
θ  if d  0  in α 0  deg
i i 
8. Calculate the vector components of F2i.

F2x  F2  cos θ
i i  i F2y  F2  sin θ
i i  i
9. Determine the total shear force at each dowel.

F   F 
2 2
F  2x 1  F2y
i i i i

10. Calculate the angle that Fi makes with the positive x axis.

i i
θF  if d  0  in atan2 F2x F1  F2y 0  i i i  
11. Summarize results (magnitude and direction in the last two columns):
A
i r θ F2x F2y F θF
 i i i i i i
2      
in in deg lbf lbf lbf deg
0.0491 2.828 45.0 1563 1563 1890 34.2 A
0.0491 2.828 -45.0 1563 -1563 2588 -52.9 B
0.0491 0.000 -90.0 0 0 500 -90.0 C
0.0491 2.828 -225.0 -1563 1563 1890 145.8 D
0.0491 2.828 -135.0 -1563 -1563 2588 -127.1 E

© 2011 Pearson Education, Inc., Upper Saddle River, NJ. All rights reserved. This publication is protected by Copyright and written permission should be
MACHINE DESIGN - An Integrated Approach, 4th Ed. 15-35a-1

PROBLEM 15-35a _____

Statement: Figure P15-3 shows a bolted and doweled joint eccentrically loaded in shear. The shear loads are
taken by dowel pins, the number and size of which are given in Table P15-3. Although the figure
shows 5 dowels, that is not the case for every row in the table. For a = 4 in, b = 4 in, l = 10 in, P =
2500 lb, and the data in the row(s) assigned from Table P15-3, find the total shear stress in each
dowel.
Given: Dimensions: a  4  in b  4  in l  10 in Eccentric load P  2500 lbf
Dowel diameters and position coordinates with respect to position C: i  1 2  5
A: d  0.250  in x  0.5 a y  0.5 b x  2.000  in y  2.000  in
1 1 1 1 1
B: d  0.250  in x  0.5 a y  0.5 b x  2.000  in y  2.000  in
2 2 2 2 2
C: d  0.250  in x  0  in y  0  in x  0.000  in y  0.000  in
3 3 3 3 3
D: d  0.250  in x  0.5 a y  0.5 b x  2.000  in y  2.000  in
4 4 4 4 4
E: d  0.250  in x  0.5 a y  0.5 b x  2.000  in y  2.000  in
5 5 5 5 5
Number of dowels in group: n  5
Solution: See Mathcad file P1535a.
1. Calculate the cross-sectional (shear) area of each dowel.

 i
π 2
A   d
i 4
2. Use equations 15.24 to find the centroid of the dowel group with respect to position C.

 A ixi  A iy i
i i
xbar  xbar  0.000  in ybar  ybar  0.000  in
 A
i  A
i
i i

3. Determine the radial distance from the group centroid to each dowel.

 
i   y i  ybar 
2 2
r  if d  0  in  x  xbar 0  in
i  i 
4. Calculate the direct shear at each dowel due to the force P acting through the centroid using equation 15.25a.

F1  if  d  0  in  0  lbf 
P
F1  500.0  lbf
i  i n  1

5. Calculate the magnitude of the moment acting on the group.


M  P ( l  xbar) M  25000  in lbf

6. Calculate the magnitude of the indirect shear at each dowel due to the moment acting on the group using
equation 15.25b.

M r
i
j  1 2  5 F2 

 rj 
i 2

7. Calculate the angle that the vector F2i makes with the positive x axis.

© 2011 Pearson Education, Inc., Upper Saddle River, NJ. All rights reserved. This publication is protected by Copyright and written permission should be
MACHINE DESIGN - An Integrated Approach, 4th Ed. 15-35a-2

s  if ( M  0  in lbf 1 1 )

   i 
α  if  x  xbar  0  in  y  ybar  0  in atan2 x  xbar y  ybar 0  deg  s 90 deg
i i i i 

i 
θ  if d  0  in α 0  deg
i i 
8. Calculate the vector components of F2i.

F2x  F2  cos θ
i i  i F2y  F2  sin θ
i i  i
9. Determine the total shear force at each dowel.

F   F 
2 2
F  2x 1  F2y
i i i i

10. Calculate the total shear stress at each dowel.

 F 
τ  if  d  0  in  0  ksi
i
i  i A 
 i 
11. Summarize results (the shear stress is in the last column):
A
i r θ F2x F2y F τ
 i i i i i i
2      
in in deg lbf lbf lbf ksi
0.0491 2.828 45.0 1563 1563 1890 38.5 A
0.0491 2.828 -45.0 1563 -1563 2588 52.7 B
0.0491 0.000 -90.0 0 0 500 10.2 C
0.0491 2.828 -225.0 -1563 1563 1890 38.5 D
0.0491 2.828 -135.0 -1563 -1563 2588 52.7 E

© 2011 Pearson Education, Inc., Upper Saddle River, NJ. All rights reserved. This publication is protected by Copyright and written permission should be
MACHINE DESIGN - An Integrated Approach, 4th Ed. 15-36a-1

PROBLEM 15-36a _____

Statement: Figure P15-3 shows a bolted and doweled joint eccentrically loaded in shear. The shear loads are
taken by alloy steel dowel pins, the number and size of which are given in Table P15-3.
Although the figure shows 5 dowel pins, that is not the case for every row in the table. For a = 4
in, b = 4 in, l = 10 in, P = 2500 lb, SAE grade 8 bolts, and the data in the row(s) assigned from
Table P15-3, find the factor of safety against yielding in shear for each dowel pin. See Table
15-12 for strength data.

Given: Dimensions: a  4  in b  4  in l  10 in Eccentric load P  2500 lbf


Dowel diameters and position coordinates with respect to position C: i  1 2  5
A: d  0.250  in x  0.5 a y  0.5 b x  2.000  in y  2.000  in
1 1 1 1 1
B: d  0.250  in x  0.5 a y  0.5 b x  2.000  in y  2.000  in
2 2 2 2 2
C: d  0.250  in x  0  in y  0  in x  0.000  in y  0.000  in
3 3 3 3 3
D: d  0.250  in x  0.5 a y  0.5 b x  2.000  in y  2.000  in
4 4 4 4 4
E: d  0.250  in x  0.5 a y  0.5 b x  2.000  in y  2.000  in
5 5 5 5 5
Number of dowels in group: n  5 Shear strength S ys  117  ksi

Solution: See Mathcad file P1536a.


1. Calculate the cross-sectional (shear) area of each dowel.

 i
π 2
A   d
i 4
2. Use equations 15.24 to find the centroid of the dowel group with respect to position C.

 A ixi  A iy i
i i
xbar  xbar  0.000  in ybar  ybar  0.000  in
 A
i  A
i
i i

3. Determine the radial distance from the group centroid to each dowel.

 
i   y i  ybar 
2 2
r  if d  0  in  x  xbar 0  in
i  i 
4. Calculate the direct shear at each dowel due to the force P acting through the centroid using equation 15.25a.

F1  if  d  0  in  0  lbf 
P
F1  500.0  lbf
i  i n  1

5. Calculate the magnitude of the moment acting on the group.


M  P ( l  xbar) M  25000  in lbf

6. Calculate the magnitude of the indirect shear at each dowel due to the moment acting on the group using
equation 15.25b.

M r
i
j  1 2  5 F2 

 rj 
i 2

j
7. Calculate the angle that the vector F2i makes with the positive x axis.
© 2011 Pearson Education, Inc., Upper Saddle River, NJ. All rights reserved. This publication is protected by Copyright and written permission should be
MACHINE DESIGN - An Integrated Approach, 4th Ed. 15-36a-2

s  if ( M  0  in lbf 1 1 )

i  i  i i 
α  if  x  xbar  0  in  y  ybar  0  in atan2 x  xbar y  ybar 0  deg  s 90 deg
i    
i 
θ  if d  0  in α 0  deg
i i 
8. Calculate the vector components of F2i.

F2x  F2  cos θ
i i  i F2y  F2  sin θ
i i  i
9. Determine the total shear force at each dowel.

F   F 
2 2
F  2x 1  F2y
i i i i

10. Calculate the total shear stress at each dowel.

 F 
τ  if  d  0  in  0  ksi
i
i  i A 
 i 
11. Using equation 5.9a, calculate the factor of safety against yielding in shear.

 S ys 
Ny  if  d  0  in  0
i i τ
 i 
12. Summarize results (the safety factor is in the last column):
A
i r θ F2x F2y F τ
 i i i i i i
2       Ny 
in in deg lbf lbf lbf ksi i
0.0491 2.828 45.0 1563 1563 1890 38.5 3.0 A
0.0491 2.828 -45.0 1563 -1563 2588 52.7 2.2 B
0.0491 0.000 -90.0 0 0 500 10.2 11.5 C
0.0491 2.828 -225.0 -1563 1563 1890 38.5 3.0 D
0.0491 2.828 -135.0 -1563 -1563 2588 52.7 2.2 E

© 2011 Pearson Education, Inc., Upper Saddle River, NJ. All rights reserved. This publication is protected by Copyright and written permission should be
MACHINE DESIGN - An Integrated Approach, 4th Ed. 15-37-1

PROBLEM 15-37
Statement: The coefficient of friction for an oil-lubricated, single-start, power screw thread-nut combination
is 0.10. Which of the American Standard Acme Threads in Table 15-3 will be self-locking for this
thread-nut combination? Which will be the least likely to back down in the presence of dynamic
loading? Which will be the most likely to back down in the presence of a dynamic load?

Given: Screw-nut coefficient of friction μ  0.10 Thread angle α  14.5 deg

Solution: See Table 15-3 and Mathcad file P1537.

1. There are 23 entries in Table 15-3. Establish an index variable with range 1 .. 23: i  1 2  23
2. Input the values of the major diameter, d; thread pitch, p; and pitch diameter, d p. Note that for a single-start
thread L = p.

Major Thread Pitch


Diameter Pitch Diameter
d  L  d p 
i i i

0.250 0.063 0.219


0.313 0.071 0.277
0.375 0.083 0.333
0.438 0.083 0.396
0.500 0.100 0.450
0.625 0.125 0.563
0.750 0.167 0.667
0.875 0.167 0.792
1.000 0.200 0.900
1.125 0.200 1.025
1.250 0.200 1.150
1.375 0.250 1.250
1.500 0.250 1.375
1.750 0.250 1.625
2.000 0.250 1.875
2.250 0.333 2.083
2.500 0.333 2.333
2.750 0.333 2.583
3.000 0.500 2.750
3.500 0.500 3.250
4.000 0.500 3.750
4.500 0.500 4.250
5.000 0.500 4.750

3. Calculate the right-hand side of inequality 15.6a.

L
i
selflock   cos( α)
i π d p
i

© 2011 Pearson Education, Inc., Upper Saddle River, NJ. All rights reserved. This publication is protected by Copyright and written permission should be
MACHINE DESIGN - An Integrated Approach, 4th Ed. 15-37-2

4. Display the selflock calculations. If the result is less than  the thread will be self locking if not, it will back dow
under a static load.

d  selflock 
i i
0.250 0.089
0.313 0.079
0.375 0.077
0.438 0.065
0.500 0.068
0.625 0.068
0.750 0.077
0.875 0.065
1.000 0.068
1.125 0.060
1.250 0.054
1.375 0.062
1.500 0.056
1.750 0.047
2.000 0.041
2.250 0.049
2.500 0.044
2.750 0.040
3.000 0.056
3.500 0.047
4.000 0.041
4.500 0.036
5.000 0.032

5. All of the selflock values are less than  so all threads will be selflocking under a static load. The 5.000-inch
major diameter thread has the lowest selflock value and will, therefore, be the least likely to back down in the
presence of a dynamic load. The 0.250-inch major diameter thread has the highest selflock value and is,
therefore, the most likely to back down in the presence of a dynamic load.

© 2011 Pearson Education, Inc., Upper Saddle River, NJ. All rights reserved. This publication is protected by Copyright and written permission should be
MACHINE DESIGN - An Integrated Approach, 4th Ed. 15-38-1

PROBLEM 15-38
Statement: The coefficient of friction for an oil-lubricated, single-start, power screw thread-nut combination
is 0.20. Which of the American Standard Acme Threads in Table 15-3 will have the greatest
efficiency (neglecting thrust-collar friction)?

Given: Screw-nut coefficient of friction μ  0.20 Thread angle α  14.5 deg

Solution: See Table 15-3 and Mathcad file P1538.

1. There are 23 entries in Table 15-3. Establish an index variable with range 1 .. 23: i  1 2  23
2. Input the values of the major diameter, d; thread pitch, p; and pitch diameter, d p. Note that for a single-start
thread L = p.

Major Thread Pitch


Diameter Pitch Diameter
d  L  d p 
i i i

0.250 0.063 0.219


0.313 0.071 0.277
0.375 0.083 0.333
0.438 0.083 0.396
0.500 0.100 0.450
0.625 0.125 0.563
0.750 0.167 0.667
0.875 0.167 0.792
1.000 0.200 0.900
1.125 0.200 1.025
1.250 0.200 1.150
1.375 0.250 1.250
1.500 0.250 1.375
1.750 0.250 1.625
2.000 0.250 1.875
2.250 0.333 2.083
2.500 0.333 2.333
2.750 0.333 2.583
3.000 0.500 2.750
3.500 0.500 3.250
4.000 0.500 3.750
4.500 0.500 4.250
5.000 0.500 4.750

3. Calculate the efficiency using equation 15.7d.

L π d p  cos( α)  μ  L
i i i
e  
i π d p π μ  d p  L  cos( α)
i i i

© 2011 Pearson Education, Inc., Upper Saddle River, NJ. All rights reserved. This publication is protected by Copyright and written permission should be
MACHINE DESIGN - An Integrated Approach, 4th Ed. 15-38-2

4. Display the efficiency calculations.

d  e 
i i
0.250 0.301
0.313 0.278
0.375 0.273
0.438 0.241
0.500 0.251
0.625 0.251
0.750 0.274
0.875 0.242
1.000 0.251
1.125 0.228
1.250 0.209
1.375 0.232
1.500 0.216
1.750 0.190
2.000 0.169
2.250 0.196
2.500 0.179
2.750 0.164
3.000 0.216
3.500 0.190
4.000 0.169
4.500 0.152
5.000 0.139

5. Determine the maximum efficiency and its corresponding major diameter.

emax  max( e) emax  0.301

The major diameter of the Acme thread with the greatest efficiency is 0.250 in. As a general rule, the greater the
major diameter the less efficient the screw thread is. However, as can be seen from the table above, this is not
always the case.

© 2011 Pearson Education, Inc., Upper Saddle River, NJ. All rights reserved. This publication is protected by Copyright and written permission should be
MACHINE DESIGN - An Integrated Approach, 4th Ed. 15-39-1

PROBLEM 15-39
Statement: Determine the number of engaged screw threads required to make the total stripping-shear area
of the engaged threads equal to twice the tensile stress area for each of the fine pitch screw
threads in Table 15-2.

Solution: See Table 15-2, Table 15-5, and Mathcad file P1539.
1. There are 14 entries in the fine-thread columns Table 15-2. Establish an index variable with range 1 .. 14:
i  1 2  14
2. Input the values of the major diameter, d; the pitch, p; the minor diameter, d r; and tensile stress area, At.

Major Minor Tensile


Diameter, Pitch, Diameter, Stress
mm mm mm Area, mm2
d  p  d r  At 
i i i i

8 1.00 6.77 39.17


10 1.25 8.47 61.20
12 1.25 10.47 92.07
14 1.50 12.16 124.55
16 1.50 14.16 167.25
18 1.50 16.16 216.23
20 1.50 18.16 271.50
22 1.50 20.16 333.06
24 2.00 21.55 384.42
27 2.00 24.55 495.74
30 2.00 27.55 621.2
33 2.00 30.55 760.80
36 3.00 32.32 864.94
39 3.00 35.32 1028.39

3. Determine the number of threads required to make the total stripping-shear area equal to twice the tensile stress
area.

n  As = 2  At

where n is the number of threads and As is given by equation 15.8a. Solving for n,

2  At
i
n 
i 0.8 π d r  p
i i

where the factor, 0.8, is taken from Table 15-5.

© 2011 Pearson Education, Inc., Upper Saddle River, NJ. All rights reserved. This publication is protected by Copyright and written permission should be
MACHINE DESIGN - An Integrated Approach, 4th Ed. 15-39-2

4. Display the values of n:

d  n 
i i
8 4.6
10 4.6
12 5.6
14 5.4
16 6.3
18 7.1
20 7.9
22 8.8
24 7.1
27 8.0
30 9.0
33 9.9
36 7.1
39 7.7

© 2011 Pearson Education, Inc., Upper Saddle River, NJ. All rights reserved. This publication is protected by Copyright and written permission should be
MACHINE DESIGN - An Integrated Approach, 4th Ed. 15-40-1

PROBLEM 15-40
Statement: Calculate the ultimate tensile loads (load that causes a tensile stress equal to the tensile strength)
for 1/2-13 UNC bolts in each SAE grade listed in Table 15-6.
Given: Bolt specification: 1/2-13 UNC
2
Bolt diameter d  0.500  in Tensile stress area At  0.1419 in

Solution: See Table 15-6 and Mathcad file P1540.

1. There are eight grade numbers in Table 15-6 that are applicable to the bolt size specified. The grade numbers an
tensile strength values are:

Grade  S ut 
i  1 2  8 i i

"1" 60 ksi


"2" 74 ksi
"4" 115  ksi
"5" 120  ksi
"5.2" 120  ksi
"7" 133  ksi
"8" 150  ksi
"8.2" 150  ksi

2. Use equation 15.2 to calculate the proof load.

Fut
Fut  At  S ut i
i i 
lbf
 "1" 
 "2"  8514
  10501
 "4"  16319
 "5"  17028
Grade   
i
 "5.2"  17028
 "7"  18873
 "8"  21285
 
 "8.2"  21285

© 2011 Pearson Education, Inc., Upper Saddle River, NJ. All rights reserved. This publication is protected by Copyright and written permission should be
MACHINE DESIGN - An Integrated Approach, 4th Ed. 15-41-1

PROBLEM 15-41
Statement: Calculate the ultimate tensile loads (load that causes a tensile stress equal to the tensile strength)
for M20 x 2.50 bolts in each class listed in Table 15-7.
Given: Bolt specification: M20 x 2.50
2
Bolt diameter d  20 mm Tensile stress area At  244.79 mm

Solution: See Table 15-7 and Mathcad file P1541.

1. There are seven grade numbers in Table 15-7. The grade numbers and tensile strength values are:

Class  S ut 
i  1 2  7 i i

"4.6" 400  MPa


"4.8" 420  MPa
"5.8" 520  MPa
"8.8" 830  MPa
"9.8" 900  MPa
"10.9" 1040 MPa
"12.9" 1220 MPa

2. Use equation 15.2 to calculate the ultimate tensile load.

Fut
Fut  At  S ut i
i i 
kN
 "4.6" 
  98
 "4.8"  103
 "5.8"  127
Class   "8.8"  203
i  
 "9.8"  220
 "10.9"  255
 "12.9"  299
 

© 2011 Pearson Education, Inc., Upper Saddle River, NJ. All rights reserved. This publication is protected by Copyright and written permission should be
MACHINE DESIGN - An Integrated Approach, 4th Ed. 15-42-1

PROBLEM 15-42
Statement: 3/8-in dia UNC bolts and nuts clamp a 0.75-in-thick aluminum cover plate to a 0.50-in-thick steel
flange. Determine the stiffness factor of the joints.
6
Given: Bolt diameter d  0.375  in Bolt modulus Ebolt  30 10  psi
6
Cover plate lc  0.75 in Cover plate Ec  10.4 10  psi
6
Flange lf  0.50 in Flange Ef  30 10  psi

Solution: See Mathcad file P1542.

1. Determine the relevant ratios for this joint from equations 15-18.

Clamped length: l  lc  lf l  1.25 in

d
Joint aspect ratio: j  j  0.3
l
Ef
Plate to bolt modulus: rH  rH  1
Ebolt
Ec
rL  rL  0.347
Ebolt

Low and high modulus


plate thicknesses: TL  lc TL  0.75 in

TH  lf TH  0.5 in

TL
Plate thickness ratio: t  t  0.6
TL  TH
Ec
Material modulus ratio: m  m  0.347
Ef

2. Calculate the Cr terms CH and CL for rH and rL , respectively, using equation 15.19 and the coefficients p i from
Table 15-8 for j  0.3 .

Coefficients from Table 15-8: p 0  0.6932 p 1  1.2426 p 2  1.1177


p 3  0.3845

3 2
CL  p 3  rL  p 2  rL  p 1  rL  p 0 CL  0.381
3 2
CH  p 3  rH  p 2  rH  p 1  rH  p 0 CH  0.184

3. Use equation 15.20 to calculate Ct. taking the coefficients from Table 15-9 for j  0.3 .

Coeficients from Table 15-9: q 0  0.0861 q 1  8.2344 q 2  22.274


q 3  13.963

3 2
Ct  q 3 t  q 2 t  q 1 t  q 0 Ct  0.024

© 2011 Pearson Education, Inc., Upper Saddle River, NJ. All rights reserved. This publication is protected by Copyright and written permission should be
MACHINE DESIGN - An Integrated Approach, 4th Ed. 15-42-2

4. Calculate a correction factor to linearize the result from the previous step using equation 15.21.

Correction factor: a  e
 0.0598 ln( j ) 30.1385 ln( j ) 20.4350 ln( j ) 2.3516
a  0.177

5. Using equation 15.22, calculate the joint stiffness factor C.

Joint stiffness factor: C  CH   t  a  Ct   CL  CH  C  0.303

© 2011 Pearson Education, Inc., Upper Saddle River, NJ. All rights reserved. This publication is protected by Copyright and written permission should be
MACHINE DESIGN - An Integrated Approach, 4th Ed. 15-43-1

PROBLEM 15-43
Statement: M14 x 2 bolts and nuts clamp a 16-mm-thick aluminum cover plate to a 12-mm-thick steel flange.
Determine the stiffness factor of the joints.

Given: Bolt diameter d  14 mm Bolt modulus Ebolt  206.8  GPa


Cover plate lc  16 mm Cover plate Ec  71.7 GPa
Flange lf  12 mm Flange Ef  206.8  GPa

Solution: See Mathcad file P1543.

1. Determine the relevant ratios for this joint from equations 15-18.

Clamped length: l  lc  lf l  28 mm

d
Joint aspect ratio: j  j  0.5
l
Ef
Plate to bolt modulus: rH  rH  1
Ebolt
Ec
rL  rL  0.347
Ebolt

Low and high modulus


plate thicknesses: TL  lc TL  16 mm

TH  lf TH  12 mm

TL
Plate thickness ratio: t  t  0.571
TL  TH
Ec
Material modulus ratio: m  m  0.347
Ef

2. Calculate the Cr terms CH and CL for rH and rL , respectively, using equation 15.19 and the coefficients p i from
Table 15-8 for j  0.5 .

Coefficients from Table 15-8: p 0  0.7580 p 1  1.2632 p 2  1.0979


p 3  0.3708

3 2
CL  p 3  rL  p 2  rL  p 1  rL  p 0 CL  0.437
3 2
CH  p 3  rH  p 2  rH  p 1  rH  p 0 CH  0.222

3. Use equation 15.20 to calculate Ct. taking the coefficients from Table 15-9 for j  0.5 .

Coeficients from Table 15-9: q 0  0.0533 q 1  7.8676 q 2  19.357


q 3  11.457

3 2
Ct  q 3 t  q 2 t  q 1 t  q 0 Ct  0.366

4. Calculate a correction factor to linearize the result from the previous step using equation 15.21.
© 2011 Pearson Education, Inc., Upper Saddle River, NJ. All rights reserved. This publication is protected by Copyright and written permission should be
MACHINE DESIGN - An Integrated Approach, 4th Ed. 15-43-2

Correction factor: a  e
 0.0598 ln( j ) 30.1385 ln( j ) 20.4350 ln( j ) 2.3516
a  0.135

5. Using equation 15.22, calculate the joint stiffness factor C.

Joint stiffness factor: C  CH   t  a  Ct   CL  CH  C  0.355

© 2011 Pearson Education, Inc., Upper Saddle River, NJ. All rights reserved. This publication is protected by Copyright and written permission should be
MACHINE DESIGN - An Integrated Approach, 4th Ed. 15-44-1

PROBLEM 15-44
Statement: M16 x 2.0 bolts with nuts clamp a 50-mm-thick aluminum cover plate to a 30-mm-thick steel
flange. Determine the stiffness factor of the joints.
Given: Bolt diameter d  16 mm Bolt modulus Ebolt  206.8  GPa
Cover plate lc  50 mm Cover plate Ec  71.7 GPa
Flange lf  30 mm Flange Ef  206.8  GPa

Solution: See Mathcad file P1544.


1. Determine the relevant ratios for this joint from equations 15-18.
Clamped length: l  lc  lf l  80 mm

d
Joint aspect ratio: j  j  0.2
l
Ef
Plate to bolt modulus: rH  rH  1
Ebolt
Ec
rL  rL  0.347
Ebolt

Low and high modulus


plate thicknesses: TL  lc TL  50 mm

TH  lf TH  30 mm

TL
Plate thickness ratio: t  t  0.625
TL  TH

2. Calculate the Cr terms CH and CL for rH and rL , respectively, using equation 15.19 and the coefficients
p i from Table 15-8 for j  0.2 .

Coefficients from Table 15-8: p 0  0.6118 p 1  1.1715 p 2  1.0875


p 3  0.3806

3 2
CL  p 3  rL  p 2  rL  p 1  rL  p 0 CL  0.320
3 2
CH  p 3  rH  p 2  rH  p 1  rH  p 0 CH  0.147

3. Use equation 15.20 to calculate Ct. taking the coefficients from Table 15-9 for j  0.2 .

Coeficients from Table 15-9: q 0  0.1010 q 1  8.5465 q 2  24.166


q 3  15.497

3 2
Ct  q 3 t  q 2 t  q 1 t  q 0 Ct  0.214

4. Calculate a correction factor to linearize the result from the previous step using equation 15.21.

Correction factor: a  e
 0.0598 ln( j ) 30.1385 ln( j ) 20.4350 ln( j ) 2.3516
a  0.214
5. Using equation 15.22, calculate the joint stiffness factor C.

Joint stiffness factor: C  CH   t  a  Ct   CL  CH  C  0.248


© 2011 Pearson Education, Inc., Upper Saddle River, NJ. All rights reserved. This publication is protected by Copyright and written permission should be
MACHINE DESIGN - An Integrated Approach, 4th Ed. 15-45-1

PROBLEM 15-45
Statement: 5/16-18 UNC bolts with nuts clamp a 1.625-in-thick gray cast iron cover plate to a 1.5-in-thick
steel flange. Determine the stiffness factor of the joints.
Given: Bolt diameter d  0.3125 in Bolt modulus Ebolt  30000  ksi
Cover plate lc  1.625  in Cover plate Ec  15000  ksi
Flange lf  1.500  in Flange Ef  30000  ksi

Solution: See Mathcad file P1545.


1. Determine the relevant ratios for this joint from equations 15-18.
Clamped length: l  lc  lf l  3.125  in

d
Joint aspect ratio: j  j  0.1
l
Ef
Plate to bolt modulus: rH  rH  1
Ebolt
Ec
rL  rL  0.5
Ebolt

Low and high modulus


plate thicknesses: TL  lc TL  1.625  in

TH  lf TH  1.5 in

TL
Plate thickness ratio: t  t  0.52
TL  TH

2. Calculate the Cr terms CH and CL for rH and rL , respectively, using equation 15.19 and the coefficients
p i from Table 15-8 for j  0.1 .

Coefficients from Table 15-8: p 0  0.4389 p 1  0.9197 p 2  0.8901


p 3  0.3187

3 2
CL  p 3  rL  p 2  rL  p 1  rL  p 0 CL  0.162
3 2
CH  p 3  rH  p 2  rH  p 1  rH  p 0 CH  0.091

3. Use equation 15.20 to calculate Ct. taking the coefficients from Table 15-9 for j  0.1 .

Coeficients from Table 15-9: q 0  0.0079 q 1  17.040 q 2  92.832


q 3  202.44 q 4  209.38 q 5  82.726

5 4 3 2
Ct  q 5 t  q 4 t  q 3 t  q 2 t  q 1 t  q 0 Ct  0.068

4. Calculate a correction factor to linearize the result from the previous step using equation 15.21.

Correction factor: a  e
 0.0598 ln( j ) 30.1385 ln( j ) 20.4350 ln( j ) 2.3516
a  0.260
5. Using equation 15.22, calculate the joint stiffness factor C.

Joint stiffness factor: C  CH   t  a  Ct   CL  CH  C  0.129


© 2011 Pearson Education, Inc., Upper Saddle River, NJ. All rights reserved. This publication is protected by Copyright and written permission should be
MACHINE DESIGN - An Integrated Approach, 4th Ed. 15-46-1

PROBLEM 15-46
Statement: M16 x 1.5 bolts with nuts clamp a 8-mm-thick titanium cover plate to a 8-mm-thick stainless
steel flange. Determine the stiffness factor of the joints.
Given: Bolt diameter d  16 mm Bolt modulus Ebolt  206.8  GPa
Cover plate lc  8  mm Cover plate Ec  113.8  GPa
Flange lf  8  mm Flange Ef  189.6  GPa

Solution: See Mathcad file P1546.


1. Determine the relevant ratios for this joint from equations 15-18.
Clamped length: l  lc  lf l  16 mm

d
Joint aspect ratio: j  j 1
l
Ef
Plate to bolt modulus: rH  rH  0.917
Ebolt
Ec
rL  rL  0.55
Ebolt

Low and high modulus


plate thicknesses: TL  lc TL  8  mm

TH  lf TH  8  mm

TL
Plate thickness ratio: t  t  0.5
TL  TH

2. Calculate the Cr terms CH and CL for rH and rL , respectively, using equation 15.19 and the coefficients
p i from Table 15-8 for j  1 .

Coefficients from Table 15-8: p 0  0.4389 p 1  0.9197 p 2  0.8901


p 3  0.3187

3 2
CL  p 3  rL  p 2  rL  p 1  rL  p 0 CL  0.149
3 2
CH  p 3  rH  p 2  rH  p 1  rH  p 0 CH  0.098

3. Use equation 15.20 to calculate Ct. taking the coefficients from Table 15-9 for j  1 .

Coeficients from Table 15-9: q 0  0.0079 q 1  17.040 q 2  92.832


q 3  202.44 q 4  209.38 q 5  82.726

5 4 3 2
Ct  q 5 t  q 4 t  q 3 t  q 2 t  q 1 t  q 0 Ct  0.124

4. Calculate a correction factor to linearize the result from the previous step using equation 15.21.

Correction factor: a  e
 0.0598 ln( j ) 30.1385 ln( j ) 20.4350 ln( j ) 2.3516
a  0.095
5. Using equation 15.22, calculate the joint stiffness factor C.

Joint stiffness factor: C  CH   t  a  Ct   CL  CH  C  0.124


© 2011 Pearson Education, Inc., Upper Saddle River, NJ. All rights reserved. This publication is protected by Copyright and written permission should be
MACHINE DESIGN - An Integrated Approach, 4th Ed. 15-47-1

PROBLEM 15-47
Statement: An M12 X 1.25 soft steel nut is assembled with a hardened steel bolt. The nut is 11 mm thick
and has a shear yield-strength of 120 MPa. Determine the axial force that will cause stripping
failure of the nut if the nut threads fail before the bolt fails.

Given: Major dia. of bolt d  12 mm Pitch p  1.25 mm


Nut thickness h  11 mm
Shear strength S ys  120  MPa

Assumptions: The nut major diameter is equal to the bolt major diameter.
Solution: See Table 15-2, Table 15-5, and Mathcad file P1547.
1. Calculate the approximate number of threads in the nut.

h
Number of threads n  n  8.8
p

2. Determine the area factor for nut thread stripping from Table 15-5.
Area factor wo  0.88

3. Calculate the shear area in the nut using equation 15.8b and the number of threads from step 1.

2
Shear area As  n  π d  wo p As  364.9 mm

4. Using equation 15.8c, calculate the tensile force that will cause the nut threads to fail in shear.

Failure load Ffail  As  S ys Ffail  43.8 kN

© 2011 Pearson Education, Inc., Upper Saddle River, NJ. All rights reserved. This publication is protected by Copyright and written permission should be
MACHINE DESIGN - An Integrated Approach, 4th Ed. 15-48-1

PROBLEM 15-48
Statement: Compare the yield loads (load that causes a tensile stress equal to the yield strength) to the proof
loads (load that causes a tensile stress equal to the proof strength) for M12 x 1.25 bolts in each
class listed in Table 15-7.
Given: Bolt specification: M12 x 1.25
2
Bolt diameter d  12 mm Tensile stress area At  92.07  mm

Solution: See Tables 15-2 and 15-7 and Mathcad file P1548.

1. There are seven grade numbers in Table 15-7. The grade numbers, proof strength, and yield strength values are:

Class  S p  S y 
i  1 2  7 i i i

"4.6" 225  MPa 240  MPa


"4.8" 310  MPa 340  MPa
"5.8" 380  MPa 420  MPa
"8.8" 600  MPa 660  MPa
"9.8" 650  MPa 720  MPa
"10.9" 830  MPa 940  MPa
"12.9" 970  MPa 1100 MPa

2. Use equation 15.2 to calculate the proof load and the yield load. The percent difference is in the last column.
Fp  At  S p Fy  At  S y
i i i i

Fy  Fp
Fp Fy i i 1
i i  
  Fp %
kN kN i
 "4.6" 
  21 22 6.7
 "4.8"  29 31 9.7
 "5.8"  35 39 10.5
Class   "8.8"  55
i   61 10.0
 "9.8"  60
66 10.8
 "10.9"  76
87 13.3
 "12.9"  89
  101 13.4

© 2011 Pearson Education, Inc., Upper Saddle River, NJ. All rights reserved. This publication is protected by Copyright and written permission should be
MACHINE DESIGN - An Integrated Approach, 4th Ed. 15-49-1

PROBLEM 15-49
Statement: An M16 x 1.50, class 4.8 bolt with cut threads is preloaded to 85% of its proof strength when
clamping a 20-mm-thick sandwich of solid steel. Find the safety factors against static yielding
and joint separation when a static 3-kN external load is applied.
Given: Bolt diameter d  16 mm Material properties for class 4.8:
Preload fraction fp  0.85 Proof strength S p  310  MPa
Clamp length l  20 mm Yield strength S y  340  MPa
Number of bolts Nbolts  1 Ultimate strength S ut  420  MPa
Applied load Ptot  3  kN Young's modulus E  206.8  GPa
Bolt modulus Ebolt  E Member modulus Ememb  E

Solution: See Mathcad file P1549.

Ptot
1. Determine the load per bolt. P  P  3  kN
Nbolts
2
2. Get the tensile stress area from Table 15-2. At  167.25 mm

3. Calculate the preload. Fi  fp  S p At Fi  44 kN


4. Determine the relevant ratios for this joint from equations 15-18a and b.
d
Joint aspect ratio: j  j  0.8
l
Ememb
Plate to bolt modulus: r  r1
Ebolt

5. Calculate Cr = C using equation 15.19 and the coefficients p i from Table 15-8 for j  0.8 .

Coefficients from Table 15-8: p 0  0.7800

p 1  1.2503

p 2  1.0672

p 3  0.3571
3 2
Joint stiffness constant: C  p 3  r  p 2  r  p 1  r  p 0 C  0.240

6. The portions of the applied load P felt by the bolt and the material can now be found from equations 15.13.
Pb  C P Pb  0.7 kN

Pm  ( 1  C)  P Pm  2.3 kN
7 Find the resulting loads in bolt and material after the load P is applied.
Fb  Fi  Pb Fb  44.8 kN

Fm  Fi  Pm Fm  41.8 kN

8 The maximum tensile stress in the bolt is

Fb
σb  σb  38.8 ksi
At
© 2011 Pearson Education, Inc., Upper Saddle River, NJ. All rights reserved. This publication is protected by Copyright and written permission should be
MACHINE DESIGN - An Integrated Approach, 4th Ed. 15-49-2

9. This is a uniaxial stress situation, so the principal stress and von Mises stress are identical to the applied
tensile stress. The safety factor against yielding for Grade 4.8 with S y  340  MPa is then

Sy
Ny  Ny  1.3
σb

10. The load required to separate the joint and the safety factor against joint separation are found from equations
15.14c and 15.14d.

Fi
P0  P0  58.0 kN
1C
P0
Nsep  Nsep  19.3
P

© 2011 Pearson Education, Inc., Upper Saddle River, NJ. All rights reserved. This publication is protected by Copyright and written permission should be
MACHINE DESIGN - An Integrated Approach, 4th Ed. 15-50-1

PROBLEM 15-50
Statement: A 15-mm-thick steel cap is to be fastened to a 15-mm-thick steel flange with 6 bolts and nuts.
The external load on the cap is 30 kN. Size and specify the bolts for a safety factor of at least
1.5 and specify the torque required on each bolt to obtain the preload if the threads are
lubricated.
Given: Clamp length l  30 mm Number of bolts Nbolts  6
Bolt modulus Ebolt  206.8  GPa Member mod. Ememb  206.8  GPa
Applied load Ptot  30 kN
Design Choices:
Use M10 x 1.5 , class 5.8 bolts.
Material properties for class 5.8: Bolt diameter d  10 mm
Proof strength S p  380  MPa
Yield strength S y  420  MPa Preload fraction fp  0.70

Solution: See Mathcad file P1550.

Ptot
1. Determine the load per bolt. P  P  5.00 kN
Nbolts

2
2. Get the tensile stress area from Table 15-2. At  57.99  mm

3. Calculate the preload. Fi  fp  S p At Fi  15.43  kN


4. Determine the relevant ratios for this joint from equations 15-18a and b.
d
Joint aspect ratio: j  j  0.333
l
Ememb
Plate to bolt modulus: r  r1
Ebolt

5. Calculate Cr = C using equation 15.19 and the coefficients p i from Table 15-8 for j  0.333 . Use linear
interpolation between ja  0.3 and jb  0.4
pb  pa
Interpolation equation: p  p a p b     j  ja   p a
jb  ja

Coefficients from Table 15-8: p 0  p ( 0.6932 0.7351) p 0  0.7072

p 1  p ( 1.2426 1.2612) p 1  1.2488

p 2  p ( 1.1177 1.1111) p 2  1.1155

p 3  p ( 0.3845 0.3779) p 3  0.3823

3 2
Joint stiffness constant: C  p 3  r  p 2  r  p 1  r  p 0 C  0.192
6. The portions of the applied load P felt by the bolt and the material can now be found from equations 15.13.
Pb  C P Pb  1.0 kN

Pm  ( 1  C)  P Pm  4.0 kN
7 Find the resulting loads in bolt and material after the load P is applied.

© 2011 Pearson Education, Inc., Upper Saddle River, NJ. All rights reserved. This publication is protected by Copyright and written permission should be
MACHINE DESIGN - An Integrated Approach, 4th Ed. 15-50-2

Fb  Fi  Pb Fb  16.4 kN

Fm  Fi  Pm Fm  11.4 kN

8 The maximum tensile stress in the bolt is

Fb
σb  σb  282.5  MPa
At

9. This is a uniaxial stress situation, so the principal stress and von Mises stress are identical to the applied
tensile stress. The safety factor against yielding for Grade 5.8 with S y  420  MPa is then

Sy
Ny  Ny  1.5
σb

10. The load required to separate the joint and the safety factor against joint separation are found from equations
15.14c and 15.14d.

Fi
P0  P0  19.1 kN
1C
P0
Nsep  Nsep  3.8
P

11. Use equation 15.23d to calculate the tightening torque.

Ti  0.21 Fi d Ti  32.4 N  m

© 2011 Pearson Education, Inc., Upper Saddle River, NJ. All rights reserved. This publication is protected by Copyright and written permission should be
MACHINE DESIGN - An Integrated Approach, 4th Ed. 15-51-1

PROBLEM 15-51
Statement: Repeat Problem 15-50 with a total external load on the six bolts that varies from 0 to 30 kN per
cycle. Design these bolts for infinite life with a factor of safety of at least 1.5. Specify their
size, class, preload, and tightening torque.

Given: Total force Ptot  30 kN Member mod. Ememb  206.8  GPa
Number of bolts Nbolts  6 Bolt modulus Ebolt  206.8  GPa

Design Choices:
Use M10 x 1.5 , class 5.8 bolts with rolled threads.
Material properties for class 5.8: Bolt diameter d  10 mm
Proof strength S p  380  MPa Preload fraction fp  0.70
Yield strength S y  420  MPa Design safety factor Nd  1.5
Ultimate strength S ut  520  MPa Clamp length l  30 mm

Solution: See Mathcad file P1551.


1. Determine the load per bolt.
Ptot
P  P  5  kN
Nbolts

Pmin  0  N Pmax  P
2
2. Get the tensile stress area from Table 15-2. At  57.99  mm

3. Calculate the preload. Fi  fp  S p At Fi  15.4 kN

4. Determine the relevant ratios for this joint from equations 15-18a and b.
d
Joint aspect ratio: j  j  0.333
l
Ememb
Plate to bolt modulus: r  r1
Ebolt

5. Calculate Cr = C using equation 15.19 and the coefficients p i from Table 15-8 for j  0.333 . Use linear
interpolation between ja  0.3 and jb  0.4
pb  pa
Interpolation equation: p  p a p b     j  ja   p a
jb  ja

Coefficients from Table 15-8: p 0  p ( 0.6932 0.7351) p 0  0.7072

p 1  p ( 1.2426 1.2612) p 1  1.2488

p 2  p ( 1.1177 1.1111) p 2  1.1155

p 3  p ( 0.3845 0.3779) p 3  0.3823

3 2
Joint stiffness constant: C  p 3  r  p 2  r  p 1  r  p 0 C  0.192

6. The portions of the applied load P felt by the bolt and the material can now be found from equations 15.13.
Pb  C P Pb  0.96 kN
© 2011 Pearson Education, Inc., Upper Saddle River, NJ. All rights reserved. This publication is protected by Copyright and written permission should be
MACHINE DESIGN - An Integrated Approach, 4th Ed. 15-51-2

Pm  ( 1  C)  P Pm  4.0 kN
7. Find the resulting loads in bolt and material after the load P is applied.
Fb  Fi  Pb Fb  16.38  kN

Fm  Fi  Pm Fm  11.38  kN
8. Calculate the alternating and mean components of the fluctuating bolt load.

Fb  Fi
Falt  Falt  0.479  kN
2
Fb  Fi
Fmean  Fmean  15.90  kN
2

9. The fatigue stress-concentration factor Kf  2.2 for rolled threads is taken from Table 15-8. The mean
stress-concentration factor is taken as Kfm  1 in this case. The mean and alternating stresses in the bolt are:

Falt
σalt  Kf  σalt  18.17  MPa
At
Fmean
σmean  Kfm σmean  274.26 MPa
At

10. The stress at the initial preload is

Fi
σinit  Kfm σinit  266.00 MPa
At

11. An endurance strength must be found for this material. Using the methods of Section 6.6 we find for
S ut  520  MPa
S'e  0.5 S ut S'e  260.0  MPa

12. From the tables and formulas in Section 6.6 we have:


Load Cload  0.70
Size Csize  1

Surface A  4.51 b  0.265


b
 Sut 
Csurf  A    Csurf  0.86
 MPa 
Temperature Ctemp  1

Reliability Creliab  0.814

and the endurance limit is

S e  Cload  Csize Csurf  Ctemp Creliab S'e S e  127.39 MPa

13. The corrected endurance strength and the ultimate tensile strength are used in equation 15.16 to find the safety
factor from the Goodman line.

© 2011 Pearson Education, Inc., Upper Saddle River, NJ. All rights reserved. This publication is protected by Copyright and written permission should be
MACHINE DESIGN - An Integrated Approach, 4th Ed. 15-51-3

S e  S ut  σinit
Nf  Nf  3.1
S e  σmean  σinit  S ut σalt

14. Calculate the maximum bolt stress and the safety factor against yielding for S y  420  MPa.
Fb
σb  σb  282.5  MPa
At
Sy
Ny  Ny  1.5
σb

15. The load required to separate the joint and the safety factor against joint separation are found from equations
15.14c and 15.14d.

Fi
Nsep  Nsep  3.8
Pmax ( 1  C)

16. Use equation 15.23d to calculate the tightening torque.

Ti  0.21 Fi d Ti  32.4 N  m

© 2011 Pearson Education, Inc., Upper Saddle River, NJ. All rights reserved. This publication is protected by Copyright and written permission should be
MACHINE DESIGN - An Integrated Approach, 4th Ed. 15-52-1

PROBLEM 15-52
Statement: A 20-mm-thick aluminum cap is to be fastened to a 20-mm-thick aluminum flange with 8 bolts
and nuts. The external load on the cap varies from 0 to 40 kN per cycle. Size and specify the
bolts for infinite life and a safety factor of at least 1.5 and specify the torque required on each
bolt to obtain the preload if the threads are lubricated.

Given: Total force Ptot  40 kN Member mod. Ememb  71.7 GPa
Number of bolts Nbolts  8 Bolt modulus Ebolt  206.8  GPa

Design Choices:
Use M10 x 1.5 , class 8.8 bolts with rolled threads.
Material properties for class 8.8: Bolt diameter d  10 mm
Proof strength S p  600  MPa Preload fraction fp  0.70
Yield strength S y  660  MPa Design safety factor Nd  1.5
Ultimate strength S ut  830  MPa Clamp length l  40 mm

Solution: See Mathcad file P1552.


1. Determine the load per bolt.
Ptot
P  P  5  kN
Nbolts

Pmin  0  N Pmax  P
2
2. Get the tensile stress area from Table 15-2. At  57.99  mm

3. Calculate the preload. Fi  fp  S p At Fi  24.4 kN


4. Determine the relevant ratios for this joint from equations 15-18a and b.
d
Joint aspect ratio: j  j  0.250
l
Ememb
Plate to bolt modulus: r  r  0.347
Ebolt

5. Calculate Cr = C using equation 15.19 and the coefficients p i from Table 15-8 for j  0.25 . Use linear
interpolation between ja  0.2 and jb  0.3
pb  pa
Interpolation equation: p  p a p b     j  ja   p a
jb  ja

Coefficients from Table 15-8: p 0  p ( 0.6118 0.6932) p 0  0.6525

p 1  p ( 1.1715 1.2426) p 1  1.2070

p 2  p ( 1.0875 1.1177) p 2  1.1026

p 3  p ( 0.3806 0.3845) p 3  0.3825

3 2
Joint stiffness constant: C  p 3  r  p 2  r  p 1  r  p 0 C  0.351

6. The portions of the applied load P felt by the bolt and the material can now be found from equations 15.13.
Pb  C P Pb  1.75 kN

© 2011 Pearson Education, Inc., Upper Saddle River, NJ. All rights reserved. This publication is protected by Copyright and written permission should be
MACHINE DESIGN - An Integrated Approach, 4th Ed. 15-52-2

Pm  ( 1  C)  P Pm  3.2 kN
7. Find the resulting loads in bolt and material after the load P is applied.
Fb  Fi  Pb Fb  26.11  kN

Fm  Fi  Pm Fm  21.11  kN
8. Calculate the alternating and mean components of the fluctuating bolt load.

Fb  Fi
Falt  Falt  0.877  kN
2
Fb  Fi
Fmean  Fmean  25.23  kN
2

9. The fatigue stress-concentration factor Kf  3.0 for rolled threads is taken from Table 15-8. The mean
stress-concentration factor is taken as Kfm  1 in this case. The mean and alternating stresses in the bolt are:

Falt
σalt  Kf  σalt  45.34  MPa
At
Fmean
σmean  Kfm σmean  435.11 MPa
At

10. The stress at the initial preload is

Fi
σinit  Kfm σinit  420.00 MPa
At

11. An endurance strength must be found for this material. Using the methods of Section 6.6 we find for
S ut  830  MPa

S'e  0.5 S ut S'e  415.0  MPa

12. From the tables and formulas in Section 6.6 we have:


Load Cload  0.70
Size Csize  1

Surface A  4.51 b  0.265


b
 Sut 
Csurf  A    Csurf  0.76
 MPa 
Temperature Ctemp  1

Reliability Creliab  0.814


and the endurance limit is

S e  Cload  Csize Csurf  Ctemp Creliab S'e S e  179.64 MPa

13. The corrected endurance strength and the ultimate tensile strength are used in equation 15.16 to find the safety
factor from the Goodman line.

© 2011 Pearson Education, Inc., Upper Saddle River, NJ. All rights reserved. This publication is protected by Copyright and written permission should be
MACHINE DESIGN - An Integrated Approach, 4th Ed. 15-52-3

S e  S ut  σinit
Nf  Nf  1.8
S e  σmean  σinit  S ut σalt

14. Calculate the maximum bolt stress and the safety factor against yielding for S y  660  MPa.
Fb
σb  σb  450.2  MPa
At
Sy
Ny  Ny  1.5
σb

15. The load required to separate the joint and the safety factor against joint separation are found from equations
15.14c and 15.14d.

Fi
Nsep  Nsep  7.5
Pmax ( 1  C)

16. Use equation 15.23d to calculate the tightening torque.

Ti  0.21 Fi d Ti  51.1 N  m

© 2011 Pearson Education, Inc., Upper Saddle River, NJ. All rights reserved. This publication is protected by Copyright and written permission should be
MACHINE DESIGN - An Integrated Approach, 4th Ed. 16-1-1

PROBLEM 16-1
Statement: A submerged-arc complete joint penetration (CJP) butt weld was made between two sections of
A36 hot-rolled steel plate. The plate is 10-in wide by 1/2-in-thick. E70 electrode was used. How
much tensile load can the assembly withstand across the weld without yielding in either base
metal or weld?

Given: Plate: L  10 in T  0.5 in S y  36 ksi


Electrode: S ut  70 ksi

Assumptions: There are no stress concentrations or residual stresses. The yield strength of the electrode material
is
S ey  0.75 S ut S ey  52.5 ksi

Solution: See Mathcad file P1601.

1. Calculate the cross-sectional area of both the plate and the weld.
2
A  L T A  5  in

2. Both the plate and weld are in pure tension so the tensile stress is the only nonzero principle stress and is
also the von Mises stress. Calculate the load that will cause failure in the weld and in the plate. The lesser of
the two is the maximum load that can be applied to the joint.

Weld Pweld  A  S ey Pweld  262.5  kip

Plate Pplate  A  S y Pplate  180  kip

The maximum load that can be applied to the joint without yielding is Pplate  180  kip

© 2011 Pearson Education, Inc., Upper Saddle River, NJ. All rights reserved. This publication is protected by Copyright and written permission should be
MACHINE DESIGN - An Integrated Approach, 4th Ed. 16-2-1

PROBLEM 16-2
Statement: The plate of Problem 16-1 has partial joint penetration (PJP) welds applied from each side. If each
weld has a throat dimension of 1/4 in, determine the maximum allowable tensile load across the
weld.

Given: Plate: L  10 in T  0.5 in S y  36 ksi


Electrode: S ut  70 ksi
Weld: t  0.25 in two places
Assumptions: The weld will fail in shear along the weld throat and, from equation 16-1, the maximum allowable
shear stress is τmax  0.3 S ut , τmax  21 ksi
Solution: See Mathcad file P1602.

1. Calculate the cross-sectional areas of the weld and the plate.


2
Aweld  L 2  t Aweld  5  in

2
Aplate  L T Aplate  5  in

2. Calculate the load that will cause failure in the weld and in the plate.

Weld: Pweld  Aweld  τmax Pweld  105  kip

Plate: Pplate  Aplate  S y Pplate  180  kip

The limiting load is Pweld  105  kip

© 2011 Pearson Education, Inc., Upper Saddle River, NJ. All rights reserved. This publication is protected by Copyright and written permission should be
MACHINE DESIGN - An Integrated Approach, 4th Ed. 16-3-1

PROBLEM 16-3
Statement: A tee bracket similar to that shown in Figure 16-11 is to have 1/2-in-thick A572 Grade 42 steel
welded with 3/16-in fillet welds along both inside corners using an E70 electrode. If it will be
subjected to a 20-kip tensile load on the leg of the tee, determine the minimum required length,
L, of the bracket based on full-length welds.

Given: Plates: T  0.5 in S put  70.3 ksi


Electrode: Exx  70 ksi
Weld: w  0.1875 in two places
Load: P  20 kip
Assumptions: The weld will fail in shear along the weld throat and, from equation 16-1, the maximum allowable
shear stress is τmax  0.3 Exx , τmax  21 ksi
Solution: See Figure 16-11, Table 16-3, and Mathcad file P1603.
1. Determine the throat dimension of the welds.
t  0.707  w t  0.133 in
2. Determine the shear area in the throats of the welds as a function of L and then solve for L using equation 4.9.

Ashear P
Ashear = L 2  t L= and τxy = τmax =
2 t Ashear
P
Solving for L, L  L  3.592 in
2  t τmax

3. Check whether this value for L gives an acceptable safety factor against yielding in the fused base metal.
The shear area on the leg along the width of the welds is Afusion  2  L w , and, from Table 16-3 the yield
strength of the base metal is S y  42 ksi. The shear yield strength of the base metal is S sy  0.577  S y . The
factor of safety against yielding in the fused base metal is:

S sy Afusion Afusion
Nyield = =  S sy Nyield   S sy Nyield  1.63
τxy P P

2
where Afusion  1.347 in and S sy  24.2 ksi

© 2011 Pearson Education, Inc., Upper Saddle River, NJ. All rights reserved. This publication is protected by Copyright and written permission should be
MACHINE DESIGN - An Integrated Approach, 4th Ed. 16-4-1

PROBLEM 16-4
Statement: Figure P16-1 shows a bar welded to a base on three sides with 3/16-in fillet welds using an E70
electrode. Material is A572 Grafde 50 hot-rolled steel. What is your recommended maximum
static load P that can safely be applied?

Given: Bar dims: a  6  in b  3  in d  2  in T  0.25 in


Electrode: Exx  70 ksi
Weld: w  0.1875 in three places
Material: S y  50 ksi S ut  65 ksi
Assumptions: The weld will fail in shear along the weld throat and, from equation 16-1, the maximum allowable
shear stress is τmax  0.3 Exx , τmax  21 ksi. Consider the weld as a line. The weld pattern
matches that of part 5 of Figure 16-15. The weld will limit the load as Category F.

Solution: See Figure P16-1, Figure 16-15, and Mathcad file P1604.
1. Determine the throat dimension of the welds.
t  0.707  w t  0.133 in
2. Determine the shear area in the throats of the welds as a line function.
2
in
Aw  d  2  b Aw  8 
in

3. Find the centroid of the weld pattern with the equation given in part 5 of Figure 16-5.
2
b
xbar  xbar  1.125 in
2 b  d

4. Find the moment arm from the applied load to the centroid.
r  a  b  xbar r  7.875 in
5. Find the unit load due to direct shear and the applied torque as functions of P.
P
fs ( P)  T ( P)  P r
Aw
6. From part 5 of Figure 16-15 the torsional geometry factor is:
3 2 2 4
( 2 b  d) b  ( b  d) in
Jw   Jw  14.542
12 2 b  d in

7. The unit load due to torsional shear is a vector that is perpendicular to a line from the centroid to any point
on the weld. It is a maximum at a point furthest from the centroid. In this case, it is the furtherest point to the
right on either the top or bottom of the bar. In either case, it will have horizontal and vertical components.
The vertical component will add to the direct shear and that total will add vectorially to the horizontal
component. The moment arm for the horizontal component is d/2, and for the vertical component it is b -
xbar.

T ( P)  d
Horizontal unit load: fth ( P) 
2  Jw

T ( P)  ( b  xbar)
Vertical unit load: ftv ( P) 
Jw

fth ( P)   fs ( P)  f tv ( P) 
2 2
Total unit load: FR( P) 
© 2011 Pearson Education, Inc., Upper Saddle River, NJ. All rights reserved. This publication is protected by Copyright and written permission should be
MACHINE DESIGN - An Integrated Approach, 4th Ed. 16-4-2

8. Solve for P setting the shear stress equal to the maximum allowable stress, which is (0.3)E70.

Guess P  3  kip

Given FR( P)
t= P  Find ( P)
τmax

Recommended maximum static load: P  2.2 kip


kip
Primary shear: fs ( P)  0.276 
in

Shear due to moment:

kip
Horizontal component: fth ( P)  1.194 
in

kip
Vertical component: ftv ( P)  2.239 
in
kip
Total unit load: FR( P)  2.784 
in

© 2011 Pearson Education, Inc., Upper Saddle River, NJ. All rights reserved. This publication is protected by Copyright and written permission should be
MACHINE DESIGN - An Integrated Approach, 4th Ed. 16-5-1

PROBLEM 16-5
Statement: Figure P16-1 shows a bar welded to a base on three sides with 3/16-in fillet welds using an E70
electrode. Material is A572 Grade 50 hot-rolled steel. What is your recommended maximum
dynamic repeated load, zero to Pmax that can be applied for 10E8 cycles with a safety factor of
1.6?

Given: Bar dims: a  6  in b  3  in d  2  in T  0.25 in


Electrode: Exx  70 ksi
Weld: w  0.1875 in three places Nfr  1.6
Assumptions: The weld will fail in shear along the weld throat. Consider the weld as a line. The weld pattern
matches that of part 5 of Figure 16-15. The weld will limit the load as Category F.
Solution: See Figure P16-1, Figure 16-15, and Mathcad file P1605.
1. Determine the throat dimension of the welds.
t  0.707  w t  0.133 in
2. Determine the shear area in the throats of the welds as a line function.
2
in
Aw  d  2  b Aw  8 
in
3. Find the centroid of the weld pattern with the equation given in part 5 of Figure 16-5.
2
b
xbar  xbar  1.125 in
2 b  d

4. Find the moment arm from the applied load to the centroid.
r  a  b  xbar r  7.875 in
5. Find the unit load due to direct shear and the applied torque as functions of P.
P
fs ( P)  T ( P)  P r
Aw
6. From part 5 of Figure 16-15 the torsional geometry factor is:
3 2 2 4
( 2 b  d) b  ( b  d) in
Jw   Jw  14.542
12 2 b  d in
7. The unit load due to torsional shear is a vector that is perpendicular to a line from the centroid to any point
on the weld. It is a maximum at a point furthest from the centroid. In this case, it is the furtherest point to the
right on either the top or bottom of the bar. In either case, it will have horizontal and vertical components.
The vertical component will add to the direct shear and that total will add vectorially to the horizontal
component. The moment arm for the horizontal component is d/2, and for the vertical component it is b -
xbar.

T ( P)  d
Horizontal unit load: fth ( P) 
2  Jw

T ( P)  ( b  xbar)
Vertical unit load: ftv ( P) 
Jw

fth ( P)   fs ( P)  f tv ( P) 
2 2
Total unit load: FR( P) 

8. From Table 16-5a, a Category F joint has a shear stress-range endurance strength of S ers  8  ksi. Using the
given safety factor, the allowable shear stress is
© 2011 Pearson Education, Inc., Upper Saddle River, NJ. All rights reserved. This publication is protected by Copyright and written permission should be
MACHINE DESIGN - An Integrated Approach, 4th Ed. 16-5-2

S ers
Allowable shear stress: τallow  τallow  5  ksi
Nfr

9. Solve for P setting the shear stress equal to the maximum allowable stress.

Guess P  1  kip

Given FR( P)
t= Pmax  Find ( P)
τallow

Recommended maximum dynamic load: Pmax  525  lbf

fs  Pmax  65.6
lbf
Primary shear:
in

Shear due to moment:

fth  Pmax  284.3 


lbf
Horizontal component:
in

ftv  Pmax  533.1 


lbf
Vertical component:
in

FR Pmax  662.8 


lbf
Total unit load:
in

© 2011 Pearson Education, Inc., Upper Saddle River, NJ. All rights reserved. This publication is protected by Copyright and written permission should be
MACHINE DESIGN - An Integrated Approach, 4th Ed. 16-6a-1

PROBLEM 16-6a
Statement: Figure P16-2 shows a bracket welded to a wall with a fillet weld using an E70 electrode. For the
row a in Table P16-1, determine the fillet weld size needed between the tube and the wall for a
static load F. The pipe and wall material is A36 steel.

Given: Bracket dims: OD  3.5 in ID  3.068  in a  2  OD l  2.5 OD


Electrode: Exx  70 ksi
Load: F  2.5 kip
Assumptions: The weld will fail in shear along the weld throat and, from equation 16-1, the maximum allowable
shear stress is τmax  0.3 Exx , τmax  21 ksi. Consider the weld as a line. The weld pattern
matches that of part 9 of Figure 16-5. The weld will limit the load as Category F.

Solution: See Figure P16-2, Figure 16-15, and Mathcad file P1606a.
1. Determine the shear area, bending factor, and torsion factor in the throats of the welds as line functions from
Figure 16-15, part 9.
2
in
Aw  π OD Aw  10.996
in
2 3
OD in
S w  π S w  9.621 
4 in

3 4
OD in
Jw  π Jw  33.674
4 in

2. The weld has a combined bending and torsion load. The maximum stress will occur at the top, center of the
pipe at the wall. There will be a direct shear component, a bending component, and a torsional moment
component.

3. Find the unit load due to direct shear.


F lbf
fs  fs  227.364 
Aw in
4. Find the unit load due to bending at the critical point.
M kip
M  F  l fb  fb  2.274 
Sw in

5. Find the unit load due to the torsional moment.


T  OD kip
T  F  a ft  ft  0.909 
2  Jw in

6. Find the magnitude of the resultant unit load at the critical point ( fs is in the negative y-direction, f b is in the
positive x-direction, and ft is in the negative z-direction).
2 2 2 kip
Total unit load: FR  fs  fb  ft FR  2.459 
in
7. Using equation 16.1, solve for the minimum required weld size.
FR
t  w  1.414  t
τmax

t  0.117 in w  0.166 in Say w = 3/16 in

© 2011 Pearson Education, Inc., Upper Saddle River, NJ. All rights reserved. This publication is protected by Copyright and written permission should be
MACHINE DESIGN - An Integrated Approach, 4th Ed. 16-7a-1

PROBLEM 16-7a
Statement: Figure P16-2 shows a bracket welded to a wall with a fillet weld using an E70 electrode. For the
row a in Table P16-1, determine the fillet weld size needed between the tube and the wall for a
dynamic load that varies from - 0.1F to +0.2F using a safety factor of 1.5. The pipe and wall
material is A36 steel.

Given: Bracket dims: OD  3.5 in ID  3.068  in a  2  OD l  2.5 OD


Electrode: Exx  70 ksi
Load: F  2.5 kip Nfr  1.5
Assumptions: The weld will fail in shear along the weld throat. Consider the weld as a line. The weld pattern
matches that of part 9 of Figure 16-15. The weld will limit the load as Category F.
Solution: See Figure P16-2, Figure 16-15, and Mathcad file P1607a.
1. Determine the dynamic load range.
∆F  0.2 F  ( 0.1F ) ∆F  750  lbf
2. Determine the shear area, bending factor, and torsion factor in the throats of the welds as line functions from
Figure 16-15, part 9.
2
in
Aw  π OD Aw  10.996
in
2 3
OD in
S w  π S w  9.621 
4 in

3 4
OD in
Jw  π Jw  33.674
4 in
3. The weld has a combined bending and torsion load. The maximum stress will occur at the top, center of the
pipe at the wall. There will be a direct shear component, a bending component, and a torsional moment
component.

4. Find the unit load due to direct shear.


∆F lbf
fs  fs  68.209
Aw in
5. Find the unit load due to bending at the critical point.
M kip
M  ∆F  l fb  fb  0.682 
Sw in

6. Find the unit load due to the torsional moment.


T  OD kip
T  ∆F  a ft  ft  0.273 
2  Jw in

7. Find the magnitude of the resultant unit load at the critical point ( fs is in the negative y-direction, f b is in the
positive x-direction, and ft is in the negative z-direction).

2 2 2 kip
Total unit load: FR  fs  fb  ft FR  0.738 
in
8. From Table 16-5a, a Category F joint has a shear stress-range endurance strength of S ers  8  ksi. Using the
given safety factor, the allowable shear stress is

© 2011 Pearson Education, Inc., Upper Saddle River, NJ. All rights reserved. This publication is protected by Copyright and written permission should be
MACHINE DESIGN - An Integrated Approach, 4th Ed. 16-7a-2

S ers
Allowable shear stress: τallow  τallow  5.333  ksi
Nfr

9. Using equation 16.1, solve for the minimum required weld size.

FR
t  w  1.414  t
τallow

t  0.138 in w  0.196 in Say w = 1/4 in

© 2011 Pearson Education, Inc., Upper Saddle River, NJ. All rights reserved. This publication is protected by Copyright and written permission should be
MACHINE DESIGN - An Integrated Approach, 4th Ed. 16-8-1

PROBLEM 16-8
Statement: Figure P16-3 shows a bracket machined from 12-mm-thick A572 Grade 50 hot-rolled steel flat
stock. It is welded to a support with a fillet weld all around using an E80 electrode. Determine
the fillet weld size needed between the bracket and the support for a static load of P = 12 kN.

Given: Distance from support to point D a  200  mm


Bracket dims: d  75 mm b  12 mm
Electrode: Exx  80 ksi Exx  552  MPa
Load: P  12 kN

Assumptions: The weld will fail in shear along the weld throat and, from equation 16-1, the maximum allowable
shear stress is τmax  0.3 Exx , τmax  165  MPa. Consider the weld as a line. The weld pattern
matches that of part 7 of Figure 16-5. The weld will limit the load as Category F.
Solution: See Figure P16-2, Figure 16-15, and Mathcad file P1608.
1. Determine the shear area and bending factor, in the throats of the welds as line functions from Figure 16-15,
part 7.
2
mm
Aw  2  b  2  d Aw  174 
mm
2 3
d mm
S w  b  d  S w  2775
3 mm

2. The weld has a combined bending and shear load. The maximum stress will occur at the top, center of the
bracket at the support. There will be a direct shear component and a bending component.
3. Find the unit load due to direct shear.
P kN
fs  fs  0.069 
Aw mm
4. Find the unit load due to bending at the critical point.
M kN
M  P a fb  fb  0.865 
Sw mm

5. Find the magnitude of the resultant unit load at the critical point.
2 2 kN
Total unit load: FR  fs  fb FR  0.868 
mm
6. Using equation 16.1, solve for the minimum required weld size.

FR
t  w  1.414  t
τmax

t  5.24 mm w  7.41 mm Say w = 8 mm

© 2011 Pearson Education, Inc., Upper Saddle River, NJ. All rights reserved. This publication is protected by Copyright and written permission should be
MACHINE DESIGN - An Integrated Approach, 4th Ed. 16-9-1

PROBLEM 16-9
Statement: Figure P16-3 shows a bracket machined from 12-mm-thick A572 Grade 50 hot-rolled steel flat
stock. It is welded to a support with a fillet weld all around using an E90 electrode. Determine
the fillet weld size needed between the bracket and the support for a dynamic load that varies
from 0 to +3 kN using a safety factor of 1.8.

Given: Distance from support to point D a  200  mm


Bracket dims: d  75 mm b  12 mm
Electrode: Exx  90 ksi Exx  621  MPa
Load: F  3  kN Nfr  1.8
Assumptions: The weld will fail in shear along the weld throat. Consider the weld as a line. The weld pattern
matches that of part 7 of Figure 16-15. The weld will limit the load as Category F.
Solution: See Figure P16-2, Figure 16-15, and Mathcad file P1609.
1. Determine the dynamic load range.
∆F  F  0  kN ∆F  3  kN

2. Determine the shear area and bending factor, in the throats of the welds as line functions from Figure 16-15,
part 7.
2
mm
Aw  2  b  2  d Aw  174 
mm
2 3
d mm
S w  b  d  S w  2775
3 mm

3. The weld has a combined bending and shear load. The maximum stress will occur at the top, center of the
bracket at the support. There will be a direct shear component and a bending component.
4. Find the unit load due to direct shear.
F kN
fs  fs  0.017 
Aw mm

5. Find the unit load due to bending at the critical point.


M kN
M  F  a fb  fb  0.216 
Sw mm

6. Find the magnitude of the resultant unit load at the critical point.
2 2 kN
Total unit load: FR  fs  fb FR  0.217 
mm

8. From Table 16-5b, a Category F joint has a shear stress-range endurance strength of S ers  55 MPa. Using
the given safety factor, the allowable shear stress is
S ers
Allowable shear stress: τallow  τallow  30.6 MPa
Nfr

9. Using equation 16.1, solve for the minimum required weld size.

FR
t  w  1.414  t
τallow

t  7.099 mm w  10.037 mm Say w = 10 mm

© 2011 Pearson Education, Inc., Upper Saddle River, NJ. All rights reserved. This publication is protected by Copyright and written permission should be
MACHINE DESIGN - An Integrated Approach, 4th Ed. 16-10-1

PROBLEM 16-10
Statement: Two 8-mm-thick by 50-mm-wide, A572 Grade 42 steel straps are welded together with fillet
welds in a lap joint using an E70 electrode. The tensile load on the straps is 45 kN. What is
your recommended weld size for the two full-length welds?

Given: Straps: T  8  mm W  50 mm S y  345  MPa


Electrode: Exx  70 ksi Exx  483  MPa
Load: P  45 kN

Assumptions: Each weld will carry half the load and will fail in shear along the weld throat and, from equation
16-1, the maximum allowable shear stress is τmax  0.3 Exx , τmax  145  MPa
Solution: See Figures 16-2 and 16-3, Table 16-1, and Mathcad file P1610.
1. Determine the transverse load carried by each weld.
P
Transverse load on weld Pt  Pt  22.5 kN
2

2. Determine the shear area in the throats of the welds as a function of t and then solve for t using equation 16.1.

Ashear Pt
Ashear = W  t t= and τxy = τmax =
W Ashear
Pt
Solving for t, t  t  3.1 mm
W  τmax

3. Convert this throat dimension to a leg width w assuming an equal-leg fillet.


t
Leg width w  w  4.4 mm
cos( 45 deg)

4. Check this against the recommended minimum weld size for this thickness strap. Table 16-2 indicates that a
8-mm-thick part needs at least a 5-mm weld leg width, so increase the weld leg width to 5 mm. Set w  5  mm

5. Check whether this value for w gives an acceptable safety factor against yielding in the fused base metal.
The shear area on the leg along the width of the welds is Afusion  W  w , and, from Table 16-3 the yield
strength of the base metal is S y  290  MPa. The shear yield strength of the base metal is S sy  0.577  S y .
The factor of safety against yielding in the fused base metal is:

S sy Afusion Afusion
Nyield = =  S sy Nyield   S sy Nyield  1.9
τxy Pt Pt

2
where Afusion  250  mm and S sy  167  MPa

This is slightly less than the safety factor against static failure in the weld (see Table 16-1) but is acceptable.

© 2011 Pearson Education, Inc., Upper Saddle River, NJ. All rights reserved. This publication is protected by Copyright and written permission should be
MACHINE DESIGN - An Integrated Approach, 4th Ed. 16-11-1

PROBLEM 16-11
Statement: Two 8-mm-thick by 50-mm-wide, A572 Grade 42 steel straps are welded together with fillet welds
in a lap joint using an E70 electrode. Determine the fillet weld size needed for a dynamic load
that varies from 0 to +12 kN using a safety factor of 1.5 for an infinite life of the two full-length
welds.

Given: Straps: T  8  mm W  50 mm S ut  485  MPa


Electrode: Exx  70 ksi Exx  483  MPa
Load: Fmin  0  kN Fmax  12 kN Nfrs  1.5

Assumptions: Each weld will carry half the load and will fail in shear along the weld throat. Since the weld
takes the load directly this is a Category F weldment.
Solution: See Figure P16-2, Figure 16-15, and Mathcad file P1611.
1. Determine the dynamic load range per weld.
Fmax  Fmin
∆F  ∆F  6  kN
2

2. From Table 16-5b, a Category F joint has a shear stress-range endurance strength of S ers  55 MPa.
Using the given safety factor, the allowable shear stress is
S ers
Allowable shear stress: τallow  τallow  36.7 MPa
Nfrs

3. Using equation 4.9, solve for the allowable shear area and the resulting minimum value of the throat
dimension t.
Allowable shear area ∆F 2
on each throat Ashear  Ashear  163.6 mm
τallow

Minimum throat Ashear


dimension t  t  3.273 mm
W

4. Convert this throat dimension to a leg width w assuming an equal-leg fillet.


t
Leg width w  w  4.6 mm
cos( 45 deg)

5. Check this against the recommended minimum weld size for this thickness strap. Table 16-2 indicates that a
8-mm-thick part needs at least a 5-mm weld leg width, so increase the weld leg width to 5 mm. Set w  5  mm
6. Check whether this value for w gives an acceptable safety factor against fatigue failure in the fused base
metal by assuming that this is a Category C weldment. The most likely point of failure is at the weld toe.
The weld leg area is Afusion  W  w , and, from Table 16-5b the endurance strength is S er  69 MPa

Fmax
Max stress at toe of σtoe  σtoe  24 MPa
each weld 2  Afusion

Factor of safety S er
in base metal Nf  Nf  2.9
σtoe

Since Nf is larger than Nfrs, the initial assumption that this was a Category F weldment was correct.

© 2011 Pearson Education, Inc., Upper Saddle River, NJ. All rights reserved. This publication is protected by Copyright and written permission should be
MACHINE DESIGN - An Integrated Approach, 4th Ed. 16-12-1

PROBLEM 16-12
Statement: Two 12-mm-thick by 50-mm-wide, weldable aluminum straps are welded together with fillet
welds in a lap joint using an aluminum electrode. Determine the fillet weld size needed for a
dynamic load that varies from 0 to +5 kN using a safety factor of 2.0 for an infinite life of the two
full-length welds.

Given: Straps: T  12 mm W  50 mm


Load: Fmin  0  kN Fmax  5  kN Nfrs  2.0

Assumptions: Each weld will carry half the load and will fail in shear along the weld throat. Since the weld
takes the load directly this is a Category F weldment.
Solution: See Figure P16-2, Figure 16-15, and Mathcad file P1612.
1. Determine the dynamic load range per weld.
Fmax  Fmin
∆F  ∆F  2.5 kN
2

2. From Table 16-5b and the notation in the text that the values in Table 16-5 can be reduced by a factor of
three for aluminum, a Category F joint has a shear stress-range endurance strength of S ers  0.333  55 MPa
S ers  18.3 MPa. Using the given safety factor, the allowable shear stress is

S ers
Allowable shear stress: τallow  τallow  9.2 MPa
Nfrs

3. Using equation 4.9, solve for the allowable shear area and the resulting minimum value of the throat
dimension t.
Allowable shear area ∆F 2
on each throat Ashear  Ashear  273 mm
τallow

Minimum throat Ashear


dimension t  t  5.46 mm
W

4. Convert this throat dimension to a leg width w assuming an equal-leg fillet.


t
Leg width w  w  7.7 mm
cos( 45 deg)

5. Check this against the recommended minimum weld size for this thickness strap. Table 16-2 indicates that
a 12-mm-thick part needs at least a 5-mm weld leg width. The calculated leg width is adequate but increase
it to the next higher mm integer. Set w  8  mm
6. Check whether this value for w gives an acceptable safety factor against fatigue failure in the fused base
metal by assuming that this is a Category C weldment. The most likely point of failure is at the weld toe.
The weld leg area is Afusion  W  w , and, from Table 16-5b the endurance strength is
S er  0.333  69 MPa
S er  23.0 MPa
Fmax
Max stress at toe of σtoe  σtoe  6.25 MPa
each weld 2  Afusion

Factor of safety S er
in base metal Nf  Nf  3.7
σtoe

Since Nf is larger than Nfrs, the initial assumption that this was a Category F weldment was correct.
© 2011 Pearson Education, Inc., Upper Saddle River, NJ. All rights reserved. This publication is protected by Copyright and written permission should be
MACHINE DESIGN - An Integrated Approach, 4th Ed. 17-1-1

PROBLEM 17-1
Statement: Find the torque that a 2-surface, dry disk clutch can transmit if the outside and inside lining
diameters are 120 mm and 70 mm, respectively, and the applied axial force is 10 kn. Assume
uniform wear and m = 0.4. Is the pressure on the lining acceptable? What lining materials
would be suitable?

Given: Number of surfaces Ns  2 Clutch dimensions:


Axial force F  10 kN Outside diameter od  120  mm
Friction coefficient μ  0.4 Inside diameter id  70 mm
Assumptions: Uniform wear model.
Solution: See Mathcad file P1701.

1. Calculate the outside and inside radii of the disk.


Outside radius ro  0.5 od ro  60 mm
Inside radius ri  0.5 id ri  35 mm

ro  ri
2. Using equation 17.6, calculate the torque capacity. T  Ns μ  F  T  380  N  m
2

3. Calculate the maximum lining pressure using equation 17.5a.

F
Maximum pressure p max 
2  π ri  ro  ri

p max  1819 kPa

4. From Table 17-1, we see that either a molded or sintered metal lining are suitable.

© 2011 Pearson Education, Inc., Upper Saddle River, NJ. All rights reserved. This publication is protected by Copyright and written permission should be
MACHINE DESIGN - An Integrated Approach, 4th Ed. 17-2-1

PROBLEM 17-2
Statement: Find the torque that a 2-surface, dry disk clutch can transmit if the outside and inside lining
diameters are 120 mm and 70 mm, respectively, and the applied axial force is 10 kn. Assume
uniform pressure and  = 0.4. Is the pressure on the lining acceptable? What lining materials
would be suitable?

Given: Number of surfaces Ns  2 Clutch dimensions:


Axial force F  10 kN Outside diameter od  120  mm
Friction coefficient μ  0.4 Inside diameter id  70 mm
Assumptions: Uniform pressure model.

Solution: See Mathcad file P1702.

1. Calculate the outside and inside radii of the disk.


Outside radius ro  0.5 od ro  60 mm

Inside radius ri  0.5 id ri  35 mm

 3  3
2  ro  ri 
2. Using equation 17.3, calculate the torque capacity. T  Ns μ  F   T  389  N  m
3  2 2
 ro  ri 
3. Calculate the lining pressure using equation 17.2c.
3 T
Maximum pressure p   p  1340 kPa
π μ  Ns  ro  ri 
2 3 3

4. From Table 17-1, we see that either a molded or sintered metal lining are suitable.

© 2011 Pearson Education, Inc., Upper Saddle River, NJ. All rights reserved. This publication is protected by Copyright and written permission should be
MACHINE DESIGN - An Integrated Approach, 4th Ed. 17-3-1

PROBLEM 17-3
Statement: Design a single-surface disk clutch to transmit 100 N-m of torque at 750 rpm using a molded
lining with a maximum pressure of 1000 kPa and  = 0.25. Assume uniform wear. Find the
outside and inside diameters required if ri = 0.577 ro. What is the power transmitted?
Given: Number of surfaces Ns  1 Torque capacity T  100  N  m
Friction coefficient μ  0.25 Maximum pressure p max  1000 kPa
Disk radius factor fr  0.577 Rotational speed ω  750  rpm

Assumptions: Uniform wear model.

Solution: See Mathcad file P1703.

1. Use equation 17.5b and the disk radius factor to solve for the required outside and inside radii.

T = π μ  ri p max   ro  ri
2 2
 ri = fr  ro

1
3
ro   T  ro  69.16  mm
  2 
 π μ  fr  p max   1  fr  
Round this to ro  70 mm

Then, the inside radius is ri  fr  ro ri  40.39  mm

Round this to ri  40 mm

2. Convert the radii to diameters.

Outside diameter od  2  ro od  140  mm

Inside diameter id  2  ri id  80 mm

3. Calculate the maximum transmitted power.

Transmitted power H  T  ω H  7.85 kW

© 2011 Pearson Education, Inc., Upper Saddle River, NJ. All rights reserved. This publication is protected by Copyright and written permission should be
MACHINE DESIGN - An Integrated Approach, 4th Ed. 17-4-1

PROBLEM 17-4
Statement: Design a single-surface disk clutch to transmit 100 N-m of torque at 750 rpm using a molded
lining with a maximum pressure of 1000 kPa and  = 0.25. Assume uniform pressure. Find the
outside and inside diameters required if ri = 0.577 ro. What is the power transmitted?
Given: Number of surfaces Ns  1 Torque capacity T  100  N  m
Friction coefficient μ  0.25 Maximum pressure p  1000 kPa
Disk radius factor fr  0.577 Rotational speed ω  750  rpm
Assumptions: Uniform pressure model.

Solution: See Mathcad file P1704.

1. Use equation 17.2c and the disk radius factor to solve for the required outside and inside radii.

 π p  μ   ro  ri
2 3 3
T=
3   Ns ri = fr  ro

1
3
 3 T 
ro  ro  61.832 mm
  3 
 2  π μ  p  N s   1  f r  
Round this to ro  62 mm

Then, the inside radius is ri  fr  ro ri  35.774 mm

Round this to ri  36 mm

2. Convert the radii to diameters.

Outside diameter od  2  ro od  124  mm

Inside diameter id  2  ri id  72 mm

3. Calculate the maximum transmitted power.

Transmitted power H  T  ω H  7.85 kW

© 2011 Pearson Education, Inc., Upper Saddle River, NJ. All rights reserved. This publication is protected by Copyright and written permission should be
MACHINE DESIGN - An Integrated Approach, 4th Ed. 17-5-1
PROBLEM 17-5
Statement: How many surfaces are needed in a wet disk clutch to transmit 120 N-m of torque at 1000 rpm
using a sintered lining with a maximum pressure of 1800 kPa and  = 0.06. Assume uniform
wear. Find the outside and inside diameters required if ri = 0.577 ro. How many disks are
needed? What is the power transmitted?
Given: Friction coefficient μ  0.06 Torque capacity T  120  N  m
Disk radius factor fr  0.577 Maximum pressure p max  1800 kPa
Rotational speed ω  1000 rpm
Assumptions: Uniform wear model.

Design Choices:
The number of surfaces depends on the disk dimensions. Let ri  30 mm

Solution: See Mathcad file P1705.


ri
1. Calculate the outside radius. ro  ro  51.993 mm ro  52 mm
fr

2. Convert the radii to diameters.


Outside diameter od  2  ro od  104  mm

Inside diameter id  2  ri id  60 mm

3. Calculate the required axial force using equation 17.5a.

Axial force F  2  π ri p max   ro  ri F  7.464  kN

4. Using equation 17.6, calculate the number of friction surfaces required.


2 T
Number surfaces Ns  ceil 
 Ns  7
 μ  F   ro  ri
5. Calculate the maximum transmitted power.

Transmitted power H  T  ω H  12.6 kW

© 2011 Pearson Education, Inc., Upper Saddle River, NJ. All rights reserved. This publication is protected by Copyright and written permission should be
MACHINE DESIGN - An Integrated Approach, 4th Ed. 17-6-1

PROBLEM 17-6
Statement: How many surfaces are needed in a wet disk clutch to transmit 120 N-m of torque at 1000 rpm
using a sintered lining with a maximum pressure of 1800 kPa and  = 0.06. Assume uniform
pressure. Find the outside and inside diameters required if ri = 0.577 ro. How many disks are
needed? What is the power transmitted?
Given: Friction coefficient μ  0.06 Torque capacity T  120  N  m
Disk radius factor fr  0.577 Pressure p  1800 kPa
Rotational speed ω  1000 rpm
Assumptions: Uniform pressure model.

Design Choices:
The number of surfaces depends on the disk dimensions. Let ri  30 mm

Solution: See Mathcad file P1706.


ri
1. Calculate the outside radius. ro  ro  51.993 mm ro  52 mm
fr

2. Convert the radii to diameters.


Outside diameter od  2  ro od  104  mm

Inside diameter id  2  ri id  60 mm


3. Calculate the required axial force using equation 17.1b.

F  π p   ro  ri
2 2
Axial force  F  10.201 kN

4. Using equation 17.3, calculate the number of friction surfaces required.

 3 T   ro2  ri2 
Number surfaces Ns  ceil
   Ns  5
  μ  F   r 3  r 3 
2 o i 

5. Calculate the maximum transmitted power.

Transmitted power H  T  ω H  12.6 kW

© 2011 Pearson Education, Inc., Upper Saddle River, NJ. All rights reserved. This publication is protected by Copyright and written permission should be
MACHINE DESIGN - An Integrated Approach, 4th Ed. 17-7-1

PROBLEM 17-7
Statement: Figure P17-1 shows a single short-shoe drum brake. Find its torque capacity and required
actuating force for the dimensions given below. What value of c will make it self-locking?
Given: Pivot to load a  100mm Drum radius r  30 mm
Pivot to y-axis b  70 mm Drum width w  50 mm
Pivot to x-axis e  20 mm Shoe angle θ  35 deg
Maximum pressure p max  1300 kPa Friction coeff. μ  0.3

Assumptions: Short-shoe theory is appropriate. The drum rotates CCW.

Solution: See Figure P17-1 and Mathcad file P1707.


1. Determine the normal force on the drum from equation 17.8.
Normal force Fn  p max  r θ  w Fn  1.191  kN

2. Use equation 17.10 to calculate the torque capacity.


Torque capacity T  μ  Fn r T  10.7 N  m
3. Determine the required actuating force from equation 17.11b and the brake geometry.

Distance c c  r  e c  10 mm
b  μ c
Actuation force Fa  Fn Fa  798  N
a
4. Check to see if the brake is self-locking using the relationship given in the text.

self_locking  return "yes" if μ  c  b


"no" otherwise

self_locking  "no"

5. Calculate the value of c that would make the brake self-locking use the above relationship.

b
Value of c to self-lock clock  clock  233.3  mm
μ

© 2011 Pearson Education, Inc., Upper Saddle River, NJ. All rights reserved. This publication is protected by Copyright and written permission should be
MACHINE DESIGN - An Integrated Approach, 4th Ed. 17-8-1

PROBLEM 17-8
Statement: Figure P17-1 shows a single short-shoe drum brake. Find its torque capacity and required
actuating force for the dimensions given below. What value of c will make it self-locking?
Given: Pivot to load a  100mm Drum radius r  30 mm
Pivot to y-axis b  70 mm Drum width w  50 mm
Pivot to x-axis e  20 mm Shoe angle θ  35 deg
Maximum pressure p max  1300 kPa Friction coefficient μ  0.3

Assumptions: Short-shoe theory is appropriate. The drum rotates CW.

Solution: See Figure P17-1 and Mathcad file P1708.


1. Determine the normal force on the drum from equation 17.8.
Normal force Fn  p max  r θ  w Fn  1.191  kN

2. Use equation 17.10 to calculate the torque capacity.


Torque capacity T  μ  Fn r T  10.7 N  m

3. Determine the required actuating force from equation 17.11b and the brake geometry modified for CW rotation.

Distance c c  r  e c  10 mm
b  μ c
Actuation force Fa  Fn Fa  870  N
a

4. Since the brake is not self-energizing with CW rotation, it cannot be self-locking for any value of c.

© 2011 Pearson Education, Inc., Upper Saddle River, NJ. All rights reserved. This publication is protected by Copyright and written permission should be
MACHINE DESIGN - An Integrated Approach, 4th Ed. 17-9-1

PROBLEM 17-9
Statement: Figure P17-1 shows a single short-shoe drum brake. Find its torque capacity and required
actuating force for the dimensions given below. What value of c will make it self-locking?

Given: Pivot to load a  8.00 in Drum radius r  5.00 in


Pivot to y-axis b  6.00 in Drum width w  1.50 in
Pivot to x-axis e  4.00 in Shoe angle θ  30 deg
Maximum pressure p max  250  psi Friction coeff. μ  0.35

Assumptions: Short-shoe theory is appropriate. The drum rotates CCW.

Solution: See Figure P17-1 and Mathcad file P1709.


1. Determine the normal force on the drum from equation 17.8.
Normal force Fn  p max  r θ  w Fn  981.7  lbf

2. Use equation 17.10 to calculate the torque capacity.


Torque capacity T  μ  Fn r T  1718 in lbf
3. Determine the required actuating force from equation 17.11b and the brake geometry.

Distance c c  r  e c  1.000  in
b  μ c
Actuation force Fa  Fn Fa  693  lbf
a
4. Check to see if the brake is self-locking using the relationship given in the text.

self_locking  return "yes" if μ  c  b


"no" otherwise

self_locking  "no"

5. Calculate the value of c that would make the brake self-locking use the above relationship.

b
Value of c to self-lock clock  clock  17.1 in
μ

© 2011 Pearson Education, Inc., Upper Saddle River, NJ. All rights reserved. This publication is protected by Copyright and written permission should be
MACHINE DESIGN - An Integrated Approach, 4th Ed. 17-10-1

PROBLEM 17-10
Statement: Figure P17-1 shows a single short-shoe drum brake. Find its torque capacity and required
actuating force for the dimensions given below. What value of c will make it self-locking?

Given: Pivot to load a  8.00 in Drum radius r  5.00 in


Pivot to y-axis b  6.00 in Drum width w  1.50 in
Pivot to x-axis e  4.00 in Shoe angle θ  30 deg
Maximum pressure p max  250  psi Friction coeff. μ  0.35

Assumptions: Short-shoe theory is appropriate. The drum rotates CW.

Solution: See Figure P17-1 and Mathcad file P1710.


1. Determine the normal force on the drum from equation 17.8.
Normal force Fn  p max  r θ  w Fn  981.7  lbf

2. Use equation 17.10 to calculate the torque capacity.


Torque capacity T  μ  Fn r T  1718 in lbf
3. Determine the required actuating force from equation 17.11b and the brake geometry modified for CW rotation.

Distance c c  r  e c  1.000  in
b  μ c
Actuation force Fa  Fn Fa  779  lbf
a

4. Since the brake is not self-energizing with CW rotation, it cannot be self-locking for any value of c.

© 2011 Pearson Education, Inc., Upper Saddle River, NJ. All rights reserved. This publication is protected by Copyright and written permission should be
MACHINE DESIGN - An Integrated Approach, 4th Ed. 17-11-1

PROBLEM 17-11
Statement: Figure P17-2 shows a double short-shoe drum brake. Find its torque capacity and required
actuating force for the dimensions given below. What value of c will make it self-locking? Hint:
Calculate the effects of each shoe separately and superpose them.

Given: Pivot to load a  90mm Drum radius r  40 mm


Pivot to y-axis b  80 mm Drum width w  60 mm
Pivot to x-axis e  30 mm Shoe angle θ  25 deg
Maximum pressure p max  1500 kPa Friction coefficient μ  0.25

Assumptions: Short-shoe theory is appropriate. The drum rotates CCW.

Solution: See Figure P17-2 and Mathcad file P1711.


Top shoe - self energizing
1. Determine the normal force on the drum from equation 17.8.
Normal force Fn1  p max  r θ  w Fn1  1.571  kN

2. Use equation 17.10 to calculate the torque capacity.


Torque capacity T1  μ  Fn1 r T1  15.71  N  m
3. Determine the required actuating force from equation 17.11b and the brake geometry.

Distance c c  r  e c  10 mm
b  μc
Actuation force Fa  Fn1 Fa  1353 N
a
4. Check to see if the brake is self-locking using the relationship given in the text.

self_locking  return "yes" if μ  c  b


"no" otherwise

self_locking  "no"

5. Calculate the value of c that would make the brake self-locking use the above relationship.

b
Value of c to self-lock clock  clock  320  mm
μ
Bottom shoe - non self energizing
6. The actuation force is the same on both shoes and is equal to the actuation force on the top shoe. Use
equation 17.11b to solve for the normal force on the bottom shoe.
a
Normal force Fn2  Fa Fn2  1476 N
b  μc

7. Use equation 17.10 to calculate the torque capacity.


Torque capacity T2  μ  Fn2 r T2  14.76  N  m

Both shoes
8. The total torque capacity is Ttot  T1  T2 Ttot  30.5 N  m

© 2011 Pearson Education, Inc., Upper Saddle River, NJ. All rights reserved. This publication is protected by Copyright and written permission should be
MACHINE DESIGN - An Integrated Approach, 4th Ed. 17-12-1

PROBLEM 17-12
Statement: Figure P17-2 shows a double short-shoe drum brake. Find its torque capacity and required
actuating force for the dimensions given below. What value of c will make it self-locking? Hint:
Calculate the effects of each shoe separately and superpose them.

Given: Pivot to load a  12.00  in Drum radius r  6.00 in


Pivot to y-axis b  8.00 in Drum width w  2.00 in
Pivot to x-axis e  3.00 in Shoe angle θ  25 deg
Maximum pressure p max  200  psi Friction coeff. μ  0.28

Assumptions: Short-shoe theory is appropriate. The drum rotates CCW.

Solution: See Figure P17-2 and Mathcad file P1712.


Top shoe - self energizing
1. Determine the normal force on the drum from equation 17.8.
Normal force Fn1  p max  r θ  w Fn1  1047 lbf

2. Use equation 17.10 to calculate the torque capacity.


Torque capacity T1  μ  Fn1 r T1  1759 in lbf
3. Determine the required actuating force from equation 17.11b and the brake geometry.

Distance c c  r  e c  3.000  in
b  μc
Actuation force Fa  Fn1 Fa  625  lbf
a
4. Check to see if the brake is self-locking using the relationship given in the text.

self_locking  return "yes" if μ  c  b


"no" otherwise

self_locking  "no"

5. Calculate the value of c that would make the brake self-locking use the above relationship.

b
Value of c to self-lock clock  clock  28.6 in
μ
Bottom shoe - non self energizing
6. The actuation force is the same on both shoes and is equal to the actuation force on the top shoe. Use
equation 17.11b to solve for the normal force on the bottom shoe.
a
Normal force Fn2  Fa Fn2  848  lbf
b  μc

7. Use equation 17.10 to calculate the torque capacity.


Torque capacity T2  μ  Fn2 r T2  1425 in lbf

Both shoes
8. The total torque capacity is Ttot  T1  T2 Ttot  3184 in lbf

© 2011 Pearson Education, Inc., Upper Saddle River, NJ. All rights reserved. This publication is protected by Copyright and written permission should be
MACHINE DESIGN - An Integrated Approach, 4th Ed. 17-13-1
PROBLEM 17-13
Statement: Figure P17-3 shows a single long-shoe drum brake. Find its torque capacity and required
actuating force for the dimensions given below.
Given: Pivot to load a x  100mm Drum radius r  30 mm
Pivot to Y-axis b x  70 mm Drum width w  50 mm
Pivot to X-axis b y  20 mm Shoe start angle θ1  25 deg
Maximum pressure p max  1300 kPa Shoe end angle θ2  125  deg
Friction coeff. μ  0.3

Assumptions: Long-shoe theory is appropriate. The drum rotates CCW and the brake is self-energizing.

Solution: See Figure P17-3 and Mathcad file P1713.

1. Determine the maximum shoe angle. θmax  return 90 deg if θ2  90 deg
θ2 otherwise
θmax  90 deg

2. Use equation 17.15 to calculate the torque capacity.

p max
  cos θ1  cos θ2 
2
Torque capacity T  μ  w r 
sin θmax

T  26.0 N  m
3. Determine the required actuating force from equations 17.14 and the brake geometry.
2 2
Dimension b b  bx  by b  72.801 mm
Normal moment

    θ2  θ1    sin 2  θ2  sin 2  θ1 


p max 1 1
MFn  w r b 
sin θmax  2 4 
MFn  184.423  N  m
Friction moment

 r  cos θ2  cos θ1  


2
  sin θ2  sin θ1
p max b 2
MFf  μ  w r
sin θmax 
 2

MFf  15.487 N  m

MFn  MFf
Actuating force Fa  Fa  1689 N
ax

© 2011 Pearson Education, Inc., Upper Saddle River, NJ. All rights reserved. This publication is protected by Copyright and written permission should be
MACHINE DESIGN - An Integrated Approach, 4th Ed. 17-14-1

PROBLEM 17-14
Statement: Figure P17-3 shows a single long-shoe drum brake. Find its torque capacity and required
actuating force for the dimensions given below.

Given: Pivot to load a x  8.00 in Drum radius r  5.00 in


Pivot to Y-axis b x  6.00 in Drum width w  1.50 in
Pivot to X-axis b y  4.00 in Shoe start angle θ1  35 deg
Maximum pressure p max  250  psi Shoe end angle θ2  155  deg
Friction coefficient μ  0.35

Assumptions: Long-shoe theory is appropriate. The drum rotates CCW and the brake is self-energizing.

Solution: See Figure P17-3 and Mathcad file P1714.

1. Determine the maximum shoe angle. θmax  return 90 deg if θ2  90 deg
θ2 otherwise
θmax  90 deg

2. Use equation 17.15 to calculate the torque capacity.

p max
  cos θ1  cos θ2 
2
Torque capacity T  μ  w r 
sin θmax

T  5662 in lbf


3. Determine the required actuating force from equations 17.14 and the brake geometry.
2 2
Dimension b b  bx  by b  7.211  in
Normal moment

    θ2  θ1    sin 2  θ2  sin 2  θ1 


p max 1 1
MFn  w r b 
sin θmax  2 4 
MFn  19925  in lbf
Friction moment

 r  cos θ2  cos θ1  


2
  sin θ2  sin θ1
p max b 2
MFf  μ  w r
sin θmax 
 2

MFf  6017 in lbf

MFn  MFf
Actuating force Fa  Fa  1738 lbf
ax

© 2011 Pearson Education, Inc., Upper Saddle River, NJ. All rights reserved. This publication is protected by Copyright and written permission should be
MACHINE DESIGN - An Integrated Approach, 4th Ed. 17-15-1
PROBLEM 17-15
Statement: Figure P17-4 shows a double long-shoe drum brake. Find its torque capacity and required
actuating force for the dimensions given below.
Given: Pivot to load a x  90mm Drum radius r  40 mm
Pivot to Y-axis b x  80 mm Drum width w  30 mm
Pivot to X-axis b y  30 mm Shoe start angle θ1  30 deg
Maximum pressure p max  1500 kPa Shoe end angle θ2  160  deg
Friction coeff. μ  0.25

Assumptions: Long-shoe theory is appropriate. The drum rotates CCW.

Solution: See Figure P17-4 and Mathcad file P1715.


Top shoe - self-energizing
1. Determine the maximum shoe angle. θmax  return 90 deg if θ2  90 deg
θ2 otherwise
θmax  90 deg

2. Use equation 17.15 to calculate the torque capacity.

p max
  cos θ1  cos θ2 
2
Torque capacity T1  μ  w r 
sin θmax

T1  32.50  N  m

3. Determine the required actuating force from equations 17.14 and the brake geometry.
2 2
Dimension b b  bx  by b  85.44  mm
Normal moment

    θ2  θ1    sin 2  θ2  sin 2  θ1 


p max 1 1
MFn  w r b 
sin θmax  2 4 
MFn  232.482  N  m
Friction moment

 r  cos θ2  cos θ1  


2
  sin θ2  sin θ1
p max b 2
MFf  μ  w r
sin θmax 
 2

MFf  35.06  N  m

MFn  MFf
Actuating force Fa  Fa  2194 N
ax

Bottom shoe - nonself-energizing

4. The magnitude of the actuation force is the same on both shoes and is equal to the actuation force on the top
shoe. This will result in a lower maximum pressure on the lower, nonself-energizing, shoe. Determine the
maximum pressure on the lower shoe by solving equation 17.14c (using the plus sign) for p max2.

© 2011 Pearson Education, Inc., Upper Saddle River, NJ. All rights reserved. This publication is protected by Copyright and written permission should be
MACHINE DESIGN - An Integrated Approach, 4th Ed. 17-15-2

a x Fa sin θmax
p max2  
w r θ θ
 2 2  2
 b  sin( θ ) dθ   μ  sin( θ )  ( r  b  cos( θ ) ) dθ
θ θ
1 1

p max2  1107 kPa

5. Use equation 17.15 to calculate the torque capacity.

p max2
  cos θ1  cos θ2 
2
Torque capacity T2  μ  w r 
sin θmax

T2  23.98  N  m

6. The torque capacity of the double brake is the sum of the torque capacity of the two.

Total torque capacity Ttot  T1  T2 Ttot  56.5 N  m

© 2011 Pearson Education, Inc., Upper Saddle River, NJ. All rights reserved. This publication is protected by Copyright and written permission should be
MACHINE DESIGN - An Integrated Approach, 4th Ed. 17-16-1
PROBLEM 17-16
Statement: Figure P17-4 shows a double long-shoe drum brake. Find its torque capacity and required
actuating force for the dimensions given below.
Given: Pivot to load a x  12.00  in Drum radius r  6.00 in
Pivot to Y-axis b x  8.00 in Drum width w  2.00 in
Pivot to X-axis b y  3.00 in Shoe start angle θ1  25 deg
Maximum pressure p max  200  psi Shoe end angle θ2  145  deg
Friction coeff. μ  0.28

Assumptions: Long-shoe theory is appropriate. The drum rotates CCW.

Solution: See Figure P17-4 and Mathcad file P1716.


Top shoe - self-energizing

1. Determine the maximum shoe angle. θmax  return 90 deg if θ2  90 deg
θ2 otherwise
θmax  90 deg

2. Use equation 17.15 to calculate the torque capacity.

p max
  cos θ1  cos θ2 
2
Torque capacity T1  μ  w r 
sin θmax

T1  6957 in lbf

3. Determine the required actuating force from equations 17.14 and the brake geometry.
2 2
Dimension b b  bx  by b  8.544  in
Normal moment

    θ2  θ1    sin 2  θ2  sin 2  θ1 


p max 1 1
MFn  w r b 
sin θmax  2 4 
MFn  30218  in lbf
Friction moment

 r  cos θ2  cos θ1  


2
  sin θ2  sin θ1
p max b 2
MFf  μ  w r
sin θmax 
 2

MFf  6525 in lbf

MFn  MFf
Actuating force Fa  Fa  1974 lbf
ax

Bottom shoe - nonself-energizing

4. The magnitude of the actuation force is the same on both shoes and is equal to the actuation force on the top
shoe. This will result in a lower maximum pressure on the lower, nonself-energizing, shoe. Determine the
maximum pressure on the lower shoe by solving equation 17.14c (using the plus sign) for p max2.
© 2011 Pearson Education, Inc., Upper Saddle River, NJ. All rights reserved. This publication is protected by Copyright and written permission should be
MACHINE DESIGN - An Integrated Approach, 4th Ed. 17-16-2

a x Fa sin θmax
p max2  
w r θ θ
 2 2  2
 b  sin( θ ) dθ   μ  sin( θ )  ( r  b  cos( θ ) ) dθ
θ θ
1 1

p max2  129.0  psi

5. Use equation 17.15 to calculate the torque capacity.

p max2
  cos θ1  cos θ2 
2
Torque capacity T2  μ  w r 
sin θmax

T2  4486 in lbf

6. The torque capacity of the double brake is the sum of the torque capacity of the two.

Total torque capacity Ttot  T1  T2 Ttot  11443  in lbf

© 2011 Pearson Education, Inc., Upper Saddle River, NJ. All rights reserved. This publication is protected by Copyright and written permission should be
MACHINE DESIGN - An Integrated Approach, 4th Ed. 17-17a-1
PROBLEM 17-17a
Statement: The short-shoe approximation is considered to be valid for brake shoes with an included angle
of up to about 45 deg. For the brake shown in Figure P17-3, calculate its torque capacity and
required actuating force by both the short-shoe method and the long-shoe method and
compare the results for the dimensions given below.

Given: Pivot to load a x  90mm Drum radius r  40 mm


Pivot to Y-axis b x  80 mm Drum width w  30 mm
Pivot to X-axis b y  30 mm Shoe start angle θ1  75 deg
Maximum pressure p max  1500 kPa Shoe end angle θ2  105  deg
Friction coefficient μ  0.3

Assumptions: The drum rotates CCW and the brake is self-energizing.

Solution: See Figure P17-3 and Mathcad file P1717a.


Short-shoe method
1. Calculate the total shoe angle. θ  θ2  θ1 θ  30 deg

2. Determine the normal force on the drum from equation 17.8.


Normal force Fn  p max  r θ  w Fn  0.942  kN

3. Use equation 17.10 to calculate the torque capacity.


Torque capacity Tshort  μ  Fn r Tshort  11.3 N  m

4. Determine the required actuating force from equation 17.11b and the brake geometry.

Distance c c  r  b y c  10 mm

bx  μ  c
Actuation force Fashort  Fn Fashort  806  N
ax

Long-shoe method

5. Determine the maximum shoe angle. θmax  return 90 deg if θ2  90 deg
θ2 otherwise
θmax  90 deg

6. Use equation 17.15 to calculate the torque capacity.


p max
  cos θ1  cos θ2 
2
Torque capacity Tlong  μ  w r 
sin θmax

Tlong  11.2 N  m
7. Determine the required actuating force from equations 17.14 and the brake geometry.
2 2
Dimension b b  bx  by b  85.44  mm
Normal moment

    θ2  θ1    sin 2  θ2  sin 2  θ1 


p max 1 1
MFn  w r b 
sin θmax  2 4 
MFn  78.711 N  m
© 2011 Pearson Education, Inc., Upper Saddle River, NJ. All rights reserved. This publication is protected by Copyright and written permission should be
MACHINE DESIGN - An Integrated Approach, 4th Ed. 17-17a-2

Friction moment

 r  cos θ2  cos θ1  


2
  sin θ2  sin θ1
p max b 2
MFf  μ  w r
sin θmax 
 2

MFf  11.181 N  m

MFn  MFf
Actuating force Falong  Falong  750  N
ax

8. Compare the results.

Method Torque Capacity Actuation Force

Short shoe Tshort  11.3 N  m Fashort  806  N


Long shoe Tlong  11.2 N  m Falong  750  N

© 2011 Pearson Education, Inc., Upper Saddle River, NJ. All rights reserved. This publication is protected by Copyright and written permission should be
MACHINE DESIGN - An Integrated Approach, 4th Ed. 17-18a-1
PROBLEM 17-18a
Statement: The short-shoe approximation is considered to be valid for brake shoes with an included angle
of up to about 45 deg. For the brake shown in Figure P17-4, calculate its torque capacity and
required actuating force by both the short-shoe method and the long-shoe method and
compare the results for the dimensions given below.

Given: Pivot to load a x  90mm Drum radius r  40 mm


Pivot to Y-axis b x  80 mm Drum width w  30 mm
Pivot to X-axis b y  30 mm Shoe start angle θ1  75 deg
Maximum pressure p max  1500 kPa Shoe end angle θ2  105  deg
Friction coeff. μ  0.3

Assumptions: The drum rotates CCW.


Solution: See Figure P17-4 and Mathcad file P1718a.
Short-shoe method
Top shoe - self energizing
1. Calculate the total shoe angle. θ  θ2  θ1 θ  30 deg
2. Determine the normal force on the drum from equation 17.8.
Normal force Fn  p max  r θ  w Fn  0.942  kN
3. Use equation 17.10 to calculate the torque capacity.
Torque capacity Tshort1  μ  Fn r Tshort1  11.3 N  m
4. Determine the required actuating force from equation 17.11b and the brake geometry.
Distance c c  r  b y c  10 mm

bx  μ  c
Actuation force Fashort  Fn Fashort  806  N
ax
Bottom shoe - nonself energizing
5. The actuation force is the same on both shoes and is equal to the actuation force on the top shoe. Use
equation 17.11b to solve for the normal force on the bottom shoe.
ax
Normal force Fn2  Fashort  Fn2  874  N
bx  μ  c

6. Use equation 17.10 to calculate the torque capacity.


Torque capacity Tshort2  μ  Fn2 r Tshort2  10.49  N  m
Both shoes
7. The total torque capacity is Tshort  Tshort1  Tshort2 Tshort  21.8 N  m
Long-shoe method
Top shoe - self energizing
8. Determine the maximum shoe angle. θmax  return 90 deg if θ2  90 deg
θ2 otherwise
θmax  90 deg
9. Use equation 17.15 to calculate the torque capacity.
p max
  cos θ1  cos θ2 
2
Torque capacity Tlong1  μ  w r 
sin θmax
© 2011 Pearson Education, Inc., Upper Saddle River, NJ. All rights reserved. This publication is protected by Copyright and written permission should be
MACHINE DESIGN - An Integrated Approach, 4th Ed. 17-18a-2

Tlong1  11.2 N  m

10. Determine the required actuating force from equations 17.14 and the brake geometry.
2 2
Dimension b b  bx  by b  85.44  mm
Normal moment

    θ2  θ1    sin 2  θ2  sin 2  θ1 


p max 1 1
MFn  w r b 
sin θmax  2 4 

MFn  78.711 N  m

Friction moment

 r  cos θ2  cos θ1  


2
  sin θ2  sin θ1
p max b 2
MFf  μ  w r
sin θmax 
 2

MFf  11.181 N  m

MFn  MFf
Actuating force Falong  Falong  750  N
ax
Bottom shoe - nonself-energizing
11. The magnitude of the actuation force is the same on both shoes and is equal to the actuation force on the top
shoe. This will result in a lower maximum pressure on the lower, nonself-energizing, shoe. Determine the
maximum pressure on the lower shoe by solving equation 17.14c (using the plus sign) for p max2.

a x Falong sin θmax


p max2  
w r θ θ
 2 2  2
 b  sin( θ ) dθ   μ  sin( θ )  ( r  b  cos( θ ) ) dθ
θ θ
1 1

p max2  1127 kPa

12. Use equation 17.15 to calculate the torque capacity.

p max2
  cos θ1  cos θ2 
2
Torque capacity Tlong2  μ  w r 
sin θmax

Tlong2  8.40 N  m

13. The torque capacity of the double brake is the sum of the torque capacity of the two.

Total torque capacity Tlong  Tlong1  Tlong2 Tlong  19.6 N  m

14. Compare the results.

Method Torque Capacity Actuation Force

Short shoe Tshort  21.8 N  m Fashort  806  N


Long shoe Tlong  19.6 N  m Falong  750  N

© 2011 Pearson Education, Inc., Upper Saddle River, NJ. All rights reserved. This publication is protected by Copyright and written permission should be
MACHINE DESIGN - An Integrated Approach, 4th Ed. 17-19a-1
PROBLEM 17-19a
Statement: The short-shoe approximation is considered to be valid for brake shoes with an included angle
of up to about 45 deg. For the brake shown in Figure P17-3, calculate its torque capacity and
required actuating force by both the short-shoe method and the long-shoe method and
compare the results for the dimensions given below.

Given: Pivot to load a x  8.00 in Drum radius r  5.00 in


Pivot to Y-axis b x  6.00 in Drum width w  1.50 in
Pivot to X-axis b y  4.00 in Shoe start angle θ1  75 deg
Maximum pressure p max  250  psi Shoe end angle θ2  105  deg
Friction coeff. μ  0.35

Assumptions: The drum rotates CCW and the brake is self-energizing.

Solution: See Figure P17-3 and Mathcad file P1719a.


Short-shoe method
1. Calculate the total shoe angle. θ  θ2  θ1 θ  30 deg

2. Determine the normal force on the drum from equation 17.8.


Normal force Fn  p max  r θ  w Fn  982  lbf

3. Use equation 17.10 to calculate the torque capacity.


Torque capacity Tshort  μ  Fn r Tshort  1718 in lbf

4. Determine the required actuating force from equation 17.11b and the brake geometry.

Distance c c  r  b y c  1.000  in

bx  μ  c
Actuation force Fashort  Fn Fashort  693  lbf
ax
Long-shoe method
5. Determine the maximum shoe angle. θmax  return 90 deg if θ2  90 deg
θ2 otherwise
θmax  90 deg

6. Use equation 17.15 to calculate the torque capacity.

p max
  cos θ1  cos θ2 
2
Torque capacity Tlong  μ  w r 
sin θmax

Tlong  1698 in lbf


7. Determine the required actuating force from equations 17.14 and the brake geometry.
2 2
Dimension b b  bx  by b  183.162  mm

Normal moment

    θ2  θ1    sin 2  θ2  sin 2  θ1 


p max 1 1
MFn  w r b 
sin θmax  2 4 

MFn  6920 in lbf


© 2011 Pearson Education, Inc., Upper Saddle River, NJ. All rights reserved. This publication is protected by Copyright and written permission should be
MACHINE DESIGN - An Integrated Approach, 4th Ed. 17-19a-2

Friction moment

 r  cos θ2  cos θ1  


2
  sin θ2  sin θ1
p max b 2
MFf  μ  w r
sin θmax 
 2

MFf  1698 in lbf

MFn  MFf
Actuating force Falong  Falong  653  lbf
ax

8. Compare the results.

Method Torque Capacity Actuation Force

Short shoe Tshort  1718 in lbf Fashort  693  lbf


Long shoe Tlong  1698 in lbf Falong  653  lbf

© 2011 Pearson Education, Inc., Upper Saddle River, NJ. All rights reserved. This publication is protected by Copyright and written permission should be
MACHINE DESIGN - An Integrated Approach, 4th Ed. 17-20a-1
PROBLEM 17-20a
Statement: The short-shoe approximation is considered to be valid for brake shoes with an included angle
of up to about 45 deg. For the brake shown in Figure P17-4, calculate its torque capacity and
required actuating force by both the short-shoe method and the long-shoe method and
compare the results for the dimensions given below.

Given: Pivot to load a x  8.00 in Drum radius r  5.00 in


Pivot to Y-axis b x  6.00 in Drum width w  2.00 in
Pivot to X-axis b y  4.00 in Shoe start angle θ1  75 deg
Maximum pressure p max  250  psi Shoe end angle θ2  105  deg
Friction coeff. μ  0.35

Assumptions: The drum rotates CCW.


Solution: See Figure P17-4 and Mathcad file P1720a.
Short-shoe method
Top shoe - self energizing
1. Calculate the total shoe angle. θ  θ2  θ1 θ  30 deg
2. Determine the normal force on the drum from equation 17.8.
Normal force Fn  p max  r θ  w Fn  1309 lbf
3. Use equation 17.10 to calculate the torque capacity.
Torque capacity Tshort1  μ  Fn r Tshort1  2291 in lbf
4. Determine the required actuating force from equation 17.11b and the brake geometry.
Distance c c  r  b y c  1.000  in

bx  μ  c
Actuation force Fashort  Fn Fashort  924  lbf
ax

Bottom shoe - nonself energizing


5. The actuation force is the same on both shoes and is equal to the actuation force on the top shoe. Use equation
17.11b to solve for the normal force on the bottom shoe.
ax
Normal force Fn2  Fashort  Fn2  1165 lbf
bx  μ  c

6. Use equation 17.10 to calculate the torque capacity.


Torque capacity Tshort2  μ  Fn2 r Tshort2  2038 in lbf
Both shoes
7. The total torque capacity is Tshort  Tshort1  Tshort2 Tshort  4329 in lbf
Long-shoe method
Top shoe - self energizing

8. Determine the maximum shoe angle. θmax  return 90 deg if θ2  90 deg
θ2 otherwise
θmax  90 deg

9. Use equation 17.15 to calculate the torque capacity.

© 2011 Pearson Education, Inc., Upper Saddle River, NJ. All rights reserved. This publication is protected by Copyright and written permission should be
MACHINE DESIGN - An Integrated Approach, 4th Ed. 17-20a-2
p max
  cos θ1  cos θ2 
2
Torque capacity Tlong1  μ  w r 
sin θmax

Tlong1  2265 in lbf

10. Determine the required actuating force from equations 17.14 and the brake geometry.
2 2
Dimension b b  bx  by b  7.211  in

Normal moment

    θ2  θ1    sin 2  θ2  sin 2  θ1 


p max 1 1
MFn  w r b 
sin θmax  2 4 

MFn  9227 in lbf


Friction moment

 r  cos θ2  cos θ1  


2
  sin θ2  sin θ1
p max b 2
MFf  μ  w r
sin θmax 
 2

MFf  2265 in lbf

MFn  MFf
Actuating force Falong  Falong  870  lbf
ax
Bottom shoe - nonself-energizing
11. The magnitude of the actuation force is the same on both shoes and is equal to the actuation force on the top
shoe. This will result in a lower maximum pressure on the lower, nonself-energizing, shoe. Determine the
maximum pressure on the lower shoe by solving equation 17.14c (using the plus sign) for p max2.

a x Falong sin θmax


p max2  
w r θ θ
 2 2  2
 b  sin( θ ) dθ   μ  sin( θ )  ( r  b  cos( θ ) ) dθ
θ θ
1 1

p max2  151.5  psi

12. Use equation 17.15 to calculate the torque capacity.

p max2
  cos θ1  cos θ2 
2
Torque capacity Tlong2  μ  w r 
sin θmax

Tlong2  1372 in lbf

13. The torque capacity of the double brake is the sum of the torque capacity of the two.
Total torque capacity Tlong  Tlong1  Tlong2 Tlong  3637 in lbf
14. Compare the results.

Method Torque Capacity Actuation Force

Short shoe Tshort  4329 in lbf Fashort  924  lbf


Long shoe Tlong  3637 in lbf Falong  870  lbf

© 2011 Pearson Education, Inc., Upper Saddle River, NJ. All rights reserved. This publication is protected by Copyright and written permission should be
MACHINE DESIGN - An Integrated Approach, 4th Ed. 17-21-1

PROBLEM 17-21
Statement: Figure P17-2 shows a double short-shoe drum brake. Find the reaction forces in the global
XY system for the dimensions given below.
Given: Pivot to load a  90 mm Drum radius r  40 mm
Pivot to y-axis b  80 mm Drum width w  60 mm
Pivot to x-axis e  30 mm Shoe angle θ  25 deg
Maximum pressure p max  1500 kPa Friction coefficient μ  0.25

Assumptions: Short-shoe theory is appropriate. The drum rotates CCW.

Solution: See Figure P17-2 and Mathcad file P1721.


Top shoe - self energizing
1. Determine the normal force on the drum from equation 17.8.
Normal force Fn1  p max  r θ  w Fn1  1571 N

2. Calculate the friction force. Ff1  μ  Fn1 Ff1  392.7  N

3. Determine the required actuating force from equation 17.11b and the brake geometry modified for CW rotation.

Distance c c  r  e c  10.000 mm
b  μc
Actuation force Fa1  Fn1 Fa1  1353 N
a
4. The reaction forces on the top shoe pivot are:
Rx1  Ff1 Rx1  392.7  N

Ry1  Fa1  Fn1 Ry1  218.2  N

Bottom shoe - nonself energizing


5. The magnitude of the actuation force is the same on both shoes and is equal to the actuation force on the top
shoe. Use equation 17.11b to solve for the normal force on the bottom shoe.

Actuation force Fa2  Fa1 Fa2  1353 N

a
Normal force Fn2  Fa2 Fn2  1476 N
b  μc

6. Calculate the friction force. Ff2  μ  Fn2 Ff2  368.9  N

7. The reaction forces on the top shoe pivot are:


Rx2  Ff2 Rx2  368.9  N

Ry2  Fa2  Fn2 Ry2  123.0  N

© 2011 Pearson Education, Inc., Upper Saddle River, NJ. All rights reserved. This publication is protected by Copyright and written permission should be
MACHINE DESIGN - An Integrated Approach, 4th Ed. 17-22-1

PROBLEM 17-22
Statement: Figure P17-2 shows a double short-shoe drum brake. Find the reaction forces in the global XY
system for the dimensions given below.
Given: Pivot to load a  12.00  in Drum radius r  6.00 in
Pivot to y-axis b  8.00 in Drum width w  2.00 in
Pivot to x-axis e  3.00 in Shoe angle θ  25 deg
Maximum pressure p max  200  psi Friction coefficient μ  0.28

Assumptions: Short-shoe theory is appropriate. The drum rotates CCW.

Solution: See Figure P17-2 and Mathcad file P1722.


Top shoe - self energizing
1. Determine the normal force on the drum from equation 17.8.
Normal force Fn1  p max  r θ  w Fn1  1047.2 lbf

2. Calculate the friction force. Ff1  μ  Fn1 Ff1  293.2  lbf

3. Determine the required actuating force from equation 17.11b and the brake geometry modified for CW
rotation.
Distance c c  r  e c  3.000  in
b  μc
Actuation force Fa1  Fn1 Fa1  624.828  lbf
a
4. The reaction forces on the top shoe pivot are:
Rx1  Ff1 Rx1  293.2  lbf

Ry1  Fa1  Fn1 Ry1  422.4  lbf

Bottom shoe - nonself energizing


5. The magnitude of the actuation force is the same on both shoes and is equal to the actuation force on the top
shoe. Use equation 17.11b to solve for the normal force on the bottom shoe.

Actuation force Fa2  Fa1 Fa2  625  lbf


a
Normal force Fn2  Fa2 Fn2  848.183  lbf
b  μc

6. Calculate the friction force. Ff2  μ  Fn2 Ff2  237.5  lbf

7. The reaction forces on the top shoe pivot are:


Rx2  Ff2 Rx2  237.5  lbf
Ry2  Fa2  Fn2 Ry2  223.4  lbf

© 2011 Pearson Education, Inc., Upper Saddle River, NJ. All rights reserved. This publication is protected by Copyright and written permission should be
MACHINE DESIGN - An Integrated Approach, 4th Ed. 17-23-1
PROBLEM 17-23
Statement: Find the reaction forces at the arm pivot in the global XY system for the brake in Figure P17-3 for
the dimensions given below.
Given: Pivot to load a x  100mm Drum radius r  30 mm
Pivot to Y-axis b x  70 mm Drum width w  50 mm
Pivot to X-axis b y  20 mm Shoe start angle θ1  25 deg
Maximum pressure p max  1300 kPa Shoe end angle θ2  125  deg
Friction coefficient μ  0.3

Assumptions: Long-shoe theory is appropriate. The drum rotates CCW and the brake is self-energizing.
Solution: See Figure P17-3 and Mathcad file P1723.
1. Determine the maximum shoe angle. θmax  return 90 deg if θ2  90 deg
θ2 otherwise
θmax  90 deg

2. Determine the required actuating force from equations 17.14 and the brake geometry.
2 2
Dimension b b  bx  by b  72.801 mm
Normal moment

    θ2  θ1    sin 2  θ2  sin 2  θ1 


p max 1 1
MFn  w r b 
sin θmax  2 4 
MFn  184.423  N  m
Friction moment

 r  cos θ2  cos θ1  


2
  sin θ2  sin θ1
p max b 2
MFf  μ  w r
sin θmax 
 2

MFf  15.487 N  m

MFn  MFf
Actuating force Fa  Fa  1689 N
ax

3. Let the angle between the global X axis and the x1 axis be .

 by 
Rotation angle α  atan  α  15.945 deg
 bx 
4. First, find the reaction forces in the x1-y1 system by summing forces in the x1 and y1 directions

 
 Fx1 Fa sin( α)   cos( θ ) dFn   sin( θ ) dFf  Rx1 = 0
 

 
 Fy1 Fa cos( α)   sin( θ ) dFn   cos( θ ) dFf  Ry1 = 0
 
where Rx1 and Ry1 are both assumed to act in their respective positive directions.

5. Expand the integrals and solve these two equations for Rx1 and Ry1, respectively.
© 2011 Pearson Education, Inc., Upper Saddle River, NJ. All rights reserved. This publication is protected by Copyright and written permission should be
MACHINE DESIGN - An Integrated Approach, 4th Ed. 17-23-2

  θ2 θ
 2 
Rx1  Fa sin( α)  w r
p max  2 
  sin( θ )  cos( θ ) dθ  μ   sin( θ ) dθ
sin θmax  θ θ 
 1 1 

Rx1  744.0  N

  θ2 θ
 2 
Ry1  Fa cos( α)  w r
p max  2
  sin( θ ) dθ  μ   sin( θ )  cos( θ ) dθ

sin θmax  θ θ 
 1 1 

Ry1  1052.9 N

6. Transform these forces to the global XY system.

RX  Rx1 cos( α)  Ry1 sin( α) RX  1005 N

RY  Rx1 sin( α)  Ry1 cos( α) RY  808  N

© 2011 Pearson Education, Inc., Upper Saddle River, NJ. All rights reserved. This publication is protected by Copyright and written permission should be
MACHINE DESIGN - An Integrated Approach, 4th Ed. 17-24-1
PROBLEM 17-24
Statement: Find the reaction forces at the arm pivot in the global XY system for the brake in Figure P17-3 for
the dimensions given below.
Given: Pivot to load a x  8.00 in Drum radius r  5.00 in
Pivot to Y-axis b x  6.00 in Drum width w  1.50 in
Pivot to X-axis b y  4.00 in Shoe start angle θ1  35 deg
Maximum pressure p max  250  psi Shoe end angle θ2  155  deg
Friction coefficient μ  0.35

Assumptions: Long-shoe theory is appropriate. The drum rotates CCW and the brake is self-energizing.
Solution: See Figure P17-3 and Mathcad file P1724.
1. Determine the maximum shoe angle. θmax  return 90 deg if θ2  90 deg
θ2 otherwise
θmax  90 deg

2. Determine the required actuating force from equations 17.14 and the brake geometry.
2 2
Dimension b b  bx  by b  7.211  in
Normal moment

    θ2  θ1    sin 2  θ2  sin 2  θ1 


p max 1 1
MFn  w r b 
sin θmax  2 4 
MFn  19925  in lbf
Friction moment

 r  cos θ2  cos θ1  


2
  sin θ2  sin θ1
p max b 2
MFf  μ  w r
sin θmax 
 2

MFf  6017 in lbf

MFn  MFf
Actuating force Fa  Fa  1738 lbf
ax

3. Let the angle between the global X axis and the x1 axis be .

 by 
Rotation angle α  atan  α  33.69  deg
 bx 
4. First, find the reaction forces in the x1-y1 system by summing forces in the x1 and y1 directions

 
 Fx1 Fa sin( α)   cos( θ ) dFn   sin( θ ) dFf  Rx1 = 0
 

 
 Fy1 Fa cos( α)   sin( θ ) dFn   cos( θ ) dFf  Ry1 = 0
 
where Rx1 and Ry1 are both assumed to act in their respective positive directions.

5. Expand the integrals and solve these two equations for Rx1 and Ry1, respectively.
© 2011 Pearson Education, Inc., Upper Saddle River, NJ. All rights reserved. This publication is protected by Copyright and written permission should be
MACHINE DESIGN - An Integrated Approach, 4th Ed. 17-24-2

  θ2 θ
 2 
Rx1  Fa sin( α)  w r
p max  2 
  sin( θ )  cos( θ ) dθ  μ   sin( θ ) dθ
sin θmax  θ θ 
 1 1 

Rx1  2072.3 lbf

  θ2 θ
 2 
Ry1  Fa cos( α)  w r
p max  2
  sin( θ ) dθ  μ   sin( θ )  cos( θ ) dθ

sin θmax   θ 
 θ1 1 

Ry1  1267.3 lbf

6. Transform these forces to the global XY system.

RX  Rx1 cos( α)  Ry1 sin( α) RX  2427 lbf

RY  Rx1 sin( α)  Ry1 cos( α) RY  95.1 lbf

© 2011 Pearson Education, Inc., Upper Saddle River, NJ. All rights reserved. This publication is protected by Copyright and written permission should be
MACHINE DESIGN - An Integrated Approach, 4th Ed. 17-25-1
PROBLEM 17-25
Statement: Find the reaction forces at the arm pivot in the global XY system for the brake in Figure P17-4 for
the dimensions given below.
Given: Pivot to load a x  90mm Drum radius r  40 mm
Pivot to Y-axis b x  80 mm Drum width w  30 mm
Pivot to X-axis b y  30 mm Shoe start angle θ1  30 deg
Maximum pressure p max  1500 kPa Shoe end angle θ2  160  deg
Friction coefficient μ  0.25

Assumptions: Long-shoe theory is appropriate. The drum rotates CCW.


Solution: See Figure P17-4 and Mathcad file P1725.
1. Determine the maximum shoe angle. θmax  return 90 deg if θ2  90 deg
θ2 otherwise
θmax  90 deg

2. Determine the required actuating force from equations 17.14 and the brake geometry.
2 2
Dimension b b  bx  by b  85.44  mm
Normal moment

    θ2  θ1    sin 2  θ2  sin 2  θ1 


p max 1 1
MFn  w r b 
sin θmax  2 4 
MFn  232.482  N  m
Friction moment

 r  cos θ2  cos θ1  


2
  sin θ2  sin θ1
p max b 2
MFf  μ  w r
sin θmax 
 2

MFf  35.06  N  m

MFn  MFf
Actuating force Fa  Fa  2194 N
ax

3. Let the angle between the global X axis and the x1 axis be .

 by 
Rotation angle α  atan  α  20.556 deg
 bx 
Top shoe - self-energizing
4. First, find the reaction forces in the x1-y1 system by summing forces in the x1 and y1 directions

 
 Fx1 Fa sin( α)   cos( θ ) dFn   sin( θ ) dFf  Rx1 = 0
 

 
 Fy1 Fa cos( α)   sin( θ ) dFn   cos( θ ) dFf  Ry1 = 0
 
where Rx1 and Ry1 are both assumed to act in their respective positive directions.

© 2011 Pearson Education, Inc., Upper Saddle River, NJ. All rights reserved. This publication is protected by Copyright and written permission should be
MACHINE DESIGN - An Integrated Approach, 4th Ed. 17-25-2

5. Expand the integrals and solve these two equations for Rx1 and Ry1, respectively.

  θ2 θ
 2 
Rx1  Fa sin( α)  w r

p max 2 
  sin( θ )  cos( θ ) dθ  μ   sin( θ ) dθ
sin θmax  θ θ 
 1 1 
Rx1  1570.2 N

  θ2 θ
 2 
Ry1  Fa cos( α)  w r

p max 2
  sin( θ ) dθ  μ   sin( θ )  cos( θ ) dθ

sin θmax  θ θ 
 1 1 
Ry1  637.2  N

6. Transform these forces to the global XY system.

RXtop  Rx1 cos( α)  Ry1 sin( α) RXtop  1694 N

RYtop  Rx1 sin( α)  Ry1 cos( α) RYtop  45.3 N

Bottom shoe - nonself-energizing


7. The magnitude of the actuation force is the same on both shoes and is equal to the actuation force on the top
shoe. This will result in a lower maximum pressure on the lower, nonself-energizing, shoe. Determine the
maximum pressure on the lower shoe by solving equation 17.14c (using the plus sign) for p max2.

a x Fa sin θmax
p max2  
w r θ θ
 2 2  2
 b  sin( θ ) dθ   μ  sin( θ )  ( r  b  cos( θ ) ) dθ
θ θ
1 1

p max2  1107 kPa

8. First, find the reaction forces in the x2-y2 system by summing forces in the x2 and y2 directions. However, for a
nonself-energizing shoe, the signs of the Ff terms are opposite to those for the self-energizing case. Note also
that the y2 axis points toward the shoe (opposite from what is shown in Figure P17-4).
 
 Fx2 Fa sin( α)   cos( θ ) dFn   sin( θ ) dFf  Rx2 = 0
 

 
 Fy2 Fa cos( α)   sin( θ ) dFn   cos( θ ) dFf  Ry2 = 0
 
where Rx2 and Ry2 are both assumed to act in their respective positive directions.

9. Expand the integrals and solve these two equations for Rx2 and Ry2, respectively.

  θ2 θ
 2 
Rx2  Fa sin( α)  w r

p max2 2 
  sin( θ )  cos( θ ) dθ  μ   sin( θ ) dθ
sin θmax  θ θ 
 1 1 

© 2011 Pearson Education, Inc., Upper Saddle River, NJ. All rights reserved. This publication is protected by Copyright and written permission should be
MACHINE DESIGN - An Integrated Approach, 4th Ed. 17-25-3
Rx2  356.6  N

  θ2 θ
 2 
Ry2  Fa cos( α)  w r
p max2  2
  sin( θ ) dθ  μ   sin( θ )  cos( θ ) dθ

sin θmax   θ 
 θ1 1 

Ry2  24.0 N

10. Transform these forces to the global XY system.

RXbot  Rx2 cos( α)  Ry2 sin( α) RXbot  325  N

RYbot  Rx2 sin( α)  Ry2 cos( α) RYbot  147.7  N

© 2011 Pearson Education, Inc., Upper Saddle River, NJ. All rights reserved. This publication is protected by Copyright and written permission should be
MACHINE DESIGN - An Integrated Approach, 4th Ed. 17-26-1
PROBLEM 17-26
Statement: Find the reaction forces at the arm pivot in the global XY system for the brake in Figure P17-4 for
the dimensions given below.
Given: Pivot to load a x  12.00  in Drum radius r  6.00 in
Pivot to Y-axis b x  8.00 in Drum width w  2.00 in
Pivot to X-axis b y  3.00 in Shoe start angle θ1  25 deg
Maximum pressure p max  200  psi Shoe end angle θ2  145  deg
Friction coefficient μ  0.28

Assumptions: Long-shoe theory is appropriate. The drum rotates CCW.


Solution: See Figure P17-4 and Mathcad file P1726.
1. Determine the maximum shoe angle. θmax  return 90 deg if θ2  90 deg
θ2 otherwise
θmax  90 deg

2. Determine the required actuating force from equations 17.14 and the brake geometry.
2 2
Dimension b b  bx  by b  8.544  in
Normal moment

    θ2  θ1    sin 2  θ2  sin 2  θ1 


p max 1 1
MFn  w r b 
sin θmax  2 4 
MFn  30218  in lbf
Friction moment

 r  cos θ2  cos θ1  


2
  sin θ2  sin θ1
p max b 2
MFf  μ  w r
sin θmax 
 2

MFf  6525 in lbf

MFn  MFf
Actuating force Fa  Fa  1974 lbf
ax

3. Let the angle between the global X axis and the x1 axis be .

 by 
Rotation angle α  atan  α  20.556 deg
 bx 
Top shoe - self-energizing
4. First, find the reaction forces in the x1-y1 system by summing forces in the x1 and y1 directions

 
 Fx1 Fa sin( α)   cos( θ ) dFn   sin( θ ) dFf  Rx1 = 0
 

 
 Fy1 Fa cos( α)   sin( θ ) dFn   cos( θ ) dFf  Ry1 = 0
 
where Rx1 and Ry1 are both assumed to act in their respective positive directions.

© 2011 Pearson Education, Inc., Upper Saddle River, NJ. All rights reserved. This publication is protected by Copyright and written permission should be
MACHINE DESIGN - An Integrated Approach, 4th Ed. 17-26-2

5. Expand the integrals and solve these two equations for Rx1 and Ry1, respectively.

  θ2 θ
 2 
Rx1  Fa sin( α)  w r

p max 2 
  sin( θ )  cos( θ ) dθ  μ   sin( θ ) dθ
sin θmax  θ θ 
 1 1 

Rx1  1503.1 lbf

  θ2 θ
 2 
Ry1  Fa cos( α)  w r

p max 2
  sin( θ ) dθ  μ   sin( θ )  cos( θ ) dθ

sin θmax  θ θ 
 1 1 

Ry1  1738.6 lbf

6. Transform these forces to the global XY system.

RXtop  Rx1 cos( α)  Ry1 sin( α) RXtop  2018 lbf

RYtop  Rx1 sin( α)  Ry1 cos( α) RYtop  1100 lbf

Bottom shoe - nonself-energizing


7. The magnitude of the actuation force is the same on both shoes and is equal to the actuation force on the top
shoe. This will result in a lower maximum pressure on the lower, nonself-energizing, shoe. Determine the
maximum pressure on the lower shoe by solving equation 17.14c (using the plus sign) for p max2.

a x Fa sin θmax
p max2  
w r θ θ
 2 2  2
 b  sin( θ ) dθ   μ  sin( θ )  ( r  b  cos( θ ) ) dθ
θ θ
1 1

p max2  129.0  psi

8. First, find the reaction forces in the x2-y2 system by summing forces in the x2 and y2 directions. However, for a
nonself-energizing shoe, the signs of the Ff terms are opposite to those for the self-energizing case. Note also
that the y2 axis points toward the shoe (opposite from what is shown in Figure P17-4).
 
 Fx2 Fa sin( α)   cos( θ ) dFn   sin( θ ) dFf  Rx2 = 0
 

 
 Fy2 Fa cos( α)   sin( θ ) dFn   cos( θ ) dFf  Ry2 = 0
 
where Rx2 and Ry2 are both assumed to act in their respective positive directions.

9. Expand the integrals and solve these two equations for Rx2 and Ry2, respectively.

  θ2 θ
 2 
Rx2  Fa sin( α)  w r

p max2 2 
  sin( θ )  cos( θ ) dθ  μ   sin( θ ) dθ
sin θmax  θ θ 
 1 1 
Rx2  61.7 lbf
© 2011 Pearson Education, Inc., Upper Saddle River, NJ. All rights reserved. This publication is protected by Copyright and written permission should be
MACHINE DESIGN - An Integrated Approach, 4th Ed. 17-26-3

 θ2 θ 
Ry2  Fa cos( α)  w r
p max2   2  2
  sin( θ ) dθ  μ   sin( θ )  cos( θ ) dθ

sin θmax  θ θ 
 1 1 

Ry2  399.3  lbf

10. Transform these forces to the global XY system.

RXbot  Rx2 cos( α)  Ry2 sin( α) RXbot  82.5 lbf

RYbot  Rx2 sin( α)  Ry2 cos( α) RYbot  396  lbf

© 2011 Pearson Education, Inc., Upper Saddle River, NJ. All rights reserved. This publication is protected by Copyright and written permission should be
MACHINE DESIGN - An Integrated Approach, 4th Ed. 17-27-1

PROBLEM 17-27
Statement: A clutch is needed for an electric motor that transmits 20 kW at 1100 rpm. The clutch will attach
directly to the motor housing face plate and is to have the same housing diameter as the motor,
which is 125 mm. The minimum radial clearance between the housing OD and the clutch disk
OD is 5 mm. The clutch output shaft will have the same diameter as the motor shaft, which is 15
mm. Design a multiple disk clutch for this application. State all assumptions and design
choices. Specify the clutch material, outside disk radius, inside disk radius, and the required
actuation force.

Given: Shaft diameter d s  15 mm


Transmitted power H  20 kW Rotational speed ω  1100 rpm
Housing dia. d h  125  mm Disk radial clearance cr  5  mm

Assumptions: Uniform pressure model.

Design choices:
Use a molded clutch material running against steel with:
Friction coefficient μ  0.25 Maximum pressure p  1000 kPa
Disk outside radius ro  0.5 d h  cr ro  57.5 mm

Solution: See Mathcad file P1727.


1. Calculate the transmitted torque.
H
T  T  173.6  N  m
ω

2. Tentatively choose an inside radius for the clutch discs and use equation 17.2c to solve for the required number
of friction faces.

 π p  μ   ro  ri
2 3 3
Let ri  0.5 d s  5  mm T=   Ns
3
3 T
Ns  Ns  1.762
2  π p  μ   ro  ri
3 3

3. Increase this to the next higher integer and solve for the required inside radius.

1
3
3 T
ri   ro  
3
Let Ns  2  ri  28.969 mm
 2  π p  μ  N s 

Round this to ri  29 mm

4. Convert the radii to diameters.


Outside diameter od  2  ro od  115  mm
Inside diameter id  2  ri id  58 mm

5. Calculate the actuation force required using equation 17.3.

3  T   ro  ri
2 2
Actuation force F 
 F  7748 N
2  μ  Ns  ro  ri
3 3

© 2011 Pearson Education, Inc., Upper Saddle River, NJ. All rights reserved. This publication is protected by Copyright and written permission should be
MACHINE DESIGN - An Integrated Approach, 4th Ed. 17-28-1

PROBLEM 17-28
Statement: A clutch is needed for an electric motor that transmits 25 hp at 800 rpm. The clutch will attach
directly to the motor housing face plate and is to have the same housing diameter as the motor,
which is 5.5 in. The minimum radial clearance between the housing OD and the clutch disk OD is
0.25 in. The clutch output shaft will have the same diameter as the motor shaft, which is 0.625 in.
Design a multiple disk clutch for this application. State all assumptions and design choices.
Specify the clutch material, outside disk radius, inside disk radius, and the required actuation
force.
Given: Shaft diameter d s  0.625  in
Transmitted power H  25 hp Rotational speed ω  800  rpm
Housing dia. d h  5.5 in Disk radial clearance cr  0.25 in
Assumptions: Uniform pressure model.
Design choices:
Use a molded clutch material running against steel with:
Friction coefficient μ  0.25 Maximum pressure p  150  psi
Disk outside radius ro  0.5 d h  cr ro  2.50 in

Solution: See Mathcad file P1728.


1. Calculate the transmitted torque.

H
T  T  1970 in lbf
ω
2. Tentatively choose an inside radius for the clutch discs and use equation 17.2c to solve for the required
number of friction faces.

T =  π p  μ   ro  ri   Ns
2 3 3
Let ri  0.5 d s  0.25 in
3
3 T
Ns  Ns  1.623
2  π p  μ   ro  ri
3 3

3. Increase this to the next higher integer and solve for the required inside radius.
1
3
3 T
ri   ro  
3
Let Ns  2  ri  1.456  in
 2  π p  μ  N s 

Round this to ri  1.50 in

4. Convert the radii to diameters.


Outside diameter od  2  ro od  5.00 in
Inside diameter id  2  ri id  3.00 in

5. Calculate the actuation force required using equation 17.3.

3  T   ro  ri
2 2
Actuation force F 
 F  1929 lbf
2  μ  Ns  ro  ri 
3 3

© 2011 Pearson Education, Inc., Upper Saddle River, NJ. All rights reserved. This publication is protected by Copyright and written permission should be
MACHINE DESIGN - An Integrated Approach, 4th Ed. 17-29-1

PROBLEM 17-29
Statement: Find the torque that a dual-pad, caliper disc brake with pad angle of 60 deg can transmit if the
outside and inside lining diameters are 160 mm and 90 mm, respectively, and the applied axial
force is 3 kN. Assume uniform wear and = 0.35. Is the pressure on the lining acceptable?
What lining materials would be suitable?

Given: Number of pads Np  2 Clutch dimensions:


Axial force F  3  kN Outside diameter od  160  mm
Friction coefficient μ  0.35 Inside diameter id  90 mm
Pad angle θ  0.333  π
Assumptions: Uniform wear model.
Solution: See Mathcad file P1729.

1. Calculate the outside and inside radii of the disk.


Outside radius ro  0.5 od ro  80 mm
Inside radius ri  0.5 id ri  45 mm

ro  ri
2. Using equation 17.6, calculate the torque capacity. T  Np μ  F  T  131.25 N  m
2

3. Calculate the maximum lining pressure using equation 17.5a.

F
Maximum pressure p max 
θ  ri  ro  ri

p max  1821 kPa

4. From Table 17-1, we see that either a molded or sintered metal lining is suitable.

© 2011 Pearson Education, Inc., Upper Saddle River, NJ. All rights reserved. This publication is protected by Copyright and written permission should be
MACHINE DESIGN - An Integrated Approach, 4th Ed. 17-30-1

PROBLEM 17-30
Statement: Find the torque that a 2-pad, caliper disc brake with pad angle of 60 deg can transmit if the
outside and inside lining diameters are 160 mm and 90 mm, respectively, and the applied axial
force is 3 kN. Assume uniform pressure and = 0.35. Is the pressure on the lining acceptable?
What lining materials would be suitable?

Given: Number of pads Np  2 Clutch dimensions:


Axial force F  3  kN Outside diameter od  160  mm
Friction coefficient μ  0.35 Inside diameter id  90 mm
Pad angle θ  0.333  π
Assumptions: Uniform pressure model.
Solution: See Mathcad file P1730.

1. Calculate the outside and inside radii of the disk.


Outside radius ro  0.5 od ro  80 mm
Inside radius ri  0.5 id ri  45 mm
3 3
2 ro  ri
2. Using equation 17.3, calculate the torque capacity. T  Np μ  F   T  134.7  N  m
3 2 2
ro  ri

3. Calculate the lining pressure using equation 17.2c.

3 T
Pressure p 
θ  μ  Np  ro  ri 
3 3

p  1311 kPa

4. From Table 17-1, we see that either a molded or sintered metal lining is suitable.

© 2011 Pearson Education, Inc., Upper Saddle River, NJ. All rights reserved. This publication is protected by Copyright and written permission should be
MACHINE DESIGN - An Integrated Approach, 4th Ed. 17-31-1

PROBLEM 17-31
Statement: Design a dual-pad caliper disc brake to provide a braking force of 240 N at the periphery of a
750-mm-dia wheel that is rotating at 670 rpm. Use an inside radius to outside radius ratio of
0.577. Assume uniform wear. State all assumptions and design choices. Specify the brake
material, outside pad radius, inside pad radius, pad angle, and the required actuation force.

Given: Number of pads Np  2 Wheel speed ω  670  rpm


Peripheral force Fw  240  N Wheel diameter d w  750  mm
Disk radius factor fr  0.577
Assumptions: Uniform wear model.
Design Choices:
Friction coefficient μ  0.30 Pad material Sintered metal
Pad angle (90 deg) θ  0.50 π Max. pressure p max  1500 kPa

Solution: See Mathcad file P1731.


1. Calculate the torque required T  0.5 d w Fw T  90 N  m

2. Using equation 17.5b and the disk radius factor, solve for the required outside and inside radii.

θ  μ  ri p max
  ro  ri
2 2
T=
2   Np ri = fr  ro

1
3
 2 T 
ro 
  2 
 θ  μ  pmax  fr   1  fr   Np
ro  69.16  mm round this to ro  70 mm
Then the inside radius is ri  fr  ro ri  40.39  mm
Round this to ri  40 mm
3. Solve equation 17.5a for the actuating force, F.

Actuating force F  θ  ri p max   ro  ri

F  2.827  kN

ro  ri
Rated torque T  Np μ  F 
2
T  93.3 N  m

4. DESIGN SUMMARY
Brake material Sintered metal
Coefficient of friction μ  0.30
Pad inside radius ri  40 mm
Pad outside radius ro  70 mm
Pad angle θ  90 deg
Actuating force F  2.83 kN
Maximum pressure p max  1500 kPa
Rated torque T  93.3 N  m

© 2011 Pearson Education, Inc., Upper Saddle River, NJ. All rights reserved. This publication is protected by Copyright and written permission should be
MACHINE DESIGN - An Integrated Approach, 4th Ed. 17-32-1

PROBLEM 17-32
Statement: Design a dual-pad caliper disc brake to provide a braking force of 240 N at the periphery of a
750-mm-dia wheel that is rotating at 670 rpm. Use an inside radius to outside radius ratio of
0.577. Assume uniform pressure. State all assumptions and design choices. Specify the
brake material, outside pad radius, inside pad radius, pad angle, and the required actuation
force.
Given: Number of pads Np  2 Wheel speed ω  670  rpm
Peripheral force Fw  240  N Wheel diameter d w  750  mm
Disk radius factor fr  0.577
Assumptions: Uniform wear model.
Design Choices:
Friction coefficient μ  0.30 Pad material Sintered metal
Pad angle (90 deg) θ  0.50 π Uniform pressure p  1500 kPa

Solution: See Mathcad file P1732.


1. Calculate the torque required T  0.5 d w Fw T  90 N  m
2. Using equation 17.2c and the disk radius factor, solve for the required outside and inside radii.

θμ p  3
  ro  ri   Np
3
T= ri = fr  ro
3
1
3
ro   
3 T
  3 
 θ  μ  p   1  fr   Np
ro  61.832 mm round this to ro  62 mm
Then the inside radius is ri  fr  ro ri  35.774 mm
Round this to ri  36 mm
3. Solve equation 17.1b for the actuating force, F.

θp  2
  ro  ri 
2
Actuating force F 
2
F  3.002  kN

3 3
2 ro  ri
Rated torque T  Np μ  F  
3 2 2
ro  ri
T  90.3 N  m

4. DESIGN SUMMARY
Brake material Sintered metal
Coefficient of friction μ  0.30
Pad inside radius ri  36 mm
Pad outside radius ro  62 mm
Pad angle θ  90 deg
Actuating force F  3.00 kN
Uniform pressure p  1500 kPa
Rated torque T  90.3 N  m

© 2011 Pearson Education, Inc., Upper Saddle River, NJ. All rights reserved. This publication is protected by Copyright and written permission should be
MACHINE DESIGN - An Integrated Approach, 4th Ed. 17-33-1

PROBLEM 17-33
Statement: An ultra-light solar racecar weighs 500 lb with driver. It has two 20-in-dia bicycle wheels in front
that are to have dual-pad caliper disk brakes on each wheel. The brakes must be capable of
bringing the car to a stop in a distance of 150 feet from a speed of 45 mph. Neglecting
aerodynamic and rolling resistance forces, design dual-pad caliper disc brakes for the car. Use
an inside radius to outside radius ratio of 0.577. Assume uniform wear. State all assumptions
and design choices. Specify the brake material, outside pad radius, inside pad radius, pad angle,
and the required actuation force.

Given: Number of pads Np  4 Car weight W  500  lbf


Stopping distance Ds  150  ft Wheel diameter d w  20 in
Disk radius factor fr  0.577 Initial speed Vi  45 mph
Assumptions: Uniform wear model Constant deceleration Level surface
Design Choices:
Friction coefficient μ  0.30 Pad material Molded
Pad angle θ  60 deg Max. pressure p max  225  psi

Solution: See Mathcad file P1733.


1. Calculate the torque required. For uniform deceleration, the stopping distance, initial velocity, and
deceleration are related by

2 2
Vi Vi 2
Ds = solving for the deceleration, d  d  14.52 ft  s
2 d 2  Ds
W
The force required between the tires and ground is Fw  d Fw  225.6  lbf
g
Thus, the torque required by the brakes is T  0.5 d w Fw T  188.04 lbf  ft

2. Using equation 17.5b and the disk radius factor, solve for the required outside and inside radii.

θ  μ  ri p max
  ro  ri
2 2
T=
2   Np ri = fr  ro

1
3
 2 T 
ro 
  2 
 θ  μ  pmax  fr   1  fr   Np
ro  3.461  in round this to ro  3.50 in
Then the inside radius is ri  fr  ro ri  2.019  in
Round this to ri  2.00 in
3. Solve equation 17.5a for the actuating force, F.

Actuating force F  θ  ri p max   ro  ri

F  706.9  lbf

ro  ri
Rated torque T  Np μ  F 
2
T  194.4  lbf  ft

© 2011 Pearson Education, Inc., Upper Saddle River, NJ. All rights reserved. This publication is protected by Copyright and written permission should be
MACHINE DESIGN - An Integrated Approach, 4th Ed. 17-33-2

4. DESIGN SUMMARY
Brake material Molded
Coefficient of friction μ  0.30
Pad inside radius ri  2.00 in
Pad outside radius ro  3.50 in
Pad angle θ  60 deg
Actuating force F  706.9  lbf
Maximum pressure p max  225  psi
Rated torque T  194  lbf  ft

© 2011 Pearson Education, Inc., Upper Saddle River, NJ. All rights reserved. This publication is protected by Copyright and written permission should be
MACHINE DESIGN - An Integrated Approach, 4th Ed. 17-34-1

PROBLEM 17-34
Statement: An ultra-light solar racecar weighs 500 lb with driver. It has two 20-in-dia bicycle wheels in
front that are to have dual-pad caliper disk brakes on each wheel. The brakes must be capable
of bringing the car to a stop in a distance of 150 feet from a speed of 45 mph. Neglecting
aerodynamic and rolling resistance forces, design dual-pad caliper disc brakes for the car. Use
an inside radius to outside radius ratio of 0.577. Assume uniform pressure. State all
assumptions and design choices. Specify the brake material, outside pad radius, inside pad
radius, pad angle, and the required actuation force.

Given: Number of pads Np  4 Car weight W  500  lbf


Stopping distance Ds  150  ft Wheel diameter d w  20 in
Disk radius factor fr  0.577 Initial speed Vi  45 mph
Assumptions: Uniform pressure model Constant deceleration Level surface
Design Choices:
Friction coefficient μ  0.30 Pad material Molded
Pad angle θ  60 deg Pressure p  225  psi
Solution: See Mathcad file P1734.

1. Calculate the torque required. For uniform deceleration, the stopping distance, initial velocity, and
deceleration are related by

2 2
Vi Vi 2
Ds = solving for the deceleration, d  d  14.52 ft  s
2 d 2  Ds
W
The force required between the tires and ground is Fw  d Fw  225.6  lbf
g
Thus, the torque required by the brakes is T  0.5 d w Fw T  188.04 lbf  ft
2. Using equation 17.2c and the disk radius factor, solve for the required outside and inside radii.

θμ p  3
  ro  ri   Np
3
T= ri = fr  ro
3
1
3
ro   
3 T
  3 
 θ  μ  p   1  fr   Np
ro  3.095  in round this to ro  3.125  in
Then the inside radius is ri  fr  ro ri  1.803  in
Round this to ri  1.750  in

3. Solve equation 17.1b for the actuating force, F.

θp  2
  ro  ri 
2
Actuating force F 
2
F  789.7  lbf
3 3
2 ro  ri
Rated torque T  Np μ  F  
3 2 2
ro  ri
T  197.6  lbf  ft

© 2011 Pearson Education, Inc., Upper Saddle River, NJ. All rights reserved. This publication is protected by Copyright and written permission should be
MACHINE DESIGN - An Integrated Approach, 4th Ed. 17-34-2

4. DESIGN SUMMARY
Brake material Molded
Coefficient of friction μ  0.30
Pad inside radius ri  1.750  in
Pad outside radius ro  3.125  in
Pad angle θ  60 deg
Actuating force F  789.7  lbf
Uniform pressure p  225  psi
Rated torque T  198  lbf  ft

© 2011 Pearson Education, Inc., Upper Saddle River, NJ. All rights reserved. This publication is protected by Copyright and written permission should be

You might also like